Sunteți pe pagina 1din 333

G.R. No.

L-5149

March 22, 1910

GREGORIO MACAPINLAC vs. MARIANO ALIMURONG,


FACTS:
Simplicia de los Santos having died on June 19, 1907, her surviving husband, Gregorio
Macapinlac, submitted her will to the Court of First Instance of Pampanga for probate.
Macario Alimurong, a nephew of the deceased, opposed the proceedings and
requested that "the will of the deceased, Doa Simplicia de los Santos, be declared null
and void for either of the two reasons" which he expresses, and which are:
(4) That, after the rough copy was amended by the addition of the abovementioned clause, a clear copy thereof was made up and was again read to the
testatrix, who approved it in all of its parts, and as she was unable to sign, she
requested Amando de Ocampo to sign for her and the latter wrote the following
words with his own hand. "At the request of the testatrix D.a Simplicia de los
Santos, I signed Amando de Ocampo." Immediately afterwards and also in
the presence of the same testatrux and of each other, the witnesses Jose Juico,
Gabino Panopio, Eusebio dayao, Juan Angeles, Jose Torres, Alejo San Pedro,
and Gregorio Sangil signed at the bottom of the will.
In view of the said factsthe lower court concludesthe will executed by
Simplicia de los Santos must be admitted to probate. The provisions of section
618 of the Code of Procedure in Civil Actions and Special Proceedings are fully
complied with. The will bears the name of the testatrix written by Amando de
Ocampo in her presence and by her express direction, and has been witnessed
and signed by more than three trustworthy witnesses, in the presence of the
testatrix and of each other.
But, besides the question of fact, the appellant submit another question of law, viz,
whether or not the will was signed in accordance with the law, and he affirms that it was
not, inasmuch as the law requires that when a person signs in place of the testator he
should write the name of the latter in the will as the signature; this was not done by
Amando de Ocampo in the will in question, as he did not sign it with the name of
testatrix.

It is shown by the evidence that the will was wholly written in the handwriting of the
subscribing witness, Gregorio Sangil, and at the foot thereof the following words appear
in a new paragraph and sufficiently apart:
At the request of the testatrix, Da. Simplicia de los Santos, I signed.
For Simplicia de los Santos.

Amando de Ocampo.

As a question of fact, the authenticity of the words "For Simplicia de los Santos,"
prefixed to the signature, is impugned as not having been written at the time of the
execution of the will.
And, as a question of law, it is claimed that the form of signing for the testatrix "At the
request of the testatrix Da. Simplicio de los Santos, I signed: Amando de Ocampo," is
not in accordance with the requirements of the law.
Regarding the first question, the trial court concluded that "the posterior insertion of the
words 'For Simplicia de los Santos' can not affect the validity of the will."
Therefore, it can be considered as nonexistent, and the other as the only fore of
signature by the testatrix, the authenticity of which has not been impugned or which the
trial court admits as conclusive, and is only one taken into account in its findings of fact.
Although the said words "For Simplicia de los Santos" be considered as inserted
subsequently, which we neither affirm nor deny, because a specific determination either
way is unnecessary, in our opinion the signature for the testatrix as if she signed the
will, and also the signature of the witness who, at her request, wrote the name of the
testatrix and signed for her, affirming the truth of this fact, attested by the other
witnesses then present. And this fully complies with the provisions of section 618 of the
Act.

G.R. No. 2586

January 19, 1906

TOMAS GUISON vs MARIA CONCEPCION


Jacoba Concepcion Salcedo made her will in Manila, on January 3, 1904. The last part
of the will is as follows:
It will be seen that the witness Feliciano Maglaqui, instead of writing the name of the
testatrix on the will, wrote his own. Probate of the will was refused in the court below on
the ground that the name of the testatrix was not signed thereto.
G.R. No. L-9150

March 31, 1915

MARIANO LEAO, vs. ARCADIO LEAO


The evidence of record satisfactorily discloses that Cristina Valdes, deceased, placed
her costs against her name, attached by some other person to the instrument offered
for probate which purports to be her last will and testament, in the presence of three
witnesses whose names are attached to the attesting clause, and that they attested
and subscribed the instrument in her presence and in the presence of each other.
We are of the opinion that the placing of the cross opposite her name at the
construction of the instrument was a sufficient compliance with the requirements of
section 618 of the Code of Civil Procedure, which prescribes that except where wills
are signed by some other person than the testator in the manner and from herein
indicated, a valid will must be signed by the testator. The right of a testator to sign his
will by mark, executed animo testandihas been uniformly sustained by the courts of last
resort of the United States in construing statutory provisions prescribing the mode of
execution of wills in language identical with, or substantially similar to that found in
section 618 of our code, which was taken from section 2349 of the Code of Vermont.
(Page on Wills, par. 173, and the cases there cited in support of the doctrine just
announced.)
The trial judge was of contrary opinion, and declined to admit the instrument to probate
as the last will and testament of the decedent. We are of opinion, however, that the
evidence of record satisfactorily establishes the execution of that instrument as and for
her last will and testament in the manner and form prescribed by law.

The judgment entered in the court below should therefore be reversed, without costs in
this instance, and the record remanded to the court below, where judgment will be
entered admitting the instrument in question to probate in accordance with the prayer of
the petitioner. So ordered.
G.R. No. L-4067

November 29, 1951

In the Matter of the will of ANTERO MERCADO, deceased. ROSARIO


GARCIA, petitioner, vs. JULIANA LACUESTA, ET AL
This is an appeal from a decision of the Court of Appeals disallowing the will of Antero
Mercado dated January 3, 1943. The will appears to have been signed by Atty.
Florentino Javier who wrote the name of Antero Mercado, followed below by "A reugo
del testator" and the name of Florentino Javier. Antero Mercado is alleged to have
written a cross immediately after his name.
In our opinion, the attestation clause is fatally defective for failing to state that Antero
Mercado caused Atty. Florentino Javier to write the testator's name under his express
direction, as required by section 618 of the Code of Civil Procedure. The herein
petitioner (who is appealing by way of certiorari from the decision of the Court of
Appeals) argues, however, that there is no need for such recital because the cross
written by the testator after his name is a sufficient signature and the signature of Atty.
Florentino Javier is a surplusage. Petitioner's theory is that the cross is as much a
signature as a thumbmark, the latter having been held sufficient by this Court.
It is not here pretended that the cross appearing on the will is the usual signature of
Antero Mercado or even one of the ways by which he signed his name. After mature
reflection, we are not prepared to liken the mere sign of the cross to a thumbmark, and
the reason is obvious. The cross cannot and does not have the trustworthiness of a
thumbmark.
G.R. No. 6845

September 1, 1914

YAP TUA, vs. YAP CA KUAN and YAP CA KUAN,


With reference to the second assignment of error to wit, that Tomasa Elizaga Yap
Caong was not of sound mind and memory at the time of the execution of the will, we
find the same conflict in the declarations of the witnesses which we found with
reference to the undue influence. While the testimony of Dr. Papa is very strong relating
to the mental condition of Tomasa Elizaga Yap Caong, yet, nevertheless, his testimony
related to a time perhaps twenty-four hours before the execution of the will in question
(Exhibit A). Several witnesses testified that at the time the will was presented to her for

her signature, she was of sound mind and memory and asked for a pen and ink and
kept the will in her possession for ten or fifteen minutes and finally signed it. The lower
court found that there was a preponderance of evidence sustaining the conclusion that
Tomasa Elizaga Yap Caong was of sound mind and memory and in the possession of
her faculties at the time she signed this will
1. There appears to be but little doubt that Tomasa Elizaga Yap Caong did execute the
will of August 6, 1909. Several witnesses testified to that fact. The mere fact, however,
that she executed a former will is no proof that she did not execute a later will. She had
a perfect right, by will, to dispose of her property, in accordance with the provisions of
law, up to the very last of moment her life. She had a perfect right to change, alter,
modify or revoke any and all of her former wills and to make a new one. Neither will the
fact that the new will fails to expressly revoke all former wills, in any way sustain the
charge that she did not make the new will.
2. "The signature of Tomasa Elizaga Yap Caong, in her first will (Exhibit 1) was not
identical with that which appears in her second will (Exhibit A)" the inference that she
had not signed the second will and all the argument of the appellants relating to said
third assignment of error is based upon the alleged fact that Tomasa Elizaga Yap
Caong did not sign Exhibit A. Several witnesses testified that they saw her write the
name "Tomasa." One of the witnesses testified that she had written her full name. We
are of the opinion, and we think the law sustains our conclusion, that if Tomasa Elizaga
Yap Caong signed any portion of her name tot he will, with the intention to sign the
same, that the will amount to a signature. It has been held time and time again that one
who makes a will may sign the same by using a mark, the name having been written by
others. If writing a mark simply upon a will is sufficient indication of the intention of the
person to make and execute a will, then certainly the writing of a portion or all of her
name ought to be accepted as a clear indication of her intention to execute the will.
In a report known as "Knox's Appeal." In this case one Harriett S. Knox died very
suddenly on the 17th of October, 1888, at the residence of her father. After her death a
paper was found in her room, wholly in her handwriting, written with a lead pencil, upon
three sides of an ordinary folded sheet of note paper and bearing the signature simply
of "Harriett." In this paper the deceased attempted to make certain disposition of her
property. The will was presented for probate. The probation was opposed upon the
ground that the same did not contain the signature of the deceased. That was the only
question presented to the court, whether the signature, in the form above indicated,
was a sufficient signature to constitute said paper the last will and testament of Harriett

S. Knox. It was admitted that the entire paper was in the handwriting of the deceased.
In deciding that question, Justice Mitchell said:
The precise case of a signature by the first name only, does not appear to have
arisen either in England or the United States; but the principle on which the
decisions already referred to were based, especially those in regard to signing
by initials only, are equally applicable to the present case, and additional force is
given to them by the decisions as to what constitutes a binding signature to a
contract.
The man who cannot write and who is obliged to make his mark simply therefor, upon
the will, is held to "sign" as effectually as if he had written his initials or his full name. It
would seem to be sufficient, under the law requiring a signature by the person making a
will, to make his mark, to place his initials or all or any part of his name thereon. In the
present case we think the proof shows, by a large preponderance, that Tomasa Elizaga
Yap Caong, if she did not sign her full name, did at least sign her given name "Tomasa,"
and that is sufficient to satisfy the statute.
3. During the trial of the cause the protestants made a strong effort to show that
Tomasa Elizaga Yap Caong did not sign her name in the presence of the witnesses and
that they did not sign their names in their presence nor in the presence of each other.
Upon that question there is considerable conflict of proof. An effort was made to show
that the will was signed by the witnesses in one room and by Tomasa in another. A plan
of the room or rooms in which the will was signed was presented as proof and it was
shown that there was but one room; that one part of the room was one or two steps
below the floor of the other; that the table on which the witnesses signed the will was
located upon the lower floor of the room. It was also shown that from the bed in which
Tomasa was lying, it was possible for her to see the table on which the witnesses
signed the will. While the rule is absolute that one who makes a will must sign the same
in the presence of the witnesses and that the witnesses must sign in the presence of
each other, as well as in the presence of the one making the will, yet, nevertheless, the
actual seeing of the signatures made is not necessary. It is sufficient if the signatures
are made where it is possible for each of the necessary parties, if they desire to see,
may see the signatures placed upon the will.
Upon a full consideration of the record, we find that a preponderance of the proof
shows that Tomasa Elizaga Yap Caong did execute, freely and voluntarily, while she
was in the right use of all her faculties, the will dated August 11, 1909 (Exhibit A).

Therefore the judgment of the lower court admitting said will to probate is hereby
affirmed with costs.
G.R. No. L-30289

March 26, 1929

SERAPIA DE GALA vs. APOLINARIO GONZALES


On November 23, 1920, Severina Gonzales executed a will in which Serapia de Gala, a
niece of Severina, was designated executrix. The testatrix died in November, 1926,
leaving no heirs by force of law, and on December 2, 1926, Serapia, through her
counsel, presented the will for probate. Apolinario Gonzales, a nephew of the
deceased, filed an opposition to the will on the ground that it had not been executed in
conformity with the provisions of section 618 of the Code of Civil Procedure. On April 2,
1927, Serapia de Gala was appointed special administratrix of the estate of the
deceased. She returned an inventory of the estate on March 31, 1927, and made
several demands upon Sinforoso Ona, the surviving husband of the deceased, for the
delivery to her of the property inventoried and of which he was in possession.
The appellants Sinforoso Ona and Apolinario Gonzales argue that the will in question
was not executed in the form prescribed by section 618 of the Code of Civil Procedure
as amended by Act No. 2645. That section reads as follows:
No will, except as provided in the preceding section, shall be valid to pass any
estate, real or personal, nor charge or affect the same, unless it be written in the
language or dialect known by the testator and signed by him, or by the testator's
name written by some other person in his presence, and by his express
direction, and attested and subscribed by three or more credible witnesses in
the presence of the testator and of each other. The testator or the person
requested by him to write his name and the instrumental witnesses of the will,
shall also sign, as aforesaid, each and every page thereof, on the left margin,
and said pages shall be numbered correlatively in letters placed on the upper
part of each sheet. The attestation shall state the number of sheets or pages
used, upon which the will is written, and the fact that the testator signed the will
and every page thereof, or caused some other person to write his name, under
his express direction, in the presence of three witnesses, and the latter
witnessed and signed the will and all pages thereof in the presence of the
testator and of each other.

The principal points raised by the appeal are (1) that the person requested to sign the
name of the testatrix signed only the latter's name and not her own; (2) that the
attestation clause does not mention the placing of the thumb-mark of the testatrix in the
will; and (3) that the fact that the will had been signed in the presence of the witnesses
was not stated in the attestation clause but only in the last paragraph of the body of the
will.
The opinion quoted is exactly in point. The testatrix thumb-mark appears in the center
of her name as written by Serapia de Gala on all of the pages of the will.
The second and third points raised by Sinforoso Ona and Apolinario Gonzales are
sufficiently refuted by quoting the last clause of the body of the will together with the
attestation clause, both of which are written in the Tagalog dialect.

As will be seen, it is not mentioned in the attestation clause that the testatrix signed by
thumb-mark, but it does there appear that the signature was affixed in the presence of
the witnesses, and the form of the signature is sufficiently described and explained in
the last clause of the body of the will. It maybe conceded that the attestation clause is
not artistically drawn and that, standing alone, it does not quite meet the requirements
of the statute, but taken in connection with the last clause of the body of the will, it is
fairly clear and sufficiently carries out the legislative intent; it leaves no possible doubt
as to the authenticity of the document.
The contention of the appellants Sinforoso Ona and Apolinario Gonzales that the fact
that the will had been signed in the presence of the witnesses was not stated in the
attestation clause is without merit; the fact is expressly stated in that clause.
In our opinion, the will is valid, and the orders appealed from are hereby affirmed
without costs. So ordered.

G.R. No. 103554 May 28, 1993


TEODORO CANEDA vs. CA
The records show that on December 5, 1978, Mateo Caballero, a widower without any
children and already in the twilight years of his life, executed a last will and testament at
his residence in Talisay, Cebu before three attesting witnesses, namely, Cipriano
Labuca, Gregorio Cabando and Flaviano Toregosa. The said testator was duly assisted
by his lawyer, Atty. Emilio Lumontad, and a notary public, Atty. Filoteo Manigos, in the
preparation of that last will. 1 It was declared therein, among other things, that the
testator was leaving by way of legacies and devises his real and personal properties to
Presentacion Gaviola, Angel Abatayo, Rogelio Abatayo, Isabelito Abatayo, Benoni G.
Cabrera and Marcosa Alcantara, all of whom do not appear to be related to the
testator. 2
Four months later, or on April 4, 1979, Mateo Caballero himself filed a petition but he
passed away before his petition could finally be heard by the probate court. Thereafter,
herein petitioners, claiming to be nephews and nieces of the testator, instituted a
second petition opposed thereat the probate of the Testator's will and the appointment
of a special administrator for his estate. 5
ISSUE:
W/N THE will in question is null and void for the reason that its attestation clause is
fatally defective since it fails to specifically state that the instrumental witnesses to the
will witnessed the testator signing the will in their presence and that they also signed
the will and all the pages thereof in the presence of the testator and of one another.
An attestation clause refers to that part of an ordinary will whereby the attesting
witnesses certify that the instrument has been executed before them and to the manner
of the execution the same. 19 It is a separate memorandum or record of the facts
surrounding the conduct of execution and once signed by the witnesses, it gives
affirmation to the fact that compliance with the essential formalities required by law has
been observed. 20 It is made for the purpose of preserving in a permanent form a record
of the facts that attended the execution of a particular will, so that in case of failure of
the memory of the attesting witnesses, or other casualty, such facts may still be
proved. 21

Under the third paragraph of Article 805, such a clause, the complete lack of which
would result in the invalidity of the will, 22 should state (1) the number of the pages
used upon which the will is written; (2) that the testator signed, or expressly caused
another to sign, the will and every page thereof in the presence of the attesting
witnesses; and (3) that the attesting witnesses witnessed the signing by the testator of
the will and all its pages, and that said witnesses also signed the will and every page
thereof in the presence of the testator and of one another.
The purpose of the law in requiring the clause to state the number of pages on which
the will is written is to safeguard against possible interpolation or omission of one or
some of its pages and to prevent any increase or decrease in the pages; 23 whereas the
subscription of the signature of the testator and the attesting witnesses is made for the
purpose of authentication and identification, and thus indicates that the will is the very
same instrument executed by the testator and attested to by the witnesses. 24
Further, by attesting and subscribing to the will, the witnesses thereby declare the due
execution of the will as embodied in the attestation clause. 25 The attestation clause,
therefore, provide strong legal guaranties for the due execution of a will and to insure
the authenticity thereof. 26 As it appertains only to the witnesses and not to the testator,
it need be signed only by them. 27Where it is left unsigned, it would result in the
invalidation of the will as it would be possible and easy to add the clause on a
subsequent occasion in the absence of the testator and its witnesses. 28
2. An examination of the last will and testament of Mateo Caballero shows that it is
comprised of three sheets all of which have been numbered correlatively, with the left
margin of each page thereof bearing the respective signatures of the testator and the
three attesting witnesses. The part of the will containing the testamentary dispositions is
expressed in the Cebuano-Visayan dialect and is signed at the foot thereof by the
testator. The attestation clause in question, on the other hand, is recited in the English
language and is likewise signed at the end thereof by the three attesting witnesses
hereto. 30
It will be noted that Article 805 requires that the witness should both attest and
subscribe to the will in the presence of the testator and of one another. "Attestation"
and "subscription" differ in meaning. Attestation is the act of senses, while subscription
is the act of the hand. The former is mental, the latter mechanical, and to attest a will is
to know that it was published as such, and to certify the facts required to constitute an

actual and legal publication; but to subscribe a paper published as a will is only to write
on the same paper the names of the witnesses, for the sole purpose of identification. 31
In Taboada vs. Rizal, 32 we clarified that attestation consists in witnessing the testator's
execution of the will in order to see and take note mentally that those things are done
which the statute requires for the execution of a will and that the signature of the
testator exists as a fact. On the other hand, subscription is the signing of the witnesses'
names upon the same paper for the purpose of identification of such paper as the will
which was executed by the testator. As it involves a mental act, there would be no
means, therefore, of ascertaining by a physical examination of the will whether the
witnesses had indeed signed in the presence of the testator and of each other unless
this is substantially expressed in the attestation.
What is fairly apparent upon a careful reading of the attestation clause herein assailed
is the fact that while it recites that the testator indeed signed the will and all its pages in
the presence of the three attesting witnesses and states as well the number of pages
that were used, the same does not expressly state therein the circumstance that said
witnesses subscribed their respective signatures to the will in the presence of the
testator and of each other.
The phrase "and he has signed the same and every page thereof, on the spaces
provided for his signature and on the left hand margin," obviously refers to the testator
and not the instrumental witnesses as it is immediately preceded by the words "as his
Last Will and Testament." On the other hand, although the words "in the presence of
the testator and in the presence of each and all of us" may, at first blush, appear to
likewise signify and refer to the witnesses, it must, however, be interpreted as referring
only to the testator signing in the presence of the witnesses since said phrase
immediately follows the words "he has signed the same and every page thereof, on the
spaces provided for his signature and on the left hand margin." What is then clearly
lacking, in the final logical analysis , is the statement that the witnesses signed the will
and every page thereof in the presence of the testator and of one another.
It is our considered view that the absence of that statement required by law is a fatal
defect or imperfection which must necessarily result in the disallowance of the will that
is here sought to be admitted to probate. Petitioners are correct in pointing out that the
aforestated defect in the attestation clause obviously cannot be characterized as
merely involving the form of the will or the language used therein which would warrant
the application of the substantial compliance rule.

SUBSTANTIAL COMPLIANCE RULE:


Art. 809. In the absence of bad faith, forgery, or fraud, or undue and
improper pressure and influence, defects and imperfections in
the form of attestation or in the language used therein shall not render
the will invalid if it is not proved that the will was in fact executed and
attested in substantial compliance with all the requirements of article
805" (Emphasis supplied.)
. . . The rule must be limited to disregarding those defects that can be supplied by an
examination of the will itself: whether all the pages are consecutively numbered;
whether the signatures appear in each and every page; whether the subscribing
witnesses are three or the will was notarized. All theses are facts that the will itself can
reveal, and defects or even omissions concerning them in the attestation clause can be
safely disregarded. But the total number of pages, and whether all persons required to
sign did so in the presence of each other must substantially appear in the attestation
clause, being the only check against perjury in the probate proceedings. (Emphasis
ours.)
The foregoing considerations do not apply where the attestation clause totally omits the
fact that the attesting witnesses signed each and every page of the will in the presence
of the testator and of each other. 35 In such a situation, the defect is not only in the form
or language of the attestation clause but the total absence of a specific element
required by Article 805 to be specifically stated in the attestation clause of a will.
It may thus be stated that the rule, as it now stands, is that omissions which can be
supplied by an examination of the will itself, without the need of resorting to extrinsic
evidence, will not be fatal and, correspondingly, would not obstruct the allowance to
probate of the will being assailed. However, those omissions which cannot be supplied
except by evidence aliunde would result in the invalidation of the attestation clause and
ultimately, of the will itself.67

G.R. No. L-9089

January 5, 1915

In re the state SOTERA BARRIENTOS. SAMUEL PERRY, vs. VICENTE ELIO


Upon the deceased of Sotera Barrientos, a resident of the municipality of Mambajao,
Province of Misamis, 68 years of age, the wife of Samuel Perry in her third marriage, in
the said municipality on August 31, 1912, two documents were presented in the Court
of First Instance of the said province, each of which, according to those who
respectively presented them, was the last will and testament of the said deceased.

him as her sole heir. There are, therefore, more than sufficient reasons for holding that
the document presented by Vicente Elio for probate as the will of Sotera Barrientos
does not express her true and spontaneous desires. All the circumstances connected
with the alleged execution of that so-called will lead us to this conclusion.

The first document was filed on September 4, 1912, that is, four days after the death of
the testatrix, by Vicente Elio, son of her first husband, and her brother-in-law; and the
second on December 20 of the same year, 1912, by Samuel Perry, her surviving
husband. Perry opposed probate of the first document, and Elio, in turn, that of the
second.

In the case at bar, so many and of such a nature are the acts that were performed by
Vicente Elio with respect to the execution of the alleged will; such was his participation
in those which in turn were performed by the five witnesses sought by him expressly for
that purpose; and such are the suspicions which, with regard to the whole matter, arise
from the very significant circumstance that, although the decedent was survived by her
husband, two sisters and a brother, he was the sole beneficiary under the alleged will, a
document which annulled the one previously executed by the said testatrix in favor of
her husband; that, after closely and carefully examining the evidence introduced at the
trial, not only do those suspicions linger, but we are convinced that the document in
question does not express the true will of the decedent.

On the other hand, the petitioner himself, Elio, prepared the document, so he testified,
from a rough draft which had been furnished him by Sotera Barrientos two or three
days before and which contained the necessary data and instructions. It was also Elio
who, on that same morning of the 26th of August, in his own house, in his presence and
under his direction, after having, as he testified, shown to the testatrix the rough draft
prepared by him, had Santos Matayabas make a clean copy of it and immediately after
the document had been written invited the four witnesses and with them and
Matayabas went to Sotera Barrientos' house. There Elio took a large if not a principal
part in all that was done and in all that happened in the immediate vicinity of the patient.
He also it was who first informed the sick woman of the reason for the presence of
them all in the house, and afterwards spoke to her about the signature. He served as
the sole and direct intermediary between the patient and Matayabas and the others to
inform them that the woman, so he testified, had chosen Matayabas to sign the said
document at her request. He afterwards witnessed all that Matayabas and the rest then
did there. Account must also be taken of the fact that Elio was the only person to be
benefited by the execution of the presented will, for, as hereinbefore stated, besides his
appearing in that document as the sole devisee of all the property of the testatrix, the
statement appears in one of its clauses, as being made by the testatrix, that her
husband, Samuel Perry, had abandoned her in serious sickness a fact that was not
proved and that she revoked and annulled the testamentary provisions previously
made by her, which were no others than those contained in the document presented by
this same Perry for probate as the will of the said deceased and in which she instituted

Furthermore, in order that a will may be deemed valid, that is executed by one person
and signed by another on account of the testator's inability to sign, the law requires
(sec. 618, Code Civ. Proc.) that it shall have been signed under the express direction or
by the express order of the testator. In the present case, as we have already seen,
when Elio and his companions took the said document to the house of Sotera
Barrientos, there to be executed as her will, it already contained a statement in the
paragraph preceding the space reserved for the signatures of the testatrix and the
witnesses, to the effect that, as the testatrix was unable to sign the will by reason of her
advanced age and her debility, she authorized and begged Santos Matayabas to do so
at her request. There is nor proof whatever that Vicente Elio was instructed by Sotera
Barrientos to have the statement inserted in the said document, when, as he testified,
the drafting and preparation of the instrument was commended to him. It is evident,
therefore, that it was all merely the idea and purpose of Elio himself. With respect to
this feature of the case, although from the testimony given by Santos Matayabas and
Feliciano Valdihuesa it is gathered that the testatrix indicated by means of signs, her
desire that the latter should sign the document as she was unable to do so, yet both the
witnesses Rivera and Sabido gave it to be understood, in referring in turn to that
incident, that is was Elio himself who named Santos Matayabas as the person who
should sign for Sotera Barrientos, and this in fact must have been done so, because
Elio said, in explaining also what then occurred there: "As she (Sotera Barrientos) was
unable to sign in spite of her insistence, I told her to choose one of the five men in order
that he might sign at her request. She replied to me in a low voice, only heard by me,

speaking in my ear the word `Santos,' and pointed him out, for he was at her side;" and,
finally, "in order that the rest might know it, I repeated in a loud voice: `Santos
Matayabas?'" and "immediately thereafter Santos Matayabas signed." These
statements all show that it was Elio's suggestion that Sotera Barrientos should select
from among them the one who should sign the document; that he was the only person
who spoke to the sick woman and in a low voice, placing his ear close to her, and who,
as he testified, heard her pronounce the name "Santos;" and, finally, also it was he who
in turn pointed out Santos Matayabas as the party designated by her for the purpose
mentioned. As in the document drawn up and prepared by Elio himself Santos
Matayabas was already designated to sign at the request of Sotera Barrientos, before
the latter was asked by Elio, as he stated, the question mentioned by him; as Elio
himself was to be benefited by the will then attempted to be executed; and as Elio's
intervention in that selection was direct and exclusive, for, as he testified, it was he
alone who heard the word "Santos" a selection which after all was entirely useless,
since the person chosen for the intended purpose was already designated in the
document by Elio himself no other conclusions can be reached than that Santos
Matayabas not only did not sign the said document under the express direction and
order of Sotera Barrientos, but also did not even do so at her request or in obedience to
her own will, because the will of Vicente Elio, who drew up and prepared the document,
was already expressed therein and to his will it appears, was that of Sotera Barrientos'
subordinated in all respects, not only with reference to the signing of the instrument, but
also with regard to all else connected with the alleged execution of the so-called will of
this testatrix.
G.R. No. 1641

January 19, 1906

GERMAN JABONETA, vs. RICARDO GUSTILO, ET AL.,


Being in the house of Arcadio Jarandilla, in Jaro, in this province, he ordered that the
document in question be written, and calling Julio Javellana, Aniceto Jalbuena, and
Isabelo Jena as witnesses, executed the said document as his will. They were all
together, and were in the room where Jaboneta was, and were present when he signed
the document, Isabelo Jena signing afterwards as a witness, at his request, and in his
presence and in the presence of the other two witnesses. Aniceto Jalbuena then signed
as a witness in the presence of the testator, and in the presence of the other two
persons who signed as witnesses. At that moment Isabelo Jena, being in a hurry to
leave, took his hat and left the room. As he was leaving the house Julio Javellana took
the pen in his hand and put himself in position to sign the will as a witness, but did not
sign in the presence of Isabelo Jena; but nevertheless, after Jena had left the room the

said Julio Javellana signed as a witness in the presence of the testator and of the
witness Aniceto Jalbuena.

The fact that Jena was still in the room when he saw Javellana moving his hand and
pen in the act of affixing his signature to the will, taken together with the testimony of
the remaining witnesses which shows that Javellana did in fact there and then sign his
name to the will, convinces us that the signature was affixed in the presence of Jena.
The fact that he was in the act of leaving, and that his back was turned while a portion
of the name of the witness was being written, is of no importance. He, with the other
witnesses and the testator, had assembled for the purpose of executing the testament,
and were together in the same room for that purpose, and at the moment when the
witness Javellana signed the document he was actually and physically present and in
such position with relation to Javellana that he could see everything which took place
by merely casting his eyes in the proper direction, and without any physical obstruction
to prevent his doing so, therefore we are of opinion that the document was in fact
signed before he finally left the room.
In the matter of Bedell (2 Connoly (N.Y.), 328) it was held that it is sufficient if the
witnesses are together for the purpose of witnessing the execution of the will,
and in a position to actually see the testator write, if they choose to do so; and
there are many cases which lay down the rule that the true test of vision is not
whether the testator actually saw the witness sign, but whether he might have
seen him sign, considering his mental and physical condition and position at the
time of the subscription. (Spoonemore vs. Cables, 66 Mo., 579.)

C.A. No. 8075

March 25, 1946

Neyra, as party, and by Dr. M. B. Abad and Eustaquio Mendoza, a protege, as


witnesses.

TRINIDAD NEYRA, vs. ENCARNACION NEYRA,


That Severo Nayra died intestate in the City of Manila, on May 6, 1938, leaving certain
properties and two children, by his first marriage, named Encarnacion Neyra and
Trinidad Neyra, and other children by his second marriage; That after the death of
Severo Neyra, the two sisters, Encarnacion Neyra and Trinidad Neyra, had serious
misunderstandings, in connection with the properties left by their deceased father.
That Encarnacion Neyra, who had remained single, and who had no longer any
ascendants, executed a will on September 14, 1939, marked Exhibit 16, disposing of
her properties in favor of a congregation but was REJECTED.
In the meanwhile, Encarnacion Neyra had become seriously ill, suffering from Addison's
disease. The sisters reconciled and agreed that they will have the cases dismissed, on
the condition that the property involved therein should be given exclusively to Trinidad
Neyra, that the latter should waive her share in the rents of said property collected by
Encarnacion, and the Trinidad had no more indebtedness to Encarnacion. They also
agreed to send for Atty. Alejandro M. Panis, to prepare the necessary document
embodying the said agreement, but Attorney Panis could come only in the afternoon of
the following day, November 2, 1942, when Encarnacion gave him instructions for the
preparation of the document embodying their agreement, and other instructions for the
preparation of her last will and testament; that Attorney Panis prepared said document
of compromise as well as the new will and testament, naming Trinidad Neyra and
Eustaquio Mendoza beneficiaries therein, pursuant to Encarnacion's express
instructions, and the two documents were prepared, in duplicate, and were ready for
signature, since the morning of November 3, 1942; that in the afternoon of that day, of
compromise and last will and testament to Encarnacion Neyra, slowly and in a loud
voice, in the presence of Father Teodoro Garcia, Dr. Moises B. Abad, Dr. Eladio
Aldecoa, Trinidad Neyra, and others, after which he asked her if their terms were in
accordance with her wishes, or if she wanted any change made in said documents; that
Encarnacion Neyra did not suggest any change, and asked for the pad and the two
documents, and, with the help of a son of Trinidad, placed her thumbmark at the foot of
each one of the two documents, in duplicate, on her bed in the sala, in the presence of
attesting witnesses, Dr. Moises B. Abad, Dr. Eladio R. Aldecoa and Atty. Alejandro M.
Panis, after which said witnesses signed at the foot of the will, in the presence of
Encarnacion Neyra, and of each other. The agreement was also signed by Trinidad

Tua vs. Yap Ca Kuan and Yap Ca Llu, 27 Phil., 579.) The testimony of the attending
physician that the deceased was suffering from diabetes and had been in a comatose
condition for several days, prior to his death, was held not sufficient to establish
testamentary incapacity, in view of the positive statement of several credible witnesses
that he was conscious and able to understand what was said to him and to
communicate his desires. (Samson vs. Corrales Tan Quintin, 44 Phil., 573.) Where the
mind of the testator is in perfectly sound condition, neither old age, nor ill health, nor the
fact that somebody had to guide his hand in order that he might sign, is sufficient to
invalidate his will (Amata and Almojuela vs. Tablizo, 48 Phil., 485.)
It may, therefore, be reasonably concluded that the mental faculties of persons suffering
from Addison's disease, like the testatrix in this case, remain unimpaired, partly due to
the fact that, on account of the sleep they enjoy, they necessarily receive the benefit of
physical and mental rest. And that like patients suffering from tuberculosis, insomnia or
diabetes, they preserve their mental faculties until the moments of their death.
The contention that the attesting witnesses were not present, at the time Encarnacion
Neyra thumbmarked the agreement and will in question, on her bed, in the sala of the
house, as they were allegedly in the caida, is untenable. It has been fully shown that
said witnesses were present, at the time of the signing and execution of the agreement
and will in question, in the sala, where the testatrix was lying on her bed. The true test
is not whether they actually saw each other at the time of the signing of the documents,
but whether they might have seen each other sign, had they chosen to do so; and the
attesting witnesses actually saw it all in this case. (Jaboneta vs. Gustilo, 5 Phil., 541.)
And the thumbmark placed by the testatrix on the agreement and will in question is
equivalent to her signature. (Yap Tua vs. Yap Ca Kuan and Yap Ca Llu, 27 Phil., 579.)
It having been shown that the said compromise or agreement had been legally signed
and executed by Encarnacion Neyra on November 3, 1942, in the presence of credible
and trustworthy witnesses, and that she was compos mentis and possessed the
necessary testamentary and mental capacity of the time.
G.R. No. 15566

September 14, 1921

EUTIQUIA AVERA, vs. MARINO GARCIA

Whethet it is essential to the validity of a will in this jurisdiction that the names of
the testator and the instrumental witnesses should be written on the left margin
of each page, as required in said Act, and not upon the right margin, as in the will
now before us; and upon this we are of the opinion that the will in question is valid. It is
true that the statute says that the testator and the instrumental witnesses shall sign
their names on the left margin of each and every page; and it is undeniable that the
general doctrine is to the effect that all statutory requirements as to the execution of
wills must be fully complied with. The same doctrine is also deducible from cases
heretofore decided by this court.
So far as concerns the authentication of the will, and of every part thereof, it can make
no possible difference whether the names appear on the left or no the right margin,
provided they are on one or the other. In Caraig vs. Tatlonghari (R. G. No. 12558,
decided March 23, 1918, not reported), this court declared a will void which was totally
lacking in the signatures required to be written on its several pages; and in the case of
Re estate of Saguinsin (41 Phil., 875), a will was likewise declared void which
contained the necessary signatures on the margin of each leaf ( folio), but not in the
margin of each page containing written matter.
The instrument now before us contains the necessary signatures on every page, and
the only point of deviation from the requirement of the statute is that these signatures
appear in the right margin instead of the left. By the mode of signing adopted every
page and provision of the will is authenticated and guarded from possible alteration in
exactly the same degree that it would have been protected by being signed in the left
margin; and the resources of casuistry could be exhausted without discovering the
slightest difference between the consequences of affixing the signatures in one margin
or the other.
The same could not be said of a case like that of Estate of Saguinsin, supra, where
only the leaves, or alternate pages, were signed and not each written page; for as
observed in that case by our late lamented Chief Justice, it was possible that in the will
as there originally executed by the testratrix only the alternative pages had been used,
leaving blanks on the reverse sides, which conceivably might have been filled in
subsequently.
The controlling considerations on the point now before us were well stated In Re will of
Abangan (40 Phil., 476, 479), where the court, speaking through Mr. Justice Avancea,
in a case where the signatures were placed at the bottom of the page and not in the
margin, said:

The object of the solemnities surrounding the execution of wills is to close the
door against bad faith and fraud, to avoid substitution o will and testaments and
to guarantee their truth and authenticity. Therefore the laws on this subject
should be interpreted in such a way as to attain these primordial ends. But, on
the other hand, also one must not lose sight of the fact that it is not the object of
the law to restrain and curtail the exercise of the right to make a will. So when
an interpretation already given assures such ends, any other interpretation
whatsoever, that adds nothing but demands more requisites entirely
unnecessary, useless and frustrative of the testator's last will, must be
disregarded.
In the case before us, where ingenuity could not suggest any possible prejudice to any
person, as attendant upon the actual deviation from the letter of the law, such deviation
must be considered too trivial to invalidate the instrument.
G.R. No. L-18979

June 30, 1964

CELSO ICASIANO, vs. NATIVIDAD ICASIANO


Appeal from an order of the Court of First Instance of Manila admitting to probate the
document and its duplicate, marked as Exhibits "A" and "A-1", as the true last will and
testament of Josefa Villacorte, deceased, and appointing as executor Celso Icasiano,
the person named therein as such.
This special proceeding was begun on October 2, 1958 by a petition for the allowance
and admission to probate of the original, Exhibit "A" as the alleged will of Josefa
Villacorte, deceased, and for the appointment of petitioner Celso Icasiano as executor
thereof.
On March 19, 1959, the petitioner proponent commenced the introduction of his
evidence; but on June 1, 1959, he filed a motion for the admission of an amended and
supplemental petition, alleging that the decedent left a will executed in duplicate with all
the legal requirements, and that he was, on that date, submitting the signed duplicate
(Exhibit "A-1"), which he allegedly found only on or about May 26, 1959. On June 17,
1959, oppositors Natividad Icasiano de Gomez and Enrique Icasiano filed their joint
opposition to the admission of the amended and supplemental petition, but by order of
July 20, 1959, the court admitted said petition, and on July 30, 1959, oppositor
Natividad Icasiano filed her amended opposition. Thereafter, the parties presented their
respective evidence, and after several hearings the court issued the order admitting the

will and its duplicate to probate. From this order, the oppositors appealed directly to this
Court, the amount involved being over P200,000.00, on the ground that the same is
contrary to law and the evidence.
The evidence presented for the petitioner is to the effect that Josefa Villacorte died in
the City of Manila on September 12, 1958; that on June 2, 1956, the late Josefa
Villacorte executed a last will and testament in duplicate at the house of her daughter
Mrs. Felisa Icasiano at Pedro Guevara Street, Manila, published before and attested by
three instrumental witnesses, namely: attorneys Justo P. Torres, Jr. and Jose V.
Natividad, and Mr. Vinicio B. Diy; that the will was acknowledged by the testatrix and by
the said three instrumental witnesses on the same date before attorney Jose Oyengco
Ong, Notary Public in and for the City of Manila; and that the will was actually prepared
by attorney Fermin Samson, who was also present during the execution and signing of
the decedent's last will and testament, together with former Governor Emilio Rustia of
Bulacan, Judge Ramon Icasiano and a little girl. Of the said three instrumental
witnesses to the execution of the decedent's last will and testament, attorneys Torres
and Natividad were in the Philippines at the time of the hearing, and both testified as to
the due execution and authenticity of the said will. So did the Notary Public before
whom the will was acknowledged by the testatrix and attesting witnesses, and also
attorneys Fermin Samson, who actually prepared the document. The latter also testified
upon cross examination that he prepared one original and two copies of Josefa
Villacorte last will and testament at his house in Baliuag, Bulacan, but he brought only
one original and one signed copy to Manila, retaining one unsigned copy in Bulacan.
The records show that the original of the will, which was surrendered simultaneously
with the filing of the petition and marked as Exhibit "A" consists of five pages, and while
signed at the end and in every page, it does not contain the signature of one of the
attesting witnesses, Atty. Jose V. Natividad, on page three (3) thereof; but the duplicate
copy attached to the amended and supplemental petition and marked as Exhibit "A-1"
is signed by the testatrix and her three attesting witnesses in each and every page.
The testimony presented by the proponents of the will tends to show that the original of
the will and its duplicate were subscribed at the end and on the left margin of each and
every page thereof by the testatrix herself and attested and subscribed by the three
mentioned witnesses in the testatrix's presence and in that of one another as witnesses
(except for the missing signature of attorney Natividad on page three (3) of the original);
that pages of the original and duplicate of said will were duly numbered; that the
attestation clause thereof contains all the facts required by law to be recited therein and
is signed by the aforesaid attesting witnesses; that the will is written in the language

known to and spoken by the testatrix that the attestation clause is in a language also
known to and spoken by the witnesses; that the will was executed on one single
occasion in duplicate copies; and that both the original and the duplicate copies were
duly acknowledged before Notary Public Jose Oyengco of Manila on the same date
June 2, 1956.
Witness Natividad who testified on his failure to sign page three (3) of the original,
admits that he may have lifted two pages instead of one when he signed the same, but
affirmed that page three (3) was signed in his presence.
Oppositors-appellants in turn introduced expert testimony to the effect that the
signatures of the testatrix in the duplicate (Exhibit "A-1") are not genuine nor were they
written or affixed on the same occasion as the original, and further aver that granting
that the documents were genuine, they were executed through mistake and with undue
influence and pressure because the testatrix was deceived into adopting as her last will
and testament the wishes of those who will stand to benefit from the provisions of the
will, as may be inferred from the facts and circumstances surrounding the execution of
the will and the provisions and dispositions thereof, whereby proponents-appellees
stand to profit from properties held by them as attorneys-in-fact of the deceased and
not enumerated or mentioned therein, while oppositors-appellants are enjoined not to
look for other properties not mentioned in the will, and not to oppose the probate of it,
on penalty of forfeiting their share in the portion of free disposal.
We have examined the record and are satisfied, as the trial court was, that the testatrix
signed both original and duplicate copies (Exhibits "A" and "A-1", respectively) of the
will spontaneously, on the same in the presence of the three attesting witnesses, the
notary public who acknowledged the will; and Atty. Samson, who actually prepared the
documents; that the will and its duplicate were executed in Tagalog, a language known
to and spoken by both the testator and the witnesses, and read to and by the testatrix
and Atty. Fermin Samson, together before they were actually signed; that the
attestation clause is also in a language known to and spoken by the testatrix and the
witnesses. The opinion of expert for oppositors, Mr. Felipe Logan, that the signatures of
the testatrix appearing in the duplicate original were not written by the same had which
wrote the signatures in the original will leaves us unconvinced, not merely because it is
directly contradicted by expert Martin Ramos for the proponents, but principally
because of the paucity of the standards used by him to support the conclusion that the
differences between the standard and questioned signatures are beyond the writer's
range of normal scriptural variation. The expert has, in fact, used as standards only
three other signatures of the testatrix besides those affixed to the original of the

testament (Exh. A); and we feel that with so few standards the expert's opinion and the
signatures in the duplicate could not be those of the testatrix becomes extremely
hazardous. This is particularly so since the comparison charts Nos. 3 and 4 fail to show
convincingly that the are radical differences that would justify the charge of forgery,
taking into account the advanced age of the testatrix, the evident variability of her
signatures, and the effect of writing fatigue, the duplicate being signed right the original.
These, factors were not discussed by the expert.
Similarly, the alleged slight variance in blueness of the ink in the admitted and
questioned signatures does not appear reliable, considering the standard and
challenged writings were affixed to different kinds of paper, with different surfaces and
reflecting power. On the whole, therefore, we do not find the testimony of the
oppositor's expert sufficient to overcome that of the notary and the two instrumental
witnesses, Torres and Natividad (Dr. Diy being in the United States during the trial, did
not testify).
Nor do we find adequate evidence of fraud or undue influence. The fact that some heirs
are more favored than others is proof of neither (see In re Butalid, 10 Phil. 27; Bugnao
vs. Ubag, 14 Phil. 163; Pecson vs. Coronal, 45 Phil. 216). Diversity of apportionment is
the usual reason for making a testament; otherwise, the decedent might as well die
intestate. The testamentary dispositions that the heirs should not inquire into other
property and that they should respect the distribution made in the will, under penalty of
forfeiture of their shares in the free part do not suffice to prove fraud or undue influence.
They appear motivated by the desire to prevent prolonged litigation which, as shown by
ordinary experience, often results in a sizeable portion of the estate being diverted into
the hands of non-heirs and speculators. Whether these clauses are valid or not is a
matter to be litigated on another occassion. It is also well to note that, as remarked by
the Court of Appeals in Sideco vs. Sideco, 45 Off. Gaz. 168, fraud and undue influence
are mutually repugnant and exclude each other; their joining as grounds for opposing
probate shows absence of definite evidence against the validity of the will.
On the question of law, we hold that the inadvertent failure of one witness to affix his
signature to one page of a testament, due to the simultaneous lifting of two pages in the
course of signing, is not per se sufficient to justify denial of probate. Impossibility of
substitution of this page is assured not only the fact that the testatrix and two other
witnesses did sign the defective page, but also by its bearing the coincident imprint of
the seal of the notary public before whom the testament was ratified by testatrix and all
three witnesses. The law should not be so strictly and literally interpreted as to penalize
the testatrix on account of the inadvertence of a single witness over whose conduct she

had no control, where the purpose of the law to guarantee the identity of the testament
and its component pages is sufficiently attained, no intentional or deliberate deviation
existed, and the evidence on record attests to the full observance of the statutory
requisites. Otherwise, as stated in Vda. de Gil. vs. Murciano, 49 Off. Gaz. 1459, at
1479 (decision on reconsideration) "witnesses may sabotage the will by muddling or
bungling it or the attestation clause".
That the failure of witness Natividad to sign page three (3) was entirely through pure
oversight is shown by his own testimony as well as by the duplicate copy of the will,
which bears a complete set of signatures in every page. The text of the attestation
clause and the acknowledgment before the Notary Public likewise evidence that no one
was aware of the defect at the time.
This would not be the first time that this Court departs from a strict and literal
application of the statutory requirements, where the purposes of the law are otherwise
satisfied. Thus, despite the literal tenor of the law, this Court has held that a testament,
with the only page signed at its foot by testator and witnesses, but not in the left margin,
could nevertheless be probated (Abangan vs. Abangan, 41 Phil. 476); and that despite
the requirement for the correlative lettering of the pages of a will, the failure to make the
first page either by letters or numbers is not a fatal defect (Lopez vs. Liboro, 81 Phil.
429). These precedents exemplify the Court's policy to require satisfaction of the legal
requirements in order to guard against fraud and bid faith but without undue or
unnecessary curtailment of the testamentary privilege.
The appellants also argue that since the original of the will is in existence and available,
the duplicate (Exh. A-1) is not entitled to probate. Since they opposed probate of
original because it lacked one signature in its third page, it is easily discerned that
oppositors-appellants run here into a dilemma; if the original is defective and invalid,
then in law there is no other will but the duly signed carbon duplicate (Exh. A-1), and
the same is probatable. If the original is valid and can be probated, then the objection to
the signed duplicate need not be considered, being superfluous and irrelevant. At any
rate, said duplicate, Exhibit A-1, serves to prove that the omission of one signature in
the third page of the original testament was inadvertent and not intentional.
That the carbon duplicate, Exhibit A-1, was produced and admitted without a new
publication does not affect the jurisdiction of the probate court, already conferred by the
original publication of the petition for probate. The amended petition did not
substantially alter the one first filed, but merely supplemented it by disclosing the
existence of the duplicate, and no showing is made that new interests were involved

(the contents of Exhibit A and A-1 are admittedly identical); and appellants were duly
notified of the proposed amendment. It is nowhere proved or claimed that the
amendment deprived the appellants of any substantial right, and we see no error in
admitting the amended petition.
IN VIEW OF THE FOREGOING, the decision appealed from is affirmed, with costs
against appellants.

G.R. No. 145545

June 30, 2008

PAZ SAMANIEGO-CELADA, petitioner, vs. LUCIA D. ABENA, respondent.

the will is the whole instrument consisting of three (3) pages inclusive of the attestation
clause and the acknowledgement. The position of the court is in consonance with the
"doctrine of liberal interpretation" enunciated in Article 809 of the Civil Code which
reads:

DECISION
Petitioner Paz Samaniego-Celada was the first cousin of decedent Margarita S.
Mayores (Margarita) while respondent was the decedents lifelong companion since
1929.
On April 27, 1987, Margarita died single and without any ascending nor descending
heirs as her parents, grandparents and siblings predeceased her. She was survived by
her first cousins Catalina Samaniego-Bombay, Manuelita Samaniego Sajonia, Feliza
Samaniego, and petitioner.
Before her death, Margarita executed a Last Will and Testament3 on February 2, 1987
where she bequeathed one-half of her undivided share of a real property located at
Singalong Manila, consisting of 209.8 square meters, and covered by Transfer
Certificate of Title (TCT) No. 1343 to respondent, Norma A. Pahingalo, and Florentino
M. Abena in equal shares or one-third portion each. She likewise bequeathed one-half
of her undivided share of a real property located at San Antonio Village, Makati,
consisting of 225 square meters, and covered by TCT No. 68920 to respondent,
Isabelo M. Abena, and Amanda M. Abena in equal shares or one-third portion each.
Margarita also left all her personal properties to respondent whom she likewise
designated as sole executor of her will.
ISSUE:
w/n will is fatally defective for the reason that its attestation clause states that the will
is composed of three (3) pages while in truth and in fact, the will consists of two
(2) pages only because the attestation is not a part of the notarial will.
While it is true that the attestation clause is not a part of the will, the court, after
examining the totality of the will, is of the considered opinion that error in the number of
pages of the will as stated in the attestation clause is not material to invalidate the
subject will. It must be noted that the subject instrument is consecutively lettered with
pages A, B, and C which is a sufficient safeguard from the possibility of an omission of
some of the pages. The error must have been brought about by the honest belief that

"In the absence of bad faith, forgery or fraud, or undue [and] improper pressure
and influence, defects and imperfections in the form of attestation or in the
language used therein shall not render the will invalid if it is proved that the will
was in fact executed and attested in substantial compliance with all the
requirements of Article 805."
G.R. No. L-21151

February 25, 1924

In re will of Antonio Vergel de Dios, deceased.


RAMON J. FERNANDEZ, vs. FERNANDO VERGEL DE DIOS, ET AL.,
As to the numbering of the sheet containing the attestation clause, it is true that it does
not appeal on the upper part of the sheet, but it does not appear in its text, the pertinent
part of which is copied hereinafter, with the words, having reference to the number of
sheets of the will, underscored, including the page number of the attestation:
* * * We certify that the foregoing document written in Spanish, a language
known by the testator Antonino Vergel de Dios, consisting of three sheet
actually used, correlatively enumerated, besides this sheet . . . .
If, as stated in this clause, the foregoing document consists of three sheets, besides
that of the clause itself, which is in singular, it is clear that such a sheet of the
attestation clause is the fourth and that the will, including said sheet, has four sheets.
This description contained in the clause in question constitutes substantial compliance
with the requirements prescribed by the law regarding the paging. So it was held by this
Court in the case ofAbangan vs. Abangan (40 Phil., 476), where the sheet containing
the attestation, as well as the preceding one, was also not paged. Furthermore the
law, as we shall see later on, does not require that the sheet containing nothing
but the attestation clause, wholly or in part, be numbered or paged.
Consequently this lack of paging on the attestation sheet does not take anything
from the validity of the will.
Turning now to the question whether or not in this clause it is stated that the testator
signed on the margin of each sheet of the will, in the presence of the witnesses and the
latter in the presence of each other, let us see what is said in said clause on this point,
and to this end its pertinent part is hereinafter transcribed and is as follows:
* * * and he (the testator) signed at the bottom of the aforesaid will in our
presence and we at his request did the same in his presence and in that of each

other as witnesses to the will, and lastly, the testator, as well as we, as
witnesses, signed in the same manner on the left margin of each sheet.
(Emphasis ours.)
Concerning the absolute absence of the signature of the testator from the sheet
containing the attestation clause, this point was already decided in the above cited case
of Abangan vs. Abangan, where this court held that:
The testator's signature is not necessary in the attestation clause because
this, as its name implies, appertains only to the witnesses and not to the
testator.
In that case of Abangan vs. Abangan it was held that the signature of the testator is not
necessary in the attestation clause, but the theory is not announced that such a clause
is unnecessary to the validity to the will.
For this reason such doctrine does not annul the judgment in the case of Uy Coque vs.
Navas L. Sioca (43 Phil., 405), where in effect the doctrine, among others, was laid
down that the attestation clause is necessary to the validity of the will. One of the points
on which greatest stress was laid in that case Uy Coque is that the requirements of the
law regarding the number of the pages used, the signing of the will and of each of its
pages by the testator in the presence of three witnesses, and the attestation and
signing of the will and of each of its pages by the witnesses in the presence of each
other cannot be proven aliunde but by the attestation clause itself which must express
the complaince of the will with such requirements. But it was not held in that case of Uy
Coque that the signature of the testator was necessary in the attestation clause, nor
was such point discussed there, which was the point at issue in the case of Abangan
vs. Abangan, supra.
In the case at bar the attestation clause in question states that the requirements
prescribed for the will were complied with, and this is enough for it, as such attestation
clause, to be held as meeting the requirements prescribed by the law for it.
The fact that in said clause the signature of the testator does not appear does not affect
its validity, for, as above stated, the law does not require that it be signed by the
testator.
G.R. No. L-1787

August 27, 1948

Testacy of Sixto Lopezvs. AGUSTIN LIBORO,


The will in question comprises two pages, each of which is written on one side of a
separate sheet. The first sheet is not paged either in letters or in Arabic numerals. This,
the appellant believes, is a fatal defect.
The purpose of the law in prescribing the paging of wills is guard against fraud, and to
afford means of preventing the substitution or of defecting the loss of any of its pages.
(Abangan vs. Abangan, 40 Phil., 476.) In the present case, the omission to put a page

number on the first sheet, if that be necessary, is supplied by other forms of


identification more trustworthy than the conventional numerical words or characters.
The unnumbered page is clearly identified as the first page by the internal sense of its
contents considered in relation to the contents of the second page. By their meaning
and coherence, the first and second lines on the second page are undeniably a
continuation of the last sentence of the testament, before the attestation clause, which
starts at the bottom of the preceding page. Furthermore, the unnumbered page
contains the caption "TESTAMENTO," the invocation of the Almighty, and a recital that
the testator was in full use of his testamentary faculty, all of which, in the logical
order of sequence, precede the direction for the disposition of the marker's property.
Again, as page two contains only the two lines above mentioned, the attestation clause,
the mark of the testator and the signatures of the witnesses, the other sheet can not by
any possibility be taken for other than page one. Abangan vs. Abangan,supra,
and Fernandez vs. Vergel de Dios, 46 Phil., 922 are decisive of this issue.
The testator affixed his thumbmark to the instrument instead of signing his name. The
reason for this was that the testator was suffering from "partial paralysis." While another
in testator's place might have directed someone else to sign for him, as appellant
contends should have been done, there is nothing curious or suspicious in the fact that
the testator chose the use of mark as the means of authenticating his will. It was a
matter of taste or preference. Both ways are good. A statute requiring a will to be
"signed" is satisfied if the signature is made by the testator's mark. (De Gala vs.
Gonzales and Ona, 53 Phil., 108; 28 R. C. L., 117.)

G.R. No. 189984

November 12, 2012

LOPEZ, Petitioner, vs. DIANA JEANNE LOPEZ,


Richard filed a petition for the probate of his father's Last Will and Testament before the
RTC of Manila with prayer for the issuance of letters testamentary in his favor.
Marybeth opposed the petition contending that the purported last will and testament
was not executed and attested as required by law, and that it was procured by undue
and improper pressure and influence on the part of Richard. The said opposition was
also adopted by Victoria.
After submitting proofs of compliance with jurisdictional requirements, Richard
presented the attesting witnesses, namely: Reynaldo Maneja; Romulo Monteiro; Ana
Maria Lourdes Manalo (Manalo); and the notary public who notarized the will, Atty.
Perfecto Nolasco (Atty. Nolasco). The instrumental witnesses testified that after the late
Enrique read and signed the will on each and every page, they also read and signed
the same in the latter's presence and of one another. Photographs of the incident were
taken and presented during trial. Manalo further testified that she was the one who
prepared the drafts and revisions from Enrique before the final copy of the will was
made.
Likewise, Atty. Nolasco claimed that Enrique had been his client for more than 20 years.
Prior to August 10, 1996, the latter consulted him in the preparation of the subject will
and furnished him the list of his properties for distribution among his children. He
prepared the will in accordance with Enrique's instruction and that before the latter and
the attesting witnesses signed it in the presence of one another, he translated the will
which was written in English to Filipino and added that Enrique was in good health and
of sound mind at that time.
On the other hand, the oppositors presented its lone witness, Gregorio B. Paraon
(Paraon), Officer-in-Charge of the Notarial Section, Office of the Clerk of Court, RTC,
Manila. His testimony centered mainly on their findings that Atty. Nolasco was not a
notary public for the City of Manila in 1996, which on cross examination was clarified
after Paraon discovered that Atty. Nolasco was commissioned as such for the years
1994 to 1997.
the CA found no valid reason to deviate from the findings of the RTC that the failure to
state the number of pages of the will in the attestation clause was fatal. It noted that
while Article 809 of the Civil Code sanctions mere substantial compliance with
the formal requirements set forth in Article 805 thereof, there was a total
omission of such fact in the attestation clause. Moreover, while the
acknowledgment of the will made mention of "7 pages including the page on which the
ratification and acknowledgment are written," the will had actually 8 pages including the
acknowledgment portion thus, necessitating the presentation of evidence aliunde to
explain the discrepancy.
G.R. No. 122880

April 12, 2006

FELIX AZUELA, Petitioner, vs. COURT OF APPEALS, GERALDA AIDA CASTILLO


substituted by ERNESTO G. CASTILLO, Respondents.
DECISION
TINGA, J.:
The three named witnesses to the will affixed their signatures on the left-hand margin of
both pages of the will, but not at the bottom of the attestation clause.
Oppositor Geralda Castillo also argued that the will was not executed and attested to in
accordance with law. She pointed out that decedents signature did not appear on the
second page of the will, and the will was not properly acknowledged. These twin
arguments are among the central matters to this petition.
As admitted by petitioner himself, the attestation clause fails to state the number of
pages of the will.12 There was an incomplete attempt to comply with this requisite, a
space having been allotted for the insertion of the number of pages in the attestation
clause. Yet the blank was never filled in; hence, the requisite was left uncomplied with.
ANDRADA CASE:
The ratio decidendi of these cases seems to be that the attestation clause must contain
a statement of the number of sheets or pages composing the will and that if this is
missing or is omitted, it will have the effect of invalidating the will if the deficiency
cannot be supplied, not by evidence aliunde, but by a consideration or examination of
the will itself. But here the situation is different. While the attestation clause does not
state the number of sheets or pages upon which the will is written, however, the
last part of the body of the will contains a statement that it is composed of eight
pages, which circumstance in our opinion takes this case out of the rigid rule of
construction and places it within the realm of similar cases where a broad and more
liberal view has been adopted to prevent the will of the testator from being defeated by
purely technical considerations." (page 165-165, supra) (Underscoring supplied)
In "Apolonio Tabaoda versus Hon. Avelino Rosal, et al."::
There is substantial compliance in the acknowledgement states that "this Last Will and
Testament consists of two pages including this page" (pages 200-201, supra)
(Underscoring supplied).
However, in the appeal at bench, the number of pages used in the will is not stated in
any part of the Will. The will does not even contain any notarial acknowledgment
wherein the number of pages of the will should be stated.21
The failure of the attestation clause to state the number of pages on which the will was
written remains a fatal flaw, despite Article 809. The purpose of the law in requiring the
clause to state the number of pages on which the will is written is to safeguard against
possible interpolation or omission of one or some of its pages and to prevent any

increase or decrease in the pages.33 The failure to state the number of pages equates
with the absence of an averment on the part of the instrumental witnesses as to how
many pages consisted the will, the execution of which they had ostensibly just
witnessed and subscribed to.
in this case, there could have been no substantial compliance with the requirements
under Article 805 since there is no statement in the attestation clause or anywhere in
the will itself as to the number of pages which comprise the will.
WITNESSES SIGNED ON THE ELFT HAND MARGIN OF THE PAGE WHRE THE
ATTESTATION IS BUT NOT AT THE BOTTOM OF THE ATTESTATION CLAUSE:
The respective intents behind these two classes of signature are distinct from each
other. The signatures on the left-hand corner of every page signify, among others, that
the witnesses are aware that the page they are signing forms part of the will. On the
other hand, the signatures to the attestation clause establish that the witnesses are
referring to the statements contained in the attestation clause itself. Indeed, the
attestation clause is separate and apart from the disposition of the will. An unsigned
attestation clause results in an unattested will. Even if the instrumental witnesses
signed the left-hand margin of the page containing the unsigned attestation clause,
such signatures cannot demonstrate these witnesses undertakings in the clause, since
the signatures that do appear on the page were directed towards a wholly different
avowal.
The Court may be more charitably disposed had the witnesses in this case signed the
attestation clause itself, but not the left-hand margin of the page containing such
clause.
NO ACKNOWLEDGEMENT ONLY NOTARIZED:
In lieu of an acknowledgment, the notary public, Petronio Y. Bautista, wrote "Nilagdaan
ko at ninotario ko ngayong10 ng Hunyo 10 (sic), 1981 dito sa Lungsod ng
Maynila."40 By no manner of contemplation can those words be construed as an
acknowledgment. An acknowledgment is the act of one who has executed a deed in
going before some competent officer or court and declaring it to be his act or deed.41 It
involves an extra step undertaken whereby the signor actually declares to the notary
that the executor of a document has attested to the notary that the same is
his/her own free act and deed.
the express requirement of Article 806 is that the will be "acknowledged", and not
merely subscribed and sworn to. The will does not present any textual proof, much less
one under oath, that the decedent and the instrumental witnesses executed or signed
the will as their own free act or deed. The acknowledgment made in a will provides for
another all-important legal safeguard against spurious wills or those made beyond the
free consent of the testator. An acknowledgement is not an empty meaningless act.43

G.R. No. 147145

January 31, 2005

BELINDA CAPONONG-NOBLEvs. ALIPIO ABAJA and NOEL ABELLAR,


Abada died sometime in May 1940.4 His widow Paula Toray ("Toray") died sometime in
September 1943. Both died without legitimate children.
The matter in dispute in the present case is the attestation clause
Caponong-Noble asserts that the will of Abada does not indicate that it is written in a
language or dialect known to the testator. Further, she maintains that the will is not
acknowledged before a notary public.
Under the Code of Civil Procedure, the intervention of a notary is not necessary in the
execution of any will.23 Therefore, Abadas will does not require acknowledgment before
a notary public.1awphi1.nt
Caponong-Noble points out that nowhere in the will can one discern that Abada knew
the Spanish language. She alleges that such defect is fatal and must result in the
disallowance of the will.
Caponong-Nobles contention must still fail. There is no statutory requirement to state in
the will itself that the testator knew the language or dialect used in the will.25 This is a
matter that a party may establish by proof aliunde.26 Caponong-Noble further argues
that Alipio, in his testimony, has failed, among others, to show that Abada knew or
understood the contents of the will and the Spanish language used in the will. However,
Alipio testified that Abada used to gather Spanish-speaking people in their place. In
these gatherings, Abada and his companions would talk in the Spanish language.27 This
sufficiently proves that Abada speaks the Spanish language.
The Attestation Clause of Abadas Will
A scrutiny of Abadas will shows that it has an attestation clause. The attestation clause
of Abadas will reads:
Caponong-Noble alleges that the attestation clause fails to state the number of pages
on which the will is written.
The allegation has no merit. The phrase "in the left margin of each and every one of the
two pages consisting of the same" shows that the will consists of two pages. The pages
are numbered correlatively with the letters "ONE" and "TWO" as can be gleaned from
the phrase "las cuales estan paginadas correlativamente con las letras "UNO" y "DOS."
Caponong-Noble further alleges that the attestation clause fails to state expressly that
the testator signed the will and its every page in the presence of three witnesses.The
first sentence of the attestation clause reads "Subscribed and professed by the testator
Alipio Abada as his last will and testament in our presence, the testator having also
signed it in our presence on the left margin of each and every one of the pages of the
same." The attestation clause clearly states that Abada signed the will and its every
page in the presence of the witnesses.

However, Caponong-Noble is correct in saying that the attestation clause does not
indicate the number of witnesses. On this point, the Court agrees with the appellate
court in applying the rule on substantial compliance in determining the number of
witnesses. While the attestation clause does not state the number of witnesses, a close
inspection of the will shows that three witnesses signed it.
We rule to apply the liberal construction in the probate of Abadas will. Abadas will
clearly shows four signatures: that of Abada and of three other persons. It is reasonable
to conclude that there are three witnesses to the will. The question on the number of
the witnesses is answered by an examination of the will itself and without the need for
presentation of evidence aliunde. The Court explained the extent and limits of the rule
on liberal construction, thus:
Finally, Caponong-Noble alleges that the attestation clause does not expressly state the
circumstances that the witnesses witnessed and signed the will and all its pages in the
presence of the testator and of each other. This Court has ruled:
Precision of language in the drafting of an attestation clause is desirable. However, it is
not imperative that a parrot-like copy of the words of the statute be made. It is sufficient
if from the language employed it can reasonably be deduced that the attestation clause
fulfills what the law expects of it.35
The last part of the attestation clause states "en testimonio de ello, cada uno de
nosotros lo firmamos en presencia de nosotros y del testador." In English, this means
"in its witness, every one of us also signed in our presence and of the testator." This
clearly shows that the attesting witnesses witnessed the signing of the will of the
testator, and that each witness signed the will in the presence of one another and of the
testator.
APOLONIO TABOADA, vs. HON. AVELINO S. ROSAL
In the petition for probate filed with the respondent court, the petitioner attached the
alleged last will and testament of the late Dorotea Perez. Written in the CebuanoVisayan dialect, the will consists of two pages. The first page contains the entire
testamentary dispositions and is signed at the end or bottom of the page by the testatrix
alone and at the left hand margin by the three (3) instrumental witnesses. The second
page which contains the attestation clause and the acknowledgment is signed at the
end of the attestation clause by the three (3) attesting witnesses and at the left hand
margin by the testatrix.
For the validity of a formal notarial will, does Article 805 of the Civil Code require that
the testatrix and all the three instrumental and attesting witnesses sign at the end of the
will and in the presence of the testatrix and of one another?
Undoubtedly, under Article 805 of the Civil Code, the will must be subscribed or signed
at its end by the testator himself or by the testator's name written by another person in

his presence, and by his express direction, and attested and subscribed by three or
more credible witnesses in the presence of the testator and of one another.
It must be noted that the law uses the terms attested and subscribed Attestation
consists in witnessing the testator's execution of the will in order to see and take note
mentally that those things are, done which the statute requires for the execution of a
will and that the signature of the testator exists as a fact. On the other hand,
subscription is the signing of the witnesses' names upon the same paper for the
purpose of Identification of such paper as the will which was executed by the testator.
(Ragsdale v. Hill, 269 SW 2d 911).
Insofar as the requirement of subscription is concerned, it is our considered view that
the will in this case was subscribed in a manner which fully satisfies the purpose of
Identification.
The signatures of the instrumental witnesses on the left margin of the first page of the
will attested not only to the genuineness of the signature of the testatrix but also the
due execution of the will as embodied in the attestation clause.
The objects of attestation and of subscription were fully met and satisfied in the present
case when the instrumental witnesses signed at the left margin of the sole page which
contains all the testamentary dispositions, especially so when the will was properly
Identified by subscribing witness Vicente Timkang to be the same will executed by the
testatrix. There was no question of fraud or substitution behind the questioned order.
We have examined the will in question and noticed that the attestation clause failed to
state the number of pages used in writing the will. This would have been a fatal defect
were it not for the fact that, in this case, it is discernible from the entire will that it is
really and actually composed of only two pages duly signed by the testatrix and her
instrumental witnesses. As earlier stated, the first page which contains the entirety of
the testamentary dispositions is signed by the testatrix at the end or at the bottom while
the instrumental witnesses signed at the left margin. The other page which is marked
as "Pagina dos" comprises the attestation clause and the acknowledgment. The
acknowledgment itself states that "This Last Will and Testament consists of two pages
including this page".
[G.R. No. L-23225. February 27, 1971.]
HERMINIO MARAVILLA v. PEDRO MARAVILLA
Attorney Manuel Villanueva, as third witness for the proponent asserted that he had
been the lawyer of the Maravillas; that 5 or 6 days before 7 October 1944 he had been
summoned through Mariano Buenaflor to the house of the Maravillas at 222 Mabini,
Ermita, Manila, and there met Digna who requested him to draft a new will, revoking
her old one, to include as additional beneficiaries Adelina Sajo, Concepcion Maravilla,
and the latters youngest daughter, Rose Mary Kohlhaas, who lived with her (Digna)

and whom she considered as her real children, having cared for them since childhood.
Digna gave Villanueva instructions concerning the will, and handed him her old will and
a handwritten list of the certificates of title of her properties, which list she asked and
obtained from her husband. Before leaving, Villanueva asked Digna to look for three
witnesses; their names were furnished him two or three days later and he sent word
that the will could be executed on 7 October 1944 (as it actually was); on that day he
brought one original and 2 copies with him, and handed them to Digna; she read the
document and while doing so the witnesses Mansueto, Hernaez and Buenaflor came.
Villanueva talked with them and satisfied himself that they were competent, whereupon
all proceeded to the dining room table. Attorney Villanueva sat at the head thereof,
Digna at his right, and Hernaez at the right of Digna; at his left was first Mansueto and
then Buenaflor. At the lawyers behest Digna Maravilla read the will in the presence of
the witnesses; after reading she called his attention to a clerical error on page 3, at the
second to the last line of paragraph 9, where Concepcion Maravilla was designated as
"hermana" ; the word was cancelled by the testatrix who wrote "cuada" above the
cancelled word, and placed her initials "D. M." beside it. She also wrote on top of each
page the words "Pagina primera," "Pagina Segunda" and so on, upon Villanuevas
instructions, and then Digna and the witnesses signed in the presence of one another
and of attorney Villanueva. 18 The latter did not ask the husband (Herminio) to join the
group when the will was executed, and Herminio remained near the window in the sala.
19 Digna appeared to the witness very healthy and spoke in Spanish intelligently. The
signing ended around 12:30 p.m., and after it all ate lunch. 20
Upon the evidence, the trial judge concluded that Mansueto did not actually see Digna
Maravilla sign the will in question, basing such conclusion upon the fact that while
Mansueto positively identified his own signature ("I identify this as my signature") but
not that of the testatrix, his five answers to the questions of counsel, in reference
thereto, being "this must be the signature of Mrs. Digna Maravilla."
In our opinion, the trial courts conclusion is far fetched, fanciful and unwarranted. It
was but natural that witness Mansueto should be positive about his own signature,
since he was familiar with it. He had to be less positive about Digna Maravillas
signature since he could not be closely acquainted with the same: for aught the record
shows, the signing of the will was the only occasion he saw her sign; he had no
opportunity to study her signature before or after the execution of Exhibit "A."
Furthermore, he witnessed Dignas signing not less than fourteen years previously. To
demand that in identifying Dignas signature Mansueto should display a positiveness
equal to the certainty shown by him in recognizing his own, exceeds the bounds of the
reasonable. The variation in the expressions used by the witness is the best evidence
that he was being candid and careful, and it is a clear badge of truthfulness rather than
the reverse.
That Mansueto, Hernaez and Buenaflor, together with the testatrix and the lawyer, sat

next to one another around one table when the will was signed is clearly established by
the uncontradicted testimony of both attorney Villanueva and Herminio Maravilla; and
that detail proves beyond doubt that each one of the parties concerned did sign in the
presence of all the others. It should be remembered, in this connection, that the test is
not whether a witness did see the signing of the will but whether he was in a position to
see if he chose to do so. 22

1. That oppositors Buenaventura Guerra and Marcelina (Martina) Guerra


are the legally adopted son and daughter, respectively, of the deceased
spouses, Florentino Guerra and Eugenia Manila;

G.R. No. L-40804 January 31, 1978

3. That, however, with respect to the parcel of riceland covered by TCT


No. T-5559 of the Register of Deeds of San Pablo City, which oppositors
believe to be the estate left and undisposed of at the time of the death of
the owner thereof, Eugenia Danila it now appears that there is a Deed of
Donation covering the same together with another parcel of coconut land
situated at Barrio San Ignacio, San Pablo City, with an area of 19,905
sq.m., and covered by Tax Declaration No. 31286, executed by the late
Eugenia Danila in favor of Adelaida Nista, as per Doc. No. 406, Page
No. 83, Series of 1966 under Notarial Register III of Notary Public Pio
Aquino of San Pablo city;

ROSARIO FELICIANO VDA. DE RAMOS, vs. COURT OF APPEALS


It appears that on June 2, 1966, Adelaida Nista who claimed to be one of
the instituted heirs, filed a petition for the probate of the alleged will and
testament dated March 9, 1963 (Exhibit H) and codicil dated April 18,
1963 (Exhibit L) of the late Eugenia Danila who died on May 21, 1966.
The petitioner prayed that after due notice and proper hearing, the
alleged will and codicil be probates and allowed and that she or any
other person be appointed as administrator of the testatrix's estate. She
also prayed that in case no opposition thereto be interposed and the
value of the estate be less than P10,000.00, said estate be summarily
settled in accordance with the Rules.
Buenaventura and Marcelina (Martina) both surnamed Guerra filed an
opposition on July 18, 1966 and an amended opposition on August 19,
1967, to the petition alleging among others that they are the legally
adopted son and daughter of the late spouses Florentino Guerra and
Eugenia Danila (Exhibit 1); that the purported will and codicil subject of
the petition (Exhibits H and L) were procured through fraud and undue
influence; that the formalities requited by law for the execution of a will
and codicil have not been complied with as the same were not properly
attested to or executed and not expressing the free will and deed of the
purported testatrix; that the late Eugenia Danila had already executed on
November 5, 1951 her last will and testament (Exhibit 3) which was duly
probated (Exhibit 4) and not revoked or annulled during the lifetime of
the testatrix, and that the petitioner is not competent and qualified to act
as administration of the estate.
On November 4, 1968, the petitioner and the oppositors, assisted by
their respective counsels, entered into a Compromise Agreement with
the following terms and conditions, thus:

2. That Florentino Guerra pre-deceased Eugenia Danila that Eugenia


Danila died on May 21, 1966, at San Pablo City, but during her lifetime,
she had already sold, donated or disposed of all her properties, some of
which to Marcelina Martina Guerra, as indicated and confirmed in
paragraph 13 of the Complaint in Civil Case No. SP620,
entitled Marcelina Guerra versus Adelaida Nista, et al., and Which We
hereby 'likewise admit and confirm;

4. That inasmuch as the above-mentioned parcel of coconut and has


been earlier donated inter vivos and validly conveyed on November 15,
1965 by the late Eugenia Danila to Marcelina (Martina) Guerra as shown
by Doc. No. 237, Page No. 49, Series of 1965, under Notarial Register
XV of Notary Public Atty. Romulo S. Brion of San Pablo City, the
inclusion of said parcel in the subsequent donation to Adelaida Nista is
admittedly considered a mistake and of no force and effect and will in no
way prejudice the ownership and right of Marcelina Martina Guerra over
the said parcel; that as a matter of fact Whatever rights and interests
Adelaida Nista has or may still have thereon are already considered
waived and renounced in favor of Marcelina Martina Guerra;
5. That in view of the fact that the riceland mentioned in paragraph 3 of
the foregoing appears to have already been disposed of by Eugenia
Danila in favor of petitioner Adelaida Nista which the parties hereto do
not now contest, there is therefore no more estate left by the said
deceased Eugenia Danila to he disposed of by the will sought to be
probated in this proceedings; that consequently, and for the sake of
peace and harmony money among the relations and kins and adopted
children of the deceased Eugenia Danila and with the further aim of
settling differences among themselves, the will and codicil of Eugenia

Danila submitted to this Honorable Court by the petitioner for probate,


are considered abrogated and set aside;
6. That as the late Eugenia Danila has incurred debts to private persons
during her lifetime, which in addition to the burial and incidental
expenses amounts to SIX THOUSAND EIGHT HUNDRED PESOS
(P6,800.00) her adopted daughter, Marcelina (Martina) Guerra is now
determined to settle the same, but herein petitioner Adelaida Nista
hereby agrees to contribute to Marcelina (Martina) Guerra for the
settlement of the said indebtedness in the amount of THREE
THOUSAND FOUR HUNDRED PESOS (P3,400.00), Philippine
Currency, the same to be delivered by Adelaida Nista to Marcelina
(Martina) Guerra at the latter's residence at Rizal Avenue, San Pablo
City, on or about February 28, 1969;
7. That should there be any other property of the deceased Eugenia
Danila that may later on be discovered to be undisposed of as yet by
Eugenia Danila during her lifetime, the same should be considered as
exclusive property of her adopted children and heirs, Buenaventura
Guerra and Marcelina (Martina) Guerra and any right of the petitioner
and signatories hereto, with respect to said property or properties, shall
be deemed waived and renounced in favor of said Buenaventura and
Marcelina (Martina) Guerra; and
8. That with the exception of the foregoing agreement, parties hereto
waived and renounce further claim against each other, and the aboveentitled case. (Exh. 6)
This Agreement was approved by the lower court in a judgment readings
as follows:
WHEREFORE, said compromise agreement, being not contrary to
public policy, law and moral, the same is hereby approved and judgment
is hereby rendered in accordance with the terms and conditions set forth
in the above- quoted compromise agreement, which is hereby made an
integral part of the dispositive portion of this decision, and the parties are
strictly enjoined to comply with the same. (Exh. 7)
On November 16, 1968, Rosario de Ramos, Miguel Danila Felix Danila
Miguel Gavino Amor Danila Consolacion Santos and Miguel Danila son
of the late Fortunato Danila filed a motion for leave to intervene as copetitioners alleging that being instituted heirs or devisees, they have
rights and interests to protect in the estate of the late Eugenia Danila
They also filed a reply partly admitting and denying the material
allegations in the opposition to the petition and alleging among other
things, that oppositors repudiated their institution as heirs and executors

when they failed to cause the recording in the Register of Deeds of San
Pablo City the will and testament dated November 5, 1951 (Exhibit 3) in
accordance with the Rules and committed acts of ingratitude when they
abandoned the testatrix and denied her support after they managed,
through fraud and undue influence, to secure the schedule of partition
dated January 15, 1962. The Intervenors prayed for the probate and/or
allowance of the will and codicil (Exhibits H and L), respectively and the
appointment of any of them in as administrator of said estate.
On December 6, 1968, the intervenors also filed a motion for new trial
and/or re-hearing and/or relief from judgment and to set aside the
judgment based on compromise dated November 5, 1968. The
oppositors interposed an opposition to the motion to which the
intervenors filed their reply.
The lower court resolved the motions in an order the dispositive portion
reading, thus:
FOR ALL THE FOREGOING the Court hereby makes the following
dispositions
(1) Movants Rosario de Ramos, Miguel C. Danila Miguela Gavino Amor
Danila Consolacion Santos, Miguel A. Danila and Raymundo Danila are
allowed and admitted to intervene to this proceeding as Party
Petitioners; and likewise admitted in their reply to the amended
opposition of November 11, 1968;
(2) The compromise agreement dated October 15, 1968 by and between
Petitioner Adelaida Nista and oppositors Buenaventura Guerra and
Marcelina Guerra Martina is disapproved, except as regards their
respective lawful rights in the subject estate; and, accordingly, the
judgment on compromise rendered by this Court on November 5, 1968
is reconsidered and set aside; and
(3) The original Petition and amended opposition to probate of the
alleged will and codicil stand.
xxx xxx xxx
The lower court also denied the motion for the appointment of a special
administrator filed by the intervenors.
xxx xxx xxx
A motion for reconsideration of the foregoing order was filed by the
intervenors co-petitioners but the motion was denied.
xxx xxx xxx

On February 9, 1971, a motion for the substitution of Irene, Crispina,


Cristina Casiano, Edilberto Felisa, Guerra in place of their father, the
oppositor Buenaventura Guerra who died on January 23, 1971, was filed
and granted by the lower court.

GAVE CREDENCE TO THE TESTIMONIES OR BIASED WITNESSES


OVER THEIR OWN ATTESTATION CLAUSES AND THE TESTIMONIAL
EVIDENCE AND NOTARIAL ACKNOWLEDGEMENT OF THE NOTARY
PUBLIC; AND

After trial on the merits, the lower court rendered its decision dated July 6, 1971
allowing the probate of the wilt In that decision, although two of the attesting witness
Odon Sarmiento and Rosendo Paz, testified that they did not see the testatrix Eugenia
Danila sign the will but that the same was already signed by her when they affixed their
own signatures thereon, the trial court gave more weight and ment to the .'straightforward and candid" testimony of Atty. Ricardo Barcenas, the Notary Public who
assisted in the execution of the wilt that the testatrix and the three (3) instrumental
witnesses signed the will in the presence of each other, and that with respect to the
codicil the same manner was likewise observed as corroborated to by the testimony of
another lawyer, Atty. Manuel Alvero who was also present during the execution of the
codicil.

(B) THAT THE COURT OF APPEALS ERRED IN HAVING DENIED THE


PROBATE OF THE WILL AND CODICIL DESPITE CONVINCING
EVIDENCE FOR THEIR ALLOWANCE.

The dispositive portion of the decision reads:


WHEREFORE, it appearing that the late Eugenia Danila had
testamentary capacity when she executed the will, Exh. H., and the
codicil Exh. L, and that said will and codicil were duly signed by her and
the three attesting witnesses and acknowledged before a Notary Public
in accordance with the formalities prescribed by law, the said will and
codicil are hereby declared probated. No evidence having been adduced
regarding the qualification and fitness of any of the intervenors- copetitioners to act as executors, the appointment of executors of the will
and codicil is held pending until after due hearing on the matter.
SO ORDERED.
Oppositors Marcelina Guam and the heirs of Buenaventura Guam appealed the
foregoing decision to the Court of Appeals The latter court, in its derision dated May 12,
1975 ruled that the lower court acted correctly in setting aside its judgment approving
the Compromise Agreement and in allowing the intervenor petitioners to participate in
the instant probate proceedings; however, it disallowed the probate of the will on the
that the evidence failed to establish that the testatrix Eugenia Danila signed her will in
the presence of the instrumental witness in accordance with Article 805 of the Civil
Code, as testified to by the two surviving instrumental witnesses.
In this present appeal petitioners vigorously insists on constitutional grounds the nullity
of the decision of respondent court but We deem it needless to consider the same as it
is not necessary in resolving this appeal on the following assigned errors:
(A) THE COURT OF APPEALS ERRED GRAVELY IN NOT HAVING
GIVEN WEIGHT TO THE MANIFESTATION CLAUSES IN THE
TESTAMENT AND CODICIL ANNEX B (PETITION) AND INSTEAD IT

We reverse the judgment of the Court of Appeals and restore the decision of the trial
court allowing probate of the will and codicil in question.
The main point in controversy here is whether or not the last testament and its
accompanying codicil were executed in accordance with the formalities of the law,
considering the complicated circumstances that two of the attesting witnesses testified
against their due execution while other non-subscribing witnesses testified to the
contrary.
Petitioners argue that the attestation clauses of the win and codicil which were signed
by the instrumental witnesses are admissions of due execution of the deeds, thus,
preventing the said witnesses from prevaricating later on by testifying against due
execution. Petitioners further maintain that it is error for respondent court to give
credence to the testimony of the biased witnesses as against their own attestation to
the fact of due execution and over the testimonial account of the Notary Public who was
also present during the execution and before whom right after, the deeds were
acknowledged.
Private respondents, on the other hand reiterate in their contention the declaration of
the two surviving witnesses, Odon Sarmiento and Rosendo Paz, that the win was not
signed by the testatrix before their presence, which is strengthened by two
photographic evidence showing only the two witnesses in the act of signing, there being
no picture of the same occasion showing the testatrix signing the will. Respondent court
holds the view that where there was an opportunity to take pictures it is not
understandable why pictures were taken of the witnesses and not of the testatrix. It
concludes that the absence of the latter's picture to complete the evidence belies the
testimony of Atty. Barcenas that the testatrix and the witnesses did sign the will and the
codicil in the presence of each other.
The oppositors' argument is untenable. There is ample and satisfactory evidence to
convince us that the will and codicil were executed in accordance with the formalities
required by law. It appears positively and convincingly that the documents were
prepared by a lawyer, Atty. Manuel Alvero The execution of the same was evidently
supervised by his associate, Atty. Ricardo Barcenas and before whom the deeds were
also acknowledged. The solemnity surrounding the execution of a will is attended by
some intricacies not usually within the comprehension of an ordinary layman. The
object is to close the door against bad faith and fraud, to avoid substitution of the will

and testament, and to guarantee their truth and authenticity. 2 If there should be any
stress on the participation of lawyers in the execution of a wig, other than an interested
party, it cannot be less than the exercise of their primary duty as members of the Bar to
uphold the lofty purpose of the law. There is no showing that the above-named lawyers
had been remiss in their sworn duty. Consequently, respondent court failed to consider
the presumption of ty in the execution of the questioned documents. There were no
incidents brought to the attention of the trial court to arouse suspicion of anomaly. While
the opposition alleged fraud and undue influence, no evidence was presented to prove
their occurrence. There is no question that each and every page of the will and codicil
carry the authentic signatures of Eugenia Danila and the three (3) attesting witnesses.
Similarly, the attestation claim far from being deficient, were properly signed by the
attesting witnesses. Neither is it disputed that these witnesses took turns in signing the
will and codicil in the presence of each other and the testatrix. Both instruments were
duly acknowledged before a Notary Public who was all the time present during the
execution.

witnesses impliedly to the truth of the facts which admit to probate, including the
sufficiency of execution, the capacity of the testatrix, the absence of undue influence,
and the like. 6

The presumption of regularity can of course be overcome by clear and convincing


evidence to the contrary, but not easily by the mere expediency of the negative
testimony of Odon Sarmiento and Rosendo Paz that they did not see the testatrix sign
the will. A negative testimony does not enjoy equal standing with a positive assertion,
and faced with the convincing appearance of the will, such negative statement must be
examined with extra care. For in this regard

In the case at bar, the s bear a disparity in the quality of the testimonies of Odon
Sarmiento and Rosendo Paz on one hand, and the Notary Public, Atty. Ricardo A.
Barcenas, on the other. The testimony of Odon Sarmiento was contradicted by his own
admission. Though his admission to the effect that "when Eugenia Danila signed the
testament (he) and the two other attesting witnesses Rosendo Paz and Calixto
Azusada were present" (t.s.n., Feb. 12, 1970, p. 115) was made extrajudicially, it was
not squarely refuted when inquired upon during the trial.

It has also been held that the condition and physical appearance of a
questioned document constitute a valuable factor which, if correctly
evaluated in the light of surrounding circumstances, may help in
determining whether it is genuine or forged. Subscribing witnesses may
forget or exaggerating what they really know, saw, heard or did; they
may be biased and, therefore, tell only half-truths to mislead the court or
favor one party to the prejudice of the others. This cannot be said of the
condition and Physical appearance of the questioned document. Both,
albeit silent, will reveal the naked truth, hiding nothing, forgetting
nothing, and exaggerating nothing. 3
Unlike other deeds, ordinary wills by necessity of law must contain an attestation clause
Which, significantly is a separate memorandum or record of the facts surrounding that
the conduct of execution. Once signed by the attesting witnesses, it that compliance
with the indispensable legal formalities had been observed. This Court had previously
hold that the attestation clause basically contracts the pretense of undue ex execution
which later on may be made by the attesting witnesses. 4 In the attestation clause, the
witnesses do not merely attest to the signature of the testatrix but also to the proper
execution of the will, and their signature following that of the testatrix show that they
have in fact at not only to the genuineness of the testatrix's signature but also to the
due execution of the will as embodied in the attention clause. 5 By signing the wilt the

In this jurisdiction, all the attesting witness to a will if available, must be called to prove
the wilt Under this circumstance, they become "forced witnesses" " and their
declaration derogatory to the probate of the will need not bind the proponent hence, the
latter may present other proof of due exemption even if contrary to the testimony of or
all of the at, testing witness. 7 As a rule, if any or all of the submitting witness testify
against the due execution of the will, or do not remember having attested to it, or are
otherwise of doubtful ability, the will may, nevertheless, be allowed if the court is
satisfied from the testimony of other witness and from all the evidence presented that
the will was executed and attested in the manner by law. 8 Accordingly, although the
subscribing witnesses to a contested will are the best witness in connection with its due
execution, to deserve full credit, their testimony must be reasonable, and unbiased; if
otherwise it may be overcome by any competent evidence, direct or circubstantial. 9

With respect to the testimony of Rosendo Paz, it had been refuted by the declaration of
Atty. Ricardo A. Barcenas. The records show that this attesting witness was fetched by
Felix Danila from his place of work in order to act as witness to a wilt Rosendo Paz did
not know what the document he signed was all about. Although he performed his
function as an attesting witness, his participation was rather passive. We do not expect,
therefore, that his testimony, "half-hearted" as that of Odon Sarmiento, be as candid
and complete as one proceeding from a keen mind fully attentive to the details of the
execution of the deeds. Quite differently, Atty. Ricardo A. Barcenas, more than a direct
witness himself, was Purposely there to oversee the accomplishment of the will and
codicil. His testimony is an account of what he actually heard and saw during the
conduct of his profession. There is no evidence to show that this lawyer was motivated
by any material interest to take sides or that his statement is truth perverted.
It has been regarded that the function of the Notary Public is, among others, to guard
against any illegal or immoral arrangements in the execution of a will. 10 In the absence
of any showing of self-interest that might possibly have warped his judgment and
twisted his declaration, the intervention of a Notary Public, in his professional capacity,
in the execution of a will deserves grave consideration. 11 An appraise of a lawyer's
participation has been succinctly stated by the Court in Fernandez v. Tantoco, supra,
this wise:

In weighing the testimony of the attesting witnesses to a will, his


statements of a competent attorney, who has been charged with the
responsibility of seeing to the proper execution of the instrument, is
entitled to greater weight than the testimony of a person casually called
to anticipate in the act, supposing of course that no motive is revealed
that should induce the attorney to prevaricate. The reason is that the
mind of the attorney being conversant of the instrument, is more likely to
become fixed on details, and he is more likely than other persons to
retain those incidents in his memory.
One final point, the absence of a photograph of the testator Eugenia Danila in the act of
signing her will. The fact that the only pictures available are those which show the
Witnesses signing the will in the presence of the testatrix and of each other does not
belie the probability that the testatrix also signed the will before the presence of the
witnesses. We must stress that the pictures are worthy only of what they show and
prove and not of what they did not speak of including the events they failed to capture.
The probate of a will is a proceeding not embued with adverse character, wherein
courts should relax the rules on evidence "to the end that nothing less than the best
evidence of which the matter is susceptible" should be presented to the court before a
reported will may be probated or denied probate. 12
We find here that the failure to imprint in photographs all the stages in the execution of
the win does not serve any persuasive effect nor have any evidentiary value to prove
that one vital and indispensable requisite has not been acted on. Much less can it
defeat, by any ordinary or special reason, the presentation of other competent evidence
intended to confirm a fact otherwise existent but not confirmed by the photographic
evidence. The probate court having satisfied itself that the win and codicil were
executed in accordance with the formalities required by law, and there being no
indication of abuse of discretion on its part, We find no error committed or any
exceptional circumstance warranting the subsequent reversal of its decision allowing
the probate of the deeds in question.
WHEREFORE, the decision of respondent Court of Appeals is hereby reversed in so
far its it disallowed the probate of the will and codicil. With costs against respondents.
SO ORDERED.

G.R. No. L-17304

May 22, 1922

In re will of Maria Roque y Paraiso, deceased.


CEFERINO ALDABA, petitioner-appellee, vs. LUDOVICO ROQUE, opponentappellant.

Platon has written my name on the left margin of all pages of this
testament, in the presence of the witnesses, and all the witnesses have
also signed all the pages of this testament on the left margin in my
presence and that of each other.
X (Her thumb mark)
MARIA ROQUE Y PARAISO,
Per VICENTE PLATON.
(Sgd.) REGINO E. MENDOZA,
Witness.

Lucero and Tengo for appellant.


Vicente Platon for appellee.
VILLAMOR, J.:
It appears from the record of the case that on July 9, 1918, Maria Roque y Paraiso, the
widow of Bruno Valenzuela, resident of the barrio of Mambog, municipality of Malolos,
Province of Bulacan, executed her last will and testament in the Tagalog dialect with the
help of Vicente Platon and in the presence of three witnesses who signed the
attestation clause and each of the four pages of the testament. Maria Roque died on
December 3, 1919, and when her will was filed in court for probate, it was contested by
Ludovico Roque on the ground that it had not been prepared nor executed in
conformity with the requirements and solemnities prescribed by law.
After due proceedings had been had, the Court of First Instance of Bulacan by its
decision rendered on February 27th of the following year, pronounced the testament in
question valid, and ordered its probate, appointing Ceferino Aldaba as the administrator
of the estate.
The errors assigned by the appellant are two, to wit: "That each and every folio of the
said testament is not paged correlatively in letter," and "that the said will lacks the
attestation clause required by law."
We have examined document Exhibit 4 which is the will in question and we find at the
end thereof the following in Tagalog which translated into English reads:
This document expresses my last and spontaneous will, and is my last
will and testament, which was drawn by the lawyer, Don Vicente Platon,
at my direction, and everything contained in this testament has been
ordained and directed by me to said Vicente Platon in order that it might
be embodied in this testament, and after this testament has been drawn
up, I directed him to read it so that I might hear all its contents, and I
have heard and understood all the contents of this document which is
my last will, wherefore, and not knowing how to write, I have requested
Don Vicente Platon to write and sign my name in my stead hereon; I
declare that this testament is composed of four sheets, actually used,
that the sheets are paged with the letter A, B, C, and d, and above my
name I have placed the thumb mark of my right hand in the presence of
the subscribing witnesses, and that all the witnesses have signed in my
presence and of each other here at Malolos, Bulacan, this 9th day of the
month of July, 1918; and I also declare that at my request Don Vicente

(Sgd.) IGNACIO ANIAG,


Witness.
(Sgd.) CEFERINO ALDABA.
Witness.
In reality, it appears that it is the testatrix who makes the declaration about the points
contained in the above described paragraph; however, as the witnesses, together with
the testatrix, have signed the said declaration, we are of the opinion and so hold that
the words above quoted of the testament constitute a sufficient compliance with the
requirements of section 1 of Act No. 2645 which provides that:
The attestation shall state the number of sheets or pages used, upon which the
will is written, and the fact that the testator signed the will and every page
thereof, or caused some other person to write his name, under his express
direction, in the presence of three witnesses, and the latter witnessed and
signed the will and all the pages thereof in the presence of the testator and of
each other.
In regard to the other assignment of error, to wit, that each of the folios of the said
testament is not paged correlatively in letters "one." "two," "three," etc., but only with the
letters A, B, C, etc., we are of the opinion that this method of indicating the paging of
the testament is a compliance with the spirit of the law, since either one of the two ways
above-mentioned indicates the correlation of the pages and serves to prevent the loss
of any of them. It might be said that the object of the law in requiring that the paging be
made in letters is to make falsification more difficult, but it should be noted that since all
the pages of the testament are signed at the margin by the testatrix and the witnesses,
the difficulty of forging the signatures in either case remains the same. In other words
the more or less degree of facility to imitate the writing of the letters A, B, C, etc., does
not make for the easiness to forge the signature. And as in the present case there
exists the guaranty of the authenticity of the testament, consisting in the signatures on
the left marging of the testament and the paging thereof as declared in the attestation
clause, the holding of this court in Abangan vs. Abangan (40 Phil, 476), might as well
be repeated:

The object of the solemnities surrounding the execution of wills is to close the
door against bad faith and fraud, to avoid substitution of wills and testaments
and to guarantee their truth and authenticity. Therefore the laws on this subject
should be interpreted in such a way as to attain these primordial ends. But, on
the other hand, also one must not lose sight of the fact that it is not the object of
the law to restrain and curtail the exercise of the right to make a will. So when
an interpretation already given assures such ends, any other interpretation
whatsoever, that adds nothing but demands more requisites entirely
unnecesary, useless, and frustrative of the testator's last will, must be
disregarded.
In that case the testament was written on one page, and the attestation clause on
another. Neither one of these pages was numbered in any way; and it was held:
In a will consisting of two sheets the first of which contains all the testamentary
dispositions and is signed at the bottom by the testator and three witnesses and
the second contains only the attestation clause and is signed also at the bottom
by the three witnesses, it is not necessary that both sheets be further signed on
their margings by the testator and the witnesses, or be paged.
This means that, according to the particular case, the omission of paging does not
necessarily render the testament invalid.
The law provides that the numbering of the pages should be in letters placed on the
upper part of the sheet, but if the paging should be place din the lower part, would the
testament be void for his sole reason? We believe not. The law also provides that the
testator and the witnesses must sign the left margin of each of the sheets of the
testament; but if they should sign on the right margin, would this fact also annul the
testament? Evidently not. This court has already held in Avera vs. Garcia and
Rodriguez (42 Phil., 145):lvvph1n+
It is true that the statute says that the testator and the instrumental witnesses
shall sign their names on the left margin of each and every page; and it is
undeniable that the general doctrine is to the effect that all statutory
requirements as to the execution of wills must be fully complied with. The same
doctrine is also deducible from cases heretofore decided by this court
Still some details at times creep into legislative enactments which are so trivial
that it would be absurd to suppose that the Legislature could have attached any
decisive importance to them. The provision to the effect that the signatures of
the testator and witnesses shall be written on the left margin of each page
rather than on the right margin seems to be of this character. So far as
concerns the authentication of the will, and of every part thereof, it can make no
possible different whether the names appear on the left or on the right margin,
provided they are on one or the other. In Caraig vs. Tatlonghari (R. G. No.
12558, decided March 23, 1918, not reported), this court declared a will void

which was totally lacking in the signatures required to be written on its several
pages; and in the case of Re estate of Saguinsin (41 Phil., 875), a will was
likewise declared void which contained the necessary signature on the margin
of each left (folio), but not on the margin of each page containing written matter.
We do not desire to intimate that the numbering in letters is a requisite of no
importance. But since its principal object is to give the correlation of the pages, we hold
that this object may be attained by writing "one." "two," "three," etc., well as by writing
A, B, C, etc. Following, therefore, the view maintained by this court in the case
ofAbangan vs. Abangan, supra, as regards the appreciation of the solemnities of a
testament, we decide that the judgment appealed from must be, as is hereby, affirmed
with costs against the appellant. So ordered.

A.C. No. 5281

February 12, 2008

MANUEL L. LEE, petitioner, vs. ATTY. REGINO B. TAMBAGO, respondent.


RESOLUTION
CORONA, J.:
In a letter-complaint dated April 10, 2000, complainant Manuel L. Lee charged
respondent Atty. Regino B. Tambago with violation of the Notarial Law and the ethics of
the legal profession for notarizing a spurious last will and testament.

Doc. 14, Page No. 4, Book No. 1, Series of 1965 refers to an AFFIDAVIT executed
by BARTOLOME RAMIREZ on June 30, 1965 and is available in this Office[s]
files.6
Respondent in his comment dated July 6, 2001 claimed that the complaint against him
contained false allegations: (1) that complainant was a son of the decedent Vicente
Lee, Sr. and (2) that the will in question was fake and spurious. He alleged that
complainant was "not a legitimate son of Vicente Lee, Sr. and the last will and
testament was validly executed and actually notarized by respondent per affidavit7 of
Gloria Nebato, common-law wife of Vicente Lee, Sr. and corroborated by the joint
affidavit8 of the children of Vicente Lee, Sr., namely Elena N. Lee and Vicente N. Lee,
Jr. xxx."9

In his complaint, complainant averred that his father, the decedent Vicente Lee, Sr.,
never executed the contested will. Furthermore, the spurious will contained the forged
signatures of Cayetano Noynay and Loreto Grajo, the purported witnesses to its
execution.

Respondent further stated that the complaint was filed simply to harass him because
the criminal case filed by complainant against him in the Office of the Ombudsman "did
not prosper."

In the said will, the decedent supposedly bequeathed his entire estate to his wife Lim
Hock Lee, save for a parcel of land which he devised to Vicente Lee, Jr. and Elena Lee,
half-siblings of complainant.

Respondent did not dispute complainants contention that no copy of the will was on file
in the archives division of the NCCA. He claimed that no copy of the contested will
could be found there because none was filed.

The will was purportedly executed and acknowledged before respondent on June 30,
1965.1 Complainant, however, pointed out that the residence certificate2 of the testator
noted in the acknowledgment of the will was dated January 5, 1962.3 Furthermore, the
signature of the testator was not the same as his signature as donor in a deed of
donation4 (containing his purported genuine signature). Complainant averred that the
signatures of his deceased father in the will and in the deed of donation were "in any
way (sic) entirely and diametrically opposed from (sic) one another in all angle[s]."5

Lastly, respondent pointed out that complainant had no valid cause of action against
him as he (complainant) did not first file an action for the declaration of nullity of the will
and demand his share in the inheritance.

Complainant also questioned the absence of notation of the residence certificates of


the purported witnesses Noynay and Grajo. He alleged that their signatures had
likewise been forged and merely copied from their respective voters affidavits.
Complainant further asserted that no copy of such purported will was on file in the
archives division of the Records Management and Archives Office of the National
Commission for Culture and the Arts (NCCA). In this connection, the certification of the
chief of the archives division dated September 19, 1999 stated:

In a resolution dated October 17, 2001, the Court referred the case to the Integrated
Bar of the Philippines (IBP) for investigation, report and recommendation.10
In his report, the investigating commissioner found respondent guilty of violation of
pertinent provisions of the old Notarial Law as found in the Revised Administrative
Code. The violation constituted an infringement of legal ethics, particularly Canon
111 and Rule 1.0112 of the Code of Professional Responsibility (CPR).13 Thus, the
investigating commissioner of the IBP Commission on Bar Discipline recommended the
suspension of respondent for a period of three months.
The IBP Board of Governors, in its Resolution No. XVII-2006-285 dated May 26, 2006,
resolved:

[T]o ADOPT and APPROVE, as it is hereby ADOPTED and APPROVED, with


modification, the Report and Recommendation of the Investigating Commissioner
of the above-entitled case, herein made part of this Resolution as Annex "A"; and,
finding the recommendation fully supported by the evidence on record and the
applicable laws and rules, and considering Respondents failure to comply with the
laws in the discharge of his function as a notary public, Atty. Regino B. Tambago is
hereby suspended from the practice of law for one year and Respondents notarial
commission is Revoked and Disqualified fromreappointment as Notary Public for
two (2) years.14

is his or her own free act and deed.21 The acknowledgment in a notarial will has a twofold purpose: (1) to safeguard the testators wishes long after his demise and (2) to
assure that his estate is administered in the manner that he intends it to be done.
A cursory examination of the acknowledgment of the will in question shows that this
particular requirement was neither strictly nor substantially complied with. For one,
there was the conspicuous absence of a notation of the residence certificates of the
notarial witnesses Noynay and Grajo in the acknowledgment. Similarly, the notation of
the testators old residence certificate in the same acknowledgment was a clear breach
of the law. These omissions by respondent invalidated the will.

We affirm with modification.


A will is an act whereby a person is permitted, with the formalities prescribed by law, to
control to a certain degree the disposition of his estate, to take effect after his death.15 A
will may either be notarial or holographic.
The law provides for certain formalities that must be followed in the execution of wills.
The object of solemnities surrounding the execution of wills is to close the door on bad
faith and fraud, to avoid substitution of wills and testaments and to guarantee their truth
and authenticity.16
A notarial will, as the contested will in this case, is required by law to be subscribed at
the end thereof by the testator himself. In addition, it should be attested and subscribed
by three or more credible witnesses in the presence of the testator and of one
another.17
The will in question was attested by only two witnesses, Noynay and Grajo. On this
circumstance alone, the will must be considered void.18 This is in consonance with the
rule that acts executed against the provisions of mandatory or prohibitory laws shall be
void, except when the law itself authorizes their validity.
The Civil Code likewise requires that a will must be acknowledged before a notary
public by the testator and the witnesses.19 The importance of this requirement is
highlighted by the fact that it was segregated from the other requirements under Article
805 and embodied in a distinct and separate provision.20
An acknowledgment is the act of one who has executed a deed in going before some
competent officer or court and declaring it to be his act or deed. It involves an extra step
undertaken whereby the signatory actually declares to the notary public that the same

As the acknowledging officer of the contested will, respondent was required to faithfully
observe the formalities of a will and those of notarization. As we held in Santiago v.
Rafanan:22
The Notarial Law is explicit on the obligations and duties of notaries public. They
are required to certify that the party to every document acknowledged before him
had presented the proper residence certificate (or exemption from the residence
tax); and to enter its number, place of issue and date as part of such certification.
These formalities are mandatory and cannot be disregarded, considering the degree of
importance and evidentiary weight attached to notarized documents.23 A notary public,
especially a lawyer,24 is bound to strictly observe these elementary requirements.
The Notarial Law then in force required the exhibition of the residence certificate upon
notarization of a document or instrument:
Section 251. Requirement as to notation of payment of [cedula] residence tax.
Every contract, deed, or other document acknowledged before a notary public shall
have certified thereon that the parties thereto have presented their proper [cedula]
residence certificate or are exempt from the [cedula] residence tax, and there shall
be entered by the notary public as a part of such certificate the number, place of
issue, and date of each [cedula] residence certificate as aforesaid.25
The importance of such act was further reiterated by Section 6 of the Residence Tax
Act26 which stated:
When a person liable to the taxes prescribed in this Act acknowledges any
document before a notary public xxx it shall be the duty of such person xxx with

whom such transaction is had or business done, to require the exhibition of the
residence certificate showing payment of the residence taxes by such person
xxx.
In the issuance of a residence certificate, the law seeks to establish the true and correct
identity of the person to whom it is issued, as well as the payment of residence taxes
for the current year. By having allowed decedent to exhibit an expired residence
certificate, respondent failed to comply with the requirements of both the old Notarial
Law and the Residence Tax Act. As much could be said of his failure to demand the
exhibition of the residence certificates of Noynay and Grajo.
On the issue of whether respondent was under the legal obligation to furnish a copy of
the notarized will to the archives division, Article 806 provides:
Art. 806. Every will must be acknowledged before a notary public by the testator
and the witness. The notary public shall not be required to retain a copy of
the will, or file another with the office of the Clerk of Court. (emphasis
supplied)
Respondents failure, inadvertent or not, to file in the archives division a copy of the
notarized will was therefore not a cause for disciplinary action.
Nevertheless, respondent should be faulted for having failed to make the necessary
entries pertaining to the will in his notarial register. The old Notarial Law required the
entry of the following matters in the notarial register, in chronological order:

7. if the instrument is a contract, a brief description of the substance of the


instrument.27
In an effort to prove that he had complied with the abovementioned rule, respondent
contended that he had crossed out a prior entry and entered instead the will of the
decedent. As proof, he presented a photocopy of his notarial register. To reinforce his
claim, he presented a photocopy of a certification28 stating that the archives division had
no copy of the affidavit of Bartolome Ramirez.
A photocopy is a mere secondary evidence. It is not admissible unless it is shown that
the original is unavailable. The proponent must first prove the existence and cause of
the unavailability of the original,29 otherwise, the evidence presented will not be
admitted. Thus, the photocopy of respondents notarial register was not admissible as
evidence of the entry of the execution of the will because it failed to comply with the
requirements for the admissibility of secondary evidence.
In the same vein, respondents attempt to controvert the certification dated September
21, 199930 must fail. Not only did he present a mere photocopy of the certification dated
March 15, 2000;31 its contents did not squarely prove the fact of entry of the contested
will in his notarial register.
Notaries public must observe with utmost care32 and utmost fidelity the basic
requirements in the performance of their duties, otherwise, the confidence of the public
in the integrity of notarized deeds will be undermined.33

3. witnesses, if any, to the signature;

Defects in the observance of the solemnities prescribed by law render the entire will
invalid. This carelessness cannot be taken lightly in view of the importance and delicate
nature of a will, considering that the testator and the witnesses, as in this case, are no
longer alive to identify the instrument and to confirm its contents.34 Accordingly,
respondent must be held accountable for his acts. The validity of the will was seriously
compromised as a consequence of his breach of duty.35

4. date of execution, oath, or acknowledgment of the instrument;

In this connection, Section 249 of the old Notarial Law provided:

1. nature of each instrument executed, sworn to, or acknowledged before him;


2. person executing, swearing to, or acknowledging the instrument;

5. fees collected by him for his services as notary;


6. give each entry a consecutive number; and

Grounds for revocation of commission. The following derelictions of duty on


the part of a notary public shall, in the discretion of the proper judge of first
instance, be sufficient ground for the revocation of his commission:
xxx

xxx

xxx

(b) The failure of the notary to make the proper entry or entries in his notarial
register touching his notarial acts in the manner required by law.
xxx

xxx

xxx

(f) The failure of the notary to make the proper notation regarding cedula
certificates.36
These gross violations of the law also made respondent liable for violation of his oath
as a lawyer and constituted transgressions of Section 20 (a), Rule 138 of the Rules of
Court37 and Canon 138 and Rule 1.0139 of the CPR.
The first and foremost duty of a lawyer is to maintain allegiance to the Republic of the
Philippines, uphold the Constitution and obey the laws of the land.40 For a lawyer is the
servant of the law and belongs to a profession to which society has entrusted the
administration of law and the dispensation of justice.41
While the duty to uphold the Constitution and obey the law is an obligation imposed on
every citizen, a lawyer assumes responsibilities well beyond the basic requirements of
good citizenship. As a servant of the law, a lawyer should moreover make himself an
example for others to emulate.42 Being a lawyer, he is supposed to be a model in the
community in so far as respect for the law is concerned.43
The practice of law is a privilege burdened with conditions.44 A breach of these
conditions justifies disciplinary action against the erring lawyer. A disciplinary sanction is
imposed on a lawyer upon a finding or acknowledgment that he has engaged in
professional misconduct.45 These sanctions meted out to errant lawyers include
disbarment, suspension and reprimand.

with the elementary formalities in the performance of his duties xxx," we find that he
acted very irresponsibly in notarizing the will in question. Such recklessness warrants
the less severe punishment of suspension from the practice of law. It is, as well, a
sufficient basis for the revocation of his commission50 and his perpetual disqualification
to be commissioned as a notary public.51
WHEREFORE, respondent Atty. Regino B. Tambago is hereby found guilty of
professional misconduct. He violated (1) the Lawyers Oath; (2) Rule 138 of the Rules
of Court; (3) Canon 1 and Rule 1.01 of the Code of Professional Responsibility; (4) Art.
806 of the Civil Code and (5) the provisions of the old Notarial Law.
Atty. Regino B. Tambago is hereby SUSPENDED from the practice of law for one year
and his notarial commission REVOKED. Because he has not lived up to the
trustworthiness expected of him as a notary public and as an officer of the court, he
is PERPETUALLY DISQUALIFIED from reappointment as a notary public.
Let copies of this Resolution be furnished to all the courts of the land, the Integrated
Bar of the Philippines and the Office of the Bar Confidant, as well as made part of the
personal records of respondent. SO ORDERED.
G.R. No. L-32213 November 26, 1973
AGAPITA N. CRUZ, petitioner, vs. HON. JUDGE GUILLERMO P. VILLASOR,
Presiding Judge of Branch I, Court of First Instance of Cebu, and MANUEL B.
LUGAY, respondents.
Paul G. Gorrez for petitioner.
Mario D. Ortiz for respondent Manuel B. Lugay.

46

Disbarment is the most severe form of disciplinary sanction. We have held in a


number of cases that the power to disbar must be exercised with great caution47 and
should not be decreed if any punishment less severe such as reprimand, suspension,
or fine will accomplish the end desired.48 The rule then is that disbarment is meted out
only in clear cases of misconduct that seriously affect the standing and character of the
lawyer as an officer of the court.49
Respondent, as notary public, evidently failed in the performance of the elementary
duties of his office. Contrary to his claims that he "exercised his duties as Notary Public
with due care and with due regard to the provision of existing law and had complied

ESGUERRA, J.:
Petition to review on certiorari the judgment of the Court First Instance of Cebu allowing
the probate of the last will a testament of the late Valente Z. Cruz. Petitioner-appellant
Agapita N. Cruz, the surviving spouse of the said decease opposed the allowance of
the will (Exhibit "E"), alleging the will was executed through fraud, deceit,
misrepresentation and undue influence; that the said instrument was execute without

the testator having been fully informed of the content thereof, particularly as to what
properties he was disposing and that the supposed last will and testament was not
executed in accordance with law. Notwithstanding her objection, the Court allowed the
probate of the said last will and testament Hence this appeal by certiorari which was
given due course.
The only question presented for determination, on which the decision of the case
hinges, is whether the supposed last will and testament of Valente Z. Cruz (Exhibit "E")
was executed in accordance with law, particularly Articles 805 and 806 of the new Civil
Code, the first requiring at least three credible witnesses to attest and subscribe to the
will, and the second requiring the testator and the witnesses to acknowledge the will
before a notary public.
Of the three instrumental witnesses thereto, namely Deogracias T. Jamaloas Jr., Dr.
Francisco Paares and Atty. Angel H. Teves, Jr., one of them, the last named, is at the
same time the Notary Public before whom the will was supposed to have been
acknowledged. Reduced to simpler terms, the question was attested and subscribed by
at least three credible witnesses in the presence of the testator and of each other,
considering that the three attesting witnesses must appear before the notary public to
acknowledge the same. As the third witness is the notary public himself, petitioner
argues that the result is that only two witnesses appeared before the notary public to
acknowledge the will. On the other hand, private respondent-appellee, Manuel B.
Lugay, who is the supposed executor of the will, following the reasoning of the trial
court, maintains that there is substantial compliance with the legal requirement of
having at least three attesting witnesses even if the notary public acted as one of them,
bolstering up his stand with 57 American Jurisprudence, p. 227 which, insofar as
pertinent, reads as follows:
It is said that there are, practical reasons for upholding a will as against
the purely technical reason that one of the witnesses required by law
signed as certifying to an acknowledgment of the testator's signature
under oath rather than as attesting the execution of the instrument.
After weighing the merits of the conflicting claims of the parties, We are inclined to
sustain that of the appellant that the last will and testament in question was not
executed in accordance with law. The notary public before whom the will was
acknowledged cannot be considered as the third instrumental witness since he cannot
acknowledge before himself his having signed the will. To acknowledge before means
to avow (Javellana v. Ledesma, 97 Phil. 258, 262; Castro v. Castro, 100 Phil. 239, 247);

to own as genuine, to assent, to admit; and "before" means in front or preceding in


space or ahead of. (The New Webster Encyclopedic Dictionary of the English
Language, p. 72; Funk & Wagnalls New Standard Dictionary of the English Language,
p. 252; Webster's New International Dictionary 2d. p. 245.) Consequently, if the third
witness were the notary public himself, he would have to avow assent, or admit his
having signed the will in front of himself. This cannot be done because he cannot split
his personality into two so that one will appear before the other to acknowledge his
participation in the making of the will. To permit such a situation to obtain would be
sanctioning a sheer absurdity.
Furthermore, the function of a notary public is, among others, to guard against any
illegal or immoral arrangement Balinon v. De Leon, 50 0. G. 583.) That function would
defeated if the notary public were one of the attesting instrumental witnesses. For them
he would be interested sustaining the validity of the will as it directly involves him and
the validity of his own act. It would place him in inconsistent position and the very
purpose of acknowledgment, which is to minimize fraud (Report of Code Commission
p. 106-107), would be thwarted.
Admittedly, there are American precedents holding that notary public may, in addition,
act as a witness to the executive of the document he has notarized. (Mahilum v. Court
Appeals, 64 0. G. 4017; 17 SCRA 482; Sawyer v. Cox, 43 Ill. 130). There are others
holding that his signing merely as notary in a will nonetheless makes him a witness
thereon (Ferguson v. Ferguson, 47 S. E. 2d. 346; In Re Douglas Will, N. Y. S. 2d. 641;
Ragsdal v. Hill, 269 S. W. 2d. 911, Tyson Utterback, 122 So. 496; In Re Baybee's
Estate 160 N. 900; W. Merill v. Boal, 132 A. 721;See also Trenwith v. Smallwood, 15
So. 1030). But these authorities do not serve the purpose of the law in this jurisdiction
or are not decisive of the issue herein because the notaries public and witnesses
referred to aforecited cases merely acted as instrumental, subscribing attesting
witnesses, and not as acknowledging witnesses. He the notary public acted not only as
attesting witness but also acknowledging witness, a situation not envisaged by Article
805 of the Civil Code which reads:
ART. 806. Every will must be acknowledged before a notary public by
the testator and the witnesses. The notary public shall not be required to
retain a copy of the will or file another with the office of the Clerk of
Court. [Emphasis supplied]
To allow the notary public to act as third witness, or one the attesting and
acknowledging witnesses, would have the effect of having only two attesting witnesses

to the will which would be in contravention of the provisions of Article 80 be requiring at


least three credible witnesses to act as such and of Article 806 which requires that the
testator and the required number of witnesses must appear before the notary public to
acknowledge the will. The result would be, as has been said, that only two witnesses
appeared before the notary public for or that purpose. In the circumstances, the law
would not be duly in observed.
FOR ALL THE FOREGOING, the judgment appealed from is hereby reversed and the
probate of the last will and testament of Valente Z. Cruz (Exhibit "E") is declared not
valid and hereby set aside.
Cost against the appellee.

[G.R. No. L-26615. April 30, 1970.]

Jesus de Praga.

REV. FATHER LUCIO V. GARCIA, ANTONIO JESUS DE PRAGA, MARIA


NATIVIDAD DE JESUS AND DR. JAIME ROSARIO, Petitioners, v. HON. CONRADO
M. VASQUEZ, as Judge of the Court of First Instance of Manila, Branch and
CONSUELO GONZALES VDA. DE PRECILLA,Respondents.

Salonga, Ordoez, Yap, Sicat & Associates for oppositors-appellants Severina


Narciso, Et. Al.
George G. Arbolario and Sixto R. Reyes & Vicente Redor for oppositorsappellants Natividad del Rosario Sarmiento, Et. Al.

[G.R. No. L-26884. April 30, 1970.]


REV. FATHER LUCIO V. GARCIA, ANTONIO JESUS DE PRAGA, MARIA
NATIVIDAD DE JESUS AND DR. JAIME ROSARIO, Petitioners, v. HON. CONRADO
M. VASQUEZ, as Judge of the Court of First Instance of Manila, Branch V,
REGISTER OF DEEDS OF MANILA, and CONSUELO GONZALES VDA. DE
PRECILLA, Respondents.
[G.R. No. L-27200. April 30, 1970.]
TESTATE ESTATE OF GLICERIA A. DEL ROSARIO, deceased CONSUELO S.
GONZALES VDA. DE PRECILLA, petitioner administratrix, v. SEVERINA
NARCISO, ROSA NARCISO, JOSEFINA NARCISO, VICENTE MAURICIO, DELFIN
MAURICIO, REMEDIOS NARCISO, ENCARNACION, NARCISO, MARIA NARCISO,
EDUARDO NARCISO, FR. LUCIO V. GARCIA, ANTONIO JESUS DE PRAGA,
MARIA NATIVIDAD DE JESUS, DR. JAIME DEL ROSARIO, ET AL., NATIVIDAD
DEL ROSARIO-SARMIENTO and PASCUALA NARCISO-MANAHAN, OppositorsAppellants.
Antonio Enrile Inton for petitioner Rev. Father Lucio V. Garcia.
Pedro V. Garcia for petitioner Antonio Jesus de Praga, Et. Al.
Leandro Sevilla & Ramon C. Aquino and Melquiades M. Virata, Jr. for respondent
Consuelo S. Gonzales Vda. de Precilla.
Lorenzo C. Gella for respondent Register of Deeds of Manila. Leandro Sevilla &
Ramon C. Aquino for petitioner administratrix.
Castro, Makalintal & Associates for oppositors-appellants Encarnacion Narciso,
Et. Al.
Pedro Garcia for oppositors-appellants Dr. Jaime Rosario, Et. Al.
Antonio Enrile Inton for oppositors-appellants Fr. Lucio V. Garcia and Antonio

SYLLABUS

1. CIVIL LAW; SUCCESSION, WILLS; PROBATE OF WILLS; GROUND FOR


DISALLOWANCE; TESTATRIXS DEFECTIVE EYESIGHT AS UNABLING HER TO
READ THE PROVISIONS OF LATER WILL. The declarations in court of the
opthalmologist as to the condition of the testatrixs eyesight fully establish the fact that
her vision remained mainly for viewing distant objects and not for reading print; that she
was, at the time of the execution of the second will on December 29, 1960, incapable of
reading and could not have read the provisions of the will supposedly signed by her.
2. ID.; ID.; ID.; ID.; ID.; IRREGULARITIES IN THE EXECUTION OF THE WILL; CASE
AT BAR. Upon its face, the testamentary provisions, the attestation clause and
acknowledgment were crammed together into a single sheet of paper, apparently to
save on space. Plainly, the testament was not prepared with any regard for the
defective vision of Da. Gliceria, the typographical errors remained uncorrected thereby
indicating that the execution thereof must have been characterized by haste. It is
difficult to understand that so important a document containing the final disposition of
ones worldly possessions should be embodied in an informal and untidy written
instrument; or that the glaring spelling errors should have escaped her notice if she had
actually retained the ability to read the purported will and had done so.
3. ID.; ID.; ID.; EXECUTION OF WILLS; REQUISITES FOR VALIDITY; ART. 808, NEW
CIVIL CODE READING OF THE WILL TWICE TO A BLIND TESTATOR; PURPOSE.
The rationale behind the requirement of reading the will to the testator if he is blind
or incapable of reading the will himself is to make the provisions thereof known to him,
so that he may be able to object if they are not in accordance with his wishes.
4. ID.; ID.; ID.; ID.; ID.; ID.; NOT COMPLIED WITH IN INSTANT CASE. Where as in
the 1960 will there is nothing in the record to show that the requisites of Art. 808 of the
Civil Code of the Philippines that "if the testator is blind, the will shall be read to him
twice," have not been complied with, the said 1960 will suffer from infirmity that affects

its due execution.


5. REMEDIAL LAW; SETTLEMENT OF ESTATE OF DECEASED PERSONS;
ADMINISTRATORS; GROUNDS FOR REMOVAL; ACQUISITION OF INTEREST
ADVERSE TO THAT OF THE ESTATE MAKES THE ADMINISTRATOR UNSUITABLE
TO DISCHARGE THE TRUST; CASE AT BAR. Considering that the alleged deed of
sale was executed when Gliceria del Rosario was already practically blind and that the
consideration given seems unconscionably small for the properties, there was
likelihood that a case for annulment might be filed against the estate or heirs of Alfonso
Precilla. And the administratrix being the widow and heir of the alleged transferee,
cannot be expected to sue herself in an action to recover property that may turn out to
belong to the estate. This, plus her conduct in securing new copies of the owners
duplicate of titles without the courts knowledge and authority and having the contract
bind the land through issuance of new titles in her husbands name, cannot but expose
her to the charge of unfitness or unsuitability to discharge the trust, justifying her
removal from the administration of the estate.
6. REMEDIAL LAW; NOTICE OF LIS PENDENS; ACTION MUST AFFECT "THE TITLE
OR THE RIGHT OF POSSESSION OF REAL PROPERTY." On the matter of lis
pendens, the provisions of the Rules of Court are clear: notice of the pendency of an
action may be recorded in the office of the register of deeds of the province in which the
property is situated, if the action affects "the title or the right of possession of (such) real
property."cralaw virtua1aw library

Gliceria Avelino del Rosario dated 29 December 1960. G.R. Nos. L-26615 and L-2684
are separate petitions for mandamus filed by certain alleged heirs of said decedent
seeking (1) to compel the probate court to remove Consuelo S. Gonzales-Precilla as
special administratrix of the estate, for conflict of interest, to appoint a new one in her
stead; and (2) to order the Register of Deeds of Manila to annotate notice of lis
pendens in TCT Nos. 81735, 81736 ,and 81737, registered in the name of Alfonso
Precilla, married to Consuelo Gonzales y Narciso, and said to be properly belonging to
the estate of the deceased Gliceria A. del Rosario.
Insofar as pertinent to the issues involved herein, the facts of these cases may be
stated as follows:chanrob1es virtual 1aw library
Gliceria Avelino del Rosario died unmarried in the City of Manila on 2 September 1965,
leaving no descendents, ascendants, brother or sister. At the time of her death, she
was said to be 90 years old more or less, and possessed of an estate consisting mostly
of real properties.
On 17 September 1965, Consuelo S. Gonzales Vda. de Precilla, a niece of the
deceased, petitioned the Court of First Instance of Manila for probate of the alleged last
will and testament of Gliceria A. del Rosario, executed on 29 December 1960, and for
her appointment as special administratrix of the latters estate, said to be valued at
about P100,000.00, pending the appointment of a regular administrator thereof.

The petition was opposed separately by several groups of alleged heirs: (1) Rev. Fr.
Lucio V. Garcia, a legatee named in an earlier will executed by Gliceria A. del Rosario
on 9 June 1956; (2) Jaime Rosario and children, relatives and legatees in both the
1956 and 1960 wills; Antonio Jesus de Praga and Marta Natividad de Jesus, wards of
the deceased and legatees in the 1956 and 1960 wills; (3) Remedios, Encarnacion, and
Eduardo, all surnamed Narciso; (4) Natividad del Rosario-Sarmiento; (5) Maria Narciso;
(6) Pascuala Narciso de Manahan; (7) Severina, Rosa and Josefa, surnamed Narciso,
and Vicente and Delfin, surnamed Mauricio, the latter five groups of persons all
claiming to be relatives of Doa Gliceria within the fifth civil degree. The oppositions
invariably charged that the instrument executed in 1960 was not intended by the
deceased to be her true will; that the signatures of the deceased appearing in the will
was procured through undue and improper pressure and influence the part of the
beneficiaries and/or other persons; that the testatrix did not know the object of her
bounty; that the instrument itself reveals irregularities in its execution, and that the
REYES, J.B.L., J.:
formalities required by law for such execution have not been complied with.

7. ID.; ID.; ID.; NOT APPLICABLE TO INSTANT CASE. The issue in controversy
here is simply the fitness or unfitness of said special administratrix to continue holding
the trust, it does not involve or affect at all the title to, or possession of, the properties
covered by TCT Nos. 81735, 81736 and 81737. Clearly, the pendency of such case (L26615) is not an action that can properly be annotated in the record of the titles to the
properties.

DECISION

G.R. No. L-27200 is an appeal from the order of the Court of First Instance of Manila (in
Sp. Proc. No. 62618) admitting to probate the alleged last will an, testament of the late

Oppositor Lucio V. Garcia, who also presented for probate the 1956 will of the
deceased, joined the group of Dr. Jaime Rosario in registering opposition to the
appointment of petitioner Consuelo S. Gonzales Vda. de Precilla as special

administratrix, on the ground that the latter possesses interest adverse to the estate.
After the parties were duly heard, the probate court, in its order of 2 October 1965,
granted petitioners prayer and appointed her special administratrix of the estate upon a
bond for P30,000.00. The order was premised on the fact the petitioner was managing
the properties belonging to the estate even during the lifetime of the deceased, and to
appoint another person as administrator or co administrator at that stage of the
proceeding would only result in further confusion and difficulties.
On 30 September 1965, oppositors Jaime Rosario, Et. Al. filed with the probate court
an urgent motion to require the Hongkong & Shanghai Bank to report all withdrawals
made against the funds of the deceased after 2 September 1965. The court denied this
motion on 22 October 1965 for being premature, it being unaware that such deposit in
the name of the deceased existed. 1
On 14 December 1965, the same sets of oppositors, Dr. Jaime Rosario and children,
Antonio Jesus de Praga, Natividad de Jesus and Fr. Lucio V. Garcia, petitioned the
court for the immediate removal of the special administratrix. It was their claim that the
special administratrix and her deceased husband, Alfonso Precilla, 2 had caused
Gliceria A. del Rosario to execute a simulated and fraudulent deed of absolute sale
dated 10 January 1961 allegedly conveying unto said spouses for the paltry sum of
P30,000.00 ownership of 3 parcels of land and the improvements thereon located on
Quiapo and San Nicolas, Manila, with a total assessed value of P334,050.00.
Oppositors contended that since it is the duty of the administrator to protect and
conserve the properties of the estate, and it may become necessary that, an action for
the annulment of the deed of sale land for recovery of the aforementioned parcels of
land be filed against the special administratrix, as wife and heir of Alfonso Precilla, the
removal of the said administratrix was imperative.
On 17 December 1965, the same oppositors prayed the court for an order directing the
Special Administratrix to deposit with the Clerk of Court all certificates of title belonging
to the estate. It was alleged that on 22 October 1965, or after her appointment,
petitioner Consuelo Gonzales Vda. de Precilla, in her capacity as special administratrix
of the estate of the deceased Gliceria A. del Rosario, filed with Branch IV of the Court
of First Instance of Manila a motion for the issuance of new copies of the owners
duplicates of certain certificates of title in the name of Gliceria del Rosario, supposedly
needed by her "in the preparation of the inventory" of the properties constituting the
estate. The motion having been granted, new copies of the owners duplicates of
certificates appearing the name of Gliceria del Rosario (among which were TCT Nos.
66201, 66202 and 66204) were issued on 15 November 1965. On 8 December 1965,
according to the oppositors, the same special administratrix presented to the Register
of Deeds the deed of sale involving properties covered by TCT Nos. 66201, 66202 and
66204 supposedly executed by Gliceria del Rosario on 10 January 1961 in favor of

Alfonso Precilla, and, in consequence, said certificates of title were cancelled and new
certificates (Nos. 81735, 81736 and 81737) were issued in the name of Alfonso
Precilla, married to Consuelo S. Gonzales y Narciso.
On 25 August 1966, the Court issued an order admitting to probate the 1960 will of
Gliceria A. del Rosario (Exhibit "D"). In declaring the due execution of the will, the
probate court took note that no evidence had been presented to establish that the
testatrix was not of sound mind when the will was executed; that the fact that she had
prepared an earlier will did not, prevent her from executing another one thereafter; that
the fact that the 1956 will consisted of 12 pages whereas the 1960 testament was
contained in one page does not render the latter invalid; that, the erasures and
alterations in the instrument were insignificant to warrant rejection; that the
inconsistencies in the testimonies of the instrumental witnesses which were noted by
the oppositors are even indicative of their truthfulness. The probate court, also
considering that petitioner had already shown capacity to administer the properties of
the estate and that from the provisions of the will she stands as the person most
concerned and interested therein, appointed said petitioner regular administratrix with a
bond for P50,000.00. From this order all the oppositors appealed, the case being
docketed in this Court as G.R. No. L-27200.
Then, on 13 September 1966, the probate court resolved the oppositors motion of 14
December 1965 for the removal of the then special administratrix, as
follows:jgc:chanrobles.com.ph
"It would seem that the main purpose of the motion to remove the special administratrix
and to appoint another one in her stead, is in order that an action may be filed against
the special administratrix for the annulment of the deed of sale executed by the
decedent on January 10, 1961. Under existing documents, the properties sold pursuant
to the said deed of absolute sale no longer forms part of the estate. The alleged conflict
of interest is accordingly not between different claimants of the same estate. If it is
desired by the movants that an action be filed by them to annul the aforesaid deed
absolute sale, it is not necessary that the special administratrix be removed and that
another one be appointed to file such action. Such a course of action would only
produce confusion and difficulties in the settlement of the estate. The movants may file
the aforesaid proceedings, preferably in an independent action, to secure the nullity of
the deed of absolute even without leave of this court:"
As regard the motion of 17 December 1965 asking for the deposit in court of the titles in
the name of the decedent, the same was also denied, for the reason that if the movants
were referring to the old titles, they could no longer be produced, and if they meant the
new duplicate copies thereof that were issued at the instance of the special
administratrix, there would be no necessity therefor, because they were already

cancelled and other certificates were issued in the name of Alfonso Precilla. This order
precipitated the oppositors filing in this Court of a petition for mandamus (G.R. No. L26615, Rev. Fr. Lucio V. Garcia, Et. Al. v. Hon. Judge Conrado M. Vasquez, Et. Al.),
which was given due course on 6 October 1966.
On 15 December 1965, with that motion for removal pending in the court, the
oppositors requested the Register of Deeds of Manila to annotate a notice of lis
pendens in the records of TCT Nos. 81735, 81736, and 81737 in the name of Alfonso
Precilla. And when said official refused to do so, they applied to the probate court (in
Sp. Proc. No. 62618) for an order to compel the Register of Deeds to annotate a lis
pendens notice in the aforementioned titles contending that the matter of removal and
appointment of the administratrix, involving TCT Nos. 81735, 81736, and 81737, was
already before the Supreme Court. Upon denial of this motion on 12 November 1966,
oppositors filed another mandamus action, this time against the probate court and the
Register of Deeds. The case was docketed and given due course in this Court as G.R.
No. L-26864.
Foremost of the questions to be determined here concerns the correctness of the order
allowing the probate of the 1960 will.
The records of the probate proceeding fully establish the fact that the testatrix, Gliceria
A. del Rosario, during her lifetime, executed two wills: one on 9 June 1956 consisting of
12 pages and written in Spanish, a language that she knew and spoke, witnessed by
Messrs. Antonio Cabrera, Jesus Y. Ayala and Valentin Marquez, and acknowledged
before notary public Jose Ayala; and another dated 29 December 1960, consisting of 1
page and written in Tagalog, witnessed by Messrs. Vicente Rosales, Francisco Decena,
and Francisco Lopez and acknowledged before notary public Remigio M. Tividad.
Called to testify on the due execution of the 1960 will, instrumental witnesses Decena,
Lopez and Rosales uniformly declared that they were individually requested by Alfonso
Precilla (the late husband of petitioner special administratrix) to witness the execution
of the last will of Doa Gliceria A. del Rosario; that they arrived at the house of the old
lady at No. 2074 Azcarraga, Manila, one after the other, in the afternoon of 29
December 1960; that the testatrix at the time was apparently of clear and sound mind,
although she was being aided by Precilla when she walked; 3 that the will, which was
already prepared, was first read "silently" by the testatrix herself before she signed it; 4
that he three witnesses thereafter signed the will in the presence of the testatrix and the
notary public and of one another. There is also testimony that after the testatrix and the
witnesses to the will acknowledged the instrument to be their voluntary act and deed,
the notary public asked for their respective residence certificates which were handed to
him by Alfonso Precilla, clipped together; 5 that after comparing them with the numbers
already written on the will, the notary public filled in the blanks in the instrument with the

date, 29 January 1960, before he affixed his signature and seal thereto. 6 They also
testified that on that occasion no pressure or influence has been exerted by any person
upon the testatrix to execute the will.
Of course, the interest and active participation of Alfonso Precilla in the signing of this
1960 will are evident from the records. The will appeared to have been prepared by one
who is not conversant with the spelling of Tagalog words, and it has been shown that
Alfonso Precilla is a Cebuano who speaks Tagalog with a Visayan accent. 7 The
witnesses to the will, two of whom are fellow Visayans, 8 admitted their relationship or
closeness to Precilla. 9 It was Precilla who instructed them to go to the house of
Gliceria del Rosario on 29 December 1960 to witness an important document, 10 and
who took their residence certificates from them a few days before the will was signed.
11 Precilla had met the notary public and witnesses Rosales and Lopez at the door of
the residence of the old woman; he ushered them to the room at the second floor
where the signing of the document took place; 12 then he fetched witness Decena from
the latters haberdashery shop a few doors away and brought him to, the house the
testatrix. 13 And when the will was actually executed Precilla was present. 14
The oppositors-appellants in the present case, however, challenging the correctness of
the probate courts ruling, maintain that on 29 December 1960 the eyesight of Gliceria
del Rosario was so poor and defective that she could not have read the provisions of
the will, contrary to the testimonies of witnesses Decena, Lopez and Rosales.
On this point, we find the declarations in court of Dr. Jesus V. Tamesis very material
and illuminating. Said ophthalmologist, whose expertise was admitted by both parties,
testified, among other things, that when Doa Gliceria del Rosario saw him for
consultation on 11 March 1960 he found her left eye to have cataract (opaque lens), 15
and that it was "above normal in pressure", denoting a possible glaucoma, a disease
that leads to blindness 16 As to the conditions of her right eye, Dr. Tamesis
declared:jgc:chanrobles.com.ph
"Q But is there anything here in the entry appearing in the other documents Exhibits 3B, 3-C and 3-D from which you could inform the court as to the condition of the vision of
the patient as to the right eve?
"A Under date of August 30, 1960, is the record of refraction. that is setting of glass by
myself which showed that the right eye with my prescription of glasses had a vision of 2
over 60 (20/60) and for the left eye with her correction 20 over 300 (20/300).
"Q In laymans language, Doctor, what is the significance of that notation that the right
had a degree of 20 over 60 (20/60)?

"A It meant that eye at least would be able to recognize objects or persons at a
minimum distance of twenty feet.
"Q But would that grade enable the patient to read print?
"A Apparently that is only a record for distance vision, for distance sight, not for
near."cralaw virtua1aw library

read?
"A No, not necessarily, only able to go around, take care of herself and see. This I can
tell you, this report was made on pure recollections and I recall she was using her
glasses although I recall also that we have to give her medicines to improve her vision,
some medicines to improve her identification some more.
x

(pages 20-21, t.s.n., hearing of 23 March 1966)


The records also show that although Dr. Tamesis operated of the left eye of the
decedent at the Lourdes Hospital on 8 August 1960; as of 23 August 1960, inspite of
the glasses her vision was only "counting fingers," 17 at five feet. The crossexamination of the doctor further elicited the following
responses:jgc:chanrobles.com.ph
"Q After she was discharged from the hospital you prescribed lenses for her, or
glasses?
"A After her discharge from the hospital, she was coming to my clinic for further
examination and then sometime later glasses were prescribed.
x

"Q And the glasses prescribed by you enabled her to read, Doctor?
"A As far as my record is concerned, with the glasses for the left eye which I prescribed
the eye which I operated she could see only forms but not read. That is on the left
eye.
"Q How about the right eye?
"A The same, although the vision on the right eye is even better than the left eye."
(pages 34. 85. t.s.n., hearing of 23 March 1966).
Then, confronted with a medical certificate (Exhibit H) issued by him on 29 November
1965 certifying that Gliceria del Rosario was provided with aphakic lenses and "had
been under medical supervision up to 1963 with apparently good vision", the doctor had
this to say:jgc:chanrobles.com.ph
"Q When yon said that she had apparently good vision you mean that she was able to

"Q What about the vision in the right eve, was that corrected by the glasses?
"A Yes, with the new prescription which I issued on 80 August 1960. It is in the clinical
record.
"Q The vision in the right eye was corrected?
"A Yes That is the vision for distant objects."cralaw virtua1aw library
(pages 38, 39, 40. t.s.n., hearing of 23 March 1966).
The foregoing testimony of the ophthalmologist who treated the deceased and,
therefore, has first hand knowledge of the actual condition of her eyesight from August,
1960 up to 1963, fully establish the fact that notwithstanding the operation and removal
of the cataract in her left eye and her being fitted with aphakic lens (used by cataract
patients), her vision remained mainly for viewing distant objects and not for reading
print. Thus, the conclusion is inescapable that with the condition of her eyesight in
August, 1960, and there is no evidence that it had improved by 29 December 1960,
Gliceria del Rosario was incapable f reading, and could not have read the provisions of
the will supposedly signed by her on 29 December 1960. It is worth noting that the
instrumental witnesses stated that she read the instrument "silently" (t.s.n., pages 164165). which is a conclusion and not a fact.
Against the background of defective eyesight of the alleged testatrix, the appearance of
the will, Exhibit "D", acquires striking significance. Upon its face, the testamentary
provisions, the attestation clause and acknowledgment were crammed together into a
single sheet of paper, to much so that the words had to be written very close on the top,
bottom and two sides of the paper, leaving no margin whatsoever; the word "and" had
to be written by the symbol" &", apparently to save on space. Plainly, the testament was
not prepared with any regard for the defective vision of Doa Gliceria. Further,
typographical errors like "HULINH" for "HULING" (last), "Alfonsa" ;or "Alfonso",
"MERCRDRS" for MERCEDES", "instrumental" for "Instrumental", and "acknowledged"

for "acknowledge, remained uncorrected, thereby indicating that execution thereof


must have been characterized by haste. It is difficult to understand that so important a
document containing the final disposition of ones worldly possessions should be
embodied in an informal and untidily written instrument; or that the glaring spelling
errors should have escaped her notice if she had actually retained the ability to read the
purported will and had done so. The record is thus convincing that the supposed
testatrix could not have physically read or understood the alleged testament, Exhibit
"D", and that its admission to probate was erroneous and should be reversed.
That Doa Gliceria should be able to greet her guests on her birthday, arrange flowers
and attend to kitchen tasks shortly prior to the alleged execution of the testament
Exhibit "D", as appears from the photographs, Exhibits "E" to "E-1", in no way proves;
that she was able to read a closely typed page, since the acts shown do not require
vision at close range. It must be remembered that with the natural lenses removed, her
eyes had lost the power of adjustment to near vision, the substituted glass lenses being
rigid and uncontrollable by her. Neither is the signing of checks (Exhibits "G" to "G-3")
by her indicative of ability to see at normal reading distances. Writing or signing of ones
name, when sufficiently practiced, becomes automatic, so that one need only to have a
rough indication of the place where the signature is to be affixed in order to be able to
write it. Indeed, a close examination of the checks, amplified in the photograph, Exhibit
"O", et seq., reinforces the contention of oppositors that the alleged testatrix could not
see at normal reading distance: the signatures in the checks are written far above the
printed base, lines, and the names of the payees as well as the amounts written do not
appear to be in the handwriting of the alleged testatrix, being in a much firmer and more
fluid hand than hers.
Thus, for all intents and purpose of the rules on probate, the deceased Gliceria del
Rosario was, as appellant oppositors contend, not unlike a blind testator, and the due
execution of her will would have required observance of the provisions of Article 808 of
the Civil Code.
"ART. 808. If the testator is blind, the will shall be read to him twice; once, by one of the
subscribing witnesses, and again, by the notary public before whom the will is
acknowledged."cralaw virtua1aw library
The rationale behind the requirement of reading the will to the testator if he is blind or
incapable of reading the will himself (as when he is illiterate), 18 is to make the
provisions thereof known to him, so that he may be able to object if they are not in
accordance with his wishes. That the aim of the law is to insure that the dispositions of
the will are properly communicated to and understood by the handicapped testator,
thus making them truly reflective of his desire, is evidenced by the requirement that the
will should be read to the latter, not only once but twice, by two different persons, and

that the witnesses have to act within the range of his (the testators) other senses. 19
In connection with the will here in question, there is nothing in the records to show that
the above requisites have been complied with. Clearly, as already stated, the 1960 will
sought to be probated suffers from infirmity that affects its due execution.
We also find merit in the complaint of oppositors Lucio V. Garcia, Et Al., against the
denial by the probate court of their petition for the removal of Consuelo Gonzales Vda.
de Precilla as special administratrix of the estate of the deceased Doa Gliceria
(Petition, G.R. No. L-26615, Annex "B").
The oppositors petition was based allegedly on the existence in the special
administratrix of an interest adverse to that of the estate. It was their contention that
through fraud her husband had caused the deceased Gliceria del Rosario to execute a
deed of sale, dated 10 January 1961, by virtue of which the latter purportedly conveyed
unto said Alfonso D. Precilla, married to Consuelo Gonzales y Narciso, the ownership
of 3 parcels of land and the improvements thereon, assessed at P334,050.00, for the
sum of P30,000.00.
In denying the petition, the probate court, in its order of 13 September 1966 (Annex "P",
Petition) reasoned out that since the properties were already sold no longer form part of
the estate. The conflict of interest would not be between the estate and third parties,
but among the different claimants of said properties, in which case, according to the
court, the participation of the special administratrix in the action for annulment that may
be brought would not be necessary.
The error in this line of reasoning lies in the fact that what was being questioned was
precisely the validity of the conveyance or sale of the properties. In short, if proper, the
action for annulment would have to be undertaken on behalf of the estate by the special
administratrix, affecting as it does the property or rights of the deceased. 20 For the
rule is that only where there is no special proceeding for the settlement of the estate of
the deceased may the legal heirs commence an action arising out of a right belonging
to their ancestor. 21
There is no doubt that to settle the question of the due execution and validity of the
deed of sale, an ordinary and separate action would have to be instituted, the matter
not falling within the competence of the probate court. 22 Considering the facts then
before it, i.e., the alleged deed of sale having been executed by Gliceria del Rosario on
10 January 1961, when she was already practically blind; and that the consideration of
P30,000.00 seems to be unconscionably small for properties with a total assessed
value of P334,050.00, there was likelihood that a case for annulment might indeed be
filed against the estate or heirs of Alfonso Precilla. And the administratrix, being the

widow and heir of the alleged transferee, cannot be expected to sue herself in an action
to recover property that may turn out to belong to the estate. 22 Not only this, but the
conduct of the special administratrix in securing new copies of the owners duplicates of
TCT Nos. 66201, 66202, and 66204, without the courts knowledge or authority, and on
the pretext that she needed them in the preparation of the inventory of the estate, when
she must have already known by then that the properties covered therein were already
"conveyed" to her husband by the deceased, being the latters successor, and having
the contract bind the land through issuance of new titles in her husbands name cannot
but expose her to the charge of unfitness or unsuitableness to discharge the trust,
justifying her removal from the administration of the estate.
With respect to the orders of the court a quo denying (1) the oppositors motion to
require the Hongkong and Shanghai Bank to report all withdrawals made against the
funds of the deceased after 2 September 1965 and (2) the motion for annotation of a lis
pendens notice on TCT Nos. 81735, 81736 and 81737, the same are to be affirmed.
The probate court pointed out in its order of 22 October 1965 (Annex "H") that it could
not have taken action on the complaint against the alleged withdrawals from the bank
deposits of the deceased, because as of that time the court had not yet been apprised
that such deposits exist. Furthermore, as explained by the special administratrix in her
pleading of 30 October 1965, the withdrawals referred to by the oppositors could be
those covered by checks issued in the name of Gliceria del Rosario during her lifetime
but cleared only after her death. That explanation, which not only appears plausible but
has not been rebutted by the petitioners-oppositors, negates any charge of grave
abuse in connection with the issuance of the order here in question.
On the matter of lis pendens (G.R. No. L-26864), the provisions of the Rules of Court
are clear: notice of the pendency of an action may be recorded in the office of the
register of deeds of the province in which the property is situated, if the action affects
"the title or the right of possession of (such) real property." 23 In the case at bar, the
pending action which oppositors seek to annotate in the records of TCT Nos. 81735,
81736, and 81737 is the mandamus proceeding filed in this Court (G.R. No. L-26615).
As previously discussed in this opinion, however, that case is concerned merely with
the correctness of the denial by the probate court of the motion for the removal of
Consuelo Gonzales Vda. de Precilla as special administratrix of the estate of the late
Gliceria del Rosario. In short, the issue in controversy there is simply the fitness or
unfitness of said special administratrix to continue holding the trust; it does not involve
or affect at all the title to, or possession of, the properties covered by said TCT Nos.
81735, 81736 and 81737. Clearly, the pendency of such case (L-26615) is not an
action that can properly be annotated in the record of the titles to the properties.
FOR THE FOREGOING REASONS, the order of the court below allowing to probate

the alleged 1960 will of Gliceria A. del Rosario is hereby reversed and set aside. The
petition in G.R. No. L-26615 being meritorious, the appealed order is set aside and the
court below is ordered to remove the administratrix, Consuelo Gonzales Vda. de
Precilla, and appoint one of the heirs intestate of the deceased Doa Gliceria Avelino
del Rosario as special administrator for the purpose of instituting action on behalf of her
estate to recover the properties allegedly sold by her to the late Alfonso D. Precilla. And
in Case G.R. No. L-26864, petition is dismissed. No costs.

G.R. No. 74695 September 14, 1993


In the Matter of the Probate of the Last Will and Testament of the Deceased
Brigido Alvarado, CESAR ALVARADO, petitioner, vs. HON. RAMON G. GAVIOLA,
JR., Presiding Justice, HON. MA. ROSARIO QUETULIO LOSA and HON. LEONOR
INES LUCIANO, Associate Justices, Intermediate Appellate Court, First Division
(Civil Cases), and BAYANI MA. RINO, respondents.
Vicente R. Redor for petitioner.
Bayani Ma. Rino for and in his own behalf.

BELLOSILLO, J.:
Before us is an appeal from the Decision dated 11 April 1986 1 of the First Civil Cases
Division of the then Intermediate Appellate Court, now Court of Appeals, which affirmed
the Order dated 27 June 1983 2 of the Regional Trial Court of Sta. Cruz, Laguna,
admitting to probate the last will and testament 3 with codicil 4 of the late Brigido
Alvarado.
On 5 November 1977, the 79-year old Brigido Alvarado executed a notarial will entitled
"Huling Habilin" wherein he disinherited an illegitimate son (petitioner) and expressly
revoked a previously executed holographic will at the time awaiting probate before
Branch 4 of the Regional Trial Court of sta. Cruz, Laguna.

the notarial will to generate cash for the testator's eye operation. Brigido was then
suffering from glaucoma. But the disinheritance and revocatory clauses were
unchanged. As in the case of the notarial will, the testator did not personally read the
final draft of the codicil. Instead, it was private respondent who read it aloud in his
presence and in the presence of the three instrumental witnesses (same as those of
the notarial will) and the notary public who followed the reading using their own copies.
A petition for the probate of the notarial will and codicil was filed upon the testator's
death on 3 January 1979 by private respondent as executor with the Court of First
Instance, now Regional Trial Court, of Siniloan, Laguna. 5Petitioner, in turn, filed an
Opposition on the following grounds: that the will sought to be probated was not
executed and attested as required by law; that the testator was insane or otherwise
mentally incapacitated to make a will at the time of its execution due to senility and old
age; that the will was executed under duress, or influence of fear and threats; that it
was procured by undue and improper pressure and influence on the part of the
beneficiary who stands to get the lion's share of the testator's estate; and lastly, that the
signature of the testator was procured by fraud or trick.
When the oppositor (petitioner) failed to substantiate the grounds relied upon in the
Opposition, a Probate Order was issued on 27 June 1983 from which an appeal was
made to respondent court. The main thrust of the appeal was that the deceased was
blind within the meaning of the law at the time his "Huling Habilin" and the codicil
attached thereto was executed; that since the reading required by Art. 808 of the Civil
Code was admittedly not complied with, probate of the deceased's last will and codicil
should have been denied.

As testified to by the three instrumental witnesses, the notary public and by private
respondent who were present at the execution, the testator did not read the final draft
of the will himself. Instead, private respondent, as the lawyer who drafted the eightpaged document, read the same aloud in the presence of the testator, the three
instrumental witnesses and the notary public. The latter four followed the reading with
their own respective copies previously furnished them.

On 11 April 1986, the Court of Appeals rendered the decision under review with the
following findings: that Brigido Alvarado was not blind at the time his last will and codicil
were executed; that assuming his blindness, the reading requirement of Art. 808 was
substantially complied with when both documents were read aloud to the testator with
each of the three instrumental witnesses and the notary public following the reading
with their respective copies of the instruments. The appellate court then concluded that
although Art. 808 was not followed to the letter, there was substantial compliance since
its purpose of making known to the testator the contents of the drafted will was served.

Meanwhile, Brigido's holographic will was subsequently admitted to probate on 9


December 1977. On the 29th day of the same month, a codicil entitled "Kasulatan ng
Pagbabago sa Ilang Pagpapasiya na Nasasaad sa Huling Habilin na may Petsa
Nobiembre 5, 1977 ni Brigido Alvarado" was executed changing some dispositions in

The issues now before us can be stated thus: Was Brigido Alvarado blind for purpose of
Art, 808 at the time his "Huling Habilin" and its codicil were executed? If so, was the
double-reading requirement of said article complied with?

Regarding the first issue, there is no dispute on the following facts: Brigido Alvarado
was not totally blind at the time the will and codicil were executed. However, his vision
on both eyes was only of "counting fingers at three (3) feet" by reason of the glaucoma
which he had been suffering from for several years and even prior to his first
consultation with an eye specialist on
14 December 1977.
The point of dispute is whether the foregoing circumstances would qualify Brigido as a
"blind" testator under Art. 808 which reads:
Art. 808. If the testator is blind, the will shall be read to him twice; once,
by one of the subscribing witnesses, and again, by the notary public
before whom the will is acknowledged.
Petitioner contends that although his father was not totally blind when the will and
codicil were executed, he can be so considered within the scope of the term as it is
used in Art. 808. To support his stand, petitioner presented before the trial court a
medical certificate issued by Dr. Salvador R. Salceda, Director of the Institute of
Opthalmology (Philippine Eye Research Institute), 6 the contents of which were
interpreted in layman's terms by Dr. Ruperto Roasa, whose expertise was admitted by
private respondent. 7 Dr. Roasa explained that although the testator could visualize
fingers at three (3) feet, he could no longer read either printed or handwritten matters
as of 14 December 1977, the day of his first consultation. 8
On the other hand, the Court of Appeals, contrary to the medical testimony, held that
the testator could still read on the day the will and the codicil were executed but chose
not to do so because of "poor eyesight." 9 Since the testator was still capable of reading
at that time, the court a quo concluded that Art. 808 need not be complied with.
We agree with petitioner in this respect.
Regardless of respondent's staunch contention that the testator was still capable of
reading at the time his will and codicil were prepared, the fact remains and this was
testified to by his witnesses, that Brigido did not do so because of his
"poor," 10 "defective," 11 or "blurred" 12 vision making it necessary for private respondent
to do the actual reading for him.
The following pronouncement in Garcia vs. Vasquez 13 provides an insight into the
scope of the term "blindness" as used in Art. 808, to wit:

The rationale behind the requirement of reading the will to the


testator if he is blind or incapable of reading the will himself (as when he
is illiterate), is to make the provisions thereof known to him, so that he
may be able to object if they are not in accordance with his wishes . . .
Clear from the foregoing is that Art. 808 applies not only to blind testators but also to
those who, for one reason or another, are "incapable of reading the(ir) will(s)." Since
Brigido Alvarado was incapable of reading the final drafts of his will and codicil on the
separate occasions of their execution due to his "poor," "defective," or "blurred" vision,
there can be no other course for us but to conclude that Brigido Alvarado comes within
the scope of the term "blind" as it is used in Art. 808. Unless the contents were read to
him, he had no way of ascertaining whether or not the lawyer who drafted the will and
codicil did so confortably with his instructions. Hence, to consider his will as validly
executed and entitled to probate, it is essential that we ascertain whether Art. 808 had
been complied with.
Article 808 requires that in case of testators like Brigido Alvarado, the will shall be read
twice; once, by one of the instrumental witnesses and, again, by the notary public
before whom the will was acknowledged. The purpose is to make known to the
incapacitated testator the contents of the document before signing and to give him an
opportunity to object if anything is contrary to his instructions.
That Art. 808 was not followed strictly is beyond cavil. Instead of the notary public and
an instrumental witness, it was the lawyer (private respondent) who drafted the eightpaged will and the five-paged codicil who read the same aloud to the testator, and read
them only once, not twice as Art. 808 requires.
Private respondent however insists that there was substantial compliance and that the
single reading suffices for purposes of the law. On the other hand, petitioner maintains
that the only valid compliance or compliance to the letter and since it is admitted that
neither the notary public nor an instrumental witness read the contents of the will and
codicil to Brigido, probate of the latter's will and codicil should have been disallowed.
We sustain private respondent's stand and necessarily, the petition must be denied.
This Court has held in a number of occasions that substantial compliance is acceptable
where the purpose of the law has been satisfied, the reason being that the solemnities
surrounding the execution of wills are intended to protect the testator from all kinds of

fraud and trickery but are never intended to be so rigid and inflexible as to destroy the
testamentary privilege. 14
In the case at bar, private respondent read the testator's will and codicil aloud in the
presence of the testator, his three instrumental witnesses, and the notary public. Prior
and subsequent thereto, the testator affirmed, upon being asked, that the contents read
corresponded with his instructions. Only then did the signing and acknowledgement
take place. There is no evidence, and petitioner does not so allege, that the contents of
the will and codicil were not sufficiently made known and communicated to the testator.
On the contrary, with respect to the "Huling Habilin," the day of the execution was not
the first time that Brigido had affirmed the truth and authenticity of the contents of the
draft. The uncontradicted testimony of Atty. Rino is that Brigido Alvarado already
acknowledged that the will was drafted in accordance with his expressed wishes even
prior to 5 November 1977 when Atty. Rino went to the testator's residence precisely for
the purpose of securing his conformity to the draft.15
Moreover, it was not only Atty. Rino who read the documents on
5 November and 29 December 1977. The notary public and the three instrumental
witnesses likewise read the will and codicil, albeit silently. Afterwards, Atty. Nonia de la
Pena (the notary public) and Dr. Crescente O. Evidente (one of the three instrumental
witnesses and the testator's physician) asked the testator whether the contents of the
document were of his own free will. Brigido answered in the affirmative. 16 With four
persons following the reading word for word with their own copies, it can be safely
concluded that the testator was reasonably assured that what was read to him (those
which he affirmed were in accordance with his instructions), were the terms actually
appearing on the typewritten documents. This is especially true when we consider the
fact that the three instrumental witnesses were persons known to the testator, one
being his physician (Dr. Evidente) and another (Potenciano C. Ranieses) being known
to him since childhood.
The spirit behind the law was served though the letter was not. Although there should
be strict compliance with the substantial requirements of the law in order to insure the
authenticity of the will, the formal imperfections should be brushed aside when they do
not affect its purpose and which, when taken into account, may only defeat the
testator's will. 17
As a final word to convince petitioner of the propriety of the trial court's Probate Order
and its affirmance by the Court of Appeals, we quote the following pronouncement
in Abangan v. Abangan, 18 to wit:

The object of the solemnities surrounding the execution of wills is to


close the door against bad faith and fraud, to avoid the substitution of
wills and testaments and to guaranty their truth and authenticity.
Therefore the laws on the subject should be interpreted in such a way as
to attain these primordial ends. But, on the other hand, also one must
not lose sight of the fact that it is not the object of the law to restrain and
curtail the exercise of the right to make a will. So when an interpretation
already given assures such ends, any other interpretation whatsoever,
that adds nothing but demands more requisites entirely unnecessary,
useless and frustrative of the testator's will, must be
disregarded (emphasis supplied).
Brigido Alvarado had expressed his last wishes in clear and unmistakable terms in his
"Huling Habilin" and the codicil attached thereto. We are unwilling to cast these aside
fro the mere reason that a legal requirement intended for his protection was not
followed strictly when such compliance had been rendered unnecessary by the fact that
the purpose of the law, i.e., to make known to the incapacitated testator the contents of
the draft of his will, had already been accomplished. To reiterate, substantial
compliance suffices where the purpose has been served.
WHEREFORE, the petition is DENIED and the assailed Decision of respondent Court
of Appeals dated 11 April 1986 is AFFIRMED. Considering the length of time that this
case has remained pending, this decision is immediately executory. Costs against
petitioner.
SO ORDERED.

G.R. No. L-37453 May 25, 1979


RIZALINA GABRIEL GONZALES, petitioner, vs. HONORABLE COURT OF
APPEALS and LUTGARDA SANTIAGO, respondents.
Francisco D. Rilloraza, Jr. for petitioners.
Angel A. Sison for private respondent.

GUERRERO, J.:
This is a petition for review of the decision of the Court of Appeals, First
Division, 1 promulgated on May 4, 1973 in CA G.R. No. 36523-R which reversed the
decision of the Court of First Instance of Rizal dated December 15, 1964 and allowed
the probate of the last will and testament of the deceased Isabel Gabriel. *
It appears that on June 24, 1961, herein private respondent Lutgarda Santiago filed a
petition with the Court of First Instance of Rizal docketed as Special Proceedings No.
3617, for the probate of a will alleged to have been executed by the deceased Isabel
Gabriel and designating therein petitioner as the principal beneficiary and executrix.
There is no dispute in the records that the late Isabel Andres Gabriel died as a widow
and without issue in the municipality of Navotas, province of Rizal her place of
residence, on June 7, 1961 at the age of eighty-five (85), having been born in 1876. It is
likewise not controverted that herein private respondent Lutgarda Santiago and
petitioner Rizalina Gabriel Gonzales are nieces of the deceased, and that private
respondent, with her husband and children, lived with the deceased at the latters
residence prior an- d up to the time of her death.
The will submitted for probate, Exhibit "F", which is typewritten and in Tagalog, appears
to have been executed in Manila on the 15th day of April, 1961, or barely two (2)
months prior to the death of Isabel Gabriel. It consists of five (5) pages, including the
pages whereon the attestation clause and the acknowledgment of the notary public
were written. The signatures of the deceased Isabel Gabriel appear at the end of the
will on page four and at the left margin of all the pages. The attestation clause, which is
found on page four, reads as follows:

PATUNAY NG MGA SAKSI


Kaming mga nakalagdang mga saksi o testigo na ang aming mga
tinitirahan ay nakasulat sa gawing kanan at kahilira ng aming mga
pangalan sa ibaba nito, ay pagpapatutuo na ipinakilala ipinaalam at
ipinahayag sa amin ni Isabel Gabriel na ang kasulatang ito na binubuo
ng Limang Dahon (Five Pages) pati na ang dahong ito, na siya niyang
TESTAMENTO AT HULING HABILIN, ngayong ika 15 ng Abril, 1961, ay
nilagdaan ng nasabing testadora na si Isabel Gabriel ang nasabing
testamento sa ibaba o ilalim ng kasulatan na nasa ika apat na dahon
(page four) at nasa itaas ng patunay naming ito, at sa kaliwang panig ng
lahat at bawat dahon (and on the left hand margin of each and every
page), sa harap ng lahat at bawat isa sa amin, at kami namang mga
saksi ay lumagda sa harap ng nasabing testadora, at sa harap ng lahat
at bawat isa sa amin, sa ilalim ng patunay ng mga saksi at sa kaliwang
panig ng lahat at bawa't dahon ng testamentong ito.
At the bottom thereof, under the heading "Pangalan", are written the signatures of
Matilde D. Orobia, Celso D. Gimpaya and Maria R. Gimpaya, and opposite the same,
under the heading "Tirahan", are their respective places of residence, 961 Highway 54,
Philamlife, for Miss Orobia, and 12 Dagala St., Navotas, Rizal, for the two Gimpayas.
Their signatures also appear on the left margin of all the other pages. The WW is paged
by typewritten words as follows: "Unang Dahon" and underneath "(Page One)",
"Ikalawang Dahon" and underneath "(Page Two)", etc., appearing at the top of each
page.
The will itself provides that the testatrix desired to be buried in the Catholic Cemetery of
Navotas, Rizal in accordance with the rites of the Roman Catholic Church, all expenses
to be paid from her estate; that all her obligations, if any, be paid; that legacies in
specified amounts be given to her sister, Praxides Gabriel Vda. de Santiago, her
brother Santiago Gabriel, and her nephews and nieces, Benjamin, Salud, Rizalina
(herein petitioner), Victoria, Ester, Andres, all surnamed Gabriel, and Evangeline,
Rudyardo Rosa, Andrea, Marcial, Numancia, Verena an surnamed Santiago. To herein
private respondent Lutgarda Santiago, who was described in the will by the testatrix as
"aking mahal na pamangkin na aking pinalaki, inalagaan at minahal na katulad ng isang
tunay na anak" and named as universal heir and executor, were bequeathed all
properties and estate, real or personal already acquired, or to be acquired, in her
testatrix name, after satisfying the expenses, debts and legacies as aforementioned.

The petition was opposed by Rizalina Gabriel Gonzales, herein petitioner, assailing the
document purporting to be the will of the deceased on the following grounds:

WHEREFORE, Exhibit "F", the document presented for probate as the


last wig and testament of the deceased Isabel Gabriel is here by
DISALLOWED.

1. that the same is not genuine; and in the alternative


2. that the same was not executed and attested as required by law;
3. that, at the time of the alleged execution of the purported wilt the
decedent lacked testamentary capacity due to old age and sickness; and
in the second alternative
4. That the purported WW was procured through undue and improper
pressure and influence on the part of the principal beneficiary, and/or of
some other person for her benefit.
Lutgarda Santiago filed her Answer to the Opposition on February 1, 1962. After trial,
the court a quo rendered judgment, the summary and dispositive portions of which
read:
Passing in summary upon the grounds advanced by the oppositor, this
Court finds:
1. That there is no iota of evidence to support the contentio that the
purported will of the deceased was procured through undue and
improper pressure and influence on the part of the petitioner, or of some
other person for her benefit;
2. That there is insufficient evidence to sustain the contention that at the
time of the alleged execution of the purported will, the deceased lacked
testamentary capacity due to old age and sickness;
3. That sufficient and abundant evidence warrants conclusively the fact
that the purported will of the deceased was not executed and attested as
required by law;
4. That the evidence is likewise conclusive that the document presented
for probate, Exhibit 'F' is not the purported win allegedly dictated by the
deceased, executed and signed by her, and attested by her three
attesting witnesses on April 15, 1961.

From this judgment of disallowance, Lutgarda Santiago appealed to respondent Court,


hence, the only issue decided on appeal was whether or not the will in question was
executed and attested as required by law. The Court of Appeals, upon consideration of
the evidence adduced by both parties, rendered the decision now under review, holding
that the will in question was signed and executed by the deceased Isabel Gabriel on
April 15, 1961 in the presence of the three attesting witnesses, Matilde Orobia, Celso
Gimpaya and Maria Gimpaya, signing and witnessing the document in the presence of
the deceased and of each other as required by law, hence allow ed probate.
Oppositor Rizalina Gabriel Gonzales moved for reconsideration 3 of the aforesaid
decision and such motion was opposed 4 by petitioner-appellant Lutgarda Santiago.
Thereafter. parties submitted their respective Memoranda, 5 and on August 28, 1973,
respondent Court, Former Special First Division, by Resolution 6 denied the motion for
reconsideration stating that:
The oppositor-appellee contends that the preponderance of evidence
shows that the supposed last wig and testament of Isabel Gabriel was
not executed in accordance with law because the same was signed on
several occasions, that the testatrix did not sign the will in the presence
of all the instrumental witnesses did not sign the will in the presence of
each other.
The resolution of the factual issue raised in the motion for
reconsideration hinges on the appreciation of the evidence. We have
carefully re-examined the oral and documentary evidence of record,
There is no reason to alter the findings of fact in the decision of this
Court sought to be set aside. 7
In her petition before this Court, oppositor Rizalina Gabriel Gonzales contends that
respondent Court abused its discretion and/or acted without or in excess of its
jurisdiction in reverssing the findings of fact and conclusions of the trial court. The
Court, after deliberating on the petition but without giving due course resolved, in the
Resolution dated Oct. 11, 1973 to require the respondents to comment thereon, which
comment was filed on Nov. 14, 1973. Upon consideration of the allegations, the issues
raised and the arguments adduced in the petition, as well as the Comment 8 of private

respondent thereon, We denied the petition by Resolution on November 26, 1973, 9 the
question raised being factual and for insufficient showing that the findings of fact by
respondent Court were unsupported by substantial evidence.

VIII. The Court of Appeals erred in holding that the grave contradictions, evasions, and
misrepresentations of witnesses (subscribing and notary) presented by the petitioner
had been explained away, and that the trial court erred in rejecting said testimonies.

Subsequently, or on December 17, 1973, petitioner Rim Gabriel Goes fried a Motion for
Reconsideration 10which private respondent answered by way of her Comment or
Opposition 11 filed on January 15, 1974. A Reply and Rejoinder to Reply followed.
Finally, on March 27, 1974, We resolved to give due course to the petition.

IX. The Court of Appeals acted in excess of its appellate jurisdiction or has so far
departed from the accepted and usual course of judicial proceedings, as to call for an
exercise of the power of supervision.

The petitioner in her brief makes the following assignment of errors:


I. The respondent Court of Appeals erred in holding that the document, Exhibit "F" was
executed and attested as required by law when there was absolutely no proof that the
three instrumental witnesses were credible witness
II. The Court of Appeals erred in reversing the finding of the lower court that the
preparation and execution of the win Exhibit "F", was unexpected and coincidental.
III. The Court of Appeals erred in finding that Atty, Paraiso was not previously furnished
with the names and residence certificates of the witnesses as to enable him to type
such data into the document Exhibit "F".
IV. The Court of Appeals erred in holding that the fact that the three typewritten lines
under the typewritten words "Pangalan" and "Tinitirahan" were left blank shows beyond
cavil that the three attesting witnesses were all present in the same occasion.
V. The Court of Appeals erred in reversing the trial court's finding that it was incredible
that Isabel Gabriel could have dictated the wilt Exhibit "F , without any note or
document, to Atty. Paraiso.
VI. The Court of Appeals erred in reversing the finding of the trial court that Matilde
Orobia was not physically present when the Will Exhibit "F" was allegedly signed on
April 15, 1961 by the deceased Isabel Gabriel and the other witnesses Celso Gimpaya
and Maria Gimpaya.
VII. The Court of Appeals erred in holding that the trial court gave undue importance to
the picture takings as proof that the win was improperly executed.

X. The Court of Appeals erred in reversing the decision of the trial court and admitting
to probate Exhibit "F", the alleged last will and testament of the deceased Isabel
Gabriel.
It will be noted from the above assignments of errors that the same are substantially
factual in character and content. Hence, at the very outset, We must again state the oftrepeated and well-established rule that in this jurisdiction, the factual findings of the
Court of Appeals are not reviewable, the same being binding and conclusive on this
Court. This rule has been stated and reiterated in a long line of cases enumerated
in Chan vs. CA (L-27488, June 30, 1970, 33 SCRA 737, 743) 12 and Tapas vs. CA (L22202, February 27; 1976, 69 SCRA 393), 13 and in the more recent cases of Baptisia
vs. Carillo and CA (L32192, July 30, 1976, 72 SCRA 214, 217) and Vda. de Catindig
vs. Heirs of Catalina Roque (L-25777, November 26, 1976, 74 SCRA 83, 88). In the
case of Chan vs. CA, this Court said:
... from Guico v. Mayuga, a 1936 decision, the opinion being penned by the then
Justice Recto, it has been well-settled that the jurisdiction of tills Court in cases brought
to us from the Court of Appeals is limited to reviewing and revising the errors of law
imputed to it, its findings of fact being conclusive. More specifically, in a decision exactly
a month later, this Court, speaking through the then Justice Laurel, it was held that the
same principle is applicable, even if the Court of Appeals was in disagreement with the
lower court as to the weight of the evidence with a consequent reversal of its findings of
fact ...
Stated otherwise, findings of facts by the Court of Appeals, when supported by
substantive evidence are not reviewable on appeal by certiorari. Said findings of the
appellate court are final and cannot be disturbed by Us particularly because its
premises are borne out by the record or based upon substantial evidence and what is
more, when such findings are correct. Assignments of errors involving factual issues
cannot be ventilated in a review of the decision of the Court of Appeals because only
legal questions may be raised. The Supreme Court is not at liberty to alter or modify the

facts as set forth in the decision of the Court of Appeals sought to be reversed. Where
the findings of the Court of Appeals are contrary to those of the trial court, a minute
scrutiny by the Supreme Court is in order, and resort to duly-proven evidence becomes
necessary. The general rule We have thus stated above is not without some recognized
exceptions.
Having laid down the above legal precepts as Our foundation, We now proceed to
consider petitioner's assignments of errors.
Petitioner, in her first assignment, contends that the respondent Court of Appeals erred
in holding that the document, Exhibit "F", was executed and attested as required by law
when there was absolutely no proof that the three instrumental witnesses were credible
witnesses. She argues that the require. ment in Article 806, Civil Code, that the
witnesses must be credible is an absolute requirement which must be complied with
before an alleged last will and testament may be admitted to probate and that to be a
credible witness, there must be evidence on record that the witness has a good
standing in his community, or that he is honest and upright, or reputed to be trustworthy
and reliable. According to petitioner, unless the qualifications of the witness are first
established, his testimony may not be favorably considered. Petitioner contends that
the term "credible" is not synonymous with "competent" for a witness may be
competent under Article 820 and 821 of the Civil Code and still not be credible as
required by Article 805 of the same Code. It is further urged that the term "credible" as
used in the Civil Code should receive the same settled and well- known meaning it has
under the Naturalization Law, the latter being a kindred legislation with the Civil Code
provisions on wigs with respect to the qualifications of witnesses.
We find no merit to petitioner's first assignment of error. Article 820 of the Civil Code
provides the qualifications of a witness to the execution of wills while Article 821 sets
forth the disqualification from being a witness to a win. These Articles state:
Art. 820. Any person of sound mind and of the age of eighteen years or
more, and not blind, deaf or dumb, and able to read and write, may be a
witness to the execution of a will mentioned in article 806 of this Code.
"Art. 821. The following are disqualified from being witnesses to a will:
(1) Any person not domiciled in the Philippines,
(2) Those who have been convicted of falsification of a document,
perjury or false testimony.

Under the law, there is no mandatory requirement that the witness testify initially or at
any time during the trial as to his good standing in the community, his reputation for
trustworthythiness and reliableness, his honesty and uprightness in order that his
testimony may be believed and accepted by the trial court. It is enough that the
qualifications enumerated in Article 820 of the Civil Code are complied with, such that
the soundness of his mind can be shown by or deduced from his answers to the
questions propounded to him, that his age (18 years or more) is shown from his
appearance, testimony , or competently proved otherwise, as well as the fact that he is
not blind, deaf or dumb and that he is able to read and write to the satisfaction of the
Court, and that he has none of the disqualifications under Article 821 of the Civil Code.
We reject petitioner's contention that it must first be established in the record the good
standing of the witness in the community, his reputation for trustworthiness and
reliableness, his honesty and uprightness, because such attributes are presumed of the
witness unless the contrary is proved otherwise by the opposing party.
We also reject as without merit petitioner's contention that the term "credible" as used
in the Civil Code should be given the same meaning it has under the Naturalization Law
where the law is mandatory that the petition for naturalization must be supported by two
character witnesses who must prove their good standing in the community, reputation
for trustworthiness and reliableness, their honesty and uprightness. The two witnesses
in a petition for naturalization are character witnesses in that being citizens of the
Philippines, they personally know the petitioner to be a resident of the Philippines for
the period of time required by the Act and a person of good repute and morally
irreproachable and that said petitioner has in their opinion all the qualifications
necessary to become a citizen of the Philippines and is not in any way disqualified
under the provisions of the Naturalization Law (Section 7, Commonwealth Act No. 473
as amended).
In probate proceedings, the instrumental witnesses are not character witnesses for they
merely attest the execution of a will or testament and affirm the formalities attendant to
said execution. And We agree with the respondent that the rulings laid down in the
cases cited by petitioner concerning character witnesses in naturalization proceedings
are not applicable to instrumental witnesses to wills executed under the Civil Code of
the Philippines.
In the case at bar, the finding that each and everyone of the three instrumental
witnesses, namely, Matilde Orobia, Celso Gimpaya and Maria Gimpaya, are competent
and credible is satisfactorily supported by the evidence as found by the respondent
Court of Appeals, which findings of fact this Tribunal is bound to accept and rely upon.

Moreover, petitioner has not pointed to any disqualification of any of the said witnesses,
much less has it been shown that anyone of them is below 18 years of age, of unsound
mind, deaf or dumb, or cannot read or write.
It is true that under Article 805 of the New Civil Code, every will, other than a
holographic will, must be subscribed at the end thereof by the testator himself or by the
testator's name written by some other person in his presence, and by his express
direction, and attested and subscribed by three or more credible witnesses in the
presence of the testator and of one another, While the petitioner submits that Article
820 and 821 of the New Civil Code speak of the competency of a witness due to his
qualifications under the first Article and none of the disqualifications under the second
Article, whereas Article 805 requires the attestation of three or more credible witnesses,
petitioner concludes that the term credible requires something more than just being
competent and, therefore, a witness in addition to being competent under Articles 820
and 821 must also be a credible witness under Article 805.
Petitioner cites American authorities that competency and credibility of a witness are
not synonymous terms and one may be a competent witness and yet not a credible
one. She exacerbates that there is no evidence on record to show that the instrumental
witnesses are credible in themselves, that is, that they are of good standing in the
community since one was a family driver by profession and the second the wife of the
driver, a housekeeper. It is true that Celso Gimpaya was the driver of the testatrix and
his wife Maria Gimpaya, merely a housekeeper, and that Matilde Orobia was a piano
teacher to a grandchild of the testatrix But the relation of employer and employee much
less the humble or financial position of a person do not disqualify him to be a
competent testamentary witness. (Molo Pekson and Perez Nable vs. Tanchuco, et al.,
100 Phil. 344; Testate Estate of Raymundo, Off. Gaz., March 18,1941, p. 788).
Private respondent maintains that the qualifications of the three or more credible
witnesses mentioned in Article 805 of the Civil Code are those mentioned in Article 820
of the same Code, this being obvious from that portion of Article 820 which says "may
be Q witness to the execution of a will mentioned in Article 805 of this Code," and cites
authorities that the word "credible" insofar as witnesses to a will are concerned simply
means " competent." Thus, in the case of Suntay vs. Suntay, 95 Phil. 500, the Supreme
Court held that "Granting that a will was duly executed and that it was in existence at
the time of, and not revoked before, the death of the testator, still the provisions of the
lost wig must be clearly and distinctly proved by at least two credible
witnesses. 'Credible witnesses' mean competent witnesses and not those who testify to
facts from or upon hearsay. " emphasissupplied).

In Molo Pekson and Perez Nable vs. Tanchuco, et al., 100 Phil. 344, the Supreme
Court held that "Section 620 of the same Code of Civil Procedure provides that any
person of sound mind, and of the age of eighteen years or more, and not blind, deaf, or
dumb and able to read and write, may be a witness to the execution of a will. This same
provision is reproduced in our New Civil Code of 1950, under Art. 820. The relation of
employer and employee, or being a relative to the beneficiary in a win, does not
disqualify one to be a witness to a will. The main qualification of a witness in the
attestation of wills, if other qualifications as to age, mental capacity and literacy are
present, is that said witness must be credible, that is to say, his testimony may be
entitled to credence. There is a long line of authorities on this point, a few of which we
may cite:
A 'credible witness is one who is not is not to testify by mental incapacity,
crime, or other cause. Historical Soc of Dauphin County vs. Kelker 74 A.
619, 226 Pix 16, 134 Am. St. Rep. 1010. (Words and Phrases, Vol. 10, p.
340).
As construed by the common law, a 'credible witness' to a will means a
'competent witness.' Appeal of Clark, 95 A. 517, 114 Me. 105, Ann. Cas.
1917A, 837. (lbid, p. 341).
Expression 'credible witness' in relation to attestation of wins means
'competent witness that is, one competent under the law to testify to fact
of execution of will. Vernon's Ann. Civ St. art. 8283. Moos vs. First State
Bank of Uvalde, Tex . Civ. App. 60 S.W. 2nd 888, 889. (Ibid, p. 342)
The term 'credible', used in the statute of wills requiring that a will shall
be attested by two credible witnesses means competent; witnesses who,
at the time of attesting the will, are legally competent to testify, in a court
of justice, to the facts attested by subscribing the will, the competency
being determined as of the date of the execution of the will and not of
the timr it is offered for probate,Smith vs. Goodell 101 N.E. 255, 256,
258 111. 145. (Ibid.)
Credible witnesses as used in the statute relating to wills, means
competent witnesses that is, such persons as are not legally
disqualified from testifying in courts of justice, by reason of mental
incapacity, interest, or the commission of crimes, or other cause
excluding them from testifying generally, or rendering them incompetent

in respect of the particular subject matter or in the particular suit. Hill vs.
Chicago Title & Trust co 152 N.E. 545, 546, 322 111. 42. (Ibid. p, 343)
In the strict sense, the competency of a person to be an instrumental witness to a will is
determined by the statute, that is Art. 820 and 821, Civil Code, whereas his credibility
depends On the appreciation of his testimony and arises from the belief and conclusion
of the Court that said witness is telling the truth. Thus, in the case ofVda. de Aroyo v. El
Beaterio del Santissimo Rosario de Molo, No. L-22005, May 3, 1968, the Supreme
Court held and ruled that: "Competency as a witness is one thing, and it is another to
be a credible witness, so credible that the Court must accept what he says. Trial courts
may allow a person to testify as a witness upon a given matter because he is
competent, but may thereafter decide whether to believe or not to believe his
testimony." In fine, We state the rule that the instrumental witnesses in Order to be
competent must be shown to have the qualifications under Article 820 of the Civil Code
and none of the disqualifications under Article 821 and for their testimony to be
credible, that is worthy of belief and entitled to credence, it is not mandatory that
evidence be first established on record that the witnesses have a good standing in the
community or that they are honest and upright or reputed to be trustworthy and reliable,
for a person is presumed to be such unless the contrary is established otherwise. In
other words, the instrumental witnesses must be competent and their testimonies must
be credible before the court allows the probate of the will they have attested. We,
therefore, reject petitioner's position that it was fatal for respondent not to have
introduced prior and independent proof of the fact that the witnesses were "credible
witnesses that is, that they have a good standing in the community and reputed to be
trustworthy and reliable.
Under the second, third, fourth, fifth, sixth, seventh and eighth assignments of errors,
petitioner disputes the findings of fact of the respondent court in finding that the
preparation and execution of the will was expected and not coincidental, in finding that
Atty. Paraiso was not previously furnished with the names and residence certificates of
the witnesses as to enable him to type such data into the document Exhibit "F", in
holding that the fact that the three typewritten lines under the typewritten words
"pangalan" and "tinitirahan" were left blank shows beyond cavil that the three attesting
witnesses were all present in the same occasion, in holding credible that Isabel Gabriel
could have dictated the will without note or document to Atty. Paraiso, in holding that
Matilde Orobia was physically present when the will was signed on April 15, 1961 by
the deceased Isabel Gabriel and the other witnesses Celso Gimpaya and Maria
Gimpaya, in holding that the trial court gave undue importance to the picture takings as
proof that the will was improperly executed, and in holding that the grave

contradictions, evasions and misrepresentations of the witnesses (subscribing and


notary) presented by the petitioner had been explained away.
Since the above errors are factual We must repeat what We have previously laid down
that the findings of fact of the appellate court are binding and controlling which We
cannot review, subject to certain exceptions which We win consider and discuss
hereinafter. We are convinced that the appellate court's findings are sufficiently justified
and supported by the evidence on record. Thus, the alleged unnaturalness
characterizing the trip of the testatrix to the office of Atty. Paraiso and bringing all the
witnesses without previous appointment for the preparation and execution of the win
and that it was coincidental that Atty. Paraiso was available at the moment impugns the
finding of the Court of Appeals that although Atty. Paraiso admitted the visit of Isabel
Gabriel and of her companions to his office on April 15, 1961 was unexpected as there
was no prior appointment with him, but he explained that he was available for any
business transaction on that day and that Isabel Gabriel had earlier requested him to
help her prepare her will. The finding of the appellate court is amply based on the
testimony of Celso Gimpaya that he was not only informed on the morning of the day
that he witnessed the will but that it was the third time when Isabel Gabriel told him that
he was going to witness the making of her will, as well as the testimony of Maria
Gimpaya that she was called by her husband Celso Gimpaya to proceed to Isabel
Gabriel's house which was nearby and from said house, they left in a car to the lawyer's
office, which testimonies are recited in the respondent Court's decision.
The respondent Court further found the following facts: that Celso Gimpaya and his
wife Maria Gimpaya obtained residence certificates a few days before Exhibit "F" was
executed. Celso Gimpaya's residence certificate No. A-5114942 was issued at Navotas,
Rizal on April 13, 1961 while Maria Gimpaya's residence certificate No. A-5114974 was
issued also at Navotas, Rizal on April 14, 1961. The respondent Court correctly
observed that there was nothing surprising in these facts and that the securing of these
residence certificates two days and one day, respectively, before the execution of the
will on April 15, 1961, far from showing an amazing coincidence, reveals that the
spouses were earlier notified that they would be witnesses to the execution of Isabel
Gabriel's will.
We also agree with the respondent Court's conclusion that the excursion to the office of
Atty. Paraiso was planned by the deceased, which conclusion was correctly drawn from
the testimony of the Gimpaya spouses that they started from the Navotas residence of
the deceased with a photographer and Isabel Gabriel herself, then they proceeded by
car to Matilde Orobia's house in Philamlife, Quezon City to fetch her and from there, all

the three witnesses (the Gimpayas and Orobia) passed by a place where Isabel Gabriel
stayed for about ten to fifteen minutes at the clinic of Dr. Chikiamco before they
proceeded to Atty. Cipriano Paraiso's office.
It is also evident from the records, as testified to by Atty. Paraiso, that previous to the
day that. the will was executed on April 15, 1961, Isabel Gabriel had requested him to
help her in the execution of her will and that he told her that if she really wanted to
execute her will, she should bring with her at least the Mayor of Navotas, Rizal and a
Councilor to be her witnesses and that he (Atty. Paraiso) wanted a medical certificate
from a physician notwithstanding the fact that he believed her to be of sound and
disposition mind. From this evidence, the appellate court rightly concluded, thus: "It is,
therefore, clear that the presence of Isabel Gabriel and her witnesses Matilde Orobia,
Celso Gimpaya and Maria Gimpaya including the photographer in the law office of Atty.
Paraiso was not coincidental as their gathering was pre-arranged by Isabel Gabriel
herself."
As to the appellate court's finding that Atty. Paraiso was not previously furnished with
the names and residence certificates of the witnesses as to enable him to type such
data into the document Exhibit ' L which the petitioner assails as contradictory and
irreconcilable with the statement of the Court that Atty. Paraiso was handed a list
(containing the names of the witnesses and their respective residence certificates)
immediately upon their arrival in the law office by Isabel Gabriel and this was
corroborated by Atty. Paraiso himself who testified that it was only on said occasion that
he received such list from Isabel Gabriel, We cannot agree with petitioner's contention.
We find no contradiction for the, respondent Court held that on the occasion of the will
making on April 15, 1961, the list was given immediately to Atty. Paraiso and that no
such list was given the lawyer in any previous occasion or date prior to April 15, 1961.
But whether Atty. Paraiso was previously furnished with the names and residence
certificates of the witnesses on a prior occasion or on the very occasion and date in
April 15, 1961 when the will was executed, is of no moment for such data appear in the
notarial acknowledgment of Notary Public Cipriano Paraiso, subscribed and sworn to
by the witnesses on April 15, 1961 following the attestation clause duly executed and
signed on the same occasion, April 15, 1961. And since Exhibit "F" is a notarial will duly
acknowledged by the testatrix and the witnesses before a notary public, the same is a
public document executed and attested through the intervention of the notary public
and as such public document is evidence of the facts in clear, unequivocal manner
therein expressed. It has in its favor the presumption of regularity. To contradict all
these, there must be evidence that is clear, convincing and more than merely

preponderant. (Yturalde vs. Azurin, 28 SCRA 407). We find no such evidence pointed
by petitioner in the case at bar.
Likewise, the conclusion of the Court of Appeals in holding that the fact that the three
typewritten lines under the typewritten words "pangalan ' and "tinitirahan" were left
blank shows beyond cavil that the three attesting witnesses were all present in the
same occasion merits Our approval because tills conclusion is supported and borne out
by the evidence found by the appellate court, thus: "On page 5 of Exhibit "F", beneath
the typewritten words "names", "Res. Tax Cert. date issued" and place issued the only
name of Isabel Gabriel with Residence Tax certificate No. A-5113274 issued on
February 24, 1961 at Navotas Rizal appears to be in typewritten form while the names,
residence tax certificate numbers, dates and places of issuance of said certificates
pertaining to the three (3) witnesses were personally handwritten by Atty. Paraiso.
Again, this coincides with Atty. Paraiso's even the sale must be made to close relatives;
and the seventh was the appointment of the appellant Santiago as executrix of the will
without bond. The technical description of the properties in paragraph 5 of Exhibit F
was not given and the numbers of the certificates of title were only supplied by Atty.
Paraiso. "
It is true that in one disposition, the numbers of the Torrens titles of the properties
disposed and the docket number of a special proceeding are indicated which Atty.
Paraiso candidly admitted were supplied by him, whereupon petitioner contends that it
was incredible that Isabel Gabriel could have dictated the will Exhibit "F" without any
note or document to Atty. Paraiso, considering that Isabel Gabriel was an old and sickly
woman more than eighty-one years old and had been suffering from a brain injury
caused by two severe blows at her head and died of terminal cancer a few weeks after
the execution of Exhibit "F". While we can rule that this is a finding of fact which is
within the competency of the respondent appellate court in determining the
testamentary capacity of the testatrix and is, therefore, beyond Our power to revise and
review, We nevertheless hold that the conclusion reached by the Court of Appeals that
the testatrix dictated her will without any note or memorandum appears to be fully
supported by the following facts or evidence appearing on record. Thus, Isabel Gabriel,
despite her age, was particularly active in her business affairs as she actively managed
the affairs of the movie business ISABELITA Theater, paying the aparatistas herself
until June 4, 1961, 3 days before her death. She was the widow of the late Eligio Naval,
former Governor of Rizal Province and acted as coadministratrix in the Intestate Estate
of her deceased husband Eligio Naval. The text of the win was in Tagalog, a dialect
known and understood by her and in the light of all the circumstances, We agree with
the respondent Court that the testatrix dictated her will without any note or

memorandum, a fact unanimously testified to by the three attesting witnesses and the
notary public himself.
Petitioner's sixth assignment of error is also bereft of merit. The evidence, both
testimonial and documentary is, according to the respondent court, overwhelming that
Matilde Orobia was physically present when the will was signed on April 15, 1961 by
the testatrix and the other two witnesses, Celso Gimpaya and Maria Gimpaya. Such
factual finding of the appellate court is very clear, thus: "On the contrary, the record is
replete with proof that Matilde Orobia was physically present when the will was signed
by Isabel Gabriel on April '15, 1961 along with her co-witnesses Celso Gimpaya and
Maria Gimpaya. The trial court's conclusion that Orobia's admission that she gave
piano lessons to the child of the appellant on Wednesdays and Saturdays and that April
15, 1961 happened to be a Saturday for which reason Orobia could not have been
present to witness the will on that day is purely conjectural. Witness Orobia did not
admit having given piano lessons to the appellant's child every Wednesday and
Saturday without fail. It is highly probable that even if April 15, 1961 were a Saturday,
she gave no piano lessons on that day for which reason she could have witnessed the
execution of the will. Orobia spoke of occasions when she missed giving piano lessons
and had to make up for the same. Anyway, her presence at the law office of Atty.
Paraiso was in the morning of April 15, 1961 and there was nothing to preclude her
from giving piano lessons on the afternoon of the same day in Navotas, Rizal."
In addition to the testimony of Matilde Orobia, Celso Gimpaya and Maria Gimpaya that
Matilde was present on April 15, 1961 and that she signed the attestation clause to the
will and on the left-hand margin of each of the pages of the will, the documentary
evidence which is the will itself, the attestation clause and the notarial acknowledgment
overwhelmingly and convincingly prove such fact that Matilde Orobia was present on
that day of April 15, 1961 and that she witnessed the will by signing her name thereon
and acknowledged the same before the notary public, Atty. Cipriano P. Paraiso. The
attestation clause which Matilde Orobia signed is the best evidence as to the date of
signing because it preserves in permanent form a recital of all the material facts
attending the execution of the will. This is the very purpose of the attestation clause
which is made for the purpose of preserving in permanent form a record of the facts
attending the execution of the will, so that in case of failure in the memory of the
subscribing witnesses, or other casualty they may still be proved. (Thompson on Wills,
2nd ed., Sec. 132; Leynez vs. Leynez, 68 Phil. 745).
As to the seventh error assigned by petitioner faulting the Court of Appeals in holding
that the trial court gave undue importance to the picture-takings as proof that the win

was improperly executed, We agree with the reasoning of the respondent court that:
"Matilde Orobia's Identification of the photographer as "Cesar Mendoza", contrary to
what the other two witnesses (Celso and Maria Gimpaya) and Atty. Paraiso said that
the photographer was Benjamin Cifra, Jr., is at worst a minor mistake attributable to
lapse of time. The law does not require a photographer for the execution and
attestation of the will. The fact that Miss Orobia mistakenly Identified the photographer
as Cesar Mendoza scarcely detracts from her testimony that she was present when the
will was signed because what matters here is not the photographer but the photograph
taken which clearly portrays Matilde Orobia herself, her co-witnesses Celso Gimpaya. "
Further, the respondent Court correctly held: "The trial court gave undue importance to
the picture takings, jumping therefrom to the conclusion that the will was improperly
executed. The evidence however, heavily points to only one occasion of the execution
of the will on April 15, 1961 which was witnessed by Matilde Orobia, Celso Gimpaya
and Maria Gimpaya. These witnesses were quite emphatic and positive when they
spoke of this occasion. Hence, their Identification of some photographs wherein they all
appeared along with Isabel Gabriel and Atty. Paraiso was superfluous."
Continuing, the respondent Court declared: "It is true that the second picture-taking was
disclosed at the cross examination of Celso Gimpaya. But this was explained by Atty.
Paraiso as a reenactment of the first incident upon the insistence of Isabel Gabriel.
Such reenactment where Matilde Orobia was admittedly no longer present was wholly
unnecessary if not pointless. What was important was that the will was duly executed
and witnessed on the first occasion on April 15, 1961 , " and We agree with the Court's
rationalization in conformity with logic, law and jurisprudence which do not require
picture-taking as one of the legal requisites for the execution or probate of a will.
Petitioner points to alleged grave contradictions, evasions and misrepresentations of
witnesses in their respective testimonies before the trial court. On the other hand, the
respondent Court of Appeals held that said contradictions, evasions and
misrepresentations had been explained away. Such discrepancies as in the description
of the typewriter used by Atty. Paraiso which he described as "elite" which to him meant
big letters which are of the type in which the will was typewritten but which was
Identified by witness Jolly Bugarin of the N.B.I. as pica the mistake in mentioning the
name of the photographer by Matilde Orobia to be Cesar Mendoza when actually it was
Benjamin Cifra, Jr. these are indeed unimportant details which could have been
affected by the lapse of time and the treachery of human memory such that by
themselves would not alter the probative value of their testimonies on the true
execution of the will, (Pascual vs. dela Cruz, 28 SCRA 421, 424) for it cannot be
expected that the testimony of every person win be Identical and coinciding with each

other with regard to details of an incident and that witnesses are not expected to
remember all details. Human experience teach us "that contradictions of witnesses
generally occur in the details of certain incidents, after a long series of questionings,
and far from being an evidence of falsehood constitute a demonstration of good faith. In
as much as not all those who witness an incident are impressed in like manner, it is but
natural that in relating their impressions, they should not agree in the minor details;
hence the contradictions in their testimony." (Lopez vs. Liboro, 81 Phil. 429).
It is urged of Us by the petitioner that the findings of the trial court should not have been
disturbed by the respondent appellate court because the trial court was in a better
position to weigh and evaluate the evidence presented in the course of the trial. As a
general rule, petitioner is correct but it is subject to well-established exceptions. The
right of the Court of Appeals to review, alter and reverse the findings of the trial court
where the appellate court, in reviewing the evidence has found that facts and
circumstances of weight and influence have been ignored and overlooked and the
significance of which have been misinterpreted by the trial court, cannot be disputed.
Findings of facts made by trial courts particularly when they are based on conflicting
evidence whose evaluation hinges on questions of credibility of contending witnesses
hes peculiarly within the province of trial courts and generally, the appellate court
should not interfere with the same. In the instant case, however, the Court of Appeals
found that the trial court had overlooked and misinterpreted the facts and
circumstances established in the record. Whereas the appellate court said that "Nothing
in the record supports the trial court's unbelief that Isabel Gabriel dictated her will
without any note or document to Atty. Paraiso;" that the trial court's conclusion that
Matilde Orobia could not have witnessed anybody signing the alleged will or that she
could not have witnessed Celso Gimpaya and Maria Gimpaya sign the same or that
she witnessed only the deceased signing it, is a conclusion based not on facts but on
inferences; that the trial court gave undue importance to the picture-takings, jumping
therefrom to the conclusion that the will was improperly executed and that there is
nothing in the entire record to support the conclusion of the court a quo that the will
signing occasion was a mere coincidence and that Isabel Gabriel made an appointment
only with Matilde Orobia to witness the signing of her will, then it becomes the duty of
the appellate court to reverse findings of fact of the trial court in the exercise of its
appellate jurisdiction over the lower courts.
Still the petitioner insists that the case at bar is an exception to the rule that the
judgment of the Court of Appeals is conclusive as to the facts and cannot be reviewed
by the Supreme Court. Again We agree with the petitioner that among the exceptions
are: (1) when the conclusion is a finding grounded entirely on speculations, surmises or

conjectures; (2) when the inference is manifestly mistaken, absurd or impossible; (3)
when there is a grave abuse of discretion; (4) when the presence of each other as
required by law. " Specifically, We affirm that on April 15, 1961 the testatrix Isabel
Gabriel, together with Matilde Orobia, Celso Gimpaya and his wife Maria Gimpaya, and
a photographer proceeded in a car to the office of Atty. Cipriano Paraiso at the Bank of
P.I. Building, Manila in the morning of that day; that on the way, Isabel Gabriel obtained
a medical certificate from one Dr. Chikiamko which she gave to Atty. Paraiso upon
arriving at the latter's office and told the lawyer that she wanted her will to be made;
that Atty. Paraiso asked Isabel Gabriel to dictate what she wanted to be written in the
will and the attorney wrote down the dictation of Isabel Gabriel in Tagalog, a language
known to and spoken by her; that Atty. Paraiso read back to her what he wrote as
dictated and she affirmed their correctness; the lawyer then typed the will and after
finishing the document, he read it to her and she told him that it was alright; that
thereafter, Isabel Gabriel signed her name at the end of the will in the presence of the
three witnesses Matilde Orobia, Celso Gimpaya and Maria Gimpaya and also at the
left-hand margin of each and every page of the document in the presence also of the
said three witnesses; that thereafter Matilde Orobia attested the will by signing her
name at the end of the attestation clause and at the left-hand margin of pages 1, 2, 3
and 5 of the document in the presence of Isabel Gabriel and the other two witnesses,
Celso Gimpaya and Maria Gimpaya; then, Celso Gimpaya signed also the will at the
bottom of the attestation clause and at the left-hand margin of the other pages of the
document in the presence of Isabel Gabriel, Matilde Orobia and Maria Gimpaya; that
Maria Gimpaya followed suit, signing her name at the foot of the attestation clause and
at the left-hand margin of every page in the presence of Isabel Gabriel, Matilde Orobia
and Celso Gimpaya; that thereafter, Atty. Paraiso notarized the will as Page No. 94,
Book No. IV, Series of 1961, in his Notarial Register. On the occasion of the execution
and attestation of the will, a photographer took pictures, one Exhibit "G", depicting
Matilde Orobia, the testatrix Isabel Gabriel, Celso Gimpaya, Maria Gimpaya and Atty.
Paraiso, taken on said occasion of the signing of the will, and another, Exhibit "H",
showing Matilde Orobia signing testimony that he had earlier advised Isabel Gabriel to
bring with her at least the Mayor and a Councilor of Navotas, Rizal to be her witnesses
for he did not know beforehand the Identities of the three attesting witnesses until the
latter showed up at his law office with Isabel Gabriel on April 15, 1961. Atty. Paraiso's
claim which was not controverted that he wrote down in his own hand the date
appearing on page 5 of Exhibit "F" dissipates any lingering doubt that he prepared and
ratified the will on the date in question."
It is also a factual finding of the Court of Appeals in holding that it was credible that
Isabel Gabriel could have dictated the will, Exhibit "F", without any note or document to

Atty. Paraiso as against the contention of petitioner that it was incredible. This ruling of
the respondent court is fully supported by the evidence on record as stated in the
decision under review, thus: "Nothing in the record supports the trial court's unbelief
that Isabel Gabriel dictated her will without any note or document to Atty. Paraiso. On
the contrary, all the three attesting witnesses uniformly testified that Isabel Gabriel
dictated her will to Atty. Paraiso and that other than the piece of paper that she handed
to said lawyer she had no note or document. This fact jibes with the evidence which
the trial court itself believed was unshaken that Isabel Gabriel was of sound
disposing memory when she executed her will.
Exhibit "F" reveals only seven (7) dispositions which are not complicated but quite
simple. The first was Isabel Gabriel's wish to be interred according to Catholic rites the
second was a general directive to pay her debts if any; the third provided for P1,000.00
for her sister Praxides Gabriel Vda. de Santiago and P2,000.00 for her brother
Santiago Gabriel; the fourth was a listing of her 13 nephews and nieces including
oppositor-appellee Rizalina Gabriel and the amount for each legatee the fifth was the
institution of the petitioner-appellant, Lutgarda Santiago as the principal heir mentioning
in general terms seven (7) types of properties; the sixth disposed of the remainder of
her estate which she willed in favor of appellant Lutgarda Santiago but prohibiting the
sale of such properties to anyone except in extreme situations in which judgment is
based on a misapprehension of facts; (5) when the findings of fact are conflicting, (6)
when the Court of Appeals, in making its findings, went beyond the issues of the case
and the same is contrary to the admissions of both appellant and appellee. (Roque vs.
Buan, et al., G.R. No. L-22459, Oct. 31, 1967; Ramos vs. Pepsi Cola Bottling Co., G.R.
No. L-22533, Feb. 9, 1967; Hilarion Jr. vs. City of Manila, G.R. No. L-19570; Sept. 14,
1967).
Petitioner's insistence is without merit. We hold that the case at bar does not fall within
any of the exceptions enumerated above. We likewise hold that the findings of fact of
the respondent appellate court are fully supported by the evidence on record. The
conclusions are fully sustained by substantial evidence. We find no abuse of discretion
and We discern no misapprehension of facts. The respondent Court's findings of fact
are not conflicting. Hence, the well-established rule that the decision of the Court of
Appeals and its findings of fact are binding and conclusive and should not be disturbed
by this Tribunal and it must be applied in the case at bar in its full force and effect,
without qualification or reservation. The above holding simply synthesize the resolutions
we have heretofore made in respect ' to petitioner's previous assignments of error and
to which We have disagreed and, therefore, rejected.

The last assignments of error of petitioner must necessarily be rejected by Us as We


find the respondent Court acted properly and correctly and has not departed from the
accepted and usual course of judicial proceedings as to call for the exercise of the
power of supervision by the Supreme Court, and as We find that the Court of Appeals
did not err in reversing the decision of the trial court and admitting to probate Exhibit
"F", the last will and testament of the deceased Isabel Gabriel.
We rule that the respondent Court's factual findings upon its summation and evaluation
of the evidence on record is unassailable that: "From the welter of evidence presented,
we are convinced that the will in question was executed on April 15, 1961 in the
presence of Matilde Orobia, Celso Gimpaya and Maria Gimpaya signing and witnessing
the same in the the will on a table with Isabel Gabriel, Celso Gimpaya and Maria
Gimpaya sitting around the table. Atty. Paraiso, after finishing the notarial act, then
delivered the original to Isabel Gabriel and retained the other copies for his file and
notarial register. A few days following the signing of the will, Isabel Gabriel, Celso
Gimpaya and another photographer arrived at the office of Atty. Paraiso and told the
lawyer that she wanted another picture taken because the first picture did not turn out
good. The lawyer told her that this cannot be done because the will was already signed
but Isabel Gabriel insisted that a picture be taken, so a simulated signing was
performed during which incident Matilde Orobia was not present.
Petitioner's exacerbation centers on the supposed incredibility of the testimonies of the
witnesses for the proponent of the will, their alleged evasions, inconsistencies and
contradictions. But in the case at bar, the three instrumental witnesses who constitute
the best evidence of the will making have testified in favor of the probate of the will. So
has the lawyer who prepared it, one learned in the law and long in the practice thereof,
who thereafter notarized it. All of them are disinterested witnesses who stand to receive
no benefit from the testament. The signatures of the witnesses and the testatrix have
been identified on the will and there is no claim whatsoever and by anyone, much less
the petitioner, that they were not genuine. In the last and final analysis, the herein
conflict is factual and we go back to the rule that the Supreme Court cannot review and
revise the findings of facts of the respondent Court of Appeals.
WHEREFORE, IN VIEW OF THE FOREGOING, the judgment appealed from is hereby
AFFIRMED, with costs against the petitioner.
SO ORDERED.

[G.R. No. L-8774. November 26, 1956.]


In the matter of the testate estate of the deceased JUANA JUAN VDA. DE MOLO.
EMILIANA MOLO-PECKSON and PILAR PEREZ-NABLE, Petitioners-Appellees,
vs. ENRIQUE TANCHUCO, FAUSTINO GOMEZ, ET AL., Oppositors-Appellants.
DECISION
MONTEMAYOR, J.:
Mariano Molo and Juana Juan was a couple possessed of much worldly wealth, but
unfortunately, not blessed with children. To fill the void in their marital life, they took into
their home and custody two baby girls, raising them from infancy, treating them as their
own daughters, sending them to school, and later to the best and exclusive centers of
higher learning, until they both graduated, one in pharmacy, and the other in law. These
two fortunate girls, now grown up women and married, are Emiliana Perez-MoloPeckson, a niece of Juana, and Pilar Perez-Nable a half sister of Emiliana.
Mariano Molo died in January, 1941, and by will bequeathed all his estate to his wife.
Juana, his widow, died on May 28, 1950, leaving no forced heirs but only collateral,
children and grandchildren of her sisters. She left considerable property worth around a
million pesos or more, and to dispose of the same, she was supposed to have
executed on May 11, 1948, about two years before her death, a document purporting to
be her last will and testament, wherein she bequeathed the bulk of her property to her
two foster children, Emiliana and Pilar. These two foster daughters, as Petitioners,
presented the document for probate in the Court of First Instance of Rizal. The other
relatives, such as Enrique Tanchuco, only son of Juanas deceased sister Modesta, and
his two children, Ester, and Gloria, both surnamed Tanchuco, and Faustino Gomez and
Fortunata Gomez, the only surviving grandchildren of another deceased sister, named
Francisca, filed opposition to the probate of the will on the ground that the instrument in
question was not the last will and testament of Juana; chan roblesvirtualawlibrarythat
the same was not executed and attested in accordance with law; chan
roblesvirtualawlibrarythat the said supposed will was secured through undue pressure
and influence on the part of the beneficiaries therein; chan roblesvirtualawlibrarythat the
signature of the testatrix was secured by fraud and that she did not intend the
instrument to be her last will; chan roblesvirtualawlibraryand that at the time the
instrument was executed, the testatrix Juana was not of sound and disposing mind.
Because of the value of the property involved, as well as the fact that the bulk of said
property was being left to Emiliana and Pilar, ignoring and practically disinheriting the
other relatives whose blood ties with the testatrix were just as close, if not closer, the
will, marked Exhibit A at the hearing, was hotly contested and considerable evidence,
oral and documentary, was introduced by both parties. After hearing, Judge Bienvenido
A. Tan, presiding over the trial court, in a well considered decision declared the
document Exhibit A to be the last will and testament of Juana Juan, and admitted the

same for probate; chan roblesvirtualawlibraryand following the provisions of the will, he
appointed Emiliana and Pilar executrices without bond. Failing to obtain a
reconsideration of this decision, the Oppositors appealed to the Court of Appeals about
the beginning of the year 1951.
Ordinarily, because of the value of the property involved in the will, which was many
times more than P50,000, the appeal should have been brought directly to this Tribunal.
However, shortly, after the execution of the instrument admitted to probate as a will, the
testatrix executed a document purporting to be a deed of donation inter vivos, donating
the great bulk of her entire property, with the exception of about P16,000 worth, to the
same beneficiaries in the will, namely, Emiliana and Pilar. If this deed of donation is
valid, then the will disposes of property valued only at about P16,000; chan
roblesvirtualawlibraryhence, the appeal to the Court of Appeals instead of the Supreme
Court. In justice to the Oppositors, it should be stated that, at the same time that they
opposed the probate of the will in the probate court, they also expressed their intention
to contest the validity of the allege donation inter vivos, either in the administration
proceedings or in a proper separate case. The appeal, for one reason or another,
remained in the Court of Appeals for sometime, and only by its resolution of July 7,
1954, was the case certified to us on the ground that, inasmuch as the validity of the
supposed donation inter vivos was being impugned and repudiated by
the Oppositors of the will, and inasmuch as the will itself covered property valued well
in excess of P50,000, the appeal should be determined by the Supreme Court.
We have carefully gone over the evidence of the record, and we are convinced that the
great preponderance thereof is in favor of the probate of the will. Not only this, but we
realize that the credibility of witnesses is very much involved in the determination of this
case, the testimony of those for the Petitioners being diametrically opposed to and
utterly conflicting with that of the witnesses for the Oppositors. His Honor, the trial judge
had the opportunity and was in a position to gauge said credibility and he evidently
found the witnesses for the Petitioners more entitled to credence, and their testimony
more reasonable. We find no reason for disturbing said finding of the probate court. We
quote with approval a portion of the decision of Judge Tan, reading as
follows:chanroblesvirtuallawlibrary
From the evidence presented in this case, both oral and documentary, it was proved to
the full satisfaction of this Court that the deceased freely and voluntarily executed
Exhibit A, her last will and testament, in the presence of her three attesting witnesses
that at the time of the execution of the said will, the deceased was of sound mind and in
good health and was fully conscious of all her acts as may be seen in Exhibits D, D1, D-2, D-3, and D-4, and also as was proven by the testimony of the two attesting
witnesses,
Petrona
P.
Navarro
and
Dr.
Cleofas
Canicosa; chan
roblesvirtualawlibrarythat said will was signed in the presence of the three attesting
witnesses, who, likewise, signed in the presence of the testatrix and in the presence of
each other; chan roblesvirtualawlibrarythat after the execution of said will or after the

signing of the same, the deceased Juana Juan Vda. de Molo took it with her and kept it
in her possession and after her death, the said will was presented in court for probate.
While the written opposition to the probate of said will consists of a litany of supposed
abuses, force and undue influence exercised on the testatrix, yet the evidence shows
that these supposed abuses, force and undue influence consist only of failure on the
part of the deceased to invite theOppositors in all the parties held in her house through
the alleged influence of Mrs. Nable, of paying more attention, care, and extending more
kindness to the Petitioners than to theOppositors in spite of the close blood relationship
existing between the testatrix and theOppositors. The Oppositors also tried to prove the
existence of another will which, according to them, was read to the Oppositor Enrique
Tanchuco three days before the departure of the testatrix for the United States, though
no evidence whatsoever was presented as to what happened to the supposed will,
where it is now, in whose hands it is, or in whose possession it could be found.
The Oppositors also tried to prove that during the illness of the testatrix in 1948 they
were unable to visit her because of the influence of Emiliana Molo-Peckson, who told
them that they could not visit the testatrix because of the advice of the doctor. This
testimony of theOppositors was satisfactorily contradicted by the testimony of Mrs.
Emiliana Molo-Peckson who denied that the testatrix was sick in the year 1948 and by
means of photographs which show that during the said period of time, which
the Oppositors alleged to be the date when Mrs. Juana Juan Vda. de Molo was sick,
the latter attended several affairs, such as sponsoring the reconstruction of the Antipolo
Church, attending a party given in the house of Gen. Aguinaldo in Kawit, Cavite, and
other social gatherings.
Neither do we find anything unusual or extraordinary in the testatrix giving practically all
her property to her foster daughters, to the exclusion of her other relatives. The two
beneficiaries, as already stated, were taken in and raised by her and her husband,
Mariano, when they were mere babies. Naturally, they became very much attached to
and came to love said two children, specially since they had none of their own. They
sent them to good, even expensive schools like the Santa Teresa, Santa Escolastica,
and the University of the Philippines, and otherwise lavished their affection and their
wealth on their two protegees. Little wonder then that Juana in making her will made
Emiliana and Pilar practically her exclusive beneficiaries, specially since, So we
understand, when these two girls had grown up to womanhood, and been highly
educated, they helped their foster parents in the administration of their extensive
properties, and later took good, kind, and tender care of them in their old age. We
repeat that it was neither unusual nor extraordinary that the testatrix, with no forced
heirs, should have made her two foster daughters, the beneficiaries in her will, to the
exclusion of her blood relatives. Said this Court in a similar case Pecson vs.
Coronel, 45 Phil. 220:chanroblesvirtuallawlibrary
The Appellants emphasize the fact that family ties in this country are very strongly knit
and that the exclusion of relatives from ones estate is an exceptional case. It is true

that the ties of relationship in the Philippines are very strong, but we understand that
cases of preterition of relatives from the inheritance are not rare. The liberty to dispose
of ones estate by will when there are no forced heirs is rendered sacred by the Civil
Code in force in the Philippines since 1889. It is so provided in the first paragraph of
article 763 in the following terms:chanroblesvirtuallawlibrary
Any person who has no forced heirs may dispose by will of all his property or any part
of it in favor of any person qualified to acquire it.
Even ignoring the precedents of this legal precept, the Code embodying it has been in
force in the Philippines for more than a quarter of a century, and for this reason it is not
tenable to say that the exercise of the liberty thereby granted is necessarily exceptional,
where it is not shown that the inhabitants of this country whose customs must have
been taken into consideration by the legislator in adopting this legal precept, are averse
to such a liberty.
Oppositors-Appellants in their printed memorandum contend that under Section 618 of
Act 190, the Old Code of Civil Procedure, which requires that a will should be attested
or subscribed by three or more credible witnesses, two of the attesting witnesses to the
will in question, namely, Miss Navarro and Miss Canicosa, who were employed as
pharmacist and salesgirl, respectively, in the drugstore of Pilar Perez-Nable, one of
beneficiaries in the will, may not be considered credible witnesses for the reason that
as such employees, they would naturally testify in favor of their employer. We find the
contention untenable. Section 620 of the same Code of Civil Procedure provides that
any person of sound mind, and of the age of eighteen years or more, and not blind,
deaf, or dumb and able to read and write, may be a witness to the execution of a will.
This same provision is reproduced in our New Civil Code of 1950, under Art. 820. The
relation of employer and employee, or being a relative to the beneficiary in a will, does
not disqualify one to be a witness to a will. The main qualification of a witness in the
attestation of wills, if other qualifications as to age, mental capacity and literacy are
present, is that said witness must be credible, that is to say, his testimony may be
entitled to credence. There is a long line of authorities on this point, a few of which we
may cite:chanroblesvirtuallawlibrary
A credible witness is one who is not disqualified to testify by mental incapacity, crime,
or other cause. Historical Soc. of Dauphin County vs. Kelker, 74 A. 619, 226 Pa. 16,
134 Am. St. Rep. 1010. (Words and Phrases, Vol. 10, p. 340).
As construed by the common law, a credible witness to a will means a competent
witness. Appeal of Clark, 95 A. 517, 114 Me. 105, Ann. Cas. 1917A, 837. (Ibid. p. 341).
Expression credible witness in relation to attestation of wills means competent
witness; chan roblesvirtualawlibrarythat is, one competent under the law to testify to
fact of execution of will. Vernons Ann. Civ. St. art. 8283. Moos vs. First State Bank of
Uvalde, Tex. Civ. App. 60 S. W. 2d 888, 889. (Ibid. p. 842)

The term credible, used in the statute of wills requiring that a will shall be attested by
two credible witnesses, means competent; chan roblesvirtualawlibrarywitnesses who,
at the time of attesting the will, are legally competent to testify, in a court of justice, to
the facts attested by subscribing the will, the competency being determined as of the
date of the execution of the will and not of the time it is offered for probate. Smith vs.
Goodell, 101 N.E. 255, 256, 258 Ill. 145. (Ibid.)
Credible witnesses, as used in the statute relating to wills, means competent
witnesses that is, such persons as are not legally disqualified from testifying in
courts of justice, by reason of mental incapacity, interest, or the commission of crimes,
or other cause excluding them from testifying generally, or rendering them incompetent
in respect of the particular subject matter or in the particular suit. Hill vs. Chicago Title &
Trust Co., 152 N. E. 545, 546, 322 III. 42. (Ibid. p. 343)
This Tribunal itself held in the case of Vda. de Roxas vs. Roxas, 48 Off. Gaz., 2177,
that the law does not bar relatives, either of the testator or of the heirs or legatees, from
acting as attesting witnesses to a will.
In view of the foregoing, finding no reversible error in the decision appealed from the
same is hereby affirmed. No costs.

G.R. Nos. 83843-44

April 5, 1990

IN THE MATTER OF THE PETITION TO APPROVE THE WILL OF MELECIO


LABRADOR. SAGRADO LABRADOR (Deceased), substituted by ROSITA
LABRADOR, ENRICA LABRADOR, and CRISTOBAL LABRADOR, petitionersappellants, vs. COURT OF APPEALS, 1 GAUDENCIO LABRADOR, and JESUS
LABRADOR, respondents-appellees.
Benjamin C. Santos Law Offices for petitioners.
Rodrigo V. Fontelera for private respondents.

PARAS, J.:
The sole issue in this case is whether or not the alleged holographic will of one Melecio
Labrador is dated, as provided for in Article 8102 of the New Civil Code.
The antecedent and relevant facts are as follows: On June 10, 1972, Melecio Labrador
died in the Municipality of Iba, province of Zambales, where he was residing, leaving
behind a parcel of land designated as Lot No. 1916 under Original Certificate of Title
No. P-1652, and the following heirs, namely: Sagrado, Enrica, Cristobal, Jesus,
Gaudencio, Josefina, Juliana, Hilaria and Jovita, all surnamed Labrador, and a
holographic will.
On July 28, 1975, Sagrado Labrador (now deceased but substituted by his heirs),
Enrica Labrador and Cristobal Labrador, filed in the court a quo a petition for the
probate docketed as Special Proceeding No. 922-I of the alleged holographic will of the
late Melecio Labrador.
Subsequently, on September 30, 1975, Jesus Labrador (now deceased but substituted
by his heirs), and Gaudencio Labrador filed an opposition to the petition on the ground
that the will has been extinguished or revoked by implication of law, alleging therein that
on September 30, 1971, that is, before Melecio's death, for the consideration of Six
Thousand (P6,000) Pesos, testator Melecio executed a Deed of Absolute Sale, selling,
transferring and conveying in favor of oppositors Jesus and Gaudencio Lot No. 1916
and that as a matter of fact, O.C.T. No. P-1652 had been cancelled by T.C.T. No. T21178. Earlier however, in 1973, Jesus Labrador sold said parcel of land to Navat for
only Five Thousand (P5,000) Pesos. (Rollo, p. 37)
Sagrado thereupon filed, on November 28, 1975, against his brothers, Gaudencio and
Jesus, for the annulment of said purported Deed of Absolute Sale over a parcel of land
which Sagrado allegedly had already acquired by devise from their father Melecio
Labrador under a holographic will executed on March 17, 1968, the complaint for
annulment docketed as Civil Case No. 934-I, being premised on the fact that the
aforesaid Deed of Absolute Sale is fictitious.

After both parties had rested and submitted their respective evidence, the trial court
rendered a joint decision dated February 28, 1985, allowing the probate of the
holographic will and declaring null and void the Deed of Absolute sale. The court a
quo had also directed the respondents (the defendants in Civil Case No. 934-I) to
reimburse to the petitioners the sum of P5,000.00 representing the redemption price for
the property paid by the plaintiff-petitioner Sagrado with legal interest thereon from
December 20, 1976, when it was paid to vendee a retro.
Respondents appealed the joint decision to the Court of Appeals, which on March 10,
1988 modified said joint decision of the court a quo by denying the allowance of the
probate of the will for being undated and reversing the order of reimbursement.
Petitioners' Motion for Reconsideration of the aforesaid decision was denied by the
Court of Appeals, in the resolution of June 13, 1988. Hence, this petition.
Petitioners now assign the following errors committed by respondent court, to wit:
I
THE COURT OF APPEALS ERRED IN NOT ALLOWING AND APPROVING
THE PROBATE OF THE HOLOGRAPHIC WILL OF THE TESTATOR MELECIO
LABRADOR; and
II
THE COURT OF APPEALS ERRED IN FINDING THAT THE ORDER OF THE
LOWER COURT DIRECTING THE REIMBURSEMENT OF THE FIVE
THOUSAND PESOS REPRESENTING THE REDEMPTION PRICE WAS
ERRONEOUS.
The alleged undated holographic will written in Ilocano translated into English, is quoted
as follows:
ENGLISH INTERPRETATION OF THE WILL OF THE
LATE MELECIO LABRADOR WRITTEN IN ILOCANO
BY ATTY. FIDENCIO L. FERNANDEZ
I First Page
This is also where it appears in writing of the place which is assigned and
shared or the partition in favor of SAGRADO LABRADOR which is the fishpond
located and known place as Tagale.
And this place that is given as the share to him, there is a measurement of more
or less one hectare, and the boundary at the South is the property and
assignment share of ENRICA LABRADOR, also their sister, and the boundary in
the West is the sea, known as the SEA as it is, and the boundary on the NORTH
is assignment belonging to CRISTOBAL LABRADOR, who likewise is also their
brother. That because it is now the time for me being now ninety three (93)
years, then I feel it is the right time for me to partition the fishponds which were

and had been bought or acquired by us, meaning with their two mothers, hence
there shall be no differences among themselves, those among brothers and
sisters, for it is I myself their father who am making the apportionment and
delivering to each and everyone of them the said portion and assignment so
that there shall not be any cause of troubles or differences among the brothers
and sisters.
II Second Page
And this is the day in which we agreed that we are making the partitioning and
assigning the respective assignment of the said fishpond, and this being in the
month of March, 17th day, in the year 1968, and this decision and or instruction
of mine is the matter to be followed. And the one who made this writing is no
other than MELECIO LABRADOR, their father.
Now, this is the final disposition that I am making in writing and it is this that
should be followed and complied with in order that any differences or troubles
may be forestalled and nothing will happen along these troubles among my
children, and that they will be in good relations among themselves, brothers and
sisters;
And those improvements and fruits of the land; mangoes, bamboos and all
coconut trees and all others like the other kind of bamboo by name of Bayog, it
is their right to get if they so need, in order that there shall be nothing that
anyone of them shall complain against the other, and against anyone of the
brothers and sisters.
III THIRD PAGE
And that referring to the other places of property, where the said property is
located, the same being the fruits of our earnings of the two mothers of my
children, there shall be equal portion of each share among themselves, and or
to be benefitted with all those property, which property we have been able to
acquire.
That in order that there shall be basis of the truth of this writing (WILL) which I
am here hereof manifesting of the truth and of the fruits of our labor which their
two mothers, I am signing my signature below hereof, and that this is what
should be complied with, by all the brothers and sisters, the children of their two
mothers JULIANA QUINTERO PILARISA and CASIANA AQUINO
VILLANUEVA Your father who made this writing (WILL), and he is, MELECIO
LABRADOR y RALUTIN (p. 46, Rollo)
The petition, which principally alleges that the holographic will is really dated, although
the date is not in its usual place, is impressed with merit.

The will has been dated in the hand of the testator himself in perfect compliance with
Article 810.1wphi1 It is worthy of note to quote the first paragraph of the second page
of the holographic will, viz:
And this is the day in which we agreed that we are making the partitioning and
assigning the respective assignment of the said fishpond, and this being in the
month of March, 17th day, in the year 1968, and this decision and or instruction
of mine is the matter to be followed. And the one who made this writing is no
other than MELECIO LABRADOR, their father. (emphasis supplied) (p.
46, Rollo)
The law does not specify a particular location where the date should be placed in the
will. The only requirements are that the date be in the will itself and executed in the
hand of the testator. These requirements are present in the subject will.
Respondents claim that the date 17 March 1968 in the will was when the testator and
his beneficiaries entered into an agreement among themselves about "the partitioning
and assigning the respective assignments of the said fishpond," and was not the date
of execution of the holographic will; hence, the will is more of an "agreement" between
the testator and the beneficiaries thereof to the prejudice of other compulsory heirs like
the respondents. This was thus a failure to comply with Article 783 which defines a will
as "an act whereby a person is permitted, with the formalities prescribed by law, to
control to a certain degree the disposition of his estate, to take effect after his death."
Respondents are in error. The intention to show 17 March 1968 as the date of the
execution of the will is plain from the tenor of the succeeding words of the paragraph.
As aptly put by petitioner, the will was not an agreement but a unilateral act of Melecio
Labrador who plainly knew that what he was executing was a will. The act of
partitioning and the declaration that such partitioning as the testator's instruction or
decision to be followed reveal that Melecio Labrador was fully aware of the nature of
the estate property to be disposed of and of the character of the testamentary act as a
means to control the disposition of his estate.
Anent the second issue of finding the reimbursement of the P5,000 representing the
redemption price as erroneous, respondent court's conclusion is incorrect. When
private respondents sold the property (fishpond) with right to repurchase to Navat for
P5,000, they were actually selling property belonging to another and which they had no
authority to sell, rendering such sale null and void. Petitioners, thus "redeemed" the
property from Navat for P5,000, to immediately regain possession of the property for its
disposition in accordance with the will. Petitioners therefore deserve to be reimbursed
the P5,000.
PREMISES CONSIDERED, the decision of the Court of Appeals dated March 10, 1988
is hereby REVERSED. The holographic will of Melecio Labrador is APPROVED and
ALLOWED probate. The private respondents are directed to REIMBURSE the
petitioners the sum of Five Thousand Pesos (P5,000.00).

SO ORDERED.

G.R. No. L-38338 January 28, 1985


IN THE MATTER OF THE INTESTATE ESTATE OF ANDRES G. DE JESUS AND
BIBIANA ROXAS DE JESUS, SIMEON R. ROXAS & PEDRO ROXAS DE
JESUS, petitioners, vs. ANDRES R. DE JESUS, JR., respondent.
Raul S. Sison Law Office for petitioners.
Rafael Dinglasan, Jr. for heir M. Roxas.

is dated "FEB./61 " and states: "This is my win which I want to be respected although it
is not written by a lawyer. ...
The testimony of Simeon R. Roxas was corroborated by the testimonies of Pedro
Roxas de Jesus and Manuel Roxas de Jesus who likewise testified that the letter dated
"FEB./61 " is the holographic Will of their deceased mother, Bibiana R. de Jesus. Both
recognized the handwriting of their mother and positively Identified her signature. They
further testified that their deceased mother understood English, the language in which
the holographic Will is written, and that the date "FEB./61 " was the date when said Will
was executed by their mother.

Ledesma, Guytingco Velasco and Associates for Ledesa and A. R. de Jesus.

GUTIERREZ, JR., J.:


This is a petition for certiorari to set aside the order of respondent Hon. Jose C.
Colayco, Presiding Judge Court of First Instance of Manila, Branch XXI disallowing the
probate of the holographic Will of the deceased Bibiana Roxas de Jesus.
The antecedent facts which led to the filing of this petition are undisputed.

Respondent Luz R. Henson, another compulsory heir filed an "opposition to probate"


assailing the purported holographic Will of Bibiana R. de Jesus because a it was not
executed in accordance with law, (b) it was executed through force, intimidation and/or
under duress, undue influence and improper pressure, and (c) the alleged testatrix
acted by mistake and/or did not intend, nor could have intended the said Will to be her
last Will and testament at the time of its execution.
On August 24, 1973, respondent Judge Jose C. Colayco issued an order allowing the
probate of the holographic Will which he found to have been duly executed in
accordance with law.

After the death of spouses Andres G. de Jesus and Bibiana Roxas de Jesus, Special
Proceeding No. 81503 entitled "In the Matter of the Intestate Estate of Andres G. de
Jesus and Bibiana Roxas de Jesus" was filed by petitioner Simeon R. Roxas, the
brother of the deceased Bibiana Roxas de Jesus.

Respondent Luz Roxas de Jesus filed a motion for reconsideration alleging inter alia
that the alleged holographic Will of the deceased Bibiana R. de Jesus was not dated as
required by Article 810 of the Civil Code. She contends that the law requires that the
Will should contain the day, month and year of its execution and that this should be
strictly complied with.

On March 26, 1973, petitioner Simeon R. Roxas was appointed administrator. After
Letters of Administration had been granted to the petitioner, he delivered to the lower
court a document purporting to be the holographic Will of the deceased Bibiana Roxas
de Jesus. On May 26, 1973, respondent Judge Jose Colayco set the hearing of the
probate of the holographic Win on July 21, 1973.

On December 10, 1973, respondent Judge Colayco reconsidered his earlier order and
disallowed the probate of the holographic Will on the ground that the word "dated" has
generally been held to include the month, day, and year. The dispositive portion of the
order reads:

Petitioner Simeon R. Roxas testified that after his appointment as administrator, he


found a notebook belonging to the deceased Bibiana R. de Jesus and that on pages
21, 22, 23 and 24 thereof, a letter-win addressed to her children and entirely written
and signed in the handwriting of the deceased Bibiana R. de Jesus was found. The will

WHEREFORE, the document purporting to be the holographic Will of


Bibiana Roxas de Jesus, is hereby disallowed for not having been
executed as required by the law. The order of August 24, 1973 is hereby
set aside.

The only issue is whether or not the date "FEB./61 " appearing on the holographic Will
of the deceased Bibiana Roxas de Jesus is a valid compliance with the Article 810 of
the Civil Code which reads:
ART. 810. A person may execute a holographic will which must be
entirely written, dated, and signed by the hand of the testator himself. It
is subject to no other form, and may be made in or out of the Philippines,
and need not be witnessed.
The petitioners contend that while Article 685 of the Spanish Civil Code and Article 688
of the Old Civil Code require the testator to state in his holographic Win the "year,
month, and day of its execution," the present Civil Code omitted the phrase Ao mes y
dia and simply requires that the holographic Will should be dated. The petitioners
submit that the liberal construction of the holographic Will should prevail.
Respondent Luz Henson on the other hand submits that the purported holographic Will
is void for non-compliance with Article 810 of the New Civil Code in that the date must
contain the year, month, and day of its execution. The respondent contends that Article
810 of the Civil Code was patterned after Section 1277 of the California Code and
Section 1588 of the Louisiana Code whose Supreme Courts had consistently ruled that
the required date includes the year, month, and day, and that if any of these is wanting,
the holographic Will is invalid. The respondent further contends that the petitioner
cannot plead liberal construction of Article 810 of the Civil Code because statutes
prescribing the formalities to be observed in the execution of holographic Wills are
strictly construed.
We agree with the petitioner.
This will not be the first time that this Court departs from a strict and literal application of
the statutory requirements regarding the due execution of Wills. We should not overlook
the liberal trend of the Civil Code in the manner of execution of Wills, the purpose of
which, in case of doubt is to prevent intestacy
The underlying and fundamental objectives permeating the provisions of
the law on wigs in this Project consists in the liberalization of the manner
of their execution with the end in view of giving the testator more
freedom in expressing his last wishes, but with sufficien safeguards and
restrictions to prevent the commission of fraud and the exercise of
undue and improper pressure and influence upon the testator.

This objective is in accord with the modem tendency with respect to the
formalities in the execution of wills. (Report of the Code Commission, p.
103)
In Justice Capistrano's concurring opinion in Heirs of Raymundo Castro v. Bustos (27
SCRA 327) he emphasized that:
xxx xxx xxx
... The law has a tender regard for the will of the testator expressed in
his last will and testament on the ground that any disposition made by
the testator is better than that which the law can make. For this reason,
intestate succession is nothing more than a disposition based upon the
presumed will of the decedent.
Thus, the prevailing policy is to require satisfaction of the legal requirements in order to
guard against fraud and bad faith but without undue or unnecessary curtailment of
testamentary privilege Icasiano v. Icasiano, 11 SCRA 422). If a Will has been executed
in substantial compliance with the formalities of the law, and the possibility of bad faith
and fraud in the exercise thereof is obviated, said Win should be admitted to probate
(Rey v. Cartagena 56 Phil. 282). Thus,
xxx xxx xxx
... More than anything else, the facts and circumstances of record are to
be considered in the application of any given rule. If the surrounding
circumstances point to a regular execution of the wilt and the instrument
appears to have been executed substantially in accordance with the
requirements of the law, the inclination should, in the absence of any
suggestion of bad faith, forgery or fraud, lean towards its admission to
probate, although the document may suffer from some imperfection of
language, or other non-essential defect. ... (Leynez v. Leynez 68 Phil.
745).
If the testator, in executing his Will, attempts to comply with all the requisites, although
compliance is not literal, it is sufficient if the objective or purpose sought to be
accomplished by such requisite is actually attained by the form followed by the testator.

The purpose of the solemnities surrounding the execution of Wills has been expounded
by this Court in Abangan v. Abanga 40 Phil. 476, where we ruled that:
The object of the solemnities surrounding the execution of wills is to
close the door against bad faith and fraud, to avoid substitution of wills
and testaments and to guaranty their truth and authenticity. ...
In particular, a complete date is required to provide against such contingencies as that
of two competing Wills executed on the same day, or of a testator becoming insane on
the day on which a Will was executed (Velasco v. Lopez, 1 Phil. 720). There is no such
contingency in this case.
We have carefully reviewed the records of this case and found no evidence of bad faith
and fraud in its execution nor was there any substitution of Wins and Testaments. There
is no question that the holographic Will of the deceased Bibiana Roxas de Jesus was
entirely written, dated, and signed by the testatrix herself and in a language known to
her. There is also no question as to its genuineness and due execution. All the children
of the testatrix agree on the genuineness of the holographic Will of their mother and
that she had the testamentary capacity at the time of the execution of said Will. The
objection interposed by the oppositor-respondent Luz Henson is that the holographic
Will is fatally defective because the date "FEB./61 " appearing on the holographic Will is
not sufficient compliance with Article 810 of the Civil Code. This objection is too
technical to be entertained.
As a general rule, the "date" in a holographic Will should include the day, month, and
year of its execution. However, when as in the case at bar, there is no appearance of
fraud, bad faith, undue influence and pressure and the authenticity of the Will is
established and the only issue is whether or not the date "FEB./61" appearing on the
holographic Will is a valid compliance with Article 810 of the Civil Code, probate of the
holographic Will should be allowed under the principle of substantial compliance.
WHEREFORE, the instant petition is GRANTED. The order appealed from is
REVERSED and SET ASIDE and the order allowing the probate of the holographic Will
of the deceased Bibiana Roxas de Jesus is reinstated.
SO ORDERED.

G.R. No. L-36342

October 8, 1932

In re Will of Francisco Varela Calderon, deceased.


FRANCISCO CARMELO VARELA, petitioner-appellee, vs. MIGUEL VARELA
CALDERON, ET AL., opponents-appellants
Araneta De Joya, Zaragoza and Araneta for appellants.
Eduardo Gutierrez Repide for appellee.

IMPERIAL, J.:
This is an appeal taken by Miguel Varela Calderon, Angel Varela Calderon, Jesus
Varela Calderon, Trinidad Varela Calderon, Paula Varela Calderon, Pilar Varela
Calderon and Maria Varela Calderon from the judgment rendered by the Honorable
Mariano A. Albert, Judge of the Court of First Instance of Manila, ordering the allowance
and probate of the document marked Exhibit B as the last will and testament of the late
Francisco Varela Calderon.
The deceased, a physician by profession, was a Filipino citizen resident of the City of
Manila where he owned real properties assessed at P188,017.81. He traveled abroad
for his health and temporarily resided in Hendaye-Plage, France. Not feeling very well,
but in the full enjoyment of his mental faculties, he decided to make his last will and
testament (Exhibit B), on April 14, 1930, in Paris, France, with the assistance of
attorneys F. de Roussy de Sales, Gething C. Miller and Henri Gadd. Sometime later,
that is on July 15, 1930, he died in the Grand-Hotel de Leysin Sanatorium in
Switzerland.
On September 20, 1930, the herein petitioner-appellee, Francisco Carmelo Varela, filed
a petition in the Court of First Instance of the City of Manila, praying that said will be
admitted to probate. Said petition was opposed by the deceased's brother Jose Miguel,
Angel, Jesus, Trinidad, Paula, Pilar and Maria, surnamed Varela Calderon, although,
later on, the first mentioned opponent withdrew his opposition giving as his reason
therefor that it was out of respect for the testator's wishes because the will was
executed in his own handwriting. The grounds of the opposition are as follows: (a) That
the will sought to be probate was not holographic in character and did not comply with
the requisites prescribed by article 970 of the French Civil Code; (b) that the witnesses
to the will did not possess the qualifications required by article 980 of the French Civil

Code; (c) that for not having complied with the requisites prescribed by the French law,
said will is null and void; (d) that neither has it the character of an open will, not having
been executed in accordance with article 1001 of the French Civil Code; and (e) that
the provisions of article 1007 of the same Code relative to the recording of wills were
not complied with in connection with the will in question.
The appellants assign the following errors alleged to have been committed by the trial
court:
1. The trial court erred in declaring that Exhibit B, purporting to be the last will
and testament of the deceased Francisco Varela Calderon, was a valid
holographic will made and executed, in accordance with the laws of the French
Republic.

including those on which I could have the power of disposition or


designation whatsoever, by equal parts in all properties, instituting them
my residuary legatee.
4. I name for my testamentary executor, without bail, my son,
FRANCISCO CARMELO VARELA, to his default my daughter CARMEN
VARELA to the default of this one my brother MIGUEL VARELA
CALDERON and to the default of my brother, Mr. ANTONIO
GUTIERREZ DEL CAMPO.
In witness whereof I have in this one my act of last wills and testament, written
dated and signed entirely by my hand, applied my signature and my seal at
Paris, this fourteenth of April nineteen hundred thirty.

2. The trial court, therefore, erred in allowing and admitting to probate said
document Exhibit B, as the last will and testament of the deceased Francisco
Varela Calderon.
In view of the nature of both assignments of error and of the arguments advanced in
their support, we shall discuss them jointly.
The will in question, Exhibit B, as translated into English reads as follows:

(Sgd.) FRANCISCO VARELA CALDERON (L. S.)


(Sgd.) HENRI GADD
(Sgd.) TH. PEYROT
Signed, sealed, published and declared by the testator above mentioned,
FRANCISCO VARELA CALDERON, on the 14 April 1930, as being his act of
last wills and his testament in the presence of we, who at his request, in his
presence, and in the presence of one another, have to these presents applied
our signatures as witnesses:

I. FRANCISCO VARELA CALDERON, Doctor in Medicine, bachelor, citizen of


the Philippine Islands and subject of the United States of America, borne and
domiciled at Manila, Philippine Islands, and actually residing temporarily at
Hendaye-Plage, France, I declare that the followings are my act of last wills and
my testament:

(Sgd.) F. DE ROUSSY DE SALES,


39 rue la Boetie, Paris, France.
(Sgd.) GETHING C. MILLER,
39 rue la Boetie, Paris, France.

1. I revoke all testamentary dispositions made by myself before this day.


(Sgd.) HENRI GADD,
39 rue la Boetie, Paris, France.

2. I desire that the inhumation of my body be made without any


ceremony.
Pour traduction certifiee conforme,
3. I give and bequeath to my children FRANCISCO CARMELO VARELA
borne at Madrid on the twenty fourth of November nineteen hundred five
and to CARMEN VARELA borne at Manila, Philippine Islands, on the
fourth of October nineteen hundred seven who live both actually with
me, the totality of my personal properties, real estates and mixed

Manille, le 15 september 1930.


The original will was executed in the French language and had been written, dated and
signed by the testator with his own hand, with the exception of the attestation clause

which appears at the bottom of the document. This fact is proved by the testimony of
the appellee and his other witnesses, including the depositions, and is admitted by the
appellants.
The petition for the allowance and probate of said will is based on the provisions of
article 970 of the French Civil Code which considers as a holographic will that which is
made or executed, dated and signed by the testator in his own handwriting without the
necessity of any other formality, and on section 635 of the Code of Civil Procedure in
force in this jurisdiction which provides that a will made out of the Philippine Islands in
accordance with the laws in force in the country in which it was made and which may
be allowed and admitted to probate therein, may, also be proved, allowed and recorded
in the Philippine Islands in the same manner and with the same effect as if executed in
the latter country. Both provisions of law literally copied from the English text, read as
follows:
(Article 970, French Civil Code)
A holographic will is not valid unless it is entirely written, dated, and signed by
the testator. No other formality is required.
(Article 635, Code of Civil Procedure)
Will made out of the Philippine Islands. A will made out of the Philippine
Islands which might be proved and allowed by the laws of the state or country in
which it was made, may be proved, allowed, and recorded in the Philippine
Islands, and shall have the same effect as if executed according to the laws of
these Islands.
As we have already said, it is an admitted fact that the will was written, dated and
signed by the deceased testator, for which reason, there is no doubt that it had been
made and executed in accordance with article 970 of the French Civil Code were it not
for the attestation clause which appears at the bottom of the document.
The appellants contend that the addition of said of clause has entirely vitiated the will,
because it ceased to be a holographic will, neither does it possess the requisites of a
public or open will in accordance with the French law. The court which originally took
cognizance of the case decided that such circumstance does not invalidate the will. We
concur in said opinion and hold that a clause drawn up in such manner is superfluous
and does not affect in any way the essential requisites prescribed for holographic wills

by the French law, and, consequently, it has not invalidated the will nor deprived it of its
holographic character. In reaching this conclusion, we base our opinion not only on the
clear and conclusive provisions of article 970 of the French Civil Code and on the
decisions of the French Court of Appeals cited in the appelee's brief, but principally on
the fact established in the depositions made by practicing attorneys F. de Roussy de
Sales, Gething C. Miller and Henri Gadd of Paris, France, who emphatically declared
that the will in question did not lose its holographic character by the addition of the
aforementioned attestation clause and that it may be allowed to probate in conformity
with the French laws under which it had been made and executed.
In view of the foregoing, we believe it unnecessary to consider and pass upon the other
grounds invoked by the opposition consisting in the incapacity of the three witnesses
who signed at the end of the attestation clause and the absence of the requisites
prescribed by the French Civil Code for the execution of open wills, inasmuch as the
attorney for the appellee has conclusively proved that the will in question is
holographic, and we have so held and decided.1awphil.net
The order appealed from, being in conformity with the law, is hereby affirmed in toto,
with costs against the appellants. So ordered.

G.R. No. L-40207 September 28, 1984


ROSA K. KALAW, petitioner, vs. HON. JUDGE BENJAMIN RELOVA, Presiding
Judge of the CFI of Batangas, Branch VI, Lipa City, and GREGORIO K.
KALAW, respondents.
Leandro H. Fernandez for petitioner.
Antonio Quintos and Jose M. Yacat for respondents.

MELENCIO-HERRERA, J.:
On September 1, 1971, private respondent GREGORIO K. KALAW, claiming to be the
sole heir of his deceased sister, Natividad K. Kalaw, filed a petition before the Court of
First Instance of Batangas, Branch VI, Lipa City, for the probate of her holographic Will
executed on December 24, 1968.
The holographic Will reads in full as follows:
My Last will and Testament
In the name of God, Amen.
I Natividad K. Kalaw Filipino 63years of age, single, and a resident of Lipa City, being of
sound and disposing mind and memory, do hereby declare thus to be my last will and
testament.
1. It is my will that I'll be burried in the cemetery of the catholic church of Lipa City. In
accordance with the rights of said Church, and that my executrix hereinafter named
provide and erect at the expose of my state a suitable monument to perpetuate my
memory.
xxx xxx xxx
The holographic Will, as first written, named ROSA K. Kalaw, a sister of the testatrix as
her sole heir. Hence, on November 10, 1971, petitioner ROSA K. Kalaw opposed

probate alleging, in substance, that the holographic Will contained alterations,


corrections, and insertions without the proper authentication by the full signature of the
testatrix as required by Article 814 of the Civil Code reading:
Art. 814. In case of any insertion, cancellation, erasure or alteration in a
holographic will the testator must authenticate the same by his full
signature.
ROSA's position was that the holographic Will, as first written, should be given effect
and probated so that she could be the sole heir thereunder.
After trial, respondent Judge denied probate in an Order, dated September 3, 197 3,
reading in part:
The document Exhibit "C" was submitted to the National Bureau of
Investigation for examination. The NBI reported that the handwriting, the
signature, the insertions and/or additions and the initial were made by
one and the same person. Consequently, Exhibit "C" was the
handwriting of the decedent, Natividad K. Kalaw. The only question is
whether the win, Exhibit 'C', should be admitted to probate although the
alterations and/or insertions or additions above-mentioned were not
authenticated by the full signature of the testatrix pursuant to Art. 814 of
the Civil Code. The petitioner contends that the oppositors are estopped
to assert the provision of Art. 814 on the ground that they themselves
agreed thru their counsel to submit the Document to the NBI FOR
EXAMINATIONS. This is untenable. The parties did not agree, nor was it
impliedly understood, that the oppositors would be in estoppel.
The Court finds, therefore, that the provision of Article 814 of the Civil
Code is applicable to Exhibit "C". Finding the insertions, alterations
and/or additions in Exhibit "C" not to be authenticated by the full
signature of the testatrix Natividad K. Kalaw, the Court will deny the
admission to probate of Exhibit "C".
WHEREFORE, the petition to probate Exhibit "C" as the holographic will
of Natividad K. Kalaw is hereby denied.
SO ORDERED.

From that Order, GREGORIO moved for reconsideration arguing that since the
alterations and/or insertions were the testatrix, the denial to probate of her holographic
Will would be contrary to her right of testamentary disposition. Reconsideration was
denied in an Order, dated November 2, 1973, on the ground that "Article 814 of the Civil
Code being , clear and explicit, (it) requires no necessity for interpretation."
From that Order, dated September 3, 1973, denying probate, and the Order dated
November 2, 1973 denying reconsideration, ROSA filed this Petition for Review on
certiorari on the sole legal question of whether or not theoriginal unaltered text after
subsequent alterations and insertions were voided by the Trial Court for lack of
authentication by the full signature of the testatrix, should be probated or not, with her
as sole heir.
Ordinarily, when a number of erasures, corrections, and interlineations made by the
testator in a holographic Will litem not been noted under his signature, ... the Will is not
thereby invalidated as a whole, but at most only as respects the particular words
erased, corrected or interlined.1 Manresa gave an Identical commentary when he said
"la omision de la salvedad no anula el testamento, segun la regla de jurisprudencia
establecida en la sentencia de 4 de Abril de 1895." 2
However, when as in this case, the holographic Will in dispute had only one substantial
provision, which was altered by substituting the original heir with another, but which
alteration did not carry the requisite of full authentication by the full signature of the
testator, the effect must be that the entire Will is voided or revoked for the simple
reason that nothing remains in the Will after that which could remain valid. To state that
the Will as first written should be given efficacy is to disregard the seeming change of
mind of the testatrix. But that change of mind can neither be given effect because she
failed to authenticate it in the manner required by law by affixing her full signature,
The ruling in Velasco, supra, must be held confined to such insertions, cancellations,
erasures or alterations in a holographic Will, which affect only the efficacy of the altered
words themselves but not the essence and validity of the Will itself. As it is, with the
erasures, cancellations and alterations made by the testatrix herein, her real intention
cannot be determined with certitude. As Manresa had stated in his commentary on
Article 688 of the Spanish Civil Code, whence Article 814 of the new Civil Code was
derived:
... No infringe lo dispuesto en este articulo del Codigo (el 688) la
sentencia que no declara la nulidad de un testamento olografo que

contenga palabras tachadas, enmendadas o entre renglones no


salvadas por el testador bajo su firnia segun previene el parrafo tercero
del mismo, porque, en realidad, tal omision solo puede afectar a la
validez o eficacia de tales palabras, y nunca al testamento mismo, ya
por estar esa disposicion en parrafo aparte de aquel que determine las
condiciones necesarias para la validez del testamento olografo, ya
porque, de admitir lo contrario, se Ilegaria al absurdo de que pequefias
enmiendas no salvadas, que en nada afectasen a la parte esencial y
respectiva del testamento, vinieran a anular este, y ya porque el
precepto contenido en dicho parrafo ha de entenderse en perfecta
armonia y congruencia con el art. 26 de la ley del Notariado que declara
nulas las adiciones apostillas entrerrenglonados, raspaduras y tachados
en las escrituras matrices, siempre que no se salven en la forma
prevenida, paro no el documento que las contenga, y con mayor
motivo cuando las palabras enmendadas, tachadas, o
entrerrenglonadas no tengan importancia ni susciten duda alguna
acerca del pensamiento del testador, o constituyan meros accidentes de
ortografia o de purez escrituraria, sin trascendencia alguna(l).
Mas para que sea aplicable la doctrina de excepcion contenida en este
ultimo fallo, es preciso que las tachaduras, enmiendas o
entrerrenglonados sin salvar saan de pala bras que no afecter4 alteren
ni uarien de modo substancial la express voluntad del testador
manifiesta en el documento. Asi lo advierte la sentencia de 29 de
Noviembre de 1916, que declara nulo un testamento olografo por no
estar salvada por el testador la enmienda del guarismo ultimo del ao
en que fue extendido 3(Emphasis ours).
WHEREFORE, this Petition is hereby dismissed and the Decision of respondent Judge,
dated September 3, 1973, is hereby affirmed in toto. No costs.
SO ORDERED.

G.R. No. 106720 September 15, 1994


SPOUSES ROBERTO AND THELMA AJERO, petitioners, vs. THE COURT OF
APPEALS AND CLEMENTE SAND, respondents.
Miguel D. Larida for petitioners.
Montilla Law Office for private respondent.

PUNO, J.:
This is an appeal by certiorari from the Decision of the Court of
Appeals 1 in CA-G.R. CV No. 22840, dated March 30, 1992, the dispositive portion of
which reads;
PREMISES CONSIDERED, the questioned decision of November 19,
1988 of the trial court is hereby REVERSED and SET ASIDE, and the
petition for probate is hereby DISMISSED. No costs.
The earlier Decision was rendered by the RTC of Quezon City, Branch 94, 2 in
Sp. Proc. No. Q-37171, and the instrument submitted for probate is the
holographic will of the late Annie Sand, who died on November 25, 1982.
In the will, decedent named as devisees, the following: petitioners Roberto and Thelma
Ajero, private respondent Clemente Sand, Meriam S. Arong, Leah Sand, Lilia Sand,
Edgar Sand, Fe Sand, Lisa S. Sand, and Dr. Jose Ajero, Sr., and their children.
On January 20, 1983, petitioners instituted Sp. Proc. No. Q-37171, for allowance of
decedent's holographic will. They alleged that at the time of its execution, she was of
sound and disposing mind, not acting under duress, fraud or undue influence, and was
in every respect capacitated to dispose of her estate by will.
Private respondent opposed the petition on the grounds that: neither the testament's
body nor the signature therein was in decedent's handwriting; it contained alterations
and corrections which were not duly signed by decedent; and, the will was procured by
petitioners through improper pressure and undue influence. The petition was likewise

opposed by Dr. Jose Ajero. He contested the disposition in the will of a house and lot
located in Cabadbaran, Agusan Del Norte. He claimed that said property could not be
conveyed by decedent in its entirety, as she was not its sole owner.
Notwithstanding the oppositions, the trial court admitted the decedent's holographic will
to probate. It found, inter alia:
Considering then that the probate proceedings herein must decide only
the question of identity of the will, its due execution and the testamentary
capacity of the testatrix, this probate court finds no reason at all for the
disallowance of the will for its failure to comply with the formalities
prescribed by law nor for lack of testamentary capacity of the testatrix.
For one, no evidence was presented to show that the will in question is
different from the will actually executed by the testatrix. The only
objections raised by the oppositors . . . are that the will was not written in
the handwriting of the testatrix which properly refers to the question of its
due execution, and not to the question of identity of will. No other will
was alleged to have been executed by the testatrix other than the will
herein presented. Hence, in the light of the evidence adduced, the
identity of the will presented for probate must be accepted, i.e., the will
submitted in Court must be deemed to be the will actually executed by
the testatrix.
xxx xxx xxx
While the fact that it was entirely written, dated and signed in the
handwriting of the testatrix has been disputed, the petitioners, however,
have satisfactorily shown in Court that the holographic will in question
was indeed written entirely, dated and signed in the handwriting of the
testatrix. Three (3) witnesses who have convincingly shown knowledge
of the handwriting of the testatrix have been presented and have
explicitly and categorically identified the handwriting with which the
holographic will in question was written to be the genuine handwriting
and signature of the testatrix. Given then the aforesaid evidence, the
requirement of the law that the holographic will be entirely written, dated
and signed in the handwriting of the testatrix has been complied with.

xxx xxx xxx


As to the question of the testamentary capacity of the testratix, (private
respondent) Clemente Sand himself has testified in Court that the
testatrix was completely in her sound mind when he visited her during
her birthday celebration in 1981, at or around which time the holographic
will in question was executed by the testatrix. To be of sound mind, it is
sufficient that the testatrix, at the time of making the will, knew
the value of the estate to be disposed of, the proper object of her bounty,
and thecharacter of the testamentary act . . . The will itself shows that
the testatrix even had detailed knowledge of the nature of her estate.
She even identified the lot number and square meters of the lots she
had conveyed by will. The objects of her bounty were likewise identified
explicitly. And considering that she had even written a nursing book
which contained the law and jurisprudence on will and succession, there
is more than sufficient showing that she knows the character of the
testamentary act.
In this wise, the question of identity of the will, its due execution and the
testamentary capacity of the testatrix has to be resolved in favor of the
allowance of probate of the will submitted herein.
Likewise, no evidence was presented to show sufficient reason for the
disallowance of herein holographic will. While it was alleged that the said
will was procured by undue and improper pressure and influence on the
part of the beneficiary or of some other person, the evidence adduced
have not shown any instance where improper pressure or influence was
exerted on the testatrix. (Private respondent) Clemente Sand has
testified that the testatrix was still alert at the time of the execution of the
will, i.e., at or around the time of her birth anniversary celebration in
1981. It was also established that she is a very intelligent person and
has a mind of her own. Her independence of character and to some
extent, her sense of superiority, which has been testified to in Court, all
show the unlikelihood of her being unduly influenced or improperly
pressured to make the aforesaid will. It must be noted that the undue
influence or improper pressure in question herein only refer to the
making of a will and not as to the specific testamentary provisions
therein which is the proper subject of another proceeding. Hence, under

the circumstances, this Court cannot find convincing reason for the
disallowance of the will herein.
Considering then that it is a well-established doctrine in the law on
succession that in case of doubt, testate succession should be preferred
over intestate succession, and the fact that no convincing grounds were
presented and proven for the disallowance of the holographic will of the
late Annie Sand, the aforesaid will submitted herein must be admitted to
probate. 3 (Citations omitted.)
On appeal, said Decision was reversed, and the petition for probate of decedent's will
was dismissed. The Court of Appeals found that, "the holographic will fails to meet the
requirements for its validity." 4 It held that the decedent did not comply with Articles 813
and 814 of the New Civil Code, which read, as follows:
Art. 813: When a number of dispositions appearing in a holographic will
are signed without being dated, and the last disposition has a signature
and date, such date validates the dispositions preceding it, whatever be
the time of prior dispositions.
Art. 814: In case of insertion, cancellation, erasure or alteration in a
holographic will, the testator must authenticate the same by his full
signature.
It alluded to certain dispositions in the will which were either unsigned and undated, or
signed but not dated. It also found that the erasures, alterations and cancellations
made thereon had not been authenticated by decedent.
Thus, this appeal which is impressed with merit.
Section 9, Rule 76 of the Rules of Court provides that will shall be disallowed in any of
the following cases:
(a) If not executed and attested as required by law;
(b) If the testator was insane, or otherwise mentally incapable to make a
will, at the time of its execution;
(c) If it was executed under duress, or the influence of fear, or threats;

(d) If it was procured by undue and improper pressure and influence, on


the part of the beneficiary, or of some other person for his benefit;
(e) If the signature of the testator was procured by fraud or trick, and he
did not intend that the instrument should be his will at the time of fixing
his signature thereto.
In the same vein, Article 839 of the New Civil Code reads:
Art. 839: The will shall be disallowed in any of the following cases;
(1) If the formalities required by law have not been
complied with;
(2) If the testator was insane, or otherwise mentally
incapable of making a will, at the time of its execution;
(3) If it was executed through force or under duress, or
the influence of fear, or threats;
(4) If it was procured by undue and improper pressure
and influence, on the part of the beneficiary or of some
other person;
(5) If the signature of the testator was procured by fraud;
(6) If the testator acted by mistake or did not intend that
the instrument he signed should be his will at the time of
affixing his signature thereto.
These lists are exclusive; no other grounds can serve to disallow a will. 5 Thus, in a
petition to admit a holographic will to probate, the only issues to be resolved are: (1)
whether the instrument submitted is, indeed, the decedent's last will and testament; (2)
whether said will was executed in accordance with the formalities prescribed by law; (3)
whether the decedent had the necessary testamentary capacity at the time the will was
executed; and, (4) whether the execution of the will and its signing were the voluntary
acts of the decedent. 6

In the case at bench, respondent court held that the holographic will of Anne Sand was
not executed in accordance with the formalities prescribed by law. It held that Articles
813 and 814 of the New Civil Code, ante, were not complied with, hence, it disallowed
the probate of said will. This is erroneous.
We reiterate what we held in Abangan vs. Abangan, 40 Phil. 476, 479 (1919), that:
The object of the solemnities surrounding the execution of wills is to
close the door against bad faith and fraud, to avoid substitution of wills
and testaments and to guaranty their truth and authenticity. Therefore,
the laws on this subject should be interpreted in such a way as to attain
these primordial ends. But, on the other hand, also one must not lose
sight of the fact that it is not the object of the law to restrain and curtail
the exercise of the right to make a will. So when an interpretation
already given assures such ends, any other interpretation whatsoever,
that adds nothing but demands more requisites entirely unnecessary,
useless and frustrative of the testator's last will, must be disregarded.
For purposes of probating non-holographic wills, these formal solemnities include the
subscription, attestation, and acknowledgment requirements under Articles 805 and
806 of the New Civil Code.
In the case of holographic wills, on the other hand, what assures authenticity is the
requirement that they be totally autographic or handwritten by the testator himself, 7 as
provided under Article 810 of the New Civil Code, thus:
A person may execute a holographic will which must be entirely written,
dated, and signed by the hand of the testator himself. It is subject to no
other form, and may be made in or out of the Philippines, and need not
be witnessed. (Emphasis supplied.)
Failure to strictly observe other formalities will not result in the disallowance of a
holographic will that is unquestionably handwritten by the testator.
A reading of Article 813 of the New Civil Code shows that its requirement affects the
validity of the dispositions contained in the holographic will, but not its probate. If the
testator fails to sign and date some of the dispositions, the result is that these
dispositions cannot be effectuated. Such failure, however, does not render the whole
testament void.

Likewise, a holographic will can still be admitted to probate, notwithstanding noncompliance with the provisions of Article 814. In the case of Kalaw vs. Relova 132
SCRA 237 242 (1984), this Court held:
Ordinarily, when a number of erasures, corrections, and interlineations
made by the testator in a holographic Will have not been noted under his
signature, . . . the Will is not thereby invalidated as a whole, but at most
only as respects the particular words erased, corrected or interlined.
Manresa gave an identical commentary when he said "la omission de la
salvedad no anula el testamento, segun la regla de jurisprudencia
establecida en la sentencia de 4 de Abril de 1985." 8 (Citations omitted.)
Thus, unless the unauthenticated alterations, cancellations or insertions were made on
the date of the holographic will or on testator's signature, 9 their presence does not
invalidate the will itself. 10 The lack of authentication will only result in disallowance of
such changes.
It is also proper to note that the requirements of authentication of changes and signing
and dating of dispositions appear in provisions (Articles 813 and 814) separate from
that which provides for the necessary conditions for the validity of the holographic will
(Article 810). The distinction can be traced to Articles 678 and 688 of the Spanish Civil
Code, from which the present provisions covering holographic wills are taken. They
read as follows:
Art. 678: A will is called holographic when the testator writes it himself in
the form and with the requisites required in Article 688.
Art. 688: Holographic wills may be executed only by persons of full age.
In order that the will be valid it must be drawn on stamped paper
corresponding to the year of its execution, written in its entirety by the
testator and signed by him, and must contain a statement of the year,
month and day of its execution.
If it should contain any erased, corrected, or interlined words, the
testator must identify them over his signature.
Foreigners may execute holographic wills in their own language.

This separation and distinction adds support to the interpretation that only the
requirements of Article 810 of the New Civil Code and not those found in Articles 813
and 814 of the same Code are essential to the probate of a holographic will.
The Court of Appeals further held that decedent Annie Sand could not validly dispose of
the house and lot located in Cabadbaran, Agusan del Norte, in its entirety. This is
correct and must be affirmed.
As a general rule, courts in probate proceedings are limited to pass only upon the
extrinsic validity of the will sought to be probated. However, in exceptional instances,
courts are not powerless to do what the situation constrains them to do, and pass upon
certain provisions of the will. 11 In the case at bench, decedent herself indubitably stated
in her holographic will that the Cabadbaran property is in the name of her late father,
John H. Sand (which led oppositor Dr. Jose Ajero to question her conveyance of the
same in its entirety). Thus, as correctly held by respondent court, she cannot validly
dispose of the whole property, which she shares with her father's other heirs.
IN VIEW WHEREOF, the instant petition is GRANTED. The Decision of the Court of
Appeals in CA-G.R. CV No. 22840, dated March 30, 1992, is REVERSED and SET
ASIDE, except with respect to the invalidity of the disposition of the entire house and lot
in Cabadbaran, Agusan del Norte. The Decision of the Regional Trial Court of Quezon
City, Branch 94 in Sp. Proc. No. Q-37171, dated November 19, 1988, admitting to
probate the holographic will of decedent Annie Sand, is hereby REINSTATED, with the
above qualification as regards the Cabadbaran property. No costs.
SO ORDERED.

G.R. No. 123486

August 12, 1999

EUGENIA RAMONAL CODOY, and MANUEL RAMONAL, petitioners, vs.


EVANGELINE R. CALUGAY, JOSEPHINE SALCEDO, and UEFEMIA
PATIGAS, respondents.
PARDO, J.:
Before us is a petition for review on certiorari of the decision of the Court of
Appeals1 and its resolution denying reconsideration, ruling:
Upon the unrebutted testimony of appellant Evangeline Calugay and witness
Matilde Ramonal Binanay, the authenticity of testators holographic will has been
established and the handwriting and signature therein (exhibit S) are hers,
enough to probate said will. Reversal of the judgment appealed from and the
probate of the holographic will in question be called for. The rule is that after
plaintiff has completed presentation of his evidence and the defendant files a
motion for judgment on demurrer to evidence on the ground that upon the facts
and the law plaintiff has shown no right to relief, if the motion is granted and the
order to dismissal is reversed on appeal, the movant loses his right to present
evidence in his behalf (Sec, 1 Rule 35 Revised Rules of Court). Judgment may,
therefore, be rendered for appellant in the instant case.
Wherefore, the order appealed from is REVERSED and judgment rendered
allowing the probate of the holographic will of the testator Matilde Seo Vda. de
Ramonal.2
The facts are as follows:
On April 6, 1990, Evangeline Calugay, Josephine Salcedo and Eufemia Patigas,
devisees and legatees of the holographic will of the deceased Matilde Seo Vda. de
Ramonal, filed with the Regional Trial Court, Misamis Oriental, Branch 18, a petition3 for
probate of the holographic will of the deceased, who died on January 16, 1990.
In the petition, respondents claimed that the deceased Matilde Seo Vda. de Ramonal,
was of sound and disposing mind when she executed the will on August 30, 1978, that
there was no fraud, undue influence, and duress employed in the person of the testator,
and will was written voluntarily.

The assessed value of the decedent's property, including all real and personal property
was about P400,000.00, at the time of her death.4
On June 28, 1990, Eugenia Ramonal Codoy and Manuel Ramonal filed an
opposition5 to the petition for probate, alleging that the holographic will was a forgery
and that the same is even illegible. This gives an impression that a "third hand" of an
interested party other than the "true hand" of Matilde Seo Vda. de Ramonal executed
the holographic will.
Petitioners argued that the repeated dates incorporated or appearing on will after every
disposition is out of the ordinary. If the deceased was the one who executed the will,
and was not forced, the dates and the signature should appear at the bottom after the
dispositions, as regularly done and not after every disposition. And assuming that the
holographic will is in the handwriting of the deceased, it was procured by undue and
improper pressure and influence on the part of the beneficiaries, or through fraud and
trickery.1wphi1.nt
Respondents presented six (6) witnesses and various documentary evidence.
Petitioners instead of presenting their evidence, filed a demurrer6 to evidence, claiming
that respondents failed to establish sufficient factual and legal basis for the probate of
the holographic will of the deceased Matilde Seo Vda. de Ramonal.
On November 26, 1990, the lower Court issued an order, the dispositive portion of
which reads:
WHEREFORE, in view of the foregoing consideration, the Demurrer to
Evidence having being well taken, same is granted, and the petition for probate
of the document (Exhibit "S") on the purported Holographic Will of the late
Matilde Seo Vda. de Ramonal, is denied for insufficiency of evidence and lack
of merits.7
On December 12, 1990, respondents filed a notice of appeal,8 and in support of their
appeal, the respondents once again reiterated the testimony of the following witnesses,
namely: (1) Augusto Neri; (2) Generosa Senon; (3) Matilde Ramonal Binanay; (4)
Teresita Vedad; (5) Fiscal Rodolfo Waga; and (6) Evangeline Calugay.
To have a clear understanding of the testimonies of the witnesses, we recite an account
of their testimonies.

Augusto Neri, Clerk of Court, Court of First Instance of Misamis Oriental, where the
special proceedings for the probate of the holographic will of the deceased was filed.
He produced and identified the records of the case. The documents presented bear the
signature of the deceased, Matilde Seo Vda. de Ramonal, for the purpose of laying
the basis for comparison of the handwriting of the testatrix, with the writing treated or
admitted as genuine by the party against whom the evidence is offered.

Finally, Evangeline Calugay, one of the respondents, testified that she had lived with
the deceased since birth, and was in fact adopted by the latter. That after a long period
of time she became familiar with the signature of the deceased. She testified that the
signature appearing in the holographic will is the true and genuine signature of Matilde
Seo Vda. de Ramonal.
The holographic will which was written in Visayan, is translated in English as follows:

Generosa Senon, election registrar of Cagayan de Oro, was presented to produced


and identify the voter's affidavit of the decedent. However, the voters' affidavit was not
produced for the same was already destroyed and no longer available.

Instruction
August 30, 1978

Matilde Ramonal Binanay, testified that the deceased Matilde Seo Vda. de Ramonal
was her aunt, and that after the death of Matilde's husband, the latter lived with her in
her parent's house for eleven (11) years from 1958 to 1969. During those eleven (11)
years of close association the deceased, she acquired familiarity with her signature and
handwriting as she used to accompany her (deceased Matilde Seo Vda. de Ramonal)
in collecting rentals from her various tenants of commercial buildings, and deceased
always issued receipts. In addition to this, she (witness Matilde Binanay) assisted the
deceased in posting the records of the accounts, and carried personal letters of the
deceased to her creditors.
Matilde Ramonal Binanay further testified that at the time of the death of Matilde Vda.
de Ramonal, she left a holographic will dated August 30, 1978, which was personally
and entirely written, dated and signed, by the deceased and that all the dispositions
therein, the dates, and the signatures in said will, were that of the deceased.
Fiscal Rodolfo Waga testified that before he was appointed City Fiscal of Cagayan de
Oro, he was a practicing lawyer, and handled all the pleadings and documents signed
by the deceased in connection with the proceedings of her late husband, as a result of
which he is familiar with the handwriting of the latter. He testified that the signature
appearing in the holographic will was similar to that of the deceased, Matilde Seo Vda.
de Ramonal, but he can not be sure.
The fifth witness presented was Mrs. Teresita Vedad, an employee of the Department
of Environment and Natural Resources, Region 10. She testified that she processed
the application of the deceased for pasture permit and was familiar with the signature of
the deceased, since the signed documents in her presence, when the latter was
applying for pasture permit.

1. My share at Cogon, Raminal Street, for Evangeline Calugay.


(Sgd) Matilde Vda de Ramonal
August 30, 1978
2. Josefina Salcedo must be given 1,500 square meters at Pinikan Street.
(Sgd) Matilde Vda de Ramonal
August 30, 1978
3. My jewelry's shall be divided among:
1. Eufemia Patigas
2. Josefina Salcedo
3. Evangeline Calugay
(Sgd) Matilde Vda de Ramonal
August 30, 1978
4. I bequeath my one (1) hectare land at Mandumol, Indahag to Evangeline R.
Calugay

(Sgd) Matilde Vda de Ramonal


August 30, 1978
5. Give the 2,500 Square Meters at Sta. Cruz Ramonal Village in favor of
Evangeline R. Calugay, Helen must continue with the Sta. Cruz, once I am no
longer around.

and producing any three witnesses; they must be witnesses "who know the
handwriting and signature of the testator" and who can declare (truthfully, of
course, even if the law does not express) "that the will and the signature are in
the handwriting of the testator." There may be no available witness acquainted
with the testator's hand; or even if so familiarized, the witness maybe unwilling
to give a positive opinion. Compliance with the rule of paragraph 1 of article 811
may thus become an impossibility. That is evidently the reason why the second
paragraph of article 811 prescribes that

(Sgd) Matilde Vda de Ramonal


August 30, 1978
6. Bury me where my husband Justo is ever buried.
(Sgd) Matilde Vda de Ramonal
August 30, 1978
Gene and Manuel:
Follow my instruction in order that I will rest peacefully.
Mama
Matilde Vda de Ramonal
On October 9, 1995, the Court of Appeals, rendered decision9 ruling that the appeal
was meritorious. Citing the decision in the case of Azaola vs. Singson, 109 Phil. 102,
penned by Mr. Justice J. B. L. Reyes, a recognized authority in civil law, the Court of
Appeals held:
. . . even if the genuineness of the holographic will were contested, we are of
the opinion that Article 811 of our present civil code can not be interpreted as to
require the compulsory presentation of three witnesses to identify the
handwriting of the testator, under penalty of having the probate denied. Since no
witness may have been present at the execution of the holographic will, none
being required by law (art. 810, new civil code), it becomes obvious that the
existence of witnesses possessing the requisite qualifications is a matter
beyond the control of the proponent. For it is not merely a question of finding

in the absence of any competent witness referred to in the preceding paragraph,


and if the court deems it necessary, expert testimony may be resorted to.
As can be see, the law foresees, the possibility that no qualified witness ma be
found (or what amounts to the same thing, that no competent witness may be
willing to testify to the authenticity of the will), and provides for resort to expert
evidence to supply the deficiency.
It may be true that the rule of this article (requiring that three witnesses be
presented if the will is contested and only one if no contest is had) was derived
from the rule established for ordinary testaments (CF Cabang vs. Delfianado, 45
PHIL 291; Tolentino v. Francisco, 57 PHIL 742). But it can not be ignored that
the requirement can be considered mandatory only in case of ordinary
testaments, precisely because the presence of at least three witnesses at the
execution of ordinary wills is made by law essential to their validity (Art. 805).
Where the will is holographic, no witness need be present (art. 10), and the rule
requiring production of three witnesses must be deemed merely permissive if
absurd results are to be avoided.
Again, under Art. 811, the resort to expert evidence is conditioned by the words
"if the court deem it necessary", which reveal that what the law deems essential
is that the court should be convinced of the will's authenticity. Where the
prescribed number of witnesses is produced and the court is convinced by their
testimony that the will is genuine, it may consider it unnecessary to call for
expert evidence. On the other hand, if no competent witness is available, or
none of those produced is convincing, the court may still, and in fact it should
resort to handwriting experts. The duty of the court, in fine, is to exhaust all
available lines of inquiry, for the state is as much interested as the proponent
that the true intention of the testator be carried into effect.

Paraphrasing Azaola vs. Singson, even if the genuineness of the holographic


will were contested, Article 811 of the civil code cannot be interpreted as to
require the compulsory presentation of three witnesses to identify the
handwriting of the testator, under penalty of the having the probate denied. No
witness need be present in the execution of the holographic will. And the rule
requiring the production of three witnesses is merely permissive. What the law
deems essential is that the court is convinced of the authenticity of the will. Its
duty is to exhaust all available lines of inquiry, for the state is as much interested
in the proponent that the true intention of the testator be carried into effect. And
because the law leaves it to the trial court to decide if experts are still needed,
no unfavorable inference can be drawn from a party's failure to offer expert
evidence, until and unless the court expresses dissatisfaction with the testimony
of the lay witnesses.10

contested holographic will, that at least three witnesses explicitly declare that the
signature in the will is the genuine signature of the testator.1wphi1.nt
We are convinced, based on the language used, that Article 811 of the Civil Code is
mandatory. The word "shall" connotes a mandatory order. We have ruled that "shall" in
a statute commonly denotes an imperative obligation and is inconsistent with the idea
of discretion and that the presumption is that the word "shall," when used in a statute is
mandatory.11
Laws are enacted to achieve a goal intended and to guide against an evil or mischief
that aims to prevent. In the case at bar, the goal to achieve is to give effect to the
wishes of the deceased and the evil to be prevented is the possibility that unscrupulous
individuals who for their benefit will employ means to defeat the wishes of the testator.

According to the Court of Appeals, Evangeline Calugay, Matilde Ramonal Binanay and
other witnesses definitely and in no uncertain terms testified that the handwriting and
signature in the holographic will were those of the testator herself.

So, we believe that the paramount consideration in the present petition is to determine
the true intent of the deceased. An exhaustive and objective consideration of the
evidence is imperative to establish the true intent of the testator.

Thus, upon the unrebutted testimony of appellant Evangeline Calugay and witness
Matilde Ramonal Binanay, the Court of Appeals sustained the authenticity of the
holographic will and the handwriting and signature therein, and allowed the will to
probate.

It will be noted that not all the witnesses presented by the respondents testified
explicitly that they were familiar with the handwriting of testator. In the case of Augusto
Neri, clerk of court, Court of First Instance, Misamis Oriental, he merely identified the
record of Special Proceedings No. 427 before said court. He was not presented to
declare explicitly that the signature appearing in the holographic was that of the
deceased.

Hence, this petition.


The petitioners raise the following issues:
(1) Whether or not the ruling of the case of Azaola vs. Singson, 109 Phil. 102,
relied upon by the respondent Court of Appeals, was applicable to the case.

Generosa E. Senon, the election registrar of Cagayan de Oro City, was presented to
identify the signature of the deceased in the voter's affidavit, which was not even
produced as it was no longer available.
Matilde Ramonal Binanay, on the other hand, testified that:

(2) Whether or not the Court of Appeals erred in holding that private
respondents had been able to present credible evidence to that the date, text,
and signature on the holographic will written entirely in the hand of the testatrix.
(3) Whether or not the Court of Appeals erred in not analyzing the signatures in
the holographic will of Matilde Seo Vda. de Ramonal.

Q. And you said for eleven (11) years Matilde Vda de Ramonal resided with
your parents at Pinikitan, Cagayan de Oro City. Would you tell the court what
was your occupation or how did Matilde Vda de Ramonal keep herself busy that
time?
A. Collecting rentals.

In this petition, the petitioners ask whether the provisions of Article 811 of the Civil Code
are permissive or mandatory. The article provides, as a requirement for the probate of a

Q. From where?

A. From the land rentals and commercial buildings at Pabayo-Gomez streets.12


xxx

xxx

A. In handwritten.14

xxx

Q. Who sometime accompany her?

xxx

xxx

xxx

A. I sometimes accompany her.

Q. In addition to collection of rentals, posting records of accounts of tenants


and deed of sale which you said what else did you do to acquire familiarity of
the signature of Matilde Vda De Ramonal?

Q. In collecting rentals does she issue receipts?

A. Posting records.

A. Yes, sir.13

Q. Aside from that?


xxx

xxx

xxx

Q. Showing to you the receipt dated 23 October 1979, is this the one you are
referring to as one of the receipts which she issued to them?

A. Carrying letters.
Q. Letters of whom?
A. Matilde.

A. Yes, sir.
Q. To whom?
Q. Now there is that signature of Matilde vda. De Ramonal, whose signature is
that Mrs. Binanay?

A. To her creditors.15

A. Matilde vda. De Ramonal.


Q. Why do you say that is the signature of Matilde Vda. De Ramonal?

xxx

xxx

xxx

Q. You testified that at time of her death she left a will. I am showing to you a
document with its title "tugon" is this the document you are referring to?

A. I am familiar with her signature.


A. Yes, sir.
Q. Now, you tell the court Mrs. Binanay, whether you know Matilde vda de
Ramonal kept records of the accounts of her tenants?

Q. Showing to you this exhibit "S", there is that handwritten "tugon", whose
handwriting is this?

A. Yes, sir.
A. My Aunt.
Q. Why do you say so?
Q. Why do you say this is the handwriting of your aunt?
A. Because we sometimes post a record of accounts in behalf of Matilde Vda.
De Ramonal.
Q. How is this record of accounts made? How is this reflected?

A. Because I am familiar with her signature.16

What Ms. Binanay saw were pre-prepared receipts and letters of the deceased, which
she either mailed or gave to her tenants. She did not declare that she saw the
deceased sign a document or write a note.

A. I presented it to the fiscal.

Further, during the cross-examination, the counsel for petitioners elicited the fact that
the will was not found in the personal belongings of the deceased but was in the
possession of Ms. Binanay. She testified that:

A. Just to seek advice.

Q. Mrs. Binanay, when you were asked by counsel for the petitioners if the late
Matilde Seno vda de Ramonal left a will you said, yes?

Q. For what purpose?

Q. Advice of what?
A. About the will.18

Q. Who was in possession of that will?

In her testimony it was also evident that Ms. Binanay kept the fact about the will from
petitioners, the legally adopted children of the deceased. Such actions put in issue her
motive of keeping the will a secret to petitioners and revealing it only after the death of
Matilde Seo Vda. de Ramonal.

A. I.

In the testimony of Ms. Binanay, the following were established:

A. Yes, sir.

Q. Since when did you have the possession of the will?

Q. Now, in 1978 Matilde Seno Vda de Ramonal was not yet a sickly person is
that correct?

A. It was in my mother's possession.


A. Yes, sir.
Q. So, it was not in your possession?
Q. She was up and about and was still uprightly and she could walk agilely and
she could go to her building to collect rentals, is that correct?

A. Sorry, yes.
Q. And when did you come into possession since as you said this was
originally in the possession of your mother?

A. Yes, sir.19
xxx

A. 1985.

xxx

xxx

17

xxx

xxx

xxx

Q. Now, Mrs. Binanay was there any particular reason why your mother left
that will to you and therefore you have that in your possession?

Q. Now, let us go to the third signature of Matilde Ramonal. Do you know that
there are retracings in the word Vda.?
A. Yes, a little. The letter L is continuous.
Q. And also in Matilde the letter L is continued to letter D?

A. It was not given to me by my mother, I took that in the aparador when she
died.
Q. After taking that document you kept it with you?

A. Yes, sir.

Q. Again the third signature of Matilde Vda de Ramonal the letter L in Matilde
is continued towards letter D.

A. Yes, sir.21
Evangeline Calugay declared that the holographic will was written, dated and signed in
the handwriting of the testator. She testified that:

A. Yes, sir.
Q. And there is a retracing in the word Vda.?

Q. You testified that you stayed with the house of the spouses Matilde and
Justo Ramonal for the period of 22 years. Could you tell the court the services if
any which you rendered to Matilde Ramonal?

A. Yes, sir.20
xxx

xxx

xxx

Q. Now, that was 1979, remember one year after the alleged holographic will.
Now, you identified a document marked as Exhibit R. This is dated January 8,
1978 which is only about eight months from August 30, 1978. Do you notice that
the signature Matilde Vda de Ramonal is beautifully written and legible?

A. During my stay I used to go with her to the church, to market and then to
her transactions.
Q. What else? What services that you rendered?
A. After my college days I assisted her in going to the bank, paying taxes and
to her lawyer.

A. Yes, sir the handwriting shows that she was very exhausted.
Q. What was your purpose of going to her lawyer?
Q. You just say that she was very exhausted while that in 1978 she was
healthy was not sickly and she was agile. Now, you said she was exhausted?
A. In writing.

A. I used to be her personal driver.


Q. In the course of your stay for 22 years did you acquire familiarity of the
handwriting of Matilde Vda de Ramonal?

Q. How did you know that she was exhausted when you were not present and
you just tried to explain yourself out because of the apparent inconsistencies?

A. Yes, sir.

A. That was I think. (sic).

Q. How come that you acquired familiarity?

Q. Now, you already observed this signature dated 1978, the same year as the
alleged holographic will. In exhibit I, you will notice that there is no retracing;
there is no hesitancy and the signature was written on a fluid movement. . . .
And in fact, the name Eufemia R. Patigas here refers to one of the petitioners?

A. Because I lived with her since birth.22

A. Yes, sir.
Q. You will also notice Mrs. Binanay that it is not only with the questioned
signature appearing in the alleged holographic will marked as Exhibit X but in
the handwriting themselves, here you will notice the hesitancy and tremors, do
you notice that?

xxx

xxx

xxx

Q. Now, I am showing to you Exhibit S which is captioned "tugon" dated


Agosto 30, 1978 there is a signature here below item No. 1, will you tell this
court whose signature is this?
A. Yes, sir, that is her signature.
Q. Why do you say that is her signature?

A. I am familiar with her signature.23

A. It is about the project partition to terminate the property, which was under
the court before.26

So, the only reason that Evangeline can give as to why she was familiar with the
handwriting of the deceased was because she lived with her since birth. She never
declared that she saw the deceased write a note or sign a document.
The former lawyer of the deceased, Fiscal Waga, testified that:
Q. Do you know Matilde Vda de Ramonal?
A. Yes, sir I know her because she is my godmother the husband is my
godfather. Actually I am related to the husband by consanguinity.

xxx

xxx

xxx

Q. Appearing in special proceeding no. 427 is the amended inventory which is


marked as exhibit N of the estate of Justo Ramonal and there appears a
signature over the type written word Matilde vda de Ramonal, whose signature
is this?
A. That is the signature of Matilde Vda de Ramonal.
Q. Also in exhibit n-3, whose signature is this?

Q. Can you tell the name of the husband?


A. This one here that is the signature of Mrs. Matilde vda de Ramonal.27
A. The late husband is Justo Ramonal.24
xxx
xxx

xxx

A. As far as I know they have no legitimate children.25


xxx

xxx

xxx

Q. Can you tell this court whether the spouses Justo Ramonal and Matilde
Ramonal have legitimate children?

xxx

xxx

Q. Aside from attending as counsel in that Special Proceeding Case No. 427
what were the other assistance wherein you were rendering professional
service to the deceased Matilde Vda de Ramonal?
A. I can not remember if I have assisted her in other matters but if there are
documents to show that I have assisted then I can recall.28

xxx
xxx

xxx

xxx

Q. You said after becoming a lawyer you practice your profession? Where?
A. Here in Cagayan de Oro City.

Q. Now, I am showing to you exhibit S which is titled "tugon", kindly go over


this document, Fiscal Waga and tell the court whether you are familiar with the
handwriting contained in that document marked as exhibit "S"?

Q. Do you have services rendered with the deceased Matilde vda de


Ramonal?

A. I am not familiar with the handwriting.

A. I assisted her in terminating the partition, of properties.

Q. This one, Matilde Vda de Ramonal, whose signature is this?

Q. When you said assisted, you acted as her counsel? Any sort of counsel as
in what case is that, Fiscal?

A. I think this signature here it seems to be the signature of Mrs. Matilde vda
de Ramonal.

Q. Now, in item No. 2 there is that signature here of Matilde Vda de Ramonal,
can you tell the court whose signature is this?
A. Well, that is similar to that signature appearing in the project of partition.
Q. Also in item no. 3 there is that signature Matilde Vda de Ramonal, can you
tell the court whose signature is that?
A. As I said, this signature also seems to be the signature of Matilde vda de
Ramonal.

In the case of Ajero vs. Court of Appeals,32 we said that "the object of the solemnities
surrounding the execution of wills is to close the door against bad faith and fraud, to
avoid substitution of wills and testaments and to guaranty their truth and authenticity.
Therefore, the laws on this subject should be interpreted in such a way as to attain
these primordial ends. But on the other hand, also one must not lose sight of the fact
that it is not the object of the law to restrain and curtail the exercise of the right to make
a will.
However, we cannot eliminate the possibility of a false document being adjudged as the
will of the testator, which is why if the holographic will is contested, that law requires
three witnesses to declare that the will was in the handwriting of the deceased.

Q. Why do you say that?


A. Because there is a similarity in the way it is being written.
Q. How about this signature in item no. 4, can you tell the court whose
signature is this?
A. The same is true with the signature in item no. 4. It seems that they are
similar.29
xxx

xxx

xxx

Q. Mr. Prosecutor, I heard you when you said that the signature of Matilde Vda
de Ramonal Appearing in exhibit S seems to be the signature of Matilde vda de
Ramonal?
A. Yes, it is similar to the project of partition.
Q. So you are not definite that this is the signature of Matilde vda de Ramonal.
You are merely supposing that it seems to be her signature because it is similar
to the signature of the project of partition which you have made?
A. That is true.30

The will was found not in the personal belongings of the deceased but with one of the
respondents, who kept it even before the death of the deceased. In the testimony of
Ms. Binanay, she revealed that the will was in her possession as early as 1985, or five
years before the death of the deceased.
There was no opportunity for an expert to compare the signature and the handwriting of
the deceased with other documents signed and executed by her during her lifetime.
The only chance at comparison was during the cross-examination of Ms. Binanay when
the lawyer of petitioners asked Ms. Binanay to compare the documents which
contained the signature of the deceased with that of the holographic will and she is not
a handwriting expert. Even the former lawyer of the deceased expressed doubts as to
the authenticity of the signature in the holographic will.
A visual examination of the holographic will convince us that the strokes are different
when compared with other documents written by the testator. The signature of the
testator in some of the disposition is not readable. There were uneven strokes,
retracing and erasures on the will.
Comparing the signature in the holographic will dated August 30, 1978,33 and the
signatures in several documents such as the application letter for pasture permit dated
December 30, 1980,34 and a letter dated June 16, 1978,35 the strokes are different. In
the letters, there are continuous flows of the strokes, evidencing that there is no
hesitation in writing unlike that of the holographic will. We, therefore, cannot be certain
that ruling holographic will was in the handwriting by the deceased.

From the testimonies of these witnesses, the Court of Appeals allowed the will to
probate and disregard the requirement of three witnesses in case of contested
holographic will, citing the decision in Azaola vs. Singson,31ruling that the requirement is IN VIEW WHEREOF, the decision appealed from is SET ASIDE. The records are
ordered remanded to the court of origin with instructions to allow petitioners to adduce
merely directory and not mandatory.

evidence in support of their opposition to the probate of the holographic will of the
deceased Matilde Seo vda. de Ramonal.1wphi1.nt
No costs.
SO ORDERED.

G.R. Nos. 75005-06 February 15, 1990


JOSE RIVERA petitioner, vs. INTERMEDIATE APPELLATE COURT and ADELAIDO
J. RIVERA, respondents.
Lorenzo O. Navarro, Jr. for petitioner.
Regalado P. Morales for private respondent.

CRUZ, J.:
Was there only one Venancio Rivera in Mabalacat, Pampanga, or were there two?
On May 30, 1975, a prominent and wealthy resident of that town named Venancio
Rivera died. On July 28, 1975, Jose Rivera, claiming to be the only surviving legitimate
son of the deceased, filed a petition for the issuance of letters of administration over
Venancio's estate. Docketed as SP No. 1076, this petition was opposed by Adelaido J.
Rivera, who denied that Jose was the son of the decedent. Adelaido averred that
Venancio was his father and did not die intestate but in fact left two holographic wills. 1
On November 7, 1975, Adelaido J. Rivera filed, also with the Regional Trial Court of
Angeles City, a petition for the probate of the holographic wills. Docketed as SP No.
1091, this petition was in turn opposed by Jose Rivera, who reiterated that he was the
sole heir of Venancio's intestate estate. 2

In support of his claim that he was the sole heir of the late Venancio Rivera, Jose
sought to show that the said person was married in 1928 to Maria Vital, who was his
mother. He submitted for this purpose Exhibit A, the marriage certificate of the couple,
and Exhibit B, his own baptismal certificate where the couple was indicated as his
parents. The petitioner also presented Domingo Santos, who testified that Jose was
indeed the son of the couple and that he saw Venancio and Jose together several
times. 5 Jose himself stressed that Adelaido considered him a half-brother and kissed
his hand as a sign of respect whenever they met. He insisted that Adelaido and his
brothers and sisters were illegitimate children, sired by Venancio with Maria Jocson. 6
Adelaido, for his part, maintained that he and his brothers and sisters were born to
Venancio Rivera and Maria Jocson, who were legally married and lived as such for
many years. He explained that he could not present his parents' marriage certificate
because the record of marriages for 1942 in Mabalacat were destroyed when the town
was burned during the war, as certified by Exhibit 6. 7 He also submitted his own birth
certificate and those of his sisters Zenaida and Yolanda Rivera, who were each
described therein as the legimitate children of Venancio Rivera and Maria
Jocson. 8 Atty. Regalado P. Morales, then 71 years of age, affirmed that he knew the
deceased and his parents, Magno Rivera and Gertrudes de los Reyes, and it was
during the Japanese occupation that Venancio introduced to him Maria Jocson as his
wife. 9 To prove that there were in fact two persons by the same name of Venancio
Rivera, Adelaido offered Venancio Rivera's baptismal certificate showing that his
parents were Magno Rivera and Gertrudes de los Reyes, 10 as contrasted with the
marriage certificate submitted by Jose, which indicated that the Venancio Rivera
subject thereof was the son of Florencio Rivera and Estrudez Reyes. 11He also denied
kissing Jose's hand or recognizing him as a brother. 12

On November 11, 1975, the two cases were consolidated. Adelaido J. Rivera was later
appointed special administrator. After joint trial, Judge Eliodoro B. Guinto found that
Jose Rivera was not the son of the decedent but of a different Venancio Rivera who
was married to Maria Vital. The Venancio Rivera whose estate was in question was
married to Maria Jocson, by whom he had seven children, including Adelaido. Jose
Rivera had no claim to this estate because the decedent was not his father. The
holographic wills were also admitted to probate.3

We find in favor of Adelaido J. Rivera.

On appeal, the decision of the trial court was affirmed by the then Intermediate
Appellate Court. 4 Its decision is now the subject of this petition, which urges the
reversal of the respondent court.

According to Article 220 of the Civil Code:

It is true that Adelaido could not present his parents' marriage certificate because, as he
explained it, the marriage records for 1942 in the Mabalacat civil registry were burned
during the war. Even so, he could still rely on the presumption of marriage, since it is
not denied that Venancio Rivera and Maria Jocson lived together as husband and wife
for many years, begetting seven children in all during that time.

In case of doubt, all presumptions favor the solidarity of the family. Thus
every intendment of the law or fact leans toward the validity of marriage,
the indissolubility of the marriage bonds, the legitimacy of children, ... .
The Rules of Court, in Rule 131, provides:
SEC. 3. Disputable presumptions. The following presumptions are
satisfactory if uncontradicted, but may be contradicted and overcome by
other evidence:
xxx xxx xxx
(aa) That a man and woman deporting themselves as husband and wife
have entered into a lawful contract of marriage.
By contrast, although Jose did present his parents' marriage certificate, Venancio was
described therein as the son of Florencio Rivera. Presumably, he was not the same
Venancio Rivera described in Exhibit 4, his baptismal certificate, as the son of Magno
Rivera. While we realize that such baptismal certificate is not conclusive evidence of
Venancio's filiation (which is not the issue here) it may nonetheless be considered to
determine his real identity. Jose insists that Magno and Florencio are one and the same
person, arguing that it is not uncommon for a person to be called by different names.
The Court is not convinced. There is no evidence that Venancio's father was called
either Magno or Florencio. What is more likely is that two or more persons may live at
the same time and bear the same name, even in the same community. That is what the
courts below found in the cases at bar.
What this Court considers particularly intriguing is why, if it is true that he was the
legitimate son of Venancio Rivera, Jose did not assert his right as such when his father
was still alive. By his own account, Jose supported himself and presumably also his
mother Maria Vital as a gasoline attendant and driver for many years. All the time,
his father was residing in the same town and obviously prospering and available
for support. His alleged father was openly living with another woman and raising
another family, but this was apparently accepted by Jose without protest, taking no step
whatsoever to invoke his status. If, as he insists, he and Venancio Rivera were on
cordial terms, there is no reason why the father did not help the son and instead left
Jose to fend for himself as a humble worker while his other children by Maria Jocson
enjoyed a comfortable life. Such paternal discrimination is difficult to understand,

especially if it is considered assuming the claims to be true that Jose was the
oldest and, by his own account, the only legitimate child of Venancio Rivera.
And there is also Maria Vital, whose attitude is no less incomprehensible. As Venancio's
legitimate wife if indeed she was she should have objected when her husband
abandoned her and founded another family by another woman, and in the same town
at that. Seeing that the children of Maria Jocson were being raised well while her own
son Jose was practically ignored and neglected, she nevertheless did not demand for
him at least support, if not better treatment, from his legitimate father. It is unnatural for
a lawful wife to say nothing if she is deserted in favor of another woman and for a
caring mother not to protect her son's interests from his wayward father's neglect. The
fact is that this forsaken wife never demanded support from her wealthy if errant
husband. She did not file a complaint for bigamy or concubinage against Venancio
Rivera and Maria Jocson, the alleged partners in crime and sin. Maria Vital was
completely passive and complaisant.
Significantly, as noted by the respondent court, Maria Vital was not even presented at
the trial to support her son's allegations that she was the decedent's lawful wife. Jose
says this was not done because she was already old and bedridden then. But there
was no impediment to the taking of her deposition in her own house. No effort was
made toward this end although her testimony was vital to the petitioner's cause. Jose
dismisses such testimony as merely "cumulative," but this Court does not agree.
Having alleged that Maria Jocson's marriage to Venancio Rivera was null and void,
Jose had the burden of proving that serious allegation.
We find from the evidence of record that the respondent court did not err in holding that
the Venancio Rivera who married Maria Jocson in 1942 was not the same person who
married Maria Vital, Jose's legitimate mother, in 1928. Jose belonged to a humbler
family which had no relation whatsoever with the family of Venancio Rivera and Maria
Vital. This was more prosperous and prominent. Except for the curious Identity of
names of the head of each, there is no evidence linking the two families or showing that
the deceased Venancio Rivera was the head of both.
Now for the holographic wills. The respondent court considered them valid because it
found them to have been written, dated and signed by the testator himself in
accordance with Article 810 of the Civil Code. It also held there was no necessity of
presenting the three witnesses required under Article 811 because the authenticity of
the wills had not been questioned.

The existence and therefore also the authenticity of the holographic wills were
questioned by Jose Rivera. In his own petition in SP No. 1076, he declared that
Venancio Rivera died intestate; and in SP No. 1091, he denied the existence of the
holographic wills presented by Adelaido Rivera for probate. In both proceedings, Jose
Rivera opposed the holographic wills submitted by Adelaido Rivera and claimed that
they were spurious. Consequently, it may be argued, the respondent court should have
applied Article 811 of the Civil Code, providing as follows:
In the probate of a holographic will, it shall be necessary that at least
one witness who knows the handwriting and signature of the testator
explicitly declare that the will and the signature are in the handwriting of
the testator. If the will is contested, at least three of such witnesses shall
be required.
The flaw in this argument is that, as we have already determined, Jose Rivera is not the
son of the deceased Venancio Rivera whose estate is in question. Hence, being a mere
stranger, he had no personality to contest the wills and his opposition thereto did not
have the legal effect of requiring the three witnesses. The testimony of Zenaida and
Venancio Rivera, Jr., who authenticated the wills as having been written and signed by
their father, was sufficient.
WHEREFORE, the petition is DENIED and the challenged decision is AFFIRMED, with
costs against the petitioner.
SO ORDERED.

G.R. No. L-12190

August 30, 1958

TESTATE ESTATE OF FELICIDAD ESGUERRA ALTO-YAP deceased. FAUSTO E.


GAN, petitioner-appellant,
vs.
ILDEFONSO YAP, oppositor-appellee.
Benedicto C. Belran, Crispin D. Baizas and Roberto H. Benitez for appellant.
Arturo M. Tolentino for appellee.
BENGZON, J.:
On November 20, 1951, Felicidad Esguerra Alto Yap died of heart failure in the
University of Santo Tomas Hospital, leaving properties in Pulilan, Bulacan, and in the
City of Manila.
On March 17, 1952, Fausto E. Gan initiated them proceedings in the Manila court of
first instance with a petition for the probate of a holographic will allegedly executed by
the deceased, substantially in these words:
Nobyembre 5, 1951.
Ako, si Felicidad E. Alto-Yap, may asawa, at ganap na pag-iisip, ay
nagsasalaysay na ang aking kayamanan sa bayan ng Pulilan, Bulacan ay aking
ipinamamana sa aking mga kamag-anakang sumusunod:
Vicente Esguerra,
Sr. .............................................

5 Bahagi

Fausto E.
Gan .........................................................

2 Bahagi

Rosario E.
Gan .........................................................

2 Bahagi

Filomena
Alto ..........................................................

1 Bahagi

Beatriz
1 Bahagi
Alto ............................................................
..
At ang aking lahat ng ibang kayamanan sa Maynila at iba panglugar ay aking
ipinamamana sa aking asawang si Idelfonso D. Yap sa kondisyong siya'y
magpapagawa ng isang Health Center na nagkakahalaga ng di kukulangin sa
halagang P60,000.00 sa bayan ng Pulilan, Bulacan, na nakaukit ang aking
pangalang Felicidad Esguerra-Alto. At kung ito ay may kakulangan man ay

bahala na ang aking asawa ang magpuno upang matupad ang aking
kagustuhan.
(Lagda) Felicidad E. Alto-Yap.
Opposing the petition, her surviving husband Ildefonso Yap asserted that the deceased
had not left any will, nor executed any testament during her lifetime.
After hearing the parties and considering their evidence, the Hon. Ramon R. San Jose,
Judge,1 refused to probate the alleged will. A seventy-page motion for reconsideration
failed. Hence this appeal.
The will itself was not presented. Petitioner tried to establish its contents and due
execution by the statements in open court of Felina Esguerra, Primitivo Reyes, Socorro
Olarte and Rosario Gan Jimenez, whose testimonies may be summarized as follows:
Sometime in 1950 after her last trip abroad, Felicidad Esguerra mentioned to her first
cousin, Vicente Esguerra, her desire to make a will. She confided however that it would
be useless if her husband discovered or knew about it. Vicente consulted with Fausto
E. Gan, nephew of Felicidad, who was then preparing for the bar examinations. The
latter replied it could be done without any witness, provided the document was entirely
in her handwriting, signed and dated by her. Vicente Esguerra lost no time in
transmitting the information, and on the strength of it, in the morning of November 5,
1951, in her residence at Juan Luna Street, Manila, Felicidad wrote, signed and dated a
holographic will substantially of the tenor above transcribed, in the presence of her
niece, Felina Esguerra (daughter of Vicente), who was invited to read it. In the
afternoon of that day, Felicidad was visited by a distant relative, Primitivo Reyes, and
she allowed him to read the will in the presence of Felina Esguerra, who again read it.
Nine days later, he had other visitors: Socorro Olarte a cousin, and Rosario Gan
Jimenez, a niece. To these she showed the will, again in the presence of Felina
Esguerra, who read it for the third time.
When on November 19, 1951, Felicidad was confined at the U.S.T. Hospital for her last
illness, she entrusted the said will, which was contained in a purse, to Felina Esguerra.
But a few hours later, Ildefonso Yap, her husband, asked Felina for the purse: and
being afraid of him by reason of his well-known violent temper, she delivered it to him.
Thereafter, in the same day, Ildefonso Yap returned the purse to Felina, only to demand
it the next day shortly before the death of Felicidad. Again, Felina handed it to him but
not before she had taken the purse to the toilet, opened it and read the will for the last
time.2
From the oppositor's proof it appears that Felicidad Esguerra had been suffering from
heart disease for several years before her death; that she had been treated by
prominent physicians, Dr. Agerico Sison, Dr. Agustin Liboro and others; that in May
1950 husband and wife journeyed to the United States wherein for several weeks she

was treated for the disease; that thereafter she felt well and after visiting interesting
places, the couple returned to this country in August 1950. However, her ailment
recurred, she suffered several attacks, the most serious of which happened in the early
morning of the first Monday of November 1951 (Nov. 5). The whole household was
surprised and alarmed, even the teachers of the Harvardian Colleges occupying the
lower floors and of by the Yap spouses. Physician's help was hurriedly called, and Dr.
Tanjuaquio arrived at about 8:00 a.m., found the patient hardly breathing, lying in bed,
her head held high by her husband. Injections and oxygen were administered.
Following the doctor's advice the patient stayed in bed, and did nothing the whole day,
her husband and her personal attendant, Mrs. Bantique, constantly at her side. These
two persons swore that Mrs. Felicidad Esguerra Yap made no will, and could have
made no will on that day.
The trial judge refused to credit the petitioner's evidence for several reasons, the most
important of which were these: (a) if according to his evidence, the decedent wanted to
keep her will a secret, so that her husband would not know it, it is strange she executed
it in the presence of Felina Esguerra, knowing as she did that witnesses were
unnecessary; (b) in the absence of a showing that Felina was a confidant of the
decedent it is hard to believe that the latter would have allowed the former to see and
read the will several times; (c) it is improbable that the decedent would have permitted
Primitivo Reyes, Rosario Gan Jimenez and Socorro Olarte to read her will, when she
precisely wanted its contents to remain a secret during her lifetime; (d) it is also
improbable that her purpose being to conceal the will from her husband she would
carry it around, even to the hospital, in her purse which could for one reason or another
be opened by her husband; (e) if it is true that the husband demanded the purse from
Felina in the U.S.T. Hospital and that the will was there, it is hard to believe that he
returned it without destroying the will, the theory of the petitioner being precisely that
the will was executed behind his back for fear he will destroy it.
In the face of these improbabilities, the trial judge had to accept the oppositor's
evidence that Felicidad did not and could not have executed such holographic will.
In this appeal, the major portion of appellant's brief discussed the testimony of the
oppositor and of his witnesses in a vigorous effort to discredit them. It appears that the
same arguments, or most of them, were presented in the motion to reconsider; but they
failed to induce the court a quo to change its mind. The oppositor's brief, on the other
hand, aptly answers the criticisms. We deem it unnecessary to go over the same
matters, because in our opinion the case should be decided not on the weakness of the
opposition but on the strength of the evidence of the petitioner, who has the burden of
proof.
The Spanish Civil Code permitted the execution of holographic wills along with other
forms. The Code of Civil Procedure (Act 190) approved August 7, 1901, adopted only
one form, thereby repealing the other forms, including holographic wills.

The New Civil Code effective in 1950 revived holographic wills in its arts. 810-814. "A
person may execute a holographic will which must be entirely written, dated, and
signed by the hand of the testator himself. It is subject to no other form and may be
made in or out of the Philippines, and need not be witnessed."
This is indeed a radical departure from the form and solemnities provided for wills
under Act 190, which for fifty years (from 1901 to 1950) required wills to be subscribed
by the testator and three credible witnesses in each andevery page; such witnesses to
attest to the number of sheets used and to the fact that the testator signed in their
presence and that they signed in the presence of the testator and of each other.
The object of such requirements it has been said, is to close the door against bad faith
and fraud, to prevent substitution of wills, to guarantee their truth and authencity
(Abangan vs. Abangan, 40 Phil., 476) and to avoid those who have no right to succeed
the testator would succeed him and be benefited with the probate of same.
(Mendoza vs. Pilapil, 40 Off. Gaz., 1855). However, formal imperfections may be
brushed aside when authenticity of the instrument is duly proved. (Rodriguez vs Yap,
40 Off. Gaz. 1st Supp. No. 3 p. 194.)
Authenticity and due execution is the dominant requirements to be fulfilled when such
will is submitted to the courts for allowance. For that purpose the testimony of one of
the subscribing witnesses would be sufficient if there is no opposition (Sec. 5, Rule 77).
If there is, the three must testify, if available. (Cabang vs. Delfinado, 34 Phil., 291;
Tolentino vs. Francisco, 57 Phil., 742). From the testimony of such witnesses (and of
other additional witnesses) the court may form its opinion as to the genuineness and
authenticity of the testament, and the circumstances its due execution.
Now, in the matter of holographic wills, no such guaranties of truth and veracity are
demanded, since as stated, they need no witnesses; provided however, that they are
"entirely written, dated, and signed by the hand of the testator himself." The law, it is
reasonable to suppose, regards the document itself as material proof of authenticity,
and as its own safeguard, since it could at any time, be demonstrated to be or not to
be in the hands of the testator himself. "In the probate of a holographic will" says the
New Civil Code, "it shall be necessary that at least one witness who knows the
handwriting and signature of the testator explicitly declare that the will and the signature
are in the handwriting of the testator. If the will is contested, at least three such
witnesses shall be required. In the absence of any such witnesses, (familiar with
decedent's handwriting) and if the court deem it necessary, expert testimony may be
resorted to."
The witnesses so presented do not need to have seen the execution of the holographic
will. They may be mistaken in their opinion of the handwriting, or they may deliberately
lie in affirming it is in the testator's hand. However, the oppositor may present other
witnesses who also know the testator's handwriting, or some expert witnesses, who
after comparing the will with other writings or letters of the deceased, have come to the

conclusion that such will has not been written by the hand of the deceased. (Sec. 50,
Rule 123). And the court, in view of such contradictory testimony may use its own visual
sense, and decide in the face of the document, whether the will submitted to it has
indeed been written by the testator.
Obviously, when the will itself is not submitted, these means of opposition, and of
assessing the evidence are not available. And then the only guaranty of authenticity3
the testator's handwriting has disappeared.
Therefore, the question presents itself, may a holographic will be probated upon the
testimony of witnesses who have allegedly seen it and who declare that it was in the
handwriting of the testator? How can the oppositor prove that such document was not
in the testator's handwriting? His witnesses who know testator's handwriting have not
examined it. His experts can not testify, because there is no way to compare the
alleged testament with other documents admittedly, or proven to be, in the testator's
hand. The oppositor will, therefore, be caught between the upper millstone of his lack of
knowledge of the will or the form thereof, and the nether millstone of his inability to
prove its falsity. Again the proponent's witnesses may be honest and truthful; but they
may have been shown a faked document, and having no interest to check the
authenticity thereof have taken no pains to examine and compare. Or they may be
perjurers boldly testifying, in the knowledge that none could convict them of perjury,
because no one could prove that they have not "been shown" a document which they
believed was in the handwriting of the deceased. Of course, the competency of such
perjured witnesses to testify as to the handwriting could be tested by exhibiting to them
other writings sufficiently similar to those written by the deceased; but what witness or
lawyer would not foresee such a move and prepare for it? His knowledge of the
handwriting established, the witness (or witnesses) could simply stick to his statement:
he has seen and read a document which he believed was in the deceased's
handwriting. And the court and the oppositor would practically be at the mercy of such
witness (or witnesses) not only as to the execution, but also as to the contents of the
will. Does the law permit such a situation?
The Rules of Court, (Rule 77) approved in 1940 allow proof (and probate) of a lost or
destroyed will by secondary evidence the testimony of witnesses, in lieu of the
original document. Yet such Rules could not have contemplated holographic wills which
could not then be validly made here. (See also Sec. 46, Rule 123; Art. 830-New Civil
Code.)
Could Rule 77 be extended, by analogy, to holographic wills?
Spanish commentators agree that one of the greatest objections to the holographic will
is that it may be lost or stolen4 an implied admission that such loss or theft renders it
useless..
This must be so, because the Civil Code requires it to be protocoled and presented to
the judge, (Art. 689) who shall subscribe it and require its identity to be established by

the three witnesses who depose that they have no reasonable doubt that the will was
written by the testator (Art. 691). And if the judge considers that the identity of the will
has been proven he shall order that it be filed (Art. 693). All these, imply presentation of
the will itself. Art. 692 bears the same implication, to a greater degree. It requires that
the surviving spouse and the legitimate ascendants and descendants be summoned so
that they may make "any statement they may desire to submit with respect to the
authenticity of the will." As it is universally admitted that the holographic will is usually
done by the testator and by himself alone, to prevent others from knowing either its
execution or its contents, the above article 692 could not have the idea of simply
permitting such relatives to state whether they know of the will, but whether in the face
of the document itself they think the testator wrote it. Obviously, this they can't do
unless the will itself is presented to the Court and to them.
Undoubtedly, the intention of the law is to give the near relatives the choice of either
complying with the will if they think it authentic, or to oppose it, if they think it
spurious.5 Such purpose is frustrated when the document is not presented for their
examination. If it be argued that such choice is not essential, because anyway the
relatives may oppose, the answer is that their opposition will be at a distinct
disadvantage, and they have the right and privilege to comply with the will, if genuine, a
right which they should not be denied by withholding inspection thereof from them.
We find confirmation of these ideas--about exhibition of the document itself--in the
decision of the Supreme Court of Spain of June 5, 1925, which denied protocolization
or probate to a document containing testamentary dispositions in the handwriting of the
deceased, but apparently mutilated, the signature and some words having been torn
from it. Even in the face of allegations and testimonial evidence (which was
controverted), ascribing the mutilation to the opponents of the will. The aforesaid
tribunal declared that, in accordance with the provision of the Civil Code (Spanish) the
will itself, whole and unmutilated, must be presented; otherwise, it shall produce no
effect.
Considerando que sentado lo anterior, y estableciendose en el parrafo segundo
del articulo 688 del Codigo civil, que para que sea valido el testamento olografo
debera estar escrito todo el y firmado por testador, con expression del ao, mes
y dia en que se otorque, resulta evidente que para la validez y eficacia de esos
testamentos, no basta la demostracion mas o menos cumplida de que cuando
se otorgaron se Ilenaron todos esos requisitos, sino que de la expresada
redaccion el precepto legal, y por el tiempo en que el verbo se emplea,
se desprende la necesidad de que el documento se encuentre en dichas
condiciones en el momento de ser presentado a la Autoridad competente, para
au adveracion y protocolizacion; y como consecuencia ineludible de ello,
forzoso es affirmar que el de autos carece de validez y aficacia, por no
estarfirmado por el testador, cualquiera que sea la causa de la falta de firma, y
sin perjuicio de las acciones que puedan ejercitar los perjudicados, bien para

pedir indemnizacion por el perjuicio a la persona culpable, si la hubiere, o su


castigo en via criminal si procediere, por constituir dicha omision un defecto
insubsanable . . . .
This holding aligns with the ideas on holographic wills in the Fuero Juzgo, admittedly
the basis of the Spanish Civil Code provisions on the matter.6
PRECEDENTES LEGALES--Fuero Juzgo, libro segundo, titulo V, ley 15--E
depues que los herederos e sus fijos ovieren esta manda, fasta ... annos
muestrenla al obispo de la tierra, o al juez fasta VI meses y el obispo o el juez
tomen otros tales tres escritos, que fuesen fechos por su mano daquel que fizo
la manda; e por aquellos escriptos, si semjara la letra de la manda, sea
confirmada la manda. E depues que todo esto fuere connoscido, el obispo o el
juez, o otras testimonios confirmen el escripto de la manda otra vez, y en esta
manera vala la manda. (Art. 689, Scaevola--Codigo Civil.)
(According to the Fuero above, the will itself must be compared with specimens of the
testators handwriting.)
All of which can only mean: the courts will not distribute the property of the deceased in
accordance with his holographic will, unless they are shown his handwriting and
signature.7
Parenthetically, it may be added that even the French Civil Law considers the loss of
the holographic will to be fatal. (Planiol y Ripert, Derecho Civil Frances, traduccion por
Diaz Cruz, 1946, Tomo V, page 555).
Taking all the above circumstances together, we reach the conclusion that the
execution and the contents of a lost or destroyed holographic will may not be proved by
the bare testimony of witnesses who have seen and/or read such will.8
Under the provisions of Art. 838 of the New Civil Code, we are empowered to adopt this
opinion as a Rule of Court for the allowance of such holographic wills. We hesitate,
however, to make this Rule decisive of this controversy, simultaneously with its
promulgation. Anyway, decision of the appeal may rest on the sufficiency, rather the
insufficiency, of the evidence presented by petitioner Fausto E. Gan.
At this point, before proceeding further, it might be convenient to explain why, unlike
holographic wills, ordinary wills may be proved by testimonial evidence when lost or
destroyed. The difference lies in the nature of the wills. In the first, the only guarantee of
authenticity is the handwriting itself; in the second, the testimony of the subscribing or
instrumental witnesses (and of the notary, now). The loss of the holographic will entails
the loss of the only medium of proof; if the ordinary will is lost, the subscribing
witnesses are available to authenticate.
In the case of ordinary wills, it is quite hard to convince three witnesses (four with the
notary) deliberately to lie. And then their lies could be checked and exposed, their
whereabouts and acts on the particular day, the likelihood that they would be called by

the testator, their intimacy with the testator, etc. And if they were intimates or trusted
friends of the testator they are not likely to end themselves to any fraudulent scheme to
distort his wishes. Last but not least, they can not receive anything on account of the
will.
Whereas in the case of holographic wills, if oral testimony were admissible9 only one
man could engineer the fraud this way: after making a clever or passable imitation of
the handwriting and signature of the deceased, he may contrive to let three honest and
credible witnesses see and read the forgery; and the latter, having no interest, could
easily fall for it, and in court they would in all good faith affirm its genuineness and
authenticity. The will having been lost the forger may have purposely destroyed it in
an "accident" the oppositors have no way to expose the trick and the error, because
the document itself is not at hand. And considering that the holographic will may consist
of two or three pages, and only one of them need be signed, the substitution of the
unsigned pages, which may be the most important ones, may go undetected.
If testimonial evidence of holographic wills be permitted, one more objectionable
feature feasibility of forgery would be added to the several objections to this kind
of wills listed by Castan, Sanchez Roman and Valverde and other well-known Spanish
Commentators and teachers of Civil Law.10
One more fundamental difference: in the case of a lost will, the three subscribing
witnesses would be testifying to a fact which they saw, namely the act of the testator of
subscribing the will; whereas in the case of a lost holographic will, the witnesses would
testify as to their opinion of the handwriting which they allegedly saw, an opinion which
can not be tested in court, nor directly contradicted by the oppositors, because the
handwriting itself is not at hand.
Turning now to the evidence presented by the petitioner, we find ourselves sharing the
trial judge's disbelief. In addition to the dubious circumstances described in the
appealed decision, we find it hard to believe that the deceased should show her will
precisely to relatives who had received nothing from it: Socorro Olarte and Primitivo
Reyes. These could pester her into amending her will to give them a share, or threaten
to reveal its execution to her husband Ildefonso Yap. And this leads to another point: if
she wanted so much to conceal the will from her husband, why did she not entrust it to
her beneficiaries? Opportunity to do so was not lacking: for instance, her husband's trip
to Davao, a few days after the alleged execution of the will.
In fine, even if oral testimony were admissible to establish and probate a lost
holographic will, we think the evidence submitted by herein petitioner is so tainted with
improbabilities and inconsistencies that it fails to measure up to that "clear and distinct"
proof required by Rule 77, sec. 6.11
Wherefore, the rejection of the alleged will must be sustained.
Judgment affirmed, with costs against petitioner.

G.R. No. L-58509 December 7, 1982


IN THE MATTER OF THE PETITION TO APPROVE THE WILL OF RICARDO B.
BONILLA deceased, MARCELA RODELAS, petitioner-appellant, vs. AMPARO
ARANZA, ET AL., oppositors-appellees, ATTY. LORENZO SUMULONG, intervenor.

(4 ) The deceased did not leave any will, holographic or otherwise,


executed and attested as required by law.
The appellees likewise moved for the consolidation of the case with
another case Sp. Proc. No, 8275). Their motion was granted by the court
in an order dated April 4, 1977.

Luciano A. Joson for petitioner-appellant.


Cesar Paralejo for oppositor-appellee.

On November 13, 1978, following the consolidation of the cases, the


appellees moved again to dismiss the petition for the probate of the will.
They argued that:

RELOVA, J.:

(1) The alleged holographic was not a last will but merely an instruction
as to the management and improvement of the schools and colleges
founded by decedent Ricardo B. Bonilla; and

This case was certified to this Tribunal by the Court of Appeals for final determination
pursuant to Section 3, Rule 50 of the Rules of Court.

(2) Lost or destroyed holographic wills cannot be proved by secondary


evidence unlike ordinary wills.

As found by the Court of Appeals:


... On January 11, 1977, appellant filed a petition with the Court of First
Instance of Rizal for the probate of the holographic will of Ricardo B.
Bonilla and the issuance of letters testamentary in her favor. The
petition, docketed as Sp. Proc. No. 8432, was opposed by the appellees
Amparo Aranza Bonilla, Wilferine Bonilla Treyes Expedita Bonilla Frias
and Ephraim Bonilla on the following grounds:
(1) Appellant was estopped from claiming that the deceased left a will by
failing to produce the will within twenty days of the death of the testator
as required by Rule 75, section 2 of the Rules of Court;
(2) The alleged copy of the alleged holographic will did not contain a
disposition of property after death and was not intended to take effect
after death, and therefore it was not a will
(3) The alleged hollographic will itself,and not an alleged copy thereof,
must be produced, otherwise it would produce no effect, as held in Gam
v. Yap, 104 Phil. 509; and

Upon opposition of the appellant, the motion to dismiss was denied by


the court in its order of February 23, 1979.
The appellees then filed a motion for reconsideration on the ground that
the order was contrary to law and settled pronouncements and rulings of
the Supreme Court, to which the appellant in turn filed an opposition. On
July 23, 1979, the court set aside its order of February 23, 1979 and
dismissed the petition for the probate of the will of Ricardo B. Bonilla.
The court said:
... It is our considered opinion that once the original copy of the
holographic will is lost, a copy thereof cannot stand in lieu of the original.
In the case of Gam vs. Yap, 104 Phil. 509, 522, the Supreme Court held
that 'in the matter of holographic wills the law, it is reasonable to
suppose, regards the document itself as the material proof of
authenticity of said wills.
MOREOVER, this Court notes that the alleged holographic will was
executed on January 25, 1962 while Ricardo B. Bonilla died on May 13,

1976. In view of the lapse of more than 14 years from the time of the
execution of the will to the death of the decedent, the fact that the
original of the will could not be located shows to our mind that the
decedent had discarded before his death his allegedly missing
Holographic Will.
Appellant's motion for reconsideration was denied. Hence, an appeal to the Court of
Appeals in which it is contended that the dismissal of appellant's petition is contrary to
law and well-settled jurisprudence.
On July 7, 1980, appellees moved to forward the case to this Court on the ground that
the appeal does not involve question of fact and alleged that the trial court committed
the following assigned errors:
I. THE LOWER COURT ERRED IN HOLDING THAT A LOST
HOLOGRAPHIC WILL MAY NOT BE PROVED BY A COPY THEREOF;
II. THE LOWER COURT ERRED IN HOLDING THAT THE DECEDENT
HAS DISCARDED BEFORE HIS DEATH THE MISSING
HOLOGRAPHIC WILL;
III. THE LOWER COURT ERRED IN DISMISSING APPELLANT'S WILL.
The only question here is whether a holographic will which was lost or cannot be found
can be proved by means of a photostatic copy. Pursuant to Article 811 of the Civil Code,
probate of holographic wills is the allowance of the will by the court after its due
execution has been proved. The probate may be uncontested or not. If uncontested, at
least one Identifying witness is required and, if no witness is available, experts may be
resorted to. If contested, at least three Identifying witnesses are required. However, if
the holographic will has been lost or destroyed and no other copy is available, the will
can not be probated because the best and only evidence is the handwriting of the
testator in said will. It is necessary that there be a comparison between sample
handwritten statements of the testator and the handwritten will. But, a photostatic copy
or xerox copy of the holographic will may be allowed because comparison can be made
with the standard writings of the testator. In the case of Gam vs. Yap, 104 PHIL. 509,
the Court ruled that "the execution and the contents of a lost or destroyed holographic
will may not be proved by the bare testimony of witnesses who have seen and/or read
such will. The will itself must be presented; otherwise, it shall produce no effect. The
law regards the document itself as material proof of authenticity." But, in Footnote 8 of

said decision, it says that "Perhaps it may be proved by a photographic or photostatic


copy. Even a mimeographed or carbon copy; or by other similar means, if any, whereby
the authenticity of the handwriting of the deceased may be exhibited and tested before
the probate court," Evidently, the photostatic or xerox copy of the lost or destroyed
holographic will may be admitted because then the authenticity of the handwriting of the
deceased can be determined by the probate court.
WHEREFORE, the order of the lower court dated October 3, 1979, denying appellant's
motion for reconsideration dated August 9, 1979, of the Order dated July 23, 1979,
dismissing her petition to approve the will of the late Ricardo B. Bonilla, is hereby SET
ASIDE.
SO ORDERED.

G.R. No. L-2538

September 21, 1951

Testate Estate of the Deceased MARIANO MOLO Y LEGASPI. JUANA JUAN VDA.
DE MOLO, petitioner-appellee, vs. LUZ, GLICERIA and CORNELIO
MOLO, oppositors-appellants.
Claro M. Recto and Serafin C. Dizon for appellants.
Delgado & Flores for appellee.
BAUTISTA ANGELO, J.:
This is an appeal from an order of the Court of First Instance of Rizal admitting to
probate the last will and testament of the deceased Mariano Molo y Legaspi executed
on August 17, 1918. The oppositors-appellants brought the case on appeal to this Court
for the reason that the value of the properties involved exceeds P50,000.
Mariano Molo y Legaspi died on January 24, 1941, in the municipality of Pasay,
province of Rizal, without leaving any forced heir either in the descending or ascending
line. He was survived, however, by his wife, the herein petitioner Juana Juan Vda. de
Molo, and by his nieces and nephew, the oppositors-appellants, Luz Gliceria and
Cornelio, all surnamed Molo, who were the legitimate children of Candido Molo y
Legaspi, deceased brother of the testator. Mariano Molo y Legaspi left two wills, one
executed on August 17, 1918, (Exhibit A) and another executed on June 20, 1939.
(Exhibit I). The later will executed in 1918.
On February 7, 1941, Juana Juan Vda. de Molo, filed in the Court of First Instance of
Rizal a petition, which was docketed as special proceeding No. 8022 seeking the
probate of the will executed by the deceased on June 20, 1939. There being no
opposition, the will was probated. However, upon petition filed by the herein oppositors,
the order of the court admitting the will to probate was set aside and the case was
reopened. After hearing, at which both parties presented their evidence, the court
rendered decision denying the probate of said will on the ground that the petitioner
failed to prove that the same was executed in accordance with law.
In view of the disallowance of the will executed on June 20, 1939, the widow on
February 24, 1944, filed another petition for the probate of the will executed by the
deceased on August 17, 1918, which was docketed as special proceeding No. 56, in
the same court. Again, the same oppositors filed an opposition to the petition based on

three grounds: (1) that petitioner is now estopped from seeking the probate of the will of
1918; (2) that said will has not been executed in the manner required by law and (3)
that the will has been subsequently revoked. But before the second petition could be
heard, the battle for liberation came and the records of the case were destroyed.
Consequently, a petition for reconstitution was filed, but the same was found to be
impossible because neither petitioner nor oppositors could produce the copies required
for its reconstitution. As a result, petitioner filed a new petition on September 14, 1946,
similar to the one destroyed, to which the oppositors filed an opposition based on the
same grounds as those contained in their former opposition. Then, the case was set for
trial, and on May 28, 1948, the court issued an order admitting the will to probate
already stated in the early part of this decision. From this order the oppositors appealed
assigning six errors, to wit.
I. The probate court erred in not holding that the present petitioner voluntarily
and deliberately frustrated the probate of the will dated June 20, 1939, in special
proceeding No. 8022, in order to enable her to obtain the probate of another
alleged will of Molo dated 191.
II. The court a quo erred in not holding that the petitioner is now estopped from
seeking the probate of Molo's alleged will of 1918.
III. The lower court erred in not holding that petitioner herein has come to court
with "unclean hands" and as such is not entitled to relief.
IV. The probate court erred in not holding that Molo's alleged will of August 17,
1918 was not executed in the manner required by law.
V. The probate court erred in not holding that the alleged will of 1918 was
deliberately revoked by Molo himself.
VI. The lower court erred in not holding that Molo's will of 1918 was
subsequently revoked by the decedent's will of 1939.
In their first assignment of error, counsel for oppositors contend that the probate court
erred in not holding that the petitioner voluntarily and deliberately frustrated the probate
of the will dated June 20, 1939, in order to enable her to obtain the probate of the will
executed by the deceased on August 17, 1918, pointing out certain facts and
circumstances with their opinion indicate that petitioner connived with the witness

Canuto Perez in an effort to defeat and frustrate the probate of the 1939 will because of
her knowledge that said will intrinsically defective in that "the one and only testamentory
disposition thereof was a "disposicion captatoria". These circumstances, counsel for the
appellants contend, constitute a series of steps deliberately taken by petitioner with a
view to insuring the realization of her plan of securing the probate of the 1918 will which
she believed would better safeguard her right to inherit from the decease.
These imputations of fraud and bad faith allegedly committed in connection with special
proceedings No. 8022, now closed and terminated, are vigorously met by counsel for
petitioner who contends that to raise them in these proceedings which are entirely new
and distinct and completely independent from the other is improper and unfair as they
find no support whatsoever in any evidence submitted by the parties in this case. They
are merely based on the presumptions and conjectures not supported by any proof. For
this reason, counsel, contends, the lower court was justified in disregarding them and in
passing them sub silentio in its decision.
A careful examination of the evidence available in this case seems to justify this
contention. There is indeed no evidence which may justify the insinuation that petitioner
had deliberately intended to frustrate the probate of the 1939 will of the deceased to
enable her to seek the probate of another will other than a mere conjecture drawn from
the apparently unexpected testimony of Canuto Perez that he went out of the room to
answer an urgent call of nature when Artemio Reyes was signing the will and the failure
of petitioner later to impeach the character of said witness in spite of the opportunity
given her by the court to do so. Apart from this insufficiency of evidence, the record
discloses that this failure has been explained by petitioner when she informed the court
that she was unable to impeach the character of her witness Canuto Perez because of
her inability to find witnesses who may impeach him, and this explanation stands
uncontradicted. Whether this explanation is satisfactory or not, it is not now, for us to
determine. It is an incident that comes within the province of the former case. The
failure of petitioner to present the testimony of Artemio Reyes at the hearing has also
been explained, and it appears that petitioner has filed because his whereabouts could
not be found. Whether this is true or not is also for this Court to determine. It is likewise
within the province and function of the court in the former case. And the unfairness of
this imputation becomes more glaring when we stock of the developments that had
taken place in these proceedings which show in bold relief the true nature of the
conduct, behavior and character of the petitioner so bitterly assailed and held in
disrepute by the oppositors.

It should be recalled that the first petition for the probate of the will executed on June
20, 1939, was filed on February 7, 1941, by the petitioner. There being no opposition,
the will was probated. Subsequently, however, upon petition of the herein oppositors,
the order of the court admitting said will to probate was set aside, over the vigorous
opposition of the herein petitioner, and the case was reopened. The reopening was
ordered because of the strong opposition of the oppositors who contended that he will
had not been executed as required by law. After the evidence of both parties had been
presented, the oppositors filed an extensive memorandum wherein they reiterated their
view that the will should be denied probate. And on the strenght of this opposition, the
court disallowed the will.
If petitioner then knew that the 1939 will was inherently defective and would make the
testamentary disposition in her favor invalid and ineffective, because it is a "disposicion
captatoria", which knowledge she may easily acquire through consultation with a
lawyer, there was no need her to go through the order of filing the petition for the
probate of the will. She could accomplish her desire by merely suppressing the will or
tearing or destroying it, and then take steps leading to the probate of the will executed
in 1918. But for her conscience was clear and bade her to take the only proper step
possible under the circumstances, which is to institute the necessary proceedings for
the probate of the 1939 will. This she did and the will was admitted to probate. But then
the unexpected happened. Over her vigorous opposition, the herein appellants filed a
petition for reopening, and over her vigorous objection, the same was granted and the
case was reopened. Her motion for reconsideration was denied. Is it her fault that the
case was reopened? Is it her fault that the order admitting the will to probate was set
aside? That was a contingency which petitioner never expected. Had appellants not
filed their opposition to the probate of the will and had they limited their objection to the
intrinsic validity of said will, their plan to defeat the will and secure the intestacy of the
deceased would have perhaps been accomplished. But they failed in their strategy. If
said will was denied probate it is due to their own effort. It is now unfair to impute bad
faith petitioner simply because she exerted every effort to protect her own interest and
prevent the intestacy of the deceased to happen.
Having reached the foregoing conclusions, it is obvious that the court did not commit
the second and third errors imputed to it by the counsel for appellants. Indeed,
petitioner cannot be considered guilty or estoppel which would prevent her from
seeking the probate of the 1918 will simply because of her effort to obtain the
allowance of the 1939 will has failed considering that in both the 1918 and 1939 wills
she was in by her husband as his universal heir. Nor can she be charged with bad faith

far having done so because of her desire to prevent the intestacy of her husband. She
cannot be blamed being zealous in protecting her interest.
The next contention of appellants refers to the revocatory clause contained in 1939 will
of the deceased which was denied probate. They contend that, notwithstanding the
disallowance of said will, the revocatory clause is valid and still has the effect of
nullifying the prior of 1918.
Counsel for petitioner meets this argument by invoking the doctrine laid down in the
case of Samson vs. Naval, (41 Phil., 838). He contends that the facts involved in that
case are on all fours with the facts of this case. Hence, the doctrine is that case is here
controlling.
There is merit in this contention. We have carefully read the facts involved in the
Samson case we are indeed impressed by their striking similarity with the facts of this
case. We do not need to recite here what those facts are; it is enough to point out that
they contain many points and circumstances in common. No reason, therefore, is seen
by the doctrine laid down in that case (which we quote hereunder) should not apply and
control the present case.
A subsequent will, containing a clause revoking a previous will, having been
disallowed, for the reason that it was not executed in conformity with the
provisions of section 618 of the Code of Civil Procedure as to the making of
wills, cannot produce the effect of annulling the previous will, inasmuch as said
revocatory clause is void. (41 Phil., 838.)
Apropos of this question, counsel for oppositors make the remark that, while they do
not disagree with the soundness of the ruling laid down in the Samson case, there is
reason to abandon said ruling because it is archaic or antiquated and runs counter to
the modern trend prevailing in American jurisprudence. They maintain that said ruling is
no longer controlling but merely represents the point of view of the minority and should,
therefore, be abandoned, more so if we consider the fact that section 623 of our Code
of Civil Procedure, which governs the revocation of wills, is of American origin and as
such should follow the prevailing trend of the majority view in the United States. A long
line of authorities is cited in support of this contention. And these authorities hold the
view, that "an express revocation is immediately effective upon the execution of the
subsequent will, and does not require that it first undergo the formality of a probate
proceeding". (p. 63, appellants' brief .

While they are many cases which uphold the view entertained by counsel for
oppositors, and that view appears to be in controlling the states where the decisions
had been promulgated, however, we are reluctant to fall in line with the assertion that is
now the prevailing view in the United States. In the search we have made of American
authorities on the subject, we found ourselves in a pool of conflicting opinions perhaps
because of the peculiar provisions contained in the statutes adopted by each State in
the subject of revocation of wills. But the impression we gathered from a review and the
study of the pertinent authorities is that the doctrine laid down in the Samson case is
still a good law. On page 328 of the American Jurisprudence Vol. 57, which is a revision
Published in 1948, we found the following passages which in our opinion truly reflect
the present trend of American jurisprudence on this matter affecting the revocation of
wills:
SEC. 471. Observance of Formalities in Execution of Instrument. Ordinarily,
statutes which permit the revocation of a will by another writing provide that to
be effective as a revocation, the writing must be executed with the same
formalities which are required to be observed in the execution of a will.
Accordingly, where, under the statutes, attestation is necessary to the making of
a valid will, an unattested non testamentary writing is not effective to revoke a
prior will. It has been held that a writing fails as a revoking instrument where it is
not executed with the formalities requisite for the execution of a will, even
though it is inscribed on the will itself, although it may effect a revocation by
cancellation or obliteration of the words of the will. A testator cannot reserve to
himself the power to modify a will by a written instrument subsequently prepared
but not executed in the manner required for a will.
SEC, 472. Subsequent Unexecuted, Invalid, or Ineffective Will or Codicil. A
will which is invalid because of the incapacity of the testator, or of undue
influence can have no effect whatever as a revoking will. Moreover, a will is not
revoked by the unexecuted draft of a later one. Nor is a will revoked by a
defectively executed will or codicil, even though the latter contains a clause
expressly revoking the former will, in a jurisdiction where it is provided by a
controlling statute that no writing other than a testamentary instrument is
sufficient to revoke a will, for the simple reason that there is no revoking will.
Similarly where the statute provides that a will may be revoked by a subsequent
will or other writing executed with the same formalities as are required in the
execution of wills, a defectively executed will does not revoke a prior will, since it
cannot be said that there is a writing which complies with the statute. Moreover,
a will or codicil which, on account of the manner in which it is executed, is

sufficient to pass only personally does not affect dispositions of real estate
made by a former will, even though it may expressly purport to do so. The intent
of the testator to revoke is immaterial, if he has not complied with the statute.
(57 Am. Jur., 328, 329.)
We find the same opinion in the American Law Reports, Annotated, edited in 1939. On
page 1400, Volume 123, there appear many authorities on the "application of rules
where second will is invalid", among which a typical one is the following:
It is universally agreed that where the second will is invalid on account of not
being executed in accordance with the provisions of the statute, or where the
testator who has not sufficient mental capacity to make a will or the will is
procured through undue influence, or the such, in other words, where the
second will is really no will, it does not revoke the first will or affect it in any
manner. Mort vs. Baker University (193-5) 229 Mo. App., 632, 78 S.W. (2d),
498.
These treaties cannot be mistaken. They uphold the view on which the ruling in the
Samson case is predicated. They reflect the opinion that this ruling is sound and good
and for this reason, we see no justification for abondoning it as now suggested by
counsel for the oppositors.
It is true that our law on the matter (sec. 623, Code Civil Procedure) provides that a will
may be some will, codicil, or other writing executed as proved in case of wills" but it
cannot be said that the 1939 will should be regarded, not as a will within the meaning of
said word, but as "other writing executed as provided in the case of wills", simply
because it was denied probate. And even if it be regarded as any other writing within
the meaning of said clause, there is authority for holding that unless said writing is
admitted to probate, it cannot have the effect of revocation. (See 57 Am. Jur. pp. 329330).
But counsel for oppositors contemned that, regardless of said revocatory clause, said
will of 1918 cannot still be given effect because of the presumption that it was
deliberately revoked by the testator himself. The oppositors contend that the testator,
after executing the 1939 will, and with full knowledge of the recovatory clause
contained said will, himself deliberately destroyed the original of the 1918 will, and for
that reason the will submitted by petitioner for probate in these proceedings is only a
duplicate of said original.

There is no evidence which may directly indicate that the testator deliberately destroyed
the original of the 1918 will because of his knowledge of the revocatory clause
contained in the will he executed in 1939. The only evidence we have is that when the
first will was executed in 1918, Juan Salcedo, who prepared it, gave the original and
copies to the testator himself and apparently they remained in his possession until he
executed his second will in 1939. And when the 1939 will was denied probate on
November 29, 1943, and petitioner was asked by her attorney to look for another will,
she found the duplicate copy (Exhibit A) among the papers or files of the testator. She
did not find the original.
If it can be inferred that the testator deliberately destroyed the 1918 will because of his
knowledge of the revocatory clause of the 1939 will, and it is true that he gave a
duplicate copy thereof to his wife, the herein petitioner, the most logical step for the
testator to take is to recall said duplicate copy in order that it may likewise be
destroyed. But this was not done as shown by the fact that said duplicate copy
remained in the possession of petitioner. It is possible that because of the long lapse of
twenty-one (21) years since the first will was executed, the original of the will had been
misplaced or lost, and forgetting that there was a copy, the testator deemed it wise to
execute another will containing exactly the same testamentary dispositions. Whatever
may be the conclusion we may draw from this chain of circumstances, the stubborn fact
is that there is no direct evidence of voluntary or deliberate destruction of the first will by
the testator. This matter cannot be inference or conjectur.
Granting for the sake of argument that the earlier will was voluntarily destroyed by the
testator after the execution of the second will, which revoked the first, could there be
any doubt, under this theory, that said earlier will was destroyed by the testator in the
honest belief that it was no longer necessary because he had expressly revoked it in
his will of 1939? In other words, can we not say that the destruction of the earlier will
was but the necessary consequence of the testator's belief that the revocatory clause
contained in the subsequent will was valid and the latter would be given effect? If such
is the case, then it is our opinion that the earlier will can still be admitted to probate
under the principle of "dependent relative revocation".
This doctrine is known as that of dependent relative revocation, and is usually
applied where the testator cancels or destroys a will or executes an instrument
intended to revoke a will with a present intention to make a new testamentary
disposition as a substitute for the old, and the new disposition is not made or, if
made, fails of effect for same reason. The doctrine is n limited to the existence

of some other document, however, and has been applied where a will was
destroyed as a consequence of a mistake of law. . . . (68 C.J.P. 799).

manner required by law. We have read their testimony and we were impressed by their
readiness and sincerity. We are convinced that they told the truth.

The rule is established that where the act of destruction is connected with the
making of another will so as fairly to raise the inference that the testator meant
the revocation of the old to depend upon the efficacy of a new disposition
intended to be substituted, the revocation will be conditional and dependent
upon the efficacy of the new disposition; and if, for any reason, the new will
intended to be made as a substitute is inoperative, the revocation fails and the
original will remains in full force. (Gardner, pp. 232, 233.)

Wherefore, the order appealed from is hereby affirmed, with costs against the
appellants.

This is the doctrine of dependent relative revocation. The failure of a new


testamentary disposition upon whose validity the revocation depends, is
equivalent to the non-fulfillment of a suspensive conditions, and hence prevents
the revocation of the original will. But a mere intent to make at some time a will
in the place of that destroyed will not render the destruction conditional. It must
appear that the revocation is dependent upon the valid execution of a new will.
(1 Alexander, p. 751; Gardner, p. 253.)
We hold therefore, that even in the supposition that the destruction of the original will by
the testator could be presumed from the failure of the petitioner to produce it in court,
such destruction cannot have the effect of defeating the prior will of 1918 because of
the fact that it is founded on the mistaken belief that the will of 1939 has been validly
executed and would be given due effect. The theory on which this principle is
predicated is that the testator did not intend to die intestate. And this intention is clearly
manifest when he executed two wills on two different occasion and instituted his wife as
his universal heir. There can therefore be no mistake as to his intention of dying testate.
The remaining question to be determined refers to the sufficiency of the evidence to
prove the due execution of the will.
The will in question was attested, as required by law, by three witnesses, Lorenzo
Morales, Rufino Enriquez, and Angel Cuenca. The first two witnesses died before the
commencement of the present proceedings. So the only instrumental witness available
was Angel Cuenca and under our law and precedents, his testimony is sufficient to
prove the due execution of the will. However, petitioner presented not only the
testimony of Cuenca but placed on the witness stand Juan Salcedo, the notary public
who prepared and notarized the will upon the express desire and instruction of the
testator, The testimony of these witnesses shows that the will had been executed in the

G.R. No. L-11823

February 11, 1918

CRISTINA SAMSON, DELFINA NAVAL, and SOR CONSOLACION


EUGENIO, petitioners-appellants, vs. MONICA NAVAL, ROSA NAVAL, and
CELESTINA NAVAL, objectors-appellants.
Guillermo Lualhati for appellants.
Perfecto Gabriel for appellees.
ARAULLO, J.:
On September 20, 1915, attorney Perfecto Gabriel presented in the Court of First
Instance of the city of Manila for allowance as the will of Simeona F. Naval, who died in
said city two days previously, a document executed by her of February 13, 1915, and in
which he was appointed executor. The case was recorded as No. 13386 and, after
hearing the petition for allowance filed by said executor, it was denied on the ground
that said document was not duly executed by the deceased as her last will and
testament, inasmuch as she did not sign it in the presence of three witness and the two
witnesses did not sign it in the presence of each other. Thereafter the nieces and
legatees of the same deceased filed in the same court for allowance as her will,
another document executed by her on October 31, 1914, and, consequently, the case
was registered under another number, which was No. 13579. The petition for allowance
was opposed by Monica Naval, Rosa Naval, and Cristina Naval on the ground that the
will, the allowance of which is asked, could not be allowed, because of the existence of
another will of subsequent date, executed during her lifetime by the same Simeona F.
Naval, and because said will has been revoked by another executed subsequently by
her during her lifetime, and further, because sail will has not been executed with the
formalities required by existing laws. Trial having taken place, at which evidence was
adduced, the court on February 8, 1916, issued an order, admitting said second
document and ordering its allowance as the last will and testament o said deceased.
From said order the opponents appealed to this court and transmitted to us the
corresponding declarations. Tow of the opponents, that is, Rosa and Cristina Naval,
assigned, as errors committed by the court, the following:
1. The finding of the court that the will of October 31, 1914, has not been revoked by
that of February 13, 1915;
2. The act of the court in permitting the petitioner to institute and proceed with the
proceedings relative to the last case for the allowance of the will, No. 13579,
notwithstanding that proceedings had already been had in the other case No. 13386
and final judgment rendered therein; and
3. The act of the court in denying the motion for continuance of the trial on the
allowance of the will of October 31, 1914, which motion was presented for the sole
purpose of introducing evidence to show the falsity of the signature appearing in said
will and submitting said signature to the Bureau of Science for analysis.

The other opponent, Monica Naval, assigned, besides the first two errors already
mentioned, the finding of the court that the disallowance of the will of said deceased,
dated February 13, 1915, on the ground that is was not executed in such form that it
could transmit real and personal property, according to section 618 of the Code of Civil
Procedure, also had the effect of annulling the revocatory clause in said will.
From the evidence it appears, as we have already stated, that the trial court declared
that the first document presented by the executor of the deceased, Simeona F. Naval,
as a will executed by her on February 13, 1915, and which was the subject-matter of
case No. 13386 of said court, could not be allowed, on the ground that it was not
executed with the requisites and formalities prescribed by law. Article 739 of the Civil
Code provides that a former will is by operation of law revoked by another valid
subsequent will, if the testator does not state in the later will his desire that the former
should subsist wholly or partly. In harmony with this provision of substantive law, we
find section 623 of the Code of Civil Procedure, which provides that no will shall be
revoked, except by implication of law, otherwise than by some will, codicil, or other
writing executed as provided in case of wills.
Therefore, according to the legal provisions, in order that the will of February 13, 1915,
that is, the first document presented as the will of the deceased Simeona F. Naval,
could have the effect of revoking that which was presented afterwards by the petitioners
as executed by the same deceased on October 31, 1914, that is, on a date previous to
the execution of the first, it was necessary and indispensable that the later will, that is,
that first presented for allowance, should be perfect or valid, that it, executed as
provided by lay in case of wills.
It also appears from the record that the opponents themselves maintained that said
later will, that is, that of February 13, 1915, was not perfect, or executed as provided by
law in case of wills, and the Court of First Instance of Manila has so held in disallowing
said documents as the will of the deceased. So that it very evident that the second will
presented, that is, that of October 31, 1914, was not and could not have been revoked
by the first, and the court was not in error in so holding in the order appealed from. We
deem it unnecessary to add a single word mere or cite well-known doctrines and
opinions of jurists in support of what has already been stated.
As to the second error assigned by the opponents, we believe it sufficient to refer to
what the court below stated in the judgment appealed from. It is as follows:
The court finds no incongruency in the presentation of a prior will when another
will of subsequent date has been disallowed. Disregarding the fact that the
petitioners in this case were not those who presented the will in No. 13386, in
which the petition was presented by the same D. Perfecto Gabriel as executor, it
is proper to take into account that the object of a petition for allowance is to ask
for an order declaring that a will has been executed in accordance with the
requisites and formalities required by law. This is a question for the court to

decide and is out of the control of the party who presents the will. The allowance
or disallowance of a will by a competent court depends upon whether the
evidence adduced at the trial shows or does not show that the formalities
required by law have been complied with, and this cannot be determined in
advance, as a general rule, by the person who presents the testament. for he
has not always concurred in or seen the execution of the will.
If, therefore, the personal who presents a will and asks that if be allowed does
not secure its allowance, and he has in his possession another will, or has
information that another exists, he does not contradict himself by asking for the
allowance of the will of earlier date merely because the later will was declared
invalid by the proper court. If in this case there is any who adopts a
contradictory position, it is the respondent himself, inasmuch as in case No.
13386 he alleged, as a ground for the disallowance of the will then presented,
that it was not executed in accordance with the law, and now he maintains the
contrary, for he claims that said will revoked that which is now presented.
With respect to the third error, it is beyond doubt that the court did not commit it, for it
appears that when the examination of the witness, Cristina Samson, was finished and
the court told Attorney Lualhati, counsel for the respondents, to continue adducing his
evidence, he said he had no more proof, although he added that he would ask the court
to grant him permission to send the will of 1914 to the Bureau of Science, which petition
was objected to by the attorney for the proponents and denied by the court.
Immediately thereafter the attorney for the opponents asked for the continuance of the
trial, which was also denied by the court, after objection was made by the proponents.
The attorney for the opponents excepted to said ruling.
Therefore, the petition of said attorney for the remission of said will to the Bureau of
Science, in the terms in which it was made to the court, after ha had stated that he had
no more evidence to present, signified that he left it to the discretion of the court to
grant it or not. Furthermore, no exception was taken to the order to the order denying
this motion, and although the attorney for the opponents excepted to the order denying
the motion for continuance of the trial, such exception was completely useless and
ineffective for the purpose of alleging before this court that the trial court erred in that
respect, for said resolution, being one of those left to the discretion of the court in the
exercise of it functions, according to section 141 of the Code of Civil Procedure, it could
not be the subject of an exception, unless the court, in denying said motion, abused its
discretional power and thereby prejudiced the essential rights of the respondents,
which is not the case here.
The error which, in addition to the first two already mentioned, has been assigned by
the opponent and appellant, Monica Naval, and refers, according to her, to the court's
action in declaring that the disallowance of the will of the deceased Simeona F. Naval,
dated February 13, 1915, for the reason that it was not executed in such manner and
from that it could transmit real and personal property, according to the provisions of

section 618 of the Code of Civil Procedure, also had the effect of annulling the
revocatory clause of said will.
First of all, it is not true that the court made such statement in the terms given in said
assignment of error, that is, it is not true that the court declared that, because said will
was not executed in the form required by law in order that it may transmit real and
personal property, according to the provisions of section 618, the disallowance of said
will also had the effect of annulling the revocatory clause therein contained. In the order
appealed from there is no declaration or conclusion made in these terms. The court did
not say that the annulment of the revocatory clause in said will was the effect or
consequence of the fact that it was not allowed on the ground that it was not executed
in the form required by law in order that it may transmit real and personal property.
Referring to the construction, given by the respondent to sections 618 and 623 of the
Code of Civil Procedure, to the effect that a subsequent will may revoke a previous will,
although the later will has not been allowed by the competent court, it being sufficient
that the intention of the testator to revoke the previous will should be clearly expressed,
and that, while the requisite of allowance is necessary in order that it may transmit
property from one person to another, it is not necessary in order that it might procedure
other effects, for example, the effect of a revocatory clause, or a clause of
aknowledgment of a child, what the court declared, we repeat, was that although the
revocation of a will should have been effected, not by means of another will or codicil,
but by mans of a document, as authorized by said section 623, which document should
have the requisites and conditions fixed in section 618, the presentation of the
document to the court was necessary in order that the latter might allow it, by declaring
that it was executed with the formalities required by law for the execution of a will, and
finally concluding that, just as to, is to be proved that the requisites of section 618 have
been complied with in order that a will may be of value through its allowance, so without
such allowance the revocatory clause like the other provisions of the will, has no value
or effect except to show extraneous matters, as, for example, the acknowledgment of
natural children, of some debt or obligation. In such case, the document could produce
effect, but not as will, but simply as a written admission made by the person executing
it. And It is beyond doubt that the revocatory clause contained in a document, like the
present, which contains provisions proper of a will, as those relating to legacies and
distribution of the properties of the testator after his death as well as the appointment of
executors, is not matter extraneous to the will, but merely a part thereof, intimately
connected with it as well as with the will or wills, the revocation of which is declared in
said clause; in short, the desire of the testator declared in the revocatory clause is
related to the desire of the same testator expressed in the provisions of the testament
in which said clause is found and to that which he might have expressed in the
testaments which he may have previously executed. There is such relation between the
revocatory clause and the will which contains it, that if the will does not produce legal
effects, because it has not been executed in accordance with the provisions of the law,
neither would the revocatory clause therein produce legal effects. And if, in the present

case, the so-called will of the deceased, Simeona F. Naval, dated February 13, 1915,
was not duly executed by her as her last will and testament, ad declared by the court in
its decision of November 19, 1915, in case No. 13386, for which reason its allowance
was denied, neither may it be maintained that the revocatory clause contained in said
will is the expression of the last will of said deceased. The disallowance of the ill,
therefore, produced the effect of annulling the revocatory clause, not exactly because
said will was not executed in such from that it could transmit real and personal property,
as inaccurately alleged by the appellant, Monica Naval, to be the court's finding, upon
which said assignment of error is based, but because it was proved that said will was
not executed or signed with the formalities and requisites required by section 618 of the
Code of Civil Procedure, a cause which also produces the nullity of the same will,
according to section 634 of said law; and of course what is invalid in law can produce
no effect whatever.
If the instrument propounded as a revocation be in form a will, it must be perfect
as such, and be subscribed and attested as is required by the statute. An
instrument intended to be a will, but filing of its effect as such on account of
some imperfection in its structure or for want of due execution, cannot be set up
for the purpose of revoking a former will. (40 Cyc., p. 1177, and cases cited
therein.)
A subsequent will containing a clause revoking an earlier will must, as a general
rule, be admitted to probate before the clause of revocation can have any effect,
and the same kind, quality, and method of proof is required for the
establishment of the subsequent will as was required for the establishment of
the former will. (40 Cyc., p. 1178, and cases cited therein.)
But admitting that the will said to have been executed by the deceased Simeona F.
Naval on February 13, 1915, notwithstanding its inefficacy to transmit property for the
reason that it has not been executed, according to the provisions of said section 618 of
the Code of Civil Procedure, should be considered as executed by her in order to
express her desire, appearing in one of its clauses, to revoke and annul any previous
will of hers, as stated in clause 13, this being the argument adduced by the appellant,
Monica naval, in support of said assignment of error neither could it be maintained
that, the allowance of said will having been denied by the court on November 11, 1915,
said revocatory clause subsists and the intention expressed by the testratrix therein is
valid and legally effective, for the simple reason that, in order that a will may be revoked
by a document, it is necessary, according to the conclusive provisions of section 623 of
said procedural law, that such documents be executed according to the provisions
relating to will in section 618, and the will in question, or, according to the respondent,
the so-called document, was not executed according to the provisions of said section,
according to the express finding of the trial court in its order of November 11, 1915,
acquiesced in by the opponent herself, and which is now final and executory. Therefore,

the disallowance of said will and the declaration that it was not executed according to
the provisions of law as to wills, produced the effect of annulling said revocatory clause.
In support of the argument advanced in her brief said appellant, Monica Naval, cites the
declaration made by the Supreme Court of Massachusetts in Wallis vs. Wallis (114
Mass., 510, 512)m which, according to the appellant herself, was in the following terms:
If it be shown that a later will was duly executed and attested, containing a
clause expressly revoking former will nothing else appearing as to its contents,
it is nevertheless good as a revocation, but it can only be made available by
setting it up in opposition to the probate of the earlier will.
In the decision of said case the finding referred to be by the appellant appears not to
have been made by the Supreme Court of Massachusetts.
The syllabus of said decision says:
When a will revoking a former will is in existence, it must be established in the
Probate Court; but when it has been lost or destroyed, and its contents cannot
be sufficiently proved to admit it to probate, it may nevertheless be availed of as
a revocation in opposition to the probate of the will revoked by it.:
And in the body of the decision there is a declaration, to which the appellant must have
desired to refer in her brief, which declaration says:
If it can be proved that a later will was duly executed, attested and subscribed,
and that it contained a clause expressly revoking all former wills, but evidence
of the rest of its contents cannot be obtained, it is nevertheless a good
revocation; and it can be made available only by allowing it to be set up in
opposition to the probate of the earlier will,. . .
The facts of the case in which this decision was rendered are different from the facts of
the case at bar. That was a case concerning a will filed by one of the children of the
testatrix, Mary Wallis, as her last will, to the allowance of which another son objected,
alleging that said will had been revoked by another executed by the same deceased
subsequent to the will that was filed, and that it had been fraudulently destroyed or
taken by his brother, the proponent and his wife, or by one of them, in order to deprive
him of the rights conferred upon him by said will. Therefore, the will said to have been
subsequently executed by the testatrix and in which, according to the oppositor, the
clause revocatory of the former will appeared, was not presented by said oppositor,
while the previous will was, in the contrary, filed for allowance by the son of the
testratrix, who appeared to be favored therein, said oppositor having alleged that the
subsequent will, that is, that containing the revocatory clause, had been drawn,
subscribed and executed in accordance with the provisions of the law, a fact which he
was ready to prove just as he was ready to prove that it had been destroyed or
suppressed by the proponent, his brother and his wife, or one of them. In the case at
bar, the subsequent will containing the revocatory clause of the previous will executed

by the deceased Simeona F. Naval was presented to the court for allowance and it was
disallowed a fact which gave opportunity to the legatees of said deceased to present
a previous will executed by her on October 31, 1914, and said two wills having been
successively presented, evidence as to them was also successively adduced for their
allowance by the court.
Therefore, the declaration made by the Supreme Court of Massachusetts in Wallis vs.
Wallis (supra), to the effect that a subsequent will containing a revocatory clause of
previous wills, constitutes a valid revocation and may be used in objecting to the
allowance of the previous will, even when it is not possible to obtain proof of the
remainder of the contents of said subsequent will, refers to the case in which the latter
had been taken away, destroyed or suppressed, and it was impossible to present it for
allowance, but requires for that purpose that it be proved that said subsequent will has
been executed, attested, and subscribed in due form and that it contained, furthermore,
that revocatory clause. This is what said declaration and, in relation thereto, also what
the syllabus of the decision thereof clearly says. The court, through Chief Justice Gray,
in giving its opinion, thus began by saying:
By our law, no will can be revoked by any subsequent instrument, other than a
"will, codicil or writing, signed, attested and subscribed in the manner provided
for making a will." And when an instrument of revocation is in existence and
capable of being propounded for probate, its validity should be tried by a direct
proceeding instituted for the purpose in the Probate Court. (Loughton vs. Atkins,
1 Pick., 535.)
It results, therefore, that while perfect parity does not exist between the case decided
by the Supreme Court of Massachusetts, to which the appellant Monica Naval refers,
and that which is not before us, it is wholly unquestionable that, whether the case deals
with a subsequent will revocatory of a previous will, which may possibly be presented to
a probate court for allowance, or of a subsequent will, also revocatory of a previous will,
which could not be presented for allowance, because it has been taken or hidding, or
mislaid in order that such will may constitute a valid revocation and be utilized in the
second case, although the remaining provisions may not be proven, in opposition to the
allowance of the previous will, it is necessary to prove that it was executed, attested,
and subscribed in due form, and, of course, also that it contained a clause expressly
revoking the previous will, or, what is the same thing, that said subsequent will has
been executed according to the provisions relating to wills, as expressed in section 623
of the procedural law in force. There can be no doubt whatever that this applies when
the revocation had been made to appear in a writing or document susceptible of
presentation for allowance, like the so-called will of the deceased Simeona F. Naval,
dated February 13, 1915, and considered by said respondent and appellant as a mere
document of revocation, for, as already seen in said decision invoked by her, the
requisite as to signing, attesting, and subscribing in the form, required by law for the
execution of wills in order that it may revoke a previous will, is also required in a will as

well as in a codicil, or in a writing, and in referring to a document of revocation, it is also


expressed that its validity should be proved in a direct proceeding, instituted for the
purpose in a probate court. In the case at bar, the document, executed by the
deceased, Simeona F. Naval, as her last will and testament, dated February 13, 1915,
has been presented for allowance; it validity has been proved by means of said
procedure in the Court of Probate of Manila, and that court denied its allowance, on the
ground that the document in question had not been duly executed by the deceased, as
her last will and testament, because she did not sign in the presence of three
witnesses, and two of these witnesses did not sign in the presence of each other, or
what is the same thing, that said document has not be attested and subscribed in the
manner established by law for the execution of will, or, in other words, as provided by
law in case of wills, as stated by section 623 of said procedural law, and this resolution
was acquiesced in, as already stated, by the respondents in this case, and is, therefore,
final and executory.
In conclusions, the doctrine laid down in the decision of the Supreme Court of
Massachusetts, invoked by the appellant, Monica Naval, is in conformity with the
provision of said section 623 of our procedural law and article 739 of the Civil Code,
and the will executed by the deceased Simeona F. Naval on October 31, 1914, not
having been revoked, according to these provisions, by the will presented and alleged
as executed by the same deceased subsequently on February 13, 1915, the allowance
of which was denied by the Court of First Instance of Manila, the court below was not in
error in ordering the allowance of said will, that is, of that of October 31, 1914, as the
last will and testament of said deceased.
Wherefore, the order appealed from is affirmed, with the costs of this instance against
the appellants. So ordered.

G.R. No. L-26317

January 29, 1927

Estate of Miguel Mamuyac, deceased.


FRANCISCO GAGO, petitioner-appellant, vs. CORNELIO MAMUYAC, AMBROSIO
LARIOSA,
FELICIANA BAUZON, and CATALINA MAMUYAC, opponents-appellees.
Nicanor Tavora for appellant.
Jose Rivera for appellees.
JOHNSON, J.:
The purpose of this action was to obtain the probation of a last will and testament of
Miguel Mamuyac, who died on the 2d day of January, 1922, in the municipality of Agoo
of the Province of La Union. It appears from the record that on or about the 27th day of
July, 1918, the said Miguel Mamuyac executed a last will and testament (Exhibit A). In
the month of January, 1922, the said Francisco Gago presented a petition in the Court
of First Instance of the Province of La Union for the probation of that will. The probation
of the same was opposed by Cornelio Mamuyac, Ambrosio Lariosa, Feliciana Bauzon,
and Catalina Mamuyac (civil cause No. 1144, Province of La Union). After hearing all of
the parties the petition for the probation of said will was denied by the Honorable C. M.
Villareal on the 2d day of November, 1923, upon the ground that the deceased had on
the 16th day of April, 1919, executed a new will and testament.
On the 21st day of February, 1925, the present action was commenced. Its purpose
was to secure the probation of the said will of the 16th day of April, 1919 (Exhibit 1). To
said petition Cornelio Mamuyac, Ambrosio Lariosa, Feliciana Bauzon, and Catalina
Mamuyac presented their oppositions, alleging (a) that the said will is a copy of the
second will and testament executed by the said Miguel Mamuyac; (b) that the same
had been cancelled and revoked during the lifetime of Miguel Mamuyac and (c) that the
said will was not the last will and testament of the deceased Miguel Mamuyac.
Upon the issue thus presented, the Honorable Anastacio R. Teodoro, judge, after
hearing the respective parties, denied the probation of said will of April 16, 1919, upon
the ground that the same had been cancelled and revoked in the year 1920. Judge
Teodoro, after examining the evidence adduced, found that the following facts had been
satisfactorily proved:
That Exhibit A is a mere carbon of its original which remained in the possession
of the deceased testator Miguel Mamuyac, who revoked it before his death as
per testimony of witness Jose Fenoy, who typed the will of the testator on April
16, 1919, and Carlos Bejar, who saw on December 30, 1920, the original Exhibit
A (will of 1919) actually cancelled by the testator Miguel Mamuyac, who assured
Carlos Bejar that inasmuch as he had sold him a house and the land where the
house was built, he had to cancel it (the will of 1919), executing thereby a new
testament. Narcisa Gago in a way corroborates the testimony of Jose Fenoy,
admitting that the will executed by the deceased (Miguel Mamuyac) in 1919 was

found in the possession of father Miguel Mamuyac. The opponents have


successfully established the fact that father Miguel Mamuyac had executed in
1920 another will. The same Narcisa Gago, the sister of the deceased, who was
living in the house with him, when cross-examined by attorney for the
opponents, testified that the original Exhibit A could not be found. For the
foregoing consideration and for the reason that the original of Exhibit A has
been cancelled by the deceased father Miguel Mamuyac, the court disallows the
probate of Exhibit A for the applicant." From that order the petitioner appealed.
The appellant contends that the lower court committed an error in not finding from the
evidence that the will in question had been executed with all the formalities required by
the law; that the same had been revoked and cancelled in 1920 before his death; that
the said will was a mere carbon copy and that the oppositors were not estopped from
alleging that fact.
With reference to the said cancellation, it may be stated that there is positive proof, not
denied, which was accepted by the lower court, that will in question had been cancelled
in 1920. The law does not require any evidence of the revocation or cancellation of a
will to be preserved. It therefore becomes difficult at times to prove the revocation or
cancellation of wills. The fact that such cancellation or revocation has taken place must
either remain unproved of be inferred from evidence showing that after due search the
original will cannot be found. Where a will which cannot be found is shown to have
been in the possession of the testator, when last seen, the presumption is, in the
absence of other competent evidence, that the same was cancelled or destroyed. The
same presumption arises where it is shown that the testator had ready access to the
will and it cannot be found after his death. It will not be presumed that such will has
been destroyed by any other person without the knowledge or authority of the testator.
The force of the presumption of cancellation or revocation by the testator, while varying
greatly, being weak or strong according to the circumstances, is never conclusive, but
may be overcome by proof that the will was not destroyed by the testator with intent to
revoke it.
In view of the fat that the original will of 1919 could not be found after the death of the
testator Miguel Mamuyac and in view of the positive proof that the same had been
cancelled, we are forced to the conclusion that the conclusions of the lower court are in
accordance with the weight of the evidence. In a proceeding to probate a will the
burden of proofs is upon the proponent clearly to establish not only its execution but its
existence. Having proved its execution by the proponents, the burden is on the
contestant to show that it has been revoked. In a great majority of instances in which
wills are destroyed for the purpose of revoking them there is no witness to the act of
cancellation or destruction and all evidence of its cancellation perishes with the testator.
Copies of wills should be admitted by the courts with great caution. When it is proven,
however, by proper testimony that a will was executed in duplicate and each copy was
executed with all the formalities and requirements of the law, then the duplicate may be

admitted in evidence when it is made to appear that the original has been lost and was
not cancelled or destroyed by the testator. (Borromeo vs. Casquijo, G.R. No. L26063.)1
After a careful examination of the entire record, we are fully persuaded that the will
presented for probate had been cancelled by the testator in 1920. Therefore the
judgment appealed from is hereby affirmed. And without any finding as to costs, it is so
ordered.

G.R. No. 17857

June 12, 1922

In re will of Josefa Zalamea y Abella, deceased.


PEDRO UNSON, petitioner-appellee, vs. ANTONIO ABELLA, ET AL., opponentsappellants.
Crispin Oben for appellants.
Pedro Guevarra and Carlos Ledesma for appellee.
VILLAMOR, J.:
On July 19, 1918, Doa Josefa Zalamea y Abella, single, 60 years old, who was
residing in the municipality of Pagsanjan, Province of Laguna, executed her last will
and testament with an attached inventory of her properties, Exhibits A and A-1, in the
presence of three witnesses, who signed with her all the pages of said documents. The
testatrix died on the 6th of January, 1921, and, as the record shows, the executor
appointed in the will, Pedro Unson, filed in the court of First Instance of Laguna on the
19th of January of the same year an application for the probate of the will and the
issuance of the proper letters of administration in his favor.
To said application an opposition was presently by Antonio Abella, Ignacia Abella,
Avicencia Abella, and Santiago Vito, alleging that the supposed will of the deceased
Zalamea was not executed in conformity with the provinces of the law, inasmuch as it
was not paged correlatively in letters, nor was there any attestation clause in it, nor was
it signed by the testatrix and the witnesses in the presence of each other.
Trial having been held, the judge a quo overruled the opposition of the contestants, and
ordered the probate of the will, Exhibit A, and the inventory, Exhibit A-1, holding that
both documents contained the true and last will of the deceased Josefa Zalamea.
From the judgment of the court below, the contestants have appealed, and in their brief
they assign three errors, which, in their opinion, justify the reversal of the judgment
appealed from.
The first error assigned by the appellants as committed by the court below is its finding
to the effect that Exhibit A, said to be the will of the deceased Josefa Zalamea, was
executed with all the solemnities required by the law.
The arguments advanced by appellants' counsel in support of the first assignment of
error tend to impeach the credibility of the witnesses for the proponent, specially that of
Eugenio Zalamea. We have made a careful examination of the evidence, but have not
found anything that would justify us in disturbing the finding of the courta quo. The
attesting witnesses, Eugenio Zalamea and Gonzalo Abaya, clearly testify that together
with the other witness to the will, Pedro de Jesus, they did sign each and every page of
the will and of the inventory in the presence of each other and of the testatrix, as the
latter did likewise sign all the pages of the will and of the inventory in their presence.

In their brief the appellants intimate that one of the pages of the will was not signed by
the testatrix, nor by the witnesses on the day of the execution of the will, that is, on the
19th of July, 1918, basing their contention on the testimony of Aurelio Palileo, who says
that on one occasion Gonzalo Abaya told him that one of the pages of the will had not
been signed by the witnesses, nor by the testatrix on the day of its execution. Palileo's
testimony is entirely contradicted by Gonzalo Abaya not only in the direct, but in the
rebuttal, evidence as well. To our mind, Palileo's testimony cannot prevail over that of
the attesting witnesses, Gonzalo Avaya and Eugenio Zalamea. The appellants impeach
the credibility of Eugenio Zalamea, for having made a sworn declaration before the
justice of the peace of Santa Cruz, Laguna, before the trial of this case, to the effect
that he was really one of the witnesses to the will in question, which fact was
corroborated by himself at the trial. The appellants take Zalamea's testimony in
connection with the dismissal of a criminal case against a nephew of his, in whose
success he was interested, and infer from this fact the partiality of his testimony. We
deem this allegation of little importance to impeach the credibility of the witness
Zalamea, especially because his testimony is corroborated by the other attesting
witness. Gonzalo Abaya, and by attorney Luis Abaya, who had prepared the testament
at the instance of the testatrix. The foregoing is sufficient for us to conclude that the first
assignment of error made by the appellants is groundless.
The appellants contend that the court below erred in admitting the will to probate
notwithstanding the omission of the proponent to produce one of the attesting
witnesses.
At the trial of this case the attorneys for the proponent stated to the court that they had
necessarily to omit the testimony of Pedro de Jesus, one of the persons who appear to
have witnessed the execution of the will, for there were reasonable grounds to believe
that said witness was openly hostile to the proponent, inasmuch as since the
announcement of the trial of the petition for the probate of the will, said witness has
been in frequent communication with the contestants and their attorney, and has
refused to hold any conference with the attorneys for the proponent. In reply to this, the
attorney for the contestants, said to the court, "without discussing for the present
whether or not in view of those facts (the facts mentioned by the attorneys for the
petitioner), in the hypothesis that the same are proven, they are relieved from
producing that witness, for while it is a matter not decided, it is a recognized rule that
the fact that a witness is hostile does not justify a party to omit his testimony; without
discussing this, I say, I move that said statement be stricken out, and if the proponent
wants these facts to stand to stand in the record, let him prove them." The court a
quo ruled, saying, "there is no need."
To this ruling of the court, the attorney for the appellants did not take any exception.
In the case of Avera vs. Garcia and Rodriguez (42 Phil., 145), recently decided by this
court, in deciding the question whether a will can be admitted to probate, where
opposition is made, upon the proof of a single attesting witness, without producing or

accounting for the absence of the other two, it was said; "while it is undoubtedly true
that an uncontested will may be proved by the testimony of only one of the three
attesting witnesses, nevertheless in Cabang vs. Delfinado (34 Phil., 291), this court
declared after an elaborate examination of the American and English authorities that
when a contest is instituted, all of the attesting witnesses must be examined, if alive
and within reach of the process of the court.
In the present case no explanation was made at the trial as to why all three of
the attesting witnesses were not produced, but the probable reason is found in
the fact that, although the petition for the probate of this will had been pending
from December 21, 1917, until the date set for the hearing, which was April 5,
1919, no formal contest was entered until the very day set for the hearing; and it
is probable that the attorney for the proponent, believing in good faith that
probate would not be contested, repaired to the court with only one of the three
attesting witnesses at hand, and upon finding that the will was contested,
incautiously permitted the case to go to proof without asking for a postponement
of the trial in order that he might produce all the attesting witnesses.
Although this circumstance may explain why the three witnesses were not
produced, it does not in itself supply any basis for changing the rule expounded
in the case above referred to; and were it not for a fact now to be mentioned,
this court would probably be compelled to reverse this case on the ground that
the execution of the will had not been proved by a sufficient number of attesting
witnesses.
It appears, however, that this point was not raised by the appellant in the lower
court either upon the submission of the cause for determination in that court or
upon the occasion of the filing of the motion for a new trial. Accordingly it is
insisted for the appellee that this question cannot now be raised for t he first
time in this court. We believe this point is well taken, and the first assignment of
error must be declared not to be well taken. This exact question has been
decided by the Supreme Court of California adversely to the contention of the
appellant, and we see no reason why the same rule of practice should not be
observed by us. (Estate of McCarty, 58 Cal., 335, 337.)

In ruling upon the point above presented we do not wish to be understood as


laying down any hard and fast rule that would prove an embarrassment to this
court in the administration of justice in the future. In one way or another we are
constantly here considering aspects of cases and applying doctrines which have
escaped the attention of all persons concerned in the litigation below; and this is
necessary if this court is to contribute the part due from it in the correct decision
of the cases brought before it. What we mean to declare is that when we believe
that substantial justice has been done in the Court of First Instance, and the
point relied on for reversal in this court appears to be one which ought properly
to have been presented in that court, we will in the exercise of a sound
discretion ignore such question upon appeal; and this is the more proper when
the question relates to a defect which might have been cured in the Court of
First Instance if attention had been called to it there. In the present case, if the
appellant had raised this question in the lower court, either at the hearing or
upon a motion for a new trial, that court would have had the power, and it would
have been its duty, considering the tardy institution of the contest, to have
granted a new trial in order that all the witnesses to the will might be brought
into court. But instead of thus calling the error to the attention of the court and
his adversary, the point is first raised by the appellant in this court. We hold that
this is too late.
Properly understood, the case of Cabang vs. Delfinado, supra, contains nothing
inconsistent with the ruling we now make, for it appears from the opinion in that
case that the proponent of the will had obtained an order for a republication and
new trial for the avowed purpose of presenting the two additional attesting
witnesses who had not been previously examined, but nevertheless
subsequently failed without any apparent reason to take their testimony. Both
parties in that case were therefore fully apprised that the question of the number
of witnesses necessar to prove the will was in issue in the lower court.

In the case at bar, we do not think this question properly to have been raised at the trial,
but in the memorandum submitted by the attorney for the appellants to the trial court,
he contended that the will could not be admitted to probate because one of the
witnesses to the will was not produced, and that the voluntary non-production of this
There are at least two reasons why the appellate tribunals are disinclined to
witness raises a presumption against the pretension of the proponent. The trial court
permit certain questions to be raised for the first time in the second instance. In
found that the evidence introduced by the proponent, consisting of the testimony of the
the first place it eliminates the judicial criterion of the Court of First Instance
two attesting witnesses and the other witness who was present at the execution, and
upon the point there presented and makes the appellate court in effect a court of had charge of the preparation of the will and the inventory, Exhibits A and A-1, was
first instance with reference to that point, unless the case is remanded for a new sufficient. As announced in Cabang vs. Delfinado, supra, the general rule is that, where
trial. In the second place, it permits, if it does not encourage, attorneys to trifle
opposition is made to the probate of a will, the attesting witnesses must be produced.
with the administration of justice by concealing from the trial court and from their But there are exceptions to this rule, for instance, when a witness is dead, or cannot be
opponent the actual point upon which reliance is placed, while they are engaged served with process of the court, or his reputation for truth has been questioned or he
in other discussions more simulated than real. These considerations are, we
appears hostile to the cause of the proponent. In such cases, the will may be admitted
think, decisive.

to probate without the testimony of said witness, if, upon the other proofs adduced in
the case, the court is satisfied that the will has been duly executed. Wherefore, we find
that the non-production of the attesting witness, Pedro de Jesus, as accounted for by
the attorney for the proponent at the trial, does not render void the decree of the court a
quo, allowing the probate.
But supposing that said witness, when cited, had testified adversely to the application,
this would not by itself have change the result reached by the court a quo, for section
632 of the Code of Civil Procedure provides that a will can be admitted to probate,
notwithstanding that one or more witnesses do not remember having attested it,
provided the court is satisfied upon the evidence adduced that the will has been
executed and signed in the manner prescribed by the law.
The last error assigned by the appellants is made to consist in the probate of the
inventory, Exhibit A-1, despite the fact that this exhibit has no attestation clause in it,
and its paging is made in Arabic numerals and not in letters.
In the third paragraph of the will, reference is made to the inventory, Exhibit A-1, and at
the bottom of said will, the testatrix Josefa Zalamea says:
In witness whereof, I sign this will composed of ten folios including the page
containing the signatures and the attestation of the witnesses; I have likewise
signed the inventory attached to this will composed of ten folios in the presence
of Messrs. Gonzalo Abaya, Eugenio Zalamea, Pedro de Jesus, in this
municipality of Pagsanjan, Laguna, Philippine Islands, this 19th of July, 1918.
And the attestation clause is as follows:
The foregoing will composed of ten folios including this one whereunto we have
affixed our signatures, as well as the inventory of the properties of Doa Josefa
Zalamea y Abella, was read to Doa Josefa Zalamea y Abella, and the latter
affixed her name to the last, and each and every page of this will and inventory
composed of ten folios in our presence; and she declared this to be her last will
and testament and at her request we have affixed hereunto our respective
signatures in her presence and in the presence of each other as witnesses to
the will and the inventory this 19th of July, 1918, at Pagsanjan, Laguna, P.I.

this court. In that case the validity of the will was assailed on the ground that its folios
were paged with the letters A, B, C, etc., instead of with the letters "one," two," "three,"
etc. It was held that this way of numbering the pages of a will is in compliance with the
spirit of the law, inasmuch as either one of these methods indicates the correlation of
the pages and serves to prevent the abstraction of any of them. In the course of the
decision, we said: "It might be said that the object of the law in requiring that the paging
be made in letters is to make falsification more difficult, but it should be noted that since
all the pages of the testament are signed at the margin by the testatrix and the
witnesses, the difficulty of forging the signatures in either case remains the same. In
other words the more or less degree of facility to imitate the writing of the letters A, B,
C, etc., does not make for the easiness to forge the signatures. And as in the present
case there exists the guaranty of the authenticity of the testament, consisting in the
signatures on the left margins of the testament and the paging thereof as declared in
the attestation clause, the holding of this court in Abangan vs. Abangan (40 Phil., 476),
might as well be repeated:
"The object of the solemnities surrounding the execution of wills is to close the
door against bad faith and fraud, to avoid substitution of wills and testaments
and to guaranty their truth and authenticity. Therefore the laws on this subject
should be interpreted in such a way as to attain these primordial ends. But, on
the other hand, also one must not lose sight of the fact that it is not the object of
the law to restrain and curtail the exercise of the right to make a will. So when
an interpretation whatsoever, that adds nothing but demands more requisites
entirely unnecessary, useless, and frustrative of the testator's last will, must be
disregarded."
In that case the testament was written on one page, and the attestation clause on
another. Neither one of these pages was numbered in any way, and it was held: "In a
will consisting of two sheets the first of which contains all the testamentary dispositions
and is signed at the bottom by the testator and three witnesses, and the second
contains only the attestation clause and is signed also at the bottom by the three
witnesses it is not necessary that both sheets be further signed on their margins by the
testator and the witnesses, or be paged."

This means that, according to the particular case, the emission of paging does not
(Sgd.) GONZALO ABAYA,
necessarily render the testament invalid.
EUGENIO ZALAMEA,
PEDRO DE JESUS. The law provides that the numbering of the pages should be in letters placed on the
upper part of the sheet, but if the paging should be placed in the lower part, would the
In view of the fact that the inventory is referred to in the will as an integral part of it, we
testament be void for this sole reason? We believe not. The law also provides that the
find that the foregoing attestation clause is in compliance with section 1 of Act No.
testator and the witnesses must sign the left margin of each of the sheets of the
2645, which requires this solemnity for the validity of a will, and makes unnecessary
testament; but if they should sign on the right margin, would this fact also annul the
any other attestation clause at the end of the inventory.
testament? Evidently not. This court has already held in Avera vs. Garcia and
As to the paging of the will in Arabic numerals, instead of in letters, we adhere to the
Rodriguez (42 Phi., 145):
doctrine announced in the case of Aldaba vs. Roque (p. 378, ante), recently decided by

"It is true that the statute says that the testator and the instrumental witnesses
shall sign their names on the left margin of each and every page; and it is
undeniable that the general doctrine is to the effect that all statutory
requirements as to the execution of wills must be fully complied with. The same
execution for wills must be fully complied with. The same doctrine is also
deducible from cases heretofore decided by this court."
"Still some details at time creep into legislative enactments which are so trivial
that it would be absurd to suppose that the Legislature could have attached any
decisive importance to them. The provision to the effect that the signatures of
the testator and witnesses shall be written on the left margin of each page
rather than on the margin seems to be of this character. So far as concerns
the authentication of the will, and of every part thereof, it can make no possible
difference whether the names appear on the left or on the right margin, provided
they are on one or the other. In Craig vs. Tatlonghari (G. R. No. 12558, decided
March 23, 1918, not reported), this court declared a will void which was totally
lacking in the signatures required to be written on its several pages; and in the
case of Re Estate of Saguinsin (41 Phil., 875) a will was likewise declared void
which contained the necessary signatures on the margin of each leaf (folio), but
not in the margin of each page containing written matter."
We do not desire to intimate that the numbering in letters is a requisite of no
importance. But since its principal object is to give the correlation of the pages, we hold
that his object may be attained by writing one, two, three, etc., as well as by writing A,
B, C, etc.
We see no reason why the same rule should not be applied where the paging is in
Arabic numerals, instead of in letters, as in the inventory in question. So that, adhering
to the view taken by this court in the case of Abangan vs. Abangan, and followed
in Aldava vs. Roque, with regard to the appreciation of the solemnities of a will, we find
that the judgement appealed from should be, as is hereby, affirmed with the costs
against the appellants. So ordered.

G.R. No. L-3378

August 22, 1951

TESTATE ESTATE OF THE DECEASED DAMASA CRISOSTOMO. NAZARIO


TRILLANA, administrator-appellee, vs. CONSORCIA P. CRISOSTOMO, ET
ALS, petitioners-appellants.
Francisco R. Capistrano and Jesus T. Quiambao for petitioners and appellants.
Jose G. Generoso and Jose B. Bautista for administrator and appellee.
FERIA, J.:
This is an appeal from an order of the Court of First Instance of Bulacan denying the
appellants' petition for relief from the judgment of the said court allowing the will of
October 19, 1948, executed by the deceased Damasa Crisostomo.

relief under sec. 2, Rule 38 of the Rules of Court, and therefore it is not necessary for
us to discuss and pass upon the other propositions of the appellant.
Where a will is duly probated after publication pursuant to 630 of the Code of
Civil Procedure, the order admitting the will is, in the absence of fraud, effective
against an persons. The fact that an heir or other interested party lives so far
away as to make it impossible for such party to be present at the date appointed
for the, probate of the will does not render the order of probate void for lack of
due process. (In re Estate of Johnson, 39 Phil. 156)

Besides, even assuming without deciding, that under sec. 3 of Rule 77, the court shall
set aside a date for proving a will even without petition when it is delivered to the, court
having jurisdiction, as contended by the appellants, the lower court was right in not
The appellants, in support of their sole assignment that the lower court erred in denying setting a date for proving the will of August 16, 1948, because this will was expressly
and absolutely revoked by the will of October 19, 1948, executed by the same executrix
their petition for relief from the judgment of January 5, 1949, admitting to probate the
or deceased, which was filed for allowance on November 1, 1948, with the same Court
will of October 19, 1948, submits to this Court three propositions, to wit: (a) "The
of First Instance of Bulacan. According to the attorneys for the appellant, the will dated
judgment of January 5 was obtained through fraud," (b) "The lower court failed to
August 16, 1948, was sent together with a writing called "Manifestation" by registered
perform its legal duty to set date for proving the will of August 16, 1948;" and (c) "The
mail on October 30, 1948, from Manila to the Court of First Instance of Bulacan, by
failure to set aside a date for proving the will of August 16 with the will of October 19
Attorney Mr. Tomas V. Barnes, and said will must have been received by the Clerk of
was entirely due to the lower court's fault or negligence."
Said Court on or after November 1, 1948, the date when the subsequent will of October
In support of their proposition (a), attorneys for the appellant allege that the fraud in
19, was filed for probate. It stands to reason that if two wills are presented for
obtaining the judgment of January 5 consisted in that the proponents of the will of
allowance but one of them revoked will cannot be included in the probate of the latter
October 19 did not cause personal notice of the hearing to be made upon the legal
subsequent will, because it would be a waste of time to allow the revoked will if the
heirs of the decedent, contrary to the requirement of Rule 77, sec. 4 of the Rules of
subsequent revoking will is allowed. The revoked will may be probated and allowed
Court. We can not consider now for the first time in this appeal the question whether the
only if the subsequent revoking will is disallowed. (11. McAra vs .MacCay, L. R. 23 Ir.,
lower court (not the proponents) complied with the requirement of said sec. 4 of Rule
138; Pepper vs. Pepper, Ir. R. 5 Eq., 85; Matter of Palmer, 58 L.J. P.D. and Adm., 44;
77 of the Rules of Court, for that question has not been raised by the appellants in the
Matter of Stephens, 22 L.T. Rep., N.S. 727.) [68 C.J. 886]
court below, either in their original petition for relief of May 12, 1949 (pp. 2-8, Record on
Besides, the appellants in the present case, who merely allege in their petition for relief
Appeal), or in their motion for reconsideration dated August 27, 1949, of the order
denying their petition for relief (pp. 67-71). And there being no evidence to the contrary, that they are "nephews and nieces and therefore legal heirs of the deceased Damasa
Crisostomo," without specifying the degree of relationship they had the latter, do not
the legal presumption is that the court which probated the will of October, 19, 1948.,
complied with its duty and acted in lawful exercise of its jurisdiction in probating said will pretend that it if the will October 19, 1949, be disallowed, they will inherit the estate left
by the testatrix. They contend that said will should be probated jointly or together with
(Sec. 69 (m) (n), Rule 123 of the Rules of Court). Besides, appellee's attorney, in the
the will of August 16, 1948, and the latter be allowed instead of the former. As in her will
statement of facts in to the appellants' petition for relief, stated that "This Honorable
of October 19, 1949, as well in that of August 16, 1948, the testatrix is leaving all her
Court set its hearing [of the petition for allowance of the will of October 19, 1948] on
properties as legacies to other persons, the appellants have no interest in the probate
December 2, 1948. Copy of this order was published in "The Star Reporter",
of said wills, and they can not appeal from the judgment which allowed one of them
newspaper of general circulation in Bulacan on November 5, 12 and 19 respectively,
and the corresponding notices served by the office of the Clerk of Court, in accordance instead of the other.
with law" (pp. 25 26, Record on Appeal). And the attorneys for the petitioners-appellant
Appellants argue that they are in interested parties and therefore may appeal in the
had not denied said statement.
present case, because in the event the will of October 19 is disallowed and in its that of
August 16 is allowed, and the legacies in the latter are declared invalid or the legatees
The petitioners-appellants having failed to show that the judgment of the lower court of
incapable to inherit, the legacies will go to appellants. This argument has no merit. In
January 5, 1948, probating the will of testatrix of October 19, was obtained through
civil actions and special proceedings, unless otherwise provided by law, the interest in
fraud, the lower court did not commit any error in denying the appellant's petition for

order that a person may be a party on appeal must be material and direct, so that he
will be materially and directly benefited or injured by the court's order, decree or
judgment: and not indirect or contingent (Espinosa vs. Barrios, 40 Off. Gaz., [8 Supp.
No. 12]. p. 145). The interest claimed by the appellants is purely contingent or
dependent upon several uncertain and future events to (1) The disallowance of the will
of October 19, 1948 (2)The allowance of the will of August 16, 1948, and (3)
invalidation of certain legacies left in said will of August 16, 1948.
In view of all the foregoing, the order appealed from is affirmed with costs against the
appellants. So ordered.

G.R. No. 168156

December 6, 2006

Sometime in January 2001, the heirs of Rosendo Lasam (son of Isabel Cuntapay by
HEIRS OF ROSENDO LASAM, Represented by Rogelio Lasam and Atty. Edward P. her second husband) filed with the MTCC a complaint for unlawful detainer against
Vicenta Umengan, who was then occupying the subject lot. Vicenta Umengan is the
Llonillo, petitioners, vs. VICENTA UMENGAN, respondent.
daughter of Abdon Turingan (son of Isabel Cuntapay by her first husband).
DECISION
In their complaint, the heirs of Rosendo Lasam alleged that they are the owners of the
CALLEJO, SR., J.:
subject lot, having inherited it from their father. Rosendo Lasam was allegedly the sole
heir of the deceased Pedro Cuntapay through Isabel Cuntapay. During his lifetime,
Before the Court is the petition for review on certiorari filed by the Heirs of Rosendo
Rosendo Lasam allegedly temporarily allowed Vicenta Umengan to occupy the subject
Lasam, represented by Rogelio M. Lasam and Atty. Edward P. Llonillo, seeking the
1
lot sometime in 1955. The latter and her husband allegedly promised that they would
reversal of the Decision dated February 16, 2005 of the Court of Appeals (CA) in CAvacate the subject lot upon demand. However, despite written notice and demand by
G.R. SP No. 80032. The assailed decision reversed and set aside the decision of the
the heirs of Rosendo Lasam, Vicenta Umengan allegedly unlawfully refused to vacate
Regional Trial Court (RTC) of Tuguegarao City, Cagayan and dismissed, for lack of
the subject lot and continued to possess the same. Accordingly, the heirs of Rosendo
merit, the complaint for unlawful detainer file by the said heirs against respondent
Lasam were constrained to institute the action for ejectment.
Vicenta Umengan.
In her Answer with Counterclaim, Vicenta Umengan specifically denied the material
The RTC decision affirmed that of the Municipal Trial Court in Cities (MTCC) of the
allegations in the complaint. She countered that when Isabel Cuntapay passed away,
same city, Branch III, which had rendered judgment in favor of the heirs of Rosendo
Lasam and directed the ejectment of respondent Vicenta Umengan from the lot subject the subject lot was inherited by her six children by her first and second marriages
through intestate succession. Each of the six children allegedly had a pro indivisoshare
of litigation.
of 1/6 of the subject lot.
The present petition likewise seeks the reversal of the CA Resolution dated May 17,
It was further alleged by Vicenta Umengan that her father, Abdon Turingan, purchased
2005 denying the motion for reconsideration filed by the heirs of Rosendo Lasam.
the respective 1/6 shares in the subject lot of his siblings Maria and Sado. These
As culled from the records, the backdrop of the present case is as follows
conveyances were allegedly evidenced by the Deed of Sale dated March 3, 1975,
The lot subject of the unlawful detainer case is situated in Tuguegarao City, Cagayan. It appearing as Doc. No. 88, Page No. 36, Book No. XIV, series of 1975 of the notarial
is the eastern half portion of Lot No. 5427 and Lot No. 990. The first lot, Lot No. 5427
book of Atty. Pedro Lagui.
containing an area of 1,037 square meters, is covered by Original Certificate of Title
Prior thereto, Rufo already sold his 1/6 share in the subject lot to Vicenta Umengan and
(OCT) No. 196. The second lot, Lot No. 990 containing an area of 118 sq m, is covered
her husband as evidenced by the Deed of Sale dated June 14, 1961, appearing as
by OCT No. 1032. These lots are registered in the names of the original owners,
Doc. No. 539, Page No. 41, Book No. V, series of 1961 of the notarial book of Atty.
spouses Pedro Cuntapay and Leona Bunagan.
Pedro Lagui. Also on June 14, 1961, Abdon donated his 1/6 share in the subject lot to
In an instrument denominated as Deed of Confirmation and acknowledged before a
her daughter Vicenta Umengan as evidenced by the Deed of Donation appearing as
notary public on June 14, 1979, the heirs of the said spouses conveyed the ownership
Doc. No. 538, Page No. 41, Book No. V, series of 1961 of the notarial book of the same
of Lots Nos. 990 and 5427 in favor of their two children, Irene Cuntapay and Isabel
notary public.
Cuntapay. In another instrument entitled Partition Agreement and acknowledged before
According to Vicenta Umengan, the children of Isabel Cuntapay by her second
a notary public on December 28, 1979, it was agreed that the eastern half portion
husband (Rosendo and Trinidad Lasam) own only 2/6 portion of the subject lot. She
(subject lot) of Lots Nos. 990 and 5427 shall belong to the heirs of Isabel Cuntapay. On
thus prayed that the complaint for ejectment be dismissed and that the heirs of
the other hand, the remaining portion thereof (the west portion) shall belong to the heirs
Rosendo Lasam be ordered to pay her damages.
of Irene Cuntapay. The subject lot (eastern half portion) has an area of 554 sq m.
The MTCC rendered judgment in favor of the heirs of Rosendo Lasam and directed the
Isabel Cuntapay had four children by her first husband, Domingo Turingan, namely:
ejectment of Vicenta Umengan. In so ruling, the MTCC gave credence to the newly
Abdon, Sado (deceased), Rufo and Maria. When Domingo Turingan passed away,
discovered last will and testament (entitledTestamento Abierto) purportedly executed by
Isabel Cuntapay remarried Mariano Lasam. She had two other children by him, namely:
Isabel Cuntapay where she bequeathed the subject lot to her son, Rosendo Lasam,
Trinidad and Rosendo.
thus:

x x x my share 1/5th (one-fifth) of the Cuntapay heirs, bordered on the North by


Sr. Elia Canapi; to the South, by Calle Aguinaldo; to the East, by Calle P. Burgos
and the West, by the late Don Luis Alonso; on the property which is my share
stands a house of light materials where I presently reside; this 1/5th (one-fifth)
share of my inheritance from the Cuntapays I leave to my son Rosendo Lasam
and also the aforementioned house of light material x x x2
The MTCC reasoned that the heirs of Rosendo Lasam anchored their claim over the
subject lot on the last will and testament of Isabel Cuntapay while Vicenta Umengan
hinged hers on intestate succession and legal conveyances. Citing jurisprudence3 and
Article 10804 of the Civil Code, the MTCC opined that testacy was favored and that
intestacy should be avoided and the wishes of the testator should prevail. It observed
that the last will and testament of Isabel Cuntapay was not yet probated as required by
law; nonetheless, the institution of a probate proceeding was not barred by prescription.
With the finding that the subject lot was already bequeathed by Isabel Cuntapay to
Rosendo Lasam, the MTCC held that the siblings Abdon, Sado, Rufo and Maria
Turingan no longer had any share therein. Consequently, they could not convey to
Vicenta Umengan what they did not own. On the issue then of who was entitled to
possession of the subject lot, the MTCC ruled in favor of the heirs of Rosendo Lasam
as it found that Vicenta Umengans possession thereof was by mere tolerance. The
dispositive portion of the MTCC decision reads:
WHEREFORE, in the light of the foregoing considerations, this Court Resolve[d]
to order the EJECTMENT of VICENTA T. UMENGAN and in her place
INSTITUTE THE HEIRS OF ROSENDO LASAM.
It is further ordered the defendant shall pay the Heirs of Rosendo Lasam the
sum of P500.00 pesos representing the monthly rental of the land from August
2000 to the time this case shall have been terminated.
Ordering the defendant to pay the plaintiffs the amount of P20,000.00 attorneys
fees plus cost of this litigation.
So Ordered.5
On appeal, the RTC affirmed in toto the decision of the MTCC. The RTC echoed the
reasoning of the MTCC that the testamentary disposition of the property of Isabel
Cuntapay should be respected, and that the heirs of Rosendo Lasam have a better
right to possess the subject lot.
Undaunted, Vicenta Umengan filed an appeal with the CA. She argued that the MTCC
had no jurisdiction over the case as it involved the recovery of ownership of the subject
lot, not merely recovery of possession or unlawful detainer. She also assailed the
RTCs and the MTCCs holding that the purported Testamento Abierto of Isabel
Cuntapay prevails over Vicenta Umengans muniments of title and, consequently, the
heirs of Rosendo Lasam have a better right to the subject lot than Vicenta Umengan.

In the assailed Decision dated February 16, 2005, the CA reversed and set aside the
decision of the RTC. The appellate court preliminarily upheld the jurisdiction of the
MTCC over the subject matter as it found that the allegations in the complaint made out
a case for unlawful detainer. The heirs of Rosendo Lasam in their complaint, according
to the CA, only sought for Vicenta Umengan to vacate and surrender possession of the
subject lot. The CA also rejected the contention of the heirs of Rosendo Lasam that the
issue of ownership of the subject lot had already been settled in another case, Civil
Case No. 4917, before RTC (Branch 3) of Tuguegarao City. The CA stated that the trial
courts order dismissing the said case was not a "judgment on the merits" as to
constitute res judicata.
However, the CA declared that the RTC, as well as the MTCC, erred in ruling that, by
virtue of the purported last will and testament of Isabel Cuntapay, the heirs of Rosendo
Lasam have a better right to the subject lot over Vicenta Umengan. The CA explained
that the said last will and testament did not comply with the formal requirements of the
law on wills.6
Specifically, the CA found that the pages of the purported last will and testament were
not numbered in accordance with the law. Neither did it contain the requisite attestation
clause. Isabel Cuntapay as testator and the witnesses to the will did not affix their
respective signatures on the second page thereof. The said instrument was likewise not
acknowledged before a notary public by the testator and the witnesses. The CA even
raised doubts as to its authenticity, noting that while Isabel Cuntapay died in 1947 and
the heirs of Rosendo Lasam claimed that they discovered the same only in 1997, a
date May 19, 1956 appears on the last page of the purported will. The CA opined
that if this was the date of execution, then the will was obviously spurious. On the other
hand, if this was the date of its discovery, then the CA expressed bafflement as to why
the heirs of Rosendo Lasam, through their mother, declared in the Partition Agreement
dated December 28, 1979 that Isabel Cuntapay died intestate.
It was observed by the CA that as against these infirmities in the claim of the heirs of
Rosendo Lasam, Vicenta Umengan presented a Deed of Sale and a Deed of Donation
to justify her possession of the subject lot. The CA noted that she has also possessed
the subject property since 1955. Such prior possession, the CA held, gave Vicente
Umengan the right to remain in the subject lot until a person with a better right lawfully
ejects her. The heirs of Rosendo Lasam do not have such a better right. The CA
stressed that the ruling on the issue of physical possession does not affect the title to
the subject lot nor constitute a binding and conclusive adjudication on the merits on the
issue of ownership. The parties are not precluded from filing the appropriate action to
directly contest the ownership of or the title to the subject lot.
The decretal portion of the assailed decision of the CA reads:
WHEREFORE, premises considered, the appeal is GRANTED. The August 29,
2003 decision of the RTC, Branch 1, Tuguegarao City, Cagayan in Civil Case

No. 5924 is hereby REVERSED and SET ASIDE. Private respondents


complaint for unlawful detainer against petitioner is dismissed for lack of merit.
SO ORDERED.7
The heirs of Rosendo Lasam sought the reconsideration thereof but their motion was
denied by the CA in its Resolution dated May 17, 2005.
The heirs of Rosendo Lasam (petitioners) now come to the Court alleging that the CA
committed reversible error in setting aside the decision of the RTC, which had affirmed
that of the MTCC, and dismissing their complaint for unlawful detainer against
respondent Vicenta Umengan.
Petitioners argue that the CA erred when it held, on one hand, that the MTCC had
jurisdiction over the subject matter of the complaint as the allegations therein make out
a case for unlawful detainer but, on the other hand, proceeded to discuss the validity of
the last will and testament of Isabel Cuntapay.
Petitioners insist that respondent is holding the subject lot by mere tolerance and that
they, as the heirs of Rosendo Lasam who was the rightful owner of the subject lot, have
a better right thereto. It was allegedly error for the CA to declare the last will and
testament of Isabel Cuntapay as null and void for its non-compliance with the formal
requisites of the law on wills. The said matter cannot be resolved in an unlawful
detainer case, which only involves the issue of material or physical possession of the
disputed property. In any case, they maintain that the said will complied with the formal
requirements of the law.
It was allegedly also erroneous for the CA to consider in respondents favor the deed of
sale and deed of donation covering portions of the subject lot, when these documents
had already been passed upon by the RTC (Branch 3) of Tuguegarao City in Civil Case
No. 4917 when it dismissed the respondents complaint for partition of the subject lot.
The said order allegedly constituted res judicata and may no longer be reviewed by the
CA.
Petitioners emphasize that in an unlawful detainer case, the only issue to be resolved is
who among the parties is entitled to the physical or material possession of the property
in dispute. On this point, the MTCC held (and the same was affirmed by the RTC) that
petitioners have a better right since the "merely tolerated" possession of the respondent
had already expired upon the petitioners formal demand on her to vacate. In support of
this claim, they point to the affidavit of Heliodoro Turingan, full brother of the
respondent, attesting that the latters possession of the subject lot was by mere
tolerance of Rosendo Lasam who inherited the same from Isabel Cuntapay.

According to petitioners, respondents predecessors-in-interest from whom she derived


her claim over the subject lot by donation and sale could not have conveyed portions
thereof to her, as she had claimed, because until the present, it is still covered by OCT
Nos. 196 and 1032 under the names of Pedro and Leona Cuntapay. Their respective
estates have not been settled up to now.
It is also the contention of petitioners that the CA should have dismissed outright
respondents petition filed therewith for failure to comply with the technical requirements
of the Rules of Court. Specifically, the petition was not allegedly properly verified,
lacked statement of material dates and written explanation on why personal service
was not made.
This last contention of petitioners deserves scant consideration. The technical
requirements for filing an appeal are not sacrosanct. It has been held that while the
requirements for perfecting an appeal must be strictly followed as they are considered
indispensable interdictions against needless delays and for orderly discharge of judicial
business, the law does admit of exceptions when warranted by circumstances.8 In the
present case, the CA cannot be faulted in choosing to overlook the technical defects of
respondents appeal. After all, technicality should not be allowed to stand in the way of
equitably and completely resolving the rights and obligations of the parties.9
The Court shall now resolve the substantive issues raised by petitioners.
It is well settled that in ejectment suits, the only issue for resolution is the physical or
material possession of the property involved, independent of any claim of ownership by
any of the party litigants. However, the issue of ownership may be provisionally ruled
upon for the sole purpose of determining who is entitled to possession de facto.10
In the present case, petitioners base their claim of right to possession on the theory that
their father, Rosendo Lasam, was the sole owner of the subject lot by virtue of the
newly discovered last will and testament of Isabel Cuntapay bequeathing the same to
him. Respondent is allegedly holding the subject lot by mere tolerance of Rosendo
Lasam and, upon the petitioners formal demand on her to vacate the same,
respondents right to possess it has expired.
On the other hand, respondent hinges her claim of possession on the legal
conveyances made to her by the children of Isabel Cuntapay by her first husband,
namely, Maria, Rufo, Sado and Abdon. These conveyances were made through the
sale and donation by the said siblings of their respective portions in the subject lot to
respondent as evidenced by the pertinent deeds.
The CA correctly held that, as between the respective claims of petitioners and
respondent, the latter has a better right to possess the subject lot.
As earlier stated, petitioners rely on the last will and testament of Isabel Cuntapay that
they had allegedly newly discovered. On the basis of this instrument, the MTCC and
RTC ruled that petitioners have a better right to the possession of the subject lot

because, following the law on succession, it should be respected and should prevail
over intestate succession.

conveyances executed in her favor by the children of Isabel Cuntapay by her first
marriage.

However, contrary to the ruling of the MTCC and RTC, the purported last will and
testament of Isabel Cuntapay could not properly be relied upon to establish petitioners
right to possess the subject lot because, without having been probated, the said last will
and testament could not be the source of any right.

Contrary to the claim of petitioners, the dismissal of respondents action for partition in
Civil Case No. 4917 before the RTC (Branch 3) of Tuguegarao City does not constitute
res judicata on the matter of the validity of the said conveyances or even as to the issue
of the ownership of the subject lot. The order dismissing respondents action for
partition in Civil Case No. 4917 stated thus:

Article 838 of the Civil Code is instructive:


Art. 838. No will shall pass either real or personal property unless it is proved
and allowed in accordance with the Rules of Court.
The testator himself may, during his lifetime, petition the court having jurisdiction
for the allowance of his will. In such case, the pertinent provisions of the Rules
of Court for the allowance of wills after the testators death shall govern.
The Supreme Court shall formulate such additional Rules of Court as may be
necessary for the allowance of wills on petition of the testator.
Subject to the right of appeal, the allowance of the will, either during the lifetime
of the testator or after his death, shall be conclusive as to its due execution.
In Caiza v. Court of Appeals,11 the Court ruled that: "[a] will is essentially ambulatory;
at any time prior to the testators death, it may be changed or revoked; and until
admitted to probate, it has no effect whatever and no right can be claimed thereunder,
the law being quite explicit: No will shall pass either real or personal property unless it
is proved and allowed in accordance with the Rules of Court."12
Dr. Tolentino, an eminent authority on civil law, also explained that "[b]efore any will can
have force or validity it must be probated. To probate a will means to prove before
some officer or tribunal, vested by law with authority for that purpose, that the
instrument offered to be proved is the last will and testament of the deceased person
whose testamentary act it is alleged to be, and that it has been executed, attested and
published as required by law, and that the testator was of sound and disposing mind. It
is a proceeding to establish the validity of the will."13 Moreover, the presentation of the
will for probate is mandatory and is a matter of public policy.14
Following the above truisms, the MTCC and RTC, therefore, erroneously ruled that
petitioners have a better right to possess the subject lot on the basis of the purported
last will and testament of Isabel Cuntapay, which, to date, has not been probated.
Stated in another manner, Isabel Cuntapays last will and testament, which has not
been probated, has no effect whatever and petitioners cannot claim any right
thereunder.
Hence, the CA correctly held that, as against petitioners claim, respondent has shown
a better right of possession over the subject lot as evidenced by the deeds of

For resolution is a motion to dismiss based on defendants [referring to the


petitioners herein] affirmative defenses consisting inter alia in the discovery of a
last will and testament of Isabel Cuntapay, the original owner of the land in
dispute.
xxx
It appears, however, that the last will and testament of the late Isabel Cuntapay
has not yet been allowed in probate, hence, there is an imperative need to
petition the court for the allowance of said will to determine once and for all the
proper legitimes of legatees and devisees before any partition of the property
may be judicially adjudicated.
It is an elementary rule in law that testate proceedings take precedence over
any other action especially where the will evinces the intent of the testator to
dispose of his whole estate.
With the discovery of the will of the late Isabel Cuntapay in favor of the
defendants, the Court can order the filing of a petition for the probate of the
same by the interested party.
WHEREFORE, in light of the foregoing considerations, let the above-entitled
case be as it is hereby DISMISSED.
SO ORDERED.15
For there to be res judicata, the following elements must be present: (1) finality of the
former judgment; (2) the court which rendered it had jurisdiction over the subject matter
and the parties; (3) it must be a judgment on the merits; and (4) there must be, between
the first and second actions, identity of parties, subject matter and causes of
action.16 The third requisite, i.e., that the former judgment must be a judgment on the
merits, is not present between the action for partition and the complaint a quo for
unlawful detainer. As aptly observed by the CA:
Our reading of the Orders (dated June 16, 1997 and October 13, 1997) in Civil
Case No. 4917 reveals that the RTC, Branch 3, Tuguegarao, Cagayan,
dismissed the complaint for partition because of the discovery of the alleged last
will and testament of Isabel Cuntapay. The court did not declare respondents
[referring to the petitioners herein] the owners of the disputed property. It simply

ordered them to petition the court for the allowance of the will to determine the
proper legitimes of the heirs prior to any partition. Instead of filing the
appropriate petition for the probate of Isabel Cuntapays will, the respondents
filed the present complaint for unlawful detainer. Viewed from this perspective,
we have no doubt that the courts Orders cited by the respondents are not
"judgments on the merits" that would result in the application of the principle
of res judicata. Where the trial court merely refrained from proceeding with the
case and granted the motion to dismiss with some clarification without
conducting a trial on the merits, there is no res judicata.17
Further, it is not quite correct for petitioners to contend that the children of Isabel
Cuntapay by her first marriage could not have conveyed portions of the subject lot to
respondent, as she had claimed, because until the present, it is still covered by OCT
Nos. 196 and 1032 under the names of Pedro and Leona Cuntapay. To recall, it was
already agreed by the heirs of the said spouses in a Partition Agreement dated
December 28, 1979 that the subject lot would belong to Isabel Cuntapay. The latter
died leaving her six children by both marriages as heirs. Considering that her purported
last will and testament has, as yet, no force and effect for not having been probated,
her six children are deemed to be co-owners of the subject lot having their
respective pro indivisoshares. The conveyances made by the children of Isabel
Cuntapay by her first marriage of their respective pro indiviso shares in the subject lot
to respondent are valid because the law recognizes the substantive right of heirs to
dispose of their ideal share in the co-heirship and/co-ownership among the heirs. The
Court had expounded the principle in this wise:
This Court had the occasion to rule that there is no doubt that an heir can sell
whatever right, interest, or participation he may have in the property under
administration. This is a matter which comes under the jurisdiction of the
probate court.
The right of an heir to dispose of the decedents property, even if the same is
under administration, is based on the Civil Code provision stating that the
possession of hereditary property is deemed transmitted to the heir without
interruption and from the moment of the death of the decedent, in case the
inheritance is accepted. Where there are however, two or more heirs, the whole
estate of the decedent is, before its partition, owned in common by such heirs.
The Civil Code, under the provisions of co-ownership, further qualifies this right.
Although it is mandated that each co-owner shall have the full ownership of his
part and of the fruits and benefits pertaining thereto, and thus may alienate,
assign or mortgage it, and even substitute another person in its enjoyment, the
effect of the alienation or the mortgage, with respect to the co-owners, shall be
limited to the portion which may be allotted to him in the division upon the
termination of the co-ownership. In other words, the law does not prohibit a co-

owner from selling, alienating or mortgaging his ideal share in the property held
in common.
As early as 1942, this Court has recognized said right of an heir to dispose of
property under administration. In the case of Teves de Jakosalem vs. Rafols, et
al., it was said that the sale made by an heir of his share in an inheritance,
subject to the result of the pending administration, in no wise, stands in the way
of such administration. The Court then relied on the provision of the old Civil
Code, Article 440 and Article 399 which are still in force as Article 533 and
Article 493, respectively, in the new Civil Code. The Court also cited the words
of a noted civilist, Manresa: "Upon the death of a person, each of his heirs
becomes the undivided owner of the whole estate left with respect to the part or
portion which might be adjudicated to him, a community of ownership being thus
formed among the co-owners of the estate which remains undivided."18
Contrary to the assertion of petitioners, therefore, the conveyances made by the
children of Isabel Cuntapay by her first marriage to respondent are valid insofar as
their pro indiviso shares are concerned. Moreover, the CA justifiably held that these
conveyances, as evidenced by the deed of donation and deed of sale presented by
respondent, coupled with the fact that she has been in possession of the subject lot
since 1955, establish that respondent has a better right to possess the same as against
petitioners whose claim is largely based on Isabel Cuntapays last will and testament
which, to date, has not been probated; hence, has no force and effect and under which
no right can be claimed by petitioners. Significantly, the probative value of the other
evidence relied upon by petitioners to support their claim, which was the affidavit of
Heliodoro Turingan, was not passed upon by the MTCC and the RTC. Their respective
decisions did not even mention the same.
In conclusion, it is well to stress the CAs admonition that
x x x our ruling on the issue of physical possession does not affect title to the
property nor constitute a binding and conclusive adjudication on the merits on
the issue of ownership. The parties are not precluded from filing the appropriate
action directly contesting the ownership of or the title to the property.19
Likewise, it is therefore in this context that the CAs finding on the validity of Isabel
Cuntapays last will and testament must be considered. Such is merely a provisional
ruling thereon for the sole purpose of determining who is entitled to possession de
facto.
WHEREFORE, premises considered, the petition is DENIED. The assailed Decision
dated February 16, 2005 and the Resolution dated May 17, 2005 of the Court of
Appeals in CA-G.R. SP No. 80032 are AFFIRMED.
SO ORDERED.

G.R. No. 115925

August 15, 2003

SPOUSES RICARDO PASCUAL and CONSOLACION SIOSON, petitioners, vs.


COURT OF APPEALS and REMEDIOS S. EUGENIO-GINO, respondents.
CARPIO, J.:
The Case
This is a petition for review of the Decision1 dated 31 January 1994 of the Court of
Appeals ordering the Register of Deeds of Metro Manila, District III, to place TCT No.
(232252) 1321 in the name of respondent Remedios S. Eugenio-Gino. The Decision
ordered the Register of Deeds to cancel the names of petitioners Ricardo Pascual and
Consolacion Sioson ("petitioners") in TCT No. (232252) 1321. The Decision also
directed petitioners to pay respondent moral and exemplary damages and attorneys
fees.
The Facts
Petitioner Consolacion Sioson ("CONSOLACION") and respondent Remedios S.
Eugenio-Gino ("REMEDIOS") are the niece and granddaughter, respectively, of the late
Canuto Sioson ("CANUTO"). CANUTO and 11 other individuals, including his sister
Catalina Sioson ("CATALINA") and his brother Victoriano Sioson ("VICTORIANO"),
were co-owners of a parcel of land in Tanza, Navotas, Metro Manila. The property,
known as Lot 2 of Plan Psu 13245, had an area of 9,347 square meters and was
covered by Original Certificate of Title No. 4207 issued by the Register of Deeds of
Rizal. CATALINA, CANUTO, and VICTORIANO each owned an aliquot 10/70 share or
1,335 square meters of Lot 2.2
On 20 November 1951, CANUTO had Lot 2 surveyed and subdivided into eight lots
(Lot Nos. 2-A to 2-H) through Subdivision Plan Psd 34713 which the Director of Lands
approved on 30 May 1952. Lot No. 2-A, with an area of 670 square meters, and Lot No.
2-E, with an area of 2,000 square meters, were placed under CANUTOs name. Three
other individuals took the remaining lots.3
On 26 September 1956, CANUTO and CONSOLACION executed a Kasulatan ng
Bilihang Tuluyan4("KASULATAN"). Under the KASULATAN, CANUTO sold his 10/70

share in Lot 2 in favor of CONSOLACION forP2,250.00. The KASULATAN, notarized by


Notary Public Jose T. de los Santos of Navotas, provides:
Na ako, CANUTO SIOSON, mamamayang Pilipino, may katampatang gulang,
kasal kay Raymunda San Diego, at naninirahan sa Tanza, Navotas, Rizal, sa
bisa at pamamagitan ng kasulatang ito ay nagpapatunay at nagpapatibay:
1. Na ako ang lubos at tunay na may-ari ng 10/70 bahaging hindi hati
(10/70 porcion pro-indiviso) ng isang lagay na lupa (Lote No. 2, Plano
Psu-13245), na nasa sa nayon ng Tanza, Municipio ng Navotas,
Provincia ng Rizal, at ang descripcion o pagkakakilanlan ng nasabing
lote ay nakasaad sa Certificado Original, de Titulo No. 4207 ng Oficina
ng Registrador de Titulos ng Rizal, gaya ng sumusunod:
xxxx
2. Na dahil at alang-alang sa halagang Dalawang Libo Dalawang Daan
at Limampung Piso (P2,250.00), salaping Pilipino, na sa akin ay
ibinayad ni CONSOLACION SIOSON, kasal kay Ricardo S. Pascual,
may sapat na gulang, mamamayang Pilipino, at naninirahan sa
Dampalit, Malabon, Rizal at ang pagkakatanggap ng nasabing halaga ay
aking inaamin at pinatutunayan, ay aking ipinagbili, inilipat at isinalin, sa
pamamagitan ng bilihang tuluyan at walang pasubali a favor [sic] sa
nasabing si CONSOLACION SIOSON, sa kanyang tagapagmana at
mapaglilipatan ang lahat ng aking titulo, karapatan at kaparti na binubuo
ng 10/70 bahaging hindi hati (10/70 porcion pro-indiviso) ng loteng
descrito or tinutukoy sa itaas nito. (Emphasis supplied)
CONSOLACION immediately took possession of Lot Nos. 2-A and 2-E. She
later declared the land for taxation purposes and paid the corresponding real
estate taxes.5
On 23 October 1968, the surviving children of CANUTO, namely, Felicidad and Beatriz,
executed a joint affidavit6("JOINT AFFIDAVIT") affirming the KASULATAN in favor of
CONSOLACION. They also attested that the lots their father had sold to
CONSOLACION were Lot Nos. 2-A and 2-E of Subdivision Plan Psd 34713. The JOINT
AFFIDAVIT reads:

KAMING sina FELICIDAD SIOSON at BEATRIZ SIOSON, pawang mga Pilipino,


kapuwa may sapat na gulang at naninirahan, ang una sa Tanza, Navotas at ang
ikalawa sa Concepcion, Malabon, lalawigan ng Rizal, sa ilalim ng isang ganap
na panunumpa alinsunod sa batas, ay malayang nagsasalaysay ng mga
sumusunod:
Na kami ang mga buhay na anak na naiwan ni CANUTO SIOSON na
nagmamay-ari ng 10/70 bahaging hindi hati (10/70 porcion pro-indiviso)
ng isang lagay na lupa (Lote No. 2, plano Psu-13245), na nasa Nayon
ng Tanza, Navotas, Rizal, at ang mga palatandaan nito ay nasasaad sa
Certificado Original de Titulo No. 4207 ng Tanggapan ng Registrador de
Titulos ng Rizal;
Na sa lubos naming kaalaman, ay ipinagbili ng aming Ama na si Canuto
Sioson ang kaniyang buong bahagi na 10/70 sa nasabing Lote No. 2,
kay CONSOLACION SIOSON, may-bahay ni Ricardo S. Pascual, na
taga Dampalit, Malabon, Rizal, sa halagang P2,250.00, salaping pilipino,
noong ika 16 [sic] ng Septiembre, 1956, sa pamamagitan ng isang
KASULATAN NG BILIHANG TULUYAN na pinagtibay sa harap ng
Notario Publico Jose T. de los Santos nang pechang nabanggit, sa
Navotas, Rizal, (Doc. No. 194, Page No. 84; Book No. IV; Series of
1956);
Na ang nasabing lupa na ipinagbili ng aming Ama kay Consolacion
Sioson ni Pascual, ay nakikilala ngayong mga Lote No. 2-A at Lote 2-E
ng Plano de Subdivision Psd-34713; na pinagtibay ng Assistant Director
of Lands noong Mayo 30, 1952;
Na aming ngayong pinatitibayan ang pagka-pagbili ng bahagi ng aming
Ama kay Consolacion Sioson ni Pascual ng ngayoy nakikilalang Lote
No. 2-A at Lote No. 2-E ng Plano de Subdivision Psd-34713. (Emphasis
supplied)
On 28 October 1968, CONSOLACION registered the KASULATAN and the JOINT
AFFIDAVIT with the Office of the Register of Deeds of Rizal ("Register of Deeds").
Based on these documents, the Register of Deeds issued to CONSOLACION Transfer
Certificate of Title No. (232252) 1321 covering Lot Nos. 2-A and 2-E of Subdivision Plan
Psd 34713 with a total area of 2,670 square meters.

On 4 February 1988, REMEDIOS filed a complaint against CONSOLACION and her


spouse Ricardo Pascual in the Regional Trial Court of Malabon, Branch 165, for
"Annulment or Cancellation of Transfer Certificate [of Title] and Damages." REMEDIOS
claimed that she is the owner of Lot Nos. 2-A and 2-E because CATALINA devised
these lots to her in CATALINAs last will and testament7 ("LAST WILL") dated 29 May
1964. REMEDIOS added that CONSOLACION obtained title to these lots through
fraudulent means since the area covered by TCT (232252) 1321 is twice the size of
CANUTOs share in Lot 2. REMEDIOS prayed for the cancellation of CONSOLACIONs
title, the issuance of another title in her name, and the payment to her of damages.
Petitioners sought to dismiss the complaint on the ground of prescription. Petitioners
claimed that the basis of the action is fraud, and REMEDIOS should have filed the
action within four years from the registration of CONSOLACIONs title on 28 October
1968 and not some 19 years later on 4 February 1988. REMEDIOS opposed the
motion, claiming that she became aware of CONSOLACIONs adverse title only in
February 1987. CONSOLACION maintained that she had timely filed her complaint
within the four-year prescriptive on 4 February 1988.
In its order of 28 April 1988, the trial court denied petitioners motion to dismiss. The
trial court held that the reckoning of the prescriptive period for filing REMEDIOS
complaint is evidentiary in nature and must await the presentation of the parties
evidence during the trial. During the pre-trial stage, REMEDIOS clarified that she was
claiming only CATALINAs 10/70 share in Lot 2, or 1,335 square meters, which
constitute of the area of Lot Nos. 2-A and 2-E.8 The trial of the case then ensued.
The Ruling of the Trial Court
On 26 November 1990, the trial court rendered judgment dismissing the case and
ordering REMEDIOS to pay petitioners P10,000 as attorneys fees and the cost of suit.
The trial court held that the action filed by REMEDIOS is based on fraud, covered by
the four-year prescriptive period. The trial court also held that REMEDIOS knew of
petitioners adverse title on 19 November 1982 when REMEDIOS testified against
petitioners in an ejectment suit petitioners had filed against their tenants in Lot Nos. 2-A
and 2-E. Thus, the complaint of REMEDIOS had already prescribed when she filed it on
4 February 1988.
The trial court further ruled that REMEDIOS has no right of action against petitioners
because CATALINAs LAST WILL from which REMEDIOS claims to derive her title has
not been admitted to probate. Under Article 838 of the Civil Code, no will passes real or

personal property unless it is allowed in probate in accordance with the Rules of Court.
The dispositive portion of the trial courts decision provides:
WHEREFORE, judgment is hereby rendered in favor of the defendants and
against plaintiff, ordering:
1. The dismissal of this case;
2. The plaintiff to pay the defendants the sum of Ten Thousand
(P10,000.00) Pesos as and for attorneys fees; and

exemplary damages ofP20,000[.00] and attorneys fees of P20,000.00


and P500.00 per appearance.10
Petitioners sought reconsideration of the ruling. However, the Court of Appeals denied
their motion in its order dated 15 June 1994.
Hence, this petition.
The Issues
Petitioners allege the following assignment of errors:

3. The plaintiff to pay the costs of suit.9


REMEDIOS appealed to the Court of Appeals.
The Ruling of the Court of Appeals
On 31 January 1994, the Court of Appeals rendered judgment reversing the decision of
the trial court. The appellate court held that what REMEDIOS filed was a suit to enforce
an implied trust allegedly created in her favor when CONSOLACION fraudulently
registered her title over Lot Nos. 2-A and 2-E. Consequently, the prescriptive period for
filing the complaint is ten years, not four. The Court of Appeals counted this ten-year
period from 19 November 1982. Thus, when REMEDIOS filed her complaint on 4
February 1988, the ten-year prescriptive period had not yet expired.
The appellate court held that CATALINAs unprobated LAST WILL does not preclude
REMEDIOS from seeking reconveyance of Lot Nos. 2-A and 2-E as the LAST WILL
may subsequently be admitted to probate. The dispositive portion of the appellate
courts ruling provides:
WHEREFORE, the decision appealed from is REVERSED and SET ASIDE. The
Registry of Deeds of Rizal or Metro Manila, District III, is ordered to place
Transfer Certificate of Title No. (232252) 1321 under the name of Remedios S.
Eugenio-Gino as executor of the will of Catalina Sioson and cancel the names
of the Spouses Ricardo Pascual and Consolacion Sioson inscribed over said
title as owners of the covered lot. Defendants-appellees spouses Ricardo
Pascual and Consolacion Sioson are ordered to pay plaintiff-appellant
Remedios S. Eugenio-Gino moral damages in the amount of P50,000.00,

I. THE COURT OF APPEALS ERRED IN HOLDING THAT PRIVATE


RESPONDENTS CAUSE OF ACTION IS NOT BARRED BY PRESCRIPTION
WHICH FINDING IS MANIFESTLY CONTRARY TO LAW AND THE
APPLICABLE DECISIONS OF THIS HONORABLE COURT.
II. THE COURT OF APPEALS ERRED IN NOT HOLDING THAT PRIVATE
RESPONDENT DOES NOT HAVE ANY TITLE AND HAS UTTERLY FAILED TO
PROVE ANY TITLE TO THE LOTS INVOLVED IN THIS CASE, AND IN
ORDERING THE CANCELLATION OF THE CERTIFICATE OF TITLE OF
PETITIONERS.
III. THE COURT OF APPEALS ACTED WITH GRAVE ABUSE OF DISCRETION
AMOUNTING TO LACK OF JURISDICTION AND IN GROSS VIOLATION OF
THE RULES OF COURT IN ORDERING THE ENTIRE PROPERTY COVERED
BY TRANSFER CERTIFICATE OF TITLE NO. (232252) 1321 TO BE PLACED
IN THE NAME OF PRIVATE RESPONDENT, BECAUSE THE CLAIM OF
PRIVATE RESPONDENT IS LIMITED ONLY TO ONE-HALF (1/2) PORTION OF
THE PROPERTY, AND THE OTHER HALF THEREOF UNQUESTIONABLY
BELONGS TO PETITIONERS.
IV. THE COURT OF APPEALS ERRED IN HOLDING THAT PETITIONERS
ACTED FRAUDULENTLY AND IN BAD FAITH IN SECURING THEIR
CERTIFICATE OF TITLE TO THE PROPERTY INVOLVED IN THIS CASE, AND
IN ORDERING PETITIONERS TO PAY PRIVATE RESPONDENTS MORAL
DAMAGES, EXEMPLARY DAMAGES AND ATTORNEYS FEES.11

The pivotal questions are: (1) whether prescription bars the action filed by REMEDIOS,
and (2) whether REMEDIOS is a real party-in-interest.
The Ruling of the Court
The petition has merit.
The Action is Barred by Prescription
The trial court held that the action filed by REMEDIOS is one based on fraud.
REMEDIOS action seeks to recover real property that petitioners allegedly acquired
through fraud. Consequently, the trial court held that the action prescribes in four years
counted from REMEDIOS actual discovery of petitioners adverse title. The trial court
concluded that REMEDIOS belatedly filed her suit on 4 February 1988 because she
actually knew of petitioners adverse title since 19 November 1982.
On the other hand, the Court of Appeals held that what REMEDIOS filed was a suit to
enforce an implied trust. REMEDIOS had ten years counted from actual notice of the
breach of trust, that is, the assertion of adverse title, within which to bring her action.
The appellate court held that REMEDIOS seasonably filed her complaint on 4 February
1988 because she allegedly discovered petitioners adverse title only on 19 November
1982.
What REMEDIOS filed was an action to enforce an implied trust but the same is
already barred by prescription.
Prescriptive Period is 10 Years Counted

REMEDIOS alleges that the excess area of 1,335 meters is not part of the sale under
the KASULATAN. REMEDIOS seeks the removal of this excess area from TCT No.
(232252) 1321 that was issued to CONSOLACION. Consequently, REMEDIOS action
is for "Annulment or Cancellation of Transfer Certificate [of Title] and Damages."14
REMEDIOS action is based on an implied trust under Article 1456 since she claims
that the inclusion of the additional 1,335 square meters in TCT No. (232252) 1321 was
without basis. In effect, REMEDIOS asserts that CONSOLACION acquired the
additional 1,335 square meters through mistake or fraud and thus CONSOLACION
should be considered a trustee of an implied trust for the benefit of the rightful owner of
the property. Clearly, the applicable prescriptive period is ten years under Article 1144
and not four years under Articles 1389 and 1391.
It is now well-settled that the prescriptive period to recover property obtained by fraud
or mistake, giving rise to an implied trust under Article 145615 of the Civil Code, is ten
years pursuant to Article 1144.16 This ten-year prescriptive period begins to run from the
date the adverse party repudiates the implied trust, which repudiation takes place when
the adverse party registers the land.17
REMEDIOS filed her complaint on 4 February 1988 or more than 19 years after
CONSOLACION registered her title over Lot Nos. 2-A and 2-E on 28 October 1968.
Unquestionably, REMEDIOS filed the complaint late thus warranting its dismissal. As
the Court recently declared in Spouses Alfredo v. Spouses Borras,18
Following Caro,19 we have consistently held that an action for reconveyance based on
an implied trust prescribes in ten years. We went further by specifying the reference
point of the ten-year prescriptive period as the date of the registration of the deed or the
issuance of the title.

From Registration of Adverse Title


The Court of Appeals Reckoning of
The four-year prescriptive period relied upon by the trial court applies only if the fraud
does not give rise to an implied trust, and the action is to annul a voidable contract
under Article 139012 of the Civil Code. In such a case, the four-year prescriptive period
under Article 139113 begins to run from the time of discovery of the mistake, violence,
intimidation, undue influence or fraud.
In the present case, REMEDIOS does not seek to annul the KASULATAN. REMEDIOS
does not assail the KASULATAN as a voidable contract. In fact, REMEDIOS admits the
validity of the sale of 1,335 square meters of land under the KASULATAN. However,

Prescriptive Period from Actual Notice


of Adverse Title Not Justified
In holding that the action filed by REMEDIOS has not prescribed, the Court of Appeals
invoked this Courts ruling in Adille v. Court of Appeals.20 In Adille, the Court reckoned
the ten-year prescriptive period for enforcing implied trusts not from registration of the

adverse title but from actual notice of the adverse title by the cestui que trust. However,
the Court, in justifying its deviation from the general rule, explained:
[W]hile actions to enforce a constructive trust prescribes (sic) in ten years, reckoned
from the date of the registration of the property, we x x x are not prepared to count the
period from such date in this case. We note the petitioners sub rosa efforts to get hold
of the property exclusively for himself beginning with his fraudulent misrepresentation in
his unilateral affidavit of extrajudicial settlement that he is "the only heir and child of his
mother Feliza["] with the consequence that he was able to secure title in his name also.
(Emphasis supplied)
Such commission of specific fraudulent conduct is absent in the present case. Other
than asserting that petitioners are guilty of fraud because they secured title to Lot Nos.
2-A and 2-E with an area twice bigger than what CANUTO allegedly sold to
CONSOLACION, REMEDIOS did not present any other proof of petitioners fraudulent
conduct akin to Adille.
CONSOLACION obtained title to Lot Nos. 2-A and 2-E through the KASULATAN
executed by CANUTO and the JOINT AFFIDAVIT executed by his surviving children,
one of whom, Felicidad, is the mother of REMEDIOS. The KASULATAN referred to the
sale of CANUTOs 10/70 share in Lot 2 without specifying the area of the lot sold. The
JOINT AFFIDAVIT referred to the "Plano de Subdivision Psd-34713" without also
specifying the area of the lot sold. However, Subdivision Plan Psd 34713, as certified
by the Assistant Director of Lands on 30 May 1952, showed an area of 2,670 square
meters in the name of CANUTO. Based on these documents, the Register of Deeds
issued TCT No. (232252) 1321 to CONSOLACION covering an area of 2,670 square
meters.

At any rate, even if we apply Adille to this case, prescription still bars REMEDIOS
complaint. As executrix of CATALINAs LAST WILL, REMEDIOS submitted to the then
Court of First Instance of Caloocan in Special Proceedings Case No. C-208 the
inventory of all the property comprising CATALINAs estate, which included Lot Nos. 2-A
and 2-E. In a motion dated 7 November 1977, CONSOLACION sought the exclusion of
these lots from the inventory, invoking her title over them. REMEDIOS was served a
copy of the motion on 8 November 1977 against which she filed an opposition.
Nevertheless, the trial court overruled REMEDIOS objection. In its order of 3 January
1978, the trial court granted CONSOLACIONs motion and ordered the exclusion of Lot
Nos. 2-A and 2-E from the estate of CATALINA. REMEDIOS did not appeal from this
ruling.
REMEDIOS thus had actual notice of petitioners adverse title on 8 November 1977.
Even if, for the sake of argument, the ten-year prescriptive period begins to run upon
actual notice of the adverse title, still REMEDIOS right to file this suit has prescribed.
REMEDIOS had until 11 November 1987 within which to file her complaint. When she
did so on 4 February 1988, the prescriptive period had already lapsed.
Respondent is Not a Real Party-in-Interest
Not only does prescription bar REMEDIOS complaint. REMEDIOS is also not a real
party-in-interest who can file the complaint, as the trial court correctly ruled.
The 1997 Rules of Civil Procedure require that every action must be prosecuted or
defended in the name of the real party-in-interest who is the party who stands to benefit
or suffer from the judgment in the suit.25 If one who is not a real party-in-interest brings
the action, the suit is dismissible for lack of cause of action.26

REMEDIOS does not assail the KASULATAN or the JOINT AFFIDAVIT as fictitious or
forged. REMEDIOS even admits the authenticity of Subdivision Plan Psd 34713 as
certified by the Assistant Director of Lands.21 Moreover, REMEDIOS has not contested
petitioners claim that CANUTO doubled his share in Lot 2 by acquiring VICTORIANOs
share.22

REMEDIOS anchored her claim over Lot Nos. 2-A and 2-E (or over its one-half portion)
on the devise of these lots to her under CATALINAs LAST WILL. However, the trial
court found that the probate court did not issue any order admitting the LAST WILL to
probate. REMEDIOS does not contest this finding. Indeed, during the trial, REMEDIOS
admitted that Special Proceedings Case No. C-208 is still pending.27

Plainly, the increase in the area sold from 1,335 square meters to 2,670 square meters
is a glaring mistake. There is, however, no proof whatsoever that this increase in area
was the result of fraud. Allegations of fraud in actions to enforce implied trusts must be
proved by clear and convincing evidence.23 Adille, which is anchored on fraud,24 cannot
apply to the present case.

Article 838 of the Civil Code states that "[N]o will shall pass either real or personal
property unless it is proved and allowed in accordance with the Rules of Court." This
Court has interpreted this provision to mean, "until admitted to probate, [a will] has no
effect whatever and no right can be claimed thereunder."28 REMEDIOS anchors her
right in filing this suit on her being a devisee of CATALINAs LAST WILL. However,

since the probate court has not admitted CATALINAs LAST WILL, REMEDIOS has not
acquired any right under the LAST WILL. REMEDIOS is thus without any cause of
action either to seek reconveyance of Lot Nos. 2-A and 2-E or to enforce an implied
trust over these lots.
The appellate court tried to go around this deficiency by ordering the reconveyance of
Lot Nos. 2-A and 2-E to REMEDIOS in her capacity as executrix of CATALINAs LAST
WILL. This is inappropriate because REMEDIOS sued petitioners not in such capacity
but as the alleged owner of the disputed lots. Thus, REMEDIOS alleged in her
complaint:
3. The plaintiff is a niece and compulsory heir of the late CATALINA SIOSON who died
single and without any child of her own and who, during her lifetime, was the owner of
those two (2) parcels of land located at Tanza, Navotas, Rizal (now Metro Manila),
formerly covered by Original Certificate of Title No. 4207 of the Registry of Deeds for
the Province of Rizal, x x x.
4. The plaintiff, aside from being the compulsory heir of the deceased CATALINA
SIOSON, has sole and exclusive claim of ownership over the above-mentioned two (2)
parcels of land by virtue of a will or "Huling Habilin at Pagpapasiya" executed by
Catalina Sioson on May 19, 1964 before Notary Public Efren Y. Angeles at Navotas,
Rizal, in which document the deceased Catalina Sioson specifically and exclusively
bequeathed to the plaintiff the above-mentioned Lots 2-A and 2-E of Psd-34713
approved by the Bureau of Lands on May 30, 1952. Copy of the "Huling Habilin at
Pagpapasiya" consisting of four (4) pages is hereto attached and forms an integral part
hereof as Annex "A;"
5. Sometime on or about February, 1987, plaintiff discovered that the above-mentioned
Lots 2-A and 2-E of subdivision plan Psd-34713 are now registered or titled in the name
of the defendants under Transfer Certificate of Title No. (232252) 1321 of the Registry
of Deeds of Rizal, now Metro-Manila District III. Copy of the title is hereto attached and
forms an integral part hereof as Annex "B;"
6. Upon further inquiry and investigation, plaintiff discovered that the defendants were
able to obtain title in their name of the said parcels of land by virtue of a "Kasulatan ng
Bilihang Tuluyan" allegedly executed by Canuto Sioson on September 26, 1956 before
Notary Public Jose [T.] de los Santos of Navotas, Metro-Manila. Copy of the said
document is hereto attached and forms an integral part hereof as Annex "C;"

7. The plaintiff also discovered that although x x x the original sale did not specify the
parcels of land sold by Canuto Sioson, the defendants submitted an alleged Affidavit
executed by Felicidad Sioson and Beatriz Sioson identifying the lots sold by Canuto
Sioson to the defendants as Lots 2-A and 2-E of subdivision plan Psd-34713. Copy of
the Affidavit dated October 3, 1968 on the basis of which the present Transfer
Certificate of Title No. (232252) 1321 was issued to the defendants is hereto attached
and forms an integral part hereof as Annex "D;"
8. The defendants are clearly guilty of fraud in presenting the aforementioned Affidavit
(Annex "D") to the Register of Deeds as the basis of their claim to Lots 2-A and 2-E in
view of the fact that the parcels sold to them by Canuto Sioson, assuming there was
such a sale, were different parcels of land, Lots 2-A and 2-E being the properties of the
late Catalina Sioson who bequeathed the same to the plaintiff.
xxxx
12. Because of the defendants fraudulent actuations on this matter, plaintiff suffered
and continious [sic] to suffer moral damages arising from anxiety, shock and wounded
feelings. Defendants should also be assessed exemplary damages by way of a lesson
to deter them from again committing the fraudulent acts, or acts of similar nature, by
virtue of which they were able to obtain title to the parcels of land involved in this case x
x x.29(Emphasis supplied)
Indeed, all throughout the proceedings below and even in her Comment to this petition,
REMEDIOS continued to pursue her claim as the alleged owner of one-half of the
disputed lots.
Other Matters Raised in the Petition
The Court deems it unnecessary to pass upon the other errors petitioners assigned
concerning the award of damages and attorneys fees to REMEDIOS. Such award
assumes that REMEDIOS is a real party-in-interest and that she timely filed her
complaint. As earlier shown, this is not the case.
WHEREFORE, we GRANT the petition. The Decision of the Court of Appeals dated 31
January 1994 and its Resolution dated 15 June 1994 are SET ASIDE. The complaint
filed by respondent Remedios Eugenio-Gino, dated 2 February 1988 is DISMISSED.
SO ORDERED.

[G.R. No. 124099. October 30, 1997]

II. I give and bequeath to my wife Asuncion Oning R. Reyes the following properties to
wit:

MANUEL G. REYES, MILA G. REYES, DANILO G. REYES, LYN AGAPE, MARITES


AGAPE, ESTABANA GALOLO, and CELSA AGAPE, petitioners, vs. COURT a. All my shares of our personal properties consisting among others of jewelries, coins,
OF APPEALS AND JULIO VIVARES, respondents.
antiques, statues, tablewares, furnitures, fixtures and the building;
DECISION
TORRES, JR., J.:
Unless legally flawed, a testators intention in his last will and testament is its life
and soul which deserves reverential observance.
The controversy before us deals with such a case.
Petitioners Manuel G. Reyes, Mila G. Reyes, Danilo G. Reyes, Lyn Agape, Marites
Agape, Estebana Galolo and Celsa Agape, the oppositors in Special Proceedings No.
112 for the probate of the will of Torcuato J. Reyes, assail in this petition for review the
decision of the Court of Appeals[1] dated November 29, 1995, the dispositive portion of
which reads:
WHEREFORE, premises considered, the judgment appealed from allowing or admitting
the will of Torcuato J. Reyes to probate and directing the issuance of Letter
Testamentary in favor of petitioner Julio A. Vivares as executor without bond is
AFFIRMED but modified in that the declaration that paragraph II of the Torcuato Reyes'
last will and testament, including subparagraphs (a) and (b) are null and void for being
contrary to law is hereby SET ASIDE, said paragraphs (a) and (b) are declared
VALID. Except as above modified, the judgment appealed from is AFFIRMED.
SO ORDERED."[2]
The antecedent facts:
On January 3, 1992, Torcuato J. Reyes executed his last will and testament
declaring therein in part, to wit:
xxx

b. All my shares consisting of one half (1/2) or 50% of all the real estates I own in
common with my brother Jose, situated in Municipalities of Mambajao, Mahinog,
Guinsiliban, Sagay all in Camiguin; real estates in Lunao, Ginoong, Caamulan,
Sugbongcogon, Boloc-Boloc, Kinoguinatan, Balingoan, Sta. Ines, Caesta, Talisayan, all
in the province of Misamis Oriental.[3]
The will consisted of two pages and was signed by Torcuato Reyes in the presence
of three witnesses: Antonio Veloso, Gloria Borromeo, and Soledad Gaputan. Private
respondent Julio A. Vivares was designated the executor and in his default or
incapacity, his son Roch Alan S. Vivares.
Reyes died on May 12, 1992 and on May 21, 1992, private respondent filed a
petition for probate of the will before the Regional Trial Court of Mambajao,
Camiguin. The petitioner was set for hearing and the order was published in the
Mindanao Daily Post, a newspaper of general circulation, once a week for three
consecutive weeks. Notices were likewise sent to all the persons named in the petition.
On July 21, 1992, the recognized natural children of Torcuato Reyes with Estebana
Galolo, namely Manuel, Mila, and Danilo all surnamed Reyes, and the deceaseds
natural children with Celsa Agape, namely Lyn and Marites Agape, filed an opposition
with the following allegations: a) that the last will and testament of Reyes was not
executed and attested in accordance with the formalities of law; and b) that Asuncion
Reyes Ebarle exerted undue and improper influence upon the testator at the time of the
execution of the will. The opposition further averred that Reyes was never married to
and could never marry Asuncion Reyes, the woman he claimed to be his wife in the will,
because the latter was already married to Lupo Ebarle who was still then alive and their
marriage was never annulled. Thus Asuncion can not be a compulsory heir for her open
cohabitation with Reyes was violative of public morals.
On July 22, 1992, the trial court issued an ordering declaring that it had acquired
jurisdiction over the petition and, therefore, allowed the presentation of evidence. After

the presentation of evidence and submission of the respective memoranda, the trial
court issued its decision on April 23, 1993.
The trial court declared that the will was executed in accordance with the
formalities prescribed by law. It, however, ruled that Asuncion Reyes, based on the
testimonies of the witnesses, was never married to the deceased Reyes, and,
therefore, their relationship was an adulterous one. Thus:
The admission in the will by the testator to the illicit relationship between him and
ASUNCION REYES EBARLE who is somebody elses, wife, is further bolstered,
strengthened, and confirmed by the direct testimonies of the petitioner himself and his
two attesting witnesses during the trial.
In both cases, the common denominator is the immoral meretrecious, adulterous and
adulterous and illicit relationship existing between the testator and the devisee prior to
the death of the testator, which constituted the sole and primary consideration for the
devise or legacy, thus making the will intrinsically invalid.[4]

Petitioners contend that the findings and conclusion of the Court of Appeals was
contrary to law, public policy and evidence on record. Torcuato Reyes and Asuncion
Oning Reyes were collateral relatives up to the fourth civil degree. Witness Gloria
Borromeo testified that Oning Reyes was her cousin as her mother and the latters
father were sister and brother. They were also nieces of the late Torcuato Reyes. Thus,
the purported marriage of the deceased Reyes and Oning Reyes was void ab initio as it
was against public policy pursuant to Article 38 (1) of the Family Code. Petitioners
further alleged that Oning Reyes was already married to Lupo Ebarle at the time she
was cohabiting with the testator hence, she could never contact any valid marriage with
the latter. Petitioners argued that the testimonies of the witnesses as well as the
personal declaration of the testator, himself, were sufficient to destroy the presumption
of marriage. To further support their contention, petitioners attached a copy of the
marriage certificate of Asuncion Reyes and Lupo Ebarle.[6]
The petition is devoid of merit.

As a general rule, courts in probate proceedings are limited to pass only upon the
extrinsic validity of the will sought to be probated.[7] Thus, the court merely inquires on
The will of Reyes was admitted to probate except for paragraph II (a) and (b) of the its due execution, whether or not it complies with the formalities prescribed by law, and
will which was declared null and void for being contrary to law and morals. Hence, Julio the testamentary capacity of the testator. It does not determine nor even by implication
Vivares filed an appeal before the Court of Appeals with the allegation that the prejudge the validity or efficacy of the wills provisions. [8] The intrinsic validity is not
oppositors failed to present any competent evidence that Asuncion Reyes was legally considered since the consideration thereof usually comes only after the will has been
proved and allowed. There are, however, notable circumstances wherein the intrinsic
married to another person during the period of her cohabitation with Torcuato Reyes.
validity was first determined as when the defect of the will is apparent on its face and
On November 29, 1995, the Court of Appeals promulgated the assailed decision the probate of the will may become a useless ceremony if it is intrinsically invalid. [9] The
which affirmed the trial courts decision admitting the will for probate but the modification intrinsic validity of a will may be passed upon because practical considerations
that paragraph II including subparagraphs (a) and (b) were declared valid. The appellee demanded it as when there is preterition of heirs or the testamentary provisions are
doubtful legality.[10] Where the parties agree that the intrinsic validity be first determined,
court stated:
the probate court may also do so.[11] Parenthetically, the rule on probate is not inflexible
and absolute. Under exceptional circumstances, the probate court is not powerless to
Considering that the oppositors never showed any competent, documentary or
do what the situation constrains it to do and pass upon certain provisions of the will.[12]
otherwise during the trial to show that Asuncion Oning Reyes marriage to the testator
was inexistent or void, either because of a pre-existing marriage or adulterous
The case at bar arose from the institution of the petition for the probate of the will of
relationship, the trial court gravely erred in striking down paragraph II (a) and (b) of the
the late Torcuato Reyes. Perforce, the only issues to be settled in the said proceeding
subject Last Will and Testament, as void for being contrary to law and morals. Said
were: (1) whether or not the testator hadanimus testandi; (2) whether or not vices of
declarations are not sufficient to destroy the presumption of marriage. Nor is it enough
[5]
consent attended the execution of the will; and (3) whether or not the formalities of the
to overcome the very declaration of the testator that Asuncion Reyes is his wife.
will had been complied with. Thus, the lower court was not asked to rule upon the
Dissatisfied with the decision of the Court of Appeals, the oppositors filed this intrinsic validity or efficacy of the provisions of the will. As a result, the declaration of the
testator that Asuncion Oning Reyes was his wife did not have to be scrutinized during
petition for review.

the probate proceedings. The propriety of the institution of Oning Reyes as one of the
devisees/legatees already involved inquiry on the wills intrinsic validity and which need
not be inquired upon by the probate court.
The lower court erroneously invoked the ruling in Nepomuceno vs. Court of
Appeals (139 SCRA 206) in the instant case. In the case aforesaid, the testator himself,
acknowledged his illicit relationship with the devisee, to wit:

was inexistent or void, either because of a pre-existing marriage or adulterous


relationship, the trial court gravely erred in striking down paragraph II (a) and (b) of the
subject Last Will and Testament, as void for being contrary to law and morals. Said
declarations are not sufficient to destroy the presumption of marriage. Nor is it enough
to overcome the very declaration of the testator that Asuncion Reyes is his wife.[14]
In the elegant language of Justice Moreland written decades ago, he said-

Art. IV. That since 1952, I have been living, as man and wife, with one Sofia J.
Nepomuceno, whom I declare and avow to be entitled to my love an [sic] affection, for
all the things which she has done for me, now and in the past; that while Sofia J.
Nepomuceno has with my full knowledge and consent, did comfort and represent
myself as her own husband, in truth and in fact, as well as in the eyes of the law, I could
not bind her to me in the holy bonds of matrimony because of my aforementioned
previous marriage.

A will is the testator speaking after death. Its provisions have substantially the same
force and effect in the probate court as if the testator stood before the court in full life
making the declarations by word of mouth as they appear in the will. That was the
special purpose of the law in the creation of the instrument known as the last will and
testament. Men wished to speak after they were dead and the law, by the creation of
that instrument, permitted them to do so. xxx All doubts must be resolved in favor of the
testators having meant just what he said. (Santos vs. Manarang, 27 Phil. 209).

Thus, the very tenor of the will invalidates the legacy because the testator admitted
he was disposing of the properties to a person with whom he had been living in
concubinage.[13] To remand the case would only be a waste of time and money since
the illegality or defect was already patent. This case is different from the Nepomuceno
case. Testator Torcuato Reyes merely stated in his will that he was bequeathing some
of his personal and real properties to his wife, Asuncion Oning Reyes. There was never
an open admission of any illicit relationship. In the case of Nepomuceno, the testator
admitted that he was already previously married and that he had an adulterous
relationship with the devisee.

Petitioners tried to refute this conclusion of the Court of Appeals by presenting


belatedly a copy of the marriage certificate of Asuncion Reyes and Lupo Ebarle. Their
failure to present the said certificate before the probate court to support their position
that Asuncion Reyes had an existing marriage with Ebarle constituted a waiver and the
same evidence can no longer be entertained on appeal, much less in this petition for
review.This Court would no try the case a new or settle factual issues since its
jurisdiction is confined to resolving questions of law which have been passed upon by
the lower courts. The settled rule is that the factual findings of the appellate court will
not be disturbed unless shown to be contrary to the evidence on the record, which
petitioners have not shown in this case.[15]

We agree with the Court of Appeals that the trial court relied on uncorroborated
testimonial evidence that Asuncion Reyes was still married to another during the time
Considering the foregoing premises, we sustain the findings of the appellate court
she cohabited with the testator. The testimonies of the witnesses were merely hearsay it appearing that it did not commit a reversible error in issuing the challenged decision.
and even uncertain as to the whereabouts or existence of Lupo Ebarle, the supposed
husband of Asuncion. Thus:
ACCORDINGLY, decision appealed from dated November 29, 1995, is hereby
AFFIRMED and the instant petition for review is DENIED for lack of merit.
The foregoing testimony cannot go against the declaration of the testator that Asuncion
Oning Reyes is his wife. In Alvarado v. City Government of Tacloban (supra) the
SO ORDERED.
Supreme Court stated that the declaration of the husband is competent evidence to
show the fact of marriage.
Considering that the oppositors never showed any competent evidence, documentary
or otherwise during the trial to show that Asuncion Oning Reyes marriage to the testator

G.R. No. L-42088 May 7, 1976


ALFREDO G. BALUYUT, petitioner, vs. HON. ERNANI CRUZ PAO, ENCARNACION
LOPEZ VDA. DE BALUYUT, JOSE ESPINO and CORAZON ESPINO, respondents.
Mary Concepcion-Bautista for petitioner.

Alfredo G. Baluyut moved for the reconsideration of that order. Acting on that motion,
the lower court in its order of March 31, 1975 appointed Baluyut and Jose Espino as
special administrators.
Mrs. Baluyut in her verified amended opposition of September 2, 1975 asked that
Espino, former governor of Nueva Vizcaya and an alleged acknowledged natural child
of Sotero Baluyut, be appointed administrator should she not be named administratrix.

Santiago, Salunat and Agbayani for respondent Encarnacion Lopez Vda. de Baluyut.

AQUINO, J.:
Sotero Baluyut died in Manila on January 6, 1975 at the age of eighty-six, leaving an
estate allegedly valued at not less than two million pesos.
A few weeks later, or on February 20, his nephew, Alfredo G. Baluyut, filed in the Court
of First Instance of Quezon City a verified petition for letters of administration. He
alleged that the deceased was survived by his widow, Encarnacion Lopez, who was
mentally incapable of acting as administratrix of the decedent's estate. Alfredo surmised
that the decedent had executed a will. He prayed that he be appointed regular
administrator and in the meantime as special administrator.
The lower court in its order of February 24, 1975 appointed Alfredo G. Baluyut as
special administrator with a bond of P100,000.
Mrs. Baluyut in her verified opposition of March 8, 1975 alleged that she was unaware
that her deceased husband executed a will. She characterized as libelous the
allegation as to her mental incapacity. She prayed that she be named administratrix
and that the appointment of Alfredo G. Baluyut as special administrator be set aside.

On November 12, 1975 Mrs. Baluyut filed an urgent motion praying that she be
appointed administratrix. She reasoned out that Alfredo G. Baluyut had no more
interest in the decedent's estate because as a collateral relative he was excluded by
Espino and other supposed descendants of the deceased who had intervened in the
proceeding, and, therefore, it was not necessary to continue with the reception of his
evidence.
Alfredo G. Baluyut opposed the urgent motion. He alleged that Espino was not a
natural child of Sotero Baluyut because Espino's parents were the spouses Elino
Espino and Josefa de Guzman. Alfredo further alleged that Mrs. Baluyut was declared
an incompetent by the Juvenile and Domestic Relations Court of Quezon City in its
order of September 25, 1975 in Special Proceeding No. QC-00939 for the guardianship
of Mrs. Baluyut. That proceeding was instituted by her sisters, Cristeta Lopez Vda. de
Cuesta and Guadalupe Lopez-Viray.
At the hearing of Mrs. Baluyut's urgent motion on November 17, 1975 no oral and
documentary evidence was presented. The lower court merely examined Mrs. Baluyut
as follows:
Court: We want also to hear her testimony.
xxx xxx xxx
Atty. Salunat: We are now therefore presenting the widow,
your Honor, to take the witness stand for examination by
the court.

The lower court in its order of March 24, 1975 cancelled Baluyut's appointment as
special administrator. In that same order the lower court noted that after asking Mrs.
Baluyut a series of questions while on the witness stand, it found that she "is healthy
and mentally qualified".
xxx xxx xxx

Court to witness: Can you testify in English?-No, your


Honor, Pampango.
Q. Ilocano? A. No, your Honor.
Atty. Salunat: She can testify in Tagalog your Honor,
which comprehensible.
Court: Your remember when you were born, Mrs.
Baluyut? A. March 25, 1901.

Atty. Salunat: We will ask the Court to (be allowed to)


submit a rejoinder, your Honor.
The probate court in its order of November 27, 1975 terminated the appointments of
Espino and Alfredo G. Baluyut as special administrators and appointed Mrs. Baluyut as
regular administratrix with a bond of P20,000. The order was based on the fact that as
surviving spouse she has a preferential right to be appointed as administratrix of her
deceased husband's estate and that she is entitled to three-fourths of the conjugal
estate: one-half in her own right and one-fourth as heir of the deceased. The lower
court said it was convinced of the widow's capacity and that her "sufficient
understanding" justified her appointment.

Q. Where did you graduate? Madres Dominicas.


Q. When did you get married to Sec. Baluyut? A. I
cannot remember the date but this was in Lingayen.
Q. What church? A. A Catholic.

Letters of administration were issued to Mrs. Baluyut after she posted her bond. She
took her oath of office on November 29, 1975.
On December 13, 1975 Alfredo G. Baluyut filed against respondent Judge, Mrs. Baluyut
and the Espino spouses this special civil action of certiorari in order to set aside the
order of November 27 appointing Mrs. Baluyut as administratrix.

Court: You want to ask more questions Attorney?


Atty. Salunat: Just a few clarificatory questions, your
Honor.
Q. Do you know Gov. Espino? A. Yes.
Q. Why do you know him? A. Because he is like a son
to me.
Q. Do you know whether Gov. Espino has any
relationship with the late Don Sotero Baluyut? A. Yes,
why not.
Q. Will you please tell us what is the relationship if there
is any? A. He is his son, sir.
Atty. Salunat: I think that would be all, your Honor.
Court: Submitted?

This court issued a restraining order enjoining the respondents from enforcing the order
of November 27 and from disposing of the funds or assets of the estate in their
possession or deposited in certain banks.
The Espino's in their comment alleged that Alfredo G. Baluyut is aware that Jose
Espino was acknowledged in a notarial instrument by Sotero Baluyut as his natural
child.
Mrs. Baluyut in her comment alleged that Alfredo G. Baluyut instituted the
administration proceeding after he had failed to get from her a cheek for P500,000
belonging to the decedent's estate and that he grossly misrepresented that she was
mentally incompetent. She further alleged that the order of the Juvenile and Domestic
Relations Court declaring her an incompetent was issued in a blitzkrieg manner
because it was based on the report of Doctor Lourdes V. Lapuz which was filed in court
just one day before the order was issued.
Mrs. Baluyut's main contention is that it is the probate court and not the Juvenile and
Domestic Relations Court that should decide the issue as to her competency to act as
administratrix.

Alfredo G. Baluyut in his manifestation of February 2, 1976 disclosed that Sotero


Baluyut executed a notarial will on April 14, 1973. In that will he bequeathed to Mrs.
Baluyut his one-half share in certain conjugal assets and one-fourth of the residue of
his estate. The remaining three-fourths were bequeated to his collateral relatives
named Irene, Erlinda, Estrellita, Eliseo and Alfredo, all surnamed Baluyut, and Emerita,
Emilio and Benjamin, all surnamed Miranda. The testator designated Mrs. Baluyut as
executrix. Espino is not mentioned in that will.
In this Court's resolution of May 7, 1976 respondents' comments were treated as their
answers. The case was deemed submitted for decision.
The issue is whether the lower court acted with grave abuse of discretion in appointing
Mrs. Baluyut as administratrix.
We hold that while the probate court correctly assumed that Mrs. Baluyut as surviving
spouse enjoys preference in the granting of letters of administration (Sec. 6[a), Rule 78,
Rules of Court), it does not follow that she should be named administratrix without
conducting a full-dress hearing on her competency to discharge that trust.
Even the directive of the testator in his will designating that a certain person should act
as executor is not binding on the probate court and does not automatically entitle him to
the issuance of letters testamentary. A hearing has to be held in order to ascertain his
fitness to act as executor. He might have been fit to act as executor when the will was
executed but supervening circumstances might have rendered him unfit for that
position.
Thus, it was held that a hearing is necessary in order to determine the suitability of the
person to be appointed administrator by giving him the opportunity to prove his
qualifications and affording oppositors a chance to contest the petition (Matute vs.
Court of Appeals, L-26106, January 31, 1969, 26 SCRA 768, 791).
In this case the probate court briefly and perfunctorily interrogated Mrs. Baluyut in order
to satisfy itself on her mental capacity. The court did not give Alfredo G. Baluyut a
chance to contest her qualifications. He had squarely raised the issue as to her
competency. The probate court assumed that
Alfredo G. Baluyut had no interest in the decedent's estate. As it now turned out, he is
one of the legatees named in the decedent's alleged will.

Moreover, it is necessary to convert the proceeding in the lower court into a


testamentary proceeding. The probate of the will cannot be dispensed with and is a
matter of public policy (Art. 838, Civil Code; See. 1, Rule 75, Rules of Court; Guevara
vs. Guevara, 74 Phil. 479 and 98 Phil. 249).
After the will is probated, the prior letters of administration should be revoked and
proceedings for the issuance of letters testamentary or of administration under the will
should be conducted (Sec. 1, Rule 82, Rules of Court; Cartajena vs. Lijauco and
Zaballa, 38 Phil. 620; Rodriguez vs. De Borja, L-21993, 64 O.G. 754, 17 SCRA 418).
Whether Sotero Baluyut died testate or intestate, it is imperative in the interest of the
orderly administration of justice that a hearing be held to determine Mrs. Baluyut's
fitness to act as executrix or administratrix. Persons questioning her capacity should be
given an adequate opportunity to be heard and to present evidence.
The lower court departed from the usual course of probate procedure in summarily
appointing Mrs. Baluyut as administratrix on the assumption that Alfredo G. Baluyut
was not an interested party. That irregularity became more pronounced after Alfredo G.
Baluyut's revelation that the decedent had executed a will. He anticipated that
development when he articulated in his petition his belief that Sotero Baluyut executed
wills which should be delivered to the court for probate.
Certiorari lies when a grave abuse of discretion was patently committed by the lower
court or if the petitioner's contention is clearly tenable or when the broader interests of
justice or public policy justify the nullification of the questioned order (Manila Electric
Company and Sheriff of Quezon City vs. Hon. Enriquez and Espinosa, 110 Phil. 499,
503; Pacheco vs. Tumangday and Fernando, 108 Phil. 238; Raneses vs. Teves, L26854, March 4, 1976).
Before closing, a pending incident herein should be resolved. Alfredo G. Baluyut in his
motion of January 15, 1976 prayed that respondent Judge be enjoined from acting on
Mrs. Baluyut's motion for the appointment of Espino as special administrator. In view of
Alfredo G. Baluyut's manifestation of
April 2, 1976 that his motion had become moot, the same is hereby denied.
WHEREFORE, the lower court's order of November 27, 1975 appointing Mrs. Baluyut
as administratrix is set aside. The letters of administration granted to her are cancelled.

The probate court is directed to conduct further proceedings in consonance with the
guidelines delineated in this decision. Costs against respondent Mrs. Baluyut.
SO ORDERED.

G.R. No. L-24742 October 26, 1973


ROSA CAYETANO CUENCO, petitioners, vs. THE HONORABLE COURT OF
APPEALS, THIRD DIVISION, MANUEL CUENCO, LOURDES CUENCO,
CONCEPCION CUENCO MANGUERRA, CARMEN CUENCO, CONSUELO CUENCO
REYES, and TERESITA CUENCO GONZALEZ, respondents.
Ambrosio Padilla Law Office for petitioner.
Jalandoni and Jamir for respondents.

The aforesaid order, however, was later suspended and cancelled and a new and
modified one released on 13 March 1964, in view of the fact that the petition was to be
heard at Branch II instead of Branch I of the said Cebu court. On the same date, a third
order was further issued stating that respondent Lourdes Cuenco's petition for the
appointment of a special administrator dated 4 March 1964 was not yet ready for the
consideration of the said court, giving as reasons the following:
It will be premature for this Court to act thereon, it not having yet
regularly acquired jurisdiction to try this proceeding, the requisite
publication of the notice of hearing not yet having been complied with.
Moreover, copies of the petition have not been served on all of the heirs
specified in the basic petition for the issuance of letters of
administration. 2

TEEHANKEE, J.:
Petition for certiorari to review the decision of respondent Court of Appeals in CA-G.R.
No. 34104-R, promulgated 21 November 1964, and its subsequent Resolution
promulgated 8 July 1964 denying petitioner's Motion for Reconsideration.
The pertinent facts which gave rise to the herein petition follow:
On 25 February 1964 Senator Mariano Jesus Cuenco died at the Manila Doctors'
Hospital, Manila. He was survived by his widow, the herein petitioner, and their two (2)
minor sons, Mariano Jesus, Jr. and Jesus Salvador, both surnamed Cuenco, all
residing at 69 Pi y Margal St., Sta. Mesa Heights, Quezon City, and by his children of
the first marriage, respondents herein, namely, Manuel Cuenco, Lourdes Cuenco,
Concepcion Cuenco Manguera, Carmen Cuenco, Consuelo Cuenco Reyes and
Teresita Cuenco Gonzales, all of legal age and residing in Cebu.
On 5 March 1964, (the 9th day after the death of the late Senator) 1 respondent Lourdes
Cuenco filed a Petition for Letters of Administration with the court of first instance of
Cebu (Sp. Proc. No. 2433-R), alleging among other things, that the late senator
died intestate in Manila on 25 February 1964; that he was a resident of Cebu at the
time of his death; and that he left real and personal properties in Cebu and Quezon
City. On the same date, the Cebu court issued an order setting the petition for hearing
on 10 April 1964, directing that due notice be given to all the heirs and interested
persons, and ordering the requisite publication thereof at LA PRENSA, a newspaper of
general circulation in the City and Province of Cebu.

In the meantime, or specifically on 12 March 1964, (a week after the filing of the Cebu
petition) herein petitioner Rosa Cayetano Cuenco filed a petition with the court of first
instance of Rizal (Quezon City) for the probate of the deceased's last will and
testament and for the issuance of letters testamentary in her favor, as the surviving
widow and executrix in the said last will and testament. The said proceeding was
docketed as Special Proceeding No. Q-7898.
Having learned of the intestate proceeding in the Cebu court, petitioner Rosa Cayetano
Cuenco filed in said Cebu court an Opposition and Motion to Dismiss, dated 30 March
1964, as well as an Opposition to Petition for Appointment of Special Administrator,
dated 8 April 1964. On 10 April 1964, the Cebu court issued an order holding in
abeyance its resolution on petitioner's motion to dismiss "until after the Court of First
Instance of Quezon City shall have acted on the petition for probate of that document
purporting to be the last will and testament of the deceased Don Mariano Jesus
Cuenco." 3 Such order of the Cebu court deferring to the probate proceedings in the
Quezon City court was neither excepted to nor sought by respondents to be
reconsidered or set aside by the Cebu court nor did they challenge the same
by certiorari or prohibition proceedings in the appellate courts.
Instead, respondents filed in the Quezon City court an Opposition and Motion to
Dismiss, dated 10 April 1964,opposing probate of the will and assailing the jurisdiction
of the said Quezon City court to entertain petitioner's petition for probate and for
appointment as executrix in Sp. Proc. No. Q-7898 in view of the alleged exclusive
jurisdiction vested by her petition in the Cebu court in Sp. Proc. No. 2433-R. Said

respondent prayed that Sp. Proc. No. Q-7898 be dismissed for lack of
jurisdiction and/or improper venue.
In its order of 11 April 1964, the Quezon City court denied the motion to dismiss, giving
as a principal reason the "precedence of probate proceeding over an intestate
proceeding." 4 The said court further found in said order that the residence of the late
senator at the time of his death was at No. 69 Pi y Margal, Sta. Mesa Heights, Quezon
City. The pertinent portion of said order follows:
On the question of residence of the decedent, paragraph 5 of the
opposition and motion to dismiss reads as follows: "that since the
decedent Don Mariano Jesus Cuenco was a resident of the City of Cebu
at the time of his death, the aforesaid petition filed by Rosa Cayetano
Cuenco on 12 March 1964 was not filed with the proper Court (wrong
venue) in view of the provisions of Section 1 of Rule 73 of the New Rules
of Court ...". From the aforequoted allegation, the Court is made to
understand that the oppositors do not mean to say that the decedent
being a resident of Cebu City when he died, the intestate proceedings in
Cebu City should prevail over the probate proceedings in Quezon City,
because as stated above the probate of the will should take precedence,
but that the probate proceedings should be filed in the Cebu City Court
of First Instance. If the last proposition is the desire of the oppositors as
understood by this Court, that could not also be entertained as proper
because paragraph 1 of the petition for the probate of the will indicates
that Don Mariano Jesus Cuenco at the time of his death was a resident
of Quezon City at 69 Pi y Margal. Annex A (Last Will and Testament of
Mariano Jesus Cuenco) of the petition for probate of the will shows that
the decedent at the time when he executed his Last Will clearly stated
that he is a resident of 69 Pi y Margal, Sta. Mesa Heights, Quezon City,
and also of the City of Cebu. He made the former as his first choice and
the latter as his second choice of residence." If a party has two
residences, the one will be deemed or presumed to his domicile which
he himself selects or considers to be his home or which appears to be
the center of his affairs. The petitioner, in thus filing the instant petition
before this Court, follows the first choice of residence of the decedent
and once this court acquires jurisdiction of the probate proceeding it is to
the exclusion of all others. 5

Respondent Lourdes Cuenco's motion for reconsideration of the Quezon City court's
said order of 11 April 1964 asserting its exclusive jurisdiction over the probate
proceeding as deferred to by the Cebu court was denied on 27 April 1964 and a second
motion for reconsideration dated 20 May 1964 was likewise denied.
On 11 May 1964, pursuant to its earlier order of 11 April 1964, the hearing for probate of
the last will of the decedent was called three times at half-hour intervals, but
notwithstanding due notification none of the oppositors appeared and the Quezon City
court proceeded at 9:00 a.m. with the hearing in their absence.
As per the order issued by it subsequently on 15 May 1964, the Quezon City court
noted that respondents-oppositors had opposed probate under their opposition and
motion to dismiss on the following grounds:
(a) That the will was not executed and attested as required by law;
(b) That the will was procured by undue and improper pressure and
influence on the part of the beneficiary or some other persons for his
benefit;
(c) That the testator's signature was procured by fraud and/or that the
testator acted by mistake and did not intend that the instrument he
signed should be his will at the time he affixed his signature thereto. 6
The Quezon City court further noted that the requisite publication of the notice of the
hearing had been duly complied with and that all the heirs had been duly notified of the
hearing, and after receiving the testimony of the three instrumental witnesses to the
decedent's last will, namely Atty. Florencio Albino, Dr. Guillermo A. Picache and Dr.
Jose P. Ojeda, and of the notary public, Atty. Braulio A. Arriola, Jr., who ratified the said
last will, and the documentary evidence (such as the decedent's residence certificates,
income tax return, diplomatic passport, deed of donation) all indicating that the
decedent was a resident of 69 Pi y Margal St., Quezon City, as also affirmed by him in
his last will, the Quezon City court in its said order of 15 May 1964 admitted to
probate the late senator's last will and testament as having been "freely and voluntarily
executed by the testator" and "with all formalities of the law" and appointed petitionerwidow as executrix of his estate without bond "following the desire of the testator" in his
will as probated.

Instead of appealing from the Quezon City court's said order admitting the will to
probate and naming petitioner-widow as executrix thereof, respondents filed a special
civil action of certiorari and prohibition with preliminary injunction with respondent Court
of Appeals (docketed as case CA-G.R. No. 34104-R) to bar the Rizal court from
proceeding with case No. Q-7898.
On 21 November 1964, the Court of Appeals rendered a decision in favor of
respondents (petitioners therein) and against the herein petitioner, holding that:
Section 1, Rule 73, which fixes the venue in proceedings for the
settlement of the estate of a deceased person, covers both testate and
intestate proceedings. Sp. Proc. 2433-R of the Cebu CFI having been
filed ahead, it is that court whose jurisdiction was first invoked and which
first attached. It is that court which can properly and exclusively pass
upon the factual issues of (1) whether the decedent left or did not leave
a valid will, and (2) whether or not the decedent was a resident of Cebu
at the time of his death.
Considering therefore that the first proceeding was instituted in the Cebu
CFI (Special Proceeding 2433-R), it follows that the said court must
exercise jurisdiction to the exclusion of the Rizal CFI, in which the
petition for probate was filed by the respondent Rosa Cayetano Cuenco
(Special Proceeding Q-7898). The said respondent should assert her
rights within the framework of the proceeding in the Cebu CFI, instead of
invoking the jurisdiction of another court.
The respondents try to make capital of the fact that on March 13, 1964,
Judge Amador Gomez of the Cebu CFI, acting in Sp. Proc. 2433-R,
stated that the petition for appointment of special administrator was "not
yet ready for the consideration of the Court today. It would be premature
for this Court to act thereon, it not having yet regularly acquired
jurisdiction to try this proceeding ... . " It is sufficient to state in this
connection that the said judge was certainly not referring to the court's
jurisdiction over the res, not to jurisdiction itself which is acquired from
the moment a petition is filed, but only to theexercise of jurisdiction in
relation to the stage of the proceedings. At all events, jurisdiction is
conferred and determined by law and does not depend on the
pronouncements of a trial judge.

The dispositive part of respondent appellate court's judgment provided as follows:


ACCORDINGLY, the writ of prohibition will issue, commanding and
directing the respondent Court of First Instance of Rizal, Branch IX,
Quezon City, and the respondent Judge Damaso B. Tengco to refrain
perpetually from proceeding and taking any action in Special Proceeding
Q-7898 pending before the said respondent court. All orders heretofore
issued and actions heretofore taken by said respondent court and
respondent Judge, therein and connected therewith, are hereby
annulled. The writ of injunction heretofore issued is hereby made
permanent. No pronouncement as to costs.
Petitioner's motion for reconsideration was denied in a resolution of respondent Court
of Appeals, dated 8 July 1965; hence the herein petition for review on certiorari.
The principal and decisive issue at bar is, theretofore, whether the appellate court erred
in law in issuing the writ of prohibition against the Quezon City court ordering it to
refrain perpetually from proceeding with the testateproceedings and annulling and
setting aside all its orders and actions, particularly its admission to probate of the
decedent's last will and testament and appointing petitioner-widow as executrix thereof
without bond in compliance with the testator's express wish in his testament. This issue
is tied up with the issue submitted to the appellate court, to wit, whether the Quezon
City court acted without jurisdiction or with grave abuse of discretion in taking
cognizance and assuming exclusive jurisdiction over the probate proceedings filed with
it, in pursuance of the Cebu court's order of 10 April 1964 expressly consenting in
deference to the precedence of probate over intestate proceedings that it (the Quezon
City court) should first act "on the petition for probate of the document purporting to be
the last will and testament of the deceased Don Mariano Jesus Cuenco" - which order
of the Cebu court respondents never questioned nor challenged by prohibition
or certiorari proceedings and thus enabled the Quezon City court to proceed without
any impediment or obstruction, once it denied respondent Lourdes Cuenco's motion to
dismiss the probate proceeding for alleged lack of jurisdiction or improper venue,
toproceed with the hearing of the petition and to admit the will to probate upon having
been satisfied as to its due execution and authenticity.
The Court finds under the above-cited facts that the appellate court erred in law in
issuing the writ of prohibition against the Quezon City court from proceeding with the
testate proceedings and annulling and setting aside all its orders and actions,
particularly its admission to probate of the deceased's last will and testament and

appointing petitioner-widow as executrix thereof without bond pursuant to the deceased


testator's express wish, for the following considerations:
1. The Judiciary Act 7 concededly confers original jurisdiction upon all Courts of First
Instance over "all matter of probate, both of testate and intestate estates." On the other
hand, Rule 73, section of the Rules of Court lays down the rule of venue, as the very
caption of the Rule indicates, and in order to prevent conflict among the different courts
which otherwise may properly assume jurisdiction from doing so, the Rule specifies that
"the court first taking cognizance of the settlement of the estate of a decedent,
shall exercise jurisdiction to the exclusion of all other courts." The cited Rule provides:
Section 1. Where estate of deceased persons settled. If the decedent is
an inhabitant of the Philippines at the time of his death, whether a citizen
or an alien, his will shall be proved, or letters of administration granted,
and his estate settled, in the Court of First Instance in the Province in
which he resides at the time of his death, and if he is an inhabitant of a
foreign country, the Court of First Instance of the province in which he
had estate. The court first taking cognizance of the settlement of the
estate of a decedent, shall exercise jurisdiction to the exclusion of all
other courts. The jurisdiction assumed by a court, so far as it depends on
the place of residence, of the decedent, or of the location of his estate,
shall not be contested in a suit or proceeding, except in an appeal from
that court, in the original case, or when the want of jurisdiction
appears on the record. (Rule 73) 8
It is equally conceded that the residence of the deceased or the location of his estate
is not an element of jurisdiction over the subject matter but merely of venue. This was
lucidly stated by the late Chief Justice Moran inSy Oa vs. Co Ho 9 as follows:
We are not unaware of existing decisions to the effect that in probate
cases the place of residence of the deceased is regarded as a question
of jurisdiction over the subject-matter. But we decline to follow this view
because of its mischievous consequences. For instance, a probate case
has been submitted in good faith to the Court of First Instance of a
province where the deceased had not resided. All the parties, however,
including all the creditors, have submitted themselves to the jurisdiction
of the court and the case is therein completely finished except for a
claim of a creditor who also voluntarily filed it with said court but on
appeal from an adverse decision raises for the first time in this Court the

question of jurisdiction of the trial court for lack of residence of the


deceased in the province. If we consider such question of residence as
one affecting the jurisdiction of the trial court over the subject-matter, the
effect shall be that the whole proceedings including all decisions on the
different incidents which have arisen in court will have to
be annulled and the same case will have to be commenced
anew before another court of the same rank in another province. That
this is ofmischievous effect in the prompt administration of justice is too
obvious to require comment. (Cf. Tanunchuan vs. Dy Buncio & Co., G.R.
No. 48206, December 31, 1942) Furthermore, section 600 of Act No.
190, 10 providing that the estate of a deceased person shall be settled in
the province where he had last resided, could not have been intended as
defining the jurisdiction of the probate court over the subject-matter,
because such legal provision is contained in a law of procedure dealing
merely with procedural matters, and, as we have said time and again,
procedure is one thing and jurisdiction over the subject matter is another.
(Attorney-General vs. Manila Railroad Company, 20 Phil. 523.) The law
of jurisdiction Act No. 136, 11Section 56, No. 5 confers upon Courts
of First Instance jurisdiction over all probate cases independently of the
place of residence of the deceased. Since, however, there are many
courts of First Instance in the Philippines, the Law of Procedure, Act No.
190, section 600, fixes the venue or the place where each case shall be
brought. Thus, the place of residence of the deceased is not an element
of jurisdiction over the subject-matter but merely of venue. And it is upon
this ground that in the new Rules of Court the province where the estate
of a deceased person shall be settled is properly called "venue".
It should be noted that the Rule on venue does not state that the court with whom the
estate or intestate petition is first filed acquires exclusive jurisdiction.
The Rule precisely and deliberately provides that "the court first taking cognizance of
the settlement of the estateof a decedent, shall exercise jurisdiction to the exclusion of
all other courts."
A fair reading of the Rule since it deals with venue and comity between courts of
equal and co-ordinate jurisdiction indicates that the court with whom the petition is
first filed, must also first take cognizance of the settlement of the estate in order
to exercise jurisdiction over it to the exclusion of all other courts.

Conversely, such court, may upon learning that a petition for probate of the decedent's
last will has been presented in another court where the decedent obviously had his
conjugal domicile and resided with his surviving widow and their minor children, and
that the allegation of the intestate petition before it stating that the decedent
died intestate may be actually false, may decline to take cognizance of the petition and
hold the petition before it in abeyance, and instead defer to the second court which has
before it the petition for probate of the decedent's alleged last will.
2. This exactly what the Cebu court did. Upon petitioner-widow's filing with it a motion to
dismiss Lourdes' intestate petition, it issued its order holding in abeyance its action on
the dismissal motion and deferred to the Quezon City court, awaiting its action on the
petition for probate before that court. Implicit in the Cebu court's order was that if the
will was duly admitted to probate, by the Quezon City court, then it would definitely
decline to take cognizance of Lourdes' intestate petition which would thereby be shown
to be false and improper, and leave the exercise of jurisdiction to the Quezon City
court, to the exclusion of all other courts. Likewise by its act of deference, the Cebu
court left it to the Quezon City court to resolve the question between the parties
whether the decedent's residence at the time of his death was in Quezon City where he
had his conjugal domicile rather than in Cebu City as claimed by respondents. The
Cebu court thus indicated that it would decline to take cognizance of
the intestate petition before it and instead defer to the Quezon City court, unless the
latter would make a negative finding as to the probate petition and the residence of the
decedent within its territory and venue.
3. Under these facts, the Cebu court could not be held to have acted without jurisdiction
or with grave abuse of jurisdiction in declining to take cognizance of
the intestate petition and deferring to the Quezon City court.
Necessarily, neither could the Quezon City court be deemed to have acted without
jurisdiction in taking cognizance of and acting on the probate petition since under Rule
73, section 1, the Cebu court must first take cognizance over the estate of the decedent
and must exercise jurisdiction to exclude all other courts, which the Cebu court declined
to do. Furthermore, as is undisputed, said rule only lays down a rule of venue and the
Quezon City court indisputably had at least equal and coordinate jurisdiction over the
estate.
Since the Quezon City court took cognizance over the probate petition before it
and assumed jurisdiction over the estate, with the consent and deference of the Cebu

court, the Quezon City court should be left now, by the same rule of venue of said Rule
73, to exercise jurisdiction to the exclusion of all other courts.
Under the facts of the case and where respondents submitted to the Quezon City court
their opposition to probate of the will, but failed to appear at the scheduled hearing
despite due notice, the Quezon City court cannot be declared, as the appellate court
did, to have acted without jurisdiction in admitting to probate the decedent's will and
appointing petitioner-widow as executrix thereof in accordance with the
testator's testamentary disposition.
4. The relatively recent case of Uriarte vs. Court of First Instance of Negros
Occidental 12 with facts analogous to the present case 13 is authority against respondent
appellate court's questioned decision.
In said case, the Court upheld the doctrine of precedence of probate proceedings over
intestate proceedings in this wise:
It can not be denied that a special proceeding intended to effect the
distribution of the estate of a deceased person, whether in accordance
with the law on intestate succession or in accordance with his will, is a
"probate matter" or a proceeding for the settlement of his estate. It is
equally true, however, that in accordance with settled jurisprudence in
this jurisdiction, testate proceedings for the settlement of the estate of a
deceased person take precedence over intestate proceedings for the
same purpose. Thus it has been held repeatedly that, if in the course of
intestate proceedings pending before a court of first instance it is found
that the decedent had left a last will, proceedings for the probate of the
latter should replace the intestate proceedings even if at that state an
administrator had already been appointed, the latter being required to
render final account and turn over the estate in his possession to the
executor subsequently appointed. This however, is understood to be
without prejudice that should the alleged last will be rejected or is
disapproved, the proceeding shall continue as an intestacy. As already
adverted to, this is a clear indication that proceedings for the probate of
a will enjoy priority over intestate proceedings. 14
The Court likewise therein upheld the jurisdiction of the second court, (in this case, the
Quezon City court) although opining that certain considerations therein "would seem to
support the view that [therein respondent] should have submitted said will for probate to

the Negros Court, [in this case, the Cebu court] either in a separate special proceeding
or in an appropriate motion for said purpose filed in the already pending Special
Proceeding No. 6344," 15 thus:
But the fact is that instead of the aforesaid will being presented for probate to the
Negros Court, Juan Uriarte Zamacona filed the petition for the purpose with the Manila
Court. We can not accept petitioner's contention in this regard that the latter court had
no jurisdiction to consider said petition, albeit we say that it was not the proper
venue therefor.
It is well settled in this jurisdiction that wrong venue is merely
a waivable procedural defect, and, in the light of the circumstances
obtaining in the instant case, we are of the opinion, and so hold, that
petitioner has waived the right to raise such objection or is precluded
from doing so by laches. It is enough to consider in this connection that
petitioner knew of the existence of a will executed by Juan Uriarte y
Goite since December 19, 1961 when Higinio Uriarte filed his opposition
to the initial petition filed in Special Proceeding No. 6344; that petitioner
likewise was served with notice of the existence (presence) of the
alleged last will in the Philippines and of the filing of the petition for its
probate with the Manila Court since August 28, 1962 when Juan Uriarte
Zamacona filed a motion for the dismissal of Special Proceeding No.
6344. All these notwithstanding, it was only on April 15, 1963 that he
filed with the Manila Court in Special Proceeding No. 51396 an Omnibus
motion asking for leave to intervene and for the dismissal and annulment
of all the proceedings had therein up to that date; thus enabling the
Manila Court not only to appoint an administrator with the will annexed
but also to admit said will to probate more than five months earlier, or
more specifically, on October 31, 1962. To allow him now to assail the
exercise of jurisdiction over the probate of the will by the Manila Court
and the validity of all the proceedings had in Special Proceeding No.
51396 would put a premium on his negligence. Moreover, it must be
remembered that this Court is not inclined to annul proceedings regularly
had in a lower court even if the latter was not the proper venue therefor,
if the net result would be to have the same proceedings repeated in
some other court of similar jurisdiction; more so in a case like the
present where the objection against said proceedings is raised too
late. 16

5. Under Rule 73, section 1 itself, the Quezon City


court's assumption of jurisdiction over the decedent's estate on the basis of the will duly
presented for probate by petitioner-widow and finding that Quezon City was the
firstchoice of residence of the decedent, who had his conjugal home and domicile
therein with the deference in comity duly given by the Cebu court could not be
contested except by appeal from said court in the original case. The last paragraph of
said Rule expressly provides:
... The jurisdiction assumed by a court, so far as it depends on the place
of residence of the decedent, or of the location of his estate, shall not be
contested in a suit or proceeding, except in an appeal from that court, in
the original case, or when the want of jurisdiction appears on the record.
(Rule 73)
The exception therein given, viz, "when the want of jurisdiction appears on the record"
could probably be properly invoked, had such deference in comity of the Cebu court to
the Quezon City court not appeared in the record, or had the record otherwise shown
that the Cebu court had taken cognizance of the petition before it and assumed
jurisdiction.
6. On the question that Quezon City established to be the residence of the late senator,
the appellate court while recognizing that "the issue is a legitimate one" held in reliance
on Borja vs. Tan 17 that.
... The issue of residence comes within the competence of whichever
court is considered to prevail in the exercise jurisdiction - in this case,
the Court of First Instance of Cebu as held by this Court. Parenthetically,
we note that the question of the residence of the deceased is a serious
one, requiring both factual and legal resolution on the basis of ample
evidence to be submitted in the ordinary course of procedure in the first
instance, particularly in view of the fact that the deceased was better
known as the Senator from Cebu and the will purporting to be his also
gives Cebu, besides Quezon City, as his residence. We reiterate that
this matter requires airing in the proper court, as so indicated in the
leading and controlling case of Borja vs. Hon. Bienvenido Tan, et al.,
G.R. L-7792, July 27, 1955.
In the case at bar, however, the Cebu court declined to take cognizance of
the intestate petition first filed with it and deferred to the testate proceedings filed with

the Quezon City court and in effect asked the Quezon City court to determine the
residence of the decedent and whether he did leave a last will and testament upon
which would depend the proper venue of the estate proceedings, Cebu or Quezon City.
The Quezon City court having thus determined in effect for both courts at the
behest and with the deference and consent of the Cebu court thatQuezon City was
the actual residence of the decedent who died testate and therefore the proper venue,
the Borja ruling would seem to have no applicability. It would not serve the practical
ends of justice to still require the Cebu court, if the Borja ruling is to be held applicable
and as indicated in the decision under review, to determine for itself the actual
residence of the decedent (when the Quezon City court had already so determined
Quezon City as the actual residence at the Cebu court's behest and respondents have
not seriously questioned this factual finding based on documentary evidence) and if the
Cebu court should likewise determine Quezon City as the actual residence, or its
contrary finding reversed on appeal, only then to allow petitioner-widow after years of
waiting and inaction to institute the corresponding proceedings in Quezon City.
7. With more reason should the Quezon City proceedings be upheld when it is taken
into consideration that Rule 76, section 2 requires that the petition for allowance of a
will must show: "(a) the jurisdictional facts." Such "jurisdictional facts" in probate
proceedings, as held by the Court in Fernando vs. Crisostomo 18 " are the death of the
decedent, his residence at the time of his death in the province where the probate court
is sitting, or if he is an inhabitant of a foreign country, his having left his estate in such
province."
This tallies with the established legal concept as restated by Moran that
"(T)he probate of a will is a proceeding in rem. The notice by publication as a prerequisite to the allowance of a will, is a constructive notice to the whole world, and
when probate is granted, the judgment of the court is binding upon everybody, even
against the State.The probate of a will by a court having jurisdiction thereof is
conclusive as to its due execution and validity." 19The Quezon City court acted
regularly within its jurisdiction (even if it were to be conceded that Quezon City was not
the proper venue notwithstanding the Cebu court's giving way and deferring to it,) in
admitting the decedent's last will to probate and naming petitioner-widow as executrix
thereof. Hence, the Quezon city court's action should not be set aside by a writ of
prohibition for supposed lack of jurisdiction as per the appellate court's appealed
decision, and should instead be sustained in line with Uriarte, supra, where the Court,
in dismissing the certiorari petition challenging the Manila court's action admitting the
decedent's will to probate and distributing the estate in accordance therewith in
the secondproceeding, held that "it must be remembered that this Court is not inclined

to annul proceedings regularly had in a lower court even if the latter was not the proper
venue therefor, if the net result would be to have the same proceedings repeated in
some other court of similar jurisdiction." As stressed by Chief Justice Moran in Sy Oa,
supra, "the mischievous effect in the administration of justice" of considering the
question of residenceas affecting the jurisdiction of the trial court and annulling the
whole proceedings only to start all over again the same proceedings before another
court of the same rank in another province "is too obvious to require comment."
8. If the question of jurisdiction were to be made to depend only on who of the
decedent's relatives gets first to file a petition for settlement of the decedent's estate,
then the established jurisprudence of the Court that Rule 73, section 1 provides only a
rule of venue in order to preclude different courts which may properly
assumejurisdiction from doing so and creating conflicts between them to the detriment
of the administration of justice, and that venue is waivable, would be set at naught. As
between relatives who unfortunately do not see eye to eye, it would be converted into a
race as to who can file the petition faster in the court of his/her choice regardless of
whether the decedent is still in cuerpo presente and in disregard of the decedent's
actual last domicile, the fact that he left a last will and testament and the right of his
surviving widow named as executrix thereof. Such dire consequences were certainly
not intended by the Rule nor would they be in consonance with public policy and the
orderly administration of justice.
9. It would finally be unjust and inequitable that petitioner-widow, who under all the
applicable rules of venue, and despite the fact that the Cebu court (where respondent
Lourdes Cuenco had filed an intestate petition in the Cebu court earlier by a week's
time on 5 March 1964) deferred to the Quezon City court where petitioner had within
fifteen days (on March 12, 1964) after the decedent's death (on February 25, 1964)
timely filed the decedent's last will and petitioned for letters testamentary and is
admittedly entitled to preference in the administration of her husband's estate, 20 would
be compelled under the appealed decision to have to go all the way to Cebu and
submit anew the decedent's will there for probate either in a new proceeding or by
asking that the intestate proceedings be converted into a testate proceeding when
under the Rules, the proper venue for the testate proceedings, as per the facts of
record and as already affirmed by the Quezon City court is Quezon City, where the
decedent and petitioner-widow had their conjugal domicile.
It would be an unfair imposition upon petitioner as the one named and entitled to be
executrix of the decedent's last will and settle his estate in accordance therewith, and a
disregard of her rights under the rule on venue and the law on jurisdiction to require her

to spend much more time, money and effort to have to go from Quezon City to the
Cebu court everytime she has an important matter of the estate to take up with the
probate court.
It would doubly be an unfair imposition when it is considered that under Rule 73,
section 2, 21 since petitioner's marriage has been dissolved with the death of her
husband, their community property and conjugal estate have to be administered and
liquidated in the estate proceedings of the deceased spouse. Under the appealed
decision, notwithstanding that petitioner resides in Quezon City, and the proper venue
of the testate proceeding was in Quezon City and the Quezon City court properly took
cognizance and exercised exclusive jurisdiction with the deference in comity and
consent of the Cebu court, such proper exercise of jurisdiction would be nullified and
petitioner would have to continually leave her residence in Quezon City and go to Cebu
to settle and liquidate even her own community property and conjugal estate with the
decedent.
10. The Court therefore holds under the facts of record that the Cebu court did not act
without jurisdiction nor with grave abuse of discretion in declining to take cognizance of
the intestate petition and instead deferring to thetestate proceedings filed just a week
later by petitioner as surviving widow and designated executrix of the decedent's last
will, since the record before it (the petitioner's opposition and motion to dismiss)
showed the falsityof the allegation in the intestate petition that the decedent had
died without a will. It is noteworthy that respondents never challenged by certiorari or
prohibition proceedings the Cebu court's order of 10 April 1964 deferring to the probate
proceedings before the Quezon City court, thus leaving the latter free (pursuant to the
Cebu court's order of deference) to exercise jurisdiction and admit the decedent's will to
probate.
For the same reasons, neither could the Quezon City court be held to have acted
without jurisdiction nor with grave abuse of discretion in admitting the decedent's will to
probate and appointing petitioner as executrix in accordance with its testamentary
disposition, in the light of the settled doctrine that the provisions of Rule 73, section 1
lay down only a rule of venue, not of jurisdiction.
Since respondents undisputedly failed to appeal from the Quezon City court's order of
May 15, 1964 admitting the will to probate and appointing petitioner as executrix
thereof, and said court concededly has jurisdiction to issue said order, the said order of
probate has long since become final and can not be overturned in a special civic action
of prohibition.

11. Finally, it should be noted that in the Supreme Court's exercise of its supervisory
authority over all inferior courts, 22 it may properly determine, as it has done in the case
at bar, that venue wasproperly assumed by and transferred to the Quezon City
court and that it is the interest of justice and in avoidance of needless delay that the
Quezon City court's exercise of jurisdiction over the testate estate of the decedent (with
the due deference and consent of the Cebu court) and its admission to probate of his
last will and testament and appointment of petitioner-widow as administratrix without
bond in pursuance of the decedent's express will and all its orders and actions taken in
the testate proceedings before it be approved and authorized rather than to annul all
such proceedings regularly had and to repeat and duplicate the same proceedings
before the Cebu court only to revert once more to the Quezon City court should the
Cebu court find that indeed and in fact, as already determined by the Quezon City court
on the strength of incontrovertible documentary evidence of record, Quezon City was
the conjugal residence of the decedent.
ACCORDINGLY, judgment is hereby rendered reversing the appealed decision and
resolution of the Court of Appeals and the petition for certiorari and prohibition with
preliminary injunction originally filed by respondents with the Court of Appeals (CA-G.R.
No. 34104-R) is ordered dismissed. No costs.

G.R. No. L-11609, September 24, 1959


IN THE MATTER OF THE TESTATE ESTATE OF THE LATE AGUSTIN DEL VALLE.
ANTONIA VENTURA, PETITIONER AND APPELLANT, VS. MAURA VENTURA,
MILAGROS P. VENTURA, DEOGRACIAS P. VENTURA, AND JACINTA P.
VENTURA, OPPOSITORS AND APPELLEES.
CONCEPCION, J.:
Appeal from an order of the Court of First Instance of Tarlac, granting appellees' motion
to dismiss appellant's petition for probate of the will of her deceased husband Agustin
del Valle, on the ground that it amounted to an attempt to reopen Special Proceedings,
No. 912, which had been finally and definitly closed, terminated and archived.
DECISION
This is an appeal from an order of the Court of first Instance of Tarlac, granting a motion
to dismiss and dismissing the petition herein.
Petitioner Antonia Ventura is the widow of Agustine del Valle, who died in Paniqui,
Tarlac, on May 19, 1955. Soon thereafter, or on June 7 of the same year, she instituted
Special Proceedings No. 912 if said court, with a petition for the probate of a document
attached thereto, as the alleged last will and testament of the deceased. On July 15,
1955, after the issuance of the corresponding notice of hearing, the publication of said
notice and the service of copies thereof upon all parties concerned, petitioner filed a
motion stating that the heirs instituted in the will had agreed to partition, among
themselves, the estate of the deceased , in accordance with the provisions of said
instrument, and praying that an order be issued "terminating and closing" the
aforementioned proceedings. On the same date, the court issued an order declaring
that said motion was well taken and would, accordingly, be granted, upon submission of
a sighed copy of the corresponding deed of extra-judicial partition, for which petitioner
was given two (2) days. Petitioner having seasonably complied with this requirement,
said Special Proceedings No. 912 was "terminated, closed and archived" by an order
dated June 19, 1955.
However, on May 9, 1956, said Antonia Ventura filed another partition for the probate of
the probate of the aforementioned last will and testament of her deceased husband,
Agustin del Valle, which petition was docketed as Special Proceedings No. 970 of the
same court. On June 22, 1956, the date set for the hearing of this petition, appellees
Maura, Milagros, Deogracias and Jacinta, all surnamed Ventura, moved for the
dismissal of the case upon the ground that it amounted to an attempt to reopen Special
Proceedings No. 912, which had already been finally and definitely closed, terminated
and archived; that the present proceedings is unnecessary and superfluous; and that

the provisions of the will in question had " already been carried out" in the
aforementioned deed of extra-judicial partition. The lower court granted this motion
upon the theory that it " no longer" had "jurisdiction to entertain" the case, "because to
do so would be for the court to reopen Special Proceedings No. 912", notwithstanding
the fact that it had been "definitely declared terminated, closed and archived" by an
order which has already "become final and executory", no appeal having been taken
therefrom. Hence, the present appeal taken by petitioner Antonia Ventura.
The aforementioned conclusion of the lower court is intenable. Although final and
executory; said order of dismissal of Special Proceedings No. 912 does not bar the
consideration of the petition herein. Pursuant to rule 30, Section 1, of the Rules of
Court.
"An action may be dismissed by the by the plaintiff without order of court by filling a
notice of dismissal at any time before service of the answer. Unless otherwise stated in
the notice, the dismissal is without prejudice, except that a notice operates as an
adjudication upon the merits when filed by a plaintiff who has once dismissed in a
competent court an action based on or including the same action."
Although found in Part I of the Rules of Court, which refers to civil actions, this rule is
applicable to Special Proceedings, for Rule 73, section 2, of said Rules provides:
"In the absence of special provisions, the rules provided for in ordinary actions shall be,
as far as practicable, applicable in special proceedings."
The order issue in Special Proceedings No. 912, directing that the same be closed,
terminated and archived, had filed a pleading or raised an issue, is deemed, therefore,
to be "without prejudice", the contrary not being stated either in said order or in the
motion that prompted its issuance. Needless to say, the aforementioned order may not
be regarded as a judgment, barring a subsequent action upon the ground of "res
adjudicata", said order not being an adjudication on the merits (Almeda v. Cruz, 47 Off.
Gaz., 1178). The case of Sandoval v. Santiago (L-1723, May 26, 1949), cited by
appellees in support of the theory that an extra-judicial partition, once approved by the
Court, becomes a judicial partition, is not in point, no such judicial approval having been
given to the deed of partition above referred to. In fact, the lower court and the
appellees herein still refer to said instrument as a deed of "extra-judicial" partition.
Moreover, in Guevarra v. Guevarra (74 Phil., 479, 487-488), it was explicitly held:
"x x x If the decedent left a will and no debts and heirs and legatees desire to make an
extra-judicial partition of the estate, they must first present that will to the court for
probate and divide the estate in accordance with the will. They may not disregard the
provisions of the will unless those provisions are contrary to law. Neither may they do

away with the presentation of the will to the court for probate, because such
suppression of the will is contrary to law and public policy. The law enjoins the probate
of the will and public policy requires it, because unless the will is probated and notice
thereof given to the whole world, the right of a person to dispose of his property by will
may rendered nugatory. Absent legatees and devises, or such of them as may have no
knowledge of the will, could be cheated of their inheritance thru the conclusion of some
of the heirs who might agrees to the partition of the estate among themselves to the
exclusion of others.
x

"Even if the decedent left no debts and nobody raises any question as to the
authenticity and due execution of the will none of the heirs may sue for the partition of
the estate in accordance with the will without first securing its allowance or probate of
the Court: first , because the law expressly provides that "no will shall pass either real
or personal estate unless it is probated and allowed in the proper court's and second,
because the probate of a will, which is a proceeding in rem, cannot be dispensed with
and substituted by any other proceeding, judicial or extra-judicial, without offending
against public policy designed to offectuate the testator's right to dispose of his property
by will in accordance with law and to protect the rights of the heirs and legatees under
the will thru the means provided by law, among which are the publication and the
personal notices to each and all of said heirs and legatees."

Hence, in "Testate Estate of the Late Asuncion Miraflores. Desedario Miraflores, et al.
vs. Soledad Mesa Magsuci, et al." (L-12166, April 29, 1959), we said:
"x x x Section 5, Rule 77 of the Rules of Court provides that 'when a will is delivered to,
or a petition for the allowance of a will is filed in, the court having jurisdiction, such court
shall fix a time and place for proving the will when all concerned may appear to contest
the allowance thereof.' Thus, in Salazar vs. The court of First Instance of Laguna (64
Phil. 785), we held that 'it is the inevitable duty of the court when a will is presented to
it, to appoint hearing for its allowance and to cause notices thereof to be given by
publication. The duty imposed by said section 3, Rule 77 of the Rules of Court is
imperative and non-compliance therewith would be a mockery of the law and of the last
will of the testator."

In other words, the lower court had, not only jurisdiction, but, also, the duty to entertain
appellant's petition for probate of the alleged will of Agustin de Valle. The practical and
legal necessity of giving due course to said petition becomes patent when we consider
that on February 8, 1952, a brother and a sister of the late Agustin del Valle, namely,

Ponciano del Valle and Severa Valle Vda. de Carlos, filed Civil Case No. 2060 of the
Court of First Instance of Tarlac, against Antonia Ventura, for the partition of the estate
of the deceased. Antonia Centura moved for the dismissla of the complaint therein,
upon the ground that is was barred by the action taken in the aforementioned Special
Proceedings No. 912. This motion was denied, on April 12, 1956, upon the ground that
no will shall pass either real or personal estate unless it is probated and allowed in the
proper court and that the probate of such will cannot be dispensed with. Hence,
Antonia Ventura felt constrained to institute the present proceedings. At any rate, under
the circumstance, public policy and public interest demand a determination of the
question whether Agustin de Valle died testate or intestate.
WHEREFORE, the order appealed from should be, as it is hereby, reversed, and lot the
record hereof be remanded to the lower court for further proceedings in consonance
with this decision, with costs against appellees herein.
IT IS SO ORDERED.

G.R. No. L-48840 December 29, 1943


ERNESTO M. GUEVARA, petitioner-appellant, vs. ROSARIO GUEVARA and her
husband PEDRO BUISON, respondent-appellees.

to her of a portion of 25 hectares of the large parcel of land of 259-odd hectares


described in plan Psu-66618. He also devised to her a portion of 5 hectares of the
same parcel of land by way of complete settlement of her usufructurary
right.1awphil.net

Primacias, Abad, Mencias & Castillo for appellant.


Pedro C. Quinto for appellees.

He set aside 100 hectares of the same parcel of land to be disposed of either by him
during his lifetime or by his attorney-in-fact Ernesto M. Guevara in order to pay all his
pending debts and to degray his expenses and those of his family us to the time of his
death.

OZAETA, J.:

The remainder of said parcel of land his disposed of in the following manner:

Ernesto M. Guevarra and Rosario Guevara, ligitimate son and natural daughter,
respectively, of the deceased Victorino L. Guevara, are litigating here over their
inheritance from the latter. The action was commenced on November 12, 1937, by
Rosario Guevara to recover from Ernesto Guevara what she claims to be her strict
ligitime as an acknowledged natural daughter of the deceased to wit, a portion of
423,492 square meters of a large parcel of land described in original certificate of title
No. 51691 of the province of Pangasinan, issued in the name of Ernesto M. Guervara
and to order the latter to pay her P6,000 plus P2,000 a year as damages for
withholding such legitime from her. The defendant answered the complaint contending
that whatever right or rights the plaintiff might have had, had been barred by the
operation of law.

(d). Toda la porcion restante de mi terreno arriba descrito, de la extension


superficial aproximada de ciento veintinueve (129) hectareas setenta (70)
areas, y veiticinco (25) centiares, con todas sus mejoras existentes en la
misma, dejo y distribuyo, pro-indiviso, a mis siguientes herederos como sigue:

It appears that on August 26, 1931, Victorino L. Guevara executed a will (exhibit A),
apparently with all the formalities of the law, wherein he made the following bequests:
To his stepdaughter Candida Guevara, a pair of earrings worth P150 and a gold chain
worth P40; to his son Ernesto M. Guevara, a gold ring worth P180 and all the furniture,
pictures, statues, and other religious objects found in the residence of the testator in
Poblacion Sur, Bayambang, Pangasinan; "a mi hija Rosario Guevara," a pair of
earrings worth P120; to his stepson Piuo Guevara, a ring worth P120; and to his wife by
second marriage, Angustia Posadas, various pieces of jewelry worth P1,020.

A mi hija natural reconocida, Rosario Guevara, veintiun (21) hectareas, sesenta


y un (61) areas y setenta y un (71) centiareas, que es la parte restante.

He also made the following devises: "A mis hijos Rosario Guevara y Ernesto M.
Guevara y a mis hijastros, Vivencio, Eduviges, Dionisia, Candida y Pio, apellidados
Guevara," a residential lot with its improvements situate in the town of Bayambang,
Pangasinan, having an area of 960 square meters and assessed at P540; to his wife
Angustia Posadas he confirmed the donation propter nuptias theretofore made by him

A mi hijo legitimo, Ernesto M. Guevara, ciento ocho (108) hectareas, ocho (8)
areas y cincuenta y cuatro (54) centiareas, hacia la parte que colinda al Oeste
de las cien (100) hectareas referidas en el inciso (a) de este parrafo del
testamento, como su propiedad absoluta y exclusiva, en la cual extension
superficial estan incluidas cuarenta y tres (43) hectareas, veintitres (23) areas y
cuarenta y dos (42) centiareas que le doy en concepto de mejora.

Duodecimo. Nombro por la presente como Albacea Testamentario a mi hijo


Ernesto M. Guevara, con relevacion de fianza. Y una vez legalizado este
testamento, y en cuanto sea posible, es mi deseo, que los herederos y
legatarios aqui nombrados se repartan extrajudicialmente mis bienes de
conformidad con mis disposiciones arriba consignadas.
Subsequently, and on July 12, 1933, Victorino L. Guevarra executed whereby he
conveyed to him the southern half of the large parcel of land of which he had
theretofore disposed by the will above mentioned, inconsideration of the sum of P1 and
other valuable considerations, among which were the payment of all his debts and
obligations amounting to not less than P16,500, his maintenance up to his death, and

the expenses of his last illness and funeral expenses. As to the northern half of the
same parcel of land, he declared: "Hago constar tambien que reconozco a mi referido
hijo Ernesto M. guevara como dueo de la mitad norte de la totalidad y conjunto de los
referidos terrenos por haberlos comprado de su propio peculio del Sr. Rafael T. Puzon
a quien habia vendido con anterioridad."
On September 27, 1933, final decree of registration was issued in land registration
case No. 15174 of the Court of First Instance of Pangasinan, and pursuant thereto
original certificate of title No. 51691 of the same province was issued on October 12 of
the same year in favor of Ernesto M. Guevara over the whole parcel of land described
in the deed of sale above referred to. The registration proceeding had been
commenced on November 1, 1932, by Victorino L. Guevara and Ernesto M. Guevara
as applicants, with Rosario, among others, as oppositor; but before the trial of the case
Victorino L. Guevara withdrew as applicant and Rosario Guevara and her co-oppositors
also withdrew their opposition, thereby facilitating the issuance of the title in the name
of Ernesto M. Guevara alone.
On September 27, 1933, Victorino L. Guevarra died. His last will and testament,
however, was never presented to the court for probate, nor has any administration
proceeding ever been instituted for the settlement of his estate. Whether the various
legatees mentioned in the will have received their respective legacies or have even
been given due notice of the execution of said will and of the dispositions therein made
in their favor, does not affirmatively appear from the record of this case. Ever since the
death of Victorino L. Guevara, his only legitimate son Ernesto M. Guevara appears to
have possessed the land adjudicated to him in the registration proceeding and to have
disposed of various portions thereof for the purpose of paying the debts left by his
father.
In the meantime Rosario Guevara, who appears to have had her father's last will and
testament in her custody, did nothing judicially to invoke the testamentary dispositions
made therein in her favor, whereby the testator acknowledged her as his natural
daughter and, aside from certain legacies and bequests, devised to her a portion of
21.6171 hectares of the large parcel of land described in the will. But a little over four
years after the testor's demise, she (assisted by her husband) commenced the present
action against Ernesto M. Guevara alone for the purpose hereinbefore indicated; and it
was only during the trial of this case that she presented the will to the court, not for the
purpose of having it probated but only to prove that the deceased Victirino L. Guevara
had acknowledged her as his natural daughter. Upon that proof of acknowledgment she
claimed her share of the inheritance from him, but on the theory or assumption that he

died intestate, because the will had not been probated, for which reason, she asserted,
the betterment therein made by the testator in favor of his legitimate son Ernesto M.
Guevara should be disregarded. Both the trial court and the Court of appeals sustained
that theory.
Two principal questions are before us for determination: (1) the legality of the procedure
adopted by the plaintiff (respondent herein) Rosario Guevara; and (2) the efficacy of the
deed of sale exhibit 2 and the effect of the certificate of title issued to the defendant
(petitioner herein) Ernesto M. Guevara.
I
We cannot sanction the procedure adopted by the respondent Rosario Guevara, it
being in our opinion in violation of procedural law and an attempt to circumvent and
disregard the last will and testament of the decedent. The Code of Civil Procedure,
which was in force up to the time this case was decided by the trial court, contains the
following pertinent provisions:
Sec. 625. Allowance Necessary, and Conclusive as to Execution. No will
shall pass either the real or personal estate, unless it is proved and allowed in
the Court of First Instance, or by appeal to the Supreme Court; and the
allowance by the court of a will of real and personal estate shall be conclusive
as to its due execution.
Sec. 626. Custodian of Will to Deliver. The person who has the custody of a
will shall, within thirty days after he knows of the death of the testator, deliver
the will into the court which has jurisdiction, or to the executor named in the will.
Sec. 627. Executor to Present Will and Accept or Refuse Trust. A person
named as executor in a will, shall within thirty days after he knows of the death
of the testor, or within thirty days after he knows that he is named executor, if he
obtained such knowledge after knowing of the death of the testor, present such
will to the court which has jurisdiction, unless the will has been otherwise
returned to said court, and shall, within such period, signify to the court his
acceptance of the trust, or make known in writing his refusal to accept it.
Sec. 628. Penalty. A person who neglects any of the duties required in the
two proceeding sections, unless he gives a satisfactory excuse to the court,
shall be subject to a fine not exceeding one thousand dollars.

Sec. 629. Person Retaining Will may be Committed. If a person having


custody of a will after the death of the testator neglects without reasonable
cause to deliver the same to the court having jurisdiction, after notice by the
court so to do, he may be committed to the prison of the province by a warrant
issued by the court, and there kept in close confinement until he delivers the
will.
The foregoing provisions are now embodied in Rule 76 of the new Rules of Court,
which took effect on July 1, 1940.
The proceeding for the probate of a will is one in rem, with notice by publication to the
whole world and with personal notice to each of the known heirs, legatees, and
devisees of the testator (section 630, C. c. P., and sections 3 and 4, Rule 77). Altho not
contested (section 5, Rule 77), the due execution of the will and the fact that the
testator at the time of its execution was of sound and disposing mind and not acting
under duress, menace, and undue influence or fraud, must be proved to the satisfaction
of the court, and only then may the will be legalized and given effect by means of a
certificate of its allowance, signed by the judge and attested by the seal of the court;
and when the will devises real property, attested copies thereof and of the certificate of
allowance must be recorded in the register of deeds of the province in which the land
lies. (Section 12, Rule 77, and section 624, C. C. P.)
It will readily be seen from the above provisions of the law that the presentation of a will
to the court for probate is mandatory and its allowance by the court is essential and
indispensable to its efficacy. To assure and compel the probate of will, the law punishes
a person who neglects his duty to present it to the court with a fine not exceeding
P2,000, and if he should persist in not presenting it, he may be committed to prision
and kept there until he delivers the will.
The Court of Appeals took express notice of these requirements of the law and held
that a will, unless probated, is ineffective. Nevertheless it sanctioned the procedure
adopted by the respondent for the following reasons:
The majority of the Court is of the opinion that if this case is dismissed ordering
the filing of testate proceedings, it would cause injustice, incovenience, delay,
and much expense to the parties, and that therefore, it is preferable to leave
them in the very status which they themselves have chosen, and to decide their
controversy once and for all, since, in a similar case, the Supreme Court applied
that same criterion (Leao vs. Leao, supra), which is now sanctioned by

section 1 of Rule 74 of the Rules of Court. Besides, section 6 of Rule 124


provides that, if the procedure which the court ought to follow in the exercise of
its jurisdiction is not specifically pointed out by the Rules of Court, any suitable
process or mode of procedure may be adopted which appears most consistent
to the spirit of the said Rules. Hence, we declare the action instituted by the
plaintiff to be in accordance with law.
Let us look into the validity of these considerations. Section 1 of Rule 74 provides as
follows:
Section 1. Extrajudicial settlement by agreement between heirs. If the
decedent left no debts and the heirs and legatees are all of age, or the minors
are represented by their judicial guardians, the parties may, without securing
letters of administration, divide the estate among themselves as they see fit by
means of a public instrument filed in the office of the register of deeds, and
should they disagree, they may do so in an ordinary action of partition. If there is
only one heir or one legatee, he may adjudicate to himself the entire estate by
means of an affidavit filed in the office of the register of deeds. It shall be
presumed that the decedent left no debts if no creditor files a petition for letters
of administration within two years after the death of the decedent.
That is a modification of section 596 of the Code of Civil Procedure, which reads as
follows:
Sec. 596. Settlement of Certain Intestates Without Legal Proceedings.
Whenever all the heirs of a person who died intestate are of lawful age and
legal capacity and there are no debts due from the estate, or all the debts have
been paid the heirs may, by agreement duly executed in writing by all of them,
and not otherwise, apportion and divide the estate among themselves, as they
may see fit, without proceedings in court.
The implication is that by the omission of the word "intestate" and the use of the word
"legatees" in section 1 of Rule 74, a summary extrajudicial settlement of a deceased
person's estate, whether he died testate or intestate, may be made under the
conditions specified. Even if we give retroactive effect to section 1 of Rule 74 and apply
it here, as the Court of Appeals did, we do not believe it sanctions the nonpresentation
of a will for probate and much less the nullification of such will thru the failure of its
custodian to present it to the court for probate; for such a result is precisely what Rule
76 sedulously provides against. Section 1 of Rule 74 merely authorizes the extrajudicial

or judicial partition of the estate of a decedent "without securing letter of


administration." It does not say that in case the decedent left a will the heirs and
legatees may divide the estate among themselves without the necessity of presenting
the will to the court for probate. The petition to probate a will and the petition to issue
letters of administration are two different things, altho both may be made in the same
case. the allowance of a will precedes the issuance of letters testamentary or of
administration (section 4, Rule 78). One can have a will probated without necessarily
securing letters testamentary or of administration. We hold that under section 1 of Rule
74, in relation to Rule 76, if the decedent left a will and no debts and the heirs and
legatees desire to make an extrajudicial partition of the estate, they must first present
that will to the court for probate and divide the estate in accordance with the will. They
may not disregard the provisions of the will unless those provisions are contrary to law.
Neither may they so away with the presentation of the will to the court for probate,
because such suppression of the will is contrary to law and public policy. The law
enjoins the probate of the will and public policy requires it, because unless the will is
probated and notice thereof given to the whole world, the right of a person to dispose of
his property by will may be rendered nugatory, as is attempted to be done in the instant
case. Absent legatees and devisees, or such of them as may have no knowledge of the
will, could be cheated of their inheritance thru the collusion of some of the heirs who
might agree to the partition of the estate among themselves to the exclusion of others.
In the instant case there is no showing that the various legatees other than the present
litigants had received their respective legacies or that they had knowledge of the
existence and of the provisions of the will. Their right under the will cannot be
disregarded, nor may those rights be obliterated on account of the failure or refusal of
the custodian of the will to present it to the court for probate.
Even if the decedent left no debts and nobdy raises any question as to the authenticity
and due execution of the will, none of the heirs may sue for the partition of the estate in
accordance with that will without first securing its allowance or probate by the court,
first, because the law expressly provides that "no will shall pass either real or personal
estate unless it is proved and allowed in the proper court"; and, second, because the
probate of a will, which is a proceeding in rem, cannot be dispensed with the
substituted by any other proceeding, judicial or extrajudicial, without offending against
public policy designed to effectuate the testator's right to dispose of his property by will
in accordance with law and to protect the rights of the heirs and legatees under the will
thru the means provided by law, among which are the publication and the personal
notices to each and all of said heirs and legatees. Nor may the court approve and allow
the will presented in evidence in such an action for partition, which is one in personam,

any more than it could decree the registration under the Torrens system of the land
involved in an ordinary action for reinvindicacion or partition.
We therefore believe and so hold that section 1 of Rule 74, relied upon by the Court of
Appeals, does not sanction the procedure adopted by the respondent.
The case of Leao vs. Leao (25 Phil., 180), cited by the Court of Appeals, like section
1 of Rule 74, sanctions the extrajudicial partition by the heirs of the properties left by a
decedent, but not the nonpresentation of a will for probate. In that case one Paulina Ver
executed a will on October 11, 1902, and died on November 1, 1902. Her will was
presented for probate on November 10, 1902, and was approved and allowed by the
Court on August 16, 1904. In the meantime, and on November 10, 1902, the heirs went
ahead and divided the properties among themselves and some of them subsequently
sold and disposed of their shares to third persons. It does not affirmatively appear in
the decision in that case that the partition made by the heirs was not in accordance with
the will or that they in any way disregarded the will. In closing the case by its order
dated September 1, 1911, the trial court validated the partition, and one of the heirs,
Cunegunda Leao, appealed. In deciding the appeal this Court said:
The principal assignment of error is that the lower court committed an error in
deciding that the heirs and legatees of the estate of Da. Paulina Ver had
voluntarily divided the estate among themselves.
In resolving that question this Court said:
In view of the positive finding of the judge of the lower court that there had been
a voluntary partition of the estate among the heirs and legatees, and in the
absence of positive proof to the contrary, we must conclude that the lower court
had some evidence to support its conclusion.
Thus it will be seen that as a matter of fact no question of law was raised and decided
in that case. That decision cannot be relied upon as an authority for the unprecedented
and unheard of procedure adopted by the respondent whereby she seeks to prove her
status as an acknowledged natural child of the decedent by his will and attempts to
nullify and circumvent the testamentary dispositions made by him by not presenting the
will to the court for probate and by claiming her legitime as an acknowledged natural
child on the basis of intestacy; and that in the face of express mandatory provisions of
the law requiring her to present the will to the court for probate.

In the subsequent case of Riosa vs. Rocha (1926), 48 Phil. 737, this Court departed
from the procedure sanctioned by the trial court and impliedly approved by this Court in
the Leao case, by holding that an extrajudicial partition is not proper in testate
succession. In the Riosa case the Court, speaking thru Chief Justice Avancea, held:
1. EXTRAJUDICIAL PARTITION; NOT PROPER IN TESTATE SUCCESSION.
Section 596 of the Code of Civil Procedure, authorizing the heirs of a person
who dies intestate to make extrajudicial partition of the property of the
deceased, without going into any court of justice, makes express reference to
intestate succession, and therefore excludes testate succession.
2. ID.; EFFECTS OF; TESTATE SUCCESSION. In the instant case, which is
a testate succession, the heirs made an extrajudicial partition of the estate and
at the same time instituted proceeding for the probate of the will and the
administration of the estate. When the time came for making the partition, they
submitted to the court the extrajudicial partition previously made by them, which
the court approved. Held: That for the purposes of the reservation and the rights
and obligations created thereby, in connection with the relatives benefited, the
property must not be deemed transmitted to the heirs from the time the
extrajudicial partition was made, but from the time said partition was approved
by the court. (Syllabus.)
The Court of Appeals also cites section 6 of Rule 124, which provides that if the
procedure which the court ought to follow in the exercise of its jurisdiction is not
specifically pointed out by the Rules of Court, any suitable process for mode of
proceeding may be adopted which appears most conformable to the spirit of the said
Rules. That provision is not applicable here for the simple reason that the procedure
which the court ought to follow in the exercise of its jurisdiction is specifically pointed
out and prescribed in detail by Rules 74, 76, and 77 of the Rules of Court.
The Court of Appeals also said "that if this case is dismissed, ordering the filing of
testate proceedings, it would cause injustice, inconvenience, delay, and much expense
to the parties." We see no injustice in requiring the plaintiff not to violate but to comply
with the law. On the contrary, an injustice might be committed against the other heirs
and legatees mentioned in the will if the attempt of the plaintiff to nullify said will by not
presenting it to the court for probate should be sanctioned. As to the inconvenience,
delay, and expense, the plaintiff herself is to blame because she was the custodian of
the will and she violated the duty imposed upon her by sections 2, 4, and 5 of Rule 76,
which command her to deliver said will to the court on pain of a fine not exceeding

P2,000 and of imprisonment for contempt of court. As for the defendant, he is not
complaining of inconvenience, delay, and expense, but on the contrary he is insisting
that the procedure prescribed by law be followed by the plaintiff.
Our conclusion is that the Court of Appeals erred in declaring the action instituted by
the plaintiff to be in accordance with law. It also erred in awarding relief to the plaintiff in
this action on the basis of intestacy of the decedent notwithstanding the proven
existence of a will left by him and solely because said will has not been probated due to
the failure of the plaintiff as custodian thereof to comply with the duty imposed upon her
by the law.
It is apparent that the defendant Ernesto M. Guevara, who was named executor in said
will, did not take any step to have it presented to the court for probate and did not
signify his acceptance of the trust or refusal to accept it as required by section 3 of Rule
76 (formerly section 627 of the Code of Civil Procedure), because his contention is that
said will, insofar as the large parcel of land in litigation is concerned, has been
superseded by the deed of sale exhibit 2 and by the subsequent issuance of the
Torrens certificate of title in his favor.
II
This brings us to the consideration of the second question, referring to the efficacy of
the deed of sale exhibit 2 and the effect of the certificate of titled issued to the
defendant Ernesto M. Guevara. So that the parties may not have litigated here in vain
insofar as that question is concerned, we deem it proper to decide it now and obviate
the necessity of a new action.
The deed of sale exhibit 2 executed by and between Victorino L. Guevara and Ernesto
M. Guevara before a notary public on July 12, 1933, may be divided into two parts: (a)
insofar as it disposes of and conveys to Ernesto M. Guevara the southern half of
Victorino L. Guevara's hacienda of 259-odd hectares in consideration of P1 and other
valuable considerations therein mentioned; and (b) insofar as it declares that Ernesto
M. Guevara became the owner of the northern half of the same hacienda by
repurchasing it with his own money from Rafael T. Puzon.
A. As to the conveyance of the southern half of the hacienda to Ernesto M. Guevara in
consideration of the latter's assumption of the obligation to pay all the debts of the
deceased, the Court of Appeals found it to be valid and efficacious because: "(a) it has
not been proven that the charges imposed as a condition is [are] less than the value of

the property; and (b) neither has it been proven that the defendant did not comply with
the conditions imposed upon him in the deed of transfer." As a matter of fact the Court
of Appeals found" "It appears that the defendant has been paying the debts left by his
father. To accomplish this, he had to alienate considerable portions of the abovementioned land. And we cannot brand such alienation as anomalous unless it is proven
that they have exceeded the value of what he has acquired by virtue of the deed of July
12, 1933, and that of his corresponding share in the inheritance." The finding of the
Court of Appeals on this aspect of the case is final and conclusive upon the respondent,
who did not appeal therefrom.
B. With regard to the northern half of the hacienda, the findings of fact and of law made
by the Court of Appeals are as follows:
The defendant has tried to prove that with his own money, he bought from
Rafael Puzon one-half of the land in question, but the Court a quo, after
considering the evidence, found it not proven; we hold that such conclusion is
well founded. The acknowledgment by the deceased, Victorino L. Guevara, of
the said transactions, which was inserted incidentally in the document of July
12, 1933, is clearly belied by the fact that the money paid to Rafael Puzon came
from Silvestre P. Coquia, to whom Victorino L. Guevara had sold a parcel of
land with the right of repurchase. The defendant, acting for his father, received
the money and delivered it to Rafael Puzon to redeem the land in question, and
instead of executing a deed of redemption in favor of Victorino L. Guevara, the
latter executed a deed of sale in favor of the defendant.
The plaintiff avers that she withdrew her opposition to the registration of the land
in the name of the defendant, because of the latter's promise that after paying
all the debt of their father, he would deliver to her and to the widow their
corresponding shares. As their father then was still alive, there was no reason to
require the delivery of her share and that was why she did not insist on her
opposition, trusting on the reliability and sincerity of her brother's promise. The
evidence shows that such promise was really made. The registration of land
under the Torrens system does not have the effect of altering the laws of
succession, or the rights of partition between coparceners, joint tenants, and
other cotenants nor does it change or affect in any other way any other rights
and liabilities created by law and applicable to unregistered land (sec. 70, Land
Registration Law). The plaintiff is not, then, in estoppel, nor can the doctrine
of res judicata be invoked against her claim. Under these circumstances, she

has the right to compel the defendant to deliver her corresponding share in the
estate left by the deceased, Victorino L. Guevara.
In his tenth to fourteenth assignments of error the petitioner assails the foregoing
findings of the Court of Appeals. But the findings of fact made by said court are final
and not reviewable by us on certiorari. The Court of Appeals found that the money with
which the petitioner repurchased the northern half of the land in question from Rafael
Puzon was not his own but his father's, it being the proceeds of the sale of a parcel of
land made by the latter to Silvestre P. Coquia. Said court also found that the
respondent withdrew her opposition to the registration of the land in the name of the
petitioner upon the latter's promise that after paying all the debts of their father he
would deliver to her and to the widow their corresponding shares. From these facts, it
results that the interested parties consented to the registration of the land in question in
the name of Ernesto M. Guevara alone subject to the implied trust on account of which
he is under obligation to deliver and convey to them their corresponding shares after all
the debts of the original owner of said land had been paid. Such finding does not
constitute a reversal of the decision and decree of registration, which merely confirmed
the petitioner's title; and in the absence of any intervening innocent third party, the
petitioner may be compelled to fulfill the promise by virtue of which he acquired his title.
That is authorized by section 70 of the Land Registration Act, cited by the Court of
Appeals, and by the decision of this Court in Severino vs. Severino, 44 Phil., 343, and
the cases therein cited.
Upon this phase of the litigation, we affirm the finding of the Court of Appeals that the
northern half of the land described in the will exhibit A and in original certificate of title
No. 51691 still belongs to the estate of the deceased Victorino L. Guevara. In the event
the petitioner Ernesto M. Guevara has alienated any portion thereof, he is under
obligation to compensate the estate with an equivalent portion from the southern half of
said land that has not yet been sold. In other words, to the estate of Victorino L.
Guevara still belongs one half of the total area of the land described in said original
certificate of title, to be taken from such portions as have not yet been sold by the
petitioner, the other half having been lawfully acquired by the latter in consideration of
his assuming the obligation to pay all the debts of the deceased.
Wherefore, that part of the decision of the Court of Appeals which declares in effect that
notwithstanding exhibit 2 and the issuance of original certificate of title No. 51691 in the
name of Ernesto M. Guevara, one half of the land described in said certificate of title
belongs to the estate of Victorino L. Guevara and the other half to Ernesto M. Guevara
in consideration of the latter's assumption of the obligation to pay all the debts of the

deceased, is hereby affirmed; but the judgment of said court insofar as it awards any
relief to the respondent Rosario Guevara in this action is hereby reversed and set
aside, and the parties herein are hereby ordered to present the document exhibit A to
the proper court for probate in accordance with law, without prejudice to such action as
the provincial fiscal of Pangasinan may take against the responsible party or parties
under section 4 of Rule 76. After the said document is approved and allowed by the
court as the last will and testament of the deceased Victorino L. Guevara, the heirs and
legatees therein named may take such action, judicial or extrajudicial, as may be
necessary to partition the estate of the testator, taking into consideration the
pronouncements made in part II of this opinion. No finding as to costs in any of the
three instances.

[G.R. No. L-5405. January 31, 1956.]


ERNESTO M. GUEVARA, Petitioner, vs. ROSARIO GUEVARA and PEDRO C.
QUINTO, Respondents.
DECISION
CONCEPCION, J.:
This is a petition for review by certiorari of a decision of the Court of Appeals. The
pertinent facts are set forth in said decision, from which we quote:
This case being the sequel to, and aftermath of, a previous litigation between the
parties that reached the Supreme Court, through the former Court of Appeals, it
becomes necessary to restate the essential antecedent facts to view the issues in
proper perspective. For this purpose, it is important to recall that on August 26, 1931,
Victorino L. Guevara, a resident of Bayambang, Pangasinan, executed a will (Exhibit
A), distributing assorted movables and a residential lot among his children, Rosario and
Ernesto Guevara, and his stepchildren, Vivencio, Eduvigis, Dionista, Candida, and Pio
Guevara. To his second wife Augustia Posadas, the testator bequeathed, in addition to
various movables, a portion of 25 hectares to be taken out of a 259 odd hectare parcel
outlined in Plan Psu-68618, plus another five (5) hectares in settlement of her widows
usufruct. The balance of the 259 odd hectares he distributed as follows:
100 hectares reserved for disposal during the testators lifetime and for payment of his
debts and family expenses;
108.0854 hectares to his legitimate son Ernesto Guevara, including therein 43.2342
hectares by way of mejora;

of the tract covered by certificate of title No. 51691 as the portion that should
correspond to her (Rosario) by way of legitime.
The case reached the former Court of Appeals in due course and was decided in
Rosario Guevaras favor (Exhibit E); but upon certiorari, the Supreme Court modified
the judgment in December, 1943, as follows (Exhibit F);
Wherefore, that part of the decision of the Court of Appeals which declares in effect
that notwithstanding exhibit 2 and the issuance of original certificate of title No. 51691
in the name of Ernesto M. Guevara, one-half of the land described in said certificate of
title belongs to the estate of Victorino L. Guevara and the other half to Ernesto M.
Guevara in consideration of the latters assumption of the obligation to pay all the debts
of the deceased, is hereby affirmed; but the judgment of said court insofar as it
awarded any relief to the Respondent Rosario Guevara in this action is hereby
reversed and set aside, and the parties are hereby ordered to present the document
Exhibit A to the proper court for probate in accordance with law, without prejudice to
such action as the provincial fiscal of Pangasinan may take against the responsible
party or parties under section 4 of Rule 76. After the said document is approved and
allowed by the court as the last will and testament of the deceased Victorino L.
Guevara, the heirs and legatees herein named may take such action, judicial or
extrajudicial, as may be necessary to partition the estate of the testator, taking into
consideration the pronouncements made in part II of this opinion. No finding as to costs
in any of the three instances. (Appellants Brief, pp. 13-14.)
Claiming to act pursuant to the foregoing decision, Rosario Guevara commenced on
October 5, 1945, special proceedings No. 2646 in the Court of First Instance of
Pangasinan for the probate of the will of Victorino Guevara. In paragraph 10 of the
petition, it was alleged:

21.6171 hectares to mi hija natural reconocida Rosario Guevara.

10. Que dicho testamento, o sus disposiciones testamentarias, ha sido de jure


revocado, o revocados, en cuanto a la parcela de terreno de 259 hectareas descrita en
Ernesto Guevara was appointed executor without bond.
dicho testamento, por haber el testador enajenado o dispuesto intervivos de la misma
On July 12, 1933, the same testator executed a deed of sale in favor of Ernesto en la forma mencionada en las tres decisiones supra-mencionadas; chan
Guevara, conveying to the latter the southern half of the 259-hectare lot heretofore roblesvirtualawlibraryy que la solicitante pide la legalizacion de dicho testamento tan
mentioned, and expressly recognized Ernesto Guevara as owner of the northern half.
solo para los efectos del reconocimiento de hija natural hecha en dicho testamento a
Prior to this sale, on November 1, 1932, Victorino and his son Ernesto had jointly favor de la demandante y en obediencia al mandato de la Corte Suprema en su
applied for registration of the big parcel (case No. 15174), but in view of the sale from decision supra. (Record on Appeal, p. 5.)
the former to the latter, the decree was issued in the name of Ernesto Guevara Notice of the petition having been duly published pursuant to Rule of Court 77, section
exclusively and for the whole tract, a certificate of title (No. 51691 of Pangasinan) being 4, Ernesto Guevara appeared and opposed the probate. Pedro L. Quinto, counsel for
issued in his sole name on October 12, 1933.
Rosario in the former litigation, was allowed to intervene in view of his duly recorded
Fifteen days previously, i.e., on September 27, 1933, Victorino Guevara died, but his
will was not filed for probate. About four years later, Rosario Guevara, claiming to be a
recognized natural child of the deceased Victorino, and on the assumption that he had
died intestate, brought suit against Ernesto Guevara to recover 423,492 square meters

attorneys lien.
On January 31, 1946, Ernesto Guevara, through counsel, filed a motion to dismiss the
petition on the grounds that (a) the petition itself alleged that the will was revoked; chan
(b) that whatever right to probate the parties may have has already prescribed (Record

on Appeal, p. 14); and (c) that the purpose of the probate was solely to
have Petitioner Rosario declared an acknowledged natural child of the deceased.

July 25, 1947

By order of December 9, 1946, Judge Sotero Rodas denied the motion to dismiss; chan
roblesvirtualawlibrarybut upon motion of reconsideration, Judge Maalac of the same
court, on June 23, 1937, reconsidered and set aside the previous resolution and
ordered the petition dismissed on the ground that Rosario Guevaras petition did not
ask for the probate in toto of the will, contrary to the order of the Supreme Court; chan
roblesvirtualawlibrarythat her right to petition for the probate of the testament of
Victorino L. Guevara had prescribed; chan roblesvirtualawlibraryand that her action for
judicial declaration of acknowledgment had likewise prescribed.

August 2, 1947

An amended petition for the probate of the will in toto and another petition to reconsider
the previous order were subsequently denied; chan roblesvirtualawlibrarythe former on
the ground that there was a radical change of theory from that embodied in the original
petition, and the second for the same reasons stated in the order of June 23, 1947.
Rosario L. Guevara and Pedro L. Quinto thereupon brought the case on appeal to this
Court, assigning no less than twenty (20) alleged errors committed by the court below.
(Guevara vs. Guevara, C.A. G. R. No. 5416-R, promulgated December 26, 1951;
see Appendix to brief for the Petitioner-Appellant, pp. 1-6.)
The dispositive part of the decision of the Court of Appeals reads as follows:

Motion for admission of Amended Petition.


Appellants motion to postpone hearing on petition for reconsideration and motion for
admission of Amended Petition.
August 10, 1947
Appellants urgent motion for continuance of hearing on
Reconsideration as well as Motion to Admit Amended Petition.

joint

petition for

August 25, 1947


Motion for extension of time to file memorandum.
September 1, 1947
Memorandum for Appellants submitted.
October 7, 1947
Memorandum for Appellee submitted.
October 14, 1947

Appellants petition for ten (10) days to file reply memorandum.


The order of dismissal of the petition for probate is reversed and the court of origin
ordered to reinstate the petition, and to hear and decide whether the will of Victorino November 1, 1947
Guevara, deceased, should be allowed to probate. Costs against Appellees in both Appellants petition to file reply memorandum on or before November 9, 1947.
instances. (Ibid.)
November 8, 1947
In his appeal therefrom, Petitioner Ernesto M. Guevara raises the following questions,
Appellants petition for extension to file reply memorandum.
to wit: (a) Did Respondents herein duly perfect their appeal from the decision of the
Court of First Instance of Pangasinan? (b) Did the Court of Appeals have jurisdiction to November 18, 1947
entertain said appeal? (c) Is the petition for probate of the alleged will of the deceased Verified reply of Appellant Rosario Guevara.
Victorino L. Guevara barred by the statute of limitations?
November 24, 1947
(1) With reference to the first question, Petitioner has submitted the following
statement 1 of the steps taken since June 23, 1947, date of the resolution of Judge Reply memorandum of Pedro C. Quinto filed.
Maalac, dismissing the petition for probate of the last will and testament of Victoriano January 12, 1948
L. Guevara:
Court denies both petitions of July 14 and 25, 1947.
June 23, 1947
January 24, 1948
Date of Resolution appealed from.
Notice of appeal to Supreme Court and petition for thirty (30) days extension
July 14, 1947
by Appellant Rosario Guevara.
Date of Joint Petition for Reconsideration filed by Appellants.

January 29, 1948

July 25, 1947

Order granting petition for extension.

Date of Amended petition for probate of will.

February 1, 1948

Another notice of appeal to Supreme Court and motion for thirty (30) days extension
by AppellantRosario Guevara.
February 28, 1948

Filing of amended joint record on appeal. (This is also again so defective and
incomplete as to constitute another mere token record on appeal as required by the
Rules.)

Appellants ex-parte petition for further extension.

August 24, 1948

March 6, 1948

Appellants petition for ten (10) days period to reply to objection, if any was to be filed.

Original joint Record on Appeal filed. (This was so defective and incomplete it consisted
of mere disjointed sheets of paper intercalated with one another and was a mere token
record on appeal.)

August 27, 1948

March 8, 1948

Appellants reply to objection.

Another joint petition for reconsideration of Appellants.

October 20, 1948

March 11, 1948


Appellees objection to record on appeal.

Court order sustaining objection and gives Appellants fifteen (15) days from notice to
redraft record on appeal.

March 17, 1948

November 3, 1948

Verified reply of Appellants to objection.


March 18, 1948

Appellants joint petition to reconsider order of disapproval of Amended Record on


Appeal.

Appellees objection to joint petition for reconsideration.

November 3, 1948

June 19, 1948

Appellants file re-amended joint record on appeal. (This again disregarded the orders of
the court regarding the contents of the record on appeal.).

Appellants memorandum in support of the joint petition for reconsideration.


July 23, 1948
Order of denial of Joint Petition and disapproving original record on appeal as
incomplete and giving Appellants within 10 days from notice.
July 26, 1948

Appellees objection to amended record on appeal.


September 8, 1948

November 22, 1948


Appellee objected to approval of re-amended joint record on appeal and prayed that
order appealed from be declared final.
March 22, 1949

July 28 and 29, 1948

Court sustains Appellees objection to record on appeal denying petition for


reconsideration andAppellants given fifteen (15) days from notice to satisfy
requirements of courts previous order.

Appellants received copy of order of July 23, 1948.

April 8, 1949

August 1, 1948

Appellants file in Supreme Court petition for certiorari and mandamus attacking order of
June 23, 1947.

Amended Notice of Appeal to the Court of Appeals instead of to the Supreme Court.

Petition for five (5) days extension to file amended Record on Appeal filed
by Appellant Pedro C. Quinto.
August 10, 1948
Appellants Joint Petition for last extension of two (2) days.
August 10, 1948

April 11, 1949


Appellant Quintos petition for fifteen (15) days extension to file Re-amended Record on
Appeal.
April 12, 1949
Supreme Court denies petition off-hand.

April 16, 1949


Appellant Rosario Guevaras motion for fifteen (15) days extension for the same
purpose.
April 21, 1949
Court granted extension prayed for to expire May 1, 1948.
April 21, 1949
Second Re-Amended Record on Appeal filed.

months until the date of this writing has elapsed, thus establishing a record-holding
delay which should not be sanctioned by the Courts as prejudicial to the administration
of justice.
(b) Because Appellants, in violation of Rule 48, section 3, did not diligently prosecute
their appeal by failing to have the record sent up to this Honorable Court within thirty
(30) days from the time their Second Re-amended Record on Appeal was approved on
September 28, 1949; and it was only so transmitted on December 8, 1949, that is after
the lapse of two (2) months and ten (10) days.

Appellees opposition to Second Re-Amended Record on Appeal.

(c) Because, at any rate, the first Amended Joint Record on Appeal was filed beyond
the extension granted by the Court and, consequently, the Appellants right to appeal
has lapsed. (Exhibit A, pp. 1-2).

June 29, 1949

The Court of Appeals denied said motion to dismiss for the following reasons:

Appellants joint notice of hearing on Second Re-Amended Record on Appeal for July
12, 1949.

A preliminary question was posed by the Appellee who prayed for the dismissal of the
appeal on the ground that Petitioners-Appellants had unreasonably delayed the
perfection of the appeal, as the Second Re-amended Joint Record on Appeal was not
certified to this Court until December, 1949. After considering the voluminous record,
and the arguments of both parties, we are of the opinion that both parties have
contributed to the delay with lengthy memoranda, and repeated motions and
objections. Moreover, the points in question are important enough to deserve adequate
consideration upon the merits. Wherefore, the motion to dismiss the appeal should be
and is hereby, overruled and denied. (Appendix to Brief for the Petitioner-Appellant, pp.
6- 7.)

June 11, 1949

July 10, 1949


Appellants joint reply to opposition.
July 12, 1949
Action on record on appeal deferred on petition of Atty. Quinto.
September 3, 1949
Appellant Quintos notice of hearing on Second Re-Amended Record on Appeal for
September 28, 1949.
September 28, 1949
Order of court approving same.
December 8, 1949
Clerk of lower court sends records to appellate court.
December 10, 1949
Appellant Quintos motion ex-parte to have records sent up to appellate court.

It is urged by Petitioner herein that Respondents appeal from the decision of the Court
of
First
Instance
of
Pangasinan
had
not
been
duly
perfected
because:chanroblesvirtuallawlibrary (a) the original of the record on appeal did not
comply with the Rules of Court; chan roblesvirtualawlibrary(b) the record on appeal was
filed after the lapse of the reglementary period; chan roblesvirtualawlibrary(c) there has
been an unprecedented delay in the filing of a satisfactory record on appeal; chan
roblesvirtualawlibraryand (d) the appeal should be deemed abandoned for violation of
Rule 48, section 3, of the Rules of Court.

The first ground is predicated upon the fact that, instead of transcribing the motions,
petitions, orders and resolutions incorporated in the original record on
(Petitioner-Appellants Brief, pp. 41-47.)
appeal, Respondents herein merely attached to the original copy of said record on
Based upon the foregoing, Oppositor and Appellee Ernesto M. Guevara filed, with the
appeal, filed with the Court of First Instance of Pangasinan, their own copies of said
Court
of
Appeals,
a
motion
praying
that
the
appeal
be
motions, petitions, orders and resolutions. Accordingly, the copy of said record on
dismissed:chanroblesvirtuallawlibrary
appeal furnished to Petitioner herein did not contain or enclose the aforementioned
(a) Because due to the Appellants many and repeated dilatory tactics, the prosecution parts of the record. It appears, however, that the Respondents were given several
of their appeal has been unduly and unreasonably delayed for a period which should extensions of time within which to comply with the pertinent provisions of the Rules of
strike anyone as totally without justification. The resolution appealed from was dictated Court and that Respondents eventually did so. There being no question about the
by the lower court on June 23, 1947, so that a period of over two (2) years and nine (9)

authority of the court of first instance to grant said extensions of time, it is clear that the
first ground, relied upon byPetitioner herein, is untenable.
In support of the second ground, it is alleged: (a) that the original record on appeal was
filed by Pedro C. Quinto only, and does not inure to the benefit of Rosario Guevara;
and (b) thatRespondents had lost their right to appeal by the lapse of the reglementary
period. As regards the first proposition, Petitioner asserts that Respondent Pedro C.
Quinto had withdrawn his appearance as counsel for Respondent Rosario
Guevara; that Quinto had, thereafter, intervened in the case in his own behalf, in order
to enforce his attorneys lien, as former counsel for Rosario Guevara; that,
consequently, the original record on appeal and the petitions for extension of time to file
an amended record on appeal, filed by Pedro C. Quinto, were good only insofar as he
is concerned, and cannot profit Rosario Guevara, she having ceased to be his client
long before the filing of said original record on appeal and petitions for extension of
time; chan roblesvirtualawlibrarythat this interest in the case arises from his rights as
former attorney for Respondent Rosario Guevara, and, as such, is subordinate to, and
dependent upon, the interest therein of said Rosario Guevara and the success of her
claim therein; chan roblesvirtualawlibraryand that, her appeal not having been duly
perfected, his appeal must be deemed to have no legal effect. There is no merit in this
pretense, for it appears, at the foot of said record on appeal, that Pedro C. Quinto had
filed the same, for himself as Appellant and in behalf of Rosario Guevara, who
authorized him to perfect the appeal for both Appellants, and that similar statements
were made in the body and at the foot of said petitions for extension of time. It is clear,
therefore, that the aforementioned record on appeal and motions should be deemed
submitted, also, by Respondent Rosario Guevara. The position then held by Pedro C.
Quinto, as special prosecutor in the office of the Solicitor General, did not nullify his
aforesaid acts on behalf of Rosario Guevara. Besides, said acts would seem to have
been performed by him, more as attorney-in- fact than as counsel for Rosario Guevara,
and this merely in connection with the perfection of her appeal. We do not find therein
anything objectionable, either legally or morally, in the light of the circumstances
surrounding the case.
The second proposition is based upon the following reasons:chanroblesvirtuallawlibrary
(a) The aforementioned record on appeal and motions for extension of time filed by
Quinto on behalf of Rosario Guevara did not inure to her benefit, for which reason the
reglementary period to appeal had expired before the perfection of her appeal. For the
reasons already adverted to, this argument is clearly untenable.
(b) The petition for reconsideration filed by Respondents on July 14, 1947, did not
suspend the running of the period to perfect the record on appeal, because said petition
did not comply with the provisions of Rule 37, section 1, of the Rules of Court, reading
as follows:chanroblesvirtuallawlibrary

Within thirty days after notice of the judgment in an action, the aggrieved party may
move the trial court to set aside the judgment end grant a new trial for one or more of
the following causes materially affecting the substantial rights of said
party:chanroblesvirtuallawlibrary
(a) Fraud, accident, mistake or excusable negligence which ordinary prudence could
not have guarded against and by reason of which such aggrieved party has probably
been impaired in his rights;
(b) Newly discovered evidence, which he could not, with reasonable diligence, have
discovered, and produced at the trial, and which if presented would probably alter the
result;
(c) Because excessive damages have been awarded, or the evidence was insufficient
to justify the decision, or it is against the law.
Said petition for reconsideration appears, however, to be predicated, in effect, upon the
ground that the evidence is insufficient to justify the decision of the court of first
instance, and that said decision is contrary to law. It partakes, therefore, of the nature of
a motion for new trial, stating specifically the reasons in support thereof, and, hence, it
suspended the period to appeal until the determination of said motion.
Relative to the alleged unprecedented delay in the filing of a satisfactory record on
appeal, we agree with the finding of the Court of Appeals to the effect that the delay
was due to the acts of the Respondents, as well as of the Petitioner herein, for both had
asked several postponements and extensions of time, filed memoranda and reply
memoranda, and raised or provoked a number of other issues or incidents which
necessarily delayed the perfection of the appeal. Obviously,Petitioner should not be
allowed to profit by said delay, to which he had actively contributed. 1
Lastly, Petitioner maintains that, although the record on appeal had been approved on
September 28, 1949, it was not forwarded to the Court of Appeals until December 8,
1949. Section 3 of Rule 48 of the Rules of Court provides:chanroblesvirtuallawlibrary
If the record on appeal is not received by the Court of Appeals within thirty days after
the approval thereof, the Appellee may, upon notice to the Appellant, move the court to
grant an order directing the clerk of the lower court forthwith to transmit such record on
appeal or to declare the same abandoned for failure to prosecute.
Considering that Respondents herein were not notified of the approval of the record on
appeal until December 8, 1949, on which date the record on appeal was forwarded to
the Court of Appeals, and that the aforementioned provision of the Rules of Court does
impose upon said court the mandatory duty to declare the appeal abandoned for failure
to prosecute, we believe that no error was committed in giving due course to the appeal
and that the same has been duly perfected.
(2) Did the Court of Appeals have jurisdiction to try the case, on appeal from the
decision of the court of first instance? Petitioner maintains the negative, upon the

ground that the appeal involved only questions of law. This is not correct, for the very
motion for reconsideration adverted to above, indicated that the appeal raised some
issues of fact, such as, for instance, whether or not the will in question was in the
possession of Respondent Rosario Guevara and whether RespondentQuinto had been
authorized by her to perfect the appeal on her behalf. At any rate, the case is now
before us and, upon examination of the record and consideration of all the issues
therein raised, we are of the opinion that, had the appeal been forwarded directly to this
Court, we would have disposed of it in the manner set forth in the decision of the Court
of Appeals, the review of which is sought by herein Appellant.
(3) The last question for determination in this case is whether or not the petition for
probate of the will of Victorino L. Guevara is barred by the statute of limitations,
considering that the testator died on September 27, 1933, and that the petition for
probate of said will was filed twelve (12) years later, or, to be exact, on October 5, 1945.
The Court of Appeals resolved the question in the negative, upon the following
grounds:chanroblesvirtuallawlibrary
We are of the opinion that the Court below was in error when it declared that the
petition for probate of the will of Victorino Guevara was barred by prescription. The
provision of Article 756 of the old Civil Code (1042 of the New) and of Rule 76 of the
Rules of Court, reiterating those of the old Code of Civil Procedure (Act 190), point out
that the presentation of a decedents will to the competent court has always been
deemed by our law as more of a duty than a right, and the neglect of such obligation
carries with it the corresponding penalty and it is inconsistent with that policy that the
court should refuse to admit wills to probate, without inquiry into their validity. The
authority given to testators to dispose freely of a portion of their estate would be
imperfectly safeguarded, unless adequate measures were provided by the state to
assure that the wishes of the deceased would be carried out. Because the decedent
may no longer act to have his testamentary dispositions duly executed, the state
authority must take over the opposite vigilance and supervision, so that free
testamentary disposition does not remain a delusion and a dream. This was expressly
recognized by the Supreme Court in its previous decision, G. R. No. 48840 (Exhibit E)
when it said:chanroblesvirtuallawlibrary
cralaw We hold that under section 1 of Rule 74, in relation to Rule 76, if the decedent
left a will and no debts and the heirs and legatees desire to make an extrajudicial
partition of the estate, they must first present that will to the court for probate and divide
the estate in accordance with the will. They may not disregard the provisions of the will
unless those provisions are contrary to law. Neither may they do away with the
presentation of the will to the court for probate, because such suppression of the will is
contrary to law and public policy. The law enjoins the probate of the will and public
policy requires it, because unless the will is probated and notice thereof given to the
whole world, the right of a person to dispose of his property by will may be rendered
nugatory, as is attempted to be done in the instant case. Absent legatees and devisees,

or such of them as may have no knowledge of the will, could be cheated of their
inheritance thru the collusion of some of the heirs who might agree to the partition of
the estate among themselves to the exclusion of others. (Italics supplied)
In holding the statute of limitations applicable to the probate of wills, the court below
failed to notice that its doctrine was destructive of the right of testamentary disposition
and violative of the owners right to control his property within the legal limits. The
appealed order in fact leaves wills at the mercy and whim of custodians and heirs
interested in their suppression. The lower court would in effect abdicate the tutelary
power that passed to the Republic from the former sovereigns, that potestad suprema
que en mi reside para velar por el puntual cumplimiento de las ultimas voluntades,
asserted as one of the royal prerogatives in the Real Cedula of March 18, 1776.
It is not without purpose that Rule of Court 77 prescribes that any person interested in
the estate may, at any time after the death of the testator, petition the court having
jurisdiction to have the will allowed. Taken from the Code of Procedure of California,
this provision has been interpreted as meaning that the statute of limitations has no
application to probate of wills. In the case of In re Humes Estate, 179 Calif. 338, 176
Pac. 681, the California Supreme Court ruled that:chanroblesvirtuallawlibrary
The chapter of the Code relating to the probate of wills does not provide for opposition
to such probate on the ground of the bar of the statute of limitations, but, in effect,
excludes it from the category of grounds allowed as a basis for such opposition.
Section 1299 declares that any person interested in the estate may at any time after
the death of the testator, petition the court having jurisdiction to have the will proved.
This implies that there is no arbitrary time limit.
As additional reasons, the same Court stated:chanroblesvirtuallawlibrary
cralaw Section 1317 declares:chanroblesvirtuallawlibrary If the court is satisfied, upon
the proof taken or from the facts found by the jury that the will was duly executed and
that the will testator at the time of its execution was of sound and disposing mind and
not acting under duress menace fraud, or undue influence, a certificate of the proof and
the facts found, signed by the judge and attested by the seal of the court, must be
attached to the will.
This excludes the bar of the statute of limitation from consideration as one of the
matters which may be shown in opposition to the probate. This is further emphasized
by section 1341, which, in substance, declares that, if upon the verdict of the jury the
facts mentioned in section 1317 as aforesaid appear to be established, the court must
admit the will to probate. Section 1314 thus makes it imperative that the court shall
admit the will to probate if the execution is proven and the grounds of opposition
authorized by section 1312 are not established. This clearly implies that no grounds of
opposition other than those enumerated in section 1312 may be set up, and it leaves
no place for the application of the statute of limitations.

It is further to be observed that, notwithstanding the positive and comprehensive


language of sections 343 and 369, if taken literally, there can be no doubt that they
cannot apply to all special proceedings of a civil nature. Proceedings for a change of
name, or in arbitration, or for voluntary dissolution of a corporation, or for guardianship,
or for a married woman to become a sole trader, are all within the definition of the
phrase, and each is enumerated, classed, and defined as such proceeding by the
Code. If the statute of limitations applied, it would begin to run against such
proceedings as soon as the right to institute them accrued. Yet from the very nature of
these proceedings it is obvious that neither of them could be subject to such limitation.
This construction of these Code provisions is confirmed by the long-continued and
uniform practice and the universal understanding of the bench and bar of the state on
the subject.
xxx

xxx

xxx

Action to quiet title frequently involve wills of persons who have died many years
before the action was begun. The section contemplates that such a will, although not
yet probated, may be construed in the action and may be afterwards probated, and it
clearly shows that the Legislature did not understand that the right to probate such will
would be barred if the testator had died more than four years before the petition for
probate was filed.
This uniform practice and understanding of the bench and bar, and of the legislative
department of the state also, is a strong argument to the effect that the statute of
limitations does not apply to such proceedings. The authorities on the effect of such
long acquiescence are numerous.
The Statute of Limitations upon which the court below has relied, sections 38 to 50 of
the old Code of Civil Procedure, Act 190, undertakes to fix limits for the filing of civil
actions, but none for special proceedings of which probate is admittedly one. The
distinction is not purely verbal, but based on differences that make the limitation to
actions inapplicable to special proceedings. In this regard, the Supreme Court of New
York
has
adequately
remarked
(In
re
Canfields
Will,
300
NYS
502):chanroblesvirtuallawlibrary
A Respondent in a private proceeding owes no legal duty or obligation to the
proponent as such, wherefore it is impossible for him to violate such non-existent
obligation. Furthermore such a proceeding is not instituted for the vindication of any
personal right to the proponent. The subject-matter is therefore wholly absent which
could give rise to any cause of action against anyRespondent therein.
The primary purpose of the proceeding is not to establish the existence of the right of
any living person, but to determine whether or not the decedent has performed the acts
specified by the pertinent statutes which are the essential prerequisites to personal
direction of the mode of devolution of his property on death. There is no legal but
merely a moral duty resting upon a proponent to attempt to validate the wishes of the

departed, and he may and frequently does receive no personal benefit from the
performance of the act.
One of the most fundamental conceptions of probate law, is that it is the duty of the
court to effectuate, in so far as may be compatible with the public interest, the
devolutionary wishes of a deceased person (Matter of Watsons Will, 262 N.Y. 284, 294,
186 N.E. 787; chan roblesvirtualawlibraryMatter of Marrimans Estate, 124 Misc. 320,
325, 208 N.Y.S. 672; chan roblesvirtualawlibraryFoley, S. affirmed 217 App. Div. 733,
216 N.Y.S. 842; chan roblesvirtualawlibraryMatter of Lensmans Estate, 137 Misc. 77,
78, 243 N.Y.S. 126, Henderson, S., Matter of Drakes Estate, 160 Misc. 587, 598, 290
N.Y.S. 581). To that end, the court is, in effect, an additional party to every litigation
affecting the disposal of the assets of the deceased. Matter of Van Valkenburghs
Estate, 164 Misc. 295, 296, 298 N.Y.S. 219. A determination, therefore, that the mere
non-action of a person upon whom no legal duty rested in this regard, could have the
effect of subverting the wishes of one who was no longer able to protect his own
unquestionable rights, would strike at the very foundation of all conceptions of justice
as administered in probate courts.
These decisions are of high persuasive value (Cu vs. Republic, G. R. L-3018, July 18,
1951); chan roblesvirtualawlibrarythey represent the trend of authority (57 Am. Jur.
585), and enable us to conclude that reason and precedent reject the applicability of
the Statute of Limitations to probate proceedings, because these are not exclusively
established in the interest of the surviving heirs, but primarily for the protection of the
testators expressed wishes, that are entitled to respect as an effect of his ownership
and right of disposition. If the probate of validly executed wills is required by public
policy, as declared by the Supreme Court in the previous case, G.R. 48840 (Exhibit E),
the state could not have intended the statute of limitations to defeat that policy.
It is true, as ruled by the trial court, that the rights of parties should not be left hanging
in uncertainty for periods of time far in excess of the maximum period of ten years
allowed by law; chan roblesvirtualawlibrarybut the obvious remedy is for the other
interested persons to petition for the production of the will and for its probate, or to
inflict upon the guilty party the penalties prescribed by Rule 76 or declare the
unworthiness of the heir under the Civil Code for concealing or suppressing the
testament; chan roblesvirtualawlibrarybut not to dismiss the petition for probate,
however belatedly submitted, and thereby refuse sanction to testamentary dispositions
executed with all the formalities prescribed by law, incidentally prejudicing also those
testamentary heirs who do not happen to be successors ab intestato. That in this
particular case the appealed rule may not work injustice would not excuse its adoption
as a general norm applicable to all cases.
It is likewise reasonable to assume that if the Supreme Court had considered the tenyear limitation applicable to probate proceedings, it would not have ordered the parties
on December 29, 1943 to present the document Exhibit A to the proper court for
probate in accordance with law, because the ten years from the death of the testator

expired in September of that same year, two months before the decision. It is safe to
assume that the high Court would not order a useless step. The reasoning that the
phrase in accordance with law was a qualification signifying if still legally possible,
appears to be far-fetched and unjustified. The plain import of the words employed by
the high Court is that the probate should follow the procedure provided for the
purpose.

shares, the order refusing the probate remains indefensible. If the other heirs were not
interested, there remained the wishes of the testator to be supported and protected, if
validly expressed. If the heirs had distributed the estate, the distribution was illegal and
improper unless the will be first probated. The Supreme Court so ruled in its previous
decision (G. R. 48840) heretofore quoted.

Even if the decedent left no debts and nobody raises any question as to the
authenticity and due execution of the will, none of the heirs may sue for the partition of
The other reasons advanced by the court a quo in support of its order dismissing the the estate in accordance with that will without first securing its allowance or probate by
petition are also untenable. The allegation contained in paragraph 10 of the original the court:chanroblesvirtuallawlibrary first, because the law expressly provides that no
petition, that the will, or its testamentary dispositions, had been de jure revoked in so will shall pass either real or personal estate unless it is proved and allowed in the
far as the parcel of 259 hectares described in said will is concerned, does not justify proper court; chan roblesvirtualawlibraryand, second, because the probate of a will,
the finding that the probate would be pointless. What is alleged is a partial revocation, which is a proceeding in rem, cannot be dispensed with and substituted by any other
only as to the parcel of land affected; chan roblesvirtualawlibrarybut as previously proceeding, judicial or extrajudicial, without offending against public policy designed to
shown, the will disposed of other property besides that one. And even granting that the effectuate the testators right to dispose of his property by will in accordance with law
next allegation to the effect that Plaintiff sought to probate only for the purposes of her and to protect the rights of the heirs and legatees under the will thru the means
acknowledgment as natural child in said will, constitutes an averment that the will had provided by law, among which are the publication and the personal notices to each and
been fully revoked, the same would at the most constitute a conclusion or inference all of said heirs and legatees. Nor may the court approve and allow the will presented in
that the lower court was not bound to admit. Because the Appellant claimed or believed evidence in such an action for partition, which is one in personam, any more than it
that the revocation of the will as to the large parcel of land, constituted a total could decree the registration under the Torrens system of the land involved in an
revocation of the testament is no reason why the court should concur in the same belief ordinary action for revindicacion or partition.
or conclusion, especially when the will itself, appended to the petition, showed that From whatever angle the case is viewed, a hearing on the allowance of the will is
there were other properties and other heirs or legatees, and the trial court had before it unavoidable. The persistent, albeit obnoxious, attempts of Rosario Guevara to sidetrack
the decision of the Supreme Court ordering the filing of the will for its probate because, the will are not remedied by dismissing the petition for probate of will, and allowing
as stated in its decision, such a step was enjoined by law and public policy. Moreover, Ernesto to retain a greater interest than that intended by the testator. (Appendix to brief
the defect, if any, incurred in failing to ask for the probate in toto of the will, was for the Petitioner-Appellant, pp. 7-15, 17-20.)
subsequently cured and corrected in the amended petition, where not only the We are fully in accord with these findings which we adopt as ours.
objectionable statements were eliminated, but others added indicating the existence of
In view of the foregoing, the decision appealed from is hereby affirmed, with the costs
a partible estate.
of this instance against the Petitioner.
Assuming that the original petition violated the order of the Supreme Court in so far as
it did not ask for the allowance of the entire will, the court below erred in dismissing the
petition, for it thereby sanctioned further disobedience to the order of the superior court.
Once again, it must be repeated that the order of dismissal failed to take into account
that the case involved not only the interests of Rosario Guevara, and those of
the Appellee Ernesto Guevara and the other legatees, but specially the express desires
of the testator; chan roblesvirtualawlibraryand that the protection and defense of the
latter developed upon the court itself, since no one else made any move to enforce
them.
xxx

xxx

xxx

Even if the other heirs had failed to show interest in the case (a fact not properly
inferable from their non-intervention in the case, because the order of publication of the
petition only called for those interested to appear to contest the allowance and not to
support it) (Rec. on App., p. 7), and even if the other heirs had already received their

GR No. L-47931 June 27, 1941


Rev. P. Testamentary the late Eleuterio Pilapil. ADRIANO MENDOZA, petitioner and
appellee, vs. CALIXTO Pilapil AND OTHERS opponents and appellants.
Filemon Sotto D. and D. G. Estenzo Numeriano representing the appellants.
Messrs. Alonso and Alonso for appellee.
DIAZ, J. :
The main issues that opponents present us for resolution, to appeal the decision of the
Court of First Instance of Cebu, issued in the record of testamentary No. 407 of the
Court, may be reduced to following the vessel.
1st Cebu If the court could appoint the March 4, 1939, the appellee as special
administrator of estate of the decedent relict P. Eleuterio Pilapil (testamentary record
No. 407), being as it was then acting as administrator of the same goods from the
February 7, 1939, the appellant Calixto Pilapil, who promoted the day before, the same
record Intestate decedent P. Eleuterio Pilapil, in this Court (file No. 399, Court of
Cebu); Y
2nd If proceeded and proceeds legalization as a will or living will of the late P. Eleuterio
Pilapil, the car obrante document as Exhibit A that is a duplicate of the coal Exhibit C.
The relevant facts must be taken into account in resolving the issues proposed are, as
reflected by the appealed decision and the same documents as the Court declare to be
last will and testament of the late P. Eleuterio Pilapil, those below are related :
Father Eleuterio Pilapil, being priest of the parish of Mualboal of the Province of Cebu,
died in the city of that name on 6 December 1935. absence of any submitted his will
after his death, at least until early February 1939, his brother Calixto Pilapil promoted
the 6th of such month and year, the record of intestacy No. 399 to ask fuesenombrado
manager of the relict goods. Received test the request submitted to elindicado end prior
publication of notices presritos by law, and hearing the court prior to the
quecomparecieron to oppose it, among which were the same appeal and Simeona
Pilapil, the Court He granted it, immediately afterwards naming him administrator of the
Intestate. Within a few days, or the March 4, 1939, the respondent promoted in turn file
No. 407 previously has been mentioned, to call for the legalization as a testament of
the late P. Eleuterio Pilapil, of Exhibit A that is doubled to carbon Exhibit C. There are
between the clauses of these two documents, which are then inserted by its relevance
to the issues raised and also the importance
I, Eleuterio Pilapil, Priest of natural Roman Catholic Church, sixty-eight years
old, of Liloan, currently pastor of the Parish of Mualboal, Province of Cebu, IF,
enjoying health and being of sound mental faculties, hereby, publish, declare
and grant the following as MY LAST WILL aND TESTAMENT:

ART. FIRST: I institute and appointed Mr. Adrian Mendoza, my nephew, married,
of legal age and resident of the Municipality of Liloan, Cebu Province, IF,
EXECUTOR-executor of this my last will and testament:Provided , That in case
of impossibility, negligence or other cause with that embarrasses enforce this
my last will and testament, through bail, dispose and ordered it to be replaced in
the position of executor-executor of this my last will and testament, by my
cousin, Jose Cabatingan, married, of legal age, resident Municipiop of
Mualboal, Cebu Province, IF, who is responsible and will make these my
following provisions are met:
xxx xxx xxx
2nd do I have and command this my Last Will and Testament not be heard by
the Court, since this last will and testament, simply corroborates claims and
ensures the legitimacy of the documents for the sale of my property;
xxx xxx xxx
ART. SECOND: Hereby, I note that this My Last Will and Testament, which
corroborates claims and ensures the legitimacy of documents by my granted to
buyers consists of two articles; It contains sixteen provisions and is written on
three pages;
xxx xxx xxx
Cebu, Cebu, IF, nowadays November 27, 1935.
(Signed) ELEUTERIO Pilapil testator ;
at the end thereof (exhibits A and C), there is this clause Witnessing:
When shall read:
HEALTH,
We who have signed below, we state: That the pre-insert Last Will and
Testament, has been signed, and jury declared by the testator, Rev. P. Eleuterio
Pilapil in the presence of all of us and beg of the testator, we signed each of us
in the presence of us, here in Cebu, Cebu, IF, nowadays November 27, 1935.
(Signed) Pilapil WENCESLAO Witness
MARCELO Pilapil Witness
EUGENE K. Pilapil Witness

The two documents, exhibits A and C, consist of three pages; and on the left of each of
the first two margin; firms that are at the end of the main body of these documents and
their attestation clause appear; and they are, according to the evidence, signatures of
the late P. Eleuterio Pilapil, and witnesses Pilapil Wenceslas, Marcelo Pilapil and
Eugene K. Pilapil.
In place of the date both of the two documents and their attestation clause, is written
the word "Cebu" about which try to scrape but may even be that said "Mualboal"; and
also shows the numeral "27" and the name of more "November", written the latter on a
scraped word that can even be too without any difficulty, at least in Exhibit A, which
says: "October". In the last paragraph on page 2 which appears below in the first two
lines of the next page (page 3), which is the last, there is the following explicit
reference: "contains sixteen provisions and is written on three pages." At the bottom of
the pages (1) and (2) there is respectively these notes: "Go to page 2."; "Go to page
3.". And it should be noted that both the one and the other of the aforementioned
Exhibits A and C, no more than two articles ( "Art. First" and "Art. Second"), and
dieceseis provisions.
The reasons for Appellants rely to argue that legalization is not appropriate for any of
the two documents expressed as a testament of the late P. Eleuterio Pilapil, are these:
( A ) They contain erasures and alterations that explain the respondent stopped;
( B ) That has not been proven that the decedent - prescindimiento of what is stated in
those documents exhibits A and C - age was competent to test;
( C ) That has not proved that the deceased possessed the Spanish is the language in
which these documents are written;
( D ) That one of the clauses of these documents is no prohibition that aired in slab
Courts;
( E ) That neither has been prepared, signed and witnessed in accordance with the
provisions of Article 618 of the Code of Civil Procedure.
With regard to the first question, it must be said that, according to tells us the same
piece of Appeal of the appellants, both Nos. 399 and 407 records were promoted in two
different Chambers of the Court of First Instance of Cebu. The first was promoted in
Room III; and the last, in Room II. Upon hearing the Judge of one of those boards that
there was a direct relationship between daub and another, provided that the two were
known by a single judge; hence both are considered as one to avoid what the judge
said said, "inconsistency in the administration of the property of the deceased,"
referring to; P. Eleuterio late Pilapil.
Certainly did not lack reason to Cebu court to appoint special administrator in file No.
407, the appellee, because there are documents sought to legalize as a testament and
living will of the late P. Eleuterio Pilapil, has commissioned express it to be. In addition,
there was not no law that prohibits the courts hearing a record of testamentary or

intestate, appoint more than one administrator; and, if I happened is that annulled the
appointment of the appellant as an administrator, then the two mentioned records were
melted. But still; If the purpose of the appellants in proposing the question of which we
are speaking, is nullifying the appointment issued to appellee as special administrator,
is vain and futile must be said purpose, because insisting on the amounts to be
appealing to a court order appointing a special administrator; and the law does not
allow for appeal against orders of this nature. It is final disposition of the law that says,
"Do not allow appeal against the appointment of the special manager." (Art. 660, Law
No. 190.)
In addition to all this must be said that if there was an error in the appointment of the
respondent as special administrator, for the reason that other property was already
appointed by the court, the error, if such it can be called, has not been such nature that
has caused any harm to anyone, least of late Probate P. Eleuterio Pilapil.
Scratches and changes that are noted in Exhibits A and C are some facts that now, for
the first time, and in this instance, want llamr attention, when it should have been done
while the case was still in court of its origin. We can not take them into account in the
present state of the proceedings because, assuming that already existed then, can and
should be, though not the Court of Cebu, I believe that these documents did not vitiate
said in express terms; it is presumption rebuttable that "all the facts concerning the
points discussed at trial were exposed to and appreciated by the court." (Art. 334, par.
16, Law No. 190.) And indeed vitiated not because it follows from the same
circumstances, they did just to put things in their proper place. The two exhibits A and C
were prepared by the late P. Eleuterio Pilapil in Mualboal where he was parish priest
before being transladado to be treated for his illness that caused his death, the
Southern Islands Hospital of Cebu, where he died. Being based the Court on these
facts proved at trial, I stated the following: "The intervention of the three attesting
witnesses of the document took place in a haphazard way, on the occasion in which
they were to visit Eleuterio Pilapil who was ill at the Southern Islands Hospital, where
the decedent today begged them to act as witnesses of the document and had then
prepared ".
To prepare the, being in Mualboal, was no more than natural that expressed in it that
there were prepared, and leave blank the date but still put the name of the month they
were placed in clean, ie October 1935.
As for the age of the testator and as to whether he spoke Spanish is the language in
which they are written the two exhibits, or not, it should be said that being a priest and
parish priest of Mualboal, Cebu, must be presumed fundademente who had age
competent to test, and understood and spoken the Spanish, then, is generally known
that to be a parish priest, one must be a priest, and to be, many years of study in
seminaries where Spanish is spoken it is necessary a language as official as
English. Moreover, no evidence has been proved that the testator did not understand
that language.

The disposition of the testator that his "Last Will and Testament not be heard by the
court" can not strip the courts of their authority to determine whether your will is referred
legalizable or not. They are stakeholders in one way or another on an issue, which can
confer or remove jurisdiction and authority to Trubunales to resolve and decide what it
wants law to be resolved and decided. It should be noted that the law mandates under
penalty, to be delivered to the court the Wills made by a testator, then this die by the
person to whom custody has been entrusted, so certainly that can determine whether
your legalization and at the same time to dispose of their property as mandated
therein; or if the contract must be declared died intestate, not be subject to legalization
which has been granted. (. 626 631 Arts, Law No. 190.) Furthermore, the testator not
being a lawyer, it is not surprising that has been entered in his will the ban that - using
his own words - "it is aired in court" .

The fraze "to beg of the testator", coupled with that signed and signed his will in the
presence of attesting witnesses, permits and justifies the inference that the testator was
present when the last there affixed their respective signatures.

And as to the exhibits A and C can not be legalized because they were not prepared or
were signed in accordance with the law, saying that their pages are not numbered with
letters; and because in its clause Witnessing is not expressed that they were signed by
the three attesting witnesses, in the presence of the testator, suffice to call attention to
the fact that the bottom of the first page is in letters note that says clearly: " Go to the
2nd page "; and the fact that, at the bottom of the second page, there is this other note:
"Go to the 3rd page"; and suffice also draw attention to the first two lines of said third
page is the last, where, to complete the provision that is contained in the last paragraph
of the previous page, or second, it is stated:

Something more than in the case of Nayve against Mojal and Aguilar (47 Phil. Rep.,
160), which was clarified by the cause of Gumban against Gorecho and other (50 Phil.
Rep., 31), there is in this case because there there was but notes: "Pag. 1"; "Page
2."; "Page 3."; and "Pag. 4" on the respective side of the four pages of which it is
composed, and present no data already mentioned and there is also the record
inserted into the first two lines of the third page of the exhibits A and C, that they are
composed of three pages, and contain two sixteen articles and provisions.

. . . It consists of two articles; contien


sixteen provisions and is written
on three pages,
which agrees faithfully with the true facts as they appear in the aforementioned two
exhibits, because actually contain two articles and sixteen provisions, not more, not
less.
Witnessing the clause in one and another copy of the subject matter Testament, by the
three attesting witnesses who signed states that
pre-insert the Last Will and Testament,
has been signed, declared and sworn by
the testator, Rev. P. Eleuterio Pilapil
in the presence of all of us;
and immediately afterwards, it is also stated by the same witnesses that:
to beg of the testator, we signed
each of us, here in Cebu, Cebu,
IF, nowadays November 27, 1935.

The purpose of the law to establish the formalities authenticity required, is undoubtedly
ensure and guarantee their authenticity against bad faith and fraud, to prevent those
who have no right to succeed the testator will happen and win-win with the legalization
of same. Has fulfilled that purpose in the case that has been talked about because, in
the same body of the will and in the same page where the clause Witnessing appears,
that is the third, expresses the will consists of three pages and that each one of the first
two leads in part the note in letters, and partly and second pages of it. These facts
clearly excluded all fear, suspicion, or any hint of doubt that has replaced some of its
pages with another.

They are therefore perfect application to the case that is what we said in the causes of
Rodriguez against Yap, RG No. 45924, May 18, 1939; and Fortunate against De
Gorostiza (57 Phil. Rep., 456). We said in these causes, respectively, the following:
The wording of the clause Witnessing this will not technically free repairs, but is
essentially a law enforcement.
We maintain the view that should be required strict compliance with the
substantive requirements of the will, to ensure its authenticity, but at the same
time we believe that they should not be considered defects that can not affect
this end and, on the other part , be taken into account, could thwart the will of
the testator.(Rodriguez against Yap, supra .)
It should not be allowed to legal formalities hinder the use of good common
sense in considering wills and frustrate the wishes of the deceased solemnly
expressed in their wills, regarding the granting of which there is not even
the shadow of bad faith or fraud. (Fortunate against De Gorostiza, supra .)
For these reasons, finding appealed according to law the decision of the Court of First
Instance of Cebu, hereby confirm it, condemned the appellants to pay the costs. So it is
ordered.

G.R. No. 126950 July 2, 1999

HEREBY DECLARE AND MAKE MANIFEST

NELSON NUFABLE, SILMOR NUFABLE and AQUILINA NUFABLE, petitioners, vs.


GENEROSA NUFABLE, VILFOR NUFABLE, MARCELO NUFABLE, and the COURT
OF APPEALS,respondents.

1. That on August 9, 1965, Rev. Fr. Esdras Nufable died


leaving (a) Last Will and Testament (marked Exh. G)
disposing (of) his properties or estate in favor of his four
legitimate children, namely: Angel Custodio Nufable,
Generosa Nufable, Vilfor Nufable and Marcelo Nufable;

GONZAGA-REYES, J.:

2. That on March 30, 1966 the said Last Will and


Testament was probated by the Honorable Court, Court of
First Instance of Negros Oriental, and is embodied in the
same order appointing an Administratrix, Generosa
Nufable, but to qualify only if she put up a necessary bond
of P1,000.00;

This petition for review on certiorari seeks to reverse and set aside the Decision dated
November 25, 1995 of the Fifth Division 1 of the Court of Appeals for allegedly being
contrary to law.
The following facts as found by the Court of Appeals are undisputed:
Edras Nufable owned at Poblacion, Manjuyod, Negros Oriental,
consisting of 948 square meters, more or less. He died on August 9,
1965 and was survived by his children, namely: Angel Custodio,
Generosa, Vilfor and Marcelo, all surnamed Nufable. Upon petition for
probate filed by said heirs and after due publication and hearing, the
then Court of First Instance of Negros Oriental (Branch II) issued an
Order dated March 30, 1966 admitting to probate the last will and
testament executed by the deceased Edras Nufable (Exhs. B, C and C1).
On June 6, 1966 the same court issued an Order approving the
Settlement of Estate submitted by the heirs of the late ESdras Nufable,
portions of which read:

3. That herein legitimate children prefer not to appoint an


Administratrix, as agreed upon (by) all the heirs, because
they have no objection as to the manner of disposition of
their share made by the testator, the expenses of the
proceedings and that they have already taken possession
of their respective shares in accordance with the will;
4. That the herein heirs agreed, as they hereby agree to
settle the estate in accordance with the terms and
condition of the will in the following manner, to wit:
a) That the parcel of land situated in Poblacion Manjuyod,
Negros Oriental remains undivided for community
ownership but respecting conditions imposed therein (sic)
in the will;

KNOW ALL MEN BY THESE PRESENTS:


xxx xxx xxx
We, ANGEL CUSTODIO NUFABLE, GENEROSA
NUFABLE, VILFOR NUFABLE and MARCELO
NUFABLE, all of legal ages (sic), Filipinos, and with
residence and postal address at Manjuyod, Negros
Oriental, Philippines,

(Exhs. "E" and "E-1")


Two months earlier, or on March 15, 1966, spouses Angel Custodio and
Aquilina Nufable mortgaged the entire property located at Manjuyod to
the Development Bank of the Philippines [DBP] (Pre-trial Order, dated

January 7, 1992, p. 103, Original Records). Said mortgagors became


delinquent for which reason the mortgaged property was foreclosed by
DBP on February 26, 1973 (id.).

4. Paragraph 4 is denied, the truth being that the late


Angel Nufable was the exclusive owner of said property,
that as such owner he mortgaged the same to the
Development Bank of the Philippines on March 15, 1966,
that said mortgage was foreclosed and the DBP became
the successful bidder at the auction sale, that ownership
was consolidated in the name of the DBP, and that
defendant Nelson Nufable bought said property from the
DBP thereafter. During this period, the plaintiffs never
questioned the transactions which were public, never filed
any third party claim nor attempted to redeem said
property as redemptioners, and that said Deed of Sale,
Annex "B" to the complaint, is fictitious, not being
supported by any consideration; (pp. 20-21, id.)

On January 11, 1980, Nelson Nufable, the son of Angel Custodio


Nufable (who died on August 29, 1978 [TSN, Testimony of Nelson
Nufable, Hearing of August 18, 1992, p. 17]), purchased said property
from DBP (Exh. "1").
Generosa, Vilfor and Marcelo, all surnamed Nufable filed with the lower
court a complaint dated July 25, 1985 "To Annul Fraudulent
Transactions, to Quiet Title and To Recover Damages' against Nelson
Nufable, and wife, Silmor Nufable and his mother Aquilina Nufable.
Plaintiffs pray:
WHEREFORE, plaintiffs pray this Honorable Court that
after trial judgment be rendered ordering:
(a) That the said Deed of Sale (Annex "C") executed by
the Development Bank of the Philippines in favor of the
defendants be declared null and void as far as the three
fourths (3/4) rights which belongs (sic) to the plaintiffs are
concerned;
(b) That the said three fourths (3/4) rights over the above
parcel in question be declared as belonging to the
plaintiffs at one fourth right to each of them;
(c) To order the defendants to pay jointly and severally to
the plaintiffs by way of actual and moral damages the
amount of P10,000.00 and another P5,000.00 as
Attorney's fees, and to pay the costs.

The Deed of Sale (Annex "B"), referred to by the parties is a notarized


Deed of Sale, dated July 12, 1966 (marked as Exhibit "H") by virtue of
which, spouses Angel and Aquilina Nufable, as vendors, sold 3/4 portion
of the subject property to herein plaintiffs for and in consideration of
P1,000.00 (Exh. "5"). 2
On November 29, 1995, the Court of Appeals rendered judgment, the dispositive
portion 3 of which reads:
WHEREFORE, the appealed decision of the lower court is REVERSED
and SET ASIDE. A new judgment is hereby entered declaring plaintiffsappellants as the rightful co-owners of the subject property and entitled
to possession of 3/4 southern portion thereof; and defendant-appellee
Nelson Nufable to 1/4 portion.
No award on damages.
No costs.

(d) Plus any other amount which this Court may deem just
and equitable. (p. 6, Original Records)

Defendants-appellees' Motion for Reconsideration was denied for lack of merit in the
Resolution of the Court of Appeals 4 dated October 2, 1996.

In their Answer, defendants contend:


Hence, the present petition. Petitioners raise the following grounds for the petition:

1. Honorable Court of Appeals erred in considering as controlling the


probate of the Last Will and Testament of Esdras Nufable, the probate
thereof not being an issue in this case;
2. The Honorable Court of Appeals erred in not considering the fact that
the Development Bank of the Philippines became absolute, exclusive,
legal and rightful owner of the land in question, from whom petitioner
Nelson Nufable acquired the same by purchase and that, therefore, no
award can be made in favor of private respondent unless and until the
Development Bank of the Philippines' title thereto is first declared null
and void by the court.
The Court of Appeals, in its decision, stated that the trial court failed to take into
consideration the probated will of the late Esdras Nufable bequeathing the subject
property to all his four children. 5 In the present petition, petitioner present the issue of
whether or not the Last Will and Testament of Esdras Nufable and its subsequent
probate are pertinent and material to the question of the right of ownership of petitioner
Nelson Nufable who purchased the land in question from, and as acquired property of,
the Development Bank of the Philippines (DBP, for short). They contend that the
probate of the Last Will Testament and of Esdras Nufable did not determine the
ownership of the land in question as against third parties.1wphi1.nt

they have already taken possession of their respective shares in accordance with the
will." Verily, it was the heirs of the late Esdras Nufable who agreed among themselves
on the disposition of their shares. The probate court simply approved the agreement
among the heirs which approval was necessary for the validity of any disposition of the
decedent's estate. 9
It should likewise be noted that the late Esdras Nufable died on August 9, 1965. When
the entire property located at Manjuyod was mortgaged on March 15, 1966 by his son
Angel Custodio with DBP, the other heirs of Esdras namely: Generosa, Vilfor and
Marcelo had already acquired successional rights over the said property. This is so
because of the principle contained in Article 777 of the Civil Code to the effect that the
rights to the succession are transmitted from the moment of death of the decedent.
Accordingly, for the purpose of transmission of rights, it does not matter whether the
Last Will and Testament of the late Esdras Nufable was admitted on March 30, 1966 or
thereafter or that the Settlement of Estate was approved on June 6, 1966 or months
later. It is to be noted that the probated will of the late Esdras Nufable specifically
referred to the subject property in stating that "the land situated in the Poblacion,
Manjuyod, Negros Oriental, should not be divided because this must remain in common
for them, but it is necessary to allow anyone of them brothers and sisters to construct a
house therein."10 It was therefor the will of the decedent that the subject property should
undivided, although the restriction should not exceed twenty (20) years pursuant to
Article 870 11 of the Civil Code.

As a general rule, courts in probate proceedings are limited only to passing upon the
extrinsic validity of the will sought to be probated, the due execution thereof, the
testator's testamentary capacity and the compliance with the requisites or solemnities
prescribes by law. Said court at this stage of the proceedings is not called to rule on the
rule on the intrinsic validity or efficacy of the will. 6 The question of the intrinsic validity of
a will normally comes only after the court has declared that the will has been duly
authenticated.

Thus, when Angel Nufable and his spouses mortgaged the subject property to DBP on
March 15, 1966, they had no right to mortgage the entire property. Angel's right over the
subject property was limited only to 1/4 pro indivisoshare. As co-owner of the subject
property, Angel's right to sell, assign or mortgage is limited to that portion that may be
allotted to him upon termination of the co-ownership. Well-entrenched is the rule that a
co-owner can only alienate his pro indiviso share in the co-owned property. 12

The records show that upon petition for probate filed by the heirs of the late Esdras
Nufable, an Order dated March 30, 1966 was issued by then Court of First Instance of
Negros Oriental, Branch II, admitting to probate the last will and testament executed by
the decedent. 7 Thereafter, on June 6, 1966, the same court approved the Settlement of
Estate submitted by the heirs of the late Esdras Nufable wherein they agreed "(T)hat
the parcel land situated in Poblacion Manjuyod, Negros Oriental remains undivided for
community ownership but respecting conditions imposed therein (sic) in the will." 8 In
paragraph 3 thereof, they stated that "they have no objection as to the manner of
disposition of their share made by the testator, the expenses of the proceeding and that

The Court of Appeals did not err in ruling that Angel Custodio Nufable "had no right to
mortgage the subject property in its entirety. His right to encumber said property was
limited only to 1/4 pro indiviso share of the property in question." 13 Article 493 of the
Civil Code spells out the rights or co-owners over a co-owned property. Pursuant to
said Article, a co-owner shall have full ownership of his part and of the fruits and
benefits pertaining thereto. He has the right to alienate, assign or mortgage it, and even
substitute another person in its enjoyment. As a mere part owner, he cannot alienate
the shares of the other co-owners. The prohibition is premised on the elementary rule
that "no one can give what he does not have." 14

Moreover, respondents stipulated that they were not aware of the mortgage by
petitioners of the subject property.15 This being the case, a co-owner does not lose his
part ownership of a co-owned property when his share is mortgaged by another coowner without the former's knowledge and consent 16 as in the case at bar. It has
likewise been ruled that the mortgage of the inherited property is not binding against
co-heirs who never benefitted. 17
Furthermore, the Deed of Sale dated June 17, 1966 marked as Exhibit "H" executed by
spouses Angel and Aquilina Nufable in favor of respondents Generosa, Vilfor and
Marcelo wherein the former sold, ceded and transferred back to the latter the 3/4
portion of the subject property bolsters respondents' claim that there was co-ownership.
Petitioner Nelson himself claimed that he was aware of the aforesaid Deed of Sale. 18
Anent the second ground of the petition, petitioners allege that the Development Bank
of the Philippines acquired ownership of the land in question through foreclosure,
purchase and consolidation of ownership. Petitioners argue that if petitioner Nelson
Nufable had not bought said land from the DBP, private respondents, in order to
acquire said property, must sue said bank for the recovery thereof, and in so doing,
must allege grounds for the annulment of documents evidencing the bank's ownership
thereof. Petitioners contend that since petitioner Nelson Nufable simply bought the
whole land from the bank, they cannot be deprived of the ownership of 3/4 without
making any pronouncement as to the legality or illegality of the bank's ownership of
said land. It is argued that there was no evidence to warrant declaration of nullity of the
bank's acquisition of said land; and that neither was there a finding by the court that the
bank illegally acquired the said property.
As adverted to above, when the subject property was mortgaged by Angel Custodio, he
had no right to mortgage the entire property but only with respect to his 1/4 pro
indiviso share as the property was subject to the successional rights of the other heirs
of the late Esdras. Moreover, in case of foreclosure; a sale would result in the
transmission of title to the buyer which is feasible only if the seller can be in a position
to convey ownership of the things sold. 19 And in one case, 20 it was held that a
foreclosure would be ineffective unless the mortgagor has title to the property to be
foreclosed. Therefore, as regards the remaining 3/4 pro indiviso share, the same was
held in trust for the party rightfully entitled thereto, 21 who are the private respondents
herein.
Pursuant to Article 1451 of the Civil Code, when land passes by succession to any
person and he causes the legal title to be put in the name of another, a trust is

established by implication of law for the benefit of the true owner. Likewise, under
Article 1456 of the same Code, if property is acquired through mistake or fraud, the
person obtaining it is, by force of law, considered a trustee of an implied trust for the
benefit of the person from whom the property comes. In the case of Noel vs. Court of
Appeals, 22 this Court held that "a buyer of a parcel of land at a public auction to satisfy
a judgment against a widow acquired only one-half interest on the land corresponding
to the share of the widow and the other half belonging to the heirs of her husband
became impressed with a constructive trust in behalf of said heirs."
Neither does the fact that DBP succeeded in consolidating ownership over the subject
property in its name terminate the existing co-ownership. Registration of property is not
a means of acquiring ownership. 23 When the subject property was sold to and
consolidated in the name of DBP, it being the winning bidder in the public auction, DBP
merely held the 3/4 portion in trust for the private respondents. When petitioner Nelson
purchased the said property, he merely stepped into the shoes of DBP and acquired
whatever rights and obligations appertain thereto.
This brings us to the issue of whether or not the DBP should have been impleaded as
party-defendant in the case at bar. Petitioners contend that DBP was never impleaded
and that due process requires that DBP be impleaded so that it can defend its sale to
petitioner Nelson Nufable; and that it was the duty of private respondents, and not of
petitioner Nelson, to implead the bank and ask for the annulment of documents
evidencing the bank's ownership of the disputed land.
In the Rejoinder to the Reply, private respondents that the non-inclusion of DBP as a
"necessary party" was not questioned by petitioners from the time the Complaint was
filed until the case was "finished." It was only after the adverse decision by the
respondent Court of Appeals that petitioners raised the issue.
At the outset, it should be stated petitioners never raised this issue in their Answers and
pursuant to Section 2, Rule 9 of the Rules of Court, defenses and objections not
pleaded either in a motion to dismiss or in the answer are deemed waived.
Nonetheless, the rule is that indispensable parties, i.e., parties in interest without whom
no final determination can be had of an action, shall be joined either as plaintiffs or
defendants; the inclusion as a party, i.e., persons who are not indispensable but ought
to be parties if complete relief is to be accorded as between those already parties, the
court may, in its discretion, proceed in the action without making such persons parties,
and the judgment rendered therein shall be without prejudice to the rights of such

persons. 25 Proper parties, therefore, have been described as parties whose presence
in necessary in order to adjudicate the whole controversy, but whose interests are so
far separable that a final decree can be made in their absence without affecting
them. 26 Any claim against a party may be severed and proceeded with separately. 27
The pivotal issue to be determined is whether DBP is an indispensable party in this
case.
Private respondents do not question the legality of the foreclosure of the mortgaged
property and the subsequent sale of the same to DBP. The subject property was
already purchased by petitioner Nelson from DBP and latter, by such sale, transferred
its rights and obligations to the former. Clearly, petitioners' interest in the controversy is
distinct and separable from the interest of DBP and a final determination can be had of
the action despite the non-inclusion of DBP as party-defendant. Hence, DBP, not being
an indispensable party, did not have to be impleaded in this case.
WHEREFORE, there being no reversible error in the decision appealed from, the
petition for review on certiorari is hereby DENIED.1wphi1.nt
SO ORDERED.

G.R. No. L-29300 June 21, 1978


PEDRO D. H. GALLANOSA, CORAZON GRECIA-GALLONOSA and ADOLFO
FORTAJADA, the deceased Pedro Gallanosa being substituted by his legal heirs,
namely his above-named widow and his children, ISIDRO GALLANOSA and LEDY
GALLANOSA, and grandchildren named IMELDA TECLA GALLANOSA and
ROSARIO BRIGIDA GALLANOSA, children of the late SIKATUNA GALLANOSA,
son of Pedro D.H. GALLONOSA, petitioners, vs. HON. UBALDO Y. ARCANGEL,
Judge of Branch I of the Court of First Instance of Sorsogon and FLORENTINO G.
HITOSIS, CASIANO G. HITOSIS, TEOTIMO G. HITOSIS, VICTORIO G. HITOSIS,
EMILIA G. HITOSIS VDA. DE CRUZ, JOAQUIN R. HITOSIS VDA. DE CRUZ,
JOAQUIN R. HITOSIS, FLORENTINO R. HITOSIS, VIRGINIA R. MITOSIS,
DEBORAH R. HITOSIS, EDILBERTO R. HITOSIS, LEONOR R. HITOSIS, NORMA R.
HITOSIS-VILLANUEVA, LEONCIO R. HITOSIS, minors ANGEL R. HITOSIS and
RODOLFO R. HITOSIS, represented by their legal guardian and mother LOURDES
RELUCIO VDA. DE HITOSIS, PETRONA HITOSIS-BALBIDO, MODESTO HITOSISGACILO, CLETO HITOSIS, AGUSTIN HITOSIS-FORTES, TOMASA HITOSISBANARES VDA. DE BORRAS, CONRADA HITOSIS-BANARES FRANCHE,
RESTITUTO HITOSIS-BANARES, DAMIAN HITOSIS-BANARES, FIDEL HITOSISBANARES, SUSANA HITOSIS-BANARES RODRIGUEZ, JOSE HITOSIS, LOLITA
HITOSIS-BANEGA, minors MILAGROS HITOSIS-BANEGA, ALICIA HITOSISBANEGA AND ELISA HITOSIS-BANEGA, represented by their legal guardian and
father ERNESTO BANEGA, FELICITAS HITOSIS-PENAFLOR, GENOVEVA
HITOSIS-ADRIATICO, MANUEL HITOSIS, PEDRO HITOSIS, LIBRATA HITOSISBALMES, JUANITA HITOSIS-GABITO VDA. DE GABAS, MAURA HITOSIS-GABITO
VDA. DE GANOLA and LEONA HITOSIS-GABITO GAMBA, respondents.
Haile Frivaldo for petitioners.
Joaquin R Mitosis for private respondents.

AQUINO, J.:
In this special civil action of certiorari, filed on July 29, 1968, the petitioners seek to
annul the orders of respondent Judge dated May 3 trial June 17, 1968, wherein he

reconsidered his order of January 10, 1968, dismissing, on the ground of prescription,
the complaint in Civil Case No. 2233 of the Court of First Instance of Sorsogon.
The case involves the sixty-one parcels of land in Sorsogon left by Florentino Hitosis,
with an estimated value of P50,000, trial claims for damages exceeding one million
pesos. The undisputed facts are as follows:
1. Florentino Hitosis executed a will in the Bicol dialect on June 19, 1938 when he was
eighty years old. He died on May 26, 1939 at Irosin, Sorsogon. A childless widower, he
as survived by his brother, Leon Hitosis. His other brothers, named Juan, Tito
(Juancito), Leoncio (Aloncio) trial Apolonio and only sister, Teodora, were all dead.
2. On June 24, 1939 a petition for the probate of his will was filed in the Court of First
Instance of Sorsogon (Special Proceeding No. 3171). The notice of hearing was duly
published. In that will, Florentino bequeathed his one-half share in the conjugal estate
to his second wife, Tecla Dollentas, and, should Tecla predecease him, as was the
case, his one-half share would be assigned to the spouses Pedro Gallanosa and
Corazon Grecia, the reason being that Pedro, Tecla's son by her first marriage, grew up
under the care of Florentino; he had treated Pedro as his foster child, and Pedro has
rendered services to Florentino and Tecla. Florentino likewise bequeathed his separate
properties consisting of three parcels of abaca land and parcel of riceland to his
protege (sasacuyang ataman), Adolfo Fortajada, a minor.
3. Opposition to the probate of the will was registered by the testator's legal heirs,
namely, his surviving brother, Leon, trial his nephews trial nieces. After a hearing,
wherein the oppositors did not present any evidence in support of their opposition,
Judge Pablo S. Rivera, in his decision of October 27, 1939, admitted the will to probate
and appointed Gallanosa as executor. Judge Rivera specifically found that the testator
executed his last will "gozando de buena salud y facultades mentales y no obrando en
virtud de amenaza, fraude o influencia indebida."
4. On October 24, 1941, the testamentary heirs, the Gallanosa spouses trial Adolfo
Fortajada, submitted a project of partition covering sixty-one parcels of land located in
various parts of Sorsogon, large cattle trial several pieces of personal property which
were distributed in accordance with Florentino's will. The heirs assumed the obligations
of the estate amounting to P7,129.27 in the portion of P2,376.42 for Adolfo Fortajada
and P4,752.85 for the Gallanosa spouses. The project of partition was approved by
Judge Doroteo Amador in his order of March 13, 1943, thus confirming the heirs'

possession of their respective shares. The testator's legal heirs did not appeal from the
decree of probate trial from the order of partition trial distribution.
5. On February 20, 1952, Leon Hitosis trial the heirs of Florentino's deceased brothers
trial sisters instituted an action in the Court of First Instance of Sorsogon against Pedro
Gallanosa for the recovery of the said sixty-one parcels of land. They alleged that they,
by themselves or through their predecessors-in-interest, had been in continuous
possession of those lands en concepto de dueo trial that Gallanosa entered those
lands in 1951 trial asserted ownership over the lands. They prayed that they be
declared the owners of the lands trial that they be restored to the possession thereof.
They also claimed damages (Civil Case No. 696).
6. Gallanosa moved to dismiss the above complaint for lack of cause of action trial on
the ground of bar by the prior judgment in the probate proceeding. Judge Anatolio C.
Maalac dismiss the complaint on the ground of res judicata in his order of August 14,
1952 wherein he said:
It also appears that the plaintiffs and/or their predecessors-in-interest
had intervened in the testate proceedings in Civil Case No. 3171 of this
Court for- the purpose of contesting the probate of the will of (the) late
Florentino Hitosis; trial had their opposition prospered trial the will denied
of probate, the proceedings would have been converted into one of
intestacy (Art. 960 Civil Code) and the settlement of the estate of the
said deceased would have been made in accordance with the provisions
of law governing legal or intestate succession ... , in which case the said
plaintiffs, as the nearest of kin or legal heirs of said Florentino Mitosis,
would have succeeded to the ownership and possession of the 61
parcels of land in question forming part of his estate (art. 1003, Civil
Code).
However, the derision of the Court was adverse to them, when it their
opposition trial ordered the probate of his will. From this decision (Annex
K) legalizing the said will, the oppositors did not file any appeal within
the period fixed by law, despite the fact that they were duly notified
thereof, so that the said decision had become final trial it now constitutes
a bar to any action that the plaintiffs may institute for the purpose of a
redetermination of their rights to inherit the properties of the late
Florentino Hitosis.

In other words, the said decision of this Court in Civil Case special ) No.
3171, in which the herein plaintiffs or their predecessors-in-interest had
intervened as parties oppositors, constitutes a final judicial determination
of the issue that the said plaintiffs, as ordinary heirs, have no legal rights
to succeed to any of the properties of the late Florentino Hitosis;
consequently, their present claim to the ownership trial possession of the
61 parcels of land in question is without any legal merit or basis.
7. The plaintiffs did not appeal from that order of dismissal which should have set the
matter at rest. But the same plaintiffs or oppositors to the probate of the will, trial their
heirs, with a persistence befitting a more meritorious case, filed on September 21,
1967, or fifteen years after the dismissal of Civil Case No. 696 trial twenty-eight years
after the probate of the will another action in the same court against the Gallanosa
spouses trial Adolfo Fortajada for the "annulment" of the will of Florentino Hitosis trial
and for the recovery of the same sixty-one parcels of land. They prayed for the
appointment of a receiver.
8. As basis of their complaint, they alleged that the Gallanosa spouses, through fraud
trial deceit, caused the execution trial simulation of the document purporting to be the
last will trial testament of Florentino Hitosis. While in their 1952 complaint the game
plaintiffs alleged that they were in possession of the lands in question, in their 1967
complaint they admitted that since 1939, or from the death of Florentino Hitosis, the
defendants (now the petitioners) have been in possession of the disputed lands (Par.
XIV of the complaint, p. 70, Rollo in Civil Case No. 555, Gubat Branch, which was
transferred to Branch I in Sorsogon town where Special Proceeding No. 3171 trial Civil
Case No. 696 were decided trial which was re-docketed as Civil Case No. 2233).
9. As already stated, that 1967 complaint, upon motion of the defendants, now the
petitioners, was dismissed by respondent Judge. The plaintiffs filed a motion for
reconsideration Respondent Judge. granted it trial set aside the order of dismissal. He
denied defendants' motion for the reconsideration of his order setting aside that
dismissal order.
The petitioners or the defendants below contend in this certiorari case that the lower
court has no jurisdiction to set aside the 1939 decree of probate trial the 1952 order of
dismissal in Civil Case No. 696 trial that it acted with grave abuse of discretion in not
dismissing private respondents' 1967 complaint.

The issue is whether, under the facts set forth above, the private respondents have a
cause of action the "annulment" of the will of Florentino Hitosis trial for the recovery of
the sixty-one parcels of land adjudicated under that will to the petitioners.

SEC. 49. Effect of judgments. The effect of a judgment or final order


rendered by a court or judge of the Philippines, having jurisdiction to
pronounce the judgment or order, may be as follows:

We hold that the lower court committed a grave abuse of discretion in reconsideration
its order of dismissal trial in ignoring the 1939 testamentary case trial the 1952 Civil
Case No. 696 which is the same as the instant 1967 case.

(a) In case of a judgment or order against a specific thing, or in respect


to the probate of a will or the administration of the estate of a deceased
person, or in respect to the personal, political, or legal condition or status
of a particular person or his relationship to another, the judgment or
order is conclusive upon the title to the thing the will or administration, or
the condition, status or relationship of the person; however, the probate
of a will or granting of letters of administration shall only be prima facie
evidence of the death of the testator or intestate;

A rudimentary knowledge of substantive law trial procedure is sufficient for an ordinary


lawyer to conclude upon a causal perusal of the 1967 complaint that it is baseless trial
unwarranted.
What the plaintiffs seek is the "annulment" of a last will trial testament duly probated in
1939 by the lower court itself. The proceeding is coupled with an action to recover the
lands adjudicated to the defendants by the same court in 1943 by virtue of the probated
will, which action is a resuscitation of The complaint of the same parties that the same
court dismissed in 1952.
It is evident from the allegations of the complaint trial from defendants' motion to
dismiss that plaintiffs' 1967 action is barred by res judicata, a double-barrelled defense,
trial by prescription, acquisitive trial extinctive, or by what are known in the jus civile trial
the jus gentium as usucapio, longi temporis possesio and praescriptio (See Ramos vs.
Ramos, L-19872, December 3, 1974, 61 SCRA 284).
Our procedural law does not sanction an action for the "annulment" of a will. In order
that a will may take effect, it has to be probated, legalized or allowed in the proper
testamentary proceeding. The probate of the will is mandatory (Art. 838, Civil Code;
sec. 1, Rule 75, formerly sec. 1, Rule 76, Rules of Court; Guevara vs. Guevara, 74 Phil.
479; Guevara vs. Guevara, 98 Phil. 249).
The testamentary proceeding is a special proceeding for the settlement of the testator's
estate. A special proceeding is distinct trial different from an ordinary action (Secs. 1
trial 2, Rule 2 trial sec. 1, Rule 72, Rules of Court).
We say that the defense of res judicata, as a ground for the dismissal of plaintiffs' 1967
complaint, is a two-pronged defense because (1) the 1939 trial 1943 decrees of
probate trial distribution in Special Proceeding No. 3171 trial (2) the 1952 order of
dismissal in Civil Case No. 696 of the lower court constitute bars by former judgment,
Rule 39 of the Rules of Court provides:

(b) In other cases the judgment or order is, with respect to the matter
directly adjudged or as to any other matter that could have been raised
in relation thereto, conclusive between the parties trial their successors
in interest by title subsequent to the commencement of the action or
special proceeding, litigating of the same thing trial under the same title
trial in the same capacity;
(c) In any other litigation between the same parties or their successors in
interest, that only is deemed to have been adjudged in a former
judgment which appears upon its face to have been so adjudged, or
which was actually trial necessarily included therein or necessary
thereto.
The 1939 decree of probate is conclusive as to the due execution or formal validity of
the will (Sec. 625, Act 190, sec. 1, Rule 76, now sec. 1, Rule 75, Rules of Court; Last
par. of art. 838, Civil Code).
That means that the testator was of sound trial disposing mind at the time when he
executed the will and was not acting under duress, menace, fraud, or undue influence;
that the will was signed by him in the presence of the required number of witnesses,
and that the will is genuine trial is not a forgery. Accordingly, these facts cannot again
be questioned in a subsequent proceeding, not even in a criminal action for the forgery
of the will. (3 Moran's Comments on the Rules of Court, 1970 Edition, p. 395; Manahan
vs. Manahan, 58 Phil. 448).

After the finality of the allowance of a will, the issue as to the voluntariness of its
execution cannot be raised anymore (Santos vs. De Buenaventura, L-22797,
September 22, 1966, 18 SCRA 47).
In Austria vs. Ventenilla, 21 Phil. 180, a "petition for annulment of a will" was not
entertained after the decree of probate had become final. That case is summarized as
follows:
Wills; Probate; Alledged Fraudulent Will; Appeal. V. died. His will was
admitted to probate without objection. No appeal was taken from said
order. It was admitted that due trial legal notice had been given to all
parties. Fifteen months after the date of said order, a motion was
presented in the lower court to have said will declared null and void, for
the reason that fraud had been practised upon the deceased in the
making of his will.
Held: That under section 625 of Act No. 190, the only time given parties
who are displeased with the order admitting to probate a will, for an
appeal is the time given for appeals in ordinary actions; but without
deciding whether or not an order admitting a will to probate will be
opened for fraud, after the time allowed for an appeal has expired, when
no appeal is taken from an order probating a will, the heirs can not, in
subsequent litigation in the same proceedings, raise questions relating
to its due execution. The probate of a will is conclusive as to its due
execution trial as to the testamentary capacity of The testator. (See
Austria vs. Heirs of Ventenilla. 99 Phil. 1069).
On the other hand, the 1943 decree of adjudication rendered by the trial court in the
testate proceeding for the settlement of the estate of Florentino Hitosis, having been
rendered in a proceeding in rem, is under the abovequoted section 49(a), binding upon
the whole world (Manalo vs. Paredes, 47 Phil. 938; In re Estate of Johnson, 39 Phil.
156; De la Cerna vs. Potot, 120 Phil. 1361, 1364; McMaster vs. Hentry Reissmann &
Co., 68 Phil. 142).
It is not only the 1939 probate proceeding that can be interposed as res judicata with
respect to private respondents' complaint, The 1952 order of dismissal rendered by
Judge Maalac in Civil Case No. 696, a judgment in personam was an adjudication on
the merits (Sec. 4, Rule 30, old Rules of Court). It constitutes a bar by former judgment
under the aforequoted section 49(b) (Anticamara vs. Ong, L-29689. April 14, 1978).

The plaintiffs or private respondents did not even bother to ask for the annulment of the
testamentary proceeding trial the proceeding in Civil Case No. 696. Obviously, they
realized that the final adjudications in those cases have the binding force of res
judicata and that there is no ground, nor is it timely, to ask for the nullification of the final
orders trial judgments in those two cases.
It is a fundamental concept in the organization of every jural system, a principle of
public policy, that, at the risk of occasional errors, judgments of courts should become
final at some definite date fixed by law. Interest rei publicae ut finis sit litum. "The very
object for which the courts were constituted was to put an end to controversies." (Dy
Cay vs. Crossfield and O'Brien, 38 Phil. 521: Pealosa vs. Tuason, 22 Phil, 303; De la
Cerna vs. Potot, supra).
After the period for seeking relief from a final order or judgment under Rule 38 of the
Rules of Court has expired, a final judgment or order can be set aside only on the
grounds of (a) lack of jurisdiction or lack of due process of law or (b) that the judgment
was obtained by means of extrinsic or collateral fraud. In the latter case, the period for
annulling the judgment is four years from the discovery of the fraud (2 Moran's
Comments on the Rules of Court, 1970 Edition, pp. 245-246; Mauricio vs. Villanueva,
106 Phil. 1159).
To hurdle over the obstacle of prescription, the trial court, naively adopting the theory of
plaintiffs' counsel, held that the action for the recovery of the lands had not prescribed
because the rule in article 1410 of the Civil Code, that "the action or defense for the
declaration of the inexistence of a contract does not prescribe", applies to wills.
That ruling is a glaring error. Article 1410 cannot possibly apply to last wills trial
testaments. The trial court trial plaintiffs' counsel relied upon the case of Dingle vs.
Guillermo, 48 0. G. 4410, allegedly decided by this Court, which cited the ruling in
Tipton vs. Velasco, 6 Phil. 67, that mere lapse of time cannot give efficacy to
voidcontracts, a ruling elevated to the category of a codal provision in article 1410.
The Dingle case was decided by the Court of Appeals. Even the trial court did not take
pains to verify the misrepresentation of plaintiffs' counsel that the Dingle case was
decided by this Court. An elementary knowledge of civil law could have alerted the trial
court to the egregious error of plaintiffs' counsel in arguing that article 1410 applies to
wills.

WHEREFORE, the lower court's orders of May 3 trial June 17, 1968 are reversed trial
set aside trial its order of dismissal dated January 10, 1968 is affirmed. Costs against
the private respondents.
SO ORDERED.

G.R. No. L-12207

December 24, 1959

JUAN PALACIOS, petitioner-appellant, vs. MARIA CATIMBANG


PALACIOS, oppositor-appellee.
Augusto Francisco and Vicente Reyes Villavicencio for appellant.
Laureano C. Alano and Enrique A. Amador for appellee.

BAUTISTA ANGELO, J.:


Juan Palacios executed his last will and testament on June 25, 1946 and availing
himself of the provisions of the new Civil Code, he filed on May 23, 1956 before the
Court of First Instance of Batangas a petition for its approval. In said will, he instituted
as his sole heirs his natural children Antonio C. Palacios and Andrea C. Palacios.
On June 21, 1956, Maria Catimbang filed a opposition to the probate of the will alleging
that she is the acknowledged natural daughter of petitioner but that she was completely
ignored in said will thus impairing here legitime.
After the presentation of petitioner's evidence relative to the essential requisites and
formalities provided by law for the validity of a will, the court on July 6, 1956 issued an
order admitting the will to probate. The court, however, set a date for the hearing of the
opposition relative to the intrinsic validity of the will and, after proper hearing concerning
this incident, the court issued another order declaring oppositor to be the natural child
of petitioner and annulling the will insofar as it impairs her legitime, with costs against
petitioner.
From this last order, petitioner gave notice of his intention to appeal directly to the
Supreme Court, and accordingly, the record was elavated to this Court.
It should be noted that petition instituted the present proceeding in order to secure the
probate of his will availing himself of the provisions of Article 838, paragraph 2, of the
new Civil Code, which permit a testator to petition the proper court during his lifetime for
the allowance of his will, but to such petition on Maria Catimbang filed an opposition
alleging that she is the acknowledged natural daughter of petitioner but that she was
completely ignored in the will thus impairing her object to the probate of the will insofar

as it due execution is concerned or on the ground that it has not complied with the
formalities prescribed by law; rather she objects to its intrinsic validity or to the legality
of the provisions of the will.
We hold that such opposition cannot be entertained in this proceeding because its only
purpose is merely to determine if the will has been executed in accordance with the
requirements of the law, much less if the purpose of the opposition is to show that the
oppositor is an acknowledged natural child who allegedly has been ignored in the will
for issue cannot be raised here but in a separate action. This is especially so when the
testator, as in the present case, is still alive and has merely filed a petition for the
allowance of his will leaving the effects thereof after his death.lawphi1.net
This is in line with our ruling in Montaano vs. Suesa, 14 Phil., 676, wherein we said:
"The authentication of the will decides no other questions than such as touch upon the
capacity of the testator and the compliance with those requisites or solemnities which
the law prescribes for the validity of a will. It does not determine nor even by implication
prejudge the validity or efficiency of the provisions; that may be impugned as being
vicious or null, notwithstanding its authentication. The questions relating to these points
remain entirely un-affected, and may be raised even after the will has been
authenticated."
On the other hand, "after a will has been probated during the lifetime of a testator, it
does not necessarily mean that he cannot alter or revoke the same before he has had a
chance to present such petition, the ordinary probate proceedings after the testator's
death would be in order" (Report of the Code Commission, pp. 53-54).The reason for
this comment is that the rights to the succession are transmitted from the moment of
the death of the decedent (Article 777, new Civil Code.).
It is clear that the trial court erred in entertaining the opposition and in annulling the
portion of the will which allegedly impairs the legitime of the oppositor on the ground
that, as it has found, she is an extraneous matter which should be treshed out in a
separate action.
Wherefore, the order appealed from is set aside, without pronouncement as to costs.

G.R. No. 108947 September 29, 1997

The Antecedent Facts

ROLANDO SANCHEZ, FLORIDA MIERLY SANCHEZ, ALFREDO T. SANCHEZ and


MYRNA T. SANCHEZ,petitioners, vs. THE HONORABLE COURT OF APPEALS,
ROSALIA S. LUGOD, ARTURO S. LUGOD, EVELYN LUGOD-RANISES and
ROBERTO S. LUGOD, respondents.

PANGANIBAN, J.:

The facts are narrated by the Court of Appeals as follows:


[Herein private respondent] Rosalia S. Lugod is the only child of spouses
Juan C. Sanchez and Maria Villafranca while [herein private
respondents] Arturo S. Lugod, Evelyn L. Ranises and Roberto S. Lugod
are the legitimate children of [herein private respondent] Rosalia.
[Herein petitioners] Rolando, Florida Mierly, Alfredo and Myrna, all
surnamed Sanchez, are the illegitimate children of Juan C. Sanchez.

Is a petition for certiorari, in lieu of appeal, the proper remedy to correct orders of a
probate court nullifying certain deeds of sale and, thus, effectively passing upon title to
the properties subject of such deeds? Is a compromise agreement partitioning inherited
properties valid even without the approval of the trial court hearing the intestate estate
of the deceased owner?

Following the death of her mother, Maria Villafranca, on September 29,


1967, [herein private respondent] Rosalia filed on January 22, 1968, thru
counsel, a petition for letters of administration over the estate of her
mother and the estate of her father, Juan C. Sanchez, who was at the
time in state of senility (Annex "B", Petition).

The Case
These questions are answered by this Court as it resolves the petition for review
on certiorari before us assailing the November 23, 1992 Decision 1 of the Court of
Appeals 2 in CA-G.R. SP No. 28761 which annulled the decision 3 of the trial court 4 and
which declared the compromise agreement among the parties valid and binding even
without the said trial court's approval. The dispositive portion of the assailed Decision
reads:
WHEREFORE, for the reasons hereinabove set forth and discussed, the
instant petition is GRANTED and the challenged decision as well as the
subsequent orders of the respondent court are ANNULLED and SET
ASIDE. The temporary restraining order issued by this Court on October
14, 1992 is made PERMANENT. The compromise agreement dated
October 30, 1969 as modified by the memorandum of agreement of April
13, 1970 is DECLARED valid and binding upon herein parties. And
Special Proceedings No. 44-M and 1022 are deemed CLOSED and
TERMINATED.
SO ORDERED. 5

On September 30, 1968, [herein private respondent] Rosalia, as


administratrix of the intestate estate of her mother, submitted an
inventory and appraisal of the real and personal estate of her late
mother (Annex "C", Petition).
Before the administration proceedings Special in Proceedings No. 44-M
could formally be terminated and closed, Juan C. Sanchez, [herein
private respondent] Rosalia's father, died on October 21, 1968.
On January 14, 1969, [herein petitioners] as heirs of Juan C. Sanchez,
filed a petition for letters of administration (Special Proceedings No.
1022) over the intestate estate of Juan C. Sanchez, which petition was
opposed by (herein private respondent) Rosalia. 6
On October 30, 1969, however, [herein private respondent] Rosalia and
[herein petitioners] assisted by their respective counsels executed a
compromise agreement (Annex "D", Petition) wherein they agreed to
divide the properties enumerated therein of the late Juan C. Sanchez.

On November 3, 1969, petitioner Rosalia was appointed by [the trial


court], and took her oath as the administratrix of her father's intestate
estate.
On January 19, 1970, [herein petitioners] filed a motion to require
administratrix, [herein private respondent] Rosalia, to deliver deficiency
of 24 hectares and or to set aside compromise agreement (Annex "E",
Petition).
Under date of April 13, 1970, (herein private respondent) Rosalia and
[herein petitioners] entered into and executed a memorandum of
agreement which modified the compromise agreement (Annex "F".
Petition)
On October 25, 1979, or nine years later, [herein petitioners] filed, thru
counsel, a motion to require [herein private respondent] Rosalia to
submit a new inventory and to render an accounting over properties not
included in the compromise agreement (Annex "G", Petition). They
likewise filed a motion to defer the approval of the compromise
agreement (Annex "H", Ibid), in which they prayed for the annulment of
the compromise agreement on the ground of fraud.
On February 4, 1980, however, counsel for [herein petitioners] moved to
withdraw his appearance and the two motions he flied, Annex "G" and
"H" (Annex "I", Petition).
On February 28, 1980, the [trial] court issued an order directing [herein
private respondent] Rosalia to submit a new inventory of properties
under her administration and an accounting of the fruits thereof, which
prompted [herein private respondent] Rosalia to file a rejoinder on March
31, 1980 (Annex "K", Petition).
On May 12, 1980, [herein petitioners], thru new counsel, filed a motion to
change administratrix (Annex "L", Petition) to which [herein private
respondent] Rosalia filed an opposition (Annex "M",Ibid).
The parties were subsequently ordered to submit their respective
position papers, which they did (Annexes "N" and "O", Petition). On

September 14, 1989, former counsel of (herein petitioners) entered his


re-appearance as counsel for (herein petitioners).
On the bases of memoranda submitted by the parties, the [trial court],
this time presided by Judge Vivencio A. Galon, promulgated its decision
on June 26, 1991, the dispositive portion of which states:
WHEREFORE, premises considered, judgment is hereby
rendered as follows by declaring and ordering:
1. That the entire intestate estate of Maria Villafranca
Sanchez under Special Proceedings No. 44-M consists of
all her paraphernal properties and one-half (1/2) of the
conjugal properties which must be divided equally
between Rosalia Sanchez de Lugod and Juan C.
Sanchez;
2. That the entire intestate estate of Juan C. Sanchez
under Special Proceedings No. 1022 consists of all his
capital properties, one-half (1/2) from the conjugal
partnership of gains and one-half (1/2) of the intestate
estate of Maria Villafranca under Special Proceedings No.
44-M;
3. That one-half (1/2) of the entire intestate estate of Juan
C. Sanchez shall be inherited by his only legitimate
daughter, Rosalia V. Sanchez de Lugod while the other
one-half (1/2) shall be inherited and be divided equally by,
between and among the six (6) illegitimate children,
namely: Patricia Alburo, Maria Ramuso Sanchez,
Rolando Pedro T. Sanchez, Florida Mierly T. Sanchez,
Alfredo T. Sanchez and Myrna T. Sanchez;
4. That all the Deed (sic) of Absolute Sales executed by
Juan C. Sanchez and Maria Villafranca in favor of Rosalia
Sanchez Lugod, Arturo S. Lugod, Evelyn S. Lugod and
Roberto S. Lugod on July 26, 1963 and June 26, 1967
are all declared simulated and fictitious and must be
subject to collation and partition among all heirs;

5. That within thirty (30) days from finality of this decision,


Rosalia Sanchez Lugod is hereby ordered to prepare a
project of partition of the intestate estate of Juan C.
Sanchez under Special Proceedings No. 1022 and
distribute and deliver to all heirs their corresponding
shares. If she fails to do so within the said thirty (30) days,
then a Board of Commissioners is hereby constituted,
who are all entitled to honorarium and per diems and
other necessary expenses chargeable to the estate to be
paid by Administratrix Rosalia S. Lugod, appointing the
Community Environment and Natural Resources Officer
(CENRO) of Gingoog City as members thereof, with the
task to prepare the project of partition and deliver to all
heirs their respective shares within ninety (90) days from
the finality of said decision;
6. That within thirty (30) days from receipt of this decision,
Administratrix Rosalia Sanchez Vda. de Lugod is hereby
ordered to submit two (2) separate certified true and
correct accounting, one for the income of all the
properties of the entire intestate estate of Maria
Villafranca under Special Proceedings No. 44-M, and
another for the properties of the entire intestate estate of
Juan C. Sanchez under Special Proceedings No. 1022
duly both signed by her and both verified by a Certified
Public Accountant and distribute and deliver to her six (6)
illegitimate brothers and sisters in equal shares, one-half
(1/2) of the net income of the estate of Juan C. Sanchez
from October 21, 1968 up to the finality of this decision;
7. For failure to render an accounting report and failure to
give cash advances to the illegitimate children of Juan C.
Sanchez during their minority and hour of need from the
net income of the estate of Juan C. Sanchez, which
adversely prejudiced their social standing and pursuit of
college education, (the trial court) hereby orders Rosalia
Sanchez Vda. de Lugod to pay her six (6) illegitimate
brothers and sisters the sum of Five Hundred Thousand
(P500,000.00) Pesos, as exemplary damages, and also

the sum of One Hundred Fifty Thousand (P150,000.00)


Pesos for attorney's fees;
8. Upon release of this decision and during its pendency,
should appeal be made, the Register of Deeds and
Assessors of the Provinces and Cities where the
properties of Juan C. Sanchez and Maria Villafranca are
located, are all ordered to register and annotate in the title
and/or tax declarations, the dispositive portion of this
decision for the protection of all heirs and all those who
may be concerned.
SO ORDERED.
[Herein private respondent] Rosalia filed a motion for reconsideration
dated July 17, 1991 (Annex "P", Petition) on August 6, 1991.
On August 13, 1991, [herein petitioners] filed a motion for execution and
opposition to [herein private respondent] Rosalia's motion for
reconsideration (Annex "Q", Petition).
On September 3, 1991, [the trial court] issued an Omnibus Order (Annex
"S", Petition) declaring, among other things, that the decision at issue
had become final and executory.
[Herein private respondent] Rosalia then filed a motion for
reconsideration of said Omnibus Order (Annex "T", Petition). Said
[herein private respondent] was allowed to file a memorandum in
support of her motion (Annex "V", Petition).
On June 26, 1991, [the trial court] issued and Order denying petitioner
Rosalia's motion for reconsideration (Annex "W", Petition). 7
Thereafter, private respondents elevated the case to the Court of Appeals via a petition
for certiorari and contended:
I
The [trial court] has no authority to disturb the compromise agreement.

II
The [trial court] has arbitrarily faulted [herein private respondent] Rosalia
S. Lugod for alleged failure to render an accounting which was
impossible.
III
The [trial court] acted without jurisdiction in derogation of the
constitutional rights of [herein private respondents] Arturo S. Lugod,
Evelyn L. Ranises and Roberto S. Lugod when [the trial court] decided
to annul the deed of sale between the said [herein private respondents]
and Juan C. Sanchez without affording them their day in court.

Sanchez Lugod, Oppositor herein, was born, thus making her the sole
and only surviving legitimate heir of her deceased parents;
2. That the said deceased Juan C. Sanchez, left illegitimate children,
Intervenors-Oppositors and Petitioners, respectively, herein namely;
(1) Patricio Alburo, born out of wedlock on
March 17, 1926 at Cebu City, Philippines,
to Emilia Alburo;
(2) Maria Ramoso Sanchez, born out of
wedlock on May 9, 1937 at Gingoog,
Misamis Oriental, now, Gingoog City, to
Alberta Ramoso;

IV
[The trial court judge] defied without rhyme or reason well-established
and entrenched jurisprudence when he determined facts sans any
evidence thereon.

(3) (a) Rolando Pedro Sanchez, born on


May 19, 1947,
(b) Florida Mierly Sanchez, born on
February 16, 1949,

V
[The trial court] grossly misinterpreted [herein private respondent]
Rosalia S. Lugod's right to appeal.8
For clarity's sake, this Court hereby reproduces verbatim the compromise
agreement 9 of the parties:
COMPROMISE AGREEMENT
COME NOW, the parties in the above-entitled case, motivated by their
mutual desire to preserve and maintain harmonious relations between
and among themselves, for mutual valuable considerations and in the
spirit of good will and fair play, and, for the purpose of this Compromise
Agreement, agree to the following:
1. That the deceased Juan C. Sanchez who died intestate on October
21, 1968 was legally married to Maria Villafranca de Sanchez, who
predeceased her on September 29, 1967, out of whose wedlock Rosalia

(c) Alfredo Sanchez, born on July 21,


1950, and
(d) Myrna Sanchez, born on June 16,
1952, all born out of wedlock to Laureta
Tampus in Gingoog City, Philippines.
3. That the deceased Juan C. Sanchez left the following properties, to
wit:
I. SEPARATE CAPITAL OF JUAN C. SANCHEZ
NATURE, DESCRIPTION AND AREA ASSESSED VALUE
(1) Agricultural Land. Covered by Tax. Decl. No. 06458, Cad. Lot No.
1041 C-2, located at Murallon, Gingoog City and bounded on the North
by Lot Nos. 1033, 1035, 1036, 1037, 1039, 1040, 1042 & 1043; South by
Lot No. 1080, 1088, 1087 & 1084; East by Lot Nos. 1089, 1061 & 2319;

West by Lot Nos. 954, 1038, 1057 & 1056, containing an area of ONE
HUNDRED EIGHTY THREE THOUSAND SIX HUNDRED SEVENTY
TWO (183, 672) sq. ms. more or less.
P21,690.00
II. CONJUGAL PROPERTY OF JUAN C. SANCHEZ AND MARIA
VILLAFRANCA DE SANCHEZ
(1) Agricultural Land. Covered by Tax Decl. No. 06447, Cad. Lot No.
2745, C-7 located at Agay-ayan, Gingoog City and bounded on the
North by Lot Nos. 2744, 2742, 2748; South by Lot No. 2739; East by Lot
No. 2746; West by Lot No. 2741, containing an area of FOURTEEN
THOUSAND SEVEN HUNDRED (14,700) sq. ms. more or less.
P1,900.00
(2) Agricultural Land. Covered by Tax Decl. No. 06449, Cad, Lot No.
3271 C-7 located at Panyangan, Lanao, Gingoog City and bounded on
the North by Lot No. 3270; South by Lot Nos. 2900 & 3462; East by
Panyangan River & F. Lumanao; and Part of Lot 3272; and West by
Samay Creek, containing an area of ONE HUNDRED FOUR
THOUSAND SIX HUNDRED (104,600) sq. ms. more or less.
P11,580.00
(3) Agricultural Land. Covered by Tax Decl. No. 06449, Cad. Lot No.
2319, Case 2, located at Murallon, Gingoog City and bounded on the
North by Lot No. 1061; South by Hinopolan Creek; East by Lot No. 1044;
and West by Lot No. 1041, containing an area of THREE THOUSAND
TWO HUNDRED TWENTY FIVE (3,225) sq. ms. more or less.
(4) Agricultural Land. Covered by Tax Decl. No. 06452, Cad. Lot No.
3272, C-7 Part 4 located at Panyangan, Lunao, Gingoog City and
bounded on the North by Lot Nos. 3270 & 3273; East by Panyangan
River; South by Panyangan River; and West by Lot Nos. 3270 & 3271,
containing an area of FIFTY FIVE THOUSAND SIX HUNDRED (55,600)
sq. ms. more or less, being claimed by Damian Querubin.

P2,370.00
(5) Agricultural Land. Covered by Tax Decl. No. 06453, Cad. Lot No.
3270 Case 7, located at Sunog, Lunao, Gingoog City and bounded on
the North by Samay Creek & Lot 3267; South by Lot Nos. 3271 & 3272;
East by Lot Nos. 3269 & 3273; and West by Samay Creek, containing an
area of FOUR HUNDRED EIGHT THREE THOUSAND SIX HUNDRED
(483,600) sq. ms. more or less.
P61,680.00
(6) Agricultural Land. Covered by Tax Decl. No. 06457, Cad. Lot No.
3273, C-7 Part 2 located at Panyangan, Lunao, Gingoog City and
bounded on the North by Lot No. 3269; South by Lot No. 3272; East by
Panyangan River; and West by Lot No. 3270, containing an area of
THIRTY FOUR THOUSAND THREE HUNDRED (34,300) sq. ms. more
or less, being claimed by Miguel Tuto.
P3,880.00
(7) Agricultural Land. Covered by Tax Decl. No. 12000, Cad. Lot No.
2806, Case 7 located at Agayayan, Gingoog City and bounded on the
North by Agayayan River; South by Victoriano Barbac; East by Isabelo
Ramoso; and West by Restituto Baol, containing an area of SIX
THOUSAND SIX HUNDRED SEVENTY SIX (6,676) sq. ms. more or
less.
P380.00
(8) Agricultural Land. Covered by Tax Decl. No. 12924, Cad. Lot No.
1206 C-1 located at Cahulogan, Gingoog City and bounded on the NW.,
by Lot No. 1209; SW., by Lot No. 1207; Eastby National Highway; and
West by Lot No. 1207; containing an area of FOUR THOUSAND FIVE
HUNDRED THIRTEEN (4,513) sq. ms. more or less.
P740.00
(9) Agricultural Land. Covered by Tax Decl. No. 12925, Cad. Lot No.
5554, located at Tinaytayan, Pigsalohan, Gingoog City and bounded on

the North by Lot Nos. 5559 & 5558; South by Lot No. 3486; East by Lot
No. 5555; and West by Lot No. 5355, containing an area of EIGHTEEN
THOUSAND FIVE HUNDRED TWENTY EIGHT (18,528) sq. ms. more
or less.
P320.00
(10) Agricultural Land. Covered by Tax Decl. No. 12926, Cad. Lot No.
5555 C-7 located at Tinaytayan, Pigsalojan, Gingoog City and bounded
on the North by Tinaytayan Creek & Lot Nos. 5557 & 5558; South by Lot
Nos. 3486, 3487, 3488, 3491 & 3496; East by Cr. & Lot No. 3496; and
West by Lot No. 5554, containing an area of SEVENTY SEVEN
THOUSAND SEVEN HUNDRED SEVENTY SIX (77,776) sq. ms. more
or less.
P1,350.00
(11) A Commercial Land. Covered by Tax Decl. No. 06454, Cad. Lot No.
61-C-1 located at Guno-Condeza Sts., Gingoog City and bounded on
the North by Lot 64; South by Road-Lot 613 Condeza St; East by Lot
Nos. 63, and 62; West by Road-Lot 614-Guno St., containing an area of
ONE THOUSAND FORTY TWO (1,042) sq. ms. more or less.

P1,050.00
(14) Agricultural Land. Covered by Tax, Decl. No. 06789, Cad. Lot No.
5157-C-7, located at Kiogat, Agayayan, Gingoog City and bounded on
the North by Lot No. 5158, 5159, 5156; South by SE-Steep Bank; East
by NW, by Lot No. 5158, Villafranca, containing an area of NINETY SIX
THOUSAND TWO HUNDRED (96,200) sq. ms. more or less.
P3,370.00
III. PERSONAL ESTATE (CONJUGAL)
NATURE AND DESCRIPTION LOCATION APPRAISAL
1. Fifty (50) shares of stock
Rural Bank of Gingoog, Inc.
at P100.00 per share P5,000.00
2. Four (4) shares of Preferred Stock
with San Miguel Corporation 400.00
4. That, the parties hereto have agreed to divide the above-enumerated
properties in the following manner, to wit:

P9,320.00
(12) A Commercial Land. Covered by Tax Decl. No. 06484, Lot No. 5,
Block 2, located at Cabuyoan, Gingoog City and bounded on the North
by Lot No. 4, block 2; South by Lot No. 8, block 2; East by Lot No. 6,
block 2, West by Subdivision Road, containing an area of FOUR
HUNDRED (400) sq. ms. more or less.

(a) To Patricio Alburo, Maria Ramoso


Sanchez, Roland Pedro T. Sanchez,
Florida Mierly Sanchez, Alfredo T. Sanchez
and Myrna T. Sanchez, in equal proindiviso shares, considering not only their
respective areas but also the
improvements existing thereon, to wit:

P12,240.00
(13) A Commercial Land. Covered by Tax Decl. No. 15798, Block No. 7A-16-0 located at Cabuyoan, Gingoog City and bounded on the North by
Lot No. 7-A-16-0; South by Lot No. 7-16-0; East by Lot No. 7-A-18Road; West by Lot No. 8, PSU-120704-Julito Arengo vs. Restituto Baol,
containing an area of TWO HUNDRED SIXTEEN (216) sq. ms. more or
less.

Agricultural Land. Covered by Tax Decl.


No. 06453, Cad. Lot No. 3270 Case 7,
located at Sunog, Lunao, Gingoog City and
bounded on the North by Samay Creek &
Lot 3267; South by Lot Nos. 3271 and
3272; East by Lot Nos. 3269 & 3273; and
West by Samay Creek, containing an area

of FOUR HUNDRED EIGHTY THREE


THOUSAND SIX HUNDRED (483,600) sq.
ms. and assessed in the sum of
P61,680.00.
(b) To Rosalia Sanchez Lugod all the rest
of the properties, both real and personal,
enumerated above with the exception of
the following:
(1) Two Preferred Shares of
Stock in the San Miguel
Corporation, indicated in
San Miguel Corporation
Stock Certificate No. 30217,
which two shares she is
ceding in favor of Patricio
Alburo;
(2) The house and lot
designated as Lot No. 5,
Block 2 together with the
improvements thereon and
identified as parcel No. II12, lot covered by Tax Decl.
No. 15798 identified as
Parcel No. II-13 in the
above enumerated, and
Cad. Lot No. 5157-C-7
together with the
improvements thereon,
which is identified as parcel
No. II-14 of the aboveenumeration of properties,
which said Rosalia S. Lugod
is likewise ceding and
renouncing in favor of
Rolando Pedro, Florida
Mierly, Alfredo and Myrna,

all surnamed Sanchez, in


equal pro-indiviso shares;
5. That Rolando Pedro, Florida Mierly, Alfredo and Myrna, all surnamed
Sanchez hereby acknowledge to have received jointly and severally in
form of advances after October 21, 1968 the aggregate sum of EIGHT
THOUSAND FIVE HUNDRED THIRTY-THREE PESOS (P8,533.94) and
NINETY-FOUR CENTAVOS;
6. That the parties hereto likewise acknowledge and recognize in the
indebtedness of the deceased Juan G. Sanchez and his deceased wife
Maria Villafranca Sanchez to the Lugod Enterprises, Inc., in the sum of
P43,064.99;
7. That the parties hereto shall be responsible for the payment of the
estate and inheritance taxes proportionate to the value of their
respective shares as may be determined by the Bureau of Internal
Revenue and shall likewise be responsible for the expenses of survey
and segregation of their respective shares;
8. That Patricio Alburo, Maria Ramoso Sanchez, Roland Pedro Sanchez,
Florida Mierly Sanchez, Alfredo Sanchez and Myrna Sanchez hereby
waive, relinquish and renounce, jointly and individually, in a manner that
is absolute and irrevocable, all their rights and interests, share and
participation which they have or might have in all the properties, both
real and personal, known or unknown and/or which may not be listed
herein, or in excess of the areas listed or mentioned herein, and/or
which might have been, at one time or another, owned by, registered or
placed in the name of either of the spouses Juan C. Sanchez or Maria
Villafranca de Sanchez or both, and which either one or both might have
sold, ceded, transferred, or donated to any person or persons or entity
and which parties hereto do hereby confirm and ratify together with all
the improvements thereon, as well as all the produce and proceeds
thereof, and particularly of the properties, real and personal listed herein,
as well as demandable obligations due to the deceased spouses Juan
C. Sanchez, before and after the death of the aforementioned spouses
Juan C. Sanchez and Maria Villafranca de Sanchez, in favor of oppositor
Rosalia S. Lugod;

9. That the expenses of this litigation including attorney's fees shall be


borne respectively by the parties hereto;

WHEREFORE, it is most respectfully prayed that the foregoing


compromise agreement be approved.

10. That Laureta Tampus for herself and guardian ad-litem of her minor
children, namely: Florida Mierly, Alfredo, and Myrna, all surnamed
Sanchez, hereby declare that she has no right, interest, share and
participation whatsoever in the estate left by Juan C. Sanchez and/or
Maria Villafranca de Sanchez, or both, and that she likewise waives,
renounces, and relinquishes whatever rigid, share, participation or
interest therein which she has or might have in favor of Rosalia S.
Lugod;

Medina, Misamis Oriental, October 30, 1969.

11. That, the parties hereto mutually waive and renounce in favor of
each other any whatever claims or actions, arising from, connected with,
and as a result of Special Proceedings Nos. 44-M and 1022 of the Court
of First Instance of Misamis Oriental, Rosalia S. Lugod, warranting that
the parcel of land ceded to the other parties herein contains 48 hectares
and 36 ares.
12. That, Rosalia S. Lugod shall assume as she hereby assumes the
payment to Lugod Enterprises, Inc., of the sum of P51,598.93
representing the indebtedness of the estate of Juan C. Sanchez and
Maria Villafranca de Sanchez and the advances made to Rolando
Pedro, Mierly, Alfredo, and Myna all surnamed Sanchez, mentioned in
paragraphs 5 hereto agree to have letters of administration issued in
favor of Rosalia S. Lugod without any bond.
That Rosalia S. Lugod likewise agrees to deliver possession and
enjoyment of the parcel of land herein ceded to petitioners and
intervenors immediately after the signing of this agreement and that the
latter also mutually agree among themselves to have the said lot
subdivided and partitioned immediately in accordance with the
proportion of one sixth (1/6) part for every petitioner and intervenor and
that in the meantime that the partition and subdivision is not yet effected,
the administrations of said parcel of land shall be vested jointly with
Laureta Tampos, guardian ad litem of petitioners and Maria Ramoso,
one of the intervenors who shall see to it that each petitioner and
intervenor is given one sixth (1/6) of the net proceeds of all agricultural
harvest made thereon.

(Sgd.) (Sgd.)
PATRICIO ALBURO ROSALIA S. LUGOD
Intervenor-Oppositor Oppositor
(Sgd.)
MARIA RAMOSO SANCHEZ ASSISTED BY:
Intervenor-Oppositor
(Sgd.)
ASSISTED BY: PABLO S. REYES
R-101-Navarro Bldg.
(Sgd.) Don A. Velez St.
REYNALDO L. FERNANDEZ Cagayan de Oro City
Gingoong City
(Sgd.) (Sgd.)
ROLANDO PEDRO T. SANCHEZ ALFREDO T. SANCHEZ
Petitioner Petitioner
(Sgd.) (Sgd.)
FLORIDA MIERLY T. SANCHEZ MYRNA T. SANCHEZ
Petitioner Petitioner
(Sgd.)
LAURETA TAMPUS
For herself and as Guardian
Ad-Litem of the minors
Florida Mierly, Alfredo, and
Myrna, all surnamed Sanchez
ASSISTED BY:

TEOGENES VELEZ, JR.


Counsel for Petitioners
Cagayan de Oro City

The respondent court grossly erred in granting the petition


for certiorari under Rule 65 considering that the special civil action
of certiorari may not be availed of as a substitute for an appeal and that,
in any event, the grounds invoked in the petition are merely alleged
errors of judgment which can no longer be done in view of the fact that
the decision of the lower court had long become final and executory.

The Clerk of Court


Court of First Instance
Branch III, Medina, Mis. Or.

II
Greetings:
Please set the foregoing compromise agreement for the approval of the
Honorable Court today, Oct. 30, 1969.
(Sgd.) (Sgd.) (Sgd.)
PABLO S. REYES TEOGENES VELEZ, JR. REYNALDO L.
FERNANDEZ
The Memorandum of Agreement dated April 13, 1970, which the parties entered into
with the assistance of their counsel, amended the above compromise. (It will be
reproduced later in our discussion of the second issue raised by the petitioners.)
The Court of Appeals, in a Resolution 10 dated September 4, 1992, initially dismissed
private respondents' petition. Acting, however, on a motion for reconsideration and a
supplemental motion for reconsideration dated September 14, 1992 and September 25,
1992, respectively, 11 Respondent Court thereafter reinstated private respondents'
petition in a resolution 12 dated October 14, 1992.

Prescinding from the foregoing, the respondent court erred in annulling


the decision of the lower court for the reason that a compromise
agreement or partition as the court construed the same to be, executed
by the parties on October 30, 1969 was void and unenforceable the
same not having been approved by the intestate court and that the same
having been seasonably repudiated by petitioners on the ground of
fraud.
III
The respondent court grossly erred in ignoring and disregarding findings
of facts of the lower court that the alleged conveyances of real properties
made by the spouses Juan C. Sanchez and Maria Villafranca just before
their death in favor of their daughter and grandchildren, private
respondents herein, are tainted with fraud or made in contemplation of
death, hence, collationable.
IV

In due course, the Court of Appeals, as earlier stated, rendered its assailed Decision
granting the petition, setting aside the trial court's decision and declaring the modified
compromise agreement valid and binding.

In any event, the respondent court grossly erred in treating the lower
court's declaration of fictitiousness of the deeds of sale as a final
adjudication of annulment.

Hence, this appeal to this Court under Rule 45 of the Rules of Court.
V
The Issues
In this appeal, petitioners invite the Court's attention to the following issues:

The respondent court grossly erred in declaring the termination of the


intestate proceedings even as the lower court had not made a final and
enforceable distribution of the estate of the deceased Juan C. Sanchez.

I
VI

Prescinding from the foregoing, the respondent court grossly erred in not
at least directing respondent Rosalia S. Lugod to deliver the deficiency
of eight (8) hectares due petitioners under the compromise agreement
and memorandum of agreement, and in not further directing her to
include in the inventory properties conveyed under the deeds of sale
found by the lower court to be part of the estate of Juan C. Sanchez. 13
The salient aspects of some issues are closely intertwined; hence, they are hereby
consolidated into three main issues specifically dealing with the following subjects: (1)
the propriety of certiorari as a remedy before the Court of Appeals, (2) the validity of the
compromise agreement, and (3) the presence of fraud in the execution of the
compromise and/or collation of the properties sold.
The Court's Ruling
The petition is not meritorious.
First Issue: Propriety of Certiorari
Before the Court of Appeals
Since private respondents had neglected or failed to file an ordinary appeal within the
reglementary period, petitioners allege that the Court of Appeals erred in allowing
private respondent's recourse to Rule 65 of the Rules of Court. They contend that
private respondents' invocation of certiorari was "procedurally defective." 14 They further
argue that private respondents, in their petition before the Court of Appeals, alleged
errors of the trial court which, being merely errors of judgment and not errors of
jurisdiction, were not correctable by certiorari. 15 This Court disagrees.
Doctrinally entrenched is the general rule that certiorari is not a substitute for a lost
appeal. However, Justice Florenz D. Regalado lists several exceptions to this rule, viz.:
"(1) where the appeal does not constitute a speedy and adequate remedy (Salvadades
vs. Pajarillo, et al., 78 Phil. 77), as where 33 appeals were involved from orders issued
in a single proceeding which will inevitably result in a proliferation of more appeals
(PCIB vs. Escolin, et al., L-27860 and 27896, Mar. 29, 1974); (2) where the orders were
also issued either in excess of or without jurisdiction (Aguilar vs. Tan, L-23600, Jun 30,
1970, Cf. Bautista, et al. vs. Sarmiento, et al., L-45137, Sept. 231985); (3) for certain
special consideration, as public welfare or public policy (See Jose vs. Zulueta, et al.
16598, May 31, 1961 and the cases cited therein); (4) where in criminal actions, the
court rejects rebuttal evidence for the prosecution as, in case of acquittal, there could

be no remedy (People vs. Abalos, L029039, Nov. 28, 1968); (5) where the order is a
patent nullity (Marcelo vs. De Guzman, et al., L-29077, June 29, 1982); and (6) where
the decision in the certiorari case will avoid future litigations (St. Peter Memorial Park,
Inc. vs. Campos, et al., L-38280, Mar. 21, 1975)." 16 Even in a case where the remedy
of appeal was lost, the Court has issued the writ of certiorari where the lower court
patently acted in excess of or outside its jurisdiction, 17 as in the present case.
A petition for certiorari under Rule 65 of the Rules of Court is appropriate and allowable
when the following requisites concur: (1) the writ is directed against a tribunal, board or
officer exercising judicial or quasi-judicial functions; (2) such tribunal, board or officer
has acted without or in excess of jurisdiction, or with grave abuse of discretion
amounting to lack or excess of jurisdiction; and (3) there is no appeal or any plain,
speedy and adequate remedy in the ordinary course of law. 18 After a thorough review
of the case at bar, we are convinced that all these requirements were met.
As a probate court, the trial court was exercising judicial functions when it issued its
assailed resolution. The said court had jurisdiction to act in the intestate proceedings
involved in this case with the caveat that, due to its limited jurisdiction, it could resolve
questions of title only provisionally. 19 It is hornbook doctrine that "in a special
proceeding for the probate of a will, the question of ownership is an extraneous matter
which the probate court cannot resolve with finality. This pronouncement no doubt
applies with equal force to an intestate proceeding as in the case at bar." 20 In the
instant case, the trial court rendered a decision declaring as simulated and fictitious all
the deeds of absolute sale which, on July 26, 1963 and June 26, 1967, Juan C.
Sanchez and Maria Villafranca executed in favor of their daughter, Rosalia Sanchez
Lugod; and grandchildren, namely, Arturo S. Lugod, Evelyn S. Lugod and Roberto S.
Lugod. The trial court ruled further that the properties covered by the said sales must
be subject to collation. Citing Article 1409 (2) of the Civil Code, the lower court nullified
said deeds of sale anddetermined with finality the ownership of the properties subject
thereof . In doing so, it clearly overstepped its jurisdiction as a probate court.
Jurisprudence teaches:
[A] probate court or one in charge of proceedings whether testate or
intestate cannot adjudicate or determine title to properties claimed to be
a part of the estate and which are claimed to belong to outside parties.
All that the said court could do as regards said properties is to determine
whether they should or should not be included in the inventory or list of
properties to be administered by the administrator. If there is not dispute,
well and good, but if there is, then the parties, the administrator, and the

opposing parties have to resort to an ordinary action for a final


determination of the conflicting claims of title because the probate court
cannot do so. 21
Furthermore, the trial court committed grave abuse of discretion when it rendered its
decision in disregard of the parties' compromise agreement. 22 Such disregard, on the
ground that the compromise agreement "was nor approved by the court," 23 is
tantamount to "an evasion of positive duty or to a virtual refusal to perform the duty
enjoined or to act in contemplation and within the bounds of law. " 24

that certiorari is not a substitute for a lost appeal because the trial court's decision and
resolutions were issued without or in excess of jurisdiction, which may thus be
challenged or attacked at any time. "A void judgment for want of jurisdiction is no
judgment at all. It cannot be the source of any right nor the creator of any obligation. All
acts performed pursuant to it and all claims emanating from it have no legal effect.
Hence, it can never become final and any writ of execution based on it is void; ' . . . it
may be said to be a lawless thing which can be treated as an outlaw and slain at sight,
or ignored wherever and whenever it exhibits its head.' " 29
Second Issue: Validity of Compromise Agreement

The foregoing issues clearly involve not only the correctness of the trial court's decision
but also the latter's jurisdiction. They encompass plain errors of jurisdiction and grave
abuse of discretion, not merely errors of judgment. 25 Since the trial court exceeded its
jurisdiction, a petition for certiorari is certainly a proper remedy. Indeed, it is well-settled
that "(a)n act done by a probate court in excess of its jurisdiction may be corrected
by certiorari." 26
Consistent with the foregoing, the following disquisition by respondent appellate court is
apt:
As a general proposition, appeal is the proper remedy of petitioner
Rosalia here under Rule 109 of the Revised Rules of Court. But the
availability of the ordinary course of appeal does not constitute sufficient
ground to [prevent] a party from making use of the extraordinary remedy
of certiorari where appeal is not an adequate remedy or equally
beneficial, speedy and sufficient (Echauz vs. Court of Appeals, 199
SCRA 381). Here, considering that the respondent court has
disregarded the compromise agreement which has long been executed
as early as October, 1969 and declared null and void the deeds of sale
with finality, which, as a probate court, it has no jurisdiction to do, We
deem ordinary appeal is inadequate. Considering further the [trial
court's] granting of [herein petitioners') motion for execution of the
assailed decision, 27 [herein private respondent] Rosalia's resort to the
instant petition [for review on certiorari] is all the more warranted under
the circumstances. 28
We thus hold that the questioned decision and resolutions of the trial court may be
challenged through a special civil action for certiorari under Rule 65 of the Rules of
Court. At the very least, this case is a clear exception to the general rule

Petitioners contend that, because the compromise agreement was executed during the
pendency of the probate proceedings, judicial approval is necessary to shroud it with
validity. They stress that the probate court had jurisdiction over the properties covered
by said agreement. They add that Petitioners Florida Mierly, Alfredo and Myrna were all
miners represented only by their mother/natural guardian, Laureta Tampus. 30
These contentions lack merit. Article 2028 of the Civil Code defines a compromise
agreement as "a contract whereby the parties, by making reciprocal concessions, avoid
a litigation or put an end to one already commenced." Being a consensual contract, it is
perfected upon the meeting of the minds of the parties. Judicial approval is not required
for its perfection. 31 Petitioners' argument that the compromise was not valid for lack of
judicial approval is not novel; the same was raised in Mayuga vs. Court of
Appeals, 32 where the Court, through Justice Irene R. Cortes, ruled:
It is alleged that the lack of judicial approval is fatal to the compromise. A
compromise is a consensual contract. As such, it is perfected upon the
meeting of the minds of the parties to the contract. (Hernandez v.
Barcelon, 23 Phil. 599 [1912]; see also De los Reyes v. de Ugarte, 75
Phil. 505 [1945].) And from that moment not only does it become binding
upon the parties (De los Reyes v. De Ugarte, supra ), it also has upon
them the effect and authority of res judicata (Civil Code, Art. 2037), even
if not judicially approved (Meneses v. De la Rosa, 77 Phil. 34 [1946];
Vda. De Guilas v. David, 132 Phil. 241, L-24280, 23 SCRA 762 [May 27,
1968]; Cochingyan v. Cloribel, L-27070-71 [April 22, 1977], 76 SCRA
361). (Emphasis found in the original.)
In the case before us, it is ineludible that the parties knowingly and freely entered into a
valid compromise agreement. Adequately assisted by their respective counsels, they

each negotiated its terms and provisions for four months; in fact, said agreement was
executed only after the fourth draft. As noted by the trial court itself, the first and second
drafts were prepared successively in July, 1969; the third draft on September 25, 1969;
and the fourth draft, which was finally signed by the parties on October 30,
1969, 33 followed. Since this compromise agreement was the result of a long drawn out
process, with all the parties ably striving to protect their respective interests and to
come out with the best they could, there can be no doubt that the parties entered into it
freely and voluntarily. Accordingly, they should be bound thereby. 34 To be valid, it is
merely required under the law to be based on real claims and actually agreed upon in
good faith by the parties thereto. 35

I Are the properties which are the object of the sale by the
deceased spouses to their grandchildren collationable?

Indeed, compromise is a form of amicable settlement that is not only allowed but also
encouraged in civil cases.36 Article 2029 of the Civil Code mandates that a "court shall
endeavor to persuade the litigants in a civil case to agree upon some fair compromise."

Furthermore, the 27-page Memorandum dated February 17, 1990 filed by petitioners
before the Regional Trial Court 40 readily reveals that they never questioned the validity
of the compromise. In their comment before the Court of Appeals, 41 petitioners based
their objection to sad compromise agreement on the solitary "reason that it was tainted
with fraud and deception," zeroing specifically on the alleged fraud committed by
private respondent Rosalia S. Lugod. 42 The issue of minority was first raised only in
petitioners' Motion for Reconsideration of the Court of Appeals' Decision; 43 thus, it "is
as if it was never duly raised in that court at all." 44 Hence, this Court cannot now, for the
first time on appeal, entertain this issue, for to do so would plainly violate the basic rule
of fair play, justice and due process. 45 We take this opportunity to reiterate and
emphasize the well-settled rule that "(a)n issue raised for the first time on appeal and
not raised timely in the proceedings in the lower court is barred by estoppel. Questions
raised on appeal must be within the issues framed by the parties and, consequently,
issues not raised in the trial court cannot be raised for the first time on appeal." 46

In opposing the validity and enforcement of the compromise agreement, petitioners


harp on the minority of Florida Mierly, Alfredo and Myna. Citing Article 2032 of the Civil
Code, they contend that the court's approval is necessary in compromises entered into
by guardians and parents in behalf of their wards or children. 37
However, we observe that although denominated a compromise agreement, the
document in this case is essentially a deed of partition, pursuant to Article 1082 of the
Civil Code which provides that "[e]very act which is intended to put an end to indivision
among co-heirs and legatees or devisees is deemed to be a partition, although it should
purport to be a sale, an exchange, a compromise, or any other transaction."
For a partition to be valid, Section 1, Rule 74 of the Rules of Court, requires the
concurrence of the following conditions: (1) the decedent left no will; (2) the decedent
left no debts, or if there were debts left, all had been paid; (3) the heirs and liquidators
are all of age, or if they are minors, the latter are represented by their judicial guardian
or legal representatives; and (4) the partition was made by means of a public
instrument or affidavit duly filed with the Register of Deeds. 38 We find that all the
foregoing requisites are present in this case. We therefore affirm the validity of the
parties' compromise agreement/partition in this case.
In any event, petitioners neither raised nor ventilated this issue in the trial court. This
new question or matter was manifestly beyond the pale of the issues or questions
submitted and threshed out before the lower court which are reproduced below, viz.:

II Are the properties which are the object of the sale by


the deceased spouses to their legitimate daughter also
collationable?
III The first and second issues being resolved, how much
then is the rightful share of the four (4) recognized
illegitimate children? 39

The petitioners likewise assail as void the provision on waiver contained in No. 8 of the
aforequoted compromise, because it allegedly constitutes a relinquishment by
petitioners of "a right to properties which were not known." 47They argue that such
waiver is contrary to law, public policy, morals or good custom. The Court disagrees.
The assailed waiver pertained to their hereditary right to properties belonging to the
decedent's estate which were not included in the inventory of the estate's properties. It
also covered their right to other properties originally belonging to the spouses Juan
Sanchez and Maria Villafranca de Sanchez which have been transferred to other
persons. In addition, the parties agreed in the compromise to confirm and ratify said
transfers. The waiver is valid because, contrary to petitioners' protestation, the parties
waived a known and existing interest their hereditary right which was already vested
in them by reason of the death of their father. Article 777 of the Civil Code provides that
"(t)he rights to the succession are transmitted from the moment of death of the

decedent." Hence, there is no legal obstacle to an heir's waiver of his/her hereditary


share "even if the actual extent of such share is not determined until the subsequent
liquidation of the estate." 48 At any rate, such waiver is consistent with the intent and
letter of the law advocating compromise as a vehicle for the settlement of civil
disputes. 49
Finally, petitioners contend that Private Respondent Rosalia T. Lugod's alleged
fraudulent acts, specifically her concealment of some of the decedent's properties,
attended the actual execution of the compromise agreement.50 This argument is
debunked by the absence of any substantial and convincing evidence on record
showing fraud on her part. As aptly observed by the appellate court:
[Herein petitioners] accuse [herein private respondent] Rosalia of fraud
or deception by alleging, inter alia, that the parcel of land given to them
never conformed to the stated area, i.e., forty-eight (48) hectares, as
stated in the compromise agreement. We find this argument
unconvincing and unmeritorious. [Herein petitioners'] averment of fraud
on the part of [herein private respondent] Rosalia becomes untenable
when We consider the memorandum of agreement they later executed
with [herein private respondent] Rosalia wherein said compromise
agreement was modified by correcting the actual area given to [herein
petitioners] from forty-eight (48) hectares to thirty-six (36) hectares only.
If the actual area allotted to them did not conform to the 48 hectare area
stated in the compromise agreement, then why did they agree to the
memorandum of agreement whereby their share in the estate of their
father was even reduced to just 36 hectares? Where is fraud or
deception there? Considering that [herein petitioners] were ably
represented by their lawyers in executing these documents and who
presumably had explained to them the import and consequences
thereof, it is hard to believe their charge that they were defrauded and
deceived by [herein private respondent] Rosalia.
If the parcel of land given to [herein petitioners], when actually surveyed,
happened to be different in area to the stated area of 48 hectares in the
compromise agreement, this circumstance is not enough proof of fraud
or deception on [herein private respondent] Rosalia's part. Note that Tax
Declaration No. 06453 plainly discloses that the land transferred to
[herein petitioners] pursuant to the compromise agreement contained an
area of 48 hectares (Annex "A", Supplemental Reply). And when [herein

petitioners] discovered that the land allotted to them actually contained


only 24 hectares, a conference between the parties took place which led
to the execution and signing of the memorandum of agreement wherein
[herein petitioners'] distributive share was even reduced to 36 hectares.
In the absence of convincing and clear evidence to the contrary, the
allegation of fraud and deception cannot be successfully imputed to
[herein private respondent] Rosalia who must be presumed to have
acted in good faith. 51
The memorandum of agreement freely and validly entered into by the parties on April
13, 1970 and referred to above reads:
MEMORANDUM OF AGREEMENT
The parties assisted by their respective counsel have agreed as they
hereby agree:
1. To amend the compromise agreement executed by them on October
30, 1969 so as to include the following:
a. Correction of the actual area being given to the
petitioners and intervenors, all illegitimate children of the
late Juan C. Sanchez, forty-eight (48) hectares, thirty-six
(36) ares as embodied in the aforementioned compromise
agreement to thirty-six (36) hectares only, thus enabling
each of them to get six (6) hectares each.
b. That the said 36-hectare area shall be taken from that
parcel of land which is now covered by O.C.T. No. 146
(Patent No. 30012) and the adjoining areas thereof
designated as Lot A and Lot C as reflected on the sketch
plan attached to the record of this case prepared by
Geodetic Engineer Olegario E. Zalles pursuant to the
Court's commission of March 10, 1970 provided,
however, that if the said 36-hectare area could not be
found after adding thereto the areas of said lots A and C,
then the additional area shall be taken from what is
designated as Lot B, likewise also reflected in the said
sketch plan attached to the records;

c. That the partition among the six illegitimate children of


the late Juan C. Sanchez (petitioners and intervenors)
shall be effective among themselves in such a manner to
be agreed upon by them, each undertaking to assume
redemption of whatever plants found in their respective
shares which need redemption from the tenants thereof
as well as the continuity of the tenancy agreements now
existing and covering the said shares or areas.
d. The subdivision survey shall be at the expense of the
said petitioners and intervenors prorata.
e. That the administratrix agrees to deliver temporary
administration of the area designated as Lot 5 of the
Valles Sketch Plan pending final survey of the said 36hectare area.
Cagayan de Oro City, April 13, 1970.
(Sgd.)
LAURETA TAMPOS
For herself and as Guardian
ad-litem of Rolando, Mierly,
Alfredo and Myrna, all
surnamed Sanchez
Assisted by:
(Sgd.)
TEOGENES VELEZ, Jr.
Counsel for Petitioners
(Sgd.)
ROSALIA S. LUGOD
Administratrix
Assisted by:

(Sgd.)
PABLO S. REYES
Counsel for Administratrix
(Sgd.)
MARIA RABOSO SANCHEZ
Intervenor 52
Not only did the parties knowingly enter into a valid compromise agreement; they even
amended it when they realized some errors in the original. Such correction emphasizes
the voluntariness of said deed.
It is also significant that all the parties, including the then minors, had
already consummated and availed themselves of the benefits of their
compromise. 53 This Court has consistently ruled that "a party to a compromise cannot
ask for a rescission after it has enjoyed its benefits." 54 By their acts, the parties are
ineludibly estopped from questioning the validity of their compromise agreement.
Bolstering this conclusion is the fact that petitioners questioned the compromise
only nine years after its execution, when they filed with the trial court their Motion to
Defer Approval of Compromise Agreement, dated October 26, 1979. 55 In hindsight, it is
not at all farfetched that petitioners filed said motion for the sole reason that they may
have felt shortchanged in their compromise agreement or partition with private
respondents, which in their view was unwise and unfair. While we may sympathize with
this rueful sentiment of petitioners, we can only stress that this alone is not sufficient to
nullify or disregard the legal effects of said compromise which, by its very nature as a
perfected contract, is binding on the parties. Moreover, courts have no jurisdiction to
look into the wisdom of a compromise or to render a decision different therefrom. 56 It is
a well-entrenched doctrine that "the law does not relieve a party from the effects of an
unwise, foolish, or disastrous contract, entered into with all the required formalities and
with full awareness of what he was doing" 57 and "a compromise entered into and
carried out in good faith will not be discarded even if there was a mistake of law or fact,
(McCarthy vs. Barber Steamship Lines, 45 Phil. 488) because courts have no power to
relieve parties from obligations voluntarily assumed, simply because their contracts
turned out to be disastrous deals or unwise investments." 58 Volenti non fit injuria.
Corollarily, the petitioners contend that the Court of Appeals gravely abused its
discretion in deeming Special Proceedings Nos. 44-M and 1022 "CLOSED and
TERMINATED," arguing that there was as yet no order of distribution of the estate
pursuant to Rule 90 of the Rules of Court. They add that they had not received their full
share thereto. 59 We disagree. Under Section 1, Rule 90 of the Rules of Court, an order

for the distribution of the estate may be made when the "debts, funeral charges, and
expenses of administration, the allowance to the widow, and inheritance tax, if any,"
had been paid. This order for the distribution of the estate's residue must contain the
names and shares of the persons entitled thereto. A perusal of the whole record,
particularly the trial court's conclusion, 60 reveals that all the foregoing requirements
already concurred in this case. The payment of the indebtedness of the estates of Juan
C. Sanchez and Maria Villafranca in the amount of P51,598.93 was shouldered by
Private Respondent Rosalia, who also absorbed or charged against her share the
advances of Rolando T. Lugod in the sum of P8,533.94, in compliance with Article 1061
of the Civil Code on collation. 61 Furthermore, the compromise of the parties, which is
the law between them, already contains the names and shares of the heirs to the
residual estate, which shares had also been delivered. On this point, we agree with the
following discussion of the Court of Appeals:
But what the (trial court) obviously overlooked in its appreciation of the
facts of this case are the uncontroverted facts that (herein petitioners)
have been in possession and ownership of their respective distributive
shares as early as October 30, 1969 and they have received other
properties in addition to their distributive shares in consideration of the
compromise agreement which they now assail. Proofs thereof are Tax
Declarations No. 20984, 20985, 20986, 20987, 20988, 20989 and 20990
(Annexes "B" to "H", Supplemental Reply) in the respective names of
(herein petitioners), all for the year 1972. (Herein petitioners) also
retained a house and lot, a residential lot and a parcel of agricultural
land (Annexes "I", "J" and "K", Ibid.) all of which were not considered in
the compromise agreement between the parties. Moreover, in the
compromise agreement per se, it is undoubtedly stated therein that cash
advances in the aggregate sum of P8,533.94 were received by (herein
petitioners) after October 21, 1968 (Compromise Agreement, par. 5) 62
All the foregoing show clearly that the probate court had essentially finished said
intestate proceedings which, consequently, should be deemed closed and terminated.
In view of the above discussion, the Court sees no reversible error on the part of the
Court of Appeals.
Third Issue: Fraud and Collation
Petitioners fault Respondent Court for not ordering Private Respondent Rosalia T.
Lugod to deliver to them the deficiency as allegedly provided under the compromise

agreement. They further contend that said court erred in not directing the provisional
inclusion of the alleged deficiency in the inventory for purposes of collating the
properties subject of the questioned deeds of sale. 63 We see no such error. In the trial
court, there was only one hearing conducted, and it was held only for the reception of
the evidence of Rosalia S. Lugod to install her as administratrix of the estate of Maria
Villafranca. There was no other evidence, whether testimonial or otherwise, "received,
formally offered to, and subsequently admitted by the probate court below"; nor was
there "a trial on the merits of the parries' conflicting claims." 64 In fact, the petitioners
"moved for the deferment of the compromise agreement on the basis of alleged
fraudulent concealment of properties NOT because of any deficiency in the land
conveyed to them under the agreements." 65Hence, there is no hard evidence on record
to back up petitioners' claims.
In any case, the trial court noted Private Respondent Rosalia's willingness to reimburse
any deficiency actually proven to exist. It subsequently ordered the geodetic engineer
who prepared the certification and the sketch of the lot in question, and who could have
provided evidence for the petitioners, "to bring records of his relocation
survey." 66 However, Geodetic Engineer Idulsa did not comply with the
court's subpoena duces tecum and ad testificandum. Neither did he furnish the required
relocation survey. 67No wonder, even after a thorough scrutiny of the records, this Court
cannot find any evidence to support petitioners' allegations of fraud against Private
Respondent Rosalia.
Similarly, petitioners' allegations of fraud in the execution of the questioned deeds of
sale are bereft of substance, in view of the palpable absence of evidence to support
them. The legal presumption of validity of the questioned deeds of absolute sale, being
duly notarized public documents, has not been overcome. 68 On the other hand, fraud is
not presumed. It must be proved by clear and convincing evidence, and not by mere
conjectures or speculations. We stress that these deeds of sale did not involve
gratuitous transfers of future inheritance; these were contracts of sale perfected by the
decedents during their lifetime. 69 Hence, the properties conveyed thereby are not
collationable because, essentially, collation mandated under Article 1061 of the Civil
Code contemplates properties conveyed inter vivos by the decedent to an heir by way
of donation or other gratuitous title.
In any event, these alleged errors and deficiencies regarding the delivery of shares
provided in the compromise, concealment of properties and fraud in the deeds of sale
are factual in nature which, as a rule, are not reviewable by this Court in petitions under
Rule 45. 70 Petitioners have failed to convince us that this case constitutes an exception

to such rule. All in all, we find that the Court of Appeals has sufficiently addressed the
issues raised by them. Indeed, they have not persuaded us that said Court committed
any reversible error to warrant a grant of their petition.
WHEREFORE, the petition is hereby DENIED and the assailed Decision of the Court of
Appeals is AFFIRMED.
SO ORDERED.

G.R. No. L-21993

June 21, 1966

ANGELA RODRIGUEZ, MARIA RODRIGUEZ, ET AL., petitioners, vs. HON. JUAN DE


BORJA, as Judge of the Court of First Instance of Bulacan, Branch III,
ANATOLIA PANGILINAN and ADELAIDA JACALAN, respondents.
Lorenzo Somulong for petitioners.
Torres and Torres for respondents.
REYES, J.B.L., J.:
Petitioners Angela, Maria, Abelardo and Antonio, surnamed Rodriguez, petition this
Court for a writ of certiorariand prohibition to the Court of First Instance of Bulacan, for
its refusal to grant their motion to dismiss its Special Proceeding No. 1331, which said
Court is alleged to have taken cognizance of without jurisdiction.
The facts and issues are succinctly narrated in the order of the respondent court, dated
June 13, 1963 (Petition, Annex 0), in this wise:
It is alleged in the motion to dismiss filed by Angela, Maria, Abelardo and
Antonio Rodriguez, through counsel, that this Court "has no jurisdiction to try the
above-entitled case in view of the pendency of another action for the settlement
of the estate of the deceased Rev. Fr. Celestino Rodriguez in the Court of First
Instance of Rizal, namely, Sp. Proceedings No. 3907 entitled 'In the matter of
the Intestate Estate of the deceased Rev. Fr. Celestino Rodriguez which was
filed ahead of the instant case".
The records show that Fr. Celestino Rodriguez died on February 12, 1963 in the
City of Manila; that on March 4, 1963, Apolonia Pangilinan and Adelaida Jacalan
delivered to the Clerk of Court of Bulacan a purported last will and testament of
Fr. Rodriguez; that on March 8, 1963, Maria Rodriguez and Angela Rodriguez,
through counsel filed a petition for leave of court to allow them to examine the
alleged will; that on March 11, 1963 before the Court could act on the petition,
the same was withdrawn; that on March 12, 1963, aforementioned petitioners
filed before the Court of First Instance of Rizal a petition for the settlement of the
intestate estate of Fr. Rodriguez alleging, among other things, that Fr.
Rodriguez was a resident of Paraaque, Rizal, and died without leaving a will
and praying that Maria Rodriguez be appointed as Special Administratrix of the

estate; and that on March 12, 1963 Apolonia Pangilinan and Adelaida Jacalan
filed a petition in this Court for the probation of the will delivered by them on
March 4, 1963. It was stipulated by the parties that Fr. Rodriguez was born in
Paraaque, Rizal; that he was Parish priest of the Catholic Church of Hagonoy,
Bulacan, from the year 1930 up to the time of his death in 1963; that he was
buried in Paraaque, and that he left real properties in Rizal, Cavite, Quezon
City and Bulacan.
The movants contend that since the intestate proceedings in the Court of First
Instance of Rizal was filed at 8:00 A.M. on March 12, 1963 while the petition for
probate was filed in the Court of First Instance of Bulacan at 11:00 A.M. on the
same date, the latter Court has no jurisdiction to entertain the petition for
probate, citing as authority in support thereof the case of Ongsingco Vda. de
Borja vs. Tan and De Borja, G.R. No. 7792, July 27, 1955.
The petitioners Pangilinan and Jacalan, on the other hand, take the stand that
the Court of First Instance of Bulacan acquired jurisdiction over the case upon
delivery by them of the will to the Clerk of Court on March 4, 1963, and that the
case in this Court therefore has precedence over the case filed in Rizal on
March 12, 1963.
The Court of First Instance, as previously stated denied the motion to dismiss on the
ground that a difference of a few hours did not entitle one proceeding to preference
over the other; that, as early as March 7, movants were aware of the existence of the
purported will of Father Rodriguez, deposited in the Court of Bulacan, since they filed a
petition to examine the same, and that movants clearly filed the intestate proceedings
in Rizal "for no other purpose than to prevent this Court (of Bulacan) from exercising
jurisdiction over the probate proceedings". Reconsideration having been denied,
movants, now petitioners, came to this Court, relying principally on Rule 73, section 1 of
the Rules of Court, and invoking our ruling in Ongsingco vs. Tan and De Borja, L-7792,
July 27, 1955.
SECTION 1. Where estate of deceased persons settled. If the decedent is an
inhabitant of the Philippines at the time of his death, whether a citizen or an
alien, his will shall be proved, or letters of administration granted, and his estate
settled, in the Court of First Instance in the province in which he resides at the
time of his death, and if he is an inhabitant of a foreign country, the Court of First
Instance of any province which he had estate. The court first taking cognizance

of the settlement of the estate of a decedent, shall exercise jurisdiction to the


exclusion of all other courts. The jurisdiction assumed by a court, as far as it
depends on the place of residence of the decedent, or of the location of his
estate, shall not be contested in a suit or proceeding, except in an appeal from
that court, in the original case, or when the want of jurisdiction appears on the
record.
We find this recourse to be untenable. The jurisdiction of the Court of First Instance of
Bulacan became vested upon the delivery thereto of the will of the late Father
Rodriguez on March 4, 1963, even if no petition for its allowance was filed until later,
because upon the will being deposited the court could, motu proprio, have taken steps
to fix the time and place for proving the will, and issued the corresponding notices
conformably to what is prescribed by section 3, Rule 76, of the Revised Rules of Court
(Section 3, Rule 77, of the old Rules):
SEC. 3. Court to appoint time for proving will. Notice thereof to be published.
When a will is delivered to, or a petition for the allowance of a will is filed in, the
Court having jurisdiction, such Court shall fix a time and place for proving the
will when all concerned may appear to contest the allowance thereof, and shall
cause notice of such time and place to be published three (3) weeks
successively, previous to the time appointed, in a newspaper of general
circulation in the province.
But no newspaper publication shall be made where the petition for probate has
been filed by the testator himself.
The use of the disjunctive in the words "when a will is delivered to OR a petition for the
allowance of a will is filed" plainly indicates that the court may act upon the mere
deposit therein of a decedent's testament, even if no petition for its allowance is as yet
filed. Where the petition for probate is made after the deposit of the will, the petition is
deemed to relate back to the time when the will was delivered. Since the testament of
Fr. Rodriguez was submitted and delivered to the Court of Bulacan on March 4, while
petitioners initiated intestate proceedings in the Court of First Instance of Rizal only on
March 12, eight days later, the precedence and exclusive jurisdiction of the Bulacan
court is incontestable.1wph1.t
But, petitioners object, section 3 of revised Rule 76 (old Rule 77) speaks of a will being
delivered to "the Court having jurisdiction," and in the case at bar the Bulacan court did
not have it because the decedent was domiciled in Rizal province. We can not

disregard Fr. Rodriguez's 33 years of residence as parish priest in Hagonoy, Bulacan


(1930-1963); but even if we do so, and consider that he retained throughout
some animus revertendi to the place of his birth in Paraaque, Rizal, that detail would
not imply that the Bulacan court lacked jurisdiction. As ruled in previous decisions, the
power to settle decedents' estates is conferred by law upon all courts of first instance,
and the domicile of the testator only affects the venue but not the jurisdiction of the
Court (In re Kaw Singco, 74 Phil. 239; Reyes vs. Diaz, 73 Phil. 484; Bernabe vs.
Vergara, 73 Phil. 676). Neither party denies that the late Fr. Rodriguez is deceased, or
that he left personal property in Hagonoy, province of Bulacan (t.s.n. p. 46, hearing of
June 11, 1963, Annex "H", Petition, Rec., p. 48). That is sufficient in the case before us.
In the Kaw Singco case (ante) this Court ruled that:
"... If we consider such question of residence as one affecting the jurisdiction of
the trial court over the subject-matter, the effect shall be that the whole
proceedings including all decisions on the different incidents which have arisen
in court will have to be annulled and the same case will have to be commenced
anew before another court of the same rank in another province. That this is of
mischievous effect in the prompt administration of justice is too obvious to
require comment. (Cf. Tanunchuan vs. Dy Buncio & Co., G.R. No. 48206,
December 31, 1942). Furthermore, section 600 of Act No. 190, providing that
the estate of a deceased person shall be settled in the province where he had
last resided, could not have been intended as defining the jurisdiction of the
probate court over the subject matter, because such legal provision is contained
in a law of procedure dealing merely with procedural matters, and, as we have
said time and again, procedure is one thing and jurisdiction over the subject
matter is another. (Attorney General vs. Manila Railroad Company, 20 Phil.
523.) The law of jurisdiction Act No. 136, Section 56, No. 5 confers upon
Courts of First Instance jurisdiction over all probate cases independently of the
place of residence of the deceased.1 Since, however, there are many Courts of
First Instance in the Philippines, the Law of Procedure, Act No. 190, section
600, fixes the venue or the place where each case shall be brought. Thus, the
place of residence of the deceased is not an element of jurisdiction over the
subject matter but merely of venue. And it is upon this ground that in the new
Rules of Court the province where the estate of a deceased person shall be
settled is properly called "venue" (Rule 75, section 1.) Motion for reconsideration
is denied.

The estate proceedings having been initiated in the Bulacan Court of First Instance
ahead of any other, that court is entitled to assume jurisdiction to the exclusion of all
other courts, even if it were a case of wrong venue by express provisions of Rule 73
(old Rule 75) of the Rules of Court, since the same enjoins that:
The Court first taking cognizance of the settlement of the estate of a decedent
shall exercise jurisdiction to the exclusion of all other courts. (Sec. 1)

(4) When the heir instituted is incapable of succeeding, except in cases


provided in this Code.
Therefore, as ruled in Castro, et al. vs. Martinez, 10 Phil. 307, "only after final decision
as to the nullity of testate succession could an intestate succession be instituted in the
form of pre-established action". The institution of intestacy proceedings in Rizal may not
thus proceed while the probate of the purported will of Father Rodriguez is pending.

This disposition presupposes that two or more courts have been asked to take
cognizance of the settlement of the estate. Of them only one could be of proper venue,
yet the rule grants precedence to that Court whose jurisdiction is first invoked, without
taking venue into account.

We rule that the Bulacan Court of First Instance was entitled to priority in the settlement
of the estate in question, and that in refusing to dismiss the probate. proceedings, said
court did not commit any abuse of discretion. It is the proceedings in the Rizal Court
that should be discontinued.

There are two other reasons that militate against the success of petitioners. One is that
their commencing intestate proceedings in Rizal, after they learned of the delivery of
the decedent's will to the Court of Bulacan, was in bad faith, patently done with a view
to divesting the latter court of the precedence awarded it by the Rules. Certainly the
order of priority established in Rule 73 (old Rule 75) was not designed to convert the
settlement of decedent's estates into a race between applicants, with the administration
of the properties as the price for the fleetest.

Wherefore, the writ of certiorari applied for is denied. Costs against petitioners
Rodriguez.

The other reason is that, in our system of civil law, intestate succession is only
subsidiary or subordinate to the testate, since intestacy only takes place in the absence
of a valid operative will. Says Article 960 of the Civil Code of the Philippines:
ART. 960. Legal or intestate succession takes place:
(1) If a person dies without a will, or with a void will, or one which has
subsequently lost its validity;
(2) When the will does not institute an heir to, or dispose of all the property
belonging to the testator. In such case, legal succession shall take place only
with respect to the property in which the testator has not disposed;
(3) If the suspensive condition attached to the institution of heir does not happen
or is not fulfilled, or if the heir dies before the testator, or repudiates the
inheritance, there being no substitution, and no right of accretion takes place;

G.R. No. L-27200 August 18, 1972


TESTATE ESTATE OF GLICERIA A. DEL ROSARIO, deceased, CONSUELO S.
GONZALES VDA. DE PRECILLA, petitioner-administratrix, vs. SEVERINA NARCISO,
ROSA NARCISO, JOSEFA NARCISO, VICENTE MAURICIO, DELFEN MAURICIO,
REMEDIOS NARCISO, ENCARNACION NARCISO, MARIA NARCISO, EDUARDO
NARCISO, FR. LUCIO V. GARCIA, ANTONIO JESUS DE PRAGA, MARIA
NATIVIDAD DE JESUS-DR. JAIME ROSARIO, ET AL., NATIVIDAD DEL ROSARIOSARMIENTO, and PASCUALA NARCISO-MANAHAN, oppositors-appellants.
Antonio Enrile Inton for petitioner Rev. Father Lucio V. Garcia.
Pedro V. Garcia for petitioner Antonio Jesus de Praga, et al.
Leandro Sevilla & Ramon C. Aquino and Melquiades M. Virata, Jr. for respondent
Consuelo S. Gonzales Vda. de Precilla.
Lorenzo C. Gella for respondent Register of Deeds of Manila.
Leandro Sevilla & Ramon C. Aquino for petitioner-administratrix.
Castro, Makalintal & Associates for oppositors-appellants Encarnacion Narciso, et al. .
Pedro Garcia for oppositors-appellant Dr. Jaime Rosario et al.
Antonio Enrile Inton for oppositors-appellants Fr. Lucio V. Garcia and Antonio Jesus de
Praga.
Salonga, Ordoez, Yap, Sicat & Associates for oppositors-appellants Severina Narciso,
et al.
George G. Arbolario and Sixto R. Reyes & Vicente Redor for oppositors-appellants
Natividad del Rosario-Sarmiento, et al.
RESOLUTION
BARREDO, J.:p
Motion for reconsideration and/or new trial filed by petitioner-appellee praying that the
decision in this case promulgated on April 30, 1970 disallowing the purported will,
Exhibit D, of the deceased Gliceria A. del Rosario, be reconsidered and set aside and
that the judgment of the trial court admitting the same to probate be affirmed, or, in the
alternative, that before finally resolving the said issue of the probate of said purported
will, this case be remanded to the court a quo in order that further evidence may be
admitted relative to the factual question of whether or not the said deceased could have
read the said document on December 29, 1960, the date of the alleged execution of the
same, considering the condition of her eyes then.
The grounds alleged in support of the present motion are:

1. Article 808 of the Civil Code applies solely to a blind testator and does
not include one with a defective eyesight or one who is illiterate or
cannot read;
2. The testimony of Dr. Jesus V. Tamesis, upon which the disallowance
of the will was based, has no probative value, because: (a) it is premised
upon a grave factual error; (b) such testimony is contradicted by his own
clinical record; and (e) it is glaringly superficial and evasive;
3. The testatrix, Doa Gliceria A. del Rosario, was not blind; she could
read and did actually read the will now in question, Exhibit "D"; and
4. Should this Honorable Court still believe that it is necessary to reopen
the case to admit additional evidence showing that Doa Gliceria A. del
Rosario was not blind, could read and did actually read the aforesaid
will, Exhibit "D", then we respectfully move for a new trial for the purpose
of admitting such evidence.
No doubt counsel's scholarly dissertation of the above first ground both in their motion
and in their reply to appellant's opposition is impressive and, insofar as the writer of this
opinion is concerned, quite logically persuasive and plausible in many respects; but
considering the view the Court takes, as hereinunder discussed, of the main factual
issue here, on the determination of which the point discussed by counsel has to be
predicated, the more fitting occasion to resolve the question thus very ably and
vigorous pressed upon would be if and when this case comes back to this Court later,
in the event that anyone of the parties should find it necessary to submit an appeal after
the court a quo has held the further proceedings herein ordered. At the moment,
consideration and resolution of the above second ground should suffice to terminate
the present proceeding in this Court.
Under the said second ground, counsel for movant contends that the testimony of Dr.
Jesus V. Tamesis, the witness of the so-called intestate heirs, upon which the Court has
relied in disallowing the supposed testament, Exhibit D, in its decision now under
review, has no probative value. In other words, it is argued that said testimony should
no have been believed.
In this connection, it should not, of course, be lost sight of that, as pointed out by
appellants in their opposition to the present motion, there are other considerations
stated in the decision additionally supporting the assailed conclusion of the Court that
"with the condition of her eyesight in August, 1960, and there is no evidence that it had
improve by 29 December, 1960, Gliceria del Rosario was incapable of reading and
could not have read the will supposedly signed by her on 29 December, 1960." For
instance, the numerous typographical errors in the document and its in formal and
untidy appearance, considering its importance were taken into account as suggestive of
the fact that the supposed testatrix might not have been capacitated to read the same,

as otherwise, it is to be normally assumed that she would have refused or, at least,
hesitated to proceed with its execution without the proper corrections being made first.
Accordingly, for the Court now to accede to movant's prayer that it overturn its previous
findings in this case, more than mere ratiocination regarding the testimony of Dr.
Tamesis is needed. The pivotal issue here is one of fact, and bare arguments, no
matter how forceful, if not based on concrete and substantial evidence which the Court
might have overlooked and which would convincingly show that said doctor's
observations and conclusions are or might be gravely erroneous cannot suffice to move
the Court to reverse itself. Stated differently, the question that confronts Us now is this:
does appellee's motion at hand allege and demonstrate that there are factual
circumstances extant in the record, related to the doctor's testimony or forming part
thereof and probably not clearly brought out before, which if considered now would tend
to show that the judgment of this Court may be materially erroneous on account
precisely of Our having relied on said testimony?
On this score, movant points out that the testimony of Dr. Tamesis is belied by the very
records referred to by said witness himself in the material portions of his declaration in
court. Certainly, this point is serious, for if it turns out to be supported by the record,
what appears to be one of the main props of the decision would naturally fall and this
may eventually cause modification of the judgment, depending on how materially
important such discordance is in the resolution of the very issue of whether or not the
deceased could have read Exhibit D on December 29, 1960. Consequently, a second
look at the relevant evidence discussed in the motion is more than justified.
Before going, however, to the points raised by movant, it bears stating that the first
thing that would strike anyone who reviews the testimony of Dr. Tamesis is that the
reason or purpose of the so-called appellant intestate heirs in presenting the same is
not clear, considering that in none of the nine formal and detailed oppositions filed in
the Court below, sometimes individually, at other times jointly by the two groups of
appellants, is there any specific indication of the ground for disallowance now under
discussion, namely, that there was failure to comply with the requirement of Article 808
of the Civil Code because Doa Gliceria, if she was not blind, had such a defective
eyesight on December 29, 1960 that she could not have read the will in question,
Exhibit D. In other words, judged from the grounds of disallowance alleged in their
formal oppositions on record before the witness was presented, there was nothing to
indicate that the good doctor was going to testify on the alleged totally defective reading
vision of the right eye of the deceased, which paucity, as We see it now, might account
for the rather ambiguous, or equivocal tenor of declarations regarding the most material
facts in dispute. It may be pointed out that the so-called testate heirs presented by
Attys. Inton and Garcia closed their case on February 23, 1966 without presenting any
evidence of their own on such alleged defective eyesight of Doa Gliceria; neither does
the record show that they adopted the testimony of Dr. Tamesis, offered by the socalled intestate heirs, thru Atty. Ordoez when the latter closed his evidence at the

hearing of March 2, 1966. Indeed, this particular ground of opposition seems not to
have surfaced until after Dr. Tamesis had already testified, for it was only his
memorandum submitted to the court after the trial, that Atty. Ordoez, the counsel for
oppositors Severina, Rosa and Josefa Narciso and Vicente and Delfin Mauricio, some
of the intestate heirs, discussed the same for the first time. Even the discussions
among counsel during the course of the doctor's testimony hardly suggest such point.
No wonder, in making reference to the oppositions filed with the lower court, the
learned trial Judge said in his decision:
The probate of the will has been opposed by several persons. For the
purpose of simplifying the proceedings and in order to avoid confusion,
the Court divided them into the so-called "testate" heirs and "intestate"
heirs. The testate heirs are oppositors who stand to inherit under a prior
will allegedly executed by the testatrix in 1956; while the intestate heirs
are those who stand to inherit in case of intestacy. The documentary
exhibits submitted by them had been marked with the appendix "Garcia,
et als." for the testate heirs, and "Narciso, et als." for the intestate heirs.
After the close of the proceedings, memoranda were filed by the
petitioner and by both the testate heirs and the intestate heirs, while a
separate memorandum was filed in behalf of oppositors Natividad del
Rosario-Sarmiento and Pascuala Narciso-Manahan:
In behalf of the intestate heirs, the probate is opposed on the following
grounds:
(1) The physical appearance of the will itself reveals irregularities in its
execution;
(2) There had been incontrovertible evidence presented that the testatrix
did not have testamentary capacity;
(3) The testimonies of the attesting witnesses and of the notary public
show irreconcilable inconsistencies on material matters; and
(4) The proponent and her family had been guilty of suppressing material
evidence and using perjured testimony in support of the probate of the
will.
On the part of the testate heirs, it is averred that Exhibit D is not the true
and authentic will of the decedent, the true will of the deceased being
one executed on June 9, 1956 (Exhibit 9-Garcia, et als.) before Notary
Public Jose Ayala; the 1960 will was not executed in accordance with
law; and inconsistencies in the testimonies of the witnesses presented
by the proponent are vital to its probate.
Oppositors Natividad del Rosario-Sarmiento and Pascuala NarcisoManahan contend that (1) from August 30, 1960 up to December 29,

1960, the testatrix was a blind woman for reading purposes and could
not have read the will; (2) there is no evidence presented as to who
prepared or typewrote the will Exhibit D; (3) the alleged will Exhibit D
was not signed by the testatrix in the presence of the witnesses and of
the notary public nor in the presence of each other on December 29,
1960; (4) the signature of the deceased to the document marked as
Exhibit D was secured through misrepresentation; (5) the testatrix was
not possessed of a sound and disposing memory on December 29,
1960; and (6) the testimonies of the witnesses for the petitioner,
including that of the petitioner herself, are highly incredible.
xxx xxx xxx
The oppositors do not question the authenticity of the will Exhibit D nor
the genuineness of the signature of the testatrix and of the attesting
witness appearing thereon. In synthesis, the opposition to the probate of
the said will Exhibit D is centered on the alleged lack of testamentary
capacity on the part of the decedent, and the supposed irregularities in
its execution.
On the testamentary capacity of the decedent the oppositors maintain
that the decedent was no longer in a position to execute a will on
December 29, 1960 because she was then more than 91 years old; that
she could not manage her own properties so much so that her properties
were under the management of Alfonso D. Precilla; that she was well on
the way to blindness due to glaucoma; that she was so weak that she
had to be assisted whenever she moved from place to place; and that as
certified to by the Clerk of Court of the Court of First Instance of Manila
on July 11, 1961 in Special Proceedings No. 43871 of this Court, the
decedent could hardly understand the questions pounded to her, that
she took much time to understand and answer the questions and that
most of her answers were not responsive to the questions.
xxx xxx xxx
It is noteworthy that aside from claiming that the decedent was much
advanced in age and that she suffered from weakened eyesight, the
oppositors have not imputed any infirmity which would affect the mental
capacity or soundness mind of the testatrix. "To be of sound mind, it is
not necessary that the testator be in full possession of all his reasoning
faculties, or that his mind be wholly unbroken, unimpaired, or
unshattered by disease, injury or other cause. It shall be sufficient if the
testator was able at the time of making the will know the nature of the
estate to be disposed of, the proper object of his bounty, and the
character of the testamentary act. The law presumes that ever person is

of sound mind, in the absence of proof to the contrary." (Articles 799,


800, Code).
We are not saying that because of these observations, the alleged blindness or
incapacity to read of the supposed testatrix and the testimony of Dr. Tamesis in regard
thereto may not be considered at this stage. Mention of the above circumstances is
here made only to stress the point that even the oppositors themselves are not
unanimous as regards this specific ground of opposition. As a matter of fact, even at
the appeal level in this Court, nowhere in the discussion of the ten errors assigned by
so-called testate heirs in their brief is mention made of this specific issue. It may,
therefore, be said that subjectively said testate heirs, who precisely would have the
greater probability of benefitting the disallowance of the will in controversy, because
they appear to be testamentary heirs under the prior will of 1956, which stands a good
chance of being the one probated if the 1960 will is ultimately rejected, might not have
considered said point to be factually tenable, as may be inferred from the fact that they
did not present any witness to prove alleged deficiency of the eyesight of the deceased,
nor they even adopt the evidence on this point of the other oppositors. They limited
themselves to the presentation of documentary evidence, none of which touched on the
said point. Only the intestate heirs presented two witnesses in their attempt to
substantiate this unannounced ground of their opposition, namely, Atty. Vidal Ranoa
and Dr. Tamesis, and at that, only the latter's testimony appears to have some
relevance.
It is clear, however, that whatever fault there might be in the tactics or exact position of
the testate heirs, We cannot now dodge this issue of the alleged blindness or incapacity
to read of the deceased. Even the failure of appellee's counsel to properly object to
evidence on said issue, for not being directly relevant to the grounds of opposition
alleged by appellants, or the apparent waiver of such possible objection as a
consequence of his act of cross-examining the doctor are of no consequence.
In Guevara v. Guevara, 74 Phil. 479, this Court postulated:
... . We hold that under section I of Rule 74, in relation to Rule 76, if the
decedent left a will and no debts and the heirs and legatees desire to
make an extra-judicial partition of the estate, they must first present that
will to the court for probate and divide the estate in accordance with the
will. They may not disregard the provision of the will unless those
provisions are contrary to law. Neither they may do away with the
presentation of the will to the court for probate, because such
suppression of the will is contrary to law and public policy. The law
enjoins the probate of the will and public policy requires it, because
unless the will is probated and notice thereof given to the whole world,
the right of a person to dispose of his property by will may be rendered
nugatory, as is attempted to be done in the instant case. Absent legatees
and devisees, or such of them as may have no knowledge of the will,

could be cheated of their inheritance thru the collusion of some of the


heirs who might agree to the partition of the estate among themselves to
the exclusion of others.
xxx xxx xxx
Even if the decedent left no debts and nobody raises any question as to
the authenticity and due execution of the will, none of the heirs may sue
for the partition of the estate in accordance with that will without first
securing it's allowance or probate by the court, first, because the law
expressly provides that "no will shall pass either real or personal estate
unless it is proved and allowed in the proper court"; and, second,
because the probate of a will, which is a proceeding in rem, cannot be
dispensed with and substituted by any other proceeding, judicial or
extrajudicial, without offending against public policy designed to
effectuate the testator's right to dispose of his property by will in
accordance with law and to protect the rights of the heirs and legatees
under the will thru the means provided by law, among which are the
publication and the personal notices to each and all of said heirs and
legatees. (Emphasis supplied)
So much is the concern of the law for the indispensability of probating a will that
Section 4 of Rule 75 penalizes a fine not exceeding two thousand pesos the failure of
custodian of a will to deliver the same to the court or the executor named therein, as
also the failure of the executor to present the will to the proper court for probate; and
under Section 5 of the same rule, such custodian may be detained by order of the court
until he makes the required delivery of the will. The court has to be convinced on
authenticity and due execution of the will even if its allowance is not opposed and the
rule requires in such a situation that, at least, one attesting witness must testify. (Sec. 5
Rule 76). A testament may not be disallowed just because the attesting witnesses
declare against its due execution; neither does it have to be necessarily allowed just
because all the attesting witnesses declare in favor of its legalization; what is decisive
is that the court is convinced by evidence before, it is not necessarily from the attesting
witnesses although they must testify, that the will was or was not duly executed in the
manner required by law (Section 11, Rule 72; Unson v. Abella, 43 Phil. 494; Fernandez
v. Tantoco, 49 Phil. 380; Tolentino v. Francisco, 57 Phil. 742; Ramirez v. Butte, 100 Phil.
635) because the proponent is not concluded by the testimonies of the attesting
witnesses. (Fernandez v. Tantoco, supra). Withal, the dismissal of a petition for probate
for non-appearance of petition counsel does not constitute res adjudicata (Arroyo v.
Albay, L-15814, Feb. 28, 1962, 4 SCRA 555), and the presentation for authentication of
a will does not prescribe. (Guevara v. Guevara,supra.) Irrespective, therefore, of
posture of any of the parties as regards the authenticity and due execution of the will
here in question, it is the mandate of the law that it is the evidence before the Court
and/or ought to be before it that is controlling.

In the case at bar, the evidence before Us with respect to the capacity of Doa Gliceria
to read on December 29, 1960, which appears on its face to be the date of the
controverted execution of the will in question boils down to this: on the part of the
appellee, direct evidence consisting of the testimonies of the three attesting witnesses,
Francisco Decena, Francisco Lopez and Vicente Rosales, who uniformly declared that
the testatrix read the will silently in their presence before signing the same and, in
addition, the records of court proceedings more or less attesting to her capacity to read
and some sworn certifications as well as checks signed by her; on the part of the
intestate heirs, only the testimonies of their two witnesses, Atty. Vidal Ranoa and Dr.
Jesus Tamesis, but actually, it was only the latter who gave what might be considered
as evidence related to the eyesight of Doa Gliceria. In other words, the evidence of
appellee consisted of the direct evidence of three attesting witnesses as to their actual
personal knowledge of the matter in dispute, corroborated by court records and other
documents, whereas the intestate heirs-oppositors had only the testimony of Dr.
Tamesis.
Referring now to said testimony of Dr. Tamesis, the record is rather vague as to
whether he was presented to testify as a professional expert or he was called for the
purpose of declaring from actual personal knowledge of the condition of the eyesight of
the testatrix on the date in question, December 29, 1960. Having been her
ophthalmologist and surgeon who operated on her left eye on August 8, 1960 and who
had her "under medical supervision up to 1963" (meaning since March 11, 1960), 1 it
would indeed have been more desirable, in Our view, if the doctor had testified more
directly from personal knowledge rather than on mere opinion, as he seems to have
done, on the basis of records, which in some vital instances were prepared, according
to him, by other persons. And, perhaps, there would have been less difficulty in arriving
at a safe conclusion as regards the vital fact now in controversy, if the doctor had only
been more categorical and definite, rather than somehow ambiguous and equivocal in
his testimony. Of course, We cannot wholly blame him alone; he might have waited for
questions of counsel that were never asked.
It is quite interesting and very revealing to follow step by step the testimony of the
doctor.
Testifying on notations made by him as to condition of the eyes of his patient (Doa
Gliceria) on March 11, 1960, the first day of consultation, he declared:
Q After the words eye grounds O.D. there appears certain
writings in ink above the two circles, will you kindly form the court
the significance of these and their meaning ?
A My assistant did not draw, but I saw. However, these are my
writings appearing here and I made notations that the optic nerve
manifested that senile degeneration and was pale.
Q That is with respect to which eye?

A With respect to the right eye marked by O.D. the letters


O.D.
Q And the notation on the other circle, Doctor, will you kindly
inform the court the significance?

Q On the right side of this document Exhibit 3-A under the


word tension there appears written notation or figures after the
symbol O.D. and O.S. Will you inform the court the significance
of those figures?

A On the other circle there was no drawing and my remarks


here were under the substance that open the pupil for
examination the optic nerve was also pale and there was even a
cupping of the optic nerve. Other details hardly noticeable.

A Tension O.D. with the eye refers to the pressure of the eye,
that is, whether the eye is soft, normal or hard.

Q When did you make these notations which appear to have


been made near the circles in Exhibit 3-A ?

A You want me to interpret ?

A They were made on the day of consultation to me, March


11, 1960.

A The right eye to me at that moment was normal in pressure,


and the left eye at that moment was above normal.

Q Can you inform the court, Doctor, in lay language what is


the significance of the notation that the optic nerve was pale
appearing on the circle above which you have written, the word
pale, Doctor ?

Q Does the term "pressure" have any effect on the ability of


the patient to see ?

ATTY. AQUINO
I object to this line of questioning, Your Honor, these questions
are absolutely immaterial in this case. The witness here, as he
has said, is an ophthalmologist. He is not a psychiatrist or a
neurology surgeon.

Q In lay language, Doctor, what is the significance of this


figure appearing under tension ?
Q Yes.

A In fact if the pressure is above normal indicates another


disease.
Q Now, in this particular document what other disease, if any,
is indicated by your notation, Doctor ?
A I have diagnosed this aside from the cataract as a possible
case of glaucoma.

COURT

Q What is the effect of glaucoma on the patient?

You just enter a general and continuing objection.

A Glaucoma invariably possesses blindness.

ATTY. AQUINO

Q And at that time you examined the patient will you please
inform the court what was the stage of that disease the patient
examined by you ?

I would like to register a general and continuing objection to


these questions.
COURT
You may proceed.
Q What does that mean, that annotation? You are asked,
Doctor, to explain ...

A Taking into consideration the findings in the eye grounds as


manifested in the notation of this big circle and there were
corroborative evidences that she has been suffering from
glaucoma that time.
Q Approximately how long a time ?

A This "pale disc senile degenerates retina", it simply meant


that the vital organs of the eye such as the retina and optic nerve
had degenerated and, therefore, function is expected or might be
below normal.

A at least a year from the changes mentioned.

ATTY. ORDOEZ

COURT

ATTY. AQUINO
Already answered.
Let him answer.

WITNESS

Q Referring to Exhibit B-B?

A Blindness is the ultimate result.

A Exhibit 3-B, here ...

COURT

ATTY. ORDOEZ

Q You refer only to the left eye affected in this case?

WITNESS:

A Yes.

A Under date of August 30, 1960, is the record of refraction,


that is setting of glass by myself which showed that the right eye
with my prescription of glasses had a vision of 20 over 60 (20/60)
and for the left eye with her correction 20 over 300 (20/300).

(pp. 14-18, t.s.n.. March 23, 1966)


So far, as may be seen, the doctor was referring principally to the left eye. As to her
right eye, his categorical declaration was:
Q Even if the left eye has glaucoma that will permit the sight of
the other eye ?
A Yes, sir.
ATTY. ORDOEZ
Q From this record is there any indication of the condition of
the right eye at the time You examined the patient?
A From the meager data set down by my assistant, however,
after examining her, the right eye would be seen to be operated
for cataract, that her optic nerve and retina showed regeneration,
that she was wearing lens with the grade that was noted by my
assistant. 2 (p. 19 id.)

ATTY. ORDOEZ
Q In layman's language, Doctor, what is the significance of
that notation that the right eye had a degree of 20 over 60
(20/60).
A It meant that the eye at least would be able to recognize
objects or persons at a minimum distance of twenty feet.
Q But would that grade enable the patient to read print ?
A Apparently that is only a record for distance vision, distance
sight, not for near. (pp. 20-21)

At this juncture, two points must be elucidated. In the first place, We have re-examined
Exhibit 3-B very carefully, and We are fully convinced that movant's contention that no
such annotation of "20/60" appears in the entry therein for August 30, 1960 is well
This was followed by references set down by his assistant which he called "farfetched
taken because what is clearly visible therein not 20/60 but 20/4 or 20/40 is written
data":
following the prescription of OD+15.25(L)-1.25x90 for right eye and above the
Q From the entries appearing in this record, would you be able prescription for the left eye reading OS+14.25 (Lic)-300x90 followed by the visual acuity
of 20/300.
to form an opinion as to the condition of the right eye of the
patient ?
In the second place, when the witness was asked, "But would that grade enable the
patient to read print?", which certainly was the pivotal and most important question,
A As to vision, from the farfetched data here there is no
answer was rather obscure and susceptible of misinterpretation. Instead of giving a
evidence on the actual visual capacity of that right eye my
direct, definite and categorical, reply, he testified thus: "Apparently that is only a record
assistant had not noted it down. (pp. 19-20, id.)
for distance vision, for distant sight, not for near." While the reference to "distance
Moving afterwards to Exhibit 3-B, because of the insistence of counsel to get more
vision, for distant sight" is apparently correct in the parlance of ophthalmologists, the
specific information as to the "condition of the vision of the patient as to the right eye",
last phrase "not for near" suffers from, at least, incompleteness in the sense that, while
he declared:
again it appears to be the ophthalmological expression, he thought his words convey
Q But is there anything herein the entry appearing in the other leaves the Court guessing as to whether or not it has same connotation as when one
documents Exhibits 3-B, 3-C, and 3-D from which you could
says "the patient can read print", for, even casual acquaintance with ophthalmological
inform the court as to the condition of the vision of the patient as
symbols teaches that the notation 20/60 by its alone, (assuming that such is the
to the right eye?
notation in Exhibit Balthough it is not, as already stated above) represents visual acuity
for far or distance and is not exactly the symbols which indicates the capacity of the
A On the underside?
patient to read print.

In other words, when the good doctor said "not for near", technically, he did not refer to
the incapacity of the patient to read print. By this statement, he simply meant that the
notation "20 over 60(20/60)", is not the usual manner of indicating whether or not the
patient can read, for, indeed as all ophthalmologists know said notation refers to the
visual acuity for far or distance vision measured on the Snellen chart. To be more
specific, said notation is a clinical symbol referring to the size of the letters on the
Snellen chart that a patient can recognize at a distance of 20 feet from the chart. The
sizes of these letters on the chart vary and the measure of visual acuity for distance
vision, not for reading, is determined according to the size of the letters on the chart
that the patient can read from a distance of 20 feet. So, "20/60" means that the size of
the letters on the Snellen chart that the patient can read from a distance of 20 feet is
that which according to the Snellen test, a person with normal vision can recognize
even at a distance of 60 feet. When an ophthalmologist may venture offhand an opinion
on the matter of the reading capacity of a patient on the basis of such notation of 20/60,
it would be somewhat speculative for him to specify what type of print such patient can
read. Indeed, according to the work or ocular diseases referred to in appellee's brief:
4. Near vision. The principles of visual acuity for distance apply for
near, remembering the added factor of accommodation. In young
individuals or with the proper addition of plus lens in presbyopes, visual
acuity for near can be determined by suitable charts usually at 14 in. The
notations of the letter size unfortunately vary, but the equivalents are
given in Table 1. Lebensohn's chart (1) has several practical virtues,
including the uses of letter, numbers and illiterate symbols, correlation of
various designations of letter size with comparable Snellen distance
acquities, a bar of music for testing intermediate distance, the two line
and duochrome tests, an attached chain measuring 14 in. and a hard
plastic surface which is easily cleaned. A normal individual can usually
detect 3 or 4 point type on testing for near vision, but larger print is
required for reading comfortably. For example, telephone directories (6
point require) 4 point acquity, newsprint (8 point) requires 5 point
magazines and books (9-10 point) require 6 point, children's textbooks
(12-18 point) requires 1-12 point and sightsaving texts (24 point) require
at least 16 point acquity. Although visual acuity for distance cannot be
related exactly to visual acquity near considering only the visual angle,
an individual with corrected vision of less "had 20/40 usually begins to
have difficulty reading ordinary newsprint." (W.F. Hughes, Office
Management of Ocular Diseases, p. 17)
and the Table 1 mentioned gives the corresponding types of print for different acquities
as follows:
(The following page is the scanned copy of p. 18 of the work where
Table 1 is found.).

VISION
LOSS OF CENTRAL VISUAL ACUITY. The usual method of
recordering vision as a fraction does not indicate the true percentage of
visual efficiency; e.g., 20/40 vision is not 50% efficient visual acuity. In
1925, a standard method of relating visual acuity of a single eye to visual
efficiency (2) was approved by the Section on Ophthalmology of the
American Medical Association, a condensation of which is given in Table
1. The percentage vision in one eye can be calculated, utilizing both rear
and distance visual acuities, according to the following method, which
weights the near vision twice.
TABLE 1. ESTIMATION OF PERCENTAGE LOSS OF CENTRAL
VISUAL ACUITY IN ONE EYE
Visual Acuity

Visual
Acuity for
Near,
Distance
in

% Visual

%
Visual

Unittent

Snellen

Meters

jacget

Point

Efficiency

Loss

20/20

14/14

0.35

100

20/25

14/18

0.44

96

20/30

14/21

0.59

...

91

20/40

14/28

0.71

84

16

20/50

14/35

0.88

77

23

20/60

14/42

1.08

...

9+

70

30

20/70

14/49

1.30

10

64

36

20/80

14/56

...

12

59

41

20/100

14/70

1.76

11

14

49

51

20/160

14/112

...

14

22

29

71

20/200

14/140

3.53

...

...

20

80

20/400

14/280

7.06

...

...

97

20/560

14/560

14.12

...

...

0.1

99.9

for Distance

calculated utilizing both near and distance visual acuities according to the following
method, which weights the vision twice.

( % acuity distance) + (2 X % acuity for near)


________________________________ = % visual for 1 eye
3
For example:
Visual Acuity = 20/100 for distance and 14/140 for
near
% Visual Acuity = 49% for distance and 20% for near
Combined % visual acuity for this eyes = 49 + 2 x 20
= 30%
3
LOSS OF VISUAL FIELD. AN estimation of the percentage efficiency
of visual field is determined according to the following method. The
peripheral visual field (isopter) is determined with a 1 white test object
(e.g., approximately a 6 mm. object at 330 mm. distance) for each of the
eight 45 meridians. A 100% field of
PHOTOSTATE OF PAGE -18W. F. HUGHES, Office Management of Ocular Diseases
WILLIAM F. HUGHES, JR., M.D.
Professor and Head, Department of Ophthalmology, University of Illinois
College of Medicine; Ophthalmologist in Chief, Illinois Eye and Ear
Infirmary and Attending Ophthalmologist, Presbyterian Hospital,
Chicago.
Copyright 1953 by The Yearbook Publishers Inc. 200 East Illinois Street,
Chicago
To reiterate, Dr. Tamesis did not give any opinion, in his above answer, as to the
capacity of his patient's right eye to read any kind of print; rather, what he must be
understood to have meant is that the annotation of 20/60 referred to in the question of
counsel was a record of visual acuity for distance which is not a definite indicium of her
capacity to read print. As may be seen in the above table, it is by reference to the
Jaeger figures that the type or size of print, according to standard points, that a patient
can read is usually indicated. In the reading chart commonly used in clinics of
ophthalmologists and optometrists, the symbol used is J-1, J-2, etc. In the absence of
any counter-reference in any of the pleadings or memoranda filed With this Court by
appellants, We consider the information contained in the above reference book, which
We have checked, to be sufficiently reliable insofar as the technical aspect of the issue
before Us is concerned.
The reference made by the witness to his prescription for lenses is even more
relevantly revealing. Here is positive proof of the utter lack of evidence in his testimony

on the reading capacity of his patient. As may be readily noticed, the prescription given
is only for distant vision. Putting aside for a while any reference to the left eye which
had been operated for glaucoma only three weeks before, hence normally it would not
have been tested for reading, it is obvious from the records referred to by the witness
that no prescription was given by him for reading even the right eye, the prescription
pointed to being obviously for distance vision only as indicated precisely by the visual
acuity noted therein. Anyone would know from common experience that the giving of a
prescription for distance vision does not necessarily mean that the patient can be given
a prescription for reading.
Counsel for oppositors would make much of the reference made by Dr. Tamesis to the
so-called "counting fingers" method to indicate the extent of Doa Gliceria's vision, but
the record of the doctor's testimony on this point is clear to the effect that he was
referring to the left eye only of the patient, not to her right eye. This is what is in the
record:
Q Will you inform the court those entries which you made after
the operation in the Lourdes Hospital?
A My record in my chart shows entries with respect to the postoperative care of the patient until she was finally given glasses
on August 30, 1960, and that she had come to my clinic on
various occasions for follow-up and other consultations.
Q During your conferences with the patient, Doctor, and after the
operation did you have any occasion to propound questions to
her relating to objects which were presented to her for
identification?
A Well, naturally in the testing of sight especially with respect to
fitting her glasses, we had to rely on what she would answer.
Q Do you have any procedure or test to indicate the extent of
vision which utilizes counting of fingers of the patient?
A We utilize counting of fingers as one of the standard
procedures when the vision is actually very poor.
Q And did you employ that method on this patient?
A I believe I did.
Q Will you kindly inform the court where you entered that
procedure of counting fingers?
A One such entry is here dated August 23.
ATTY. ORDOEZ
Witness pointing to entry on Exhibit 3-B.

WITNESS

A Yes, sir.

A Referring to division of newly operated left eye and in spite of


the glasses at that time for reflection, her vision was only
counting fingers at five feet.

Q What was her principal complaint?

ATTY. ORDOEZ

Q She told you that personally?

May I request to read previous to the last answer of the witness.

A Yes, sir.

STENOGRAPHER

Q Was she walking when she went to your clinic?

We utilize counting of fingers as one of the standard procedures


when the vision is actually very poor.

A Yes, sir.

A She was dizzy. The fact that was her main complaint that she
was dizzy and she had practically poor vision of the left eye.

ATTY. ORDOEZ

Then, when the witness was asked how successful was the operation on his patient's
left eye on August 8, 1960.

Q What do you actually mean by the phrase "actually very poor"


which justified your method of counting fingers?

he explained:

A It is manifested by the inability of the patient or the person to


see clearly with curve glasses, and when we see objectively
changes on the eye which would block the passage of the light
rays towards the retina.
Thus, ended the doctor's direct testimony, and so far already explained, it cannot be
said therefrom that there is adequate basis for the conclusion that the right eye of Doa
Gliceria could or could not read on August 30, 1060. Indeed, and more importantly, it
should be noted that witness much less made any reference, as he could not have had
any safe basis to do so, to the condition of eyes of his patient on the very date in issue,
December 29, 1960.
Coming now to the cross-examination, the impression easily gets from the tendency
thereof is that counsel was more interested in extracting facts from the witness
indicative of the capacity of the patient's right eye to rather than to directly belie the
points attempted to be proven by oppositors during the direct examination, vague and
equivocal tho' they were. For example:
Q Before you performed the operation on Da. Gliceria A. del
Rosario, she went to your clinic?
A Yes, sir, for consultation.
Q And you talked with her?
A Yes, sir.
Q You propounded questions to her?
A Yes, sir.
Q And she answered your questions?

Q The operation performed by you was successful?


A Technically successful.
Q In fact according to this Exhibit 4 Narciso, the record of the
hospital, there is a statement here "condition on discharge" with
the handwritten answer "fair"?
A Yes, sir.
Q Can you elaborate on the meaning of the word "fair" here?
A Well, this was written by the resident on duty in the hospital,
and that signified only that the patient was physically healthy as
far as ordinary activities are concerned. She can walk.
Q And because her condition was fair you consented to have her
discharged from the hospital?
A Yes, sir.
Perhaps, it was because at this stage, the cross-examiner understandably avoided for
obvious reasons, asking directly the opponent's witness about the patient's capacity to
read that nothing was mentioned by the doctor on that point, but nonetheless, he later
declared more relevantly thus:
Q After she was discharged from the hospital you prescribed
lenses for her, or glasses?
A After her discharge from the hospital she was coming to my
clinic for further examination and then sometimes later glasses
were prescribed.
Q If I remember correctly, according to the test of August 30,
1960?

A Because here it is marked Rx prescription.


Q And the glasses prescribed by you enabled her to read,
Doctor?
A As far as my record is concerned, with the glasses for the left
eye which I prescribed the eye which I operated she could
see only forms but not read. That is on the left eye.
Q What about the right eye?
A The same, although the vision on the right eye is even better
than the left eye.
Q After that did you Prescribe other glasses?
A No other glasses were prescribed, but subsequent visits were
made to my clinic because we gave her some vitamins and drugs
to improve the vision of the eyes.
Of course, the looseness or ambiguity of this testimony is manifest. As can readily be
noted, although the doctor was testifying on a most crucial point, which, under the
circumstances, it may be presumed he should have had personal knowledge of as
Doa Gliceria's attending physician, hence he could be considered as giving direct
evidence rather than merely the opinion of one professionally engaged as an expert
witness, he based his answers neither on present nor on past recollection but simply on
"as far as my record is concerned." And then his answer to the question, "What about
the right eye?" saying "the same, although the vision on the right eye is even better
than the left eye." is as devoid of any concrete information on the point at issue as his
later declaration as follows:
Q In this certificate you stated, and I quote: "Patient was provided
with aphakic lens and she had been under medical supervision
up to 1963 with apparently good condition"?
A Yes.
Q When you said that she had apparently good vision you mean
that she was able to read?
A No, not necessarily, only able to go around, take care of
herself, and see. This I can tell you, this report was made on pure
recollections and I recall she was using her glasses although I
recall also that we have to give her medicines to improve her
vision, some medicines to improve her identification some more.
Q Will you kindly inform the Honorable Court what you mean by
aphakic glasses?
A The cataract glasses.

Q Those lenses were used by the patients who had a cataract


operation?
A Yes.
It is immediately noteworthy that to the direct question whether or not "apparently good
vision" meant that his patient "was able to read", the answer "No, not necessarily etc.,"
and, at that, based, according to the doctor himself on his "report (which) was made on
pure recollections, etc." is hardly the clear and categorical answer expected of an
attending physician having personal knowledge of the condition of his patient. Withal,
the aphakic glasses the doctor mentioned in the above testimony may not have been
prescribed by him but by Dr. Ocampo who had performed the operation on the right
eye.
In any event, it is generally known that almost normal vision is usually restored after a
successful operation for cataract such as that which Doa Gliceria underwent in 1956:
There are numerous operations now developed which are relatively
simple and which are quite safe in the majority of instances. Moreover,
good vision follow in 97% of the operations for cataract. After cataract is
removed, the person wears what are known as cataract glasses. These
are made so that they help in forming the image properly on the retina.
By use of a hormone called chymotrypsin the tissues surrounding the
lens may be softened and this has greatly facilitated the operation for
cataract.
The person who has been unable to see for some time because of the
development of clouding, who has been unable to play golf or get about,
and who then recovers his eyesight by a simple operation that any
competent specialist in diseases of the eye can perform, is one of the
most appreciative of all people benefited by the modern medical and
surgical science. When the cataract is removed, the result is like
defrosting a window or raising the window shade: the light comes in
without obstruction, and the individual is able to see. (p. 940, Illustrated
Medical and Health Encyclopedia, Edited by Morris Fishbein M.D., Vols.
3 and 4, 1966 ed.)
In other words, and as may be confirmed from anyone who has undergone an
operation for cataract, with the use of aphakic glasses, vision for reading as well as for
far is generally nearly as normal as anybody else who uses ordinary corrective eye
glasses or even one who does not have defective eyes. Testifying further, the doctor
declared:
Q She was wearing eyeglasses?
A Yes, sir, she had to wear eyeglasses.

Q What about the vision in the right eye, was that


corrected by the glasses?

was uneventful and she was able to be discharged


without any complication.

A Yes, with the new prescription which I issued on August


30, 1960. It is in the clinical record.

lest it be misunderstood, refers to the left eye he operated on and not to the right eye of
Doa Gliceria.

Q The vision in the right eye was corrected?

In any event, in such state of the evidence before Us ambiguous, indefinite,


equivocal and even misleading, as to a very vital issue of fact and considering the
importance of the probate of a will, as explained above, not to mention the substantial
value of the estate herein involved, We are persuaded, in the light of the points raised
by appellee in her motion for reconsideration, that the best interests of justice will be
better served by reopening this case.

A Yes, that is the vision for distant objects.


Here again, the answer of the good doctor were as safe as any equivocal testimony
can be. Obviously, he was referring to his prescription noted on Exhibit 3-B Narciso
which is indicated thereon as having been given on August 30, 1960 and which earlier
he said was based on the visual acuity of his patient of 20/60 (which have already seen
is actually 20/4 or 20/40). Evidently, therefore, the correction the doctor was talking
about must have been necessarily with reference to what is shown by the entry for
September 22, 1960 on the same Exhibit 3-B Narciso indicating that, indeed, the vision
of his patient's right eye had improved its visual acuity to 20/3 or 20/30, approximating
almost the normal acuity of 20/20. Considering again that any ophthalmologist knows
that 20/60 means that a person with such visual acuity can distinguish in the Snellen
chart at a distance of only 20 feet letters of the size or type that, according to the same
chart, a person with normal vision can see at 60 feet, naturally, 20/30 means that the
vision had greatly improved, such that smaller letters in the chart which can be read by
the normal eye at 30 feet distance could already be read by the patient at 20 feet. But
instead of being more categorical about what such improvement signifies, the good
doctor merely made reference to "the vision for distant objects", leaving unsaid what
such notations of the visual acuity for distance of his patient indicate regarding her
capacity to read ordinary print. As We have already discussed above, ophthalmological
parlance, the capacity to read is indicated by the Jaeger symbols, and since the doctor
limited himself only to the entries on the record regarding visual acuity for distance on
the Snellen chart, his testimony may be quite accurate but certainly, it is incomplete as
to the very point at issue. Why he did not care to testify on present or past recollection
and why he did not elucidate directly and specifically by the use of unequivocal words
on whether or not his patient could read ordinary print on any particular date are
incomprehensively unexplained.

The very existence of a purported testament is in itself prima facie proof that the
supposed testator has willed that his estate be distributed in the manner therein
provided, and it is incumbent upon the state that, if legally tenable, such desire be given
full effect independent of the attitude of the parties affected thereby. No less than public
policy requires adherence to this view. While, as indicated earlier, no will may be
probated unless the court is convinced of its authenticity and due execution on the
basis of the evidence before it, it is as important a matter of public interest that a
purported will is not denied legalization on dubious grounds. Otherwise, the very
institution of testamentary succession will be shaken to its foundation, for even if a will
has been duly executed in fact, whether or not it will be probated would have to depend
largely on the attitude of those interested in his estate. All they have to do is control the
available evidence make it good or bad, depending on whether they like to follow the
wishes of the testator or they don't. We hold that the underlying principles of the law on
probate dictate that subjective considerations should not be the determinative factor as
to whether a will should be allowed or not. We are of the considered view that in
probate cases, the courts should disregard the ordinary rules of procedure and of
evidence to the end that nothing less than the evidence of which the matter is
susceptible should be inquired to be presented to it before a document purport to be
legalized as a will is to be probated or to be denied probate. Whenever the court has
reasonable doubt as the material facts concerning the proper execution of a will, it
should take the initiative of requiring the parties to take the appropriate step to erase
such doubt. Unlike in adversary proceedings wherein the rule is that the party whose
Finally, his testimony to the following effect:
favor the evidence preponderates should be awarded the palm of victory, probate
proceedings are instituted that the state may see to it that a testator's will, if duly
Q When you said that the operation was successful and
that you provided glasses for the patient, according to you executed, may be legalized, and the courts, as agents of the state, should not stop
short of being fully convincing that there has been or has not been compliance with the
she would be able to see?
requirements of the formalities required by law in the execution of the testament, so
A She would have been able to see very clearly if her
much so that if the opposing parties, for any reason, should leave material evidence
retina and optic nerve were also normal. But the operation
unpresented or unclarified, and from the circumstances, the court has reason to believe
by itself was successful in the sense that the cataract was
that better evidence exists, the court should try to remedy the situation by calling for
successfully removed and her post-operative recovery
more complete and clearer evidence; and so it is that when it appears to the appellate

court that the evidence is incomplete or insufficient to either establish or disprove any
essential fact in an appealed probate proceeding, it is not only within its authority, but
more of an obligation on its part to return the case for further proceedings, that the
evidence may be completed and in order that the court may be provided with adequate
and concrete basis for deciding the issues of authenticity and due execution of the will
before it with as much degree of certainty as all the attendant circumstances will permit.
Thus, in the case at bar, it is really of no moment that, as contended by appellants, the
alternative motion for new trial of appellee is not based on the ground of newly
discovered evidence within the contemplation of Section 1 of Rule 53, for, aside from
what has just been said of the nature of probate proceedings and the duties of the
courts in relation to the evidence of the parties therein, Section 3 of Rule 51 expressly
and definitely empowers this Court to order a new trial for reasons other than newly
discovered evidence, such as when the court feels that the reception of additional
evidence on some points is needed in order to clear doubts engendered by the
evidence already in the record. Indeed, when We consider again the nature of probate
proceedings and the imperativeness of attaining the most approximate certainly
possible in determining the authenticity and due execution of a purported testament,
this power of the court to order, upon its own motion, the reception of additional
evidence becomes even more handy and appropriate. Thus, in Testate Estate
of Jovellana v. Jovellana, 106 Phil. 1073, the Court could not ascertain from the meager
evidence on record whether or not the testator knew the Spanish language in which the
will therein in question was written, but instead of denying probate for insufficiency of
the evidence of the petitioner, as should ordinarily be done when the plaintiff in a civil
action fails to show preponderance of the evidence in his favor, the Court ordered the
remand of the case to the trial court in order "to afford the parties an opportunity to
present evidence, if they so desire, on this controversial issue." The Court held:
But petitioner-appellees insist in their brief that the burden is on the
oppositors to allege and prove that the testator did not know the Spanish
language in the face of the legal presumptions that "the law has been
obeyed," "that a will executed in the Philippines must be presumed to
have been executed in conformity with the laws of the Philippines" and
"that things have happened in accordance with the ordinary course of
nature and the ordinary habits of life," concluding that it would certainly
be contrary to the ordinary habits of life for a person to execute his will in
a language unknown to him. This, we believe, is, to use a colloquial
term, begging the question. If the argument of counsel is correct, then
every unopposed will may be probated upon its mere presentation in
court, without need of producing evidence regarding its execution.
Counsel' statement is its own refutation.
We find, however, in the record some indicia, although in sufficient to
give rise to the presumption, that the testator might, in fact, have known

the Spanish language. In oppositors own Exhibit 3 (a letter admittedly


written by the testator) appear the salutation "Querido Primo" and the
compliment ending "Su primo" which are Spanish terms. Having found
that all the formal requisites for the validity of the will have been
satisfactorily established, except the language requirement, we deem it
in the interest of justice to afford the parties an opportunity to present
evidence, if they so desire, on this controverted issue.
Wherefore, let the records of this case be remanded to the court of origin
for further proceedings as above indicated, without costs. It is so
ordered.
Echoing the foregoing, in the case at bar, We hold, after a careful review of the entire
record and upon consideration of the points raised by counsel in the motion for
reconsideration and/or new trial now before Us, that it is but consistent with the
fundamental principles governing probate proceedings that there be more definite, clear
and unequivocal evidence, which We believe exists, as to whether or not Doa Gliceria
could read print on December 29, 1960. Without in any manner passing on their
probative weight, the Court feels that the affidavits of Dr. Gemeniano de Ocampo and
Dr. Roberto N. Sunga attached to the present motion of appellee, and against which
nothing technically authoritative has been offered by appellants in their oppositions,
sufficiently indicate that better evidence is available upon which the Court can decide
the vital issue before it with a more satisfactory degree of certainty. As already
explained, the direct evidence consisting of the unanimous testimonies of the three
attesting witnesses to the effect that the testatrix read the will, Exhibit D, silently in the
presence of all three of them stands uncontradicted by any direct evidence of the
oppositors. Only the intestate heirs have attempted to overthrow the weight of this
evidence of the proponents by presenting Dr. Tamesis, but, as already demonstrated
above, the good doctor never referred to any personal knowledge of his that Doa
Gliceria could or could not have read ordinary print on August 30, 1960. and much less
did he give any direct evidence that she could not have read Exhibit D, the testament in
question, on December 29, 1960, there being nothing either in Exhibits 3-B and 3-C
Narciso or in his whole testimony indicating that he had relevantly examined Doa
Gliceria for such purpose anywhere near the latter date. As a whole, as already
demonstrated above, his testimony on the facts most vital and material to the specific
issue on hand leaves many important and decisive questions unanswered. On the
other hand, while indeed some circumstances extant on the face of the document in
controversy to lend significance to the testimony of Dr. Tamesis, it cannot be denied
that there are also in the record circumstances tending to show that it is not improbable
that Doa Gliceria could have read the said document on the day of its execution as
testified to by the attesting witnesses, such as, mainly the complete silence of the
testate heirs regarding this particular issue before Our decision of April 30, 1970 and, of
course, the exhibits presented by appellee (Exhibits C to E-13, G and the two G-3's as

well as Exhibit I) which, taken together with the testimonies of the witnesses who
testified on them, are not entirely without probative weight in regard to the point in
dispute. What we are saying is that, all in all, in the state of the evidence before Us, the
Court does not find itself in a position to determine with fairness and justice to all
concerned the pivotal question with the condition of her eyes on December 29,
1960, could Doa Gliceria have read the will, Exhibit D, before she signed the same on
that day? The evidence in the record is short of what the nature of probate proceedings
require. We have, therefore no other recourse than to remand this case to the court a
quo in order that the parties may present additional evidence, more definite,
unequivocal and convincing on said point in controversy.
WHEREFORE, the decision in this case of April 30, 1970 is hereby set aside, and this
case is ordered remanded to the court a quo for the purposes above indicated, after
which said court may render the appropriate decision. No costs.

G.R. No. L-41947

January 16, 1936

In re Will of the deceased Silvestra Baron.


VIVENCIO CUYUGAN, petitioner-appellant, vs. FAUSTINA BARON and GUILLERMO
BARON, oppositors-appellees.
Pedro Abad Santos, Quirino Abad Santos, Francisco M. Ramos, and Aurelio Pineda for
appellant.
Arturo Joven for appellee Guillermo Baron.
Vicente J. Francisco and Jesus E. Blanco for appellee Faustina Baron.
BUTTE, J.:
This is an appeal from a judgment of the Court of First Instance of Pampanga denying
all the petition of Vivencio Cuyugan for the probate of the will of Silvestra Baron.
The petition which was filed on February 1, 1933, recites among other things that
Silvestra Baron died on January 30, 1933. The death certificate recites that she was
eighty-six years of age and died of heart failure. The petition further recites that she left
an estate exceeding in value the sum of P80,000 which she disposed of by will dated
December 17, 1932, that she died single without forced heirs.
The will appointed Vivencio Cuyugan, her nephew, as executor and contains the
following paragraphs which dispose of her estate:
Que despues de pagados todos los gastos quese han de ocasioner desde que
me caiga enferma hasta el entiero de mi cadaver, los bienes y propiedades que
he de dejar se repartiran buenamente y en partes iguales mis hermanos
Ilamados Guillermo Baron, con exception de todo el dinero en metalico y mi
casa de materiales fuertes construida en el barrio del Pilar, San Fernando,
Pampanga que actualmente habita mi hermano Guillermo Baron, porque estos
los doy de una manera absoluta como herencia de mi sobrino Vivencio
Cuyugan.
Que a la muerte de mis hermanos Guillermo y Faustina Baron, todos los
terrenos que en virtud de este test tamento les dejo en herencia, los doy
herencia a mi so brino VIVENCIO CUYUGAN, por lo que, encargo y prohibo a
mis citados hermanos Guillermo y Faustina Baron, que graven o pongan
cualquiera clase de obligacion sobre los bienes que les dejo en herencia.
The original of this will is signed "Silestra On" and the copy is signed "Silestra Baron"
(t.s.n. pp. 170, 171). Both copies are written in the Pampanga dialect and consist of
one sheet and are witnessed in due form by Vicente David, Valeriano Silva and
Zacarias Nuguid (known to the testator).
The petition for probate recites:
9. That on the date of the execution of said will, that is to say, on December 17,
1932, the said testatrix was about 80 years old, more or less, and was found

and disposing mind, and not acting under duress, menace, fraud, or undue
influence, and was in every respect competent to dispose of her estate by will.
The amended oppositions of Guillermo Baron, brother of the deceased, and Faustina
Baron, sister of the deceased, allege in substance first, that at the time of the execution
of the alleged will, Silvestra Baron was mentally and physically incapacitated for the
execution of a will; and, second, that her signature and alleged consent to the said will
was obtained and the attorney who prepared the document and the witnesses who
affixed their signatures thereto.
Upon the issues thus drawn by the pleadings of the judge of the Court of First Instance,
after and extended trial and a full consideration of the evidence, came to the following
conclusion:
Opinamos que influyeron indebidamente e impropia mente en la voluntad ya
debilitada de doa Silvestra Baron por su avanzada edad la presencia de sus
sobrinos Vivencio Cuyugan y Regino Cuyugan durante el otorgamiento del
Exhibit A; la ausencia de Faustina Baron impedida de presenciarlo por algunos
soldados de la Constabularia y el Jefe de Policia Municipal, Zacarias Nuguid; la
oposicion de Regino Cuyugan a que ella firmase el documento preparado por el
abogado Narciso declarando que no habia otorgado testamento el dia anterior a
su translado forzoso a San Fernando para que no se hisciese firmar documento
analogo y la presencia del cabo Morales y del algunos otros soldados, no
solamente cuando se otorgo el testamento, sino cuando ella fue transladada de
casa contra su voluntad y cuando se le hizo firmar el Exhibit 10, y por lo tanto,
que ella no gozo de una completa libertad para disponer de sus bienes en
testamento, o con pleno conocimiento del alcance de su contendido. Solo asi
se explica el que ella haya dejado toda la propiedad de sus bienes a sus
sobrinos, con quienes habia estado en pleito, con pretericion de sus hermanos,
especialmente de la opositora Faustina Baron, con quien habia conviviendo
durante 40 aos . . . .
Sentadas las premisas de hecho y de derecho que an teceden, el Juzgado no
puede menos de llegar a la con ser legalizado como el testamento y ultima
voluntad de la finada Silvestra Baron. Cuando existen pruebas suficien tes para
concencer al Juzgado de que se ha ejercido in fluencia indebida en el animo de
la testadora y que como resultado de dicha influencia indebida esta ha otorgado
el testamento de la voluntad de la supuesta testadora sino de los que sobre ella
ejercieron la influencia indebida.
An instrument purporting to be a will executed and witnessed in accordance with the
formalities required by the statute is entitled to the presumption of regularity. But the
burden of the evidence passed to the proponent when the oppositors submit credible
evidence tending to show that the supposed testator did not possess testamentary
capacity at the time or that the document was not the free and voluntary expression of

the alleged testator or that the will, for any other reason, is void in law. The finding that
the will was executed under due influence or by the fraud of another presupposes
testamentary capacity. In the present case the learned trial judge refused the probate of
the alleged will on the ground that it was executed under the due influence of other
persons and we think the record warrants his findings in this respect. The trial court
also made findings of fact tending to show actual lack of testamentary capacity of
Silvestra Baron and we have preferred to base our conclusion on that finding. The
testamentary capacity of Silvestra Baron at the time she executed the said purported
will.

P. El seor Quirino Abad Santos le ha dicho algo a ella? R. Nada. No he


oido.

The evidence shows that the same morning when Silvestra Baron signed the alleged
will she suffered a physical collapse of such a serious nature that a physician and a
nurse were immediately called in. By reason of her advanced age and the gravity of her
illness, she was unable to do anything for herself. Her grandniece, Epifania Sampang,
who reached the house about one hour or so after the old lady's collapse, telephoned a
message to Vivencio Cuyugan at San Fernando, some fourteen kilometers distant, that
Silvestra had an attack and was in a serious condition and requested that a doctor be
sent immediately, Doctor Teopaco and a nurse arrived at about ten o'clock and treated
the patient with a plaster on her back and ice packs over her heart and the doctor gave
her a hypodermic injection in the arm. As the doctor and the nurse were leaving,
Vivencio Cuyugan, with an attorney and three witnesses, entered the house prepared
to obtain the will of Silvestra Baron. Neither the doctor nor the nurse were presented as
witnesses by the proponent. Epifania Sampang, admittedly an intelligent young woman,
who was the first to reach Silvestra Baron and remained throughout the morning
attended to her, testified that when she reached the house she found her grandaunt
lying in bed, very pale and unconscious; that she called to her but she did not answer
and only groaned; that her mouth was twisted and her lower lip swollen. She went out
to call a doctor but all the doctors in Magalang were out whereupon she telephoned as
stated to San Fernando for a doctor.

P. Cuando el senor Silva termino de leer el testamento, dijo algo la vieja? R.


No he oido que dijera algo.

The subscribing witnesses stated that it was their belief that Silvestra understood the
alleged will which she signed, but all of them admitted that although they were in her
house about two hours not one of them exchanged a single word of conversation with
Silvestra. The subscribing witness Zacarias Nuguid testified in part as follows:
P. Desde que los tres abogados Abad Santos, Silva y David y usted y Vivencio
Cuyugan se acercaron a la cama de la finada, hasta que tanto ella como
usteded firmaron el testamento, ha pronunciado ella alguna palabra? ha
dicho ella algo o no? R. No recuerdo.
P. Pero, por lo que usted recuerda, ha dicho ella algo o no ha dicho nada?
R. No recuerdo.
P. Usted ha dicho algo a ella? R. Nada.

P. Los otros abogados Silva y David le han dicho algo ? R. No he oido.


P. Ella ha dicho algo a cualquiera de osos tres o a los abogados? R. No he
oido que dijera algo.
P. Si ella hubiese dicho algo a los abogados, asi como los abogados hubieran
hablado a ella, usted hubiera oido porque usted estaba cerca, no es verdad?
R. Si seor, hubiera podido oir.

There is no evidence that Silvestra Baron took any active part in the preparation of the
alleged will except that when she was asked if she wished to include her sister Faustina
in the will she said "Yes" in Pampanga. There is no affirmative evidence that she
understood the document when it was read to her. The person who read the will to her
testified as follows:
R. Despues de leido el testamento, tuve que entregarlo a doa Silvestra, y lo
miro algun rato.
P. Y ella, efectivamente, cogio el testamento de manos de usted? R. Lo
entregue a sus manos.
P. Y ella lo cogio con sus manos? R. Si seor.
P. Y lo tuvo en sus manos leyendo, mirando? R. Mirandolo asi.
P. Pero, no lo leia? R. Lo estuvo mirando por mucho tiempo asi.
Standing at her bedside was the attorney with three witnesses and the chief beneficiary,
Vivencio Cuyugan, and yet so far as this record shows, not a word was exchanged
between any of them and the suffering old woman. We don't know what drug the doctor
administered but it is clear to us from the evidence that in her dazed physical and
mental condition she had no adequate understanding of what she was doing at that
time. She could not even sign her name to the original will properly or correctly, and
when this defect was noted by one of the astute subscribing witnesses, he suggested
that they have her sign another copy (t.s.n. page 109) which was done.
She never saw the alleged will at any time again prior to her death which occurred
forty-four days later. It was immediately taken away by an attorney who kept it in his
possession alleging that she had instructed him to keep it secret. There is, however,
credible evidence in the record that before her death she had denied to several persons
that she made any will.
This belief on her part that she had not made any will explains her failure to do any act
of revocation in the forty-four days during which she lingered in this life. The doctrine

that where the testator has had an opportunity to revoke his will subsequent to the
operation of an alleged undue influence upon him but makes no change in it, the courts
will consider this fact as weighing heavily against the testimony of undue influence, has
no application to cases in which there has been an initial lack of testamentary capacity.
It has no application, moreover, where from the day of execution until the death of the
testator his mental condition is such that he cannot judge the propriety of revoking the
will. Nor obviously does it apply to a case where the alleged testator harbors the belief
that he had not executed the will in question.
In view of the premises, the judgment appealed from is affirmed with costs against the
appellant.

G.R. No. L-16763 December 22, 1921


PASCUAL COSO, petitioner-appellant, vs. FERMINA FERNANDEZ DEZA, ET
AL., objectors-appellees.
Eduardo Gutierrez Repide & Felix Socias for appellant.
Jose Varela Calderon & Benito Jimenez Zoboli for appellees.

OSTRAND, J.:
This is an appeal from a decision of the Court of First Instance of Manila setting aside a
will on the ground of undue influence alleged to have been exerted over the mind of a
testator by one Rosario Lopez. The will gives the tercio de libre disposicion to an
illegitimate son had by the testator with said Rosario Lopez, and also provides for the
payment to her of nineteen hundred Spanish duros by way the reimbursement for
expenses incurred by her in taking care of the testator in Barcelona during the years
1909 to 1916, when he is alleged to have suffered from a severe illness.
The evidence shows that the testator, a married man and resident of the Philippine
Islands, became acquainted with Rosario Lopez in Spain in 1898 and that he had illicit
returns with her for many years thereafter. After his return to the Philippines she
followed him, arriving in Manila in February, 1918, and remained in close
communication with him until his death in February, 1919. There is no doubt that she
exercised some influence over him and the only question for our determination is
whether this influence was of such a character as to vitiate the will.
The English and American rule in regard to undue influence is thus stated in 40 Cyc.,
1144-1149.
Mere general or reasonable influence over a testator is not sufficient to
invalidate a will; to have that effect the influence must be "undue." The rule as to
what constitutes "undue influence" has been variously stated, but the substance
of the different statements is that, to be sufficient to avoid a will, the influence
exerted must be of a kind that so overpowers and subjugates the mind of the
testator as to destroy his free agency and make his express the will of another,
rather than his own.1awphil.net

. . . such influence must be actually exerted on the mind of the testator in regard
to the execution of the will in question, either at time of the execution of the will,
or so near thereto as to be still operative, with the object of procuring a will in
favor of particular parties, and it must result in the making of testamentary
dispositions which the testator would not otherwise have made. . . .
. . . and while the same amount of influence may become "undue" when
exercised by one occupying an improper and adulterous relation to testator, the
mere fact that some influence is exercised by a person sustaining that relation
does not invalidate a will, unless it is further shown that the influence destroys
the testator's free agency.
The burden is upon the parties challenging the will to show that undue influence, in the
sense above expressed, existed at the time of its execution and we do not think that
this burden has been carried in the present case. While it is shown that the testator
entertained strong affections for Rosario Lopez, it does not appear that her influence so
overpowered and subjugated his mind as to "destroy his free agency and make him
express the will of another rather than his own." He was an intelligent man, a lawyer by
profession, appears to have known his own mind, and may well have been actuated
only by a legitimate sense of duty in making provisions for the welfare of his illegitimate
son and by a proper feeling of gratitude in repaying Rosario Lopez for the sacrifices she
had made for him. Mere affection, even if illegitimate, is not undue influence and does
not invalidate a will. No imposition or fraud has been shown in the present case.
Influence gained by kindness and affection will not be regarded as `undue,' if no
imposition or fraud be practiced, even though it induces the testator to make an
unequal and unjust disposition of his property in favor of those who have
contributed to his comfort and ministered to his wants, if such disposition is
voluntarily made. (Mackall vs. Mackall, 135 U. S., 1677.)
It may be further observed that under the Civil Law the right of a person with legal heirs
to dispose of his property by will is limited to only a portion of his estate, and that under
the law in force in these Islands before the enactment of the Code of Civil Procedure,
the only outside influences affecting the validity of a will were duress, deceit, and fraud.
The present doctrine of undue influence originated in a legal system where the right of
the testator to dispose of his property by will was nearly unlimited. Manifestly, greater
safeguards in regard to execution of wills may be warranted when the right to so
dispose of property is unlimited than when it is restricted to the extent it is in this

jurisdiction. There is, therefore, certainly no reason for giving the doctrine of undue
influence a wider scope here than it enjoys in the United States.
For the reasons stated, the decision of the lower court disallowing the will of Federico
Gimenez Zoboli is hereby reversed and it is ordered that the will be admitted to
probate. No costs will be allowed. So ordered.

G.R. No. L-14474

October 31, 1960

ONESIMA D. BELEN, petitioner-appellant, vs. BANK OF THE PHILIPPINE ISLANDS


and MILAGROS BELEN DE OLAGUERA, oppositors-appellees.
E. A. Beltran for appellant.
E. P. Villar for appellees.
R. F. Aviado for Trustee Bank.

On November 7, 1944, Benigno Diaz died; and the aforesaid codicil, together with the
will, was admitted to probate in Special Proceedings No. 894 of the same Court of First
Instance of Manila. The proceedings for the administration of the estate of Benigno
Diaz were closed in 1950 and the estate was thereafter put under the administration of
the appellee Bank of the Philippine Islands, as trustee for the benefit of the legatees.
Filomena Diaz died on February 8, 1954, leaving two legitimate children, Milagros
Belen de Olsguera, married, with seven (7) legitimate children, and Onesima D. Belen,
single.

REYES, J.B.L., J.:


Appeal from an order, dated May 23, 1958 of the Court of First Instance of Manila in
Special Proceedings No. 9226, denying appellant's petition therein as hereafter
discussed.
Briefly, the facts and circumstances that brought about this present appeal may be
narrated as follows:
Benigno Diaz executed a codicil on September 29, 1944, the pertinent provisions of
which read:
9.0 En caso de muerte de alguno o de todos los legatarios nom brados por
mi, seran beneficiarios o sea parasan los legados a favor solamente de los
descendientes y ascendientes legitimos, pero no a los viudos conyuges.
10.0 Transcurridos diez o quince aos despues de mi muerte todas
mispropiedades, muebles o inmuebles, derechos y ventajosos, pueden
proceder a la venta de todos dando preferencia a los legatarios y de su importe
total se deduciran mil pesos (P1,000) para los cuartrohijos de mi difunto
hermano Fabian, todos los gastos y reservando una cantidad suficiente y bein
calcumada para sufrugar se distriburia a las siguientes personas que aun
vuiven, o a sus descendientes legitimos:
A Isabel M. de Santiago cincuente por ciento (50%)
Los hijos de Domingo Legarda treinta por ciente (30%)
Filomena Diaz diez por ciento (10%)
Nestor M. Santiago diez por ciento (10%)

On March 19, 1958, Onesima D. Belen filed a petition in Special Proceedings No. 9226,
contending that the amount that would have appertained to Filomena Diaz under the
codicil should now be divided(equally) only between herself and Milagros Belen de
Olaguera, as the surviving children of the said deceased, to the exclusion, in other
words, of the seven (7) legitimate children of Milagros Belen de Olaguera. The court, in
its order on May 23, 1958 denied, as initially pointed out Onesima's petition. More
specifically, the court said:
After due consideration of the petition filed by Onesima D. Belen on March 19,
1958, wherein it is prayed that the trustee Bank of the Philippine Island be
directed to deliver to her "one-half of whatever share is due to the deceased
Filomena Diaz as legatee in the will and codicil of the deceased testator
Benigno Diaz y Heredia, subject of trusteeship in these proceedings," this Court
of the resolution of September 28, 1959, in which resolution the following was
declared:
"That the share of Filomena Diaz in the residue of the proceeds of the
sale of the properties covered in paragraph 10 of the codicil aforesaid
does not and should not from part of her estate; it pertains to her
legitimate descendants; and
"That the aforesaid share of Filomena Diaz should be distributed not
only between her children, Milagros Belen de Olaguera and Onesima D.
Belen, but also among her other legitimate descendants, if any,
for descendientes include not only children but also grandchildren, etc.,
and in this connection. it is not amiss to observe that one may be a
descendant and not yet not be an heir, and vice versa, one may be an
heir and yet not be a descendant.

From this order Onesima D. Belen has appealed to this Court, insisting that (1) the
Court below was in error in holding that its former resolution of September 16, 1955
had been affirmed by our decision of February 28, 1958 in the case of Arguelles vs.
Belen de Olaguera, G.R. No. L-10164 Feb. 28, 1958; and (2) that the term "sus
descendeintes legitimos," as used in the codicil, should be interpreted to mean
descendants nearest in the degree to the original legatee Filomena Diaz. In the present
case, they are her two daughters (Milagros and Onesima Belen), thereby excluding the
seven grandchildren of said legatee.
As to her first point, the appellant is the correct ion her view that the trial court's
interpretation of clause 10 of the codicil to the will of Benigno Diaz has not been
affirmed in our previous decision (G.R. No. L-10164). Perusal of that judgment will
show that this Court left the issue open at the time, contenting itself with pointing out
that the then appellant Administrator of the estate of Folimena Diaz was not the proper
party to the raise the particular issue.
As the actual meaning of the provision
El restro se distribuira a las siguientes personas que aun viven, o a sus
descendientes legitimos,
it is undeniable that but this cluase the testator ordained a simple substitution
(sustitucion vulgar) with a plurality of substitutes for each legatee. This form of
substitution authorized by the first poart of Article 860 of the Civil Code (Art. 778 of the
Code of 1889):
Two or more persons may be substituted for one and one person for two or
more heirs.
The issue is now squarely before us : do the words "sus descendientes legitimos" refer
conjointly to all living descendant (children and grandchildren) of the legatee, as a
class; or they refer to the descendants nearest in degree?
Appellant Onesima Belen contends that the phrase should be taken to mean the
relatives nearest in degree to Filomena Diaz; and that the legacy should be therefore
divided equally between her and her sister Milagros Belen de Olaguera, to the
exclusion of the latter's sons and daughters, grand children of the original legatee,
Filomena Diaz. As authority in support of her thesis, appellant invokes Article 959 of the
Civil Code of the Philippines (reproducing ne varieter Article 751 of the Code of 1889):

A distribution made in general terms in favor of the testator's relatives shall be


understood as made in favor of those nearest in degree.
The argument fails to note that this article is specifically limited in its application to the
case where the beneficiaries are relatives of the testator, not those of the legatee. In
such an event, the law assumes that the testator intended to refer to the rules of
intestacy, in order to benefit the relatives closest to him, because, as Manresa
observes,
la razon y la logica ha cen fundadamente suponer que, al procurar este
favorecer a sus parientes, habria de ajustarse mas a ligadas al mismo
(testador) por los vinculos de la sanger y de la familia (6 Manresa, Comm., 7th
Ed., p. 72).
But the ratios legis (that among a testator's relative the closest are dearest) obviously
does not supply where the beneficiaries are relatives of another person (the legatee)
and noot of the testator . There is no logical reason in this case to presume that the
testator intended to refer to the rules of intestacy, for he precisely made a testament
and provided substitutes for each legatee; nor can it be said that his affections would
prefer the nearest relatives of the legatee to those more distant, since he envisages all
of them in a group, and only as mere substitutes for a preferred beneficiary.
Should Article 959 (old Art. 751) be applied by anology? There are various reasons
against this. The most important one is that under this article, as recognized by the
principal commentators on the Code of 1889, the nearest of exclude all the farther
relatives and right of representation does not operate. Castan, in his monograph "El
derecho de representacion y mecanimos jurididos afines en la sucesion testamentaria"
(Reus, 1942), says on this question (Pp. 13, 14, 15):
En el subgrupo ibericio de Europia y America predomina, aunque haya ex
excepciones, cuando menos en principio, no tiene cabida en la sucesion
testamentaria. Asi, por ejemplo, lo establece la doctrina cientifica en Portugal y
en la Argentina y lo ha sancionado la jurisprudencia en Cuba.
En igual sentido, en la doctrina espaola es opinion general que el derecho de
representacion, dentro del Codigo civil, no tiene lugar mas que en la sucesion
intestada, y en la testamentaria en la parte refernte a las legitimas. MUCIUS
SCAEVOLA juzga que la reopresentacion, atraida por la herencia legitima, es
repelida por la testada, y apunta, como razon de ello, la de que "la primera

descansa en la ley de la sangre, en el parentesco con su consiguiente atributo


de linea y grado, elementos propios o indispensabnles para la repretascion , en
tanto que l asegunda se basa exclusivamente en la voluntad del testador,
elemento diverso, en la orderen legal, al de la naturaleza o de la sandre". Y el
maestro DE DIEGO, con orientacion anologa, piensa que como el titulo de la
sucesion testada es de origen voluntario y caracter personalismo, es evidente
que no hay terminos habiles para el derecho de representacion: los
llamamientos son individuales y la premoriencia del instituido, como su
incapacidad, aniquilan la institucion.
In the second place, the history of Article 751 (of the 1889 Code) shown that the right of
representation wasdeliberately suppressed. Says Castan (op. cit., 24):
En nuestra Patria opino GARCIA GOYENA que debia tener a los parientos mas
representacion aun cuando el testator llame abiertamente la voluntad del
testador, debe "observarse el orden de la sucesion legitima, al que se presume
que en todo lo demas quiso atemperase. Poe ello, el art. 562 Proyecto de 1851
quedo redactadso asi: "La disposicion hecha simple y generalmente a favor de
los parientes del testador, se entiende hecha en favor de los mas proximos en
grado ; pero habra lugar al derecho de representacion con todos sus efectos,
con arreglo al tittulo siguiento".
Con poco acierto, a nuestro juicio, los autores del vigente Codigo han suprimido
esta salvedad del Proyecto del 51, y con ello han instaurado una norma rigida,
distanciada de lo que exige la equidad y de lo que suelen establecer los
Codiogos extranjeros. Los commentaristas convienen en que la supresion ha
sido intencionada, y por consiguiente el proposito del legaslador es que en esta
clase de llamamientos no se da el derecho, de representacion. Dice Manresa
que el art. 751 "tiene por favorecidos con tal institucion, no a los parientes de
mejor derecho, sino a los mas proximos en grado y, por lo tanto, los de primer
grado excluiran a los de segundo y asi sucesivamente, toda vez que la art.
915". La misma interpretacion dan al articulo de referencia NAVARRO AMANDI,
MUCIUS SCAEVOLA, SANCHEZ ROMAN y VALVERDE.
The result would be that by applying to the descendants of Filorema Diaz the "nearest
relatives" rule of Article 959, the inheritance would be limited to her children, or anyone
of them, excluding the grandchildren altogether. This could hardly be the intention of
the testator who, in the selfsame clause 10 of his council (ante), speaks of "cuatro hijos
de mi difunto hermano Fabian" and of "los hijos de Domingo Legarda," as well as of

"descendientes legitimos" of the other legates, to us indicating clearly that he


understood well that hijos and descendientes are not synonymous terms. Observe that,
in referring to the substitutes of Filomena Diaz, Nestor Santiago and Isabel M. de
Santiago, the testator, does not even use the description "sus hijos o descendientes,"
but only "descendientes".
It is suggested that "descendientes legitimos" could mean the nearest descendant but
with the right of representation in favor of the more distant relatives. Unquestionably,
the testator was at liberty to provide a series of successive substitutions in the order of
proximity of relationship to the original legatee. And he, likewise, was free to ordain that
the more distant descendants should enjoy the right of representation as in intestate
succession. But to arrive at such conclusion, we must declare that the testator had:.
(a) Rejected, or intended to reject, the right of accretion among co-heirs and colegatees, as established for testamentary successions by Articles 10016 (old Art. 982)
and 1019, and intended to replace such accretion with representation;
ART. 1016. In order that the right of accretion may take place in a testamentary
succession, it shall be necessary:
(1) That two or more persons be called to the same inheritance, or to the same
portion thereof, pro indiviso; and
(2) That one of the persons thus called die before the testator or renounce the
inheritance, or be incapacitated to receive it.
xxx

xxx

xxx

ART. 1019. The heirs to whom the petition goes by the right of accretion take it
in the same proportion that they inherit.
(b) Refused, likewise, the rule of Article 846 (reproduced from Article 765 of the Code of
1889) providing that:
Heirs instituted without designation of shares shall inherit in equal parts,
which would not obtain if the right of representation were to apply;

(c) Rejected finally the rule of Article 1022 (old Art. 986), that vacancies in the free part
should be filed according to the rules of accretion or substitution (not representation);
and in default of these two, ultimately inherited by the testator's own heirs intestate:
ART. 1022. In testamentary succession, when the right of accretion does not
take place, the vacant portion of the instituted heirs, if no substitute has been
designated, shall pass to the legal heirs of the testator, who shall receive it with
the same charges and obligations.
There is no doubt that, the testator's intention being the cardinal rule of succession in
the absence of compulsory (forced) heirs, he could have rendered inoperative all the
articles mentioned, if he had so desired. But without any other supporting
circumstances, we deem expression "o a sus desecendientes legitimos," the testator
Benigno Diaz did intend to circumvent all the legal provisions heretofore quoted. It was
incumbent upon appellant to prove such intention on the part of the testator; yet she
has not done so.
It is interesting to note that even under the Anglo-Saxon doctrine, the courts are divided
on the question whether a bequest to "relatives" or "issue," made in general terms,
gives rise to succession per capita or per stripes. InWyeth, et al., vs. Crane, 174 N.E.
871, the Supreme Court of Illinois said;
The meaning of the word "descendants", when used in a will or deed to
designate a class to take property passing by the will or deed, has been
frequently considered and decided by the Court of England and the United
States. They established rule in England from an early date was that the word
"descendants" or the word "issued" unexplained by anything in the context of
the instrument, means all persons descending lineally from another, to the
remotest degree, and includes persons descended, even though their parents
are living, and that such descendants take per capita stripes.
The courts of this country are divided on the question of whether in case of a gift
or conveyance to "descendants" or "issue", children take concurrently with their
parents. The so- called English rule has been adhered to in New York, New
Jersey, and Tenessee. . . . On the other hand, the courts of Massachusetts,
Maine, Rhode Island and South Carolina have held that, in case of a gift or
conveyance to descendants or issue, unexplained by anything in the context of
instrument, children do not take currently with their parents.

We conclude that in the absence of other indications of contrary intent, the proper rule
to apply in the instant case is that the testator, by designating a class or group of
legatees, intended all members thereof to succeed per capita, in consonance with
article 846. So that the original legacy to Filomena Diaz should be equally divided
among her surviving children and grandchidren.
The order appealed from is affirmed, with costs to the appellant..

G.R. No. L-8927

March 10, 1914

ASUNCION NABLE JOSE, ET AL., plaintiff-appellants, vs. MARIA IGNACIA USON,


ET AL., defendants-appellees.
Ramon Salinas for appellants.
Pedro M.a Sison for appellees.
MORELAND, J.:
The question involved in this appeal arises from the interpretation of the first and
second clauses of a codicil to the will of Filomena Uson. They read as follows:
First. I declare that all the property which belongs to me as conjugal property,
referred to in my said testament, shall be the property of my aforesaid husband,
Don Rafael Sison; in case all or part of said property exists at my husband's
death, it is my will that at his death my sisters and nieces hereinafter named
succeed him as heirs.
Second. I declare to be my sisters in lawful wedlock the persons named Doa
Antonia Uson, now deceased, who has left tow daughters called Maria Rosario,
widow, and Maria Paz, unmarried; Maria Romualda Uson, widow of Estanislao
Lengson; Ignacia Uson, married to Don Vicente Puson; Eufemia Uson, now
deceased, who is survived by three daughters called Maria Salud, Maria
Amparo, and Maria Asuncion; and Maria Pilar Uson; Maria Manaoag Uson,
unmarried, issued had by our deceased after Don Daniel Uson with one
Leonarda Fernandez, alias Andao de Lingayen, so that they may have and
enjoy it in equal parts as good sisters and relatives.
The court below found that the children of the deceased sisters should take only that
portion which their respective mothers would have taken if they been alive at the time
the will was made; that the property should be divided into six equal parts
corresponding to the number of sisters; that each living sister should take one-sixth,
and the children of each deceased sister should also take one-sixth, each one- sixth to
be divided among said children equally.
This appeal is taken from the judgment entered upon that finding, appellants asserting
that under a proper construction of the paragraphs of the codicil above-quoted the

property should be divided equally between the living sisters and the children of the
deceased sisters, share and share alike, a niece taking the same share that a sister
receives.
We are of the opinion that the appellants' contention is well founded. We see no words
appellants in the clauses quoted which lead necessarily to the construction placed upon
those paragraphs by the learned court below. On the other hand, we find expressions
which seem to indicate with fair clearness that it was the intention of the testatrix to
divide her property equally between her sisters and nieces. The court below based its
construction upon the theory that the other construction would be "an admission that
the testatrix desired to favor her deceased sister Eufemia Uson, who left three children,
more than her other deceased sister Antonia Uson, who left two children, and moreover
both would be more favored than any of the other four surviving sisters, one of whom
was married at the time of the execution of the said codicil and without doubt had
children."
As we look at the codicil we observe, first, that the testatrix, in the first paragraph
thereof, declares that after her husband's death she desires that "my sisters and
nieces, as hereinafter named, shall succeed him as heirs."
We note, in the second place, that the testatrix, in the second paragraph of the codicil,
names and identifies each one of her heirs then living, in each one of the persons
whom she desires shall succeed her husband in the property. Among those mentioned
specially are the nieces as well as the sisters. The nieces are referred to in no way
different from the sisters. Each one stands out in the second paragraph of the codicil as
clearly as the other and under exactly the same conditions.
In the third place, we note, with interest, the last clause of the second paragraph of the
codicil which, it seems to us, taken together with the last clause of the first paragraph of
the codicil, is decisive of the intention of the testatrix. In the last clause she says that
she names all of the persons whom she desires to take under her will be name "so that
they must take and enjoy the property in equal parts as good sisters and relatives."
We have then in the first paragraph a declaration as to who the testatrix desires shall
become the owners of her property on the death of her husband. Among them we find
the names of the nieces as well as of the sisters. We have also the final declaration of
the testatrix that she desires that the sisters and the nieces shall take and enjoy the
property in equal parts. That being so, it appears to us that the testatrix's intention is

fairly clear, so clear in fact that it is unnecessary to bring in extraneous arguments to


reach a conclusion as to what she intended.
The judgment appealed from is hereby modified by declaring that, of the property
passing under the codicil herein above referred to, the living sisters and the children of
the deceased sisters shall take per capita and in equal parts, and as so modified the
judgment is affirmed. No costs in this instance.

G.R. No. L-23079 February 27, 1970


RUBEN AUSTRIA, CONSUELO AUSTRIA-BENTA and LAURO AUSTRIA
MOZO, petitioners, vs. HON. ANDRES REYES, Judge, Court of First Instance of
Rizal, PERFECTO CRUZ, BENITA CRUZ-MENEZ ISAGANI CRUZ, ALBERTO CRUZ
and LUZ CRUZ-SALONGA respondents.
Salonga, Ordoez, Yap, Sicat and Associates for petitioners.

of Basilia, and that the five respondents Perfecto Cruz, et al., had not in fact been
adopted by the decedent in accordance with law, in effect rendering these respondents
mere strangers to the decedent and without any right to succeed as heirs.
Notwithstanding opposition by the respondent Perfecto Cruz, as executor of the estate,
the court a quo allowed the petitioners' intervention by its order of December 22, 1959,
couched in broad terms, as follows: "The Petition in Intervention for Partition filed by the
above-named oppositors [Ruben Austria, et al.,] dated November 5, 1959 is hereby
granted."

Ruben Austria for himself and co-petitioners.


De los Santos, De los Santos and De los Santos for respondent Perfecto Cruz.
Villareal, Almacen, Navarra and Amores for other respondents.

CASTRO, J.:
On July 7, 1956 Basilia Austria vda. de Cruz filed with the Court of First Instance of
Rizal (Special Proceedings 2457) a petition for probate, ante mortem, of her last will
and testament. The probate was opposed by the present petitioners Ruben Austria,
Consuelo Austria-Benta and Lauro Austria Mozo, and still others who, like the
petitioner, are nephews and nieces of Basilia. This opposition was, however, dismissed
and the probate of the will allowed after due hearing.
The bulk of the estate of Basilia, admittedly, was destined under the will to pass on to
the respondents Perfecto Cruz, Benita Cruz-Meez, Isagani Cruz, Alberto Cruz, and
Luz Cruz-Salonga, all of whom had been assumed and declared by Basilia as her own
legally adopted children.
On April 23, 1959, more than two years after her will was allowed to probate, Basilia
died. The respondent Perfecto Cruz was appointed executor without bond by the same
court in accordance with the provisions of the decedent's will, notwithstanding the
blocking attempt pursued by the petitioner Ruben Austria.
Finally, on November 5, 1959, the present petitioners filed in the same proceedings a
petition in intervention for partition alleging in substance that they are the nearest of kin

In the meantime, the contending sides debated the matter of authenticity or lack of it of
the several adoption papers produced and presented by the respondents. On motion of
the petitioners Ruben Austria, et al., these documents were referred to the National
Bureau of Investigation for examination and advice. N.B.I. report seems to bear out the
genuineness of the documents, but the petitioners, evidently dissatisfied with the
results, managed to obtain a preliminary opinion from a Constabulary questioneddocument examiner whose views undermine the authenticity of the said documents.
The petitioners Ruben Austria, et al., thus moved the lower court to refer the adoption
papers to the Philippine Constabulary for further study. The petitioners likewise located
former personnel of the court which appeared to have granted the questioned adoption,
and obtained written depositions from two of them denying any knowledge of the
pertinent adoption proceedings.
On February 6, 1963, more than three years after they were allowed to intervene, the
petitioners Ruben Austria, let al., moved the lower court to set for hearing the matter of
the genuineness of the adoption of the respondents Perfecto Cruz, et al., by the late
Basilia. Before the date set by the court for hearing arrived, however, the respondent
Benita Cruz-Meez who entered an appearance separately from that of her brother
Perfecto Cruz, filed on February 28, 1963 a motion asking the lower court, by way of
alternative relief, to confine the petitioners' intervention, should it be permitted, to
properties not disposed of in the will of the decedent.
On March 4, 1963, the lower court heard the respondent Benita's motion. Both sides
subsequently submitted their respective memoranda, and finally, the lower court issued
an order on June 4, 1963, delimiting the petitioners' intervention to the properties of the
deceased which were not disposed of in the will.

The petitioners moved the lower court to reconsider this latest order, eliciting thereby an
opposition, from the respondents. On October 25, 1963 the same court denied the
petitioners' motion for reconsideration.
A second motion for reconsideration which set off a long exchange of memoranda from
both sides, was summarily denied on April 21, 1964.
Hence this petition for certiorari, praying this Court to annul the orders of June 4 and
October 25, 1963 and the order of April 21, 1964, all restricting petitioners' intervention
to properties that were not included in the decedent's testamentary dispositions.
The uncontested premises are clear. Two interests are locked in dispute over the bulk
of the estate of the deceased. Arrayed on one side are the petitioners Ruben Austria,
Consuelo Austria-Benta and Lauro Austria Mozo, three of a number of nephews and
nieces who are concededly the nearest surviving blood relatives of the decedent. On
the other side are the respondents brothers and sisters, Perfecto Cruz, Benita CruzMeez, Isagani Cruz, Alberto Cruz and Luz Cruz-Salonga, all of whom heirs in the will
of the deceased Basilia, and all of whom claim kinship with the decedent by virtue of
legal adoption. At the heart of the controversy is Basilia's last will immaculate in its
extrinsic validity since it bears the imprimatur of duly conducted probate proceedings.
The complaint in intervention filed in the lower court assails the legality of the tie which
the respondent Perfecto Cruz and his brothers and sisters claim to have with the
decedent. The lower court had, however, assumed, by its orders in question, that the
validity or invalidity of the adoption is not material nor decisive on the efficacy of the
institution of heirs; for, even if the adoption in question were spurious, the respondents
Perfecto Cruz, et al., will nevertheless succeed not as compulsory heirs but as
testamentary heirs instituted in Basilia's will. This ruling apparently finds support in
article, 842 of the Civil Code which reads:

free-wheeling testamentary disposition. The petitioners' interest is confined to


properties, if any, that have not been disposed of in the will, for to that extent intestate
succession can take place and the question of the veracity of the adoption acquires
relevance.
The petitioners nephews and niece, upon the other hand, insist that the entire estate
should descend to them by intestacy by reason of the intrinsic nullity of the institution of
heirs embodied in the decedent's will. They have thus raised squarely the issue of
whether or not such institution of heirs would retain efficacy in the event there exists
proof that the adoption of the same heirs by the decedent is false.
The petitioners cite, as the controlling rule, article 850 of the Civil Code which reads:
The statement of a false cause for the institution of an heir shall be
considered as not written, unless it appears from the will that the testator
would not have made such institution if he had known the falsity of such
cause.
Coming closer to the center of the controversy, the petitioners have called the attention
of the lower court and this Court to the following pertinent portions of the will of the
deceased which recite:
III
Ang aking mga sapilitang tagapagmana (herederos forzosos) ay ang
aking itinuturing na mga anak na tunay (Hijos legalmente adoptados) na
sina Perfecto, Alberto, Luz, Benita at Isagani, na pawang may
apelyidong Cruz.
xxx xxx xxx

One who has no compulsory heirs may dispose of by will all his estate or
any part of it in favor of any person having capacity to succeed.
One who has compulsory heirs may dispose of his estate provided he
does not contravene the provisions of this Code with regard to the
legitime of said heirs.
The lower court must have assumed that since the petitioners nephews and niece are
not compulsory heirs, they do not possess that interest which can be prejudiced by a

Kung ako ay bawian ng Dios ng buhay, ay aking ipinamamana ang aking


mga ari-ariang maiiwan, sa kaparaanang sumusunod:
A.Aking ipinamamana sa aking nabanggit na limang anak na sina
Perfecto, Alberto, Luz, Benita at Isagani, na pawang may apelyidong
Cruz, na parepareho ang kaparti ng bawa't isa at walang lamangan (en
partes iguales), bilang kanilang sapilitang mana (legiti[ma]), ang kalahati
() ng aking kaparti sa lahat ng aming ari-ariang gananciales ng aking

yumaong asawang Pedro Cruz na napapaloob sa Actuacion Especial


No. 640 ng Hukumang Unang Dulugan ng Rizal at itinutukoy sa No. 1 ng
parafo IV ng testamentong ito, ang kalahati () ng mga lagay na lupa at
palaisdaan na nasa Obando at Polo, Bulacan, na namana ko sa aking
yumaong ama na si Calixto Austria, at ang kalahati () ng ilang lagay na
lupa na nasa Tinejeros, Malabon, Rizal, na aking namana sa yumao
kong kapatid na si Fausto Austria.
The tenor of the language used, the petitioners argue, gives rise to the inference that
the late Basilia was deceived into believing that she was legally bound to bequeath
one-half of her entire estate to the respondents Perfecto Cruz, et al. as the latter's
legitime. The petitioners further contend that had the deceased known the adoption to
be spurious, she would not have instituted the respondents at all the basis of the
institution being solely her belief that they were compulsory heirs. Proof therefore of the
falsity of the adoption would cause a nullity of the institution of heirs and the opening of
the estate wide to intestacy. Did the lower court then abuse its discretion or act in
violation of the rights of the parties in barring the petitioners nephews and niece from
registering their claim even to properties adjudicated by the decedent in her will?
Before the institution of heirs may be annulled under article 850 of the Civil Code, the
following requisites must concur: First, the cause for the institution of heirs must be
stated in the will; second, the cause must be shown to be false; and third, it must
appear from the face of the will that the testator would not have made such institution if
he had known the falsity of the cause.
The petitioners would have us imply, from the use of the terms, "sapilitang
tagapagmana" (compulsory heirs) and "sapilitang mana" (legitime), that the impelling
reason or cause for the institution of the respondents was the testatrix's belief that
under the law she could not do otherwise. If this were indeed what prompted the
testatrix in instituting the respondents, she did not make it known in her will. Surely if
she was aware that succession to the legitime takes place by operation of law,
independent of her own wishes, she would not have found it convenient to name her
supposed compulsory heirs to their legitimes. Her express adoption of the rules on
legitimes should very well indicate her complete agreement with that statutory scheme.
But even this, like the petitioners' own proposition, is highly speculative of what was in
the mind of the testatrix when she executed her will. One fact prevails, however, and it
is that the decedent's will does not state in a specific or unequivocal manner the cause
for such institution of heirs. We cannot annul the same on the basis of guesswork or
uncertain implications.

And even if we should accept the petitioners' theory that the decedent instituted the
respondents Perfecto Cruz, et al. solely because she believed that the law commanded
her to do so, on the false assumption that her adoption of these respondents was valid,
still such institution must stand.
Article 850 of the Civil Code, quoted above, is a positive injunction to ignore whatever
false cause the testator may have written in his will for the institution of heirs. Such
institution may be annulled only when one is satisfied, after an examination of the will,
that the testator clearly would not have made the institution if he had known the cause
for it to be false. Now, would the late Basilia have caused the revocation of the
institution of heirs if she had known that she was mistaken in treating these heirs as her
legally adopted children? Or would she have instituted them nonetheless?
The decedent's will, which alone should provide the answer, is mute on this point or at
best is vague and uncertain. The phrases, "mga sapilitang
tagapagmana" and "sapilitang mana," were borrowed from the language of the law on
succession and were used, respectively, to describe the class of heirs instituted and the
abstract object of the inheritance. They offer no absolute indication that the decedent
would have willed her estate other than the way she did if she had known that she was
not bound by law to make allowance for legitimes. Her disposition of the free portion of
her estate (libre disposicion) which largely favored the respondent Perfecto Cruz, the
latter's children, and the children of the respondent Benita Cruz, shows a perceptible
inclination on her part to give to the respondents more than what she thought the law
enjoined her to give to them. Compare this with the relatively small devise of land which
the decedent had left for her blood relatives, including the petitioners Consuelo AustriaBenta and Lauro Mozo and the children of the petitioner Ruben Austria. Were we to
exclude the respondents Perfecto Cruz, et al. from the inheritance, then the petitioners
and the other nephews and nieces would succeed to the bulk of the testate by intestacy
a result which would subvert the clear wishes of the decedent.
Whatever doubts one entertains in his mind should be swept away by these explicit
injunctions in the Civil Code: "The words of a will are to receive an interpretation which
will give to every expression some effect, rather than one which will render any of the
expressions inoperative; and of two modes of interpreting a will, that is to be preferred
which will prevent intestacy." 1
Testacy is favored and doubts are resolved on its side, especially where the will evinces
an intention on the part of the testator to dispose of practically his whole estate,2 as was
done in this case. Moreover, so compelling is the principle that intestacy should be

avoided and the wishes of the testator allowed to prevail, that we could even vary the
language of the will for the purpose of giving it effect.3 A probate court has found, by
final judgment, that the late Basilia Austria Vda. de Cruz was possessed of
testamentary capacity and her last will executed free from falsification, fraud, trickery or
undue influence. In this situation, it becomes our duty to give full expression to her will.4
At all events, the legality of the adoption of the respondents by the testatrix can be
assailed only in a separate action brought for that purpose, and cannot be the subject
of a collateral attack.5
To the petitioners' charge that the lower court had no power to reverse its order of
December 22, 1959, suffice it to state that, as borne by the records, the subsequent
orders complained of served merely to clarify the first an act which the court could
legally do. Every court has the inherent power to amend and control its processes and
orders so as to make them conformable to law and justices.6 That the court a quo has
limited the extent of the petitioners' intervention is also within its powers as articulated
by the Rules of Court.7
ACCORDINGLY, the present petition is denied, at petitioners cost.

G.R. No. L-29789

December 22, 1928

In re estate of Jose Macrohon Tiahua. FRANCISCO BARRIOS, administratorappellee, vs. EDUARDA ENRIQUEZ, ET AL., heirs-appellants.
IGNACIO MACROHON, appellant.
P. J. M. Moore for heirs and appellants.
Pablo Lorenzo, Juan S. Alvarez, Perfecto Gabriel, and Delfin Joven for appellant
Macrohon.
No appearance for appellee.

Now come the herein surviving spouse, Eduarda Enriquez, and the legitimate
heirs of Jose Macrohon Tiahua, deceased, by their undersigned attorney, and
allege that under the law and the evidence in the above-entitled proceeding, the
lower court committed the following errors:
I. In not approving the project of distribution of the estate of the herein
deceased, Jose Macrohon Tiahua, as submitted by the herein executor,
Francisco Barrios.
II. In holding that the deceased, Jose Macrohon Tiahua, had any right to
institute and name his bastard son, Ignacio Macrohon, as an heir in his will,
together and in a like manner with his legitimate descendants and
heirs.1awphi1.net

VILLA-REAL, J.:
This is an appeal taken by Eduarda Enriquez, surviving spouse of the deceased Jose
Macrohon Tiahua, and the latter's legitimate children on the one side, and Ignacio
Macrohon, his adulterous son, on the other, from an order of the Court of First Instance
of Zamboanga, laying down the following conclusions of law: (1) That an adulterous
child may be instituted heir within the limits provided by law; (2) that in making Ignacio
Macrohon an heir under his will, the testator did not observe the limitations prescribed
by law; (3) that the institution of Ignacio Macrohon as heir under the will ought not to be
declared absolutely void, but he should so share in the inheritance as not to prejudice
the legitimate of the other heirs; (4) that as Exhibit 1 deals with certain acts contrary to
law, such as not presenting the will to the court, and as some minors took part in it
through their guardian without the latter being authorized by the court to enter into the
transaction in their behalf, said exhibit cannot bind the parties, nor do the admissions
made by them therein constitute estoppel; whereupon it disapproved the scheme of
partition presented by the administrator and ordered him to file another in consonance
with the conclusions therein laid down.
In support of their appeal, Eduarda Enriquez, widow of Jose Macrohon Tiahua, and the
latter's legitimate children assign the following alleged errors as committed by the trial
court in its judgment, to wit:

III. And in holding that the said bastard son, Ignacio Macrohon, having been so
instituted and named in the will of the said Jose Macrohon Tiahua, deceased,
said Ignacio Macrohon had a right under the terms of the will and the law, to
inherit a portion of the estate of the deceased, to the prejudice of the legitimate
descendants and heirs of the said Jose Macrohon Tiahua, deceased.
Ignacio Macrohon, adulterous son of the deceased Jose Macrohon Tiahua in turn,
assigns the following alleged errors as committed by the trial court in its judgment, in
support of his appeal, to wit:
Comes now the appellant Ignacio Macrohon, thru the undersigned attorney's
and to this Honorable Court respectfully submits that the lower court erred, as
follows:
1. In holding that its order dated November 10, 1926, did not constitute res
adjudicata as to question of the right of the appellant Ignacio Macrohon to
inherit from his deceased father, Jose Marcohon Tiahua, under and in
accordance with the will of the latter.
2. In not holding that the right of the said Ignacio Macrohon as heir of said
deceased cannot, by reason of the doctrine of estoppel, be questioned by the
other heirs.

3. In holding that the manner the institution of the heirs was made in the will of
the testator herein falls under, or is the case contemplated by, article 765 of the
Civil Code.
4. In not allowing said Ignacio Marcrohon the full share allotted to him in and by
the will of the testator, that is, a portion equal to that granted in said will to each
of the legitimate of the deceased, or one-tenth of the whole hereditary estate.
The questions, all of law, raised by the respective parties in their respective appeals,
may be reduced to the following:
(1) Did the deceased Jose Macrohon Tiahua have a right to dispose of a part of his
estate by will in favor of his adulterous son? (2) If so, has the deceased Jose Macrohon
Tiahua infringed the limitations prescribed by the law in putting his adulterous son
Ignacio Macrohon on the same footing as his legitimate children by giving him a share
equal to that of each of the latter?
As to the first question, while it is true that article 845 of the Civil Code provides that
"illegitimate children who have not the status of natural children shall be entitled to
support only," and therefore cannot demand anything more of those bound by law to
support them, it does not prohibit said illegitimate children from receiving, nor their
parents from giving them, something more than support, so long as the legitimate
children are not prejudiced. If the law permits a testator to dispose of the free third of
his hereditary estate in favor of a stranger (art. 808 of the Civil Code), there is no legal,
moral or social reason to prevent him from making over that third to his illlegitimate son
who has not the status of a natural son. On the contrary, by reason of blood, the son,
although illegitimate, has a preferential right over a stranger unless by his behaviour he
has become unworthy of such consideration. For these reasons, we are of opinion and
so hold, that Jose Macrohon Tiahua could dispose of the free third of his estate in favor
of his adulterous son, Ignacio Macrohon. With respect to the second question of law,
Jose Macrohon Tiahua states the following in his will:
After all my debts, obligations, and funeral expenses have been paid, I hereby
bequeath and devise all my property, real, personal, and mixed, as follows, to
wit:
One-half () pro indiviso of my whole estate to my wife Eduarda Enriquez, and
the other half () in equal parts pro indiviso to each of my children, including
Fernando Quintas and Julia Quintas, son and daughter, respectively, of my

deceased daughter, Gregoria Macrohon, who shall receive the portion


corresponding to the share of my said daughter, that is, 1/44 for each of the two.
Included among the children mentioned by the testator in said will, and to whom he
gave the one-half of the property corresponding to him from the conjugal partnership, is
the herein appellant Ignacio Macrohon, his adulterous son. Dividing this half, that is tentwentieth parts (10/20), among his nine legitimate children and his adulterous son,
Ignacio Macrohon, into equal parts, each of them will be entitled to one-twentieth of the
whole estate.
Now then, does the twentieth part corresponding to the share of Ignacio Macrohon
impair the legitimate corresponding to each of the nine legitimate children?
According to article 808 of the Civil Code, the legitime of legitimate children and
descendants consists of two-thirds of the hereditary estate of the father and of the
mother, the latter being allowed to dispose of one of said two parts in order to give it as
a betterment to their legitimate children or descendants. In the present case the testator
has not disposed of any of the two parts forming the legitime in order to give it as
betterment to any of his children, and the said legitime therefore remains intact, and
according to article 806 of the same Code, is by the law reserved for the forced heirs
and the testator cannot dispose of it in any other way.
Hence, the nine-legitimate children are entitled to two-thirds of said half, or two-sixths of
the whole, which divided equally among them would give to each, two fifty- fourths or
one twenty-seventh of the whole estate. When Jose Macrohon Tiahua, therefore,
provided in his will that the one-half of the conjugal property belonging to him was to be
divided equally among his nine legitimate children and one adulterous son, each to
receive one-twentieth part, he did not go beyond the limits provided by law for such
cases, because, one-twentieth for each of his legitimate children is more than each of
his legitimate children should receive as his legitime, which only amounts to one
twenty-seventh. In other words, since Jose Macrohon Tiahua could dispose of the free
third of his herediatary estate in favor of his adulterous son, Ignacio Macrohon, and as
he only gave a part of said free third to the latter, he did not infringe any legal
prohibition and his testamentary disposition to this effect is valid and effective.
Having arrived at these conclusions there is no need to discuss the other questions of
law raised by the parties in their respective assignments of error.

By virtue whereof, the order appealed from is modified and it is ordered that the
dispositons made by Jose Macrohon Tiahua in his last will and testament be strictly
complied with, without special pronouncement of costs. So ordered.

G.R. No. L-26545 December 16, 1927


Testate Estate of Florencia R. Mateo. PERFECTO GABRIEL, petitioner-appellee, vs.
RITA R. MATEO, ET AL., opponents-appellants.
R. Gonzales Lloret, Carlos S. Basa, Thomas Cary Welch and Camus, Delgado and
Recto for appellants.
The appellee in his own behalf.

AVANCEA, C. J.:
The judgment appealed from allowed the will of Florencia Mateo dated February 6,
1923, composed of two used sheets to probate. The will appears to be signed by the
testatrix and three witnesses on the left margin of each of the sheets, by the testatrix
alone at the bottom, and by the three witnesses after the attestation clause. The
testatrix died on August 13, 1925. Opposition to such probate was filed by Rita Mateo,
the testatrix's sister, and by other relatives.
The three attesting witnesses to this will, testifying in this case, declared that the
signature of the testatrix were written in their presence and that they signed their
names in the presence of the testatrix and of each other.
The testatrix from girlhood knew how to sign her name and did so with her right hand;
but as the right side of her body later became paralyzed, she learned to sign with her
left hand and for many years thereafter, up to the time of her death, she used to sign
with that hand. Opponents allege that Florencia Mateo did not sign this will.
There are three salient arguments among those adduced by the opponents in support
of their opposition.
The attesting witnesses testified that the testratrix signed before they did. The
signatures of the testatrix on the left margin of the two sheets of the will are between
the signatures of the two witnesses Vidal Raoa and Julio Gabriel, and below her
surname is the signature of the other witness Felicisimo Gabriel. The signatures of
Vidal Raoa and Julio Gabriel are on a level with each other, while that of Felicisimo
Gabriel is found a little lower down. The testatrix's signatures start on the line with

Felicisimo Gabriel's signature, but tend to rise and her surname reaches a level with
Julio Gabriel's signature.
It is said that this direction of the testatrix's signature was due to the fact that when it
was written Felicisimo Gabriel's signature was already there, and so she had to write
her surname upwards in order to avoid interfering with that Felicisimo Gabriel, which
would have been the case had she continued on the horizontal line on which she had
written her first name. From this detail it is pretended to draw the inference that the
attesting witnesses signed before testatrix, contrary to their testimony that she singed
before they did. This deduction, however, is unnecessary. It may be inferred with equal,
if not greater, logic that the testatrix signed before him, and when it came to the witness
Gabriel's turn, he, finding the space below the testatrix signature free, signed his name
there. On the other hand, it may be noted that the testatrix's other signature at the
bottom of the will also shows a more or less marked tendency to rise, notwithstanding
the fact that there was no signature with which she might interfere if she continued to
write in a straight horizontal line. Furthermore, if, as the opposition alleges, the
testatrix's signature is not genuine and was placed there by another person, it is
strange that the latter should have done so in such a way as to write it above Gabriel's
signature while following the horizontal line, when this could have been avoided by
simply putting it a little higher. And this may be attributed to carelessness in the first
case, but it cannot be so explained in the second.
Attention is also called to the apparently different kinds of ink used by the testatrix in
her signature and by the attesting witnesses. Really an examination of these signature
reveals a somewhat deeper intensity of ink in the signature of the testatrix than in those
of the attesting witnesses. It is alleged that this circumstance cannot be reconciled with
the declaration of the attesting witnesses that they used the same pen and ink as the
testatrix. But, only one of these witnesses declared this. The other one was not sure of
it and said that he said that he did not perfectly remember this detail. The third scarcely
made reference to this particular. At all events, this apparent difference in ink may be
merely
due supposing that the same ink and pen were used to the difference in pressure
employed in writing these signatures, as is reasonable to suppose when we consider
that the testatrix was a paralytic and wrote with her left hand; or it may have been due
to the fact that the attesting witnesses dipped lightly in the ink while the testatrix dipped
the pen so as to take up the ink from the bottom of the well. To bring out this irregularity,
the opposition presented the expert Del Rosario who asserted, among other things, that
the signature of the testatrix is more recent than that of the attesting witnesses. If this

opinion is correct and if, as alleged, the testatrix's signature is forged, it would mean
that the forgers, after having prepared the will and made the witnesses sign, allowed
sometime to elapsed before forging the testatrix's signature, which supposition is not all
probable, nor has it been explained.lawphi1.net
At all events, even admitting that there is a certain question as to whether the attesting
witnesses signed before or after the testatrix, or whether or not they signed with the
same pen and ink, these are details of such trivial importance, considering that this will
was signed two years before the date on which these witnesses gave their testimony,
that it is not proper to set aside the will for this reason alone.
The attesting witnesses to this will, who testified also as witnesses at the trial of this
case, showed themselves to be intelligent and honest, one of them being a lawyer of
twelve year's practice, and there is no reason to reject their testimony, and to suppose
that they were untruthful in testifying, and that they falsified the will in question.
Lastly, attention is called to the unreasonableness of the testatrix in not leaving
anything to the principal opponent, her sister Rita Mateo, and to her nephews and
nieces, to whom she had been so affectionate during life. But as to the affectionate
relations between the deceased and the opponents, only the opponent Rita Mateo
testified, and she only stated that she was on good terms with her sister during the
latter's lifetime; that the said sister used to give her a sack or some gantas of rice, and,
a times, a little money; that she held all her nephews and nieces in equal regard. But
even supposing that this were so, there is nothing strange in the testatrix having left
nothing to the opponents, or in her having left all of her estate to the only heir instituted
in her will, Tomas Mateo, who is also one of her nieces. And not only is it not strange,
but it seems reasonable, since, according to the evidence of the testatrix when the
former was but 3 years old, and from then on up to the time of her death had never
been separated from her.
The opposition presented Doctor Banks as expert. He testified that the signatures of
the testatrix in the will are not genuine. The petitioner, on the other hand, presented
another expert, Pedro Serrano Laktao, who affirmed that these signatures are genuine.
But, over the testimony of these experts, we have the categorical and positive
declaration of veracious witnesses who affirm that these signatures were written by the
testatrix herself.
The judgment appealed from is affirmed, with costs against the appellants. So ordered.

G.R. No. L-24561 June 30, 1970


MARINA DIZON-RIVERA, executrix-appellee, vs. ESTELA DIZON, TOMAS V. DIZON,
BERNARDITA DIZON, JOSEFINA DIZON, ANGELINA DIZON and LILIA
DIZON, oppositors-appellants.
Punzalan, Yabut & Eusebio for executrix-appellee.
Leonardo Abola for oppositors-appellants.

TEEHANKEE, J.:
Appeal from orders of the Court of First Instance of Pampanga approving the Executrixappellee's project of partition instead of Oppositors-Appellants' proposed counterproject of partition. 1
On January 28, 1961, the testatrix, Agripina J. Valdez, a widow, died in Angeles,
Pampanga, and was survived by seven compulsory heirs, to wit, six legitimate children
named Estela Dizon, Tomas V. Dizon, Bernardita Dizon, Marina Dizon (herein
executrix-appellee), Angelina Dizon and Josefina Dizon, and a legitimate
granddaughter named Lilia Dizon, who is the only legitimate child and heir of Ramon
Dizon, a pre-deceased legitimate son of the said decedent. Six of these seven
compulsory heirs (except Marina Dizon, the executrix-appellee) are the oppositorsappellants.
The deceased testatrix left a last will executed on February 2, 1960 and written in the
Pampango dialect. Named beneficiaries in her will were the above-named compulsory
heirs, together with seven other legitimate grandchildren, namely Pablo Rivera, Jr.,
Gilbert D. Garcia, Cayetano Dizon, Francisco Rivera, Agripina Ayson, Jolly Jimenez
and Laureano Tiambon.
In her will, the testatrix divided, distributed and disposed of all her properties appraised
at P1,801,960.00 (except two small parcels of land appraised at P5,849.60, household
furniture valued at P2,500.00, a bank deposit in the sum of P409.95 and ten shares of
Pampanga Sugar Development Company valued at P350.00) among her above-named
heirs.

Testate proceedings were in due course commenced 2 and by order dated March 13,
1961, the last will and testament of the decedent was duly allowed and admitted to
probate, and the appellee Marina Dizon-Rivera was appointed executrix of the testatrix'
estate, and upon her filing her bond and oath of office, letters testamentary were duly
issued to her.
After the executrix filed her inventory of the estate, Dr. Adelaido Bernardo of Angeles,
Pampanga was appointed commissioner to appraise the properties of the estate. He
filed in due course his report of appraisal and the same was approved in toto by the
lower court on December 12, 1963 upon joint petition of the parties.
The real and personal properties of the testatrix at the time of her death thus had a total
appraised value of P1,811,695.60, and the legitime of each of the seven compulsory
heirs amounted to P129,362.11. 3 (/7 of the half of the estate reserved for the legitime
of legitimate children and descendants). 4 In her will, the testatrix "commanded that her
property be divided" in accordance with her testamentary disposition, whereby she
devised and bequeathed specific real properties comprising practically the entire bulk of
her estate among her six children and eight grandchildren. The appraised values of the
real properties thus respectively devised by the testatrix to the beneficiaries named in
her will, are as follows:
1. Estela Dizon ....................................... P 98,474.80
2. Angelina Dizon .................................. 106,307.06
3. Bernardita Dizon .................................. 51,968.17
4. Josefina Dizon ...................................... 52,056.39
5. Tomas Dizon ....................................... 131,987.41
6. Lilia Dizon .............................................. 72,182.47
7. Marina Dizon ..................................... 1,148,063.71
8. Pablo Rivera, Jr. ...................................... 69,280.00
9. Lilia Dizon, Gilbert Garcia,
Cayetano Dizon, Francisco Rivera,
Agripina Ayson, Dioli or Jolly
Jimenez, Laureano Tiamzon ................. 72,540.00
Total Value ...................... P1,801,960.01
The executrix filed her project of partition dated February 5, 1964, in substance
adjudicating the estate as follows:

(1) with the figure of P129,254.96 as legitime for a basis Marina


(exacultrix-appellee) and Tomas (appellant) are admittedly considered to
have received in the will more than their respective legitime, while the
rest of the appellants, namely, Estela, Bernardita, Angelina, Josefina and
Lilia received less than their respective legitime;
(2) thus, to each of the latter are adjudicated the properties respectively
given them in the will, plus cash and/or properties, to complete their
respective legitimes to P129,254.96; (3) on the other hand, Marina and
Tomas are adjudicated the properties that they received in the will less
the cash and/or properties necessary to complete the prejudiced legitime
mentioned in number 2 above;
(4) the adjudications made in the will in favor of the grandchildren remain
untouched.<re||an1w>
On the other hand oppositors submitted their own counter-project of
partition dated February 14, 1964, wherein they proposed the
distribution of the estate on the following basis:
(a) all the testamentary dispositions were proportionally reduced to the
value of one-half () of the entire estate, the value of the said one-half
() amounting to P905,534.78; (b) the shares of the OppositorsAppellants should consist of their legitime, plus the devises in their favor
proportionally reduced; (c) in payment of the total shares of the
appellants in the entire estate, the properties devised to them plus other
properties left by the Testatrix and/or cash are adjudicated to them; and
(d) to the grandchildren who are not compulsory heirs are adjudicated
the properties respectively devised to them subject to reimbursement by
Gilbert D. Garcia, et al., of the sums by which the devise in their favor
should be proportionally reduced.
Under the oppositors' counter-project of partition, the testamentary disposition made by
the testatrix of practically her whole estate of P1,801,960.01, as above stated, were
proposed to be reduced to the amounts set forth after the names of the respective heirs
and devisees totalling one-half thereof as follows:
1. Estela Dizon ........................................... P 49,485.56
2. Angelina Dizon ......................................... 53,421.42

3. Bernardita Dizon ....................................... 26,115.04


4. Josefina Dizon .......................................... 26,159.38
5. Tomas V. Dizon ......................................... 65,874.04
6. Lilia Dizon .................................................. 36,273.13
7. Marina Dizon ........................................... 576,938.82
8. Pablo Rivera, Jr. ......................................... 34,814.50
9. Grandchildren Gilbert Garcia et al .......... 36,452.80
T o t a l ................................................... P905,534.78
while the other half of the estate (P905,534.78) would be deemed as constituting the
legitime of the executrix-appellee and oppositors-appellants, to be divided among them
in seven equal parts of P129,362.11 as their respective legitimes.
The lower court, after hearing, sustained and approved the executrix' project of
partition, ruling that "(A)rticles 906 and 907 of the New Civil Code specifically provide
that when the legitime is impaired or prejudiced, the same shall be completed and
satisfied. While it is true that this process has been followed and adhered to in the two
projects of partition, it is observed that the executrix and the oppositors differ in respect
to the source from which the portion or portions shall be taken in order to fully restore
the impaired legitime. The proposition of the oppositors, if upheld, will substantially
result in a distribution of intestacy, which is in controversion of Article 791 of the New
Civil Code" adding that "the testatrix has chosen to favor certain heirs in her will for
reasons of her own, cannot be doubted. This is legally permissible within the limitation
of the law, as aforecited." With reference to the payment in cash of some P230,552.38,
principally by the executrix as the largest beneficiary of the will to be paid to her five coheirs, the oppositors (excluding Tomas Dizon), to complete their impaired legitimes, the
lower court ruled that "(T)he payment in cash so as to make the proper adjustment to
meet with the requirements of the law in respect to legitimes which have been impaired
is, in our opinion, a practical and valid solution in order to give effect to the last wishes
of the testatrix."
From the lower court's orders of approval, oppositors-appellants have filed this appeal,
and raise anew the following issues: .
1. Whether or not the testamentary dispositions made in the testatrix' will are in the
nature of devises imputable to the free portion of her estate, and therefore subject to
reduction;

2. Whether the appellants are entitled to the devise plus their legitime under Article
1063, or merely to demand completion of their legitime under Article 906 of the Civil
Code; and
3. Whether the appellants may be compelled to accept payment in cash on account of
their legitime, instead of some of the real properties left by the Testatrix;
which were adversely decided against them in the proceedings below.
The issues raised present a matter of determining the avowed intention of the testatrix
which is "the life and soul of a will." 5 In consonance therewith, our Civil Code included
the new provisions found in Articles 788 and 791 thereof that "(I)f a testamentary
disposition admits of different interpretations, in case of doubt, that interpretation by
which the disposition is to be operative shall be preferred" and "(T)he words of a will
are to receive an interpretation which will give to every expression some effect, rather
than one which will render any of the expressions inoperative; and of two modes of
interpreting a will, that is to be preferred which will prevent intestacy." In Villanueva vs.
Juico 6 for violation of these rules of interpretation as well as of Rule 123, section 59 of
the old Rules of Court, 7 the Court, speaking through Mr. Justice J.B.L. Reyes,
overturned the lower court's decision and stressed that "the intention and wishes of the
testator, when clearly expressed in his will, constitute the fixed law of interpretation, and
all questions raised at the trial, relative to its execution and fulfillment, must be settled
in accordance therewith, following the plain and literal meaning of the testator's words,
unless it clearly appears that his intention was otherwise." 8
The testator's wishes and intention constitute the first and principal law in the matter of
testaments, and to paraphrase an early decision of the Supreme Court of
Spain, 9 when expressed clearly and precisely in his last will amount to the only law
whose mandate must imperatively be faithfully obeyed and complied with by his
executors, heirs and devisees and legatees, and neither these interested parties nor
the courts may substitute their own criterion for the testator's will. Guided and restricted
by these fundamental premises, the Court finds for the appellee.
1. Decisive of the issues at bar is the fact that the testatrix' testamentary disposition
was in the nature of a partition of her estate by will. Thus, in the third paragraph of her
will, after commanding that upon her death all her obligations as well as the expenses
of her last illness and funeral and the expenses for probate of her last will and for the
administration of her property in accordance with law, be paid, she expressly provided
that "it is my wish and I command that my property be divided" in accordance with the

dispositions immediately thereafter following, whereby she specified each real property
in her estate and designated the particular heir among her seven compulsory heirs and
seven other grandchildren to whom she bequeathed the same. This was a valid
partition 10 of her estate, as contemplated and authorized in the first paragraph of Article
1080 of the Civil Code, providing that "(S)hould a person make a partition of his estate
by an act inter vivos or by will, such partition shall be respected, insofar as it does not
prejudice the legitime of the compulsory heirs." This right of a testator to partition his
estate is subject only to the right of compulsory heirs to their legitime. The Civil Code
thus provides the safeguard for the right of such compulsory heirs:
ART. 906. Any compulsory heir to whom the testator has left by any title
less than the legitime belonging to him may demand that the same be
fully satisfied.
ART. 907. Testamentary dispositions that impair or diminish the legitime
of the compulsory heirs shall be reduced on petition of the same, insofar
as they may be inofficious or excessive.
This was properly complied with in the executrix-appellee's project of
partition, wherein the five oppositors-appellants namely Estela,
Bernardita, Angelina, Josefina and Lilia, were adjudicated the properties
respectively distributed and assigned to them by the testatrix in her will,
and the differential to complete their respective legitimes of P129,362.11
each were taken from the cash and/or properties of the executrixappellee, Marina, and their co-oppositor-appellant, Tomas, who
admittedly were favored by the testatrix and received in the partition by
will more than their respective legitimes.
2. This right of a testator to partition his estate by will was recognized even in Article
1056 of the old Civil Code which has been reproduced now as Article 1080 of the
present Civil Code. The only amendment in the provision was that Article 1080 "now
permits any person (not a testator, as under the old law) to partition his estate by
actinter vivos." 11 This was intended to repeal the then prevailing doctrine 12 that for a
testator to partition his estate by an act inter vivos, he must first make a will with all the
formalities provided by law. Authoritative commentators doubt the efficacy of the
amendment 13 but the question does not here concern us, for this is a clear case of
partition by will, duly admitted to probate, which perforce must be given full validity and
effect. Aside from the provisions of Articles 906 and 907 above quoted, other codal
provisions support the executrix-appellee's project of partition as approved by the lower

court rather than the counter-project of partition proposed by oppositors-appellants


whereby they would reduce the testamentary disposition or partition made by the
testatrix to one-half and limit the same, which they would consider as mere devises or
legacies, to one-half of the estate as the disposable free portion, and apply the other
half of the estate to payment of the legitimes of the seven compulsory heirs. Oppositors'
proposal would amount substantially to a distribution by intestacy and pro tanto nullify
the testatrix' will, contrary to Article 791 of the Civil Code. It would further run counter to
the provisions of Article 1091 of the Civil Code that "(A) partition legally made confers
upon each heir the exclusive ownership of the property adjudicated to him."

appear from the whole context of the will and the disposition by the testatrix of her
whole estate (save for some small properties of little value already noted at the
beginning of this opinion) that her clear intention was to partition her whole estate
through her will. The repeated use of the words "I bequeath" in her testamentary
dispositions acquire no legal significance, such as to convert the same into devises to
be taken solely from the free one-half disposable portion of the estate. Furthermore, the
testatrix' intent that her testamentary dispositions were by way of adjudications to the
beneficiaries as heirs and not as mere devisees, and that said dispositions were
therefore on account of the respective legitimes of the compulsory heirs is expressly
borne out in the fourth paragraph of her will, immediately following her testamentary
adjudications in the third paragraph in this wise: "FOURTH: I likewise command that in
3. In Habana vs. Imbo, 14 the Court upheld the distribution made in the will of the
deceased testator Pedro Teves of two large coconut plantations in favor of his daughter, case any of those I named as my heirs in this testament any of them shall die before I
do, his forced heirs under the law enforced at the time of my death shall inherit the
Concepcion, as against adverse claims of other compulsory heirs, as being a partition
properties I bequeath to said deceased." 17
by will, which should be respected insofar as it does not prejudice the legitime of the
compulsory heirs, in accordance with Article 1080 of the Civil Code. In upholding the
sale made by Concepcion to a stranger of the plantations thus partitioned in her favor in Oppositors' conclusions necessarily are in error. The testamentary dispositions of the
the deceased's will which was being questioned by the other compulsory heirs, the
testatrix, being dispositions in favor of compulsory heirs, do not have to be taken only
Court ruled that "Concepcion Teves by operation of law, became the absolute owner of
from the free portion of the estate, as contended, for the second paragraph of Article
said lots because 'A partition legally made confers upon each heir the exclusive
842 of the Civil Code precisely provides that "(O)ne who has compulsory heirsmay
ownership of the property adjudicated to him' (Article 1091, New Civil Code), from the
dispose of his estate provided he does not contravene the provisions of this Code with
death of her ancestors, subject to rights and obligations of the latter, and, she can not
regard to the legitime of said heirs." And even going by oppositors' own theory of
be deprived of her rights thereto except by the methods provided for by law (Arts. 657,
bequests, the second paragraph of Article 912 Civil Code covers precisely the case of
659, and 661, Civil Code). 15 Concepcion Teves could, as she did, sell the lots in
the executrix-appellee, who admittedly was favored by the testatrix with the large bulk
question as part of her share of the proposed partition of the properties, especially
of her estate in providing that "(T)he devisee who is entitled to a legitime may retain the
when, as in the present case, the sale has been expressly recognized by herself and
entire property,provided its value does not exceed that of the disposable portion and of
her co-heirs ..."
the share pertaining to him as legitime." For "diversity of apportionment is the usual
reason for making a testament; otherwise, the decedent might as well die intestate." 18
Fundamentally, of course, the dispositions by the testatrix constituted a partition by will,
4. The burden of oppositors' contention is that the testamentary dispositions in their
which by mandate of Article 1080 of the Civil Code and of the other cited codal
favor are in the nature of devises of real property, citing the testatrix' repeated use of
provisions upholding the primacy of the testator's last will and testament, have to be
the words "I bequeath" in her assignment or distribution of her real properties to the
respected insofar as they do not prejudice the legitime of the other compulsory heirs.
respective heirs. From this erroneous premise, they proceed to the equally erroneous
conclusion that "the legitime of the compulsory heirs passes to them by operation of law
and that the testator can only dispose of the free portion, that is, the remainder of the
Oppositors' invoking of Article 1063 of the Civil Code that "(P)roperty left by will is not
estate after deducting the legitime of the compulsory heirs ... and all testamentary
deemed subject to collation, if the testator has not otherwise provided, but the legitime
dispositions, either in the nature of institution of heirs or of devises or legacies, have to
shall in any case remain unimpaired" and invoking of the construction thereof given by
16
be taken from the remainder of the testator's estate constituting the free portion."
some authorities that "'not deemed subject to collation' in this article really means not
imputable to or chargeable against the legitime", while it may have some
plausibility 19 in an appropriate case, has no application in the present case. Here, we
Oppositors err in their premises, for the adjudications and assignments in the testatrix'
will of specific properties to specific heirs cannot be considered all devises, for it clearly have a case of a distribution and partition of the entire estate by the testatrix, without

her having made any previous donations during her lifetime which would require
collation to determine the legitime of each heir nor having left merely some properties
by will which would call for the application of Articles 1061 to 1063 of the Civil Code on
collation. The amount of the legitime of the heirs is here determined and undisputed.

could have received them earlier, like Bernardita, at the time of approval of the project
of partition and when the peso's purchasing value was higher, is due to their own
decision of pursuing the present appeal.
ACCORDINGLY, the orders appealed from are hereby affirmed. Without cost.

5. With this resolution of the decisive issue raised by oppositors-appellants, the


secondary issues are likewise necessarily resolved. Their right was merely to demand
completion of their legitime under Article 906 of the Civil Code and this has been
complied with in the approved project of partition, and they can no longer demand a
further share from the remaining portion of the estate, as bequeathed and partitioned
by the testatrix principally to the executrix-appellee.
Neither may the appellants legally insist on their legitime being completed with real
properties of the estate instead of being paid in cash, per the approved project of
partition. The properties are not available for the purpose, as the testatrix had
specifically partitioned and distributed them to her heirs, and the heirs are called upon,
as far as feasible to comply with and give effect to the intention of the testatrix as
solemnized in her will, by implementing her manifest wish of transmitting the real
properties intact to her named beneficiaries, principally the executrix-appellee. The
appraisal report of the properties of the estate as filed by the commissioner appointed
by the lower court was approved in toto upon joint petition of the parties, and hence,
there cannot be said to be any question and none is presented as to fairness of
the valuation thereof or that the legitime of the heirs in terms of cash has been
understated. The plaint of oppositors that the purchasing value of the Philippine peso
has greatly declined since the testatrix' death in January, 1961 provides no legal basis
or justification for overturning the wishes and intent of the testatrix. The transmission of
rights to the succession are transmitted from the moment of death of the decedent
(Article 777, Civil Code) and accordingly, the value thereof must be reckoned as of
then, as otherwise, estates would never be settled if there were to be a revaluation with
every subsequent fluctuation in the values of the currency and properties of the estate.
There is evidence in the record that prior to November 25, 1964, one of the oppositors,
Bernardita, accepted the sum of P50,000.00 on account of her inheritance, which, per
the parties' manifestation, 20 "does not in any way affect the adjudication made to her in
the projects of partition of either party as the same is a mere advance of the cash that
she should receive in both projects of partition." The payment in cash by way of making
the proper adjustments in order to meet the requirements of the law on non-impairment
of legitimes as well as to give effect to the last will of the testatrix has invariably been
availed of and sanctioned. 21 That her co-oppositors would receive their cash
differentials only now when the value of the currency has declined further, whereas they

G.R. No. 198994, February 03, 2016


IRIS MORALES, Petitioners, v. ANA MARIA OLONDRIZ, ALFONSO JUAN
OLONDRIZ, JR., ALEJANDRO MORENO OLONDRIZ, ISABEL ROSA OLONDRIZ
AND FRANCISCO JAVIER MARIA OLONDRIZ, Respondent.

OLONDRIZ, (4) ISABEL OLONDRIZ, (5) ANGELO OLONDRIZ, and their


mother (6) MARIA ORTEGAS OLONDRIZ, SR.3
Notably, the will omitted Francisco Javier Maria Bautista Olondriz, an illegitimate son of
the decedent.

DECISION

On September 1, 2003, Morales filed a manifestation in Sp. Proc. Case No. SP-03BRION, J.: 0060 and moved to suspend the intestate proceedings in order to give way to the
probate proceedings in Sp. Proc. Case No. SP-03-0069. The respondent heirs
This is a petition for review on certiorari filed by Iris Morales from the May 27, 2011
opposed Morales' motion for suspension and her petition for allowance of the will.
decision and October 12, 2011 resolution of the Court of Appeals (CA) in CA-G.R. SP
No. 102358.1 The CA denied Morales' petition for certiorari from the Regional Trial
On November 27, 2003, the RTC consolidated Sp. Proc. Case No. SP-03Court's (RTC) July 12, 2007 and October 30, 2007 orders in SP. Proc. No. 03-0060
2
0060 with Sp. Proc. Case No. SP-03-0069.
and SP. Proc. No. 03-0069. chanroblesvirtuallawlibrary
Antecedents
Alfonso Juan P. Olondriz, Sr. (the decedent) died on June 9, 2003. He was survived by
his widow, Ana Maria Ortigas de Olondriz, and his children: Alfonso Juan O. Olondriz,
Jr., Alejandro Marino O. Olondriz, Isabel Rosa O. Olondriz, Angelo Jose O. Olondriz,
and Francisco Javier Maria Bautista Olondriz. His widow and children are collectively
referred to as the respondent heirs.

On January 6, 2004, the respondent heirs moved to dismiss the probate proceedings
because Francisco was preterited from the will.
On January 10, 2006, Morales agreed to the holding of an evidentiary hearing to
resolve the issue of preterition. Thus, the RTC ordered the parties to submit their
factual allegations to support or negate the existence of preterition. Only the
respondent heirs complied with this order.

Believing that the decedent died intestate, the respondent heirs filed a petition with the
Las Pias RTC for the partition of the decedent's estate and the appointment of a
special administrator on July 4, 2003. The case was raffled to Branch 254 and
docketed as Sp. Proc. Case No. SP-03-0060.

After several postponements at the instance of Morales, the reception of evidence for
the evidentiary hearing was scheduled on May 29, 2006. However, Morales failed to
appear, effectively waiving her right to present evidence on the issue of preterition.

On July 11, 2003, the RTC appointed Alfonso Juan O. Olondriz, Jr. as special
administrator.

On June 23, 2006, the RTC, through Judge Gloria Butay Aglugub, suspended the
intestate proceedings in Sp. Proc. Case No. SP-03-0060 and set the case for probate.
The RTC reasoned that probate proceedings take precedence over intestate
proceedings.

However, on July 28, 2003, Iris Morales filed a separate petition with the RTC alleging
that the decedent left a will dated July 23, 1991. Morales prayed for the probate of the
will and for hex appointment as special administratrix. Her petition was also raffled
to Branch 254 and docketed asSp. Proc. Case No. SP-03-0069.
The pertinent portions of the decedent's will reads:chanRoblesvirtualLawlibrary
1. Upon my death, IRIS MORALES OLONDRIZ shall be the executor
hereof and administrator of my estate until its distribution in accordance
herewith, x x x
2. My entire estate shall be divided into six (6) parts to be distributed
equally among and between (1) IRIS MORALES OLONDRIZ, my
children (2) ALFONSO JUAN OLONDRIZ, JR., (3) ALEJANDRO

The respondent heirs moved for reconsideration of the suspension order but the RTC
denied the motion on September 1, 2006. The RTC also summarily revoked the Letters
of Administration previously issued to Alfonso Jr.
The respondent heirs moved for reconsideration of the summary revocation of the
Letters of Administration. They also moved for the inhibition of Judge Aglugub of
Branch 254.
On November 16, 2006, the RTC granted the motion for inhibition. The case was
transferred toBranch 253 presided by Judge Salvador V. Timbang, Jr.
On July 12, 2007, the RTC resolved (1) the respondent heirs' motion for

reconsideration of the revocation of the Letters of Administration and (2) Morales'


motion to be appointed Special Administratrix of the estate. The RTC noted that while
testacy is preferred over intestacy, courts will not hesitate to set aside probate
proceedings if it appears that the probate of the will might become an idle ceremony
because the will is intrinsically void.
The RTC observed: (1) that Morales expressly admitted that Francisco Javier Maria
Bautista Olondriz is an heir of the decedent; (2) that Francisco was clearly omitted from
the will; and (3) that based on the evidentiary hearings, Francisco was clearly
preterited. Thus, the RTC reinstated Alfonso Jr. as administrator of the estate and
ordered the case to proceed in intestacy.
Morales moved for reconsideration which the RTC denied on October 30, 2007, for lack
of merit.
On February 7, 2008, Morales filed a petition for certiorari against the orders of the
RTC. Morales alleged that the RTC acted with grave abuse of discretion in proceeding
intestate despite the existence of the will. The petition was docketed as CA-G.R. SP
No. 102358.
On May 27, 2011, the CA dismissed Morales' petition for certiorari. The CA reasoned
that while probate proceedings take precedence over intestate proceedings, the
preterition of a compulsory heir in the direct line annuls the institution of heirs in the will
and opens the entire inheritance into intestate succession.4 Thus, the continuation of
the probate proceedings would be superfluous and impractical because the inheritance
will be adjudicated intestate. The CA concluded that the RTC did not act with grave
abuse of discretion.
Morales moved for reconsideration which the CA denied on October 12, 2011. Hence,
she filed the present petition for review on certiorari on December 5, 2011.
The Petition
Morales maintains that the RTC committed grave abuse of discretion when it ordered
the case to proceed intestate because: (1) the probate of a decedent's will is
mandatory; (2) the RTC Branch 254 already ordered the case to proceed into probate;
(3) the order setting the case for probate already attained finality; (3) the probate court
cannot touch on the intrinsic validity of the will; and (4) there was no preterition because
Francisco received a house and lot inter vivos as an advance on his legitime.
The respondent heirs counter: (1) that it is within the RTC's jurisdiction to reverse or
modify an interlocutory order setting the case for probate; (2) that the petitioner failed to

mention that she did not appear in any of the evidentiary hearings to disprove their
allegation of preterition; (3) that the RTC and the CA both found that Francisco was
preterited from the will; and (4) that Francisco's preterition annulled the institution of
heirs and opened the case into intestacy. They conclude that the RTC did not exceed
its jurisdiction or act with grave abuse of discretion when it reinstated Alfonso Jr. as the
administrator of the estate and ordered the case to proceed intestate.
Our Ruling
We join the ruling of the CA.
Preterition consists in the omission of a compulsory heir from the will, either because
he is not named or, although he is named as a father, son, etc., he is neither instituted
as an heir nor assigned any part of the estate without expressly being disinherited tacitly depriving the heir of his legitime.5Preterition requires that the omission is total,
meaning the heir did not also receive any legacies, devises, or advances on his
legitime.6
In other words, preterition is the complete and total omission of a compulsory heir from
the testator's inheritance without the heir's express disinheritance.
Article 854 of the Civil Code states the legal effects of
preterition:chanRoblesvirtualLawlibrary
Art. 854. The preterition or omission of one, some, or all of the compulsory heirs in
the direct line, whether living at the time of the execution of the will or born after the
death of the testator, shall annul the institution of heir; but the devises and legacies
shall be valid insofar as they are not inofficious.
If the omitted compulsory heirs should die before the testator, the institution shall be
effectual, without prejudice to the right of representation, (emphasis
supplied)cralawlawlibrary
Under the Civil Code, the preterition of a compulsory heir in the direct line shall annul
the institution of heirs, but the devises and legacies shall remain valid insofar as the
legitimes are not impaired. Consequently, if a will does not institute any devisees or
legatees, the preterition of a compulsory heir in the direct line will result in total
intestacy.7
In the present case, the decedent's will evidently omitted Francisco Olondriz as an heir,
legatee, or devisee. As the decedent's illegitimate son, Francisco is a compulsory heir
in the direct line. Unless Morales could show otherwise, Francisco's omission from the
will leads to the conclusion of his preterition.

During the proceedings in the RTC, Morales had the opportunity to present evidence
that Francisco received donations inter vivos and advances on his legitime from the
decedent. However, Morales did not appear during the hearing dates, effectively
waiving her right to present evidence on the issue. We cannot fault the RTC for
reaching the reasonable conclusion that there was preterition.
We will not entertain the petitioner's factual allegation that Francisco was not preterited
because this Court is not a trier of facts. Furthermore, the CA concurred with the RTC's
conclusion. We see no cogent reason to deviate from the factual findings of the lower
courts.
The remaining question is whether it was proper for the RTC to (1) pass upon the
intrinsic validity of the will during probate proceedings and (2) order the case to proceed
intestate because of preterition.

jurisdiction.16 As discussed, it is well within the jurisdiction of the probate court to pass
upon the intrinsic validity of the will if probate proceedings might become an idle
ceremony due to the nullity of the will.
On the other hand, grave abuse of discretion is the capricious and whimsical exercise
of judgment equivalent to an evasion of positive duty, or a virtual refusal to act at all in
contemplation of the law.17 It is present when power is exercised in a despotic manner
by reason, for instance, of passion and hostility. Morales failed to show that the RTC
acted in such a capricious and despotic manner that would have warranted the CA's
grant of her petition for certiorari. On the contrary, the RTC acted appropriately in
accordance with the law and jurisprudence.cralaw-red
WHEREFORE, the petition is DISMISSED. Costs against the petitioner.
SO ORDERED.

The general rule is that in probate proceedings, the scope of the court's inquiry is
limited to questions on the extrinsic validity of the will; the probate court will only
determine the will's formal validity and due execution.8 However, this rule is not
inflexible and absolute.9 It is not beyond the probate court's jurisdiction to pass upon the
intrinsic validity of the will when so warranted by exceptional circumstances.10 When
practical considerations demand that the intrinsic validity of the will be passed upon
even before it is probated, the probate court should meet the issue.11
The decedent's will does not contain specific legacies or devices and Francisco's
preterition annulled the institution of heirs. The annulment effectively caused the total
abrogation of the will, resulting in total intestacy of the inheritance.12 The decedent's
will, no matter how valid it may appear extrinsically, is null and void. The conduct of
separate proceedings to determine the intrinsic validity of its testamentary provisions
would be superfluous. Thus, we cannot attribute error - much less grave abuse of
discretion - on the RTC for ordering the case to proceed intestate.
Finally, there is no merit in the petitioner's argument that the previous order setting the
case for probate barred the RTC from ordering the case to proceed intestate. The
disputed order is merely interlocutory and can never become final and executory in the
same manner that a final judgment does.13 An interlocutory order does not result in res
judicata.14 It remains under the control of the court and can be modified or rescinded at
any time before final judgment.15
Certiorari is a limited form of review confined to errors of jurisdiction. An error of
jurisdiction is one where the officer or tribunal acted without or in excess of its
jurisdiction, or with grave abuse of discretion amounting to lack or excess of

G.R. Nos. 140371-72

November 27, 2006

DY YIENG SEANGIO, BARBARA D. SEANGIO and VIRGINIA D.


SEANGIO, Petitioners, vs. HON. AMOR A. REYES, in her capacity as Presiding
Judge, Regional Trial Court, National Capital Judicial Region, Branch 21, Manila,
ALFREDO D. SEANGIO, ALBERTO D. SEANGIO, ELISA D. SEANGIO-SANTOS,
VICTOR D. SEANGIO, ALFONSO D. SEANGIO, SHIRLEY D. SEANGIO-LIM, BETTY
D. SEANGIO-OBAS and JAMES D. SEANGIO, Respondents.
DECISION
AZCUNA, J.:
This is a petition for certiorari1 with application for the issuance of a writ of preliminary
injunction and/or temporary restraining order seeking the nullification of the orders,
dated August 10, 1999 and October 14, 1999, of the Regional Trial Court of Manila,
Branch 21 (the RTC), dismissing the petition for probate on the ground of preterition, in
the consolidated cases, docketed as SP. Proc. No. 98-90870 and SP. Proc. No. 9993396, and entitled, "In the Matter of the Intestate Estate of Segundo C. Seangio v.
Alfredo D. Seangio, et al." and "In the Matter of the Probate of the Will of Segundo C.
Seangio v. Dy Yieng Seangio, Barbara D. Seangio and Virginia Seangio."
The facts of the cases are as follows:
On September 21, 1988, private respondents filed a petition for the settlement of the
intestate estate of the late Segundo Seangio, docketed as Sp. Proc. No. 9890870 of
the RTC, and praying for the appointment of private respondent Elisa D. Seangio
Santos as special administrator and guardian ad litem of petitioner Dy Yieng Seangio.
Petitioners Dy Yieng, Barbara and Virginia, all surnamed Seangio, opposed the petition.
They contended that: 1) Dy Yieng is still very healthy and in full command of her
faculties; 2) the deceased Segundo executed a general power of attorney in favor of
Virginia giving her the power to manage and exercise control and supervision over his
business in the Philippines; 3) Virginia is the most competent and qualified to serve as
the administrator of the estate of Segundo because she is a certified public accountant;
and, 4) Segundo left a holographic will, dated September 20, 1995, disinheriting one of
the private respondents, Alfredo Seangio, for cause. In view of the purported
holographic will, petitioners averred that in the event the decedent is found to have left

a will, the intestate proceedings are to be automatically suspended and replaced by the
proceedings for the probate of the will.

(signed)
Segundo Seangio

On April 7, 1999, a petition for the probate of the holographic will of Segundo, docketed
as SP. Proc. No. 9993396, was filed by petitioners before the RTC. They likewise
reiterated that the probate proceedings should take precedence over SP. Proc. No. 98
90870 because testate proceedings take precedence and enjoy priority over intestate
proceedings.2

Nilagdaan sa harap namin


(signed)
Dy Yieng Seangio (signed)

The document that petitioners refer to as Segundos holographic will is quoted, as


follows:

Unang Saksi ikalawang saksi

Kasulatan sa pag-aalis ng mana

(signed)

Tantunin ng sinuman

ikatlong saksi

Ako si Segundo Seangio Filipino may asawa naninirahan sa 465-A Flores St., Ermita,
Manila at nagtatalay ng maiwanag na pag-iisip at disposisyon ay tahasan at hayagang
inaalisan ko ng lahat at anumang mana ang paganay kong anak na si Alfredo Seangio
dahil siya ay naging lapastangan sa akin at isan beses siya ng sasalita ng masama
harapan ko at mga kapatid niya na si Virginia Seangio labis kong kinasama ng loob ko
at sasabe rin ni Alfredo sa akin na ako nasa ibabaw gayon gunit daratin ang araw na
ako nasa ilalim siya at siya nasa ibabaw.

On May 29, 1999, upon petitioners motion, SP. Proc. No. 9890870 and SP. Proc. No.
9993396 were consolidated.4

Labis kong ikinasama ng loob ko ang gamit ni Alfredo ng akin pagalan para
makapagutang na kuarta siya at kanya asawa na si Merna de los Reyes sa China
Bangking Corporation na millon pesos at hindi ng babayad at hindi ng babayad ito ay
nagdulot sa aking ng malaking kahihiya sa mga may-ari at stockholders ng China
Banking.
At ikinagalit ko pa rin ang pagkuha ni Alfredo at ng kanyang asawa na mga custome[r]
ng Travel Center of the Philippines na pinagasiwaan ko at ng anak ko si Virginia.
Dito ako nagalit din kaya gayon ayoko na bilanin si Alfredo ng anak ko at hayanan kong
inaalisan ng lahat at anoman mana na si Alfredo at si Alfredo Seangio ay hindi ko siya
anak at hindi siya makoha mana.
Nila[g]daan ko ngayon ika 20 ng Setyembre 1995 sa longsod ng Manila sa harap ng
tatlong saksi. 3

On July 1, 1999, private respondents moved for the dismissal of the probate
proceedings5 primarily on the ground that the document purporting to be the
holographic will of Segundo does not contain any disposition of the estate of the
deceased and thus does not meet the definition of a will under Article 783 of the Civil
Code. According to private respondents, the will only shows an alleged act of
disinheritance by the decedent of his eldest son, Alfredo, and nothing else; that all other
compulsory heirs were not named nor instituted as heir, devisee or legatee, hence,
there is preterition which would result to intestacy. Such being the case, private
respondents maintained that while procedurally the court is called upon to rule only on
the extrinsic validity of the will, it is not barred from delving into the intrinsic validity of
the same, and ordering the dismissal of the petition for probate when on the face of the
will it is clear that it contains no testamentary disposition of the property of the
decedent.
Petitioners filed their opposition to the motion to dismiss contending that: 1) generally,
the authority of the probate court is limited only to a determination of the extrinsic
validity of the will; 2) private respondents question the intrinsic and not the extrinsic
validity of the will; 3) disinheritance constitutes a disposition of the estate of a decedent;
and, 4) the rule on preterition does not apply because Segundos will does not
constitute a universal heir or heirs to the exclusion of one or more compulsory heirs.6

On August 10, 1999, the RTC issued its assailed order, dismissing the petition for
probate proceedings:
A perusal of the document termed as "will" by oppositors/petitioners Dy Yieng Seangio,
et al., clearly shows that there is preterition, as the only heirs mentioned thereat are
Alfredo and Virginia. [T]he other heirs being omitted, Article 854 of the New Civil Code
thus applies. However, insofar as the widow Dy Yieng Seangio is concerned, Article
854 does not apply, she not being a compulsory heir in the direct line.
As such, this Court is bound to dismiss this petition, for to do otherwise would amount
to an abuse of discretion. The Supreme Court in the case of Acain v. Intermediate
Appellate Court [155 SCRA 100 (1987)] has made its position clear: "for respondents
to have tolerated the probate of the will and allowed the case to progress when, on its
face, the will appears to be intrinsically void would have been an exercise in futility. It
would have meant a waste of time, effort, expense, plus added futility. The trial court
could have denied its probate outright or could have passed upon the intrinsic validity of
the testamentary provisions before the extrinsic validity of the will was
resolved (underscoring supplied).

THE RESPONDENT JUDGE, WITHOUT EVEN COMPLYING WITH SECTIONS 3 AND


4 OF RULE 76 OF THE RULES OF COURT ON THE PROPER PROCEDURE FOR
SETTING THE CASE FOR INITIAL HEARING FOR THE ESTABLISHMENT OF THE
JURISDICTIONAL FACTS, DISMISSED THE TESTATE CASE ON THE ALLEGED
GROUND THAT THE TESTATORS WILL IS VOID ALLEGEDLY BECAUSE OF THE
EXISTENCE OF PRETERITION, WHICH GOES INTO THE INTRINSIC VALIDITY OF
THE WILL, DESPITE THE FACT THAT IT IS A SETTLED RULE THAT THE
AUTHORITY OF PROBATE COURTS IS LIMITED ONLY TO A DETERMINATION OF
THE EXTRINSIC VALIDITY OF THE WILL, I.E., THE DUE EXECUTION THEREOF,
THE TESTATORS TESTAMENTARY CAPACITY AND THE COMPLIANCE WITH THE
REQUISITES OR SOLEMNITIES PRESCRIBED BY LAW;
II
EVEN ASSUMING ARGUENDO THAT THE RESPONDENT JUDGE HAS THE
AUTHORITY TO RULE UPON THE INTRINSIC VALIDITY OF THE WILL OF THE
TESTATOR, IT IS INDUBITABLE FROM THE FACE OF THE TESTATORS WILL THAT
NO PRETERITON EXISTS AND THAT THE WILL IS BOTH INTRINSICALLY AND
EXTRINSICALLY VALID; AND,

WHEREFORE, premises considered, the Motion to Suspend Proceedings is hereby


DENIED for lack of merit. Special Proceedings No. 9993396 is hereby DISMISSED
without pronouncement as to costs.

III
RESPONDENT JUDGE WAS DUTY BOUND TO SUSPEND THE PROCEEDINGS IN
THE INTESTATE CASE CONSIDERING THAT IT IS A SETTLED RULE THAT
TESTATE PROCEEDINGS TAKE PRECEDENCE OVER INTESTATE PROCEEDINGS.

SO ORDERED.7
Petitioners motion for reconsideration was denied by the RTC in its order dated
October 14, 1999.
Petitioners contend that:
THE RESPONDENT JUDGE ACTED IN EXCESS OF HER JURISDICTION OR WITH
GRAVE ABUSE OF DISCRETION AMOUNTING TO LACK OR EXCESS OF
JURISDICTION AND DECIDED A QUESTION OF LAW NOT IN ACCORD WITH LAW
AND JURISPRUDENCE IN ISSUING THE QUESTIONED ORDERS, DATED 10
AUGUST 1999 AND 14 OCTOBER 1999 (ATTACHMENTS "A" AND "B" HEREOF)
CONSIDERING THAT:
I

Petitioners argue, as follows:


First, respondent judge did not comply with Sections 3 and 4 of Rule 76 of the Rules of
Court which respectively mandate the court to: a) fix the time and place for proving the
will when all concerned may appear to contest the allowance thereof, and cause notice
of such time and place to be published three weeks successively previous to the
appointed time in a newspaper of general circulation; and, b) cause the mailing of said
notice to the heirs, legatees and devisees of the testator Segundo;
Second, the holographic will does not contain any institution of an heir, but rather, as its
title clearly states,Kasulatan ng Pag-Aalis ng Mana, simply contains a disinheritance of
a compulsory heir. Thus, there is no preterition in the decedents will and the
holographic will on its face is not intrinsically void;

Third, the testator intended all his compulsory heirs, petitioners and private
respondents alike, with the sole exception of Alfredo, to inherit his estate. None of the
compulsory heirs in the direct line of Segundo were preterited in the holographic will
since there was no institution of an heir;
Fourth, inasmuch as it clearly appears from the face of the holographic will that it is
both intrinsically and extrinsically valid, respondent judge was mandated to proceed
with the hearing of the testate case; and,
Lastly, the continuation of the proceedings in the intestate case will work injustice to
petitioners, and will render nugatory the disinheritance of Alfredo.

(3) When a child or descendant has been convicted of adultery or concubinage


with the spouse of the testator;
(4) When a child or descendant by fraud, violence, intimidation, or undue
influence causes the testator to make a will or to change one already made;
(5) A refusal without justifiable cause to support the parents or ascendant who
disinherit such child or descendant;
(6) Maltreatment of the testator by word or deed, by the child or descendant;8
(7) When a child or descendant leads a dishonorable or disgraceful life;

The purported holographic will of Segundo that was presented by petitioners was
dated, signed and written by him in his own handwriting. Except on the ground of
preterition, private respondents did not raise any issue as regards the authenticity of
the document.
The document, entitled Kasulatan ng Pag-Aalis ng Mana, unmistakably showed
Segundos intention of excluding his eldest son, Alfredo, as an heir to his estate for the
reasons that he cited therein. In effect, Alfredo was disinherited by Segundo.
For disinheritance to be valid, Article 916 of the Civil Code requires that the same must
be effected through a will wherein the legal cause therefor shall be specified. With
regard to the reasons for the disinheritance that were stated by Segundo in his
document, the Court believes that the incidents, taken as a whole, can be considered a
form of maltreatment of Segundo by his son, Alfredo, and that the matter presents a
sufficient cause for the disinheritance of a child or descendant under Article 919 of the
Civil Code:
Article 919. The following shall be sufficient causes for the disinheritance of children
and descendants, legitimate as well as illegitimate:
(1) When a child or descendant has been found guilty of an attempt against the
life of the testator, his or her spouse, descendants, or ascendants;
(2) When a child or descendant has accused the testator of a crime for which
the law prescribes imprisonment for six years or more, if the accusation has
been found groundless;

(8) Conviction of a crime which carries with it the penalty of civil interdiction.
Now, the critical issue to be determined is whether the document executed by Segundo
can be considered as a holographic will.
A holographic will, as provided under Article 810 of the Civil Code, must be entirely
written, dated, and signed by the hand of the testator himself. It is subject to no other
form, and may be made in or out of the Philippines, and need not be witnessed.
Segundos document, although it may initially come across as a mere disinheritance
instrument, conforms to the formalities of a holographic will prescribed by law. It is
written, dated and signed by the hand of Segundo himself. An intent to dispose mortis
causa[9] can be clearly deduced from the terms of the instrument, and while it does not
make an affirmative disposition of the latters property, the disinheritance of Alfredo,
nonetheless, is an act of disposition in itself. In other words, the disinheritance results in
the disposition of the property of the testator Segundo in favor of those who would
succeed in the absence of Alfredo.10
Moreover, it is a fundamental principle that the intent or the will of the testator,
expressed in the form and within the limits prescribed by law, must be recognized as
the supreme law in succession. All rules of construction are designed to ascertain and
give effect to that intention. It is only when the intention of the testator is contrary to law,
morals, or public policy that it cannot be given effect.11
Holographic wills, therefore, being usually prepared by one who is not learned in the
law, as illustrated in the present case, should be construed more liberally than the ones

drawn by an expert, taking into account the circumstances surrounding the execution of
the instrument and the intention of the testator.12 In this regard, the Court is convinced
that the document, even if captioned as Kasulatan ng Pag-Aalis ng Mana, was intended
by Segundo to be his last testamentary act and was executed by him in accordance
with law in the form of a holographic will. Unless the will is probated,13 the
disinheritance cannot be given effect.14
With regard to the issue on preterition,15 the Court believes that the compulsory heirs in
the direct line were not preterited in the will. It was, in the Courts opinion, Segundos
last expression to bequeath his estate to all his compulsory heirs, with the sole
exception of Alfredo. Also, Segundo did not institute an heir16 to the exclusion of his
other compulsory heirs. The mere mention of the name of one of the petitioners,
Virginia, in the document did not operate to institute her as the universal heir. Her name
was included plainly as a witness to the altercation between Segundo and his son,
Alfredo.1wphi1

G.R. No. 72706 October 27, 1987


CONSTANTINO C. ACAIN, petitioner, vs. HON. INTERMEDIATE APPELLATE
COURT (Third Special Cases Division), VIRGINIA A. FERNANDEZ and ROSA
DIONGSON, respondents.

PARAS, J.:
This is a petition for review on certiorari of the decision * of respondent. Court of
Appeals in AC-G.R. SP No. 05744 promulgated on August 30, 1985 (Rollo, p. 108)
ordering the dismissal of the petition in Special Proceedings No, 591 ACEB and its
Resolution issued on October 23, 1985 (Rollo, p. 72) denying respondents' (petitioners
herein) motion for reconsideration.

Considering that the questioned document is Segundos holographic will, and that the
law favors testacy over intestacy, the probate of the will cannot be dispensed with.
Article 838 of the Civil Code provides that no will shall pass either real or personal
property unless it is proved and allowed in accordance with the Rules of Court. Thus,
unless the will is probated, the right of a person to dispose of his property may be
rendered nugatory.17

The dispositive portion of the questioned decision reads as follows:

In view of the foregoing, the trial court, therefore, should have allowed the holographic
will to be probated. It is settled that testate proceedings for the settlement of the estate
of the decedent take precedence over intestate proceedings for the same purpose.18

The antecedents of the case, based on the summary of the Intermediate Appellate
Court, now Court of Appeals, (Rollo, pp. 108-109) are as follows:

WHEREFORE, the petition is GRANTED. The Orders of the Regional Trial Court of
Manila, Branch 21, dated August 10, 1999 and October 14, 1999, are set aside.
Respondent judge is directed to reinstate and hear SP Proc. No. 99-93396 for the
allowance of the holographic will of Segundo Seangio. The intestate case or SP. Proc.
No. 98-90870 is hereby suspended until the termination of the aforesaid testate
proceedings.
No costs.
SO ORDERED.

WHEREFORE, the petition is hereby granted and respondent Regional


Trial Court of the Seventh Judicial Region, Branch XIII (Cebu City), is
hereby ordered to dismiss the petition in Special Proceedings No. 591
ACEB No special pronouncement is made as to costs.

On May 29, 1984 petitioner Constantino Acain filed on the Regional Trial Court of Cebu
City Branch XIII, a petition for the probate of the will of the late Nemesio Acain and for
the issuance to the same petitioner of letters testamentary, docketed as Special
Proceedings No. 591 ACEB (Rollo, p. 29), on the premise that Nemesio Acain died
leaving a will in which petitioner and his brothers Antonio, Flores and Jose and his
sisters Anita, Concepcion, Quirina and Laura were instituted as heirs. The will allegedly
executed by Nemesio Acain on February 17, 1960 was written in Bisaya (Rollo, p. 27)
with a translation in English (Rollo, p. 31) submi'tted by petitioner without objection
raised by private respondents. The will contained provisions on burial rites, payment of
debts, and the appointment of a certain Atty. Ignacio G. Villagonzalo as the executor of
the testament. On the disposition of the testator's property, the will provided:

THIRD: All my shares that I may receive from our properties. house,
lands and money which I earned jointly with my wife Rosa Diongson
shall all be given by me to my brother SEGUNDO ACAIN Filipino,
widower, of legal age and presently residing at 357-C Sanciangko
Street, Cebu City. In case my brother Segundo Acain pre-deceased me,
all the money properties, lands, houses there in Bantayan and here in
Cebu City which constitute my share shall be given to me to his children,
namely: Anita, Constantino, Concepcion, Quirina, laura, Flores, Antonio
and Jose, all surnamed Acain.
Obviously, Segundo pre-deceased Nemesio. Thus it is the children of Segundo who are
claiming to be heirs, with Constantino as the petitioner in Special Proceedings No. 591
ACEB
After the petition was set for hearing in the lower court on June 25, 1984 the oppositors
(respondents herein Virginia A. Fernandez, a legally adopted daughter of tile deceased
and the latter's widow Rosa Diongson Vda. de Acain filed a motion to dismiss on the
following grounds for the petitioner has no legal capacity to institute these proceedings;
(2) he is merely a universal heir and (3) the widow and the adopted daughter have
been pretirited. (Rollo, p. 158). Said motion was denied by the trial judge.
After the denial of their subsequent motion for reconsideration in the lower court,
respondents filed with the Supreme Court a petition for certiorari and prohibition with
preliminary injunction which was subsequently referred to the Intermediate Appellate
Court by Resolution of the Court dated March 11, 1985 (Memorandum for Petitioner, p.
3; Rollo, p. 159).
Respondent Intermediate Appellate Court granted private respondents' petition and
ordered the trial court to dismiss the petition for the probate of the will of Nemesio Acain
in Special Proceedings No. 591 ACEB
His motion for reconsideration having been denied, petitioner filed this present petition
for the review of respondent Court's decision on December 18, 1985 (Rollo, p. 6).
Respondents' Comment was filed on June 6, 1986 (Rollo, p. 146).

Petitioner raises the following issues (Memorandum for petitioner, p. 4):


(A) The petition filed in AC-G.R. No. 05744 for certiorari and prohibition
with preliminary injunction is not the proper remedy under the premises;
(B) The authority of the probate courts is limited only to inquiring into the
extrinsic validity of the will sought to be probated and it cannot pass
upon the intrinsic validity thereof before it is admitted to probate;
(C) The will of Nemesio Acain is valid and must therefore, be admitted to
probate. The preterition mentioned in Article 854 of the New Civil Code
refers to preterition of "compulsory heirs in the direct line," and does not
apply to private respondents who are not compulsory heirs in the direct
line; their omission shall not annul the institution of heirs;
(D) DICAT TESTATOR ET MERIT LEX. What the testator says will be
the law;
(E) There may be nothing in Article 854 of the New Civil Code, that
suggests that mere institution of a universal heir in the will would give
the heir so instituted a share in the inheritance but there is a definite
distinct intention of the testator in the case at bar, explicitly expressed in
his will. This is what matters and should be in violable.
(F) As an instituted heir, petitioner has the legal interest and standing to
file the petition in Sp. Proc. No. 591 ACEB for probate of the will of
Nemesio Acain and
(G) Article 854 of the New Civil Code is a bill of attainder. It is therefore
unconstitutional and ineffectual.
The pivotal issue in this case is whether or not private respondents have been
pretirited.
Article 854 of the Civil Code provides:

On August 11, 1986 the Court resolved to give due course to the petition (Rollo, p.
153). Respondents' Memorandum was filed on September 22, 1986 (Rollo, p. 157); the
Memorandum for petitioner was filed on September 29, 1986 (Rollo, p. 177).

Art. 854. The preterition or omission of one, some, or all of the


compulsory heirs in the direct line, whether living at the time of the
execution of the will or born after the death of the testator, shall annul

the institution of heir; but the devisees and legacies shall be valid insofar
as they are not; inofficious.
If the omitted compulsory heirs should die before the testator, the
institution shall he effectual, without prejudice to the right of
representation.
Preterition consists in the omission in the testator's will of the forced heirs or anyone of
them either because they are not mentioned therein, or, though mentioned, they are
neither instituted as heirs nor are expressly disinherited (Nuguid v. Nuguid, 17 SCRA
450 [1966]; Maninang v. Court of Appeals, 114 SCRA 478 [1982]). Insofar as the widow
is concerned, Article 854 of the Civil Code may not apply as she does not ascend or
descend from the testator, although she is a compulsory heir. Stated otherwise, even if
the surviving spouse is a compulsory heir, there is no preterition even if she is omitted
from the inheritance, for she is not in the direct line. (Art. 854, Civil code) however, the
same thing cannot be said of the other respondent Virginia A. Fernandez, whose legal
adoption by the testator has not been questioned by petitioner (.Memorandum for the
Petitioner, pp. 8-9). Under Article 39 of P.D. No. 603, known as the Child and Youth
Welfare Code, adoption gives to the adopted person the same rights and duties as if he
were a legitimate child of the adopter and makes the adopted person a legal heir of the
adopter. It cannot be denied that she has totally omitted and preterited in the will of the
testator and that both adopted child and the widow were deprived of at least their
legitime. Neither can it be denied that they were not expressly disinherited. Hence, this
is a clear case of preterition of the legally adopted child.
Pretention annuls the institution of an heir and annulment throws open to intestate
succession the entire inheritance including "la porcion libre (que) no hubiese dispuesto
en virtual de legado mejora o donacion" Maniesa as cited in Nuguid v. Nuguid, supra;
Maninang v. Court of Appeals, 114 SCRA [1982]). The only provisions which do not
result in intestacy are the legacies and devises made in the will for they should stand
valid and respected, except insofar as the legitimes are concerned.
The universal institution of petitioner together with his brothers and sisters to the entire
inheritance of the testator results in totally abrogating the will because the nullification
of such institution of universal heirs-without any other testamentary disposition in the
will-amounts to a declaration that nothing at all was written. Carefully worded and in
clear terms, Article 854 of the Civil Code offers no leeway for inferential interpretation
(Nuguid v. Nuguid), supra. No legacies nor devises having been provided in the will the
whole property of the deceased has been left by universal title to petitioner and his

brothers and sisters. The effect of annulling the "Institution of heirs will be, necessarily,
the opening of a total intestacy (Neri v. Akutin, 74 Phil. 185 [1943]) except that proper
legacies and devises must, as already stated above, be respected.
We now deal with another matter. In order that a person may be allowed to intervene in
a probate proceeding he must have an interest iii the estate, or in the will, or in the
property to be affected by it either as executor or as a claimant of the estate and an
interested party is one who would be benefited by the estate such as an heir or one
who has a claim against the estate like a creditor (Sumilang v. Ramagosa, 21 SCRA
1369/1967). Petitioner is not the appointed executor, neither a devisee or a legatee
there being no mention in the testamentary disposition of any gift of an individual item
of personal or real property he is called upon to receive (Article 782, Civil Code). At the
outset, he appears to have an interest in the will as an heir, defined under Article 782 of
the Civil Code as a person called to the succession either by the provision of a will or
by operation of law. However, intestacy having resulted from the preterition of
respondent adopted child and the universal institution of heirs, petitioner is in effect not
an heir of the testator. He has no legal standing to petition for the probate of the will left
by the deceased and Special Proceedings No. 591 A-CEB must be dismissed.
As a general rule certiorari cannot be a substitute for appeal, except when the
questioned order is an oppressive exercise of j judicial authority (People v. Villanueva,
110 SCRA 465 [1981]; Vda. de Caldito v. Segundo, 117 SCRA 573 [1982]; Co Chuan
Seng v. Court of Appeals, 128 SCRA 308 [1984]; and Bautista v. Sarmiento, 138 SCRA
587 [1985]). It is axiomatic that the remedies of certiorari and prohibition are not
available where the petitioner has the remedy of appeal or some other plain, speedy
and adequate remedy in the course of law (DD Comendador Construction Corporation
v. Sayo (118 SCRA 590 [1982]). They are, however, proper remedies to correct a grave
abuse of discretion of the trial court in not dismissing a case where the dismissal is
founded on valid grounds (Vda. de Bacang v. Court of Appeals, 125 SCRA 137 [1983]).
Special Proceedings No. 591 ACEB is for the probate of a will. As stated by respondent
Court, the general rule is that the probate court's authority is limited only to the extrinsic
validity of the will, the due execution thereof, the testator's testamentary capacity and
the compliance with the requisites or solemnities prescribed by law. The intrinsic validity
of the will normally comes only after the Court has declared that the will has been duly
authenticated. Said court at this stage of the proceedings is not called upon to rule on
the intrinsic validity or efficacy of the provisions of the will (Nuguid v. Nuguid, 17 SCRA
449 [1966]; Sumilang v. Ramagosa, supra; Maninang v. Court of Appeals, 114 SCRA

478 [1982]; Cayetano v. Leonides, 129 SCRA 522 [1984]; and Nepomuceno v. Court of
Appeals, 139 SCRA 206 [1985]).
The rule, however, is not inflexible and absolute. Under exceptional circumstances, the
probate court is not powerless to do what the situation constrains it to do and pass
upon certain provisions of the will (Nepomuceno v. Court of Appeals, supra). In Nuguid
v. Nuguid the oppositors to the probate moved to dismiss on the ground of absolute
preteriton The probate court acting on the motion held that the will in question was a
complete nullity and dismissed the petition without costs. On appeal the Supreme Court
upheld the decision of the probate court, induced by practical considerations. The Court
said:
We pause to reflect. If the case were to be remanded for probate of the
will, nothing will be gained. On the contrary, this litigation will be
protracted. And for aught that appears in the record, in the event of
probate or if the court rejects the will, probability exists that the case will
come up once again before us on the same issue of the intrinsic validity
or nullity of the will. Result: waste of time, effort, expense, plus added
anxiety. These are the practical considerations that induce us to a belief
that we might as well meet head-on the issue of the validity of the
provisions of the will in question. After all there exists a justiciable
controversy crying for solution.
In Saguimsim v. Lindayag (6 SCRA 874 [1962]) the motion to dismiss the petition by the
surviving spouse was grounded on petitioner's lack of legal capacity to institute the
proceedings which was fully substantiated by the evidence during the hearing held in
connection with said motion. The Court upheld the probate court's order of dismissal.
In Cayetano v. Leonides, supra one of the issues raised in the motion to dismiss the
petition deals with the validity of the provisions of the will. Respondent Judge allowed
the probate of the will. The Court held that as on its face the will appeared to have
preterited the petitioner the respondent judge should have denied its probate outright.
Where circumstances demand that intrinsic validity of testamentary provisions be
passed upon even before the extrinsic validity of the will is resolved, the probate court
should meet the issue. (Nepomuceno v. Court of Appeals,supra; Nuguid v.
Nuguid, supra).
In the instant case private respondents filed a motion to dismiss the petition in Sp.
Proceedings No. 591 ACEB of the Regional Trial Court of Cebu on the following

grounds: (1) petitioner has no legal capacity to institute the proceedings; (2) he is
merely a universal heir; and (3) the widow and the adopted daughter have been
preterited (Rollo, p. 158). It was denied by the trial court in an order dated January 21,
1985 for the reason that "the grounds for the motion to dismiss are matters properly to
be resolved after a hearing on the issues in the course of the trial on the merits of the
case (Rollo, p. 32). A subsequent motion for reconsideration was denied by the trial
court on February 15, 1985 (Rollo, p. 109).
For private respondents to have tolerated the probate of the will and allowed the case
to progress when on its face the will appears to be intrinsically void as petitioner and his
brothers and sisters were instituted as universal heirs coupled with the obvious fact that
one of the private respondents had been preterited would have been an exercise in
futility. It would have meant a waste of time, effort, expense, plus added futility. The trial
court could have denied its probate outright or could have passed upon the intrinsic
validity of the testamentary provisions before the extrinsic validity of the will was
resolved (Cayetano v. Leonides, supra; Nuquid v. Nuguid, supra. The remedies of
certiorari and prohibition were properly availed of by private respondents.
Thus, this Court ruled that where the grounds for dismissal are indubitable, the
defendants had the right to resort to the more speedy, and adequate remedies of
certiorari and prohibition to correct a grave abuse of discretion, amounting to lack of
jurisdiction, committed by the trial court in not dismissing the case, (Vda. de Bacang v.
Court of Appeals, supra) and even assuming the existence of the remedy of appeal, the
Court harkens to the rule that in the broader interests of justice, a petition for certiorari
may be entertained, particularly where appeal would not afford speedy and adequate
relief. (Maninang Court of Appeals, supra).
PREMISES CONSIDERED, the petition is hereby DENIED for lack of merit and the
questioned decision of respondent Court of Appeals promulgated on August 30, 1985
and its Resolution dated October 23, 1985 are hereby AFFIRMED.
SO ORDERED.

G.R. No. L-47799

June 13, 1941

Administration of the estate of Agripino Neri y Chavez. ELEUTERIO NERI, ET


AL., petitioners, vs. IGNACIA AKUTIN AND HER CHILDREN, respondents.
Ozamiz & Capistrano for petitioners.
Gullas, Leuterio, Tanner & Laput for respondents.
MORAN, J.:
Agripino Neri y Chavez, who died on December 12, 1931, had by his first marriage six
children named Eleuterio, Agripino, Agapito, Getulia, Rosario and Celerina; and by his
second marriage with Ignacia Akutin, five children named Gracia, Godofredo, Violeta,
Estela Maria, and Emma. Getulia, daughter in the first marriage, died on October 2,
1923, that is, a little less than eight years before the death of said Agripino Neri y
Chavez, and was survived by seven children named Remedios, Encarnacion, Carmen,
Trinidad, Luz, Alberto and Minda. In Agripino Neri's testament, which was admitted to
probate on March 21, 1932, he willed that his children by the first marriage shall have
no longer any participation in his estate, as they had already received their
corresponding shares during his lifetime. At the hearing for the declaration of heirs, the
trial court found, contrary to what the testator had declared in his will, that all his
children by the first and second marriages intestate heirs of the deceased without
prejudice to one-half of the improvements introduced in the properties during the
existence of the last conjugal partnership, which should belong to Ignacia Akutin. The
Court of Appeals affirmed the trial court's decision with the modification that the will was
"valid with respect to the two-thirds part which the testator could freely dispose of. "This
judgment of the Court of Appeals is now sought to be reviewed in this petition
forcertiorari.

Disinheritance made without a statement of the cause, or for a cause the truth
of which, if contradicted, is not proven, ... shall annul the institution of the heir in
so far as it prejudices the person disinherited; but the legacies, betterments, and
other testamentary dispositions, in so far as they do no encroach upon the
legitime, shall be valid.
The appellate court thus seemed to have rested its judgment upon the impression that
the testator had intended to disinherit, though ineffectively, the children of the first
marriage. There is nothing in the will that supports this conclusion. True, the testator
expressly denied them any share in his estate; but the denial was predicated, not upon
the desire to disinherit, but upon the belief, mistaken though it was, that the children by
the first marriage had already received more than their corresponding shares in his
lifetime in the form of advancement. Such belief conclusively negatives all inference as
to any intention to disinherit, unless his statement to that effect is prove to be
deliberately fictitious, a fact not found by the Court of Appeals. The situation
contemplated in the above provision is one in which the purpose to disinherit is clear,
but upon a cause not stated or not proved, a situation which does not obtain in the
instant case.
The Court of Appeals quotes Manresa thus:
En el terreno de los principios, la solucion mas justa del problema que hemos
hecho notar al comentar el articulo, seria distinguir el caso en que el heredero
omitido viviese al otorgarse el testamento, siendo conocida su existencia por el
testador, de aquel en que, o naciese despues, o se ignorase su existencia,
aplicando en el primer caso la doctrina del articulo 851, y en el segundo la del
814. (6 Manresa, 354-355.)

The decisive question here raised is whether, upon the foregoing facts, the omission of
the children of the first marriage annuls the institution of the children of the first
marriage as sole heirs of the testator, or whether the will may be held valid, at least with
respect to one-third of the estate which the testator may dispose of as legacy and to the
other one-third which he may bequeath as betterment, to said children of the second
marriage.

But it must be observed that this opinion is founded on mere principles (en el terreno de
los principios) and not on the express provisions of the law. Manresa himself admits
that according to law, "no existe hoy cuestion alguna en esta materia: la pretericion
produce siempre los mismos efectos, ya se refiera a personas vivas al hacer el
testamento o nacidas despues. Este ultimo grupo solo puede hacer relacion a los
descendientes legitimos, siempre que ademas tengan derecho a legitima." (6 Manresa,
381.)

The Court of Appeals invoked the provisions of article 851 of the Civil Code, which read
in part as follows:

Appellants, on the other hand, maintain that the case is one of voluntary preterition of
four of the children by the first marriage, and of involuntary preterition of the children by

the deceased Getulia, also of the first marriage, and is thus governed by the provisions
of article 814 of the Civil Code, which read in part as follows:
The preterition of one or all of the forced heirs in the direct line, whether living at
the time of the execution of the will or born after the death of the testator, shall
void the institution of heir; but the legacies and betterments shall be valid, in so
far as they are not inofficious.
Preterition consists in the omission in the testator's will of the forced heirs or anyone of
them, either because they are not mentioned therein, or, though mentioned, they are
neither instituted as heirs nor are expressly disinherited.(Cf. 6 Manresa, 346.) In the
instant case, while the children of the first marriage were mentioned in the will, they
were not accorded any share in the heriditary property, without expressly being
disinherited. It is, therefore, a clear case of preterition as contended by appellants. The
omission of the forced heirs or anyone of them, whether voluntary or involuntary, is a
preterition if the purpose to disinherit is not expressly made or is not at least manifest.
Except as to "legacies and betterments" which "shall be valid in so far as they are not
inofficious" (art. 814 of the Civil Code), preterition avoids the institution of heirs and
gives rise to intestate succession. (Art. 814, Civil Code; Decisions of the Supreme
Court of Spain of June 17, 1908 and February 27, 1909.) In the instant case, no such
legacies or betterments have been made by the testator. "Mejoras" or betterments must
be expressly provided, according to articles 825 and 828 of the Civil Code, and where
no express provision therefor is made in the will, the law would presume that the
testator had no intention to that effect. (Cf. 6 Manresa, 479.) In the will here in question,
no express betterment is made in favor of the children by the second marriage; neither
is there any legacy expressly made in their behalf consisting of the third available for
free disposal. The whole inheritance is accorded the heirs by the second marriage upon
the mistaken belief that the heirs by the first marriage have already received their
shares. Were it not for this mistake, the testator's intention, as may be clearly inferred
from his will, would have been to divide his property equally among all his children.
Judgment of the Court of Appeals is reversed and that of the trial court affirmed, without
prejudice to the widow's legal usufruct, with costs against respondents.

G.R. No. L-57848 June 19, 1982


RAFAEL E. MANINANG and SOLEDAD L. MANINANG, petitioners, vs. COURT OF
APPEALS, HON. RICARDO L. PRONOVE, JR., as Judge of the Court of First
Instance of Rizal and BERNARDO S. ASENETA, respondents.

On June 9, 1977, petitioner Soledad Maninang filed a Petition for probate of the Will of
the decedent with the Court of First Instance-Branch IV, Quezon City (Sp. Proc. No. Q23304, hereinafter referred to as the Testate Case).
On July 25, 1977, herein respondent Bernardo Aseneta, who, as the adopted son,
claims to be the sole heir of decedent Clemencia Aseneta, instituted intestate
proceedings with the Court of First Instance-Branch XI, Pasig, Rizal (Sp. Proc. No.
8569, called hereinafter the Intestate Case" for brevity).

MELENCIO-HERRERA, J.:
A Petition to Review the Decision of April 28, 1981 of respondent Appellate Court in CAG.R. No. 12032-R entitled "Rafael E. Maninang and Soledad L. Maninang vs. Hon.
Ricardo Pronove, Judge of the Court of First Instance of Rizal, Pasig, Branch XI, and
Bernardo S. Aseneta".
Pertinent to the controversy are the following antecedental facts:
On May 21, 1977, Clemencia Aseneta, single, died at the Manila Sanitarium Hospital at
age 81. She left a holographic will, the pertinent portions of which are quoted
hereunder:
xxx xxx xxx
It is my will that all my real properties located in Manila, Makati, Quezon
City, Albay and Legaspi City and all my personal properties shagllbe
inherited upon my death by Dra. Soledad L. Maninang with whose family
I have lived continuously for around the last 30 years now. Dra.
Maninang and her husband Pamping have been kind to me. ... I have
found peace and happiness with them even during the time when my
sisters were still alive and especially now when I am now being troubled
by my nephew Bernardo and niece Salvacion. I am not incompetent as
Nonoy would like me to appear. I know what is right and wrong. I can
decide for myself. I do not consider Nonoy as my adopted son. He has
made me do things against my will.
xxx xxx xxx

On December 23, 1977, the Testate and Intestate Cases were ordered consolidated
before Branch XI, presided by respondent Judge.
Respondent Bernardo then filed a Motion to Dismiss the Testate Case on the ground
that the holographic will was null and void because he, as the only compulsory heir,
was preterited and, therefore, intestacy should ensue. In support of said Motion to
Dismiss, respondent Bernardo cited the cases of Neri vs. Akutin (72 Phil. 322); Nuguid
vs. Nuguid (17 SCRA 449), and Ramos vs. Baldovino (2 CA Rep. 2nd, 878). 1
In her Opposition to said Motion to Dismiss, petitioner Soledad averred that it is still the
rule that in a case for probate of a Will, the Court's area of inquiry is limited to an
examination of and resolution on the extrinsic validity of the will; and that respondent
Bernardo was effectively disinherited by the decedent. 2
On September 8, 1980, the lower Court ordered the dismissal of the Testate Case in
this wise:
For reasons stated in the motion to dismiss filed by petitioner Bernardo
S. Aseneta which the Court finds meritorious, the petition for probate of
will filed by Soledad L. Maninang and which was docketed as Sp. Proc.
No. Q-23304 is DISMISSED, without pronouncement as to costs.
On December 19, 1980, the lower Court denied reconsideration for lack of merit and in
the same Order appointed Bernardo as the administrator of the intestate estate of the
deceased Clemencia Aseneta "considering that he is a forced heir of said deceased
while oppositor Soledad Maninang is not, and considering further that Bernardo
Aseneta has not been shown to be unfit to perform the duties of the trust. "

Petitioners Maninang resorted to a certiorari Petition before respondent Court of


Appeals alleging that the lower Court exceeded its jurisdiction in issuing the Orders of
dismissal of the Testate Case (September 8, 1980) and denial of reconsideration
(December 19, 1980).
On April 28, 1981, respondent Court 3 denied certiorari and ruled that the trial Judge's
Order of dismissal was final in nature as it finally disposed of the Testate Case and,
therefore, appeal was the proper remedy, which petitioners failed to avail of.
Continuing, it said that even granting that the lower Court committed errors in issuing
the questioned Orders, those are errors of judgment reviewable only by appeal and not
by Certiorari. 'Thus, this Petition before us.
We find that the Court a quo a quo acted in excess of its jurisdiction when it dismissed
the Testate Case. Generally, the probate of a Will is mandatory.
No will shall pass either real or personal property unless it is proved and
allowed in accordance with the Rules of Court. 4
The law enjoins the probate of the Will and public policy requires it, because unless the
Will is probated and notice thereof given to the whole world, the right of a person to
dispose of his property by Will may be rendered nugatory. 5
Normally, the probate of a Will does not look into its intrinsic validity.
... The authentication of a will decides no other question than such as
touch upon the capacity of the testator and the compliance with those
requisites or solemnities which the law prescribes for the validity of wills.
It does not determine nor even by implication prejudge the validity or
efficiency (sic) of the provisions, these may be impugned as being
vicious or null, notwithstanding its authentication. The que0stions
relating to these points remain entirely unaffected, and may be raised
even after the will has been authenticated .... 6
Opposition to the intrinsic validity or legality of the provisions of the will
cannot be entertained in Probate proceeding because its only purpose is
merely to determine if the will has been executed in accordance with the
requirements of the law. 7

Respondent Bernardo, however, relies on the pronouncement in Nuguid vs. Nuguid 8,


reading:
In a proceeding for the probate of a will, the Court's area of inquiry is
limited to an examination of, and resolution on, the extrinsic validity of
the will, the due execution thereof, the testatrix's testamentary capacity
and the compliance with the requisites or solemnities prescribed by law.
The intrinsic validity of the will normally comes only after the court has
declared that the will has been duly authenticated. However, where
practical considerations demand that the intrinsic validity of the will be
passed upon, even before it is probated, the Court should meet that
issue. (Emphasis supplied)
Our ruling in Balanay vs. Hon. Martinez 9 had a similar thrust:
The trial court acted correctly in passing upon the will's intrinsic validity
even before its formal validity had been established. The probate of a
will might become an Idle ceremony if on its face it appears to be
intrinsically void. Where practical considerations demand that the
intrinsic validity of the will be passed upon, even before it is probated,
the court should meet the issue.
The Nuguid and the Balanay cases provide the exception rather than the rule. The
intrinsic validity of the Wills in those cases was passed upon even before probate
because "practical considerations" so demanded. Moreover, for the parties in
the Nuguid case, the "meat of the controversy" was the intrinsic validity of the Will; in
fact, the parties in that case "shunted aside the question of whether or not the Will
should be allowed probate." Not so in the case before us now where the probate of the
Will is insisted on by petitioners and a resolution on the extrinsic validity of the Will
demanded.
Moreover, in the Nuguid case, this Court ruled that the Will was intrinsically invalid as it
completely preterited the parents of the testator. In the instant case, a crucial issue that
calls for resolution is whether under the terms of the decedent's Will, private respondent
had been preterited or disinherited, and if the latter, whether it was a valid
disinheritance. Preterition and disinheritance are two diverse concepts.
... Preterition "consists in the omission in the testator's will of the forced
heirs or anyone of them, either because they are not mentioned therein,

or, though mentioned, they are neither instituted as heirs nor are
expressly disinherited." (Neri vs. Akutin, 72 Phil. 325). Disinheritance, in
turn, "is a testamentary disposition depriving any compulsory heirs of his
share in the legitimate for a cause authorized by law." (Justice J.B.L.
Reyes and R.C. Puno, "An Outline of Philippine Civil Law", 1956 ed.,
Vol. III, p. 8, citing cases) Disinheritance is always "voluntary", preterition
upon the other hand, is presumed to be "involuntary" (Sanchez Roman,
Estudios de Derecho Civil 2nd edition, Volume 2.o p. 1131). 10
The effects of preterition and disinheritance are also totally different.
... The effects flowing from preterition are totally different from those of
disinheritance. Pretention under Article 854 of the New Civil Code shall
annul the institution of heir. This annulment is in toto, unless in the wail
there are, in addition, testamentary dispositions in the form of devises or
legacies. In ineffective disinheritance under Article 918 of the same
Code, such disinheritance shall also "annul the institution of heirs", but
only "insofar as it may prejudice the person disinherited", which last
phrase was omitted in the case of preterition (III Tolentino, Civil Code of
the Philippines, 1961 Edition, p. 172). Better stated yet, in disinheritance
the nullity is limited to that portion of the estate of which the disinherited
heirs have been illegally deprived. 11
By virtue of the dismissal of the Testate Case, the determination of that controversial
issue has not been thoroughly considered. We gather from the assailed Order of the
trial Court that its conclusion was that respondent Bernardo has been preterited We are
of opinion, however, that from the face of the Will, that conclusion is not indubitable.
As held in the case of Vda. de Precilla vs. Narciso 12
... it is as important a matter of public interest that a purported will is not
denied legalization on dubious grounds. Otherwise, the very institution of
testamentary succession will be shaken to its foundation, ...
Coming now to the procedural aspect, suffice it to state that in view of our finding that
respondent Judge had acted in excess of his jurisdiction in dismissing the Testate
Case, certiorari is a proper remedy. An act done by a Probate Court in excess of its
jurisdiction may be corrected by Certiorari. 13 And even assuming the existence of the
remedy of appeal, we harken to the rule that in the broader interests of justice, a

petition for certiorari may be entertained, particularly where appeal would not afford
speedy and adequate relief.
WHEREFORE, the Decision in question is set aside and the Orders of the Court of First
Instance-Branch XI, Rizal, dated September 8, 1980 and December 19, 1980, are
nullified. Special Proceeding No. Q-23304 is hereby remanded to said Court of First
Instance-Branch XI. Rizal, therein to be reinstated and consolidated with Special
Proceeding No. 8569 for further proceedings.
No pronouncement as to costs.
SO ORDERED.

G.R. No. L-24365

June 30, 1966

IN THE MATTER OF THE INTESTATE ESTATE OF EDWARD E. CHRISTENSEN,


deceased.
ADOLFO C. AZNAR, executor and appellee, vs. MARIA LUCY CHRISTENSEN
DUNCAN, oppositor and appellant.
MARIA HELEN CHRISTENSEN, oppositor and appellee.
J. Salonga and L. M. Abellera for oppositor and appellee.
Carlos Dominguez, Jr. for executor-appellee.
M. R. Sotelo for appellant.
MAKALINTAL, J.:
Edward E. Christensen, a citizen of California with domicile in the Philippines, died
leaving a will executed on March 5, 1951. The will was admitted to probate by the Court
of First Instance of Davao in its decision of February 28, 1954. In that same decision
the court declared that Maria Helen Christensen Garcia (hereinafter referred to as
Helen Garcia) was a natural child of the deceased. The declaration was appealed to
this Court, and was affirmed in its decision of February 14, 1958 (G.R. No. L-11484).

legacies left in favor of certain other persons, which legacies have been duly approved
by the lower court and distributed to the legatees.
The case is once more before us on appeal, this time by Lucy Duncan, on the sole
question of whether the estate, after deducting the legacies, should pertain to her and
to Helen Garcia in equal shares, or whether the inheritance of Lucy Duncan as
instituted heir should be merely reduced to the extent necessary to cover the legitime of
Helen Garcia, equivalent to 1/4 of the entire estate.
The will of Edward E. Christensen contains, among others, the following clauses which
are pertinent to the issue in this case:
3. I declare ... that I have but ONE (1) child, named MARIA LUCY
CHRISTENSEN (Now Mrs. Bernard Daney), who was born in the Philippines
about twenty-eight years ago, who is now residing at No. 665 Rodger Young
Village, Los Angeles, California, U.S.A.
4. I further declare that I now have no living ascendants, and no descendants
except my above-named daughter, MARIA LUCY CHRISTENSEN DANEY.
xxx

In another incident relative to the partition of the deceased's estate, the trial court
approved the project submitted by the executor in accordance with the provisions of the
will, which said court found to be valid under the law of California. Helen Garcia
appealed from the order of approval, and this Court, on January 31, 1963, reversed the
same on the ground that the validity of the provisions of the will should be governed by
Philippine law, and returned the case to the lower court with instructions that the
partition be made as provided by said law (G.R. No. L-16749).
On October 29, 1964, the Court of First Instance of Davao issued an order approving
the project of partition submitted by the executor, dated June 30, 1964, wherein the
properties of the estate were divided equally between Maria Lucy Christensen Duncan
(named in the will as Maria Lucy Christensen Daney, and hereinafter referred to as
merely Lucy Duncan), whom the testator had expressly recognized in his will as his
daughter (natural) and Helen Garcia, who had been judicially declared as such after his
death. The said order was based on the proposition that since Helen Garcia had been
preterited in the will the institution of Lucy Duncan as heir was annulled, and hence the
properties passed to both of them as if the deceased had died intestate, saving only the

xxx

xxx

7. I give, devise, and bequeath unto MARIA HELEN CHRISTENSEN, now


married to Eduardo Garcia, about eighteen years of age and who,
notwithstanding the fact that she was baptized Christensen, is not in any way
related to me, nor has she been at any time adopted by me, and who, from all
information I have now resides in Egpit, Digos, Davao, Philippines, the sum of
THREE THOUSAND SIX HUNDRED PESOS (P3,600.00), Philippine Currency,
the same to be deposited in trust for the said Maria Helen Christensen with the
Davao Branch of the Philippine National Bank, and paid to her at the rate of
One Hundred Pesos (P100.00), Philippine Currency per month until the
principal thereof as well as any interest which may have accrued thereon, is
exhausted.
xxx

xxx

xxx

12. I hereby give, devise and bequeath, unto my well-beloved daughter, the said
MARIA LUCY CHRISTENSEN DANEY (Mrs. Bernard Daney) now residing, as

aforesaid, at No. 665 Rodger Young Village, Los Angeles, California, U.S.A., all
the income from the rest, remainder, and residue of my property and estate,
real, personal and/or mixed, of whatsoever kind or character, and wheresoever
situated, of which I may be possessed at my death and which may have come
to me from any source whatsoever, during her lifetime; Provided, however, that
should the said MARIA LUCY CHRISTENSEN DANEY at anytime prior to her
decease having living issue, then and in that event, the life interest herein given
shall terminate, and if so terminated, then I give, devise, and bequeath to my
daughter, the said MARIA LUCY CHRISTENSEN DANEY the rest, remainder
and residue of my property with the same force and effect as if I had originally
so given, devised and bequeathed it to her; and provided, further, that should
the said MARIA LUCY CHRISTENSEN DANEY die without living issue, then,
and in that event, I give, devise and bequeath all the rest, remainder and
residue of my property one-half (1/2) to my well-beloved sister, Mrs. CARRIE
LOUISE C. BORTON, now residing at No. 2124, Twentieth Street, Bakersfield,
California, U.S.A., and one-half (1/2) to the children of my deceased brother,
JOSEPH C. CHRISTENSEN, namely: Mrs. Carol F. Ruggaver, of Los Angeles,
California, U.S.A., and Joseph Raymond Christensen, of Manhattan Beach,
California, U.S.A., share and share alike, the share of any of the three above
named who may predecease me, to go in equal parts to the descendants of the
deceased; and, provided further, that should my sister Mrs. Carol Louise C.
Borton die before my own decease, then, and in that event, the share of my
estate devised to her herein I give, devise and bequeath to her children,
Elizabeth Borton de Trevio, of Mexico City Mexico; Barbara Borton Philips, of
Bakersfield, California, U.S.A., and Richard Borton, of Bakersfield, California,
U.S.A., or to the heirs of any of them who may die before my own decease,
share and share alike.
The trial court ruled, and appellee now maintains, that there has been preterition of
Helen Garcia, a compulsory heir in the direct line, resulting in the annulment of the
institution of heir pursuant to Article 854 of the Civil Code, which provides:
ART. 854. The preterition or omission of one, some, or all of the compulsory
heirs in the direct line, whether living at the time of the execution of the will or
born after the death of the testator, shall annul the institution of heir; but the
devises and legacies shall be valid insofar as they are not inofficious.
On the other hand, appellant contends that this is not a case of preterition, but is
governed by Article 906 of the Civil Code, which says: "Any compulsory heir to whom

the testator has left by any title less than the legitime belonging to him may demand
that the same be fully satisfied." Appellant also suggests that considering the provisions
of the will whereby the testator expressly denied his relationship with Helen Garcia, but
left to her a legacy nevertheless although less than the amount of her legitime, she was
in effect defectively disinherited within the meaning of Article 918, which reads:
ART. 918. Disinheritance without a specification of the cause, or for a cause the
truth of which, if contradicted, is not proved, or which is not one of those set
forth in this Code, shall annul the institution of heirs insofar as it may prejudice
the person disinherited; but the devices and legacies and other testamentary
dispositions shall be valid to such extent as will not impair the legitimate.
Thus, according to appellant, under both Article 906 and 918, Helen Garcia is entitled
only to her legitime, and not to a share of the estate equal that of Lucy Duncan as if the
succession were intestate.
Article 854 is a reproduction of Article 814 of the Spanish Civil Code; and Article 906 of
Article 815. Commenting on Article 815, Manresa explains:
Como dice Goyena, en el caso de pretericion puede presumirse ignorancia o
falta de memoria en el testador; en el de dejar algo al heredero forzoso
no. Este no se encuentra plivado totalmente de su legitima: ha recibido por
cualquir titulo una porcion de los bienes hereditarios, porcion que no alcanza a
completar la legitima, pero que influeye poderosamente en el animo del
legislador para decidirle a adoptar una solucion bien diferente de la sealada
para el caso de pretericion.
El testador no ha olvidado por completo al heredero forzoso; le ha dejado
bienes; pero haciendo un calculo equivocado, ha repartido en favor de extraos
o en favor de otros legitimarios por via de legado donacion o mejora mayor
cantidad de la que la ley de consentia disponer. El heredero forzoso no puede
perder su legitima, pero tampoco puede pedir mas que la misma. De aqui su
derecho a reclamar solamente lo que le falta; al complemento de la porcion que
forzosamente la corresponde.
... Dejar el testador por cualquier titulo, equivale a disponer en testamento por
titulo de herencia legado o mejora, y en favor de legitimarios, de alguna
cantidad o porcion de bienes menos que la legitima o igual a la misma. Tal
sentido, que es el mas proprio en al articulo 815, no pugna tampoco con la

doctrina de la ley.Cuando en el testamento se deja algo al heredero forzoso, la


pretericion es incompleta: es mas formularia que real. Cuando en el testamento
nada se deja el legitimario, hay verdadera pretericion. (6 Manresa, 7th Ed.,
1951, p. 437.)

el articulo 815 desvanece esta duda. Aquel se ocupa de privacion completa o


total, tacita este, de la privacion parcial. Los efectos deben ser y son, como
veremos completamente distintos (6 Manresa, p. 428.)
La privacion de la legitima puede ser total o parcial.

On the difference between preterition of a compulsory heir and the right to ask for
completion of his legitime, Sanchez Roman says:
La desheredacion, como expresa, es siempre voluntaria; la pretericion puede
serlo pero se presume involuntaria la omision en que consiste en cuanto olvida
o no atiende el testador en su testamento a la satisfaccion del derecho a la
legitima del heredero forzoso preterido, prescindiendo absoluta y totalmente de
el y no mencionandole en ninguna de sus disposiciones testamentarias, o
no instituyendole en parte alguna de la herencia, ni por titulo de heredero ni por
el de legatar o aunque le mencionara o nombrara sin dejarle mas o menos
bienes. Si le dejara algunos, por pocos que sean e insuficientes para cubrir su
legitima, ya no seria caso de pretericion, sino de complemento de aquella. El
primer supuesto o de pretericion se regula por el articulo 814, y produce accion
de nulidad de la institucion de heredero; y el segundo, o de complemento de
legitima por el 815 y solo original la accion ad suplementum, para completar la
legitima. (Sanchez Roman, Tomo VI, Vol. 2, p. 1131.)
Manresa defines preterition as the omission of the heir in the will, either by not naming
him at all or, while mentioning him as father, son, etc., by not instituting him as heir
without disinheriting him expressly, nor assigning to him some part of the properties.
Manresa continues:
Se necesita pues (a) Que la omision se refiera a un heredero forzoso; (b) Que
la omision sea completa; que el heredero forzoso nada reciba en el
testamento.1wph1.t
xxx

xxx

xxx

B. Que la omision sea completa Esta condicion se deduce del mismo Articulo
814 y resulta con evidencia al relacionar este articulo con el 815. El heredero
forzoso a quien el testador deja algo por cualquier titulo en su testamento, no
se halla propiamente omitido pues se le nombra y se le reconoce participacion
en los bienes hereditarios. Podria discutirse en el Articulo 814 si era o no
necesario que se reconociese el derecho del heredero como tal heredero, pero

Privar totalmente de la legitima es negarla en absoluto al legitimario, despojarle


de ella por completo. A este caso se refiere el articulo 814. Privar parcialmente
de la legitima, es menguarla o reducirla dejar al legitimario una porcion, menor
que la que le corresponde. A este caso se refiere el articulo 815. El 813 sienta,
pues, una regla general, y las consecuencias del que brantamiento de esta
regla se determina en los articulos 814 y 815. (6 Manresa p. 418.)
Again Sanchez Roman:
QUE LA OMISSION SEA TOTAL. Aunque el articulo 814 no consigna de
modo expreso esta circunstancia de que la pretericion o falta de mencion e
institucion o disposicion testamentaria a su favor, sea total, completa y
absoluta, asi se deduce de no hacer distincion o salvedad alguna empleandola
en terminos generales; pero sirve a confirmarlo de un modo indudable el
siguiente articulo 815, al decir que el heredero forzoso a quien el testador haya
dejado por cualquier titulo, menos de la legitima que la corresponda, podria
pedir el complemento de la misma, lo cual ya no son el caso ni los efectos de la
pretericion, que anula la institucion, sino simplemente los del suplemento
necesario para cubrir su legitima. (Sanchez Roman Tomo VI, Vol. 2.0 p.
1133.)
The question may be posed: In order that the right of a forced heir may be limited only
to the completion of his legitime (instead of the annulment of the institution of heirs) is it
necessary that what has been left to him in the will "by any title," as by legacy, be
granted to him in his capacity as heir, that is, a titulo de heredero? In other words,
should he be recognized or referred to in the will as heir? This question is pertinent
because in the will of the deceased Edward E. Christensen Helen Garcia is not
mentioned as an heir indeed her status as such is denied but is given a legacy of
P3,600.00.
While the classical view, pursuant to the Roman law, gave an affirmative answer to the
question, according to both Manresa (6 Manresa 7th 3rd. 436) and Sanchez Roman
(Tomo VI, Vol. 2.0 p. 937), that view was changed by Article 645 of the "Proyecto de

Codigo de 1851," later on copied in Article 906 of our own Code. Sanchez Roman, in
the citation given above, comments as follows:
RESPECTO DEL COMPLEMENTO DE LA LEGITIMA. Se inspira el Codigo
en esta materia en la doctrina clasica del Derecho romano y patrio (2); pero con
alguna racional modificacion. Concedian aquellos precedentes legales al
heredero forzoso, a quien no se le dejaba por titulo de tal el completo de su
legitima, la accion para invalidar la institucion hecha en el testamento y
reclamar y obtener aquella mediante el ejercicio de la querella de inoficioso, y
aun cuando resultara favorecido como donotario, por otro titulo que no fuera el
de heredero, sino al honor de que se le privaba no dandole este caracter, y solo
cuando era instituido heredero en parte o cantidad inferior a lo que le
correspondiera por legitima, era cuando bastaba el ejercicio de la accion ad
suplementum para completarla, sin necesidad de anular las otras instituciones
de heredero o demas disposiciones contenidas en el testamento.
El Articulo 851 se aparta de este criterio estricto y se ajusta a la unica
necesidad que le inspira cual es la de que se complete la legitima del heredero
forzoso, a quien por cualquier titulo se haya dejado menos de lo que le
corresponda, y se le otorga tan solo el derecho de pedir el complemento de la
misma sin necesidad de que se anulen las disposiciones testamentarias, que
se reduciran en lo que sean inoficiosas conforme al articulo 817, cuya
interpretacion y sentido tienen ya en su apoyo la sancion de la jurisprudencia
(3); siendo condicion precisa que lo que se hubiere dejado de menos de la
legitima al heredero forzoso, lo haya sido en el testamento, o sea por
disposicion del testador, segun lo revela el texto del articulo, "el heredero
forzoso a quien el testador haya dejado, etc., esto es por titulo de legado o
donacion mortis causa en el testamento y, no fuera de al. (Sanchez Roman,
Tomo VI, Vol. 2.0 p. 937.)
Manresa cites particularly three decisions of the Supreme Court of Spain dated January
16, 1895, May 25, 1917, and April 23, 1932, respectively. In each one of those cases
the testator left to one who was a forced heir a legacy worth less than the legitime, but
without referring to the legatee as an heir or even as a relative, and willed the rest of
the estate to other persons. It was held that Article 815 applied, and the heir could not
ask that the institution of heirs be annulled entirely, but only that the legitime be
completed. (6 Manresa, pp. 438, 441.)

The foregoing solution is indeed more in consonance with the expressed wishes of the
testator in the present case as may be gathered very clearly from the provisions of his
will. He refused to acknowledge Helen Garcia as his natural daughter, and limited her
share to a legacy of P3,600.00. The fact that she was subsequently declared judicially
to possess such status is no reason to assume that had the judicial declaration come
during his lifetime his subjective attitude towards her would have undergone any
change and that he would have willed his estate equally to her and to Lucy Duncan,
who alone was expressly recognized by him.
The decision of this Court in Neri, et al. v. Akutin, 74 Phil. 185, is cited by appellees in
support of their theory of preterition. That decision is not here applicable, because it
referred to a will where "the testator left all his property by universal title to the children
by his second marriage, and (that) without expressly disinheriting the children by his
first marriage, he left nothing to them or, at least, some of them." In the case at bar the
testator did not entirely omit oppositor-appellee Helen Garcia, but left her a legacy of
P3,600.00.
The estate of the deceased Christensen upon his death consisted of 399 shares of
stocks in the Christensen Plantation Company and a certain amount in cash. Onefourth (1/4) of said estate descended to Helen Garcia as her legitime. Since she
became the owner of her share as of the moment of the death of the decedent (Arts.
774, 777, Civil Code), she is entitled to a corresponding portion of all the fruits or
increments thereof subsequently accruing. These include the stock dividends on the
corporate holdings. The contention of Lucy Duncan that all such dividends pertain to
her according to the terms of the will cannot be sustained, for it would in effect impair
the right of ownership of Helen Garcia with respect to her legitime.
One point deserves to be here mentioned, although no reference to it has been made
in the brief for oppositor-appellant. It is the institution of substitute heirs to the estate
bequeathed to Lucy Duncan in the event she should die without living issue. This
substitution results in effect from the fact that under paragraph 12 of the will she is
entitled only to the income from said estate, unless prior to her decease she should
have living issue, in which event she would inherit in full ownership; otherwise the
property will go to the other relatives of the testator named in the will. Without deciding
this, point, since it is not one of the issues raised before us, we might call attention to
the limitations imposed by law upon this kind of substitution, particularly that which says
that it can never burden the legitime (Art. 864 Civil Code), which means that the
legitime must descend to the heir concerned in fee simple.

Wherefore, the order of the trial court dated October 29, 1964, approving the project of
partition as submitted by the executor-appellee, is hereby set aside; and the case is
remanded with instructions to partition the hereditary estate anew as indicated in this
decision, that is, by giving to oppositor-appellee Maria Helen Christensen Garcia no
more than the portion corresponding to her as legitime, equivalent to one-fourth (1/4) of
the hereditary estate, after deducting all debts and charges, which shall not include
those imposed in the will of the decedent, in accordance with Article 908 of the Civil
Code. Costs against appellees in this instance.

G.R. No. L-41971 November 29, 1983


ZONIA ANA T. SOLANO, petitioner, vs. THE COURT OF APPEALS, BIENVENIDO S.
GARCIA, and EMETERIA S. GARCIA, respondents.
Benjamin H. Aquino for petitioner.

In the hearing of May 13, 1970, the Trial Court specified the legal issues to be treated in
the parties' respective Memoranda as: 1) the question of recognition of the GARCIAS;
2) the correct status of ZONIA, and 3) the hereditary share of each of them in view of
the probated Will. 2
On July 14, 1970, the Trial Court, presided by Judge Ezequiel S. Grageda, rendered
judgment the dispositive portion of which decrees: t.hqw

Alfredo Kallos for respondents.

MELENCIO HERRERA, J.:+.wph!1


A Petition for Review on certiorari of the Decision of the then Court of Appeals affirming
the judgment rendered by the former Court of First Instance of Albay, Branch II, in Civil
Case No. 3956, an action for Recognition.
On July 7, 1969, Bienvenido Garcia and Emeteria Garcia (GARCIAS), claiming to be
illegitimate children of Dr. Meliton SOLANO, filed an action for recognition against him.
In his Answer, SOLANO denied paternity. On February 3, 1970, during the pendency of
the suit, SOLANO died. Petitioner ZONIA Ana Solano was ordered substituted for the
DECEDENT as the only surviving heir mentioned in his Last Will and Testament
probated on March 10, 1969, or prior to his death, in Special Proceedings No. 842 of
the same Court. ZONIA entered her formal appearance as a "substitute defendant" on
March 4, 1970 claiming additionally that she was the sole heir of her father, SOLANO,
and asking that she be allowed to assume her duties as executrix of the probated Will
with the least interference from the GARCIAS who were "mere pretenders to be
illegitimate children of SOLANO".
On April 6, 1970, the GARCIAS filed their "Reply to ZONIA's Appearance and
Supplemental Cause of Action" impugning the recognition of ZONIA as an
acknowledged natural child with the prayer that she be declared instead, like them, as
an adulterous child of the DECEDENT. ZONIA did not file any responsive pleading and
the case proceeded to trial. The GARCIAS further moved for the impleading of the
SOLANO estate in addition to ZONIA, which was opposed by the latter, but which the
Trial Court granted in its Order dated April 15, 1970. 1

WHEREFORE, judgment is hereby rendered declaring the plaintiffs


Bienvenido S. Garcia and Emeteria S. Garcia and the defendant Sonia
Ana Tuagnon as the illegitimate children of the late Dr. Meliton Solano
under the class of ADULTEROUS CHILDREN, with all the rights granted
them by law. The institution of Sonia Ana Solano as sole and universal
heir of the said deceased in the will is hereby declared null and void and
the three (3) children shall share equally the estate or one- third (1/3)
each, without prejudice to the legacy given to Trinidad Tuagnon and the
right of any creditors of the estate. No pronouncement as to costs.
Appealed to the Court of Appeals by ZONIA, said Court affirmed the judgment in
toto (CA-G.R. No. 49018).
ZONIA seeks a reversal of that affirmance in this petition, which was given due course.
At the outset, we should state that we are bound by the findings of fact of both the Trial
Court and the Appellate Court, particularly, the finding that the GARCIAS and ZONIA
are, in fact, illegitimate children of the DECEDENT. The oral testimony and the
documentary evidence of record inevitably point to that conclusion, as may be gleaned
from the following background facts: SOLANO, a resident of Tabaco, Albay, married
Pilar Riosa. The latter died. On a world tour he met a French woman, Lilly Gorand, who
became his second wife in 1928. The union was short-lived as she left him in 1929. In
the early part of 1930, SOLANO started having amorous relations with Juana Garcia,
out of which affair was born Bienvenido Garcia on March 24, 1931 (Exhibits "A" & "3");
and on November 3, 1935, Emeteria Garcia was born (Exhibits "B " & "2"). Their birth
certificates and baptismal certificates mention only the mother's name without the
father's name. The facts establish, however, that SOLANO during his lifetime
recognized the GARCIAS as his children by acts of support and provisions for their
education.

In 1935, SOLANO started living with Trinidad Tuagnon. Three children were born out of
this relation but only petitioner ZONIA Ana Tuagnon, born on July 26, 1941, is living. In
her Birth Certificate, her status was listed as "illegitimate"; her mother as Trinidad
Tuagnon; her father as "P.N.C. " (Exhibit "V"), or "padre no conocido".
During the Japanese occupation, SOLANO obtained a divorce from Lilly Gorand on
November 29, 1943 (Exhibits "R-1" and "S-1"). On December 22, 1943, SOLANO and
Trinidad Tuagnon executed an "Escritura de Reconocimiento de Unit Hija Natural"
(Exhibit "Q"; "7"), acknowledging ZONIA as a "natural child" and giving her the right to
use the name ZONIA Ana Solano y Tuagnon. The document was registered with the
Local Civil Registrar on the same date.
On January 18, 1969, SOLANO executed his "Ultima Voluntad y Testamento" (Exhibit
"11"), instituting ZONIA as his universal heir to all his personal and real properties in
Camalig, Tabaco and Malinao, all in the province of Albay, except for five parcels of
land in Bantayan, Tabaco, Albay, which were given to Trinidad Tuagnon in usufruct
Upon SOLANO's petition (Exhibit "10"), the Will was duly probated on March 10, 1969
in Special Proceedings No. 842 of the Court of First Instance of Albay, Branch II, in a
Decision also rendered by Judge Ezequiel S. Grageda (Exhibit "12").
As above stated, these facts are not in question.
Petitioner maintains, however, that: t.hqw
I
The Court of Appeals, as well as the trial Court, acted without jurisdiction
or in excess of jurisdiction in declaring substitute defendant Zonia Ana
Solano, now petitioner, an illegitimate child of the late Dr. Meliton Solano
in an action where private respondents, as plaintiffs in the Court below,
sought recognition as natural children of Dr. Meliton Solano.
II
The Court of Appeals, as well as the trial Court, acted without jurisdiction
or in excess of jurisdiction in ordering the division of the estate of Dr.
Meliton Solano between the petitioner and private respondents, when
said estate is under the jurisdiction and control of the probate Court in
Special Proceedings No. 842.

III
The Court of Appeals, as well as the trial Court, acted without jurisdiction
or in excess of jurisdiction in declaring nun and void the institution of heir
in the last will and testament of Dr. Meliton Solano, which was duly
probated in special proceedings No. 842 of the Court of First Instance of
Albay, and in concluding that total intestacy resulted there from. 3
Directly challenged is the jurisdiction of the lower Court, in an action for recognition: 1)
to declare ZONIA as an illegitimate child of SOLANO; 2) to order the division of the
estate in the same action despite the pendency of Special Proceedings No. 842; and 3)
to declare null and void the institution of heir in the Last Win and Testament of
SOLANO, which was duly probated in the same Special Proceedings No. 842, and
concluding that total intestacy resulted.
It is true that the action below was basically one for recognition. However, upon notice
of SOLANO's death, the Trial Court ordered his substitution by ZONIA, "the only
surviving heir ... as of as of now" 4 In her "Appearance of Substitute Defendant Zonia
Ana T. Solano ... Sole and Universal Heir", ZONIA specifically prayed that she be 6
allowed to assume her duties as executrix and administratrix of the probated will and
testament of the late Dr. Meliton Solano, under Special Proceedings No. 842, which is
already final and executory, with least interference from the plaintiffs (GARCIAS) who
may be classified for the moment as only pretenders to be illegitimate children". In other
words, ZONIA did not only rely upon SOLANO's Answer already of record but asserted
new rights in her capacity as sole and universal heir, "executrix and administratrix, "and
challenged the right of the GARCIAS to recognition. Thus, she was not defending the
case as a mere representative of the deceased but asserted rights and defenses in her
own personal capacity. So it was that the GARCIAS filed a "Reply to Appearance of
ZONIA ... and Supplemental Cause of Action ... "vigorously denying that ZONIA was
SOLANO's sole and universal heir; that ZONIA could not legally be considered as
SOLANO's acknowledged natural child because of a legal impediment; that the
admission to probate of SOLANO's Will was merely conclusive as to its due execution;
that the supposed recognition under a notarial instrument of ZONIA as an
acknowledged natural child was fraudulent and a product of misrepresentation; that
ZONIA's recognition in the Will as an acknowledged natural child is subject to
nullification and that at most ZONIA is, like them, an adulterous child of SOLANO with
Trinidad Tuagnon.

During the trial, the GARCIAS presented evidence to prove their allegations not only in
their main complaint but also in their "Reply to Appearance and Supplemental Cause of
Action". ZONIA presented no objection to the presentation by the GARCIAS of their oral
and documentary evidence and even cross-examined their witnesses. ZONIA, for her
part, presented her own testimonial and documentary evidence, denied the relationship
of the GARCIAS' to SOLANO and presented the notarial recognition in her favor as an
acknowledged natural child by SOLANO and Trinidad Tuagnon (Exhibit "Q"). Thus, as
raised by the parties in their own pleadings and pursuant to their respective evidence
during the trial, the litigation was converted into a contest between the GARCIAS and
ZONIA precisely as to their correct status as heirs and their respective rights as such.
No error was committed by either the Trial Court or the Appellate Court, therefore, in
resolving the issue of ZONIA's status.
ZONIA additionally assails the jurisdiction of the Trial Court in declaring null and void
the institution of heir in SOLANO's will; in concluding that total intestacy resulted
therefrom; and distributing the shares of the parties in SOLANO's estate when said
estate was under the jurisdiction and control of the Probate Court in Special
Proceedings No. 842.
Normally, this would be the general rule. However, a peculiar situation is thrust upon us
here. It should be recalled that SOLANO himself instituted the petition for probate of the
Will during his lifetime. That proceeding was not one to settle the estate of a deceased
person that would be deemed terminated only upon the final distribution of the residue
of the hereditary estate. With the Will allowed to probate, the case would have
terminated except that it appears that the parties, after SOLANO's death, continued to
file pleadings therein. Secondly, upon motion of the GARCIAS, and over the objection
of ZONIA, the Trial Court ordered the impleading of the estate of SOLANO and
proceeded on that basis. In effect, therefore, the two cases were consolidated. The
records further disclose that the action for recognition (Civil Case No. 3956) and Spec.
Procs. No. 842 were pending before the same Branch of the Court and before the
same presiding Judge. Thirdly, it is settled that the allowance of a Will is conclusive only
as to its due execution. 5 A probate decree is not concerned with the intrinsic validity or
legality of the provisions of the Will. 6
Thus, the Trial Court and the Appellate Court had jurisdiction to conclude that, upon the
facts, the GARCIAS and ZONIA were in the same category as illegitimate children; that
ZONIA's acknowledgment as a "natural child" in a notarial document executed by
SOLANO and Trinidad Tuagnon on December 22, 1943 was erroneous because at the
time of her birth in 1941, SOLANO was still married to Lilly Gorand, his divorce having

been obtained only in 1943, and, therefore, did not have the legal capacity to contract
marriage at the time of ZONIA's conception, 7that being compulsory heirs, the
GARCIAS were, in fact, pretended from SOLANO's Last' Will and Testament; and that
as a result of said preterition, the institution of ZONIA as sole heir by SOLANO is null
and void pursuant to Article 854 of the Civil Code. t.hqw
The preterition or omission of one, some, or all of the compulsory heirs
in the direct line, whether living at the time of the execution of the will or
born after the death of the testator, shall annul the institution of heir; but
the devises and legacies shall be valid insofar as they are not inofficious.
... 8
As provided in the foregoing provision, the disposition in the Will giving the usufruct in
favor of Trinidad Tuagnon over the five parcels of land in Bantayan, Tabaco, Albay, is a
legacy, recognized in Article 563 of the Civil Code, 9and should be respected in so far
as it is not inofficious. 10
So also did the Trial Court have jurisdiction in resolving the issue of the hereditary
shares of the GARCIAS and ZONIA. However, contrary to the conclusions of the Courts
below, holding that the entire Will is void and intestacy ensues, the pretention of the
GARCIAS should annul the institution of ZONIA as heir only insofar as the legitime of
the omitted heirs is impaired. The Will, therefore, is valid subject to that limitation. 11 It is
a plain that the intention of the testator was to favor ZONIA with certain portions of his
property, which, under the law, he had a right to dispose of by Will, so that the
disposition in her favor should be upheld as to the one-half (1/2) portion of the property
that the testator could freely dispose of. 12 Since the legitime of illegitimate children
consists of one half (1/2) of the hereditary estate, 13 the GARCIAS and ZONIA each
have a right to participation therein in the proportion of one-third (1/3) each. ZONIA's
hereditary share will, therefore, be 1/2 + (1/3 of 1/2) or 4/6 of the estate, while the
GARCIAS will respectively be entitled to 1/3 of 1/2 or 1/6 of the value of the estate.
As heretofore stated, the usufruct in favor of Trinidad Tuagnon over the properties
indicated in the Will is valid and should be respected.
The case of Nuguid vs. Nuguid, et al., 14 reiterating the ruling in Neri, et al. vs. Akutin, et
al., 15which held that where the institution of a universal heir is null and void due to
pretention, the Will is a complete nullity and intestate succession ensues, is not
applicable herein because in the Nuguid case, only a one-sentence Will was involved
with no other provision except the institution of the sole and universal heir; there was no

specification of individual property; there were no specific legacies or bequests. It was


upon that factual setting that this Court declared: t.hqw
The disputed order, we observe, declares the will in question 'a complete
nullity. Article 854 of the Civil Code in turn merely nullifies 'the institution
of heir'. Considering, however, that the will before us solely provides for
the institution of petitioner as universal heir, and nothing more, the result
is the same. The entire will is null." (at p. 459)
In contrast, in the case at bar, there is a specific bequest or legacy so that Article 854 of
the Civil Code, supra, applies merely annulling the "institution of heir".
Lastly, it should be pointed out that the jurisdiction of the Trial Court and the Appellate
Court was never questioned before either Court. ZONIA herself had gone, without
objection, to trial on the issues raised and as defined by the Trial Court. Neither had
ZONIA assigned lack of jurisdiction of the Trial Court as an error before the Appellate
Court. She should now be held estopped to repudiate that jurisdiction to which she had
voluntarily submitted, after she had received an unfavorable judgment, The leading
case of Tijam vs. Sibonghanoy, 16 on this point, declared: t.hqw
A party cannot invoke the jurisdiction of a court to secure affirmative
relief against his opponent and after failing to obtain such relief,
repudiate or question the same jurisdiction. The question whether the
court has jurisdiction either of the subject matter of the action or of the
parties is not because the judgment or order of the court is valid and
conclusive as an adjudication but for the reason that such practice
cannot be tolerated obviously for reasons of public policy. After
voluntarily submitting a cause and encountering an adverse decision on
the merits, it is too late for the loser to question the jurisdiction or power
of the court.
WHEREFORE, the judgment under review is hereby modified in that the hereditary
share in the estate of the decedent of petitioner Zonia Ana T. Solano is hereby declared
to be (1/2 + (1/3 of 1/2) or 4/6 of said estate, while that of private respondents,
Bienvenido S. Garcia and Emeteria S. Garcia, shall each be (1/3 of 1/2) or (1/6) of the
estate. The usufruct in favor of Trinidad Tuagnon shall be respected. The judgment is
affirmed in all other respects. No costs.
SO ORDERED.1

G.R. No. 179859

August 9, 2010

IN RE: PETITION FOR PROBATE OF LAST WILL AND TESTAMENT OF BASILIO


SANTIAGO,
MA. PILAR SANTIAGO and CLEMENTE SANTIAGO, Petitioners, vs. ZOILO S.
SANTIAGO, FELICIDAD SANTIAGO-RIVERA, HEIRS OF RICARDO SANTIAGO,
HEIRS OF CIPRIANO SANTIAGO, HEIRS OF TOMAS SANTIAGO, Respondents.
FILEMON SOCO, LEONILA SOCO, ANANIAS SOCO, URBANO SOCO,
GERTRUDES SOCO AND HEIRS OF CONSOLACION SOCO, Oppositors.
DECISION
CARPIO MORALES, J.:
Basilio Santiago (Basilio) contracted three marriagesthe first to Bibiana Lopez, the
second to Irene Santiago, and the third to Cecilia Lomotan. Basilio and his first wife
bore two offsprings, Irene and Marta, the mother of herein oppositors Felimon, Leonila,
Consolacion, Ananias, Urbano, and Gertrudes, all surnamed Soco.
Basilio and his second wife had six offsprings, Tomas, Cipriano, Ricardo, respondents
Zoilo and Felicidad, and petitioner Ma. Pilar, all surnamed Santiago.
Basilio and his third wife bore three children, Eugenia herein petitioner Clemente, and
Cleotilde, all surnamed Santiago.1
After Basilio died testate on September 16, 1973, his daughter by the second marriage
petitioner Ma. Pilar filed before the Regional Trial Court (RTC) of Bulacan2 a petition for
the probate of Basilios will, docketed as SP No. 1549-M. The will was admitted to
probate by Branch 10 of the RTC and Ma. Pilar was appointed executrix.
The will contained the following provisions, among others:
4. Ang mga ari-arian ko na nasasaysay sa itaas ay INIWAN, IPINAGKAKALOOB,
IBINIBIGAY, at IPINAMAMANA ko sa aking mga nasabing tagapagmana sa ilalim ng
gaya ng sumusunod:
xxxx

c) ang aking anak na si Ma. Pilar ang magpapalakad at mamamahala ng


balutan na nasa Santiago, Malolos, Bulacan, na nasasaysay sa itaas na 2(y);
d) Sa pamamahala ng bigasan, pagawaan ng pagkain ng hayop at lupat bahay
sa Maynila, ang lahat ng solar sa danay ng daang Malolos-Paombong na nasa
Malolos, Bulacan, kasali at kasama ang palaisdaan na nasa likuran niyon, ay
ililipat sa pangalan nila Ma. Pilar at Clemente; ngunit ang kita ng palaisdaan ay
siyang gagamitin nila sa lahat at anomang kailangang gugol, maging majora o
roperacion [sic], sa lupat bahay sa Lunsod ng Maynila na nasasaysay sa itaas
na 2(c);
e) Ang lupat bahay sa Lunsod ng Maynila na nasasaysay sa itaas na 2(c)
ay ililipat at ilalagay sa pangalan nila Ma. Pilar at Clemente hindi bilang pamana
ko sa kanila kundi upang pamahalaan at pangalagaan lamang nila at nang ang
sinoman sa aking mga anak sampu ng apo at kaapuapuhan ko sa habang
panahon ay may tutuluyan kung magnanais na mag-aral sa Maynila o kalapit na
mga lunsod x x x.
f) Ang bigasan, mga makina at pagawaan ng pagkain ng hayop ay
ipinamamana ko sa aking asawa, Cecilia Lomotan, at mga anak na Zoilo, Ma.
Pilar, Ricardo, Cipriano, Felicidad, Eugenia, Clemente, at Cleotilde nang parepareho. Ngunit, sa loob ng dalawampong (20) taon mula sa araw ng aking
kamatayan, hindi nila papartihin ito at pamamahalaan ito ni Clemente at ang
maghahawak ng salaping kikitain ay si Ma. Pilar na siyang magpaparte. Ang
papartihin lamang ay ang kita ng mga iyon matapos na ang gugol na
kakailanganin niyon, bilang reparacion, pagpapalit o pagpapalaki ay maawas
na. Ninais ko ang ganito sa aking pagmamahal sa kanila at pagaaring
ibinubuhay ko sa kanila lahat, bukod sa yaon ay sa kanila ding kapakinabangan
at kabutihan.
g) Ang lahat ng lupa, liban sa lupat bahay sa Lunsod ng Maynila, ay
ipinapamana ko sa aking nasabing asawa, Cecilia Lomotan, at mga anak na
Tomas, Zoilo, Ma. Pilar, Ricardo, Cipriano, Felicidad, Eugenia, Clemente at
Cleotilde nang pare-pareho. Datapwat, gaya din ng mga bigasan, makina at
gawaan ng pagkain ng hayop, ito ay hindi papartihin sa loob ng dalawampong
(20) taon mula sa aking pagpanaw, at pamamahalaan din nila Ma. Pilar at
Clemente. Ang mapaparte lamang ay ang kita o ani ng nasabing mga pag-aari
matapos bayaran ang buwis at/o patubig at iba pang mga gugol na kailangan.

Si Ma. Pilar din ang hahawak ng ani o salaping manggagaling dito. (emphasis
and underscoring supplied)3
The oppositors-children of Marta, a daughter of Basilio and his first wife, were, on their
motion, allowed to intervene.4
After the executrix-petitioner Ma. Pilar filed a "Final Accounting, Partition and
Distribution in Accordance with the Will,"5 the probate court approved the will by Order
of August 14, 1978 and directed the registers of deeds of Bulacan and Manila to
register the certificates of title indicated therein.6 Accordingly, the titles to Lot Nos. 786,
837, 7922, 836 and 838 in Malolos, Bulacan and Lot No. 8-C in Manila were transferred
in the name of petitioners Ma. Pilar and Clemente.7
The oppositors thereafter filed a Complaint-in-Intervention8 with the probate court,
alleging that Basilios second wife was not Irene but a certain Maria Arellano with whom
he had no child; and that Basilios will violates Articles 979-981 of the Civil Code.9
The probate court dismissed the Complaint-in-Intervention, citing its previous approval
of the "Final Accounting, Partition, and Distribution in Accordance with the Will."10
The oppositors-heirs of the first marriage thereupon filed a complaint for completion of
legitime before the Bulacan RTC, docketed as Civil Case No. 562-M-90,11 against the
heirs of the second and third marriages.
In their complaint, oppositors-heirs of the first marriage essentially maintained that they
were partially preterited by Basilios will because their legitime was reduced.12 They
thus prayed, inter alia, that an inventory and appraisal of all the properties of Basilio be
conducted and that Ma. Pilar and Clemente be required to submit a fresh accounting of
all the incomes of the properties from the time of Basilios death up to the time of the
filing of Civil Case No. 562-M-90.13
RTC-Branch 17 decided Civil Case No. 562-M-90 (for completion of legitime) in favor of
the oppositors-heirs of the first marriage.

WHEREFORE, premises considered, the Appeal is hereby GRANTED. The Decision in


Civil Case No. 562-M-90 is hereby ANNULLED on the ground of res judicata. Let the
Decree of Distribution of the Estate of Basilio Santiago remain UNDISTURBED.
SO ORDERED.16 (emphasis in the original; underscoring supplied)
Oppositors-heirs of the first marriage challenged the appellate courts decision in CA
G.R. No. 45801 by petition for review, docketed as G.R. No. 155606, which this Court
denied.17 The denial became final and executory on April 9, 2003.18
In the interregnum, or on October 17, 2000, respondent-heirs of the second marriage
filed before the probate court (RTC-Branch 10) a Motion for Termination of
Administration, for Accounting, and for Transfer of Titles in the Names of the
Legatees.19 Citing the earlier quoted portions of Basilios will, they alleged that:
x x x x the twenty (20) year period within which subject properties should be under
administration of [Ma.] Pilar Santiago and Clemente Santiago expired on September
16, 1993.
Consequently, [Ma.] Pilar Santiago and Clemente Santiago should have ceased as
such administrator[s] way back on September 16, 1993 and they should have
transferred the above said titles to the named legatees in the Last Will and Testament
of the testator by then. Said named legatees in the Last Will and Testament are no[ne]
other than the following:
xxxx
Said [Ma.] Pilar Santiago and Clemente Santiago should have also rendered an
accounting of their administration from such death of the testator up to the present or
until transfer of said properties and its administration to the said legatees.
x x x x20
Respondents prayed that petitioners be ordered:

On appeal (docketed as CA G.R. No. 45801), the Court of Appeals, by Decision of


January 25, 2002,14 annulled the decision of RTC-Branch 17, holding that the RTC
Branch 17 dismissal of the Complaint-in-Intervention in SP No. 1549-M and its August
14, 1978 Order approving the probate of the will constitute res judicata with respect to
Civil Case No. 562-M-90.15 Thus the appellate court disposed:

1) To surrender the above-enumerated titles presently in their names to [the]


Honorable Court and to transfer the same in the names of the designated
legatees in the Last Will and Testament, to wit:

1) asawa, Cecilia Lomotan, at mga anak na


2) Tomas
3) Zoilo
4) Ma. Pilar
5) Ricardo
6) Cipriano
7) Felicidad
8) Eugenia
9) Clemente at
10) Cleotilde
(all surnamed SANTIAGO)
2) To peacefully surrender possession and administration of subject properties,
including any and all improvements thereon, to said legatees.
3) To render an accounting of their administration of said properties and other
properties of the testator under their administration, from death of testator
Basilio Santiago on September 16, 1973 up to the present and until possession
and administration thereof is transferred to said legatees.21
Opposing the motion, petitioners argued that with the approval of the Final Accounting,
Partition and Distribution in Accordance with the Will, and with the subsequent issuance
of certificates of title covering the properties involved, the case had long since been
closed and terminated.22
The probate court, finding that the properties in question would be transferred to
petitioners Ma. Pilar and Clemente for purposes of administration only, granted the
motion, by Order of September 5, 2003,23 disposing as follows:

WHEREFORE, premises considered, the Motion for Termination of Administration, for


Accounting, and for Transfer of Titles in the Names of the Legatees dated October 3,
2000 filed by some heirs of the testator Basilio Santiago xxx is hereby GRANTED.
Accordingly, the administratrix [sic] Ma. Pilar Santiago and Mr. Clemente Santiago are
hereby DIRECTED, as follows:
a.) To surrender the above-enumerated titles presently in their names to this
Honorable Court and to transfer the same in the names of the designated
legatees in the Last Will and Testament, to wit: 1.) asawa, Cecilia Lomotan at
mga anak na 2.) Tomas 3). Zoilo 4.) Ma. Pilar 5.) Ricardo 6.) Cipriano 7.)
Felicidad 8.) Eugenia 9.) Clemente and 10.) Cleotilde all named SANTIAGO.
b.) To peacefully surrender possession and administration of subject properties
including any and all improvements thereon, to said legatees; and
c.) To render an accounting of their administration of subject properties,
including any and all improvements thereon, to said legatees; and
d.) To submit an accounting of their administration of the above-mentioned
estate of the testator or all the above said lots including the rice mill, animal
feeds factory, and all improvements thereon from August 14, 1978 up to the
present.
e.) To submit a proposed Project of Partition, indicating how the parties may
actually partition or adjudicate all the above said properties including the
properties already in the name of all the said legatees xxx.
x x x x.
Further, the Register of Deeds of Bulacan are hereby DIRECTED to cancel and
consider as no force and effects Transfer Certificates of Title Nos. T-249177 (RT-46294)
[Lot No. 786], T-249175 (RT-46295) [Lot No. 837], T-249174 (RT-46296) [Lot No. 7922],
T-249173 (RT-46297) [Lot No. 836], and T-249176 (RT-46293) [Lot No. 838] in the
names of Ma. Pilar Santiago and Clemente Santiago and to issue new ones in the lieu
thereof in the names of Cecilia Lomotan-Santiago, Tomas Santiago, Zoilo Santiago,
Ma. Pilar Santiago, Ricardo Santiago, Cipriano Santiago, Felicidad Santiago, Eugenia
Santiago, Clemente Santiago, and Cleotilde Santiago.

Moreover, the Register of Deeds of Manila is hereby DIRECTED to cancel and consider
as no force and effect Transfer Certificate of Title No. 131044 [Lot No. 8-C] in the
names of Ma. Pilar Santiago and Clemente Santiago and to issue new ones in lieu
thereof in the names of the Heirs of Bibiana Lopez, the Heirs of Irene Santiago, and the
Heirs of Cecilia Lomotan.

Petitioners, together with the oppositors, filed a motion for reconsideration,26 which the
probate court denied, drawing them to appeal to the Court of Appeals which docketed it
as CA G.R. No. 83094.
The Court of Appeals affirmed the decision of the probate court,27 hence, the
petition28 which raises the following grounds:

The Motion to Suspend Proceedings filed by Filemon, Leonila, Ma. Concepcion,


Ananias, Urbano and Gertrudes, all surnamed Soco, dated December 3, 2002, is
hereby DENIED for lack of merit.24

I.
"CAN THE HONORABLE COURT OF APPEALS REVERSE ITSELF"

Respecting petitioners argument that the case had long been closed and terminated,
the trial court held:
x x x x [I]t is clear from the Last Will and Testament that subject properties cannot
actually be partitioned until after 20 years from the death of the testator Basilio
Santiago x x x x. It is, therefore, clear that something more has to be done after the
approval of said Final Accounting, Partition, and Distribution. The testator Basilio
Santiago died on September 16, 1973, hence, the present action can only be filed after
September 16, 1993. Movants cause of action accrues only from the said date and for
which no prescription of action has set in.
The principle of res judicata does not apply in the present probate proceeding which is
continuing in character, and terminates only after and until the final distribution or
settlement of the whole estate of the deceased in accordance with the provision of the
will of the testator. The Order dated August 14, 1978 refers only to the accounting,
partition, and distribution of the estate of the deceased for the period covering from the
date of the filing of the petition for probate on December 27, 1973 up to August 14,
1978. And in the said August 14, 1978 order it does not terminate the appointment of
petitioner[s] Ma. Pilar Santiago and Clemente Santiago as executrix and administrator,
respectively, of the estate of the deceased particularly of those properties which were
prohibited by the testator to be partitioned within 20 years from his death. Since then up
to the present, Ma. Pilar Santiago and Clemente Santiago remain the executor and
administrator of the estate of the deceased and as such, they are required by law to
render an accounting thereof from August 14, 1978 up to the present; there is also now
a need to partition and distribute the aforesaid properties as the prohibition period to do
so has elapsed. (emphasis and underscoring supplied)25

A. THE COURT OF APPEALS ERRED IN NOT BINDING ITSELF WITH ITS


PREVIOUS DECISION INVOLVING THE SAME PARTIES AND SAME
PROPERTIES;
B. THE COURT OF APPEALS ERRED IN AFFIRMING THE RTC AS IT
AGREED WITH THE RTC THAT THIS CASE IS NOT BARRED BY RES
JUDICATA;
C. IN C.A.-G.R. NO. 45801, THE HONORABLE COURT OF APPEALS HELD
THAT THERE WAS RES JUDICATA; IN C.A.-G.R. CV NO. 83094, THERE WAS
NO RES JUDICATA.
II.
"GRANTING THAT THE COURT OF APPEALS HAS ALL THE COMPETENCE AND
JURISDICTION TO REVERSE ITSELF, STILL THE COURT OF APPEALS ERRED IN
AFFIRMING THE RTCS ORDER TO TRANSFER THE MANILA PROPERTY
COVERED BY TCT NO. 131004 TO THE NAMES OF CECILIA LOMOTAN, TOMAS,
ZOILO, MA. PILAR, RICARDO, CIPRIANO FELICIDAD, EUGENIA, CLEMENTE AND
CLEOTILDE, ALL SURNAMED SANTIAGO."29 (emphasis in the original)
The petition lacks merit.
Petitioners argument that the decision of the appellate court in the earlier CA-G.R. NO.
45801 (upheld by this Court in G.R. No. 155606) constitutes res judicata to the
subsequent CA G.R. No. 83094 (the subject of the present petition for review) fails.

Res judicata has two aspects, which are embodied in Sections 47 (b) and 47 (c) of Rule
39 of the Rules of Civil Procedure.30 The first, known as "bar by prior judgment,"
proscribes the prosecution of a second action upon the same claim, demand or cause
of action already settled in a prior action.31 The second, known as "conclusiveness of
judgment," ordains that issues actually and directly resolved in a former suit cannot
again be raised in any future case between the same parties involving a different cause
of action.32
Both aspects of res judicata, however, do not find application in the present case. The
final judgment regarding oppositors complaint on the reduction of their legitime in CAG.R. NO. 45801 does not dent the present petition, which solely tackles the propriety of
the termination of administration, accounting and transfer of titles in the names of the
legatees-heirs of the second and third marriages. There is clearly no similarity of claim,
demand or cause of action between the present petition and G.R. No. 155606.
While as between the two cases there is identity of parties, "conclusiveness of
judgment" cannot likewise be invoked. Again, the judgment in G.R. No. 155606 would
only serve as an estoppel as regards the issue on oppositors supposed preterition and
reduction of legitime, which issue is not even a subject, or at the very least even
invoked, in the present petition.
What is clear is that petitioners can invoke res judicata insofar as the judgment in G.R.
No. 155606 is concerned against the oppositors only. The records reveal, however, that
the oppositors did not appeal the decision of the appellate court in this case and were
only impleaded pro forma parties.
Apparently, petitioners emphasize on the directive of the appellate court in CA G.R. No.
45801 that the decree of distribution of the estate of Basilio should remain undisturbed.
But this directive goes only so far as to prohibit the interference of the oppositors in the
distribution of Basilios estate and does not pertain to respondents supervening right to
demand the termination of administration, accounting and transfer of titles in their
names.
Thus, the Order of September 5, 2003 by the probate court granting respondents
Motion for Termination of Administration, for Accounting, and for Transfer of Titles in the
Names of the Legatees is a proper and necessary continuation of the August 14, 1978
Order that approved the accounting, partition and distribution of Basilios estate. As did
the appellate court, the Court notes that the August 14, 1978 Order was yet to become
final pending the whole settlement of the estate. And final settlement of the estate, in

this case, would culminate after 20 years or on September 16, 1993, when the
prohibition to partition the properties of the decedent would be lifted.
Finally, petitioners object to the inclusion of the house and lot in Manila, covered by
TCT No. 131044, among those to be transferred to the legatees-heirs as it would
contravene the testators intent that no one is to own the same.1avvphi1
The Court is not persuaded. It is clear from Basilios will that he intended the house and
lot in Manila to be transferred in petitioners names for administration purposes only,
and that the property be owned by the heirs in common, thus:
e) Ang lupat bahay sa Lunsod ng Maynila na nasasaysay sa itaas na 2(c) ay
ililipat at ilalagay sa pangalan nila Ma. Pilar at Clemente hindi bilang pamana ko
sa kanila kundi upang pamahalaan at pangalagaan lamang nila at nang ang
sinoman sa aking mga anak sampu ng apo at kaapuapuhan ko sa habang
panahon ay may tutuluyan kung magnanais na mag-aral sa Maynila o kalapit na
mga lunsod sa medaling salita, ang bahay at lupang itoy walang magmamayari bagkus ay gagamitin habang panahon ng sinomang magnanais sa aking
kaapuapuhan na tumuklas ng karunungan sa paaralan sa Maynila at katabing
mga lunsod x x x x33 (emphasis and underscoring supplied)
But the condition set by the decedent on the propertys indivisibility is subject to a
statutory limitation. On this point, the Court agrees with the ruling of the appellate court,
viz:
For this Court to sustain without qualification, [petitioners]s contention, is to go against
the provisions of law, particularly Articles 494, 870, and 1083 of the Civil Code, which
provide that the prohibition to divide a property in a co-ownership can only last for
twenty (20) years x x x x
xxxx
x x x x Although the Civil Code is silent as to the effect of the indivision of a property for
more than twenty years, it would be contrary to public policy to sanction co-ownership
beyond the period expressly mandated by the Civil Code x x x x34
WHEREFORE, the petition is DENIED.
Costs against petitioners.

SO ORDERED.

G.R. No. 168660

June 30, 2009

HILARION, JR. and ENRICO ORENDAIN, represented by FE D.


ORENDAIN, Petitioners,1 vs. TRUSTEESHIP OF THE ESTATE OF DOA
MARGARITA RODRIGUEZ, Respondent.
DECISION
NACHURA, J.:
This petition for certiorari, filed under Rule 65 of the Rules of Court, assails the
Order2 of the Regional Trial Court (RTC) of Manila, Branch 4 in SP. PROC. No. 51872
which denied petitioners (Hilarion, Jr. and Enrico Orendain, heirs of Hilarion Orendain,
Sr.) Motion to Dissolve the Trusteeship of the Estate of Doa Margarita Rodriguez.
First, we revisit the long settled facts.
On July 19, 1960, the decedent, Doa Margarita Rodriguez, died in Manila, leaving a
last will and testament. On September 23, 1960, the will was admitted to probate by
virtue of the order of the Court of First Instance of Manila City (CFI Manila) in Special
Proceeding No. 3845. On August 27, 1962, the CFI Manila approved the project of
partition presented by the executor of Doa Margarita Rodriguezs will.
At the time of her death, the decedent left no compulsory or forced heirs and,
consequently, was completely free to dispose of her properties, without regard to
legitimes,3 as provided in her will. Some of Doa Margarita Rodriguezs testamentary
dispositions contemplated the creation of a trust to manage the income from her
properties for distribution to beneficiaries specified in the will, to wit:
xxxx
CLAUSULA SEGUNDA O PANG-DALAWA: - x x x Ipinaguutos ko na matapos magawa
ang pagaayos ng aking Testamentaria at masara na ang Expediente ng aking
Testamentaria, ang lahat ng pagaare ko sa aking ipinaguutos na pangasiwaan sa
habang panahon ay ipagbukas sa Juzgado ng tinatawag na "FIDEICOMISO" at ang
ilalagay na "fideicomisario" ang manga taong nasabi ko na sa itaas nito, at ang
kanilang gaganahin ay ang nasasabi sa testamentong ito na gaganahen ng
tagapangasiwa at albacea. x x x x

CLAUSULA TERCERA O PANG-TATLO: - Ipinaguutos ko na ang kikitain ng lahat ng


aking pagaare, na ang hindi lamang kasama ay ang aking lupain na nasasabi sa
Certificado de Transferencia de Titulo No. 7156 (Lote No. 1088-C), Certificado Original
de Titulo No. 4588 (LOTE No. 2492), Certificado Original de Titulo No. 4585 (Lote No.
1087) ng lalawigan ng Quezon, at ang bahaging maytanim na palay ng lupang
nasasaysay sa Certificado Original de Titulo No. 4587 (Lote No. 1180) ng Quezon, ay
IIPUNIN SA BANCO upang maibayad sa anillaramiento, ang tinatawag na "estate Tax",
ang "impuesto de herencia" na dapat pagbayaran ng aking pinagbibigyan na kasama
na din ang pagbabayaran ng "Fideicomiso", gastos sa abogado na magmamakaalam
ng testamentaria at gastos sa Husgado. Ngunit bago ipasok sa Banco ang kikitaen ng
nabangit na manga gagaare, ay aalisin muna ang manga sumusunod na gastos:
xxxx
CLAUSULA DECIMA O PANG-SAMPU: - Ipinaguutos ko na ang manga pagaareng
nasasabi sa Clausulang ito ay pangangasiwaan sa habang panahon, at ito nga ang
ipagbubukas ng "Fideicomiso" sa Jusgado pagkatapos na maayos ang naiwanan kong
pagaare. Ang pangangasiwaang pagaare ay ang manga sumusunod:
xxxx
Ang lahat ng pagaaring nasasabe sa Clusulang ito (hindi kasama ang "generator" at
automovil) hindi maisasanla o maipagbibili kailan man, maliban sa pagaaring nasa
Quezon Boulevard, Maynila, na maaring isanla kung walang fondo na gagamitin sa
ipagpapaigui o ipagpapagawa ng panibago alinsunod sa kaayusang hinihingi ng
panahon.
xxxx
CLAUSULA DECIMA SEGUNDA O PANG-LABING DALAWA: - Ang kuartang matitipon
sa Banco ayon sa tagubilin na nasasaysay sa Clausulang sinusundan nito ay gagamitin
sa manga sumusunod na pagkakagastusan; at ganito din ang gagawin sa lahat ng
aking pagaare na nasasakop ng fideicomiso at walang ibang pinaguukulan. Ang
pagkakagastusan na ito ay ang sumusunod:
xxxx

CLAUSULA VIGESIMA CUARTA O PANG-DALAWANGPU AT APAT: - Ipinaguutos ko


sa aking manga Tagapangasiwa na sa fondong ipinapasok sa Banco para sa gastos ng
Nia Maria, Misa at iba pa, kukuha sila na kakailanganin para maitulong sa manga
sumusunod: Florentina Luna, Roberta Ponce, Marciada Ponce, Benita Ponce,
Constancia Pineda, Regino Pineda, Tomas Payumo, Rosito Payumo, Loreto Payumo,
Brigido Santos at Quintin Laino, Hilarion Orendain at manga anak. Ang manga dalaga
kung sakali at inabutan ng pagkamatay ko na ako ay pinagtiisan at hindi humiwalay sa
akin, kung magkasakit ay ipagagamot at ibabayad sa medico, at ibibili ng gamot, at
kung kailangan ang operacion ay ipaooperacion at ipapasok sa Hospital na
kinababagayan ng kaniyang sakit, at kahit maypagkakautang pa sa "impuesto de
herencia at estate tax" ay ikukuha sa nasabing fondo at talagang ibabawas doon, at
ang paggagamot ay huag pagtutuusan, at ang magaalaga sa kanya ay bibigyan ng
gastos sa pagkain at sa viaje at iba pa na manga kailangan ng nagaalaga. Kung nasa
provincia at dadalhin ditto sa Maynila ay bibigyan ng gastos sa viaje ang maysakit at
ang kasama sa viaje, at ang magaalaga ay dito tutuloy sa bahay sa Tuberias at
Tanduay na natatalaga sa manga may servicio sa akin, at kung mamatay at gusting
iuwi sa provincia ang bangkay ay iupa at doon ilibing at dapit ng Pare at hated sa nicho
na natotoka sa kanya. Ganito din ang gagawain kung mayasawa man ay nasa poder
ko ng ako ay mamatay. Ang wala sa poder ko datapua at nagservicio sa akin, kaparis
ng encargado, ang gagawaing tulong ay ipagagamot, ibibili ng gamot at kung kailangan
ang operacion o matira sa Hospital, ipaooperacion at ipagbabayad sa
Hospital.4 (emphasis supplied)
xxxx
As regards Clause 10 of the will which explicitly prohibits the alienation or mortgage of
the properties specified therein, we had occasion to hold, in Rodriguez, etc., et al. v.
Court of Appeals, et al.,5 that the clause, insofar as the first twenty-year period is
concerned, does not violate Article 8706 of the Civil Code. We declared, thus:
The codal provision does not need any interpretation. It speaks categorically. What is
declared void is the testamentary disposition prohibiting alienation after the twenty-year
period. In the interim, such a provision does not suffer from the vice of invalidity. It
cannot be stricken down. Time and time again, We have said, and We now repeat, that
when a legal provision is clear and to the point, there is no room for interpretation. It
must be applied according to its literal terms.
Even with the purpose that the testatrix had in mind were not as unequivocal, still the
same conclusion emerges. There is no room for intestacy as would be the effect if the

challenged resolution of January 8, 1968 were not set aside. The wishes of the testatrix
constitute the law. Her will must be given effect. This is so even if there could be an
element of uncertainty insofar as the ascertainment thereof is concerned. In the
language of a Civil Code provision: "If a testamentary disposition admits of different
interpretations, in case of doubt, that interpretation by which the disposition is to be
operative shall be preferred." Nor is this all. A later article of the Civil Code equally calls
for observance. Thus: "The words of a will are to receive an interpretation which will
give to every expression some effect, rather than one which will render any of the
expressions inoperative; and of two modes of interpreting a will, that is to be preferred
which will prevent intestacy."
xxxx
Nothing can be clearer, therefore, than that [Petra, Antonia and Rosa, all surnamed
Rodriguez] could not challenge the provision in question. [They] had no right to
vindicate. Such a right may never arise. The twenty-year period is still with us. What
would transpire thereafter is still locked up in the inscrutable future, beyond the power
of mere mortals to foretell. At any rate, We cannot anticipate. Nor should We. We do
not possess the power either of conferring a cause of action to a party when, under the
circumstances disclosed, it had none.7
Almost four decades later, herein petitioners Hilarion, Jr. and Enrico Orendain, heirs of
Hilarion Orendain, Sr. who was mentioned in Clause 24 of the decedents will, moved to
dissolve the trust on the decedents estate, which they argued had been in existence for
more than twenty years, in violation of Articles 8678 and 870 of the Civil Code, and
inconsistent with our ruling in Rodriguez v. Court of Appeals.9
On April 18, 2005, the RTC issued the herein assailed Order:10
The above-cited provisions of the civil code find no application in the present motion to
dissolve the trust created by the testatrix. There is no question that the testamentary
disposition of Doa Margarita Rodriguez prohibiting the mortgage or sale of properties
mentioned in clause X of her Last Will and Testament forevermore is void after the
lapse of the twenty year period. However, it does not mean that the trust created by
[the] testatrix in order to carry out her wishes under clauses 12, 13 and 24 will also
become void upon expiration of the twenty year period. As ruled by the Supreme Court
in Emetrio Barcelon v. CA, "the codal provision cited in Art. 870 is clear and
unequivocal and does not need any interpretation. What is declared void is the
testamentary disposition prohibiting alienation after the twenty year period." Hence, the

trustees may dispose of the properties left by the testatrix in order to carry out the
latters testamentary disposition.
The question as to whether a trust can be perpetual, the same finds support in Article
1013[,] paragraph 4 of the Civil Code, which provides that "the Court, at the instance of
an interested party or its motion, may order the establishment of a permanent trust so
that only the income from the property shall be used." In the present case, the testatrix
directed that all the twenty five (25) pieces of property listed in the tenth clause should
be placed under the trusteeship and should be perpetually administered by the trustees
and a certain percentage of the income from the trust estate should be deposited in a
bank and should be devoted for the purposes specifically indicated in the clauses 12,
13 and 24.1awphi1
The wishes of the testatrix constitute the law. Her will must be given effect. This is even
if there could be an element of uncertainty insofar as the ascertainment thereof is
concerned. This Court so emphatically expressed it in a decision rendered more than
sixty years ago. Thus, respect for the will of a testator as [an] expression of his last
testamentary disposition, constitutes the principal basis of the rules which the law
prescribes for the correct interpretation of all of the clauses of the will; the words and
provision therein written must be plainly construed in order to avoid a violation of his
intentions and real purpose. The will of the testator clearly and explicitly stated must be
respected and complied with as an inviolable law among the parties in interest. Such is
the doctrine established by the Supreme Court of Spain, constantly maintained in a
great number of decisions.
Hence, this petition, positing the following issues:
1. WHETHER THE TRUSTEESHIP OVER THE PROPERTIES LEFT BY DOA
MARGARITA RODRIGUEZ CAN BE DISSOLVED APPLYING ARTICLES 867
AND 870 OF THE CIVIL CODE.
2. WHETHER THE LOWER COURT IS CORRECT IN STATING THAT THE
ABOVE-CITED PROVISIONS OF THE CIVIL CODE FINDS NO APPLICATION
IN THE PRESENT MOTION TO DISSOLVE THE TRUST CREATED BY THE
TESTATRIX.
3. CONCOMITANT THERETO, [WHETHER] THE LOWER COURT [IS]
CORRECT IN APPLYING ARTICLE 1013 PARAGRAPH 4 OF THE CIVIL
CODE.11

Before we delve into the foregoing issues, it is noteworthy that the present petition,
albeit captioned as a petition for certiorari, is actually a petition for review on certiorari,
raising only pure questions of law. On more than one occasion, we have allowed
erroneously labeled actions based on the averments contained in the petition or
complaint.12 Thus, we now disregard the incorrect designation and treat this as a
petition for review on certiorari under Rule 45 of the Rules of Court.
The petition is impressed with merit.
The issues being intertwined, we shall discuss them jointly.
Quite categorical from the last will and testament of the decedent is the creation of a
perpetual trust for the administration of her properties and the income accruing
therefrom, for specified beneficiaries. The decedent, in Clause 10 of her will, listed a
number of properties to be placed under perpetual administration of the trust. In fact,
the decedent unequivocally forbade the alienation or mortgage of these properties. In
all, the decedent did not contemplate the disposition of these properties, but only
sought to bequeath the income derived therefrom to various sets of beneficiaries.
On this score, we held in Rodriguez v. Court of Appeals13 that the perpetual prohibition
was valid only for twenty (20) years. We affirmed the CAs holding that the trust
stipulated in the decedents will prohibiting perpetual alienation or mortgage of the
properties violated Articles 867 and 870 of the Civil Code. However, we reversed and
set aside the CAs decision which declared that that portion of the decedents estate,
the properties listed in Clause 10 of the will, ought to be distributed based on intestate
succession, there being no institution of heirs to the properties covered by the perpetual
trust.
As previously quoted, we reached a different conclusion and upheld the trust, only
insofar as the first twenty-year period is concerned. We refrained from forthwith
declaring the decedents testamentary disposition as void and the properties
enumerated in Clause 10 of the will as subject to intestate succession. We held that, in
the interim, since the twenty-year period was then still upon us, the wishes of the
testatrix ought to be respected.
Thus, at present, there appears to be no more argument that the trust created over the
properties of the decedent should be dissolved as the twenty-year period has, quite
palpably, lapsed.

Notwithstanding the foregoing, the RTC ruled otherwise and held that: (a) only the
perpetual prohibition to alienate or mortgage is declared void; (b) the trust over her
properties stipulated by the testatrix in Clauses 12, 13 and 24 of the will remains valid;
and (c) the trustees may dispose of these properties in order to carry out the latters
testamentary disposition.

xxxx

We disagree.

(2) When the will does not institute an heir to, or dispose of all the property belonging to
the testator. In such case, legal succession shall take place only with respect to the
property of which the testator has not disposed;

Apparent from the decedents last will and testament is the creation of a trust on a
specific set of properties and the income accruing therefrom. Nowhere in the will can it
be ascertained that the decedent intended any of the trusts designated beneficiaries to
inherit these properties. The decedents will did not institute any heir thereto, as clearly
shown by the following:
1. Clause 2 instructed the creation of trust;
2. Clause 3 instructed that the remaining income from specified properties, after
the necessary deductions for expenses, including the estate tax, be deposited in
a fund with a bank;
3. Clause 10 enumerated the properties to be placed in trust for perpetual
administration (pangasiwaan sa habang panahon);
4. Clauses 11 and 12 directed how the income from the properties ought to be
divided among, and distributed to the different beneficiaries; and
5. Clause 24 instructed the administrators to provide medical support to certain
beneficiaries, to be deducted from the fund deposits in the bank mentioned in
Clauses 2 and 3.
Plainly, the RTC was mistaken in denying petitioners motion to dissolve and ordering
the disposition of the properties in Clause 10 according to the testatrixs wishes. As
regards these properties, intestacy should apply as the decedent did not institute an
heir therefor. Article 782, in relation to paragraph 2, Article 960 of the Civil Code,
provides:
Art. 782. An heir is a person called to the succession either by the provision of a will or
by operation of law.

Art. 960. Legal or intestate succession takes place:


xxxx

xxx
We find as erroneous the RTCs holding that paragraph 4,14 Article 1013 of the same
code specifically allows a perpetual trust, because this provision of law is inapplicable.
Suffice it to state that the article is among the Civil Code provisions on intestate
succession, specifically on the State inheriting from a decedent, in default of persons
entitled to succeed. Under this article, the allowance for a permanent trust, approved by
a court of law, covers property inherited by the State by virtue of intestate succession.
The article does not cure a void testamentary provision which did not institute an heir.
Accordingly, the article cannot be applied to dispose of herein decedents properties.
We are not unmindful of our ruling in Palad, et al. v. Governor of Quezon Province, et
al.15 where we declared, thus:
Article 870 of the New Civil Code, which regards as void any disposition of the testator
declaring all or part of the estate inalienable for more than 20 years, is not violated by
the trust constituted by the late Luis Palad; because the will of the testator does not
interdict the alienation of the parcels devised. The will merely directs that the income of
said two parcels be utilized for the establishment, maintenance and operation of the
high school.
Said Article 870 was designed "to give more impetus to the socialization of the
ownership of property and to prevent the perpetuation of large holdings which give rise
to agrarian troubles." The trust herein involved covers only two lots, which have not
been shown to be a large landholding. And the income derived therefrom is being
devoted to a public and social purpose the education of the youth of the land. The
use of said parcels therefore is in a sense socialized. There is no hint in the record that
the trust has spawned agrarian conflicts.16

In this case, however, we reach a different conclusion as the testatrix specifically


prohibited the alienation or mortgage of her properties which were definitely more than
the two (2) properties in the aforecited case. The herein testatrixs large landholdings
cannot be subjected indefinitely to a trust because the ownership thereof would then
effectively remain with her even in the afterlife.

G.R. No. L-31703

In light of the foregoing, therefore, the trust on the testatrixs properties must be
dissolved and this case remanded to the lower court to determine the following:

L. D. Lockwood and Jose M. Casal for appellants.


Eduardo Gutierrez Repide and Leoncio B. Monzon for appellee.

1. The properties listed in Clause 10 of the will, constituting the perpetual trust,
which are still within reach and have not been disposed of as yet; and
2. The intestate heirs of the decedent, with the nearest relative of the deceased
entitled to inherit the remaining properties.
One final note. To obviate confusion, we clarify that the petitioners, although correct in
moving for the dissolution of the trust after the twenty-year period, are not necessarily
declared as intestate heirs of the decedent. Our remand of the case to the RTC means
that the probate court should now make a determination of the heirship of the intestate
heirs of the decedent where petitioners, and all others claiming to be heirs of the
decedent, should establish their status as such consistent with our ruling in Heirs of
Yaptinchay v. Hon. del Rosario.17
WHEREFORE, premises considered, the petition is GRANTED. The Order of the
Regional Trial Court of Manila, Branch 4 in SP. PROC. No. 51872 is REVERSED and
SET ASIDE. The trust approved by the Regional Trial Court of Manila, Branch 4 in SP.
PROC. No. 51872 is DISSOLVED. We ORDER the Regional Trial Court of Manila,
Branch 4 in SP. PROC. No. 51872 to determine the following:
1. the properties listed in Clause 10 of Doa Margarita Rodriguezs will,
constituting the perpetual trust, which are still within reach and have not been
disposed of as yet; and
2. the intestate heirs of Doa Margarita Rodriguez, with the nearest relative of
the decedent entitled to inherit the remaining properties.
SO ORDERED.

February 13, 1930

CARMEN G. DE PEREZ, trustee of the estate of Ana Maria Alcantara, plaintiffappellee, vs. MARIANO GARCHITORENA, and JOSE CASIMIRO, Sheriff of the
Court of First Instance of Manila,defendants-appellants.

ROMUALDEZ, J.:
The amount of P21,428.58 is on deposit in the plaintiff's name with the association
known as La Urbana in Manila, as the final payment of the liquidated credit of Ana
Maria Alcantara, deceased, whose heiress is said plaintiff, against Andres
Garchitorena, also deceased, represented by his son, the defendant Mariano
Garchitorena.
And as said Mariano Garchitorena held a judgment for P7,872.23 against Joaquin
Perez Alcantara, husband of the plaintiff, Carmen G. de Perez, the sheriff pursuant to
the writ of execution issued in said judgment, levied an attachment on said amount
deposited with La Urbana.
The plaintiff, alleging that said deposit belongs to the fideicommissary heirs of the
decedent Ana Maria Alcantara, secured a preliminary injunction restraining the
execution of said judgment on the sum so attached. The defendants contend that the
plaintiff is the decedent's universal heiress, and pray for the dissolution of the
injunction.
The court below held that said La Urbana deposit belongs to the plaintiff's children as
fideicommissary heirs of Ana Maria Alcantara, and granted a final writ of injunction.
The defendants insist in their contentions, and, in their appeal from the decision of the
trial court, assign the following errors:
1. The lower court erred in holding that a trust was created by the will of Doa
Ana Maria Alcantara.

2. The lower court erred in concluding and declaring that the amount of
P21,428.58 deposited with La Urbana is the property of the children of the
plaintiff as "herederos fidei-comisarios."
3. The lower court erred in making the injunction permanent and condemning
defendant to pay the costs.
The question here raised is confined to the scope and meaning of the institution of heirs
made in the will of the late Ana Maria Alcantara already admitted to probate, and whose
legal force and effect is not in dispute.
The clauses of said will relevant to the points in dispute, between the parties are the
ninth, tenth, and eleventh, quoted below:
Ninth. Being single and without any forced heir, to show my gratitude to my
niece-in-law, Carmen Garchitorena, of age, married to my nephew, Joaquin
Perez Alcantara, and living in this same house with me, I institute her as my
sole and universal heiress to the remainder of my estate after the payment of
my debts and legacies, so that upon my death and after probate of this will, and
after the report of the committee on claims and appraisal has been rendered
and approved, she will receive from my executrix and properties composing my
hereditary estate, that she may enjoy them with God's blessing and my own.
Tenth. Should my heiress Carmen Garchitorena die, I order that my whole
estate shall pass unimpaired to her surviving children; and should any of these
die, his share shall serve to increase the portions of his surviving brothers (and
sisters) by accretion, in such wise that my estate shall never pass out of the
hands of my heiress or her children in so far as it is legally possible.
Eleventh. Should my aforesaid heiress, Carmen Garchitorena, die after me
while her children are still in their minority, I order that my estate be
administered by my executrix, Mrs. Josefa Laplana, and in her default, by
Attorney Ramon Salinas and in his default, by his son Ramon Salinas; but the
direction herein given must not be considered as an indication of lack of
confidence in my nephew Joaquin Perez Alcantara, whom I relieve from the
duties of administering my estate, because I recognize that his character is not
adapted to management and administration.

The appellants contend that in these clauses the testatrix has ordered a simple
substitution, while the appellee contends that it is a fideicommissary substitution.
This will certainly provides for a substitution of heirs, and of the three cases that might
give rise to a simple substitution (art. 774, Civil Code), only the death of the instituted
heiress before the testatrix would in the instant case give place to such substitution,
inasmuch as nothing is said of the waiver of inheritance, or incapacity to accept it. As a
matter of fact, however, clause XI provides for the administration of the estate in case
the heiress instituted should die after the testatrix and while the substitute heirs are still
under age. And it is evident that, considering the nature of simple substitution by the
heir's death before the testator, and the fact that by clause XI in connection with clause
X, the substitution is ordered where the heiress instituted dies after the testatrix, this
cannot be a case of simple substitution.
The existence of a substitution in the will is not and cannot be denied, and since it
cannot be a simple substitution in the light of the considerations above stated, let us
now see whether the instants case is a fideicommissary substitution.
In clause IX, the testatrix institutes the plaintiff herein her sole and universal heiress,
and provides that upon her death (the testatrix's) and after probate of the will and
approval of the report of the committee on claims and appraisal, said heiress shall
receive and enjoy the whole hereditary estate. Although this clause provides nothing
explicit about substitution, it does not contain anything in conflict with the idea of
fideicommissary substitution. The fact that the plaintiff was instituted the sole and
universal heiress does not prevent her children from receiving, upon her death and in
conformity with the express desire of the testatrix, the latter's hereditary estate, as
provided in the following (above quoted) clauses which cannot be disregarded if we are
to give a correct interpretation of the will. The word sole does not necessarily exclude
the idea of substitute heirs; and taking these three clauses together, such word means
that the plaintiff is the sole heiress instituted in the first instance.
The disposition contained in clause IX, that said heiress shall receive and enjoy the
estate, is not incompatible with a fideicommissary substitution (it certainly is
incompatible with the idea of simple substitution, where the heiress instituted does not
receive the inheritance). In fact the enjoyment of the inheritance is in conformity with
the idea of fideicommissary substitution, by virtue of which the heir instituted receives
the inheritance and enjoys it, although at the same time he preserves it in order to pass
it on the second heir. On this point the illustrious Manresa, in his Civil Code (Vol. 6, pp.
142 and 143, 5th ed.), says:

Or, what amounts to the same thing, the fideicommissary substitution, as held in
the Resolution of June 25, 1895, February 10, 1899, and July 19, 1909, requires
three things:
1. A first heir called primarily to the enjoyment of the estate.
2. An obligation clearly imposed upon him to preserve and transmit to a third
person the whole or a part of the estate.
3. A second heir.
To these requisites, the decision of November 18, 1918 adds another, namely
that the fideicommissarius be entitled to the estate from the time the testator
dies, since he is to inherit from the latter and not from the fiduciary. (Emphasis
ours.)
It appears from this quotation that the heir instituted or the fiduciary, as referred to in
articles 783 of the Civil Code, is entitled to enjoy the inheritance. And it might here be
observed, as a timely remark, that the fideicommissum arising from a fideicommissary
substitution, which is of Roman origin, is not exactly equivalent to, nor may it be
confused with, the English "trust."
It should also be noted that said clause IX vests in the heiress only the right to enjoy
but not the right to dispose of the estate. It says, she may enjoy it, but does not say she
may dispose of it. This is an indication of the usufruct inherent in fideicommissary
substitution.
Clause X expressly provides for the substitution. It is true that it does not say whether
the death of the heiress herein referred to is before or after that of the testatrix; but from
the whole context it appears that in making the provisions contained in this clause X,
the testatrix had in mind a fideicommissary substitution, since she limits the
transmission of her estate to the children of the heiress by this provision, "in such wise
that my estate shall never pass out of the hands of my heiress or her children in so far
as it is legally possible." Here it clearly appears that the testatrix tried to avoid the
possibility that the substitution might later be legally declared null for transcending the
limits fixed by article 781 of the Civil Code which prescribed that fideicommissary
substitutions shall be valid "provided they do not go beyond the second degree."

Another clear and outstanding indication of fideicommissary substitution in clause X is


the provision that the wholeestate shall pass unimpaired to the heiress's children, that
is to say the heiress is required to preserve the whole estate, without diminution, in
order to pass it on in due time to the fideicommissary heirs. This provision complies
with another of the requisites of fideicommissary substitution according to our quotation
from Manresa inserted above.
Lastly, clause XI more clearly indicates the idea of fideicommissary substitution, when a
provision is therein made in the event the heiress should die after the testatrix. That is,
said clause anticipates the case where the instituted heiress should die after the
testatrix and after receiving and enjoying the inheritance.
The foregoing leads us to the conclusion that all the requisites of a fideicommissary
substitution, according to the quotation from Manresa above inserted, are present in the
case of substitution now under consideration, to wit:
1. At first heir primarily called to the enjoyment of the estate. In this case the
plaintiff was instituted an heiress, called to the enjoyment of the estate,
according to clause IX of the will.
2. An obligation clearly imposed upon the heir to preserve and transmit to a third
person the whole or a part of the estate. Such an obligation is imposed in clause
X which provides that the "whole estate shall pass unimpaired to her (heiress's)
surviving children;" thus, instead of leaving the heiress at liberty to dispose of
the estate by will, or of leaving the law to take its course in case she dies
intestate, said clause not only disposes of the estate in favor of the heiress
instituted, but also provides for the disposition thereof in case she should die
after the testatrix.
3. A second heir. Such are the children of the heiress instituted, who are
referred to as such second heirs both in clause X and in clause XI.
Finally, the requisite added by the decision of November 18, 1918, to wit, that the
fideicommissarius or second heir should be entitled to the estate from the time of the
testator's death, which in the instant case, is, rather than a requisite, a necessary
consequence derived from the nature of the fideicommissary substitution, in which the
second heir does not inherit from the heir first instituted, but from the testator.

By virtue of this consequence, the inheritance in question does not belong to the
heiress instituted, the plaintiff herein, as her absolute property, but to her children, from
the moment of the death of the testatrix, Ana Maria Alcantara.
Therefore, said inheritance, of which the amount referred to at the beginning, which is
on deposit with the association known as La Urbana in the plaintiff's name, is a part,
does not belong to her nor can it be subject to the execution of the judgment against
Joaquin Perez, who is not one of the fideicommissary heirs.
The judgment appealed from is affirmed, with costs against the appellant, Mariano
Garchitorena. So ordered.

G.R. No. L-13876

February 28, 1962

CONSOLACION FLORENTINO DE CRISOLOGO, ET AL., plaintiffs-appellees, vs. DR.


MANUEL SINGSON, defendant-appellant.

3. That in the event the said parties shall fail to do so, this Court will appoint the
corresponding commissioners to make the partition in accordance with law;
and .
4. Without special pronouncement as to costs." .

Felix V. Vergara for defendant-appellant.


B. Martinez for plaintiffs-appellees.
DIZON, J.:
Action for partition commenced by the spouses Consolacion Florentino and Francisco
Crisologo against Manuel Singson in connection with a residential lot located a Plaridel
St., Vigan, Ilocos Sur, with an area of approximately 193 square meters, and the
improvements existing thereon, covered by Tax No. 10765-C. Their complaint alleged
that Singson owned one-half pro-indiviso of said property and that Consolacion
Florentino owned the other half by virtue of the provisions of the duly probated last will
of Da. Leona Singson, the original owner, and the project of partition submitted to, and
approved by the Court of First Instance of Ilocos Sur in special Proceeding No. 453;
that plaintiffs had made demands for the partition of said property, but defendant
refused to accede thereto, thus compelling them to bring action.
Defendant's defense was that Consolacion Florentino was a mere usufructuary of, and
not owner of one-half pro-indiviso of the property in question, and that, therefore, she
was not entitled to demand partition thereof.
After trial upon the issue thus posed, the lower court rendered judgment as follows:
1. Declaring that the plaintiff is a co-owner pro-indiviso with the defendant of the
house and lot described in the complaint to the extent of each of an undivided
1/2 portion thereof; .
2. Ordering the aforesaid co-owners to execute an agreement of partition of the
said property within 30 days from receipt of this judgment unless it be shown
that the division thereof may render it unserviceable, in which case the
provisions of Art. 498 of the New Civil Code may be applied; .1wph1.t

From the above judgment, defendant Singson appealed.


It is admitted that Da. Leona Singson, who died single on January 13, 1948, was the
owner of the property in question at the time of her death. On July 31, 1951 she
executed her last will which was admitted to probate in Special Proceeding No. 453 of
the lower court whose decision was affirmed by the Court of Appeals in G.R. No. 3605R. At the time of the execution of the will, her nearest living relatives were her brothers
Evaristo, Manuel and Dionisio Singson, her nieces Rosario, Emilia and Trinidad, and
her grandniece Consolation, all surnamed Florentino.
Clause IX of her last will reads as follows: .
NOVENO. Ordeno que se de a mi nieta por parte de mi hermana mia y que
al mismo tiempo vive en mi casa, y, por tanto, bajo mi proteccion, y es la
CONSOLACION FLORENTINO:
(A). La mitad de mi casa de materials fuertes con techo de hierro galvanizado,
incluyendo la mitad de su solar, ubicado en la Poblacion de Vigan, Ilocos Sur,
Calle Plaridel, actualmente arrendada por los hermanos Fortunato, Teofilo y
Pedro del appellido Kairuz. Pero si falleciere antes o despues que yo mi citada
nieta, esta propiedad se dara por partes iguales entre mis tres hermanos
Evaristo, Manuel y Dionisio, o a sus herederos forzosos en el caso de que
alguno de ellas murieie antes ... (Exhibit F.)
The issue to be decided is whether the testamentary disposition above-quoted provided
for what is calledsustitucion vulgar or for a sustitucion fideicomisaria. This issue is, we
believe, controlled by the pertinent provisions of the Civil Code in force in the
Philippines prior to the effectivity of the New Civil Code, in view of the fact that the
testatrix died on January 13, 1948. They are the following: .

Art. 774. The testator may designate one or more persons to substitute the heir
or heirs instituted in case such heir or heirs should die before him, or should not
wish or should be unable to accept the inheritance.
A simple substitution, without a statement of the cases to which it is to apply,
shall include the three mentioned in the next preceeding paragraph, unless the
testator has otherwise provided:
Art. 781. Fidei-commissary substitutions by virtue of which the heir is charged to
preserve and transmit to a third person the whole or part of the inheritance shall
be valid and effective, provided they do not go beyond the second degree, or
that they are made in favor of persons living at the time of the death of the
testator." .
Art. 785. The following shall be inoperative: .
1. Fiduciary substitutions not made expressly, either by giving them this name or
by imposing upon the fiduciary the absolute obligation of delivering the property
to a second heir." ....
In accordance with the first legal provision quoted above, the testator may not only
designate the heirs who will succeed him upon his death, but also provide for
substitutes in the event that said heirs do not accept or are in no position to accept the
inheritance or legacies, or die ahead of him.
The testator may also bequeath his properties to a particular person with the obligation,
on the part of the latter, to deliver the same to another person, totally or partially, upon
the occurrence of a particular event (6 Manresa, p. 1112).

undoubtedly be entitled to partition, but not in the latter. As Manresa says, if the
fiduciary did not acquire full ownership of the property bequeathed by will, but mere
usufructuary rights thereon until the time came for him to deliver said property to the
fideicomisario, it is obvious that the nude ownership over the property, upon the death
of the testatrix, passed to and was acquired by another person, and the person cannot
be other than the fideicomisario (6 Manresa p. 145).
It seems to be of the essence of a fideicommissary substitution that an obligation be
clearly imposed upon the first heir to preserve and transmit to another the whole or part
of the estate bequeathed to him, upon his death or upon the happening of a particular
event. For this reason, Art. 785 of the old Civil Code provides that a fideicommissary
substitution shall have no effect unless it is made expressly ("de una manera expresa")
either by giving it such name, or by imposing upon the first heir the absolute obligation
("obligacion terminante") to deliver the inheritance to a substitute or second heir. In this
connection Manresa says: .
Para que la sustitucion sea fideicomisaria, es preciso segun el art. 781, que se
ordeno o encargue al primer heredero, cuando sea tal, que conserve y
transmita a una tercera persona o entidad el todo a parte de la herencia. O lo
que es lo mismo, la sustitucion fideicomisaria, como declaran las resoluciones
de 25 de Junio de 1895, 10 de Febrero de 1899 y 19 de Julio de 1909, exige
tres requisitos: .
1.o Un primer heredero llamado al goce de los bienes preferentemente.
2.o Obligacion claramente impuesta al mismo de conservar y transmitir a un
tercero el todo o parte del caudal.
3.o Un segundo heredero.

It is clear that the particular testamentary clause under consideration provides for a
substitution of the heir named therein in this manner: that upon the death of
Consolacion Florentino whether this occurs before or after that of the testatrix the
property bequeathed to her shall be delivered ("se dara") or shall belong in equal parts
to the testatrix's three brothers, Evaristo, Manuel and Dionisio, or their forced heirs,
should anyone of them die ahead of Consolacion Florentino. If this clause created what
is known as sustitucion vulgar, the necessary result would be that Consolacion
Florentino, upon the death of the testatrix, became the owner of one undivided half of
the property, but if it provided for a sustitution fideicomisaria, she would have acquired
nothing more than usufructuary rights over the same half. In the former case, she would

A estos requisitos anade la sentencia de 18 de Noviembre de 1918, otro mas, el


del que el fideicomisario tenga derecho a los bienes de la herencia desde el
momento de la muerte del testador, puesto que ha de suceder a este y no al
fiduciario.
Por tanto, cuando el causante se limita a instituir dos herederos, y por
fallecimiento de ambos o de cualquiera de ellos, asigna la parte del fallecido o
fallecidos, a los herederos legitimos o a otras personas, solo existe una
sustitucion vulgar, porque falta el requisito de haberse impuesto a los primeros

herederos la obligacion de conservar y transmitir los bienes, y el articulo 789,


en su parrafo primero, evige que la sustitucion sea expresa, ya dandole el
testador el nombre de sustitucion fideicomisaria, ya imponiendo al sustituido la
obligacion terminante de conservar y transmitir los bienes a un segundo
heredero.
A careful perusal of the testamentary clause under consideration shows that the
substitution of heirs provided for therein is not expressly made of the fideicommissary
kind, nor does it contain a clear statement to the effect that appellee, during her
lifetime, shall only enjoy usufructuary rights over the property bequeathed to her, naked
ownership thereof being vested in the brothers of the testatrix. As already stated, it
merely provides that upon appellee's death whether this happens before or after that
of the testatrix her share shall belong to the brothers of the testatrix.
In the light of the foregoing, we believe, and so hold, that the last will of the deceased
Da. Leona Singson, established a mere sustitucion vulgar, the substitution
Consolacion Florentino by the brothers of the testatrix to be effective or to take place
upon the death of the former, whether it happens before or after that of the testatrix.
IN VIEW OF THE FOREGOING, the appealed judgment is affirmed, with costs.

G.R. No. L-56249 May 29, 1987


IN THE MATTER OF THE TESTATE ESTATE OF THE DECEASED REV. FATHER
TEODORO ARANAS, RAMONA B. VDA. DE ARANAS, ADELIA B. ARANASFERNANDEZ, HEIRS OF THE LATE RODULFO B. ARANAS, ETC., ET
AL., petitioners, vs. VICENTE B. ARANAS AND HON. LUIS B. MANTA, respondents.

PARAS, J.:
This is a petition for certiorari which seeks to declare the orders of respondent Judge
dated July 16, 1980 and September 23, 1980 as an exercise of a gross abuse of
discretion amounting to lack of jurisdiction, by ruling that the properties under Group C
of the testate estate of the late Fr.Teodoro Aranas are subject to remunerative legacies.
The antecedent facts of the case are as follows:
Fr. Teodoro Aranas, a priest of the Roman Catholic Church, died on January 19, 1953.
He had executed on June 6, 1946 his Last Will and Testament which was admitted to
probate on August 31, 1956. In said Last Will and Testament, Fr. Teodoro Aranas
stipulated the following:
A. The return to Aniceto Aranas or his heirs of all properties acquired by Fr. Aranas from
his brother Aniceto Aranas and ten (10) parcels of land described in the Will inherited
by the testator from his parents.
B. The return to Carmelo Aranas or his heirs of all properties acquired by Fr. Aranas
from his brother Carmelo Aranas and ten (10) parcels of land described in the Will
inherited by the testator from his parents.
C. The special administration of the remainder of the estate of the testator by Vicente
Aranas, a faithful and serviceable nephew and designating him also as recipient of 1/2
of the produce of said properties after deducting the expenses for the administration
and the other 1/2 of the produce to be given to the Catholic Church for the eternal
repose of the testator's soul. Said pertinent provision 1 reads as follows:

Fourth. It is my will that the lands I had bought from other persons
should be converged and placed under a "special administrator." The
special administrator of these lands, for his office, should receive one
half of all the produce from which shall be deducted the expenses for the
administration, and the other half of the produce should be received by
the Roman Catholic Church and should be spent for my soul, Vicente B.
Aranas (Tingting), because he is a faithful and serviceable nephew,
should be the first special administrator of said properties, without bond,
until his death or until he should not want to hold the said office
anymore. Anyone of the sons of my brother Carmelo Aranas can hold
the said office of special administrator, and none other than they. Their
father, my brother Carmelo Aranas shall be the one to decide who
among them shall hold the said office, but upon the death of my said
brother Carmelo Aranas, his said sons will have power to select the one
among them ourselves. The special administration is perpetual.
The lower court in its Order 2 dated November 17, 1977 ruled, upon petitioners' (in Sp.
Proc. No. 303) "Motion for the Declaration of Heirs and Partition; and for Removal of
the Administrator (Vicente Aranas) and/or for his Permission to Resign, and
appointment of His Successor" that the "perpetual inalienability and administration of
the portion of the estate of the late Rev. Fr. Teodoro Aranas, administered by Vicente
Aranas, is nun and void after twenty years from January 19, 1954 ... " and declared in
the same order the heirs of the late Fr. Teodoro Aranas. It also declared that "the
removal of Vicente Aranas will, therefore, not serve the ends of justice and for the best
interest of all the heirs, particularly with respect to the portion of the estate taken by the
heirs of Aniceto Aranas, represented by the petitioners herein and the rest of the heirs
of Carmelo, represented by the intervenors, coheirs of Administrator Vicente Aranas." 3
However, the abovesaid Order was subsequently set aside upon the "Urgent Motion for
Reconsideration and to Declare Testate and Intestate Heirs of the late Fr. Teodoro
Aranas," filed by the administrator Vicente Aranas on the allegation that said order was
violative of due process and without legal and factual basis because only the issue for
the removal of the administrator was heard and not the matter of the declaration of
heirs. Thus, the lower court declared in its Order, 4 dated July 16, 1980 that the Order
dated November 17, 1977 is "set aside and in the interest of justice, reopened in order
that other heirs, successors-in-interest of Felino Aranas, 5 could likewise assert their
claims, as in the case of the heirs of Aniceto Aranas and Carmelo Aranas." 6

Their Motion for Reconsideration having been denied by the lower court in its order
dated September 23, 1980, petitioners now come before Us by certiorari raising the
issue that the lower court erred in setting aside its order dated November 17, 1977 and
in not applying the provisions on Usufruct of the New Civil Code with respect to the
properties referred to as Group "C" in the Last Will and Testament.
The court ruled in its questioned order that this particular group of properties (Group
"C") is subject to the following:
1. Remunerative legacy by way of usufruct of the net proceeds of 1/2 of
the estate after deducting expenses for administration in favor of Vicente
Aranas, during his lifetime and shall continue an administrator of the
estate, and, who, upon his death or refusal to continue such usufruct,
may be succeeded by any of the brothers of the administrator as
selected by their father, Carmelo Aranas, if still alive or one selected by
his sons if, he, Carmelo, is dead; Pursuant to the Will. (Article 562, 563,
564 and 603 of the New Civil Code).
2. Legacy in favor of the Roman Catholic Church, particularly the
Archbishop diocese of Cagayan de Oro City Represented by the
Reverend Archbishop Patrick H. Cronin over one-half of the proceeds of
the properties under Group "C." (Article 603, New Civil Code) and to last
for a period of Fifty years from the effective date of the legacy, Article
605, New Civil Code). (Annex "L-14," p. 87, Rollo)
Assailing the aforementioned ruling, petitioners rely heavily on the doctrine laid down in
Art. 870 of the New Civil Code to wit:
Art. 870. The dispositions of the testator declaring all or part of the estate
inalienable for more than twenty years are void.
A cursory reading of the English translation of the Last Will and Testament shows that it
was the sincere intention and desire of the testator to reward his nephew Vicente
Aranas for his faithful and unselfish services by allowing him to enjoy one-half of the
fruits of the testator's third group of properties until Vicente's death and/or refusal to act
as administrator in which case, the administration shall pass to anyone chosen by
Carmelo Aranas among his sons and upon Carmelo's death, his sons will have the
power to select one among themselves. Vicente Aranas therefore as a usufructuary has
the right to enjoy the property of his uncle with all the benefits which result from the

normal enjoyment (or exploitation) of another's property, with the obligation to return, at
the designated time, either the same thing, or in special cases its equivalent. This right
of Vicente to enjoy the fruits of the properties is temporary and therefore not perpetual
as there is a limitation namely his death or his refusal. Likewise his designation as
administrator of these properties is limited by his refusal and/or death and therefore it
does not run counter to Art. 870 of the Civil Code relied upon by the petitioners. Be it
noted that Vicente Aranas is not prohibited to dispose of the fruits and other benefits
arising from the usufruct. Neither are the naked owners (the other heirs) of the
properties, the usufruct of which has been given to Vicente Aranas prohibited from
disposing of said naked ownership without prejudice of course to Vicente's continuing
usufruct. To void the designation of Vicente Aranas as usufructuary and/or administrator
is to defeat the desire and the dying wish of the testator to reward him for his faithful
and unselfish services rendered during the time when said testator was seriously ill or
bed-ridden. The proviso must be respected and be given effect until the death or until
the refusal to act as such of the instituted usufructuary/administrator, after which period,
the property can be properly disposed of, subject to the limitations provided in Art. 863
of the Civil Code concerning a fideicommissary substitution, said Article says:
A fideicommissary substitution by virtue of which the fiduciary or first heir
instituted is entrusted with the obligation to preserve and to transmit to a
second heir the whole or part of the inheritance, shall be valid and shall
take effect, provided such substitution does not go beyond one degree
from the heir originally instituted, and provided further, that the fiduciary
or first heir and the second heir are living at the time of the death of the
testator.
It is contended by petitioners that the ruling made by respondent court dated November
17, 1977 was already final and not subject to correction as what was set aside and to
be reheard was only regarding the determination of additional heirs. Such contention is
not worthy of credence. Respondents in their Memorandum allege and it is not disputed
by petitioners that the order of November 17, 1977 has not yet become final because it
was received only on January 12, 1978 by the counsel for respondent Vicente Aranas
and the Motion for Reconsideration and to declare testamentary and intestate heirs
dated January 17, 1978 was filed by the said respondent within the reglementary
period. Besides the validity or invalidity of the usufructuary dispositions would affect the
determination of heirs.
As to petitioners' allegation that the order of July 16, 1980 is without basis, the record
shows that during the hearing of the urgent motion for reconsideration and to declare

testamentary and intestate heirs, it was proven conclusively by the said respondent
Vicente B. Aranas that he was instituted as a remunerative legatee per mandate of the
Last Will and Testament by way of usufructuary. Likewise the right of the Roman
Catholic Church as the other usufructuary legatee for the duration of the statutory
lifetime of a corporation, that is, 50 years from the date of the effectivity of said legacy,
was also established. 7
WHEREFORE, the instant petition is hereby dismissed.
SO ORDERED.

G.R. No. L-15737

February 28, 1962

LEONOR VILLAFLOR VDA. DE VILLANUEVA, plaintiff-appellant, vs. DELFIN N.


JUICO, in his capacity as Judicial Administrator of the testate estate of FAUSTA
NEPOMUCENO, defendant-appellee.
Amado G. Salazar for plaintiff-appellant.
Sycip, Salazar, Luna and Associates for defendant-appellee.
REYES, J.B.L., J.:
Subject to this direct appeal to us on points of law is the decision of the Court of First
Instance of Rizal, in its Civil Case No. Q-2809, dismissing plaintiff-appellant's complaint
for the recovery of certain properties that were originally owned by the plaintiff's
granduncle, Nicolas Villaflor, and which he granted to his widow, Doa Fausta
Nepomuceno, bequeathing to her "su uso y posesion mientras viva y no se case en
segundas nupcias".
The following facts appear of record: On October 9, 1908, Don Nicolas Villaflor, a
wealthy man of Castillejos, Zambales, executed a will in Spanish in his own
handwriting, devising and bequeathing in favor of his wife, Dona Fausta Nepomuceno,
one-half of all his real and personal properties, giving the other half to his brother Don
Fausto Villaflor.
Clause 6th, containing the institution of heirs, reads as follows: .
SEXTO En virtud de las facultades que me conceden las leyes, instituyo per
mis unicos y universales herederos de todos mis derechos y acciones a mi
hermano D. Fausto Villaflor y a mi esposa Da. Fausta Nepomuceno para que
partan todos mis bienes que me pertenescan, en iguales partes, para despues
de mi muerte, exceptuando las donaciones y legados que, abajo mi mas
expontanea voluntad, lo hago en la forma siguiente: .
SEPTIMO: Lego para dispues de mi muerte a mi esposa Da. Fausta
Nepomuceno, en prueba de mi amor y carino, los bienes, alhajas y muebles
que a continuacion se expresan; .

OCTAVO: Que estos legades disfrutaria mi referida esposa Da. Fausta


Nepomuceno su uso y posesion mientras viva y no se case en segundas
nupcias, de la contrario, pasara a ser propiedad estos dichos legados de mi
sobrina nieta Leonor Villaflor.
The 12th clause of the will provided, however, that Clauses 6th and 7th thereof would
be deemed annulled from the moment he bore any child with Doa Fausta
Nepomuceno. Said Clause 12th reads as follows: .
DUODECIMO: Quedan anulados las parrafos 6.0 y 7.0 de este testamento
que tratan de institucion de herederos y los legados que se haran despues de
mi muerte a favor de mi esposa, en el momento que podre tener la dicha de
contrar con hijo y hijos legitimos o legitimados, pues estos, conforme a ley
seran mis herederos.
Don Nicolas Villaflor died on March 3, 1922, without begetting any child with his wife
Doa Fausta Nepomuceno. The latter, already a widow, thereupon instituted Special
Proceeding No. 203 of the Court of First Instance of Zambales, for the settlement of her
husband's estate and in that proceeding, she was appointed judicial administratrix. In
due course of administration, she submitted a project of partition, now Exhibit "E". In
the order of November 24, 1924, now exhibit "C", the probate court approved the
project of partition and declared the proceeding closed. As the project of partition,
Exhibit "E", now shows Doa Fausta Nepomuceno received by virtue thereof the
ownership and possession of a considerable amount of real and personal estate. By
virtue also of the said project of partition, she received the use and possession of all the
real and personal properties mentioned and referred to in Clause 7th of the will. The
order approving the project of partition (Exh. "C"), however, expressly provided that
approval thereof was "sin perjuicio de lo dispuesto en la clausula 8.o del testamento de
Nicolas Villaflor." .
On May 1, 1956, Doa Fausta Nepomuceno died without having contracted a second
marriage, and without having begotten any child with the deceased Nicolas Villaflor. Her
estate is now being settled in Special Proceeding No. Q-1563 in the lower court, with
the defendant Delfin N. Juico as the duly appointed and qualified judicial administrator.
The plaintiff Leonor Villaflor Vda. de Villanueva is admitted to be the same Leonor
Villaflor mentioned by Don Nicolas Villaflor in his will as his "sobrina nieta Leonor
Villaflor".

Plaintiff Leonor Villaflor instituted the present action against the administrator of the
estate of the widow Fausta Nepomuceno, on February 8, 1958, contending that upon
the widow's death, said plaintiff became vested with the ownership of the real and
personal properties bequeathed by the late Nicolas Villaflor to clause 7 of his will,
pursuant to its eight (8th) clause. Defendant's position, adopted by the trial court, is that
the title to the properties aforesaid became absolutely vested in the widow upon her
death, on account of the fact that she never remarried.
We agree with appellant that the plain desire and intent of the testator, as manifested in
clause 8 of his testament, was to invest his widow with only a usufruct or life tenure in
the properties described in the seventh clause, subject to the further condition
(admitted by the appellee) that if the widow remarried, her rights would thereupon
cease, even during her own lifetime. That the widow was meant to have no more than a
life interest in those properties, even if she did not remarry at all, is evident from the
expressions used by the deceased "uso y posesion mientras viva" (use and possession
while alive) in which the first half of the phrase "uso y posesion" instead of "dominio" or
"propiedad") reinforces the second ("mientras viva"). The testator plainly did not give
his widow the full ownership of these particular properties, but only the right to their
possession and use (or enjoyment) during her lifetime. This is in contrast with the
remainder of the estate in which she was instituted universal heir together with the
testator's brother (clause 6). 1wph1.t
SEXTO: En virtud de las facultades que me conceden las leyes, instituyo por
mis unicos y universales herederos de todos mis derechos y acciones a mi
hermano D. Fausto Villaflor y a mi esposa Da. Fausta Nepomuceno para que
parten todos mis bienes que me pertenescan, en iguales partes, para despues
de mi muerte, exceptuando las donaciones y legados que, abajo mi mas
expontanea voluntad, lo hago en la forma siguiente.
The court below, in holding that the appellant Leonor Villaflor, as reversionary legatee,
could succeed to the properties bequeathed by clause 7 of the testament only in the
event that the widow remarried, has unwarrantedly discarded the expression "mientras
viva," and considered the words "uso y posesion" as equivalent to "dominio"
(ownership). In so doing, the trial court violated Article 791 of the Civil Code of the
Philippines, as well as section 59 of Rule 123 of the Rules of Court.

ART. 791. The words of a will are to receive an interpretation which will give to
every expression some effect, rather than one which will render any of the
expressions inoperative; and of two modes of interpreting a will, that one is to
be preferred which will prevent intestacy." .
SEC. 59. Instrument construed so as to give effect to all provisions. In the
construction of an instrument where there are several provisions or particulars,
such a construction is, if possible, to be adopted as will give effect to all." .
Speculation as to the motives of the testator in imposing the conditions contained in
clause 7 of his testament should not be allowed to obscure the clear and unambiguous
meaning of his plain words, which are over the primary source in ascertaining his intent.
It is well to note that if the testator had intended to impose as sole condition the nonremarriage of his widow, the words "uso y posesion mientras viva" would have been
unnecessary, since the widow could only remarry during her own lifetime.
The Civil Code, in Article 790, p. 1 (Article 675 of the Code of 1889), expressly enjoins
the following: .
ART. 790. The words of a will are to be taken in their ordinary and grammatical
sense, unless a clear intention to use them in another sense can be gathered,
and that other can be ascertained." .
Technical words in a will are to be taken in their technical sense, unless the
context clearly indicates a contrary intention, or unless it satisfactorily appears
that the will was drawn solely by the testator, and that he was unacquainted with
such technical sense. (675a)
In consonance with this rule, this Supreme Court has laid the doctrine in In re Estate of
Calderon, 26 Phil., 233, that the intention and wishes of the testator, when clearly
expressed in his will, constitute the fixed law of interpretation, and all questions raised
at the trial, relative to its execution and fulfillment, must be settled in accordance
therewith, following the plain and literal meaning of the testator's words, unless
it clearly appears that his intention was otherwise. The same rule is adopted by the
Supreme Court of Spain (TS. Sent. 20 Marzo 1918; 28 Mayo 1918; 30 Abril 1913; 16
Enero 1915; 23 Oct. 1925).
La voluntad del testador, clara, precisa y constantemente expresada al ordenar
su ultimo voluntad, es ley unica, imperativa y obligatoria que han de obedecer y

cumplir fieldmente albaceas, legatarios y heredera, hoy sus sucesores, sin que
esa voluntad patente, que no ha menester de interpretaciones, pues no ofrece
la menor duda, pueda sustituirse, pues no ofrece la menor duda, pueda
sustituirse por ningun otro criterio de alguna de los interesados, ni tampoco por
el judicial. (Tribunal Supremo of Spain, Sent. 20 March 1918) .
The American decisions invoked by appellee in his brief inapplicable, because they
involve cases where the only condition imposed on the legatee was that she should
remain a widow. As already shown, the testament of Don Nicolas Villaflor clearly and
unmistakably provided that his widow should have the possession and use of the
legacies while alive and did not remarry. It necessarily follows that by the express
provisions of the 8th clause of his will, the legacies should pass to the testator's
"sobrinanieta", appellant herein, upon the widow's death, even if the widow never
remarried in her lifetime. Consequently, the widow had no right to retain or dispose of
the aforesaid properties, and her estate is accountable to the reversionary legatee for
their return, unless they had been lost due to fortuitous event, or for their value should
rights of innocent third parties have intervened.
PREMISES CONSIDERED, the decision appealed from is reversed, and the appellant
Leonor Villaflor Vda. de VILLANUEVA is declared entitled to the ownership and fruits of
the properties described in clause 7 of the will or testament, from the date of the death
of Doa Fausta Nepomuceno. The records are ordered remanded to the court of origin
for liquidation, accounting and further proceedings conformably to this decision. Costs
against the Administrator-appellee.

G.R. No. L-22595

November 1, 1927

Testate Estate of Joseph G. Brimo, JUAN MICIANO, administrator, petitionerappellee, vs. ANDRE BRIMO, opponent-appellant.
Ross, Lawrence and Selph for appellant.
Camus and Delgado for appellee.

ROMUALDEZ, J.:
The partition of the estate left by the deceased Joseph G. Brimo is in question in this
case.
The judicial administrator of this estate filed a scheme of partition. Andre Brimo, one of
the brothers of the deceased, opposed it. The court, however, approved it.
The errors which the oppositor-appellant assigns are:
(1) The approval of said scheme of partition; (2) denial of his participation in the
inheritance; (3) the denial of the motion for reconsideration of the order approving the
partition; (4) the approval of the purchase made by the Pietro Lana of the deceased's
business and the deed of transfer of said business; and (5) the declaration that the
Turkish laws are impertinent to this cause, and the failure not to postpone the approval
of the scheme of partition and the delivery of the deceased's business to Pietro Lanza
until the receipt of the depositions requested in reference to the Turkish laws.
The appellant's opposition is based on the fact that the partition in question puts into
effect the provisions of Joseph G. Brimo's will which are not in accordance with the
laws of his Turkish nationality, for which reason they are void as being in violation or
article 10 of the Civil Code which, among other things, provides the following:
Nevertheless, legal and testamentary successions, in respect to the order of
succession as well as to the amount of the successional rights and the intrinsic
validity of their provisions, shall be regulated by the national law of the person
whose succession is in question, whatever may be the nature of the property or
the country in which it may be situated.

But the fact is that the oppositor did not prove that said testimentary dispositions are
not in accordance with the Turkish laws, inasmuch as he did not present any evidence
showing what the Turkish laws are on the matter, and in the absence of evidence on
such laws, they are presumed to be the same as those of the Philippines. (Lim and
Lim vs. Collector of Customs, 36 Phil., 472.)
It has not been proved in these proceedings what the Turkish laws are. He, himself,
acknowledges it when he desires to be given an opportunity to present evidence on this
point; so much so that he assigns as an error of the court in not having deferred the
approval of the scheme of partition until the receipt of certain testimony requested
regarding the Turkish laws on the matter.
The refusal to give the oppositor another opportunity to prove such laws does not
constitute an error. It is discretionary with the trial court, and, taking into consideration
that the oppositor was granted ample opportunity to introduce competent evidence, we
find no abuse of discretion on the part of the court in this particular. There is, therefore,
no evidence in the record that the national law of the testator Joseph G. Brimo was
violated in the testamentary dispositions in question which, not being contrary to our
laws in force, must be complied with and executed. lawphil.net
Therefore, the approval of the scheme of partition in this respect was not erroneous.
In regard to the first assignment of error which deals with the exclusion of the herein
appellant as a legatee, inasmuch as he is one of the persons designated as such in
will, it must be taken into consideration that such exclusion is based on the last part of
the second clause of the will, which says:
Second. I like desire to state that although by law, I am a Turkish citizen, this
citizenship having been conferred upon me by conquest and not by free choice,
nor by nationality and, on the other hand, having resided for a considerable
length of time in the Philippine Islands where I succeeded in acquiring all of the
property that I now possess, it is my wish that the distribution of my property
and everything in connection with this, my will, be made and disposed of in
accordance with the laws in force in the Philippine islands, requesting all of my
relatives to respect this wish, otherwise, I annul and cancel beforehand
whatever disposition found in this will favorable to the person or persons who
fail to comply with this request.

The institution of legatees in this will is conditional, and the condition is that the
instituted legatees must respect the testator's will to distribute his property, not in
accordance with the laws of his nationality, but in accordance with the laws of the
Philippines.
If this condition as it is expressed were legal and valid, any legatee who fails to comply
with it, as the herein oppositor who, by his attitude in these proceedings has not
respected the will of the testator, as expressed, is prevented from receiving his legacy.
The fact is, however, that the said condition is void, being contrary to law, for article 792
of the civil Code provides the following:
Impossible conditions and those contrary to law or good morals shall be
considered as not imposed and shall not prejudice the heir or legatee in any
manner whatsoever, even should the testator otherwise provide.
And said condition is contrary to law because it expressly ignores the testator's national
law when, according to article 10 of the civil Code above quoted, such national law of
the testator is the one to govern his testamentary dispositions.
Said condition then, in the light of the legal provisions above cited, is considered
unwritten, and the institution of legatees in said will is unconditional and consequently
valid and effective even as to the herein oppositor.
It results from all this that the second clause of the will regarding the law which shall
govern it, and to the condition imposed upon the legatees, is null and void, being
contrary to law.
All of the remaining clauses of said will with all their dispositions and requests are
perfectly valid and effective it not appearing that said clauses are contrary to the
testator's national law.
Therefore, the orders appealed from are modified and it is directed that the distribution
of this estate be made in such a manner as to include the herein appellant Andre Brimo
as one of the legatees, and the scheme of partition submitted by the judicial
administrator is approved in all other respects, without any pronouncement as to costs.
So ordered.

G.R. No. L-22797

September 22, 1966

TESTACY OF MAXIMA SANTOS VDA. DE BLAS. ROSALINA SANTOS


(Executrix), petitioner and appellee, vs. FLORA BLAS DE BUENAVENTURA
(Legatee), oppositor and appellant.
Vicente J. Francisco for oppositor and appellant.
J.T. de los Santos and R.M. Caluag for petitioner and appellee.

BENGZON, J.P., J.:


This is an appeal from two orders of the Court of First Instance of Rizal in Special
Proceedings No. 2524 regarding the testacy of the deceased Maxima Santos Vda. de
Blas.
On October 22, 1956, Rosalina Santos filed a petition with the Court of First Instance of
Rizal for the probate of the last will allegedly executed on September 22, 1956 by the
deceased Maxima Santos Vda. de Blas. 1 The nearest of kin of the deceased were her
brothers and a sister, nephews and nieces. Rosalinda Santos, petitioner-appellee
herein, is one of said nieces. Among the legatees or more accurately, devisees
mentioned in the will is Flora Blas de Buenaventura. She is not related by blood to the
deceased.
Flora Blas de Buenaventura and Justo Garcia filed on November 28, 1956 an
opposition to the probate of said will.
Among the grounds for the opposition of Flora Blas and Justo Garcia were that the will
was not executed in accordance with law; that undue and improper pressure was
exerted upon the testatrix Maxima Santos in the execution thereof; that the signature of
Maxima was secured through fraud; and that at the time of the execution of the will
Maxima was mentally incapable of making a will.2
After the probate court had received the evidence for both the petitioner and oppositors,
but before the latter could close their evidence, Flora Blas on November 6, 1957 filed a
manifestation that she is withdrawing her opposition to the probate of the will, quoted as
follows:

Oppositor FLORA BLAS BUENAVENTURA, assisted by her counsel, unto this


Honorable Court respectfully manifests:
1. That she is hereby withdrawing her opposition to the petition for the probate
of the will of the deceased Maxima Santos Vda. de Blas;
2. That being a legatee named in the will, to protect and preserve her rights and
interests, she hereby makes of record that she is joining the proponent of said
will for the legalization of the same.
Some circumstances leading to said withdrawal may be noted. Flora had to sell her
house for P5,000 to pay for stenographic notes of this case. Rosalina Santos thereafter
gave a party at the Manila Hotel, aimed at settling the case amicably. And there Atty.
Jose T. de los Santos appellee's lawyer took Flora aside and told her that he
learned she had sold her house, that it was a foolish thing to have done, and that for
her sake and her children's, she should withdraw her opposition and receive her legacy,
so that from its rent she could start a business.
The proceedings continued however as to the opposition of Justo Garcia.
On December 24, 1957, the court below issued an order allowing the probate of the
will. After the order had become final and executory, Flora Blas on February 27, 1958,
filed a petition praying for the delivery to her of a fishpond as a specific devise in her
favor under Item No. 3, Clause No. 6, of the will. To this petition, inspite of apparent
understanding, Rosalina Santos filed an opposition predicated on the ground that said
specific devise in favor of Flora was forfeited in favor of the other residuary heirs,
pursuant to a provision of the will that should any of the heirs, devisees or legatees
contest or oppose its probate, the latter shall lose his or her right to receive any
inheritance or benefit under it, which shall be forfeited in favor of the other heirs,
devisees and legatees.
The pertinent provisions of the will, translated into English from Tagalog, reads as
follows:
Fourteenth.I request all my heirs, devisees and legatees to look after each
other, love and help one another and accept with thanks what I have
bequeathed to them, and treasure, love and cherish the same. Any one of them
who contests or opposes the probate of my will or the carrying out of its

provisions shall lose any right to receive any inheritance or benefit under my
will, and their inheritance or share shall pertain to the other heirs who have not
opposed.3
This is known in Anglo-American jurisdiction as the "no contest and forfeiture" clause of
a will. 1awphl.nt
In its order of April 30, 1958, the court a quo sustained the theory that the "no-contest
and forfeiture" clause of the will was valid and had the effect of depriving Flora of her
devise in view of her previous opposition to its probate, which it held not justified under
the circumstances. Accordingly, it denied the motion for delivery of the specific devise,
declaring the same forfeited in favor of the other residuary heirs. Flora's motion for
reconsideration, superseded by a subsequent amended motion to the same effect, was
denied by the probate court in its order dated March 7, 1959.
From the foregoing two orders of the trial court, Flora Blas interposed an appeal to the
Court of Appeals. Said Court, in its resolution of March 25, 1964, certified the appeal to
Us as calling for determination of questions purely of law.
This appeal raises two issues: (1) Did Flora's actuations, under the facts and
circumstances herein, amount to a violation of the "no-contest and forfeiture" clause of
the will; and (2) Is the "no-contest and forfeiture" provision of the will valid?
Anent the second issue, the parties herein, relying mostly upon Spanish and AngloAmerican authorities, advance conflicting theories. Petitioner-appellee argues that the
"no-contest and forfeiture" clause is a valid, legal and efficacious testamentary
condition. Against this position, however, the devisee-appellant maintains that such
provision in a will is null and void because it is contrary to public policy.
It is, however, the first issue that We will now discuss. For this purpose, the point to
determine initially is whether or not appellant's filing of her opposition was justified
under the particular circumstances of the case; and then, whether or not a timely
withdrawal of said opposition had precluded violation of the "no contest and forfeiture
clause"
The court a quo's conclusion is that "there is no justification for her to oppose or contest
the probate of said will" because "from the evidence given by her and by her witnesses
during the pendency of the probate of the will ..., it appears that Flora Blas was aware
of the true facts surrounding the execution of the will and of the mental state of mind of

the said testatrix at the time of the execution of the will in question, and yet she has
charge her benefactor, the late Maxima Santos, as not enjoying sound mind when the
latter executed her will on September 22, 1956", and that "there is no proof to show that
the said Flora Blas was in any manner related by blood to Maxima Santos Vda. de Blas
so that her contest of the said will cannot benefit her."4
We disagree with the above conclusion of the lower court, which is not the inference
borne out by the facts and the evidence both testimonial and documentary
adduced in the case.
Appellant knew about the existence of another will executed earlier in 1953 in which
she stood to receive more much more than what is devised to her in the 1956
will.5 Since 1953 up to the death of the testatrix, appellant did not fall out of the good
graces of the deceased. Their relationship stayed as close as ever. She did not give
any cause to alienate the deceased's affections. Why, then, the supposed change of
heart?
She was addressed as Flora Buendia in the will,6 yet she has been using the name
Flora Blas as far as she could remember, apparently with the knowledge and consent
of the deceased. This is supported by her school records from grade school up to first
year pharmacy. Admittedly, it was the deceased who reared and spent for the education
of the appellant, and therefore she must have known that the latter was using the family
name Blas. If, indeed, the testatrix was not agreeable to such an arrangement why did
she not take steps to correct the same? We can only conclude that appellant's use of
the family name Blas was with the acquiescence of the testatrix. Why should she
change her mind after all the years and speak of appellant in her will as Flora Buendia
instead of Flora Blas?
There was also the coincidence that the three attesting witnesses to the will, all
brothers, are likewise the lawyers of the executrix (who will receive the biggest single
share under the will) and compadres of the assistant executrix, while the notary public
is also a compadre of one of the attesting brothers-lawyers.
Furthermore, the nurse who attended to the deceased on September 22, 1956 the
date when the will was supposedly typed and signed by that testatrix in her room at the
Manila Doctors Hospital told the appellant that there was no one inside the testatrix's
room when she went to administer medications to the old woman at the precise time
when the attesting witnesses and the notary public testified they were inside the said
room. The nurse admitted this likewise under oath (Tsn., June 10, 1957, p. 23).

But the most important single factor that should engender reasonable doubt as to the
physical and mental capacity of a person to execute a will, was the condition of Maxima
Blas as gleaned from the records of the case. She was an old woman more than 86
years old who suffered from various ailments like rheumatoid arthritis, catarrh of the
eyes, jaundice, cirrhosis of the liver, anemia, edema of the lower legs and fracture in
the vertebrae. From August 1, 1956 to September 23, 1956 she received seven blood
transfusions, as follows: one on August 1; two on September 22 (the alleged date of the
execution of the will), with barely three hours intervening; one each on September 24,
25, 26 and 29, 1956. She was also given dextrose vinoclysis on September 22,
because she could not take food through the mouth; and on September 23, 1956 she
started to bleed by mouth, compelling her doctor to cancel her trip to the United States
scheduled for September 25, 1956. Several documents executed by her before the
alleged date of execution of the will, were no longer signed but merely thumbmarked by
her,7whereas the will appealed to have been signed.
It is difficult for Us to imagine that one situated and equally faced with the above
enumerated facts and circumstances as the appellant was, should keep her peace. She
had her doubts, and to resolve them she had to conduct inquiries and investigations.
Her findings all the more strengthened her belief that there was something untoward
about the execution of the will. Thus, in her desire to know the truth and to protect her
rights, she opposed the probate of the will.
After all, had the contest been continued and the will held invalid on any of the grounds
provided by law for the disallowance of a will,8 she would have contributed in no small
measure to the cause of the truth which the courts have been in a position to apply the
proper legal provisions which are for the greater interests of the testatrix since all of
them are ordained to the idea that the truth of her last thoughts may be duly assured
and guaranteed.
Above all, the factor that preponderates in favor of appellant is that, after realizing her
mistake in contesting the will a mistake committed in good faith because grounded
on strong doubts she withdrew her opposition and joined the appellee in the latter's
petition for the probate of the will. She must not now be penalized for rectifying her
error. After all, the intentions of the testatrix had been fulfilled, her will had been
admitted and allowed probate within a reasonably short period, and the disposition of
her property can now be effected.
It should be pointed out that, contrary to the translation accorded to Paragraph
Fourteen of the will, the testatrix enjoins not a mere contest or opposition to its probate,

but a contest or opposition to the probate of the will and the carrying out of its
provisions. This is so because the questioned clause speaks of "pagpapatibay at pagbibigay-bisa" instead of "pagpapatibay o pag-bibigay-bisa."9 This furnishes a significant
index into the intention of the testatrix, namely, that she was more concerned in insuring
the carrying out of her testamentary provisions than in precluding any contest or
opposition to it. By the withdrawal of the contest which appellant brought in good faith,
no prejudice has been done into the intention of the testatrix. The dispositions of her will
can now be safely carried out.
The most that can be said, if at all, is that Flora Blas' actuations were also impelled by
some desire to gain. But who among the heirs can assume a posture of innocence and
cast the first stone? None of them can safely claim that he is not thus similarly
motivated.
From the foregoing premises it cannot be said that Flora's actuations impaired the true
intention of the testatrix in regard to the "no-contest and forfeiture" clause of the will.
Flora's act of withdrawing her opposition before she had rested her case contributed to
the speedy probation of the will. Since the withdrawal came before Flora had rested her
case, it precluded the defeat of the probate upon the strength of Flora's evidence.
Through said withdrawal, Flora conformed to the testatrix's wish that her dispositions of
her properties under the will be carried out. It follows that, taken as a whole, Flora's
actuations subserved rather than violated the testatrix's intention.
There is, therefore, no further need to discuss the second issue on the validity of a "no
contest and forfeiture" clause in this jurisdiction, since, at any rate, said clause was not
violated in this case.
Wherefore, the appealed orders dated April 30, 1958 and March 7, 1959 are hereby
reversed, and this case is remanded to the court a quo with the instruction that
appellant's devise under the will be forthwith delivered to her. No costs. So ordered.
Concepcion, C.J., Reyes, J.B.L., Barrera, Dizon, Makalintal, Zaldivar, Sanchez and
Castro, JJ., concur.
Regala J., took no part.
RESOLUTION AMENDING DECISION
November 29, 1966
BENGZON, J.P., J.:

Flora Blas de Buenaventura, oppositor-appellant, moved for reconsideration of this


Court's decision herein rendered on September 22, 1966. Against this motion the
petitioner-appellee and executrix, Rosalina Santos, filed an opposition. And appellant
filed a reply thereto.

state that appellant is also entitled to, and appellee should deliver to her, the fruits or
rents of the devised fishpond accruing after the testatrix's death. The precise
determination of the same, however, should be threshed out in the court below, before
which appellee must render an accounting.

Appellant-movant contends, first, that she is entitled to and should be awarded, not only
the devised fishpond, but all the fruits or rents of said property from the death of the
testatrix on October 5, 1956 up to the time said property will be delivered to her.
Appellant, it be noted, did not expressly seek recovery of fruits or rents in her petition
for delivery of specific legacy (devise) filed below. She started to mention also the fruits
or rents in her amended motion for reconsideration of the court a quo's denial of said
petition. And, thereafter she has raised the point in her third assignment of error in the
present appeal.

Appellee, in this regard, would bring up in this proceedings and at this very late stage,
some new matters: that allegedly the testatrix owned only 65.38-2/3% of the property
devised, so that the fruits or rents pertaining to appellant should likewise be only 65.382/3%. And in support of this, appellee refers to final decisions of this Court in two other
cases, L-14070, "Maria Gervacio Blas, et al. v. Rosalina Santos" promulgated March
29, 1961, and L-19270, "Manuel Gervacio Blas, et al. v. Hon. Cecilia Muoz-Palma, et
al.," promulgated March 31, 1962. In said decisions, it is contended that the rulings are
to the effect that the properties therein litigated belonged to Maxima Santos, the
testatrix herein, only to the extent of 65.38-2/3%, the rest being owned by her husband
Simeon Blas, represented by the plaintiffs therein. The property involved here is
allegedly one of the properties litigated therein. The foregoing cannot avail appellee
herein. She is not the proper party to raise it, since she represents the testatrix and not
Simeon Blas or his heirs. For her to do so would in effect be to assert an interest
adverse to that of the testatrix, even when those to whom said alleged interest pertains
Simeon Blas and his heirs do not advance it.

This notwithstanding, We believe that appellant should receive the fruits of the property
given to her in devise. The provisions of law regarding devised proper are emphatic in
stating that a devise of a specific things includes its fruits and income accruing after the
testator's death, ordering that these shall be delivered with the thing devised:
ART. 948. If the legacy or devise is of a specific and determinate thing
pertaining to the testator, the legatee or devisee acquires the ownership thereof
upon the death of the testator, as well as any growing fruits, or unborn offspring
of animals, or uncollected income; but not the income which was due and
unpaid before the latter's death.
From the moment of the testator's death, the thing bequeathed shall be at the
risk of the legatee or devisee, who shall, therefore, bear its loss or deterioration,
and shall be benefited by its increase or improvement, without prejudice to the
responsibility of the executor or administrator.
ART. 951. The thing bequeathed shall be delivered with all its accessions and
accessories and in the condition in which it may be upon the death of the
testator. (Civil Code)
Furthermore, since fruits or rents are accessions (Arts. 441, 442, Civil Code), strictly
speaking, there was really no need to mention them in the petition or the decision.
Article 1166 of the Civil Code applies: "The obligation to give a determinate thing
includes that of delivering all its accessions and accessories, even though they may not
have been mentioned." To remove doubts on the matter, however, We here expressly

As to appellee's reiterated contention that appellant had violated the no contest and
forfeiture clause of the will, the same has already been sufficiently discussed and
resolved in our decision. As therein stated, due to appellant's timely withdrawal of her
opposition to the probate of the will, it was as if there had been no opposition by her at
all, as far as the purpose underlying the aforestated clause is concerned.
The next argument is on appellant's claim to interest upon the fruits or rents. The rule is
that interest does not run, unless stipulated, where there is yet no delay (Arts. 1169,
2209, Civil Code). And in settlement proceedings, there is no delay on the part of the
administratrix until after the court orders her to make delivery of the legacy or devise
(Ongpin v. Rivera, 44 Phil. 808). In this case, the court a quo not having issued such an
order, appellee has not incurred in delay and is thus not liable for interest.
Appellant-movant also prays for moral and exemplary damages and would rest this
claim upon fraud allegedly committed on two different occasions: First, in the
preparation of the will; and, second, during the supposed negotiation for the withdrawal
of her opposition preparatory to the delivery of her devise.

As to the first, appellant would hereby be assailing the very basis of the right she is
asserting as devisee, for if the will was not a voluntary act of the testatrix as she would
contend, the devise in question would suffer the same defect. It should be remembered
also that the will has already been admitted to probate, so that its due execution and
authenticity, are already deemed established for purposes of this proceeding.
As to the second alleged occasion of fraud, We have on record only the parties'
allegations and denials, and the affidavit of the devisee-claimant. Fraud being a serious
charge, it is difficult to see how the same can be sustained on so insufficient an
evidence. And moreover, this being a factual issue, We cannot consider the same, for
this appeal is confined to questions purely of law. Appellant-movant's prayer for moral
and exemplary damages, therefore, is hereby denied.
As to attorney's fees, however, this Court, considering all the circumstances; believes it
reasonable and equitable to award under Article 2208, par. 11, of the Civil Code,
P5,000 in appellant's favor.
In view of the foregoing, the dispositive portion of the decision herein promulgated on
September 22, 1966 is hereby amended to read as follows:
"WHEREFORE, the appealed orders April 30, 1958 and March 7, 1959 are hereby
reversed and this case is remanded to the court a quo, with the instruction that
appellant's specific devise under the will be forthwith delivered to her by appellee
executrix, with all the fruits or rents thereof acquired from the death of the testatrix on
October 5, 1956 until its delivery, and for this purpose said appellee executrix shall
render an accounting to the court a quo. Lastly, attorney's fee of P5,000 is hereby
awarded in appellant's favor against appellee. No costs. So ordered.

G.R. No. L-3891 December 19, 1907


ELENA MORENTE, petitioner-appellant, vs. GUMERSINDO DE LA
SANTA, respondent-appellee.
Agoncillo and Ilustre, for appellant.
Agustin Alvares, for appellee.

WILLARD, J.:
The will of Consuelo Morente contains the following clauses:lawphil.net
1. I hereby order that all real estate which may belong to me shall pass to my
husband, Gumersindo de la Santa.
2. That my said husband shall not leave my brothers after my death, and that he
shall not marry anyone; should my said husband have children by anyone, he
shall not convey any portion of the property left by me, except the one-third part
thereof and the two remaining thirds shall be and remain for my brother Vicente
or his children should he have any.
3. After my death I direct my husband to dwell in the camarin in which the
bakery is located, which is one of the properties belonging to me.
Her husband, Gumersindo de la Santa, married again within four months of the death
of the testatrix. Elena Morente, a sister of the deceased, filed a petition in the
proceeding relating to the probate of the will of Consuelo Morente pending in the Court
of First Instance of the Province of Tayabas in which she alleged the second marriage
of Gumersindo de la Santa and asked that the legacy to him above-mentioned be
annulled. Objection was made in the court below by the husband to the procedure
followed by the petitioner. The court below, however, held that the proceeding was
proper and from that holding the husband did not appeal. From the judgment of the
court below, the petitioner, Elena Morente, appealed.

In its judgment the court denied the petition. It was said, however, in the decision, as
we understand it, that the husband having married, he had the right to the use of all the
property during his life and that at his death two-thirds thereof would pass to Vicente, a
brother of the testatrix, and one-third thereof could be disposed of by the husband. The
construction given to the will by the court below is not accepted by the appellant. She
claims that by the mere act of marriage the husband at once lost all rights acquired by
the will. It is neither alleged nor proven that any children have been born to the
husband since the death of the testatrix. lawphil.net
Article 790 of the Civil Code provides that testamentary provisions may be made
conditional and article 793 provides that a prohibition against another marriage may in
certain cases be validly imposed upon the widow or widower. But the question in this
case is, Did the testatrix intend to impose a condition upon the absolute gift which is
contained in the first clauses of the will? It is to be observed that by the second clause
she directs that her husband shall not leave her sisters. It is provided in the third clause
that he must continue to live in a certain building. It is provided in the second clause
that he shall not marry again. To no one of these orders is attached the condition that if
he fails to comply with them he shall lose the legacy given to him by the first clause of
the will. It is nowhere expressly said that if he does leave the testatrix's sisters, or does
not continue to dwell in the building mentioned in the will he shall forfeit the property
given him in the first clause; nor is it anywhere expressly said that if he marries again
he shall incur such a loss. But it is expressly provided that if one event does happen the
disposition of the property contained in the first clause of the will shall be changed. It is
said that if he has children by anyone, two-thirds of that property shall pass to Vicente,
the brother of the testatrix.
We are bound to construe the will with reference to all the clauses contained therein,
and with reference to such surrounding circumstances as duly appear in the case, and
after such consideration we can not say that it was the intention of the testatrix that if
her husband married again he should forfeit the legacy above mentioned. In other
words, there being no express condition attached to that legacy in reference to the
second marriage, we can not say that any condition can be implied from the context of
the will. In the case of Chiong Joc-Soy vs. Jaime Vano (8 Phil. Rep., 119), we held that
the legacy contained in the will therein mentioned was not conditional. It is true that
case arose under article 797 of the Civil Code, which perhaps is not strictly applicable
to this case, but we think that it may be argued from what is said in article 797 that, in
order to make a testamentary provision conditional, such condition must fairly appear
from the language used in the will.

Whether the children mentioned in the second clause of the will are natural children or
legitimate children we do not decide, for no such question is before us, the contingency
mentioned in that part of the clause not having arisen, and we limit ourselves to saying
merely that by the subsequent marriage of the husband he did not forfeit the legacy
given to him by the first part of the will. That was the only question before the court
below. the judgment of that court, denying the petition, is accordingly affirmed, with the
costs of this instance against the appellant. So ordered.

G.R. No. 149926

February 23, 2005

UNION BANK OF THE PHILIPPINES, petitioner, vs. EDMUND SANTIBAEZ and


FLORENCE SANTIBAEZ ARIOLA, respondents.
DECISION
CALLEJO, SR., J.:
Before us is a petition for review on certiorari under Rule 45 of the Revised Rules of
Court which seeks the reversal of the Decision1 of the Court of Appeals dated May 30,
2001 in CA-G.R. CV No. 48831 affirming the dismissal2 of the petitioners complaint in
Civil Case No. 18909 by the Regional Trial Court (RTC) of Makati City, Branch 63.
The antecedent facts are as follows:
On May 31, 1980, the First Countryside Credit Corporation (FCCC) and Efraim M.
Santibaez entered into a loan agreement3 in the amount of P128,000.00. The amount
was intended for the payment of the purchase price of one (1) unit Ford 6600
Agricultural All-Purpose Diesel Tractor. In view thereof, Efraim and his son, Edmund,
executed a promissory note in favor of the FCCC, the principal sum payable in five
equal annual amortizations ofP43,745.96 due on May 31, 1981 and every May 31st
thereafter up to May 31, 1985.
On December 13, 1980, the FCCC and Efraim entered into another loan
agreement,4 this time in the amount ofP123,156.00. It was intended to pay the balance
of the purchase price of another unit of Ford 6600 Agricultural All-Purpose Diesel
Tractor, with accessories, and one (1) unit Howard Rotamotor Model AR 60K. Again,
Efraim and his son, Edmund, executed a promissory note for the said amount in favor
of the FCCC. Aside from such promissory note, they also signed a Continuing Guaranty
Agreement5 for the loan dated December 13, 1980.
Sometime in February 1981, Efraim died, leaving a holographic will.6 Subsequently in
March 1981, testate proceedings commenced before the RTC of Iloilo City, Branch 7,
docketed as Special Proceedings No. 2706. On April 9, 1981, Edmund, as one of the
heirs, was appointed as the special administrator of the estate of the decedent.7 During
the pendency of the testate proceedings, the surviving heirs, Edmund and his sister
Florence Santibaez Ariola, executed a Joint Agreement8 dated July 22, 1981, wherein

they agreed to divide between themselves and take possession of the three (3)
tractors; that is, two (2) tractors for Edmund and one (1) tractor for Florence. Each of
them was to assume the indebtedness of their late father to FCCC, corresponding to
the tractor respectively taken by them.
On August 20, 1981, a Deed of Assignment with Assumption of Liabilities9 was
executed by and between FCCC and Union Savings and Mortgage Bank, wherein the
FCCC as the assignor, among others, assigned all its assets and liabilities to Union
Savings and Mortgage Bank.
Demand letters10 for the settlement of his account were sent by petitioner Union Bank of
the Philippines (UBP) to Edmund, but the latter failed to heed the same and refused to
pay. Thus, on February 5, 1988, the petitioner filed a Complaint11 for sum of money
against the heirs of Efraim Santibaez, Edmund and Florence, before the RTC of
Makati City, Branch 150, docketed as Civil Case No. 18909. Summonses were issued
against both, but the one intended for Edmund was not served since he was in the
United States and there was no information on his address or the date of his return to
the Philippines.12 Accordingly, the complaint was narrowed down to respondent
Florence S. Ariola.
On December 7, 1988, respondent Florence S. Ariola filed her Answer13 and alleged
that the loan documents did not bind her since she was not a party thereto. Considering
that the joint agreement signed by her and her brother Edmund was not approved by
the probate court, it was null and void; hence, she was not liable to the petitioner under
the joint agreement.
On January 29, 1990, the case was unloaded and re-raffled to the RTC of Makati City,
Branch 63.14Consequently, trial on the merits ensued and a decision was subsequently
rendered by the court dismissing the complaint for lack of merit. The decretal portion of
the RTC decision reads:
WHEREFORE, judgment is hereby rendered DISMISSING the complaint for lack of
merit.15
The trial court found that the claim of the petitioner should have been filed with the
probate court before which the testate estate of the late Efraim Santibaez was
pending, as the sum of money being claimed was an obligation incurred by the said
decedent. The trial court also found that the Joint Agreement apparently executed by

his heirs, Edmund and Florence, on July 22, 1981, was, in effect, a partition of the
estate of the decedent. However, the said agreement was void, considering that it had
not been approved by the probate court, and that there can be no valid partition until
after the will has been probated. The trial court further declared that petitioner failed to
prove that it was the now defunct Union Savings and Mortgage Bank to which the
FCCC had assigned its assets and liabilities. The court also agreed to the contention of
respondent Florence S. Ariola that the list of assets and liabilities of the FCCC assigned
to Union Savings and Mortgage Bank did not clearly refer to the decedents account.
Ruling that the joint agreement executed by the heirs was null and void, the trial court
held that the petitioners cause of action against respondent Florence S. Ariola must
necessarily fail.
The petitioner appealed from the RTC decision and elevated its case to the Court of
Appeals (CA), assigning the following as errors of the trial court:
1. THE COURT A QUO ERRED IN FINDING THAT THE JOINT AGREEMENT
(EXHIBIT A) SHOULD BE APPROVED BY THE PROBATE COURT.

The appellate court found that the appeal was not meritorious and held that the
petitioner should have filed its claim with the probate court as provided under Sections
1 and 5, Rule 86 of the Rules of Court. It further held that the partition made in the
agreement was null and void, since no valid partition may be had until after the will has
been probated. According to the CA, page 2, paragraph (e) of the holographic will
covered the subject properties (tractors) in generic terms when the deceased referred
to them as "all other properties." Moreover, the active participation of respondent
Florence S. Ariola in the case did not amount to a waiver. Thus, the CA affirmed the
RTC decision, viz.:
WHEREFORE, premises considered, the appealed Decision of the Regional Trial Court
of Makati City, Branch 63, is hereby AFFIRMED in toto.
SO ORDERED.18
In the present recourse, the petitioner ascribes the following errors to the CA:
I.

2. THE COURT A QUO ERRED IN FINDING THAT THERE CAN BE NO VALID


PARTITION AMONG THE HEIRS UNTIL AFTER THE WILL HAS BEEN
PROBATED.
3. THE COURT A QUO ERRED IN NOT FINDING THAT THE DEFENDANT
HAD WAIVED HER RIGHT TO HAVE THE CLAIM RE-LITIGATED IN THE
ESTATE PROCEEDING.16
The petitioner asserted before the CA that the obligation of the deceased had passed to
his legitimate children and heirs, in this case, Edmund and Florence; the unconditional
signing of the joint agreement marked as Exhibit "A" estopped respondent Florence S.
Ariola, and that she cannot deny her liability under the said document; as the
agreement had been signed by both heirs in their personal capacity, it was no longer
necessary to present the same before the probate court for approval; the property
partitioned in the agreement was not one of those enumerated in the holographic will
made by the deceased; and the active participation of the heirs, particularly respondent
Florence S. Ariola, in the present ordinary civil action was tantamount to a waiver to relitigate the claim in the estate proceedings.
On the other hand, respondent Florence S. Ariola maintained that the money claim of
the petitioner should have been presented before the probate court.17

THE HONORABLE COURT OF APPEALS ERRED IN FINDING THAT THE JOINT


AGREEMENT SHOULD BE APPROVED BY THE PROBATE COURT.
II.
THE COURT OF APPEALS ERRED IN FINDING THAT THERE CAN BE NO VALID
PARTITION AMONG THE HEIRS OF THE LATE EFRAIM SANTIBAEZ UNTIL AFTER
THE WILL HAS BEEN PROBATED.
III.
THE COURT OF APPEALS ERRED IN NOT FINDING THAT THE RESPONDENT HAD
WAIVED HER RIGHT TO HAVE THE CLAIM RE-LITIGATED IN THE ESTATE
PROCEEDING.
IV.

RESPONDENTS CAN, IN FACT, BE HELD JOINTLY AND SEVERALLY LIABLE WITH


THE PRINCIPAL DEBTOR THE LATE EFRAIM SANTIBAEZ ON THE STRENGTH
OF THE CONTINUING GUARANTY AGREEMENT EXECUTED IN FAVOR OF
PETITIONER-APPELLANT UNION BANK.
V.
THE PROMISSORY NOTES DATED MAY 31, 1980 IN THE SUM OF P128,000.00
AND DECEMBER 13, 1980 IN THE AMOUNT OF P123,000.00 CATEGORICALLY
ESTABLISHED THE FACT THAT THE RESPONDENTS BOUND THEMSELVES
JOINTLY AND SEVERALLY LIABLE WITH THE LATE DEBTOR EFRAIM
SANTIBAEZ IN FAVOR OF PETITIONER UNION BANK.19
The petitioner claims that the obligations of the deceased were transmitted to the heirs
as provided in Article 774 of the Civil Code; there was thus no need for the probate
court to approve the joint agreement where the heirs partitioned the tractors owned by
the deceased and assumed the obligations related thereto. Since respondent Florence
S. Ariola signed the joint agreement without any condition, she is now estopped from
asserting any position contrary thereto. The petitioner also points out that the
holographic will of the deceased did not include nor mention any of the tractors subject
of the complaint, and, as such was beyond the ambit of the said will. The active
participation and resistance of respondent Florence S. Ariola in the ordinary civil action
against the petitioners claim amounts to a waiver of the right to have the claim
presented in the probate proceedings, and to allow any one of the heirs who executed
the joint agreement to escape liability to pay the value of the tractors under
consideration would be equivalent to allowing the said heirs to enrich themselves to the
damage and prejudice of the petitioner.
The petitioner, likewise, avers that the decisions of both the trial and appellate courts
failed to consider the fact that respondent Florence S. Ariola and her brother Edmund
executed loan documents, all establishing thevinculum juris or the legal bond between
the late Efraim Santibaez and his heirs to be in the nature of a solidary obligation.
Furthermore, the Promissory Notes dated May 31, 1980 and December 13, 1980
executed by the late Efraim Santibaez, together with his heirs, Edmund and
respondent Florence, made the obligation solidary as far as the said heirs are
concerned. The petitioner also proffers that, considering the express provisions of the
continuing guaranty agreement and the promissory notes executed by the named
respondents, the latter must be held liable jointly and severally liable thereon. Thus,
there was no need for the petitioner to file its money claim before the probate court.

Finally, the petitioner stresses that both surviving heirs are being sued in their
respective personal capacities, not as heirs of the deceased.
In her comment to the petition, respondent Florence S. Ariola maintains that the
petitioner is trying to recover a sum of money from the deceased Efraim Santibaez;
thus the claim should have been filed with the probate court. She points out that at the
time of the execution of the joint agreement there was already an existing probate
proceedings of which the petitioner knew about. However, to avoid a claim in the
probate court which might delay payment of the obligation, the petitioner opted to
require them to execute the said agreement.1a\^/phi1.net
According to the respondent, the trial court and the CA did not err in declaring that the
agreement was null and void. She asserts that even if the agreement was voluntarily
executed by her and her brother Edmund, it should still have been subjected to the
approval of the court as it may prejudice the estate, the heirs or third parties.
Furthermore, she had not waived any rights, as she even stated in her answer in the
court a quo that the claim should be filed with the probate court. Thus, the petitioner
could not invoke or claim that she is in estoppel.
Respondent Florence S. Ariola further asserts that she had not signed any continuing
guaranty agreement, nor was there any document presented as evidence to show that
she had caused herself to be bound by the obligation of her late father.
The petition is bereft of merit.
The Court is posed to resolve the following issues: a) whether or not the partition in the
Agreement executed by the heirs is valid; b) whether or not the heirs assumption of the
indebtedness of the deceased is valid; and c) whether the petitioner can hold the heirs
liable on the obligation of the deceased.1awphi1.nt
At the outset, well-settled is the rule that a probate court has the jurisdiction to
determine all the properties of the deceased, to determine whether they should or
should not be included in the inventory or list of properties to be administered.20 The
said court is primarily concerned with the administration, liquidation and distribution of
the estate.21
In our jurisdiction, the rule is that there can be no valid partition among the heirs until
after the will has been probated:

In testate succession, there can be no valid partition among the heirs until after the will
has been probated. The law enjoins the probate of a will and the public requires it,
because unless a will is probated and notice thereof given to the whole world, the right
of a person to dispose of his property by will may be rendered nugatory. The
authentication of a will decides no other question than such as touch upon the capacity
of the testator and the compliance with those requirements or solemnities which the law
prescribes for the validity of a will.22
This, of course, presupposes that the properties to be partitioned are the same
properties embraced in the will.23In the present case, the deceased, Efraim Santibaez,
left a holographic will24 which contained, inter alia, the provision which reads as follows:
(e) All other properties, real or personal, which I own and may be discovered later after
my demise, shall be distributed in the proportion indicated in the immediately preceding
paragraph in favor of Edmund and Florence, my children.
We agree with the appellate court that the above-quoted is an all-encompassing
provision embracing all the properties left by the decedent which might have escaped
his mind at that time he was making his will, and other properties he may acquire
thereafter. Included therein are the three (3) subject tractors. This being so, any
partition involving the said tractors among the heirs is not valid. The joint
agreement25 executed by Edmund and Florence, partitioning the tractors among
themselves, is invalid, specially so since at the time of its execution, there was already
a pending proceeding for the probate of their late fathers holographic will covering the
said tractors.
It must be stressed that the probate proceeding had already acquired jurisdiction over
all the properties of the deceased, including the three (3) tractors. To dispose of them in
any way without the probate courts approval is tantamount to divesting it with
jurisdiction which the Court cannot allow.26 Every act intended to put an end to
indivision among co-heirs and legatees or devisees is deemed to be a partition,
although it should purport to be a sale, an exchange, a compromise, or any other
transaction.27 Thus, in executing any joint agreement which appears to be in the nature
of an extra-judicial partition, as in the case at bar, court approval is imperative, and the
heirs cannot just divest the court of its jurisdiction over that part of the estate. Moreover,
it is within the jurisdiction of the probate court to determine the identity of the heirs of
the decedent.28 In the instant case, there is no showing that the signatories in the joint
agreement were the only heirs of the decedent. When it was executed, the probate of
the will was still pending before the court and the latter had yet to determine who the

heirs of the decedent were. Thus, for Edmund and respondent Florence S. Ariola to
adjudicate unto themselves the three (3) tractors was a premature act, and prejudicial
to the other possible heirs and creditors who may have a valid claim against the estate
of the deceased.
The question that now comes to fore is whether the heirs assumption of the
indebtedness of the decedent is binding. We rule in the negative. Perusing the joint
agreement, it provides that the heirs as parties thereto "have agreed to divide between
themselves and take possession and use the above-described chattel and each of
them to assume the indebtedness corresponding to the chattel taken as herein after
stated which is in favor of First Countryside Credit Corp."29 The assumption of liability
was conditioned upon the happening of an event, that is, that each heir shall take
possession and use of their respective share under the agreement. It was made
dependent on the validity of the partition, and that they were to assume the
indebtedness corresponding to the chattel that they were each to receive. The partition
being invalid as earlier discussed, the heirs in effect did not receive any such tractor. It
follows then that the assumption of liability cannot be given any force and effect.
The Court notes that the loan was contracted by the decedent.l^vvphi1.net The
petitioner, purportedly a creditor of the late Efraim Santibaez, should have thus filed its
money claim with the probate court in accordance with Section 5, Rule 86 of the
Revised Rules of Court, which provides:
Section 5. Claims which must be filed under the notice. If not filed barred; exceptions.
All claims for money against the decedent, arising from contract, express or implied,
whether the same be due, not due, or contingent, all claims for funeral expenses for the
last sickness of the decedent, and judgment for money against the decedent, must be
filed within the time limited in the notice; otherwise they are barred forever, except that
they may be set forth as counterclaims in any action that the executor or administrator
may bring against the claimants. Where an executor or administrator commences an
action, or prosecutes an action already commenced by the deceased in his lifetime, the
debtor may set forth by answer the claims he has against the decedent, instead of
presenting them independently to the court as herein provided, and mutual claims may
be set off against each other in such action; and if final judgment is rendered in favor of
the defendant, the amount so determined shall be considered the true balance against
the estate, as though the claim had been presented directly before the court in the
administration proceedings. Claims not yet due, or contingent, may be approved at their
present value.

The filing of a money claim against the decedents estate in the probate court is
mandatory.30 As we held in the vintage case of Py Eng Chong v. Herrera:31

the requisite notoriety exists; and every reasonable doubt upon the subject should be
promptly resolved in the negative." (Republic vs. Court of Appeals, 107 SCRA 504).36

This requirement is for the purpose of protecting the estate of the deceased by
informing the executor or administrator of the claims against it, thus enabling him to
examine each claim and to determine whether it is a proper one which should be
allowed. The plain and obvious design of the rule is the speedy settlement of the affairs
of the deceased and the early delivery of the property to the distributees, legatees, or
heirs. `The law strictly requires the prompt presentation and disposition of the claims
against the decedent's estate in order to settle the affairs of the estate as soon as
possible, pay off its debts and distribute the residue.32

This being the case, the petitioners personality to file the complaint is wanting.
Consequently, it failed to establish its cause of action. Thus, the trial court did not err in
dismissing the complaint, and the CA in affirming the same.

Perusing the records of the case, nothing therein could hold private respondent
Florence S. Ariola accountable for any liability incurred by her late father. The
documentary evidence presented, particularly the promissory notes and the continuing
guaranty agreement, were executed and signed only by the late Efraim Santibaez and
his son Edmund. As the petitioner failed to file its money claim with the probate court, at
most, it may only go after Edmund as co-maker of the decedent under the said
promissory notes and continuing guaranty, of course, subject to any defenses Edmund
may have as against the petitioner. As the court had not acquired jurisdiction over the
person of Edmund, we find it unnecessary to delve into the matter further.
We agree with the finding of the trial court that the petitioner had not sufficiently shown
that it is the successor-in-interest of the Union Savings and Mortgage Bank to which the
FCCC assigned its assets and liabilities.33 The petitioner in its complaint alleged that
"by virtue of the Deed of Assignment dated August 20, 1981 executed by and between
First Countryside Credit Corporation and Union Bank of the Philippines"34 However,
the documentary evidence35 clearly reflects that the parties in the deed of assignment
with assumption of liabilities were the FCCC, and the Union Savings and Mortgage
Bank, with the conformity of Bancom Philippine Holdings, Inc. Nowhere can the
petitioners participation therein as a party be found. Furthermore, no documentary or
testimonial evidence was presented during trial to show that Union Savings and
Mortgage Bank is now, in fact, petitioner Union Bank of the Philippines. As the trial
court declared in its decision:
[T]he court also finds merit to the contention of defendant that plaintiff failed to prove
or did not present evidence to prove that Union Savings and Mortgage Bank is now the
Union Bank of the Philippines. Judicial notice does not apply here. "The power to take
judicial notice is to [be] exercised by the courts with caution; care must be taken that

IN LIGHT OF ALL THE FOREGOING, the petition is hereby DENIED. The assailed
Court of Appeals Decision is AFFIRMED. No costs.
SO ORDERED.

G.R. No. 138842

October 18, 2000

NATIVIDAD P. NAZARENO, MAXIMINO P. NAZARENO, JR., petitioners, vs. COURT


OF APPEALS, ESTATE OF MAXIMINO A. NAZARENO, SR., ROMEO P. NAZARENO
and ELIZA NAZARENO, respondents.

DEED OF ABSOLUTE SALE


KNOW ALL MEN BY THESE PRESENTS:
I, MAXIMINO A. NAZARENO, Filipino, married to Aurea Poblete-Nazareno, of legal age
and a resident of the Mun. of Naic, Prov. of Cavite, Philippines,

DECISION
-WITNESSETHMENDOZA, J.:
This is a petition for review on certiorari of the decision1 of the Court of Appeals in CAGR CV No. 39441 dated May 29, 1998 affirming with modifications the decision of the
Regional Trial Court, Branch 107, Quezon City, in an action for annulment of sale and
damages.
The facts are as follows:
Maximino Nazareno, Sr. and Aurea Poblete were husband and wife. Aurea died on April
15, 1970, while Maximino, Sr. died on December 18, 1980. They had five children,
namely, Natividad, Romeo, Jose, Pacifico, and Maximino, Jr. Natividad and Maximino,
Jr. are the petitioners in this case, while the estate of Maximino, Sr., Romeo, and his
wife Eliza Nazareno are the respondents.

That I am the absolute registered owner of six (6) parcels of land with the
improvements thereon situated in Quezon City, Philippines, which parcels of land are
herewith described and bounded as follows, to wit:
"TRANS. CERT. OF TITLE NO. 140946"
"A parcel of land (Lot 3-B of the subdivision plan Psd-47404, being a portion of Lot 3,
Block D-3 described on plan Bsd-10642, G.L.R.O. Record No.) situated in the Quirino
District, Quezon City. Bounded on the N., along line 1-2 by Lot 15, Block D-3 of plan
Bsd - 10642; along line 2-3 by Lot 4, Block D-3 of plan Bsd-10642; along line 3-4 by
Aurora Boulevard (Road Lot-1, Bsd-10642); and along line 4-1 by Lot 3-D of the
subdivision plan. Beginning at a point marked "1" on plan, being S.29 deg. 26E.,
1156.22 m. from B.L.L.M. 9, Quezon City,

During their marriage, Maximino Nazareno, Sr. and Aurea Poblete acquired properties
in Quezon City and in the Province of Cavite. It is the ownership of some of these
properties that is in question in this case.

thence N. 79 deg. 53E., 12.50 m. to point 2;

It appears that after the death of Maximino, Sr., Romeo filed an intestate case in the
Court of First Instance of Cavite, Branch XV, where the case was docketed as Sp. Proc.
No. NC-28. Upon the reorganization of the courts in 1983, the case was transferred to
the Regional Trial Court of Naic, Cavite. Romeo was appointed administrator of his
fathers estate.

thence S. 79 deg. 53W., 12.50 m. to point 4;

In the course of the intestate proceedings, Romeo discovered that his parents had
executed several deeds of sale conveying a number of real properties in favor of his
sister, Natividad. One of the deeds involved six lots in Quezon City which were
allegedly sold by Maximino, Sr., with the consent of Aurea, to Natividad on January 29,
1970 for the total amount of P47,800.00. The Deed of Absolute Sale reads as follows:

thence S. 10 deg. 07E., 40.00 m. to point 3;

thence N. 10 deg. 07W., 40.00 m. to the point


of beginning; containing an area of FIVE HUNDRED (500) SQUARE METERS. All
points referred to are indicated on the plan and are marked on the ground as follows:
points "1" and "4" by P.L.S. Cyl. Conc. Mons. bearings true; date of the original survey,
April 8-July 15, 1920 and that of the subdivision survey, March 25, 1956."
"TRANS. CERT. OF TITLE NO. 132019"

"A parcel of land (Lot 3, Block 93 of the subdivision plan Psd-57970 being a portion of
Lot 6, Pcs-4786, G.L.R.O. Rec. No. 917) situated in Quirino District Quezon City.
Bounded on the NW., along line 1-2, by Lot 1, Block 93; on the NE., along line 2-3, by
Road Lot 101; on the SE., along line 3-4, by Road Lot 100; on the SW., along line 4-1,
by Lot 4, Block 93; all of the subdivision plan. Beginning at point marked "1" on plan,
being S. 65 deg. 40 3339.92 m. from B.L.L.M. No. 1, Marikina, Rizal;

thence N. 64 deg. 59E., 29.99 m. to the point of


beginning; containing an area of THREE HUNDRED SIXTY SQUARE METERS (360),
more or less. All points referred to are indicated on the plan and on the ground are
marked by P.L.S. Conc. Mons. 15 x 60 cm.; bearings true; declination 0 deg. 50E., date
of the original survey, April 8 to July 15, 1920, and that of the consolidation and
subdivision survey, April 24 to 26, 1941."

thence N. 23 deg. 28 min. E., 11.70 m. to point "2";


"TRANS. CERT. OF TITLE NO. 118886"
thence S. 66 deg. 32 min. E., 18.00 m. to point "3";
thence S. 23 deg. 28 min. W., 11.70 m. to point "4";
thence N. 66 deg. 32. min. W., 18.00 m. to the point
of beginning; containing an area of TWO HUNDRED TEN SQUARE METERS AND
SIXTY SQUARE DECIMETERS (210.60). All points referred to are indicated on the
plan and are marked on the ground by B.L. Cyl. Conc. Mons. 15 x 60 cm.; bearings
true; date of the original survey, Nov. 10, 1920 and Jan. 31-March 31, 1924 and that of
the subdivision survey, February 1 to September 30, 1954. Date approved - March 9,
1962."

"A parcel of land (Lot No. 11, of the consolidation and subdivision plan Pcs-988, being a
portion of the consolidated Lot No. 26, Block No. 6, Psd-127, and Lots Nos. 27-A and
27-B, Psd-14901, G.L.R.O. Record No. 917), situated in the District of Cubao, Quezon
City, Island of Luzon. Bounded on the NE., by Lot No. 4 of the consolidation and
subdivision plan; on the SE., by Lot No. 12 of the consolidation and subdivision plan; on
the SW., by Lot No. 3 of the consolidation and subdivision plan; on the NW., by Lot No.
10 of the consolidation and subdivision plan. Beginning at a point marked "1" on plan,
being S. 79 deg. 07W., 4264.00 m. more or less from B.L.L.M. No. 1, Mp. of Mariquina;
thence S. 64 deg. 59W., 29.99 m. to point "2";
thence N. 25 deg. 00W., 12.00 m. to point "3";

"TRANS. CERT. OF TITLE NO. 118885"


thence N. 64 deg. 59E., 29.99 m. to point "4";
"A parcel of land (Lot No. 10, of the consolidation and subdivision plan Pcs-988, being a
portion of the consolidated Lot No. 26, Block No. 6, Psd-127, and Lots Nos. 27-A and
27-B, Psd-14901, G.L.R.O. Record No. 917), situated in the District of Cubao, Quezon
City, Island of Luzon. Bounded on the NE., by Lot No. 4 of the consolidation and
subdivision plan; on the SE., by Lot No. 11 of the consolidation and subdivision plan; on
the SW., by Lot No. 3 of the consolidation and subdivision plan; and on the NW., by Lot
No. 9 of the consolidation and subdivision plan. Beginning at a point marked "1" on the
plan, being S. 7 deg. 26W., 4269.90 m. more or less from B.L.L.M. No. 1, Mp. of
Mariquina;
thence S. 25 deg. 00E., 12.00 m. to point "2";
thence S. 64 deg. 59W., 29.99 m. to point "3";
thence N. 25 deg. 00W., 12.00 m to point "4";

thence S. 26 deg. 00E., 12.00 m. to the point of


beginning; containing an area of THREE HUNDRED SIXTY SQUARE METERS (360),
more or less. All points referred to are indicated on the plan and on the ground, are
marked by P.L.S. Conc. Mons. 15 x 60 cm.; bearings true; declination 0 deg. 50E.; date
of the original survey, April 8 to July 15, 1920, and that of the consolidation and
subdivision survey, April 24 to 26, 1941."
"A parcel of land (Lot No. 13 of the consolidation and subdivision plan Pcs-988, being a
portion of the consolidated Lot No. 26, Block No. 6, Psd-127, and Lots Nos. 27-A and
27-B, Psd-14901, G.L.R.O. Record No. 917), situated in the District of Cubao, Quezon
City, Island of Luzon. Bounded on the NE., by Lot No. 4 of the consolidation and
subdivision plan; on the SE., by Lot No. 14, of the consolidation; and subdivision plan;
on the SW., by Lot No. 3 of the consolidation and subdivision plan; and on the NW., by

Lot No. 12, of the consolidation and subdivision plan. Beginning at the point marked "1"
on plan, being S.78 deg. 48W., 4258.20 m. more or less from B.L.L.M. No. 1, Mp. of
Mariquina;
thence S. 64 deg. 58W., 30.00 m. to point "2";
thence N. 25 deg. 00W., 12.00 m. to point "3";
thence N. 64 deg. 59E., 29.99 m. to point "4";
thence S.25 deg. 00E., 12.00 m. to point of
beginning; containing an area of THREE HUNDRED SIXTY SQUARE METERS (360,
more or less. All points referred to are indicated on the plan and on the ground are
marked by P.L.S. Conc. Mons. 15 x 60 cm.; bearings true; declination 0 deg. 50E., date
of the original survey, April 8 to July 15, 1920, and that of the consolidation and
subdivision survey, April 24 to 26, 1941."
"A parcel of land (Lot No. 14, of the consolidation and subdivision plan Pcs-988, being a
portion of the consolidated Lot No. 26, Block No. 6, Psd-127, and Lots Nos. 27-A and
27-B, Psd-14901, G.L.R.O. Record No. 917), situated in the District of Cubao, Quezon
City, Island of Luzon. Bounded on the NE., by Lot No. 4 of the consolidation and
subdivision plan; on the SE., by Lot No. 15, of the consolidation and subdivision plan;
on the SW., by Lot No. 3 of the consolidation and subdivision plan; and on the NW., by
Lot No. 13 of the consolidation and subdivision plan. Beginning at the point marked "1"
on plan, being S.78 deg. 48W., 4258.20 m. more or less from B.L.L.M. No. 1, Mp. of
Mariquina;
thence S. 25 deg. 00E., 12.00 m. to point "2";
thence S. 65 deg. 00W., 30.00 m. to point "3";
thence S. 65 deg. 00W., 12.00 m. to point "4";
thence N.64 deg. 58E., 30.00 m. to the point of
beginning; containing an area of THREE HUNDRED SIXTY SQUARE METERS (360),
more or less. All points referred to are indicated on the plan and on the ground are
marked by P.L.S. Conc. Mons. 15 x 60 cm.; bearings true; declination 0 deg. 50E., date

of the original survey, April 8 to July 15, 1920, and that of the consolidation and
subdivision survey, April 24 to 26, 1941."
That for and in consideration of the sum of FORTY THREE THOUSAND PESOS
(P43,000.00) PHILIPPINE CURRENCY, to me in hand paid by NATIVIDAD P.
NAZARENO, Filipino, single, of legal age and a resident of the Mun. of Naic, Prov. of
Cavite, Philippines, the receipt whereof is acknowledged to my entire satisfaction, I do
hereby CEDE, SELL, TRANSFER, CONVEY and ASSIGN unto the said Natividad P.
Nazareno, her heirs, administrators and assigns, all my title, rights, interests and
participations to the abovedescribed parcels of land with the improvements thereon,
with the exception of LOT NO. 11 COVERED BY T.C.T. NO. 118886, free of any and all
liens and encumbrances; and
That for and in consideration of the sum of FOUR THOUSAND EIGHT HUNDRED
PESOS (P4,800.00) PHILIPPINE CURRENCY, to me in hand paid by NATIVIDAD P.
NAZARENO, Filipino, single, of legal age and a resident of the Mun. of Naic, Prov. of
Cavite, Philippines, the receipt whereof is acknowledged to my entire satisfaction, I do
hereby CEDE, SELL, TRANSFER, CONVEY and ASSIGN unto the said Natividad P.
Nazareno, her heirs, administrators and assigns, all my title, rights, interests and
participations in and to Lot No. 11 covered by T.C.T. No. 118886 above-described, free
of any and all liens and encumbrances, with the understanding that the title to be
issued in relation hereto shall be separate and distinct from the title to be issued in
connection with Lots Nos. 13 and 14, although covered by the same title.
IN WITNESS WHEREOF, I have hereunto signed this deed of absolute sale in the City
of Manila, Philippines, this 29th day of January, 1970.2
By virtue of this deed, transfer certificates of title were issued to Natividad, to wit: TCT
No. 162738 (Lot 3-B),3 TCT No. 162739 (Lot 3),4 TCT No. 162735 (Lot 10),5 TCT No.
162736 (Lot 11),6 and TCT No. 162737 (Lots 13 and 14),7 all of the Register of Deeds of
Quezon City.
Among the lots covered by the above Deed of Sale is Lot 3-B which is registered under
TCT No. 140946. This lot had been occupied by Romeo, his wife Eliza, and by
Maximino, Jr. since 1969. Unknown to Romeo, Natividad sold Lot 3-B on July 31, 1982
to Maximino, Jr.,8 for which reason the latter was issued TCT No. 293701 by the
Register of Deeds of Quezon City.9

When Romeo found out about the sale to Maximino, Jr., he and his wife Eliza locked
Maximino, Jr. out of the house. On August 4, 1983, Maximino, Jr. brought an action for
recovery of possession and damages with prayer for writs of preliminary injunction and
mandatory injunction with the Regional Trial Court of Quezon City. On December 12,
1986, the trial court ruled in favor of Maximino, Jr. In CA-G.R. CV No. 12932, the Court
of Appeals affirmed the decision of the trial court.10
On June 15, 1988, Romeo in turn filed, on behalf of the estate of Maximino, Sr., the
present case for annulment of sale with damages against Natividad and Maximino, Jr.
The case was filed in the Regional Trial Court of Quezon City, where it was docketed as
Civil Case No. 88-58.11 Romeo sought the declaration of nullity of the sale made on
January 29, 1970 to Natividad and that made on July 31, 1982 to Maximino, Jr. on the
ground that both sales were void for lack of consideration.
On March 1, 1990, Natividad and Maximino, Jr. filed a third-party complaint against the
spouses Romeo and Eliza.12 They alleged that Lot 3, which was included in the Deed of
Absolute Sale of January 29, 1970 to Natividad, had been surreptitiously appropriated
by Romeo by securing for himself a new title (TCT No. 277968) in his name.13 They
alleged that Lot 3 is being leased by the spouses Romeo and Eliza to third persons.
They therefore sought the annulment of the transfer to Romeo and the cancellation of
his title, the eviction of Romeo and his wife Eliza and all persons claiming rights from
Lot 3, and the payment of damages.
The issues having been joined, the case was set for trial. Romeo presented evidence to
show that Maximino and Aurea Nazareno never intended to sell the six lots to Natividad
and that Natividad was only to hold the said lots in trust for her siblings. He presented
the Deed of Partition and Distribution dated June 28, 1962 executed by Maximino Sr.
and Aurea and duly signed by all of their children, except Jose, who was then abroad
and was represented by their mother, Aurea. By virtue of this deed, the nine lots subject
of this Deed of Partition were assigned by raffle as follows:
1. Romeo - Lot 25-L (642 m2)
2. Natividad - Lots 23 (312 m2) and 24 (379 m2)
3. Maximino, Jr. - Lots 6 (338 m2) and 7 (338 m2)
4. Pacifico - Lots 13 (360 m2) and 14 (360 m2)

5. Jose - Lots 10 (360 m2) and 11 (360 m2)


Romeo received the title to Lot 25-L under his name,14 while Maximino, Jr. received
Lots 6 and 7 through a Deed of Sale dated August 16, 1966 for the amount
of P9,500.00.15 Pacifico and Joses shares were allegedly given to Natividad, who
agreed to give Lots 10 and 11 to Jose, in the event the latter came back from abroad.
Natividads share, on the other hand, was sold to third persons16 because she allegedly
did not like the location of the two lots. But, Romeo said, the money realized from the
sale was given to Natividad.
Romeo also testified that Lot 3-B was bought for him by his father, while Lot 3 was sold
to him for P7,000.00 by his parents on July 4, 1969.17 However, he admitted that a
document was executed by his parents transferring six properties in Quezon City, i.e.,
Lots 3, 3-B, 10, 11, 13, and 14, to Natividad.
Romeo further testified that, although the deeds of sale executed by his parents in their
favor stated that the sale was for a consideration, they never really paid any amount for
the supposed sale. The transfer was made in this manner in order to avoid the payment
of inheritance taxes.18 Romeo denied stealing Lot 3 from his sister but instead claimed
that the title to said lot was given to him by Natividad in 1981 after their father died.
Natividad and Maximino, Jr. claimed that the Deed of Partition and Distribution
executed in 1962 was not really carried out. Instead, in December of 1969, their
parents offered to sell to them the six lots in Quezon City, i.e., Lots 3, 3-B, 10, 11, 13
and 14. However, it was only Natividad who bought the six properties because she was
the only one financially able to do so. Natividad said she sold Lots 13 and 14 to RosAlva Marketing Corp.19 and Lot 3-B to Maximino, Jr. for P175,000.00.20 Natividad
admitted that Romeo and the latters wife were occupying Lot 3-B at that time and that
she did not tell the latter about the sale she had made to Maximino, Jr.
Natividad said that she had the title to Lot 3 but it somehow got lost. She could not get
an original copy of the said title because the records of the Registrar of Deeds had
been destroyed by fire. She claimed she was surprised to learn that Romeo was able to
obtain a title to Lot 3 in his name.
Natividad insisted that she paid the amount stated in the Deed of Absolute Sale dated
January 29, 1970. She alleged that their parents had sold these properties to their
children instead of merely giving the same to them in order to impose on them the
value of hardwork.

Natividad accused Romeo of filing this case to harass her after Romeo lost in the action
for recovery of possession (Civil Case No. Q-39018) which had been brought against
him by Maximino, Jr. It appears that before the case filed by Romeo could be decided,
the Court of Appeals rendered a decision in CA-GR CV No. 12932 affirming the trial
courts decision in favor of Maximino, Jr.
On August 10, 1992, the trial court rendered a decision, the dispositive portion of which
states:
WHEREFORE, judgment is hereby rendered declaring the nullity of the Deed of Sale
dated January 29, 1970. Except as to Lots 3, 3-B, 13 and 14 which had passed on to
third persons, the defendant Natividad shall hold the rest in trust for Jose Nazareno to
whom the same had been adjudicated. The Register of Deeds of Quezon City is
directed to annotate this judgment on Transfer Certificate of Titles Nos. 162735 and
162736 as a lien in the titles of Natividad P. Nazareno.
The defendants counterclaim is dismissed. Likewise, the third-party complaint is
dismissed.
The defendants are hereby directed to pay to the plaintiff jointly and severally the sum
of P30,000 as and for attorneys fees. Likewise, the third-party plaintiff is directed to pay
the third-party defendants attorneys fees ofP20,000.
All other claims by one party against the other are dismissed.
SO ORDERED.21
Natividad and Maximino, Jr. filed a motion for reconsideration. As a result, on October
14, 1992 the trial court modified its decision as follows:
WHEREFORE, the plaintiffs Partial Motion for Reconsideration is hereby granted. The
judgment dated August 10, 1992 is hereby amended, such that the first paragraph of its
dispositive portion is correspondingly modified to read as follows:
"WHEREFORE, judgment is hereby rendered declaring the nullity of the Deeds of Sale
dated January 29, 1970 and July 31, 1982.
"Except as to Lots 3, 13 and 14 which had passed on to third person, the defendant
Natividad shall hold the rest OF THE PROPERTIES COVERED BY THE DEED OF

SALE DATED JANUARY 29, 1970 (LOTS 10 and 11) in trust for Jose Nazareno to
whom the same had been adjudicated.
"The Register of Deeds of Quezon City is directed to annotate this judgment on
Transfer Certificates of Title No. 162735 and 162736 as a lien on the titles of Natividad
P. Nazareno.
"LIKEWISE, THE SAID REGISTER OF DEEDS IS DIRECTED TO CANCEL TCT NO.
293701 (formerly 162705) OVER LOT 3-B AND RESTORE TCT NO. 140946 IN THE
NAME OF MAXIMINO NAZARENO SR. AND AUREA POBLETE."22
On appeal to the Court of Appeals, the decision of the trial court was modified in the
sense that titles to Lot 3 (in the name of Romeo Nazareno) and Lot 3-B (in the name of
Maximino Nazareno, Jr.), as well as to Lots 10 and 11 were cancelled and ordered
restored to the estate of Maximino Nazareno, Sr. The dispositive portion of the decision
dated May 29, 1998 reads:
WHEREFORE, the appeal is GRANTED. The decision and the order in question are
modified as follows:
1. The Deed of Absolute Sale dated 29 January 1970 and the Deed of Absolute
Sale dated 31 July 1982 are hereby declared null and void;
2. Except as to Lots 13 and 14 ownership of which has passed on to third
persons, it is hereby declared that Lots 3, 3-B, 10 and 11 shall form part of the
estate of the deceased Maximino Nazareno, Sr.;
3. The Register of Deeds of Quezon City is hereby ordered to restore TCT No.
140946 (covering Lot 3-B), TCT No. 132019 (covering Lot 3), TCT No. 118885
(covering Lot 10), and TCT No. 118886 (covering Lot 11).23
Petitioners filed a motion for reconsideration but it was denied in a resolution dated May
27, 1999. Hence this petition.
Petitioners raise the following issues:
1. WHETHER OR NOT THE UNCORROBORATED TESTIMONY OF PRIVATE
RESPONDENT ROMEO P. NAZARENO CAN DESTROY THE FULL FAITH
AND CREDIT ACCORDED TO NOTARIZED DOCUMENTS LIKE THE DEED

OF ABSOLUTE SALE DATED JANUARY 29, 1970 (EXH. 1) EXECUTED BY


THE DECEASED SPOUSES MAXIMINO A. NAZARENO, SR. AND AUREA
POBLETE IN FAVOR OF PETITIONER NATIVIDAD P. NAZARENO.
2. WHETHER OR NOT THE RESPONDENT COURT GROSSLY
MISAPPRECIATED THE FACTS OF THE CASE WITH RESPECT TO THE
VALIDITY OF THE SAID DEED OF ABSOLUTE SALE DATED JANUARY 29,
1970 (EXH. 1) IN THE LIGHT OF THE FOLLOWING:
A) THE DOCUMENTARY EVIDENCE, ALL OF WHICH ARE
NOTARIZED, EXECUTED BY THE DECEASED SPOUSES DURING
THEIR LIFETIME INVOLVING SOME OF THEIR CONJUGAL
PROPERTIES.
B) THE EXECUTION OF AN EXTRA-JUDICIAL PARTITION WITH
WAIVER OF RIGHTS AND CONFIRMATION OF SALE DATED MAY 24,
1975 (EXH. 14A) OF THE ESTATE OF AUREA POBLETE BY THE
DECEASED MAXIMINO A. NAZARENO, SR. AND THEIR CHILDREN
INVOLVING THE ONLY REMAINING ESTATE OF AUREA POBLETE
THUS IMPLIEDLY ADMITTING THE VALIDITY OF PREVIOUS
DISPOSITIONS MADE BY SAID DECEASED SPOUSES ON THEIR
CONJUGAL PROPERTIES, HALF OF WHICH WOULD HAVE BECOME
A PART OF AUREA POBLETES ESTATE UPON HER DEMISE.
C) THE ADMISSION MADE BY MAXIMINO A. NAZARENO, SR. IN HIS
TESTIMONY IN OPEN COURT ON AUGUST 13, 1980 DURING HIS
LIFETIME IN CIVIL CASE NO. NC-712 (EXH. 81, 81B) THAT HE HAD
SOLD CERTAIN PROPERTIES IN FAVOR OF NATIVIDAD P.
NAZARENO THUS BELYING THE CLAIM OF ROMEO P. NAZARENO
THAT THE DEED OF ABSOLUTE SALE DATED JANUARY 29, 1970 IS
ONE AMONG THE DOCUMENTS EXECUTED BY THE DECEASED
SPOUSES TO BE WITHOUT CONSIDERATION.

PROPERTIES IN QUESTION THAT THE SAID PROPERTY IS OWNED


BY PETITIONER NATIVIDAD P. NAZARENO.
E) THE PARTIAL PROJECT OF PARTITION DATED MAY 24, 1995
WHICH WAS APPROVED BY THE INTESTATE COURT IN SP. PROC.
NO. NC-28 AND EXECUTED IN ACCORDANCE WITH THE LATTER
COURTS FINAL ORDER DATED JULY 9, 1991 DETERMINING WHICH
WERE THE REMAINING PROPERTIES OF THE ESTATE.
3. WHETHER OR NOT THE DEED OF ABSOLUTE SALE DATED JANUARY
29, 1970 EXECUTED BY THE DECEASED SPOUSES MAXIMINO A.
NAZARENO, SR. AND AUREA POBLETE DURING THEIR LIFETIME
INVOLVING THEIR CONJUGAL PROPERTIES IS AN INDIVISIBLE
CONTRACT? AND IF SO WHETHER OR NOT UPON THEIR DEATH, THE
ESTATE OF MAXIMINO A. NAZARENO, SR. ALONE CAN SEEK THE
ANNULMENT OF SAID SALE?
4. WHETHER OR NOT THE SALE OF LOT 3 UNDER THE DEED OF
ABSOLUTE SALE DATED JANUARY 29, 1970 IN FAVOR OF PETITIONER
NATIVIDAD P. NAZARENO, IS VALID CONSIDERING THAT AS PER THE
ORDER OF THE LOWER COURT DATED NOVEMBER 21, 1990. ROMEO
NAZARENO ADMITTED THAT HE DID NOT PAY THE CONSIDERATION
STATED IN THE DEED OF ABSOLUTE SALE DATED JULY 4, 1969
EXECUTED BY THE DECEASED SPOUSES IN HIS FAVOR (EXH. M-2).
5. WHETHER OR NOT AS A CONSEQUENCE, THE TITLE ISSUED IN THE
NAME OF ROMEO P. NAZARENO, TCT NO. 277968 (EXH. M) SHOULD BE
CANCELLED AND DECLARED NULL AND VOID AND A NEW ONE ISSUED IN
FAVOR OF NATIVIDAD P. NAZARENO PURSUANT TO THE DEED OF
ABSOLUTE SALE EXECUTED IN THE LATTERS FAVOR ON JANUARY 29,
1970 BY THE DECEASED SPOUSES.24
We find the petition to be without merit.

D) THE ADMISSIONS MADE BY ROMEO P. NAZARENO HIMSELF


CONTAINED IN A FINAL DECISION OF THE RESPONDENT COURT
IN CA-GR CV NO. 12932 DATED AUGUST 31, 1992 AND AN ANNEX
APPEARING IN HIS ANSWER TO THE COMPLAINT IN CIVIL CASE
NO. Q-39018 (EXH. 11-B) INVOLVING LOT 3B, ONE OF THE

First. Petitioners argue that the lone testimony of Romeo is insufficient to overcome the
presumption of validity accorded to a notarized document.
To begin with, the findings of fact of the Court of Appeals are conclusive on the parties
and carry even more weight when these coincide with the factual findings of the trial

court. This Court will not weigh the evidence all over again unless there is a showing
that the findings of the lower court are totally devoid of support or are clearly erroneous
so as to constitute serious abuse of discretion.25 The lone testimony of a witness, if
credible, is sufficient. In this case, the testimony of Romeo that no consideration was
ever paid for the sale of the six lots to Natividad was found to be credible both by the
trial court and by the Court of Appeals and it has not been successfully rebutted by
petitioners. We, therefore, have no reason to overturn the findings by the two courts
giving credence to his testimony.
The fact that the deed of sale was notarized is not a guarantee of the validity of its
contents. As held in Suntay v. Court of Appeals:26
Though the notarization of the deed of sale in question vests in its favor the
presumption of regularity, it is not the intention nor the function of the notary public to
validate and make binding an instrument never, in the first place, intended to have any
binding legal effect upon the parties thereto. The intention of the parties still and always
is the primary consideration in determining the true nature of a contract.
Second. Petitioners make capital of the fact that in C.A.-G.R. CV No. 12932, which was
declared final by this Court in G.R. No. 107684, the Court of Appeals upheld the right of
Maximino, Jr. to recover possession of Lot 3-B. In that case, the Court of Appeals held:
As shown in the preceding disquisition, Natividad P. Nazareno acquired the property in
dispute by purchase in 1970. She was issued Transfer Certificate of Title No. 162738 of
the Registry of Deeds of Quezon City. When her parents died, her mother Aurea
Poblete-Nazareno in 1970 and her father Maximino A. Nazareno, Sr. in 1980, Natividad
P. Nazareno had long been the exclusive owner of the property in question. There was
no way therefore that the aforesaid property could belong to the estate of the spouses
Maximino Nazareno, Sr. and Aurea Poblete. The mere fact that Romeo P. Nazareno
included the same property in an inventory of the properties of the deceased Maximino
A. Nazareno, Sr. will not adversely affect the ownership of the said realty. Appellant
Romeo P. Nazarenos suspicion that his parents had entrusted all their assets under the
care and in the name of Natividad P. Nazareno, their eldest living sister who was still
single, to be divided upon their demise to all the compulsory heirs, has not progressed
beyond mere speculation. His barefaced allegation on the point not only is without any
corroboration but is even belied by documentary evidence. The deed of absolute sale
(Exhibit "B"), being a public document (Rule 132, Secs. 19 and 23, Revised Rules on
Evidence), is entitled to great weight; to contradict the same, there must be evidence
that is clear, convincing and more than merely preponderant (Yturralde vs. Aganon, 28

SCRA 407; Favor vs. Court of Appeals, 194 SCRA 308). Defendants-appellants own
conduct disproves their claim of co-ownership over the property in question. Being
themselves the owner of a ten-unit apartment building along Stanford St., Cubao
Quezon City, defendants-appellants, in a letter of demand to vacate addressed to their
tenants (Exhibits "P", "P-1" and "P-2") in said apartment, admitted that the house and
lot located at No. 979 Aurora Blvd., Quezon City where they were residing did not
belong to them. Also, when they applied for a permit to repair the subject property in
1977, they stated that the property belonged to and was registered in the name of
Natividad P. Nazareno. Among the documents submitted to support their application for
a building permit was a copy of TCT No. 162738 of the Registry of Deeds of Quezon
City in the name of Natividad Nazareno (Exhibit "O" and submarkings; tsn March 15,
1985, pp. 4-5).27
To be sure, that case was for recovery of possession based on ownership of Lot 3-B.
The parties in that case were Maximino, Jr., as plaintiff, and the spouses Romeo and
Eliza, as defendants. On the other hand, the parties in the present case for annulment
of sale are the estate of Maximino, Sr., as plaintiff, and Natividad and Maximino, Jr., as
defendants. Romeo and Eliza were named third-party defendants after a third-party
complaint was filed by Natividad and Maximino, Jr. As already stated, however, this
third-party complaint concerned Lot 3, and not Lot 3-B.
The estate of a deceased person is a juridical entity that has a personality of its
own.28 Though Romeo represented at one time the estate of Maximino, Sr., the latter
has a separate and distinct personality from the former. Hence, the judgment in CA-GR
CV No. 12932 regarding the ownership of Maximino, Jr. over Lot 3-B binds Romeo and
Eliza only, and not the estate of Maximino, Sr., which also has a right to recover
properties which were wrongfully disposed.
Furthermore, Natividads title was clearly not an issue in the first case. In other words,
the title to the other five lots subject of the present deed of sale was not in issue in that
case. If the first case resolved anything, it was the ownership of Maximino, Jr. over Lot
3-B alone.
Third. Petitioners allege that, as shown by several deeds of sale executed by
Maximino, Sr. and Aurea during their lifetime, the intention to dispose of their real
properties is clear. Consequently, they argue that the Deed of Sale of January 29, 1970
should also be deemed valid.

This is a non-sequitur. The fact that other properties had allegedly been sold by the
spouses Maximino, Sr. and Aurea does not necessarily show that the Deed of Sale
made on January 29, 1970 is valid.
Romeo does not dispute that their parents had executed deeds of sale. The question,
however, is whether these sales were made for a consideration. The trial court and the
Court of Appeals found that the Nazareno spouses transferred their properties to their
children by fictitious sales in order to avoid the payment of inheritance taxes.
Indeed, it was found both by the trial court and by the Court of Appeals that Natividad
had no means to pay for the six lots subject of the Deed of Sale.
All these convince the Court that Natividad had no means to pay for all the lots she
purportedly purchased from her parents. What is more, Romeos admission that he did
not pay for the transfer to him of lots 3 and 25-L despite the considerations stated in the
deed of sale is a declaration against interest and must ring with resounding truth. The
question is, why should Natividad be treated any differently, i.e., with consideration for
the sale to her, when she is admittedly the closest to her parents and the one staying
with them and managing their affairs? It just seems without reason. Anyway, the Court
is convinced that the questioned Deed of Sale dated January 29, 1970 (Exh. "A" or "1")
is simulated for lack of consideration, and therefore ineffective and void.29
In affirming this ruling, the Court of Appeals said:
Facts and circumstances indicate badges of a simulated sale which make the Deed of
Absolute Sale dated 29 January 1970 void and of no effect. In the case of Suntay vs.
Court of Appeals (251 SCRA 430 [1995]), the Supreme Court held that badges of
simulation make a deed of sale null and void since parties thereto enter into a
transaction to which they did not intend to be legally bound.
It appears that it was the practice in the Nazareno family to make simulated transfers of
ownership of real properties to their children in order to avoid the payment of
inheritance taxes. Per the testimony of Romeo, he acquired Lot 25-L from his parents
through a fictitious or simulated sale wherein no consideration was paid by him. He
even truthfully admitted that the sale of Lot 3 to him on 04 July 1969 (Deed of Absolute
Sale, Records, Vol. II, p. 453) likewise had no consideration. This document was signed
by the spouses Max, Sr. and Aurea as vendors while defendant-appellant Natividad
signed as witness.30

Fourth. Petitioners argue further:


The Deed of Absolute Sale dated January 29, 1970 is an indivisible contract founded
on an indivisible obligation. As such, it being indivisible, it can not be annulled by only
one of them. And since this suit was filed only by the estate of Maximino A. Nazareno,
Sr. without including the estate of Aurea Poblete, the present suit must fail. The estate
of Maximino A. Nazareno, Sr. can not cause its annulment while its validity is sustained
by the estate of Aurea Poblete.31
An obligation is indivisible when it cannot be validly performed in parts, whatever may
be the nature of the thing which is the object thereof. The indivisibility refers to the
prestation and not to the object thereof.32 In the present case, the Deed of Sale of
January 29, 1970 supposedly conveyed the six lots to Natividad. The obligation is
clearly indivisible because the performance of the contract cannot be done in parts,
otherwise the value of what is transferred is diminished. Petitioners are therefore
mistaken in basing the indivisibility of a contract on the number of obligors.
In any case, if petitioners only point is that the estate of Maximino, Sr. alone cannot
contest the validity of the Deed of Sale because the estate of Aurea has not yet been
settled, the argument would nonetheless be without merit. The validity of the contract
can be questioned by anyone affected by it.33 A void contract is inexistent from the
beginning. Hence, even if the estate of Maximino, Sr. alone contests the validity of the
sale, the outcome of the suit will bind the estate of Aurea as if no sale took place at all.
Fifth. As to the third-party complaint concerning Lot 3, we find that this has been
passed upon by the trial court and the Court of Appeals. As Romeo admitted, no
consideration was paid by him to his parents for the Deed of Sale. Therefore, the sale
was void for having been simulated. Natividad never acquired ownership over the
property because the Deed of Sale in her favor is also void for being without
consideration and title to Lot 3 cannot be issued in her name.
Nonetheless, it cannot be denied that Maximino, Sr. intended to give the six Quezon
City lots to Natividad. As Romeo testified, their parents executed the Deed of Sale in
favor of Natividad because the latter was the only "female and the only unmarried
member of the family."34 She was thus entrusted with the real properties in behalf of her
siblings. As she herself admitted, she intended to convey Lots 10 and 11 to Jose in the
event the latter returned from abroad. There was thus an implied trust constituted in her
favor.1wphi1 Art. 1449 of the Civil Code states:

There is also an implied trust when a donation is made to a person but it appears that
although the legal estate is transmitted to the donee, he nevertheless is either to have
no beneficial interest or only a part thereof.
There being an implied trust, the lots in question are therefore subject to collation in
accordance with Art. 1061 which states:
Every compulsory heir, who succeeds with other compulsory heirs, must bring into the
mass of the estate any property or right which he may have received from the
decedent, during the lifetime of the latter, by way of donation, or any other gratuitous
title, in order that it may be computed in the determination of the legitime of each heir,
and in the account of the partition.
As held by the trial court, the sale of Lots 13 and 14 to Ros-Alva Marketing, Corp. on
April 20, 197935 will have to be upheld for Ros-Alva Marketing is an innocent purchaser
for value which relied on the title of Natividad. The rule is settled that "every person
dealing with registered land may safely rely on the correctness of the certificate of title
issued therefor and the law will in no way oblige him to go behind the certificate to
determine the condition of the property."36
WHEREFORE, the decision of the Court of Appeals is AFFIRMED.
SO ORDERED.

G.R. No. 106401

September 29, 2000

SPOUSES FLORENTINO ZARAGOZA and ERLINDA ENRIQUEZZARAGOZA, petitioners, vs. THE HONORABLE COURT OF APPEALS, ALBERTA
ZARAGOZA MORGAN, respondents.
DECISION

Petitioners, in their Answer, admitted their affinity with private respondent and the
allegations on the properties of their father. They, however, denied knowledge of an
alleged distribution by way of deeds of sale to them by their father. They said that lot
871 is still registered in their father's name, while lot 943 was sold by him to them for a
valuable consideration. They denied knowledge of the alleged intention of their father to
convey the cited lots to Alberta, much more, the reason for his failure to do so because
she became an American citizen. They denied that there was partitioning of the estate
of their father during his lifetime.

QUISUMBING, J.:
Before the Court is a petition for review on certiorari, which seeks (1) the reversal of the
decision1 of the Court of Appeals promulgated on March 27, 1992 in CA - G.R. CV No.
12587, which affirmed the decision2 of the Regional Trial Court in Civil Case No. 14178,
except the dismissal of private respondent's claim over lot 943; (2) the dismissal of the
complaint filed by private respondent in the Regional Trial Court of Iloilo; and (3) the
declaration of the deed of sale executed by Flavio Zaragoza covering Lot 943 as valid.
The facts of the case as found by the Court of Appeals and on record are as follows:
Flavio Zaragoza Cano was the registered owner of certain parcels of land situated at
the Municipalities of Cabatuan, New Lucena and Sta. Barbara, Province of Iloilo. He
had four children: Gloria, Zacariaz, Florentino and Alberta, all surnamed Zaragoza. On
December 9, 1964, he died without a will and was survived by his four children.
On December 28, 1981, private respondent Alberta Zaragoza-Morgan filed a complaint
with the Court of First Instance of Iloilo against Spouses Florentino and Erlinda, herein
petitioners, for delivery of her inheritance share, consisting of Lots 943 and 871, and for
payment of damages. She claims that she is a natural born Filipino citizen and the
youngest child of the late Flavio. She further alleged that her father, in his lifetime,
partitioned the aforecited properties among his four children. The shares of her brothers
and sister were given to them in advance by way of deed of sale, but without valid
consideration, while her share, which consists of lots no. 871 and 943, was not
conveyed by way of deed of sale then. She averred that because of her marriage, she
became an American citizen and was prohibited to acquire lands in the Philippines
except by hereditary succession. For this reason, no formal deed of conveyance was
executed in her favor covering these lots during her father's lifetime.

On November 23, 1983, petitioners filed a Motion to Dismiss, on the ground that the
complaint did not state a cause of action and it failed to implead indispensable parties.
The resolution of said Motion was deferred by the lower court until the case was tried
on the merits.
On October 7, 1986, the Regional Trial Court of Iloilo promulgated its decision, the
decretal portion of which reads:
WHEREFORE, in view of the above findings, judgment is hereby rendered,
adjudicating Lot 871 in the name of Flavio Zaragoza Cano to plaintiff Alberta ZaragozaMorgan as appertaining her share in his estate and ordering defendants to vacate its
premises and deliver immediately the portion occupied by them to herein plaintiff.
Plaintiff's claim against defendants over Lot 943 is dismissed as well as claims for
damages interposed against each other.3
In the above decision, the RTC found that Flavio partitioned his properties during his
lifetime among his three children by deeds of sales; that the conveyance of Lot 943 to
petitioners was part of his plan to distribute his properties among his children during his
lifetime; and that he intended Lot 871 to be the share of private respondent.4
Not satisfied with the above decision, both parties interposed an appeal in the Court of
Appeals docketed as CA -GR CV No. 12587.
On March 27, 1992, respondent court rendered the assailed decision, the decretal
portion of which reads:
WHEREFORE, WE reverse the decision appealed from, insofar as defendantappellants, spouses Florentino Zaragoza and Erlinda E. Zaragoza, were adjudged

owner of Lot 943. In all other respects, the decision appealed from is hereby
AFFIRMED.5

3. NO WITNESSES WAS EVER PRESENTED TO ASSAIL THE


GENUINENESS OF THE SIGNATURE OF FLAVIO ZARAGOZA.

The appellate court gave weight to the testimonial and documentary evidence
presented by private respondent to support its finding that Lots 871 and 943 were
inheritance share of private respondent. Specifically, it noted the admission by
petitioner in his letter in 1981 to private respondent's counsel, that their father had given
them their inheritance.6 Further, public respondent found that the alleged sale of lot 943
in favor of petitioner Florentino was fictitious and void. The signature of Don Flavio in
the said document was markedly different from his other signatures appearing in other
documents he signed from January to February 1957.7 The Motion for Reconsideration
was denied in a Resolution8 dated June 26, 1992.

4. THE SAID DEED OF SALE WAS EVEN WITNESSED BY HIS OTHER


DAUGHTER GLORIA ZARAGOZA NUEZ AND NOTARIZED BY NOTARY
PUBLIC ATTY. EDURESE.

Hence, this petition for review on certiorari, with a supplemental petition, raising the
following assigned errors:
A. THE COURT OF APPEALS ERRED IN HOLDING THAT LOTS 871 AND 943 ARE
THE INHERITANCE SHARE OF THE PRIVATE RESPONDENT NOTWITHSTANDING
THE FACT THAT THE DECEDENT FLAVIO ZARAGOZA HAS NOT EXECUTED ANY
WILL NOR ANY DOCUMENT GIVING THESE TWO PROPERTIES IN FAVOR OF
PRIVATE RESPONDENT;
B. THE COURT OF APPEALS ERRED IN ADMITTING AND GIVING WEIGHT TO THE
TESTIMONIES OF PRIVATE RESPONDENT'S WITNESSES TO THE EFFECT THAT
LOTS 871 AND 943 ARE THE INHERITANCE SHARE OF PRIVATE RESPONDENT
AS TOLD TO THEM BY FLAVIO ZARAGOZA DESPITE THE FACT THAT THESE
TESTIMONIES ARE HEARSAY;
C. THE COURT OF APPEALS ERRED IN CONCLUDING THAT THE DEED OF SALE
EXECUTED BY FLAVIO ZARAGOZA IN FAVOR OF PETITIONER OVER LOT 943 IS A
FORGERY, NOTWITHSTANDING THE FACT THAT:
1. THE THEORY THAT THE AFORESAID DEED OF SALE WAS A FORGERY
WAS NEVER RAISED IN THE COMPLAINT NOR ESTABLISHED BY
EVIDENCE.

D. RESPONDENT COURT OF APPEALS ERRED IN MODIFYING THE DECISION OF


THE LOWER COURT WITH RESPECT TO LOT 943 WHEN THE LATTER
SUSTAINED THE GENUINENESS OF THE SIGNATURE OF PETITIONER'S FATHER
FOUND IN EXH. I.
E. THE COURT OF APPEALS ERRED IN APPLYING THE DOCTRINE OF
ESTOPPEL, IGNORING THE FACT THAT IT IS THE LAW ON INTESTATE
SUCCESSION, AND THE CORRESPONDING RULES OF COURT ON THE
SETTLEMENT OF THE ESTATE THAT IS APPLICABLE ON THIS CASE.10
In their Supplemental Petition for Review dated October 29, 1992, petitioners
additionally raised:
I. THE HONORABLE COURT OF APPEALS ERRED IN NOT DISMISSING THE
COMPLAINT FILED BEFORE THE TRIAL COURT FOR FAILURE TO STATE A
CAUSE OF ACTION,
II. ALTERNATIVELY, THE COURT ERRED IN NOT CONSIDERING THAT
LOTS TRANSFERRED INTER VIVOS TO THE OTHER HEIRS SHOULD HAVE
BEEN COLLATED TO THE MASS OF THE ESTATE OF THE DECEASED
FLAVIO ZARAGOZA (y) CANO.
III. THE HONORABLE COURT OF APPEALS ERRED IN AFFIRMING AS
LAWFUL AND VALID ALL THE DISPOSITIONS MADE BY THE DECEASED
FLAVIO ZARAGOZA (y) CANO EXCEPT LOT # 943 DESPITE UTTER LACK
OF EVIDENCE TO SUPPORT ITS FINDING THAT THE SIGNATURE OF THE
LATE FLAVIO ZARAGOZA (y) CANO IN EXH. "M-11-A" APPEARING IN THE
DEED OF SALE DATED FEBRUARY 5, 1957 (EXH. "1", FLORENTINO) WAS A
FORGERY.

2. THE SAID DEED OF SALE WAS IN A PUBLIC INSTRUMENT.


IV. THE COURT ERRED IN NOT CONSIDERING THAT TRANSFER
CERTIFICATE OF TITLE NO. T-35946 (EXHIBIT 2) COVERING LOT 943 IN

FAVOR OF THE PETITIONER CONCLUSIVELY EVIDENCES THE LATTER'S


OWNERSHIP THEREOF.11
Essentially, we are asked to resolve two issues: (1) whether the partition inter vivos by
Flavio Zaragoza Cano of his properties, which include Lots 871 and 943, is valid; and
(2) whether the validity of the Deed of Sale and consequently, the Transfer Certificate of
Title over Lot 943 registered in the name of the petitioners, can be a valid subject
matter of the entire proceeding for the delivery of inheritance share.
On the first issue. It is the main contention of the petitioner that the adjudication of Lots
943 and 871 in favor of private respondent, as her inheritance share, has no legal basis
since there is no will nor any document that will support the transfer.
Both the trial court and the public respondent found that during the lifetime of Flavio, he
already partitioned and distributed his properties among his three children, excepting
private respondent, through deeds of sale. A deed of sale was not executed in favor of
private respondent because she had become an American citizen and the Constitution
prohibited a sale in her favor. Petitioner admitted Lots 871 and 943 were inheritance
shares of the private respondent. These are factual determinations of the Court of
Appeals, based on documentary and testimonial evidence. As a rule, we are bound by
findings of facts of the Court of Appeals.12 Was the partition done during the lifetime of
Flavio Zaragoza Cano valid? We think so. It is basic in the law of succession that a
partition inter vivos may be done for as long as legitimes are not prejudiced. Art. 1080
of the Civil Code is clear on this.13 The legitime of compulsory heirs is determined after
collation, as provided for in Article 1061:
Every compulsory heir, who succeeds with other compulsory heirs, must bring into the
mass of the estate any property or right which he may have received from the
decedent, during the lifetime of the latter, by way of donation, or any other gratuitous
title in order that it may be computed in the determination of the legitime of each heir,
and in the account of the partition.
Unfortunately, collation can not be done in this case where the original petition for
delivery of inheritance share only impleaded one of the other compulsory heirs. The
petition must therefore be dismissed without prejudice to the institution of a new
proceeding where all the indispensable parties are present for the rightful determination
of their respective legitime and if the legitimes were prejudiced by the partitioning inter
vivos.

We now come to the second issue. Private respondent, in submitting her petition for the
delivery of inheritance share, was in effect questioning the validity of the deed of sale
covering Lot 943 in favor of petitioner and consequently, the Transfer Certificate of Title
issued in the latter's name. Although the trial court, as an obiter, made a finding of
validity of the conveyance of Lot 943 in favor of petitioners, since according to it, private
respondent did not question the genuineness of the signature of the deceased,
nevertheless, when the case was elevated to the Court of Appeals, the latter declared
the sale to be fictitious because of finding of marked differences in the signature of
Flavio in the Deed of Sale vis--vis signatures found in earlier documents. Could this be
done? The petition is a collateral attack. It is not allowed by Sec. 48 of the Presidential
Decree No. 1529, otherwise known as the Property Registration Decree, which
provides:
Sec. 48. Certificate not subject to collateral attack. - A certificate of title shall not be
subject to collateral attack. It can not be altered, modified, or cancelled except in a
direct proceeding in accordance with law.
We have reiterated this rule in the case of Halili vs. Court of Industrial Relations,14 citing
the earlier cases ofConstantino vs. Espiritu15 and Co vs. Court of Appeals.16 In Halili, we
held that a certificate of title accumulates in one document a precise and correct
statement of the exact status of the fee held by its owner. The certificate, in the
absence of fraud, is the evidence of title and shows exactly the real interest of its
owner. The title once registered, with very few exceptions, should not thereafter be
impugned, altered, changed, modified, enlarged or diminished, except in some direct
proceeding permitted by law. Otherwise, all security in registered titles would be lost.
In Constantino, the Court decided that the certificate, in the absence of fraud, is the
evidence of title and shows exactly the real interest of its owner. The title once
registered, with very few exceptions, should not thereafter be impugned, altered,
changed, modified, enlarged or diminished, except in some direct proceeding permitted
by law. Otherwise all security in registered titles would be lost. And in Co, we stated that
a Torrens title cannot be collaterally attacked. The issue on the validity of title, i.e.,
whether or not it was fraudulently issued, can only be raised in an action expressly
instituted for that purpose.
ACCORDINGLY, judgment is hereby rendered GRANTING the instant petition for
review. The decision of the Court of Appeals dated March 27, 1992 in CA-G.R. CV No.
12587, entitled Alberta Zaragoza-Morgan vs. Spouses Florentino Zaragoza and Erlinda
Enriquez-Zaragoza is VACATED and SET ASIDE. The complaint for delivery of
inheritance share in the Regional Trial Court, for failure to implead indispensable

parties, is also DISMISSEDwithout prejudice to the institution of the proper


proceedings.
No pronouncement as to costs.
SO ORDERED.

G.R. No. 143989

July 14, 2003

ISABELITA S. LAHOM, petitioner, vs. JOSE MELVIN SIBULO (previously referred to


as "DR. MELVIN S. LAHOM"), respondent.
VITUG, J.:

xxx

xxx

xxx

"13. That herein petitioner being a widow, and living alone in this city with only
her household helps to attend to her, has yearned for the care and show of
concern from a son, but respondent remained indifferent and would only come
to Naga to see her once a year.

The bliss of marriage and family would be to most less than complete without children.
The realization could have likely prodded the spouses Dr. Diosdado Lahom and
Isabelita Lahom to take into their care Isabelita's nephew Jose Melvin Sibulo and to
bring him up as their own. At the tender age of two, Jose Melvin enjoyed the warmth,
love and support of the couple who treated the child like their own. Indeed, for years,
Dr. and Mrs. Lahom fancied on legally adopting Jose Melvin. Finally, in 1971, the
couple decided to file a petition for adoption. On 05 May 1972, an order granting the
petition was issued that made all the more intense than before the feeling of affection of
the spouses for Melvin. In keeping with the court order, the Civil Registrar of Naga City
changed the name "Jose Melvin Sibulo" to "Jose Melvin Lahom."

"14. That for the last three or four years, the medical check-up of petitioner in
Manila became more frequent in view of a leg ailment, and those were the times
when petitioner would need most the care and support from a love one, but
respondent all the more remained callous and utterly indifferent towards
petitioner which is not expected of a son.

A sad turn of events came many years later. Eventually, in December of 1999, Mrs.
Lahom commenced a petition to rescind the decree of adoption before the Regional
Trial Court (RTC), Branch 22, of Naga City. In her petition, she averred

"16. That in view of respondent's insensible attitude resulting in a strained and


uncomfortable relationship between him and petitioner, the latter has suffered
wounded feelings, knowing that after all respondent's only motive to his
adoption is his expectancy of his alleged rights over the properties of herein
petitioner and her late husband, clearly shown by his recent filing of Civil Case
No. 99-4463 for partition against petitioner, thereby totally eroding her love and
affection towards respondent, rendering the decree of adoption, considering
respondent to be the child of petitioner, for all legal purposes, has been negated
for which reason there is no more basis for its existence, hence this petition for
revocation,"1

"7. That x x x despite the proddings and pleadings of said spouses, respondent
refused to change his surname from Sibulo to Lahom, to the frustrations of
petitioner particularly her husband until the latter died, and even before his
death he had made known his desire to revoke respondent's adoption, but was
prevented by petitioner's supplication, however with his further request upon
petitioner to give to charity whatever properties or interest may pertain to
respondent in the future.
xxx

xxx

xxx

"10. That respondent continued using his surname Sibulo to the utter disregard
of the feelings of herein petitioner, and his records with the Professional
Regulation Commission showed his name as Jose Melvin M. Sibulo originally
issued in 1978 until the present, and in all his dealings and activities in
connection with his practice of his profession, he is Jose Melvin M. Sibulo.

"15. That herein respondent has recently been jealous of petitioner's nephews
and nieces whenever they would find time to visit her, respondent alleging that
they were only motivated by their desire for some material benefits from
petitioner.

Prior to the institution of the case, specifically on 22 March 1998, Republic Act (R.A.)
No. 8552, also known as the Domestic Adoption Act, went into effect. The new statute
deleted from the law the right of adopters to rescind a decree of adoption.
Section 19 of Article VI of R.A. No. 8552 now reads:
"SEC. 19. Grounds for Rescission of Adoption. Upon petition of the adoptee,
with the assistance of the Department if a minor or if over eighteen (18) years of
age but is incapacitated, as guardian/counsel, the adoption may be rescinded

on any of the following grounds committed by the adopter(s): (a) repeated


physical and verbal maltreatment by the adopter(s) despite having undergone
counseling; (b) attempt on the life of the adoptee; (c) sexual assault or violence;
or (d) abandonment and failure to comply with parental obligations.
"Adoption, being in the best interest of the child, shall not be subject to
rescission by the adopter(s). However, the adopter(s) may disinherit the
adoptee for causes provided in Article 919 of the Civil Code." (emphasis
supplied)
Jose Melvin moved for the dismissal of the petition, contending principally (a) that the
trial court had no jurisdiction over the case and (b) that the petitioner had no cause of
action in view of the aforequoted provisions of R.A. No. 8552. Petitioner asseverated,
by way of opposition, that the proscription in R.A. No. 8552 should not retroactively
apply, i.e., to cases where the ground for rescission of the adoption vested under the
regime of then Article 3482of the Civil Code and Article 1923 of the Family Code.
In an order, dated 28 April 2000, the trial court held thusly:
"On the issue of jurisdiction over the subject matter of the suit, Section 5(c) of
R.A. No. 8369 confers jurisdiction to this Court, having been designated Family
Court in A.M. No. 99-11-07 SC.
"On the matter of no cause of action, the test on the sufficiency of the facts
alleged in the complaint, is whether or not, admitting the facts alleged, the Court
could render a valid judgment in accordance with the prayer of said complaint
(De Jesus, et al. vs. Belarmino, et al., 95 Phil. 365).
"Admittedly, Section 19, Article VI of R.A. No. 8552 deleted the right of an
adopter to rescind an adoption earlier granted under the Family Code.
Conformably, on the face of the petition, indeed there is lack of cause of action.
"Petitioner however, insists that her right to rescind long acquired under the
provisions of the Family Code should be respected. Assuming for the sake of
argument, that petitioner is entitled to rescind the adoption of respondent
granted on May 5, 1972, said right should have been exercised within the period
allowed by the Rules. From the averments in the petition, it appears clear that
the legal grounds for the petition have been discovered and known to petitioner
for more than five (5) years, prior to the filing of the instant petition on December

1, 1999, hence, the action if any, had already prescribed. (Sec. 5, Rule 100
Revised Rules of Court)
"WHEREFORE, in view of the foregoing consideration, the petition is ordered
dismissed."4
Via a petition for review on certiorari under Rule 45 of the 1997 Rules of Court,
petitioner raises the following questions; viz:
1. May the subject adoption, decreed on 05 May 1972, still be revoked or
rescinded by an adopter after the effectivity of R.A. No. 8552?
2. In the affirmative, has the adopter's action prescribed?
A brief background on the law and its origins could provide some insights on the
subject. In ancient times, the Romans undertook adoption to assure male heirs in the
family.5 The continuity of the adopter's family was the primary purpose of adoption and
all matters relating to it basically focused on the rights of the adopter. There was hardly
any mention about the rights of the adopted.6 Countries, like Greece, France, Spain
and England, in an effort to preserve inheritance within the family, neither allowed nor
recognized adoption.7 It was only much later when adoption was given an impetus in
law and still later when the welfare of the child became a paramount concern.8 Spain
itself which previously disfavored adoption ultimately relented and accepted the Roman
law concept of adoption which, subsequently, was to find its way to the archipelago.
The Americans came and introduced their own ideas on adoption which, unlike most
countries in Europe, made the interests of the child an overriding consideration.9 In the
early part of the century just passed, the rights of children invited universal attention;
the Geneva Declaration of Rights of the Child of 1924 and the Universal Declaration of
Human Rights of 1948,10 followed by the United Nations Declarations of the Rights of
the Child,11 were written instruments that would also protect and safeguard the rights of
adopted children. The Civil Code of the Philippines12 of 1950 on adoption, later modified
by the Child and Youth Welfare Code13 and then by the Family Code of the
Philippines,14gave immediate statutory acknowledgment to the rights of the adopted. In
1989, the United Nations initiated the Convention of the Rights of the Child. The
Philippines, a State Party to the Convention, accepted the principle that adoption was
impressed with social and moral responsibility, and that its underlying intent was geared
to favor the adopted child. R.A. No. 8552 secured these rights and privileges for the
adopted. Most importantly, it affirmed the legitimate status of the adopted child, not only
in his new family but also in the society as well. The new law withdrew the right of an

adopter to rescind the adoption decree and gave to the adopted child the sole right to
sever the legal ties created by adoption.

It was months after the effectivity of R.A. No. 8552 that herein petitioner filed an action
to revoke the decree of adoption granted in 1975. By then, the new law,22 had already
abrogated and repealed the right of an adopter under the Civil Code and the Family
Petitioner, however, would insist that R.A. No. 8552 should not adversely affect her right Code to rescind a decree of adoption. Consistently with its earlier pronouncements, the
Court should now hold that the action for rescission of the adoption decree, having
to annul the adoption decree, nor deprive the trial court of its jurisdiction to hear the
been initiated by petitioner after R.A. No. 8552 had come into force, no longer could be
case, both being vested under the Civil Code and the Family Code, the laws then in
pursued.
force.
The concept of "vested right" is a consequence of the constitutional guaranty of due
process15 that expresses apresent fixed interest which in right reason and natural
justice is protected against arbitrary state action;16 it includes not only legal or equitable
title to the enforcement of a demand but also exemptions from new obligations created
after the right has become vested.17 Rights are considered vested when the right to
enjoyment is a present interest,18 absolute, unconditional, and perfect19 or fixed and
irrefutable.
In Republic vs. Court of Appeals,20 a petition to adopt Jason Condat was filed by
Zenaida C. Bobiles on 02 February 1988 when the Child and Youth Welfare Code
(Presidential Decree No. 603) allowed an adoption to be sought by either spouse
or both of them. After the trial court had rendered its decision and while the case was
still pending on appeal, the Family Code of the Philippines (Executive Order No.
209), mandating joint adoption by the husband and wife, took effect. Petitioner Republic
argued that the case should be dismissed for having been filed by Mrs. Bobiles alone
and without being joined by the husband. The Court concluded that the jurisdiction of
the court is determined by the statute in force at the time of the commencement of
the action. The petition to adopt Jason, having been filed with the court at the time
when P.D. No. 603 was still in effect, the right of Mrs. Bobiles to file the petition, without
being joined by her husband, according to the Court had become vested. InRepublic
vs. Miller,21 spouses Claude and Jumrus Miller, both aliens, sought to adopt Michael
Madayag. On 29 July 1988, the couple filed a petition to formalize Michael's adoption
having theretofore been taken into their care. At the time the action was commenced,
P.D. No. 603 allowed aliens to adopt. After the decree of adoption and while on appeal
before the Court of Appeals, the Family Code was enacted into law on 08 August 1988
disqualifying aliens from adopting Filipino children. The Republic then prayed for the
withdrawal of the adoption decree. In discarding the argument posed by the Republic,
the Supreme Court ruled that the controversy should be resolved in the light of the law
governing at the time the petition was filed.

Interestingly, even before the passage of the statute, an action to set aside the adoption
is subject to the five-year bar rule under Rule 10023 of the Rules of Court and that the
adopter would lose the right to revoke the adoption decree after the lapse of that
period. The exercise of the right within a prescriptive period is a condition that could not
fulfill the requirements of a vested right entitled to protection. It must also be
acknowledged that a person has no vested right in statutory privileges.24 While adoption
has often been referred to in the context of a "right," the privilege to adopt is itself not
naturally innate or fundamental but rather a right merely created by statute.25 It is a
privilege that is governed by the state's determination on what it may deem to be for the
best interest and welfare of the child.26 Matters relating to adoption, including the
withdrawal of the right of an adopter to nullify the adoption decree, are subject to
regulation by the State.27 Concomitantly, a right of action given by statute may be taken
away at anytime before it has been exercised.28
While R.A. No. 8552 has unqualifiedly withdrawn from an adopter a consequential right
to rescind the adoption decree even in cases where the adoption might clearly turn out
to be undesirable, it remains, nevertheless, the bounden duty of the Court to apply the
law. Dura lex sed lex would be the hackneyed truism that those caught in the law have
to live with. It is still noteworthy, however, that an adopter, while barred from severing
the legal ties of adoption, can always for valid reasons cause the forfeiture of certain
benefits otherwise accruing to an undeserving child. For instance, upon the grounds
recognized by law, an adopter may deny to an adopted child his legitime and, by a will
and testament, may freely exclude him from having a share in the disposable portion of
his estate.
WHEREFORE, the assailed judgment of the court a quo is AFFIRMED. No costs.
SO ORDERED.

G.R. No. 82233 March 22, 1990


JOSE BARITUA and EDGAR BITANCOR, petitioners, vs. HONORABLE COURT OF
APPEALS, NICOLAS NACARIO and VICTORIA RONDA NACARIO, respondents.
Domingo Lucenario for petitioners.
Ernesto A. Atienza for private respondents.
SARMIENTO, J.:
This petition for review on certiorari assails as erroneous and contrary to existing
relevant laws and applicable jurisprudence the decision 1 of the Court of Appeals dated
December 11, 1987 which reversed and set aside that of the Regional Trial Court,
Branch XXXII, at Pili, Camarines Sur. 2 The challenged decision adjudged the
petitioners liable to the private respondents in the total amount of P20,505.00 and for
costs.
The facts are as follows:
In the evening of November 7, 1979, the tricycle then being driven by Bienvenido
Nacario along the national highway at Barangay San Cayetano, in Baao, Camarines
Sur, figured in an accident with JB Bus No. 80 driven by petitioner Edgar Bitancor and
owned and operated by petitioner Jose Baritua. 3 As a result of that accident
Bienvenido and his passenger died 4 and the tricycle was damaged. 5 No criminal case
arising from the incident was ever instituted. 6
Subsequently, on March 27, 1980, as a consequence of the extra-judicial settlement of
the matter negotiated by the petitioners and the bus insurer Philippine First
Insurance Company, Incorporated (PFICI for brevity) Bienvenido Nacario's widow,
Alicia Baracena Vda. de Nacario, received P18,500.00. In consideration of the amount
she received, Alicia executed on March 27, 1980 a "Release of Claim" in favor of the
petitioners and PFICI, releasing and forever discharging them from all actions, claims,
and demands arising from the accident which resulted in her husband's death and the
damage to the tricycle which the deceased was then driving. Alicia likewise executed
an affidavit of desistance in which she formally manifested her lack of interest in
instituting any case, either civil or criminal, against the petitioners. 7

On September 2, 1981, or about one year and ten months from the date of the accident
on November 7, 1979, the private respondents, who are the parents of Bienvenido
Nacario, filed a complaint for damages against the petitioners with the then Court of
First Instance of Camarines Sur. 8 In their complaint, the private respondents alleged
that during the vigil for their deceased son, the petitioners through their representatives
promised them (the private respondents) that as extra-judicial settlement, they shall be
indemnified for the death of their son, for the funeral expenses incurred by reason
thereof, and for the damage for the tricycle the purchase price of which they (the private
respondents) only loaned to the victim. The petitioners, however, reneged on their
promise and instead negotiated and settled their obligations with the long-estranged
wife of their late son. The Nacario spouses prayed that the defendants, petitioners
herein, be ordered to indemnify them in the amount of P25,000.00 for the death of their
son Bienvenido, P10,000.00 for the damaged tricycle, P25,000.00 for compensatory
and exemplary damages, P5,000.00 for attorney's fees, and for moral damages. 9
After trial, the court a quo dismissed the complaint, holding that the payment by the
defendants (herein petitioners) to the widow and her child, who are the preferred heirs
and successors-in-interest of the deceased Bienvenido to the exclusion of his parents,
the plaintiffs (herein private respondents), extinguished any claim against the
defendants (petitioners). 10
The parents appealed to the Court of Appeals which reversed the judgment of the trial
court. The appellate court ruled that the release executed by Alicia Baracena Vda. de
Nacario did not discharge the liability of the petitioners because the case was instituted
by the private respondents in their own capacity and not as "heirs, representatives,
successors, and assigns" of Alicia; and Alicia could not have validly waived the
damages being prayed for (by the private respondents) since she was not the one who
suffered these damages arising from the death of their son. Furthermore, the appellate
court said that the petitioners "failed to rebut the testimony of the appellants (private
respondents) that they were the ones who bought the tricycle that was damaged in the
incident. Appellants had the burden of proof of such fact, and they did establish such
fact in their testimony . . . 11Anent the funeral expenses, "(T)he expenses for the funeral
were likewise shouldered by the appellants (the private respondents). This was never
contradicted by the appellees (petitioners). . . . Payment (for these) were made by the
appellants, therefore, the reimbursement must accrue in their favor. 12
Consequently, the respondent appellate court ordered the petitioners to pay the private
respondents P10,000.00 for the damage of the tricycle, P5,000.00 for "complete"

funeral services, P450.00 for cemetery lot, P55.00 fororacion adulto, and P5,000.00 for
attorney's fees. 13 The petitioners moved for
a reconsideration of the appellate court's decision 14 but their motion was
denied. 15 Hence, this petition.
The issue here is whether or not the respondent appellate court erred in holding that
the petitioners are still liable to pay the private respondents the aggregate amount of
P20,505.00 despite the agreement of extrajudicial settlement between the petitioners
and the victim's compulsory heirs.
The petition is meritorious.
Obligations are extinguished by various modes among them being by payment. Article
1231 of the Civil Code of the Philippines provides:

Art 1240. Payment shall be made to the person in whose favor the
obligation has been constituted, or his successor in interest, or any
person authorized to receive it.
Certainly there can be no question that Alicia and her son with the deceased are the
successors in interest referred to in law as the persons authorized to receive payment.
The Civil Code states:
Article 887. The following are compulsory heirs:
1. Legitimate children and descendants, with respect to their legitimate
parents and ascendants;
2. In default of the foregoing, legitimate parents and ascendants with
respect to their legitimate children and decendants;

Art. 1231. Obligations are extinguished:


3. The widow or widower;
(1) By payment or performance;
4. Acknowledged natural children and natural children by legal fiction;
(2) By the loss of the thing due;
5. Other illegitimate children referred to in Article 287.
(3) By the condonation or remission of the debt;
(4) By the confusion or merger of the rights of creditor and debtor;

Compulsory heirs mentioned in Nos. 3, 4 and 5 are not excluded by


those in Nos. 1 and 2. Neither do they exclude one another. (Emphasis
ours.)

(5) By compensation;
(6) By novation.

Article 985. In default of legitimate children and descendants of the


deceased, his parents and ascendants shall inherit from him, to the
exclusion of collateral relatives.

(Emphasis ours.)
(Emphasis ours.)
There is no denying that the petitioners had paid their obligation petition arising from
the accident that occurred on November 7, 1979. The only question now is whether or
not Alicia, the spouse and the one who received the petitioners' payment, is entitled to
it.
Article 1240 of the Civil Code of the Philippines enumerates the persons to whom
payment to extinguish an obligation should be made.

It is patently clear that the parents of the deceased succeed only when the latter dies
without a legitimate descendant. On the other hand, the surviving spouse concurs with
all classes of heirs. As it has been established that Bienvenido was married to Alicia
and that they begot a child, the private respondents are not successors-in-interest of
Bienvenido; they are not compulsory heirs. The petitioners therefore acted correctly in
settling their obligation with Alicia as the widow of Bienvenido and as the natural
guardian of their lone child. This is so even if Alicia had been estranged from

Bienvenido. Mere estrangement is not a legal ground for the disqualification of a


surviving spouse as an heir of the deceased spouse.
Neither could the private respondents, as alleged creditors of Bienvenido, seek relief
and compensation from the petitioners. While it may be true that the private
respondents loaned to Bienvenido the purchase price of the damaged tricycle and
shouldered the expenses for his funeral, the said purchase price and expenses are but
money claims against the estate of their deceased son. 16 These money claims are not
the liabilities of the petitioners who, as we have said, had been released by the
agreement of the extra-judicial settlement they concluded with Alicia Baracena Vda. de
Nacario, the victim's widow and heir, as well as the natural guardian of their child, her
co-heir. As a matter of fact, she executed a "Release Of Claim" in favor of the
petitioners.
WHEREFORE, the petition is GRANTED; the decision of the Court of Appeals is
REVERSED and SET ASIDE and the decision of the Regional Trial Court is hereby
REINSTATED. Costs against the private respondents.
SO ORDERED.

G.R. No. 7890

September 29, 1914

FILOMENA PECSON, as administratix of the last will and testament of Florencio


Pecson, et al., plaintiffs-appellants, vs. ROSARIO MEDIAVILLO, defendant-appellee.
S. E. Imperial for appellants.
Tomas Lorayes for appellee.
JOHNSON, J.:
It appears from the record that some time prior to the 17th day of September, 1910, the
last will and testament of Florencio Pecson was presented to the Court of First Instance
of the Province of Albay for probate. Mr. Tomas Lorayes, an attorney at law, opposed
the legislation of the will on the ground that it had not been authorized nor signed by the
deceased, in accordance with the provisions of the Code of Civil Procedure. After
hearing the respective parties, the Honorable Percy M. Moir, judge, found that the will
had been signed and executed in accordance with the provisions of law, and denied the
opposition on the 17th day of September, 1910.
On the 18th day of September, 1910, the said Tomas Lorayes, representing Basiliso
Mediavillo and Rosario Mediavillo, presented a motion in the words following:

make the testator's died without succession, but is represented now by his
father, Basiliso Mediavillo), participants in the estate left by their grandfather;
and, finally, that the court grant such other relief as it may deem just and
equitable.
After a consideration of the question presented by said motion, the lower court, on the
22d day of September, 1911, rendered the following decision:
This case has come up to-day for a hearing on the declaration of heirs of the
decease Florencio Pecson, who died in Daraga, about the year 1910.
From the evidence it appears that the deceased had eight children by his wife
Nicolasa Manjares, likewise deceased, which children are those named
Emerenciano, Teresa, Filomena, Asunsion, Rufino, Zoila, Emiliano, and
Perfecto, all surnamed Pecson. It also appears that Rufino Pecson absented
himself from these Islands twenty-five years ago, going to Australia, and that
nothing has been heard of him for the past twenty years. The said Rufino
Pecson left no children in the Philippines and was unmarried when he
emigrated. As nothing has been heard of him for twenty years, it is presumed
that he died and it is held that the part of this estate to which he was entitled
must be divided among the other heirs.

3. That the interested party did not commit such an act, and if perhaps she did,
it was due to the derangement of her mental faculties which occurred a long
time ago and from which she now suffers in periodical attacks.

It also appears from the evidence that Teresa Pecson married Basiliso
Mediavillo, by whom she had two children, Joaquin and Rosario Mediavillo.
Teresa also died, leaving these two children and her husband, Basiliso
Mediavillo. Her son Joaquin died, unmarried and childless, before the death of
the testator, Florencio Pecson. Rosario is the only living daughter of Teresa and
the latter's husband, Basiliso Mediavillo, is also living. The evidence shows that
this girl Rosario became insane in 1895, when she went to Nueva Caceres to
study in college, and it has been proved that it was previous to this date that she
disobeyed her grandfather and raised her hand against him, and, as the testator
states in the third paragraph of his will, he disinherited her. This court
understands that this Rosario, who was then 14 years of age, and who shortly
afterwards became insane, was not responsible for her acts and should not
have been disinherited by her grandfather.

By reason of all the foregoing and because the disinheriting clause 3 of the will
is unfounded, the undersigned prays the court to annul the said clause and to

The court therefore decrees that this part of the will is contrary to law and sets it
aside as being of no force or value whatever. The court further holds that

1. That Rosario Mediavillo is and Joaquin Mediavillo was a legitimate child of


the deceased Teresa Pecson, who also was a daughter of the testator, Florencio
Pecson, and therefore the first mentioned is and the second was a grandchild of
the latter.
2. That the said granddaughter, Rosario Mediavillo y Pecson, was disinherited
by her grandfather, the testator Florencio Pecson, according to clause 3 of the
will, because she failed to show him due respect and on a certain occasion
raised her hand against him.

Rosario Mediavillo, the daughter of Teresa Pecson, is the heiress of the one-half
of the share of this estate pertaining to the said Teresa, and that her father, as
the heir of his son Joaquin, also Teresa's son, is the heris of the other one-half
of the said share pertaining to Teresa that is, of the one-seventh of this estate
that pertains to the latter. Moreover, the court decrees that, besides the two
heirs just above mentioned, Emerciano, Filomena, Asuncion, Zoila, Emiliano,
and Perfecto, surnamed Pecson, and the children of Teresa, are also heirs of
the estate of Florencio Pecson.
From the decision the plaintiff appealed to this court and made the following
assignments of error:
FIRST ERROR
The lower court erred in finding that the part of the will which disinherits Rosario
Mediavillo is contrary to law, and in setting it aside as being of no force or value
whatever.
SECOND ERROR
The lower court erred by decreeing that Basaliso Mediavillo, the father of
Joaquin Mediavillo, is the heir by representation of the one-half of the one
seventh of this estate pertaining to Joaquin Mediavillo.
With reference to the first assignment of error it may be said that from the record it
appears that during the lifetime of Florencio Pecson he had been married to Nicolasa
Manjares, with whom he had eight children, named Filomena, Asuncion, Zoila,
Emerenciano, Emiliano, Perfecto, Rufino and Teresa Pecson; that before the death of
Florencio Pecson he executed and delivered the will in question. The will made no
provision for the said Rufino Pecson, neither was there any provision in the will for the
said Teresa. All of the other children were named as heirs in said will. It appears that
Teresa had been married with one Basiliso Mediavillo, and that some time before the
making of the will in question she died, leaving her husband and two children, Joaquin
Mediavillo and Rosario Mediavillo, as her heirs. It also appears from the record that
Joaquin Mediavillo died without heirs, leaving as the only heirs of the said Teresa
Pecson, her husband, Basilio Mediavillo and the said Rosario Mediavillo. The said
Joaquin Mediavillo died before his grandfather, Florencio Pecson, and probably before
the will in question was made.

Paragraph 3 of the will disinherited Rosario Mediavillo in the following language:


I declare that one of my daughters, named Teresa, now deceased, left a
legitimate daughter named Rosario Mediavillo. I also declare that I disinherit my
granddaughter, the said Rosario Mediavillo, because she was grossly
disrespectful to me and because on one occasion, when it was I do not
remember, she raised her hand against me. Therefore, it is my will that the said
Rosario Mediavillo shall have no share in my property.
The defendant, Rosario Mediavillo, in the motion which she presented and which is
copied above, alleges that she was disinherited without case. Upon a consideration of
that question, the lower court found that she had been disinherited without cause and
annulled said paragraph 3 of the will. That order of the lower court constitutes the error
complained of by the appellant in her first assignment of error.
By reference to said paragraph 3 above quoted, it will be seen that Florencio Pecson
disinherited the said Rosario Mediavillo "because she was grossly disrespectful to me
and because on one occasion, when it was I do not remember, she raised her hand
against me. Therefore it is my will that she, the said Rosario Mediavillo, shall have no
share in my property."
The lower court admitted proof the question of the responsibility of the said Rosario
Mediavillo at the time she offered the offense to her grandfather, Florencio Pecson.
After hearing the proof, the lower court reached the following conclusion:
The evidence shows that this girl Rosario became insane in 1895, when she
went to Nueva Caceres to study in college, and it has been proved that it was
previous to this date that she disobeyed her grandfather and raised her hand
against him, and, as the testator states in the third paragraph of his will, he
disinherited her. This court understands that this Rosario, who was then 14
years of age, and who shortly afterwards became insane, was not responsible
for her acts and should not have been disinherited by her grandfather.
The first assignment of error presents the question whether or not the courts, when a
parent disinherits his children, may inquire into the cause of the disinheritance and
decide that there was or was not ground for such disinheritance. The Civil Code (art.
848) provides that disinheritance shall only take place for one of the causes expressly
fixed by law. In accordance with the provisions of that article (848) we find that articles
756 and 853 provide the cases or causes for disinheritance; or, in other words, the

cases or causes in which the ancestors may by will disinherit their heirs. Article 849 of
the Civil Code provides that the disinheritance can only be effected by the testament, in
which shall be mentioned the legal grounds or causes for such disinheritance. If it is
true that heirs can be disinherited only by will, and for causes mentioned in the Civil
Code, it would seen to follow that the courts might properly inquire whether the
disinheritance has been made properly and for the causes provided for by law. The
right of the courts to inquire into the causes and whether there was sufficient cause for
the disinheritance or not, seems to be supported by express provisions of the Civil
Code. Article 850 provides that "the proof of the truthfulness of the reason for
disinheritance shall be established by the heirs of the testator, should the disinherited
person deny it." It would appear then that if the person disinherited should deny the
truthfulness of the cause of disinheritance, he might be permitted to support his
allegation by proof. The right of the court to inquire whether or not the disinheritance
was made for just cause is also sustained by the provisions of article 851, which in part
provides that:
Disinheritance made without statement of the reason, or for a cause the truth of
which, if contradicted, should not be proven . . . shall annul the designation of
heirship, in so far as it prejudices the person disinherited.
It seems clear from the above-quoted provisions, that the courts may inquire into the
justice of a disinheritance such as was attempted in the present case, and if they find
that the disinheritance was without cause, that part of the testament or will may be
pronounced null and void. It remains, however, to be seen whether the evidence
adduced during the trial of the present cause was sufficient to show that the
disinheritance made in paragraph 3 of the will was made for just cause. It appears from
the record that when Rosario Mediavillo was about 14 years of age, she had received
some attentions from a young man that she had received a letter from him and
that her grandfather, Florencio Pecson, took occasion to talk to her about the relations
between her and the said young man; that it was upon that occasion when, it is alleged,
the disobedience and disrespect were shown to her grandfather, and that was the
cause for her disinheritance by her grandfather. The record shows that very soon after
said event she lost the use of her mental powers and that she has never regained
them, except for very brief periods, up to the present time. The lower court, taking into
consideration her tender years, and the fact that she very soon thereafter lost the use
of her mental faculties, reached the conclusion that she was probably not responsible
for the disrespect and disobedience shown to her grandfather in the year 1894 or 1895.

After a careful consideration of the record, we are inclined to believe that the same
supports the conclusions of the lower court and that the same supports the conclusions
of the lower court that he did not commit the error complained of in the first assignment
of error.
With reference to the second assignment of error, it will be remembered that Teresa
Pecson, the mother of Rosario Mediavillo, at the time of her death left two children,
Rosario and Joaquin, and her husband Basiliso Mediavillo, and that said Joaquin
Mediavillo died without heirs. The lower court gave one-half of the inheritance of the
said Teresa Pecson to Rosario Mediavillo and the share that would have gone to
Joaquin Mediavillo, and the share that would have gone to Joaquin Mediavillo, to his
father Basiliso Mediavillo. In that conclusion of the lower court we think error was
committed. The appellant relies upon the provisions of article 925 of the Civil Code, in
his contention that the lower court committed an error. Article 925 provides that:
The right of representation shall always take place in the direct descending line,
but never in the ascending. In collateral lines, it shall take place only in favor of
the children of brothers or sisters, whether they be of the whole or half blood.
The appellee, in support of the conclusions of the lower court, cites articles 935 and
936 of the Civil Code. Article 935 provides that:
In the absence of legitimate children and descendants of the deceased, his
ascendants shall inherit from him, to the exclusion of collaterals.
Article 936 provides that:
The father and mother, if living shall inherits share and share alike. If one of
them only survive, he or she shall succeed to the son's entire estate.
It will be remembered that the whole argument of the appellants with reference to the
first assignment of error was that Rosario Mediavillo had been disinherited and the
court evidently believed that there were no "legitimate children, descendants of the
deceased, surviving," and that therefore the father or mother of said legitimate children
would inherit as ascendants. Inasmuch, however, as there was a descendant in the
direct line, surviving, the inheritance could not ascend, and for the reason the lower
court committed an error in declaring that Basiliso Mediavillo was entitled to inherit that
share of the estate that would have belonged to Joaquin Mediavillo, had he been living.
Therefore, and for all the foregoing, that part of the judgment of the lower court

nullifying and setting aside paragraph 3 of the will is hereby affirmed, and that art of
said judgment which decrees to Basiliso Mediavillo one-half of the estate of Florencio
Pecson, belonging to Teresa Pecson and which would have been given to Joaquin
Mediavillo, had he been surviving, is hereby revoked. And without any findings as to
costs, it is hereby ordered that the cause be remanded to the lower court, with direction
that judgment be entered in accordance herewith, and that such further proceedings be
had as the interested parties may deem necessary, for the purpose of disposing of that
part of the inheritance of Teresa Pecson would have belonged to Joaquin Mediavillo,
had he been surviving.

G.R. No. L-10111

August 31, 1960

Trusteeship of the Estate of BENIGNO DIAZ Y HEREDIA, deceased.


BANK OF THE PHILIPPINE ISLANDS, trustee.
SOLEDAD ROBLES, ET AL., petitioners-appellees, vs. ISABEL MANAHAN DE
SANTIAGO and NESTOR M. SANTIAGO, oppositors-appellants.

Septimo Cualquier legatario que impugna sin exito la legalizacion este


testamento, o que no respete cualquiera disposicion del mismo una vez
legalizado, perdera su derecho al legado y a cualquier beneficio a su favor. El
legado y beneficios a su favor engrosaran el fondo de la testamentaria al
principio y del fideicomiso despues.
In the will, a trust estate was created out of the properties not otherwise disposed of,
which the Bank of the Philippine Islands was designated as Trustee.

Quijano and Azores for appellants.


Alejandro M. Panis for appellees.

After the death of his wife, Da. Rafaela Mercado y Beleti on August 8, 1944, Diaz, then
still living, caused the preparation of a codicil (to his will), dated September 29, 1944,
providing, inter alia

BARRERA, J.:
Benigno Diaz y Heredia, in his will dated July 5, 1944, provided, among others, the
following:
B Lego la renta liquida mensual, deducidos el amillaramiento y otros gastos
indispensables para la conservacion de la finca situada en (Pagina Segunda) la
calle Rosario Nos. 123, 125, 127 y 129, Binondo, Manila, la proporcion que me
corresponde y la de mi esposa Da. Rafaela que representa la mitad, a las
siguientes personas:
"A Domingo Legarda Veinte por ciento (20%)
Soledad Robles Diez por ciento (10%)
Paz Legarda y Robles Veinte por ciento (20%)
Pilar Legarda y Robles Quince por ciento (15%)
Jose Legarda y Robles Diez por ciento (10%)
Vicente Legarda y Robles Diez por ciento (10%)
Angeles Legarda y Robles para su hija Leti Cinco por ciento (5%)
y Colegio de la Consolacion de las M.M. Agustinas Diez por ciento
(10%)
xxx

xxx

xxx

"A Isabel M. de Santiago Cincuenta por ciento (50%); Los hijos de


Domingo Legarda Treinta por ciento (30%); Filomena Diaz Diez
por ciento (10%); Nestor M. Santiago Diez por ciento (10%)
11. Es aplicable a este codicilo, mutatis mutandi, el Parrafo 7.o, Pag. 6 de
mi testamento.
The testator died on November 7, 1944, and his will and the codicil were accordingly
probated (In Sp. Proc. No. 894).

xxx

(Pagina Sexta)
xxx

10. Transcurridos dies o quince aos despues de mi muerte 3: 3 todas mis


propiedades, muebles e inmuebles, derechos y acciones, cuando asi convenga
a los legatarios y los precios sean ventajosos, pueden proceder a la venta de
todos, dando preferencia a los legatarios y de su importe total se deducieran mil
pesos (P1,000.00) para los cuatro hijos de mi difunto hermano Fabian, todos
los gastos y reservando una cantidad suficiente y bien calculada para sufragar
los gastos para otros diez aos para las mandas y misas. El resto se distribuira
a las siguientes personas que aun viven, o a sus descendientes legitimos:

xxx

On November 1, 1949, the Bank of the Philippine Islands was appointed Trustee of the
trust created by the testator in his will and codicil (in Sp. Proc. No. 9226, Trusteeship of
Benigno Diaz), for the purpose of paying the monthly and yearly legacies of the
legatees named therein, to which office it duly qualified.

Pursuant to the terms of the codicil, and 10 years having already elapsed from the
testator's death on November 7, 1944, the Trustee, on January 13, 1955, petitioned the
court, with the consent of all the legatees, for authority to sell all the properties and
liquidate the estate. This petition was granted. Accordingly, the property located at
Rosario, Binondo, Manila, was sold to legatee Isabel Manahan de Santiago on March
18, 1955.
Immediately thereafter, or on March 21 of the same year, legatees Soledad Robles and
her children Pilar, Jose, Vicente and Angeles, all surnamed Legarda (her husband
Domingo and daughter Paz died in 1945) filed a motion praying that the trustee be
ordered to deliver to them, in accordance with the provisions of the will, 90 per cent of
the rentals collected from the property in Rosario during the period of liquidation of the
estate, that is, from January 13 to March 18, 1955, which the Trustee refused to deliver,
the latter contending that upon the authorization of the court on January 19, 1955, of
the sale of the properties, the trusteeship ceased already and the rentals collected
thereafter not only from the Rosario property but also from all the other properties of the
estate, constitute the mass of the residuary estate to be distributed among the legatees
in accordance with the terms of the codicil, i.e., only 30 per cent and not 90 per cent to
the heirs of Domingo Legarda (supra).
On April 11, 1955, appellees Soledad Robles and children filed a second motion
likewise claiming 90 per cent of the rentals from the Rosario property from 1946 to
1949 which had not yet been paid to them. The Trustee Bank countered that said
rentals were liable first to the payment of the expenses of the Testate Estate of Rafaela
Mercado, the Testate Estate of Benigno Diaz, and the Trusteeship of Benigno Diaz
before they could be distributed; that movants, having agreed not to collect the
proceeds of their legacies from 1946 to 1949, they were estopped from claiming the
same; and that the right to claim said legacies from 1946 to 1949, had already
prescribed.
After due hearing on the matter, the court issued a resolution dated May 24, 1955,
granting both motions and directing the Trustee to deliver to Soledad Robles and
children, their shares in the rentals collected from the Rosario property, during both
periods, from 1946 to 1949 and from January 19 to March 18, 1955, after deducting
therefrom the real estate tax, the indispensable expenses for the conservation of the
property, and the corresponding estate and inheritance taxes. From this resolution, the
present appeal was interposed by legatees Isabel Manahan de Santiago, et al., who, in
the distribution of the residuary estate, would get 50 pr cent according to the codicil.

As may be seen from the provisions of the will quoted above, the legacies given to
appellees Soledad Robles and her children were not made subject to any condition;
hence, the right thereto passed to the said legatees upon the death of the testator1 on
November 7, 1944. As the Rosario property (land only, the building having been burned
during the battle for the liberation of Manila) was leased from November 4, 1946 to
March 18, 1955, it follows that the legatees were entitled to their share of the rentals for
the duration of said period. Since they have already been paid their share from
November, 1949 to January 19, 1955, the only question now to be resolved is whether
said legatees- appellees may still demand their share of the rentals from November 4,
1946 to October, 1949, and from January 20 to March 18, 1955.
Against appellees' demands for the rentals during the period 1946-1949, appellants first
set up the defense of prescription, alleging that, being money-claim, the claim should
have been made within 4 years. And, as the demand was made only in 1955, it was
argued the same was already barred.
The testator specifically provided in his will:
G. . . . La finca motivo del legado arriba descrito, deberan conservarse
indefinidamente siempre que sea posible, pero en el imprescindible case de ser
necesario vender, el importe total de la venta debera invertirse en la compra de
otra finca de buenas condiciones que produscan una renta segura igual o mas
que la finca de la Calle Rosario. El deseo mio es que los legatarios sigan
recibiendo sus legados. . . .
And in his codicil, appears the following:
. . . En relacion con esta finca de la Calle Rosario repito que se conserva y no
se venda mientras sea posible, pero en el caso imprescindible de realizar la
venta, se procurara conseguir el mejor precio posible y que cuyo importe se
invierta ya por intereses en un hipoteca segura y ventajosa o ya por alquileres
de otra buena finca que se adquiera y que produzca una renta mayor. . . .
From the above-quoted testamentary provisions, it seems clear that the testator
intended the enjoyment by the legatees, of their respective legacies for the entire
duration of the trust estate, even the specified properties are sold, the proceeds of
which have been directed to be invested in mortgages with interest, or in the purchase
of other rental-bearing properties. The legacies should, therefore, be viewed as one
whole, continuing obligation, to be carried out by the trustee. The fact that the rentals

are to be delivered monthly, did not make each delivery a separate, distinct prestation,
or render the obligation divisible, for to treat it as such, would destroy or alter the
essence of the legacy. Considering that the obligation terminated upon the sale of the
said property on March 18, 1955, the demand for complete delivery of the inheritance
made in April, 1955, has not yet prescribed. Besides, as contended by the appellees
the claim is based upon a specific legacy contained in a probated will. Hence, it is an
obligation based upon a judgment (the will becomes binding upon probate) and the
prescription period is 10 years.
Appellants also contend that, having received their share in the rentals and agreed to
the approval to the Trustee's statement of accounts, appellees are now estopped from
claiming for additional share. This allegation was countered by appellees by claiming
that they merely desisted from pressing the collection of their shares for 1946 to 1949,
by reason of the Trustee's representation that the money would be needed for expense
of administration, and assurance that they would be paid before the division of the
residuary estate among the heirs. Clearly, the matter calls for a determination of
whether or not such agreement took place a question of fact which this Court may
not legally pass upon. And since the trial court inferentially rejected this contention of
appellants and the latter, having appealed directly to us on purely legal issues,
appellants should be deemed to have waived this defense.2

Section 1, Rule 91, of the Rules of Court, should be distributed after deducting
therefrom the corresponding share in the charges, taxes, and expenses of
administration.
The will, it must be remembered, imposed on the legatees merely the payment of the
real estate tax and expenses for the preservation of the Rosario property, and no other.
On the other hand, said will and the codicil thereto, respectively, provide:
Sexta. Todas las demas propiedas, derechos y acciones que no aparecen
dispuetas en este mi testamento y ultima voluntad, o que yo adquiera en lo
futuro, estaran a cargo del fideicomisario por mis encargos y lo que pueda
quedarse de los frutos, en beneficio de los legatarios mencionados por mi este
mi testamento, en partes iguales, o de sus respectivos herederos por
stirpes, despues de deducir los gastos de la administracion.
3.o Es mi deaso que, con la intervencion del Sr. Vicente Santiago, todos los
intereses y otros ingresos de la testamentaria of fideicomiso despues de
deducir los gastos imprescindibles se distribuye el 31 de Diciembre de cada
ao a las siguientes personas:
xxx

It was also alleged that the appellees' filing of an opposition to the probate of the will
caused the forfeiture of their right to the legacies. This contention is similarly
unmeritorious.
True it may be, that under the will, the unsuccessful opposition, by any legatee, to the
probate of the will and non-compliance with its provisions after the probate shall cause
the forfeiture of his right thereunder. It is also true that appellees' counsel filed an
opposition to the probate of the will on January 24, 1946. However, the same
waswithdrawn by the appellees themselves personally soon thereafter, or on March 1,
1946, and the will was finally probated on April 1, 1948. Appellees having voluntarily
desisted from their opposition long before the probate of the will and, apparently, even
before presentation of evidence on the part of the proponent, such an opposition could
not have been the opposition contemplated in the will, as otherwise the provision
prohibiting opposition would be null and void.
The ruling of the court a quo, subjecting the legacies to the payment only of real estate
tax, necessary expenses for conservation of the property, and for estate and
inheritance taxes, was also assailed on the invoked ground that the legacies, under

xxx

xxx

La distribucion que precede es completamente independiente de las pensiones


de la renta de la finca en la Calle Rosario y se hara despues de separar un
fondo de reserva para los gastos de impuesto y gastos de conservacion de las
fincas, en particular de la Calle Rosario. (Emphasis supplied.).
It is evident therefrom that the testator intended the expenses incidental to the
execution of his will to be defrayed and deductible from the residuary estate, excluding
the rents from the Rosario property. On this point, the Rules of Court provide:
SEC. 2. Part of estate from which debts paid where provision made by will.
If the testator makes provision by his will, or designates the estate to be
appropriated for the payment of his debts, the expenses of administration, or the
family expenses, they shall be paid according to the provisions of the will; but if
the provision made by the will or the estate appropriated, is not sufficient for that
purpose, such part of the estate of the testator, real or personal, as is not
disposed of by the will, if any, shall be appropriated for that purpose. (Rule 89.)

There is nothing on record to show that the provisions made by the testator in the will
are inadequate to cover the expenses of administration. On the contrary, it appears that
as of the date of the filing of the appellees' demand, there were still properties of the
estate yet unsold. In the absence of valid and cogent reasons, the provision of the will
as to how the expenses would be satisfied must be respected.
In the belief that the court's order of January 19, 1955, allowing the sale of the
properties of the estate, had the effect of terminating the trust and putting an end to the
legatees' right to the fruits of the Rosario property, appellants contend that appellees'
claim over rentals terminated on said date, hence, appellees are not entitled to the
rentals from January 20 to March 18, 1955, date when the Rosario property was
actually sold. This is equally without merit. The trust has been created to carry out the
dispositions made in the will, principally the payment of the legacies. The codicil
provides that after the lapse of ten or fifteen years (sic) from the date of the testator's
death (November 7, 1944), the properties may be sold upon agreement of all the
legatees and the proceeds of the sale, after deducting certain amounts, are to be
distributed in the manner indicated therein. The approval of the petition to sell did not
automatically terminate the trusteeship, nor did it constitute full accomplishment of the
trust. It was only after the actual sale of the properties on March 18, 1955, and the
distribution of the proceeds directed by the testator that the trust could be considered
as accomplished and terminated. Consequently, the appellees are entitled to their
share of the rentals during the liquidation of the trusteeship from January 20 to March
18, 1955.
Wherefore, in view of the foregoing considerations, the resolution appealed from, is
hereby affirmed, in toto, with costs against the appellants. So ordered.

G.R. No. L-30634 August 27, 1980


BRENDA J. DEBUQUE, ESTRELLA L. JAVELONA, ROSENDO L. JAVELONA JR.,
ARTURO L. JAVELONA, YSMAEL L. JAVELONA, VIVIAN L. JAVELONA,
ROBERTO L. JAVELONA and EDUARDO L. JAVELONA,petitioners, vs.
HONORABLE RAFAEL CLIMACO, Judge of the Court of First Instance of Silay
City, Negros Occidental, RENATO JAVELONA, EDNA JAVELONA, EDMUNDO
JAVELONA, ERLINDA JAVELONA, and LILIA LIBOON, Judicial Guardian for the
Minors, CAMILO JAVELONA, BONITO JAVELONA, BELENDA JAVELONA,
ARMANDO JAVELONA and EDWIN JAVELONA, respondents.

The estate was first administered by Arturo Javelona, a legitimate son who was
appointed Special Administrator on July 5, 1962 (p. 9, CFI rec.). He was replaced by his
elder brother, Rosendo Javelona Jr., who was appointed Judicial Administrator on
December 13, 1962 (p. 48, CFI rec.).
On July 15, 1964, the parties, to avoid a protracted and expensive court litigation,
entered into an Amicable Settlement Compromise Agreement whereby they agreed that
the first set of heirs will receive 71.62% of the decedent's net estate which shall be
equally divided among them, while the second set of heirs wig receive 28.38% of the
net estate, likewise to be equally divided among them (pp. 192-194, CFI rec.). This was
in accordance with the Amended Project of Partition drawn by the Judicial Administrator
and the second set of heirs (pp. 183-190, rec.), and approved by respondent Judge on
July 20, 1964 (p. 195, CFI rec.).

MAKASIAR, J.:
Petition for certiorari to review the order dated February 15, 1969 of the Court of First
Instance of Negros Occidental, Branch I of Silay City, distributing the balance of the
intestate estate of the late Rosendo Javelona Sr. in Special Proceeding No. 270 (6688)
entitled "Intestate Estate of the late Rosendo Javelona Sr., Rosendo Javelona Jr.,
Administrator," which awarded the amount of P12,081.51 to petitioners and P42,587.88
to private respondents.
The following facts are not disputed:
Rosendo Javelona Sr. died intestate on June 22, 1962 in Silay City (p. 2, Petitioners'
Brief found on p. 185, rec.). The deceased left two sets of heirs, i.e., the first set
consisting of his widow Estrella Libo-on Javelona and their seven (7) legitimate
children, and the second set consisting of his nine (9) illegitimate children. They are the
petitioners and private respondents herein, respectively.
At the time of the institution of these intestate proceedings before the Court of First
Instance of Negros Occidental, Brenda Javelona, the youngest legitimate child was still
a minor (P. 6, CFI rec.), and was placed under the guardianship of her mother, Estrella
Libo-on Javelona. On the other hand, only the eldest, Renato Javelona, was of age
among the illegitimate children (p. 47, CFI rec.) so that the minors were likewise placed
under the guardianship of, first their mother, Serena, Liboon (p. 62, CFI rec.), then of
Renato Javelona upon the death of their mother (p. 101, CFI rec.), and later of Eulalia
Libo-on (p. 311, CFI rec.).

Since the bulk of the estate consisted in decedent's 1/3 share in Hacienda Banilad,
which he owned in common with his two brothers under the partnership "Javelona
Brothers" (pp. 3 & 108, CFI rec.), the judicial administrator was authorized to participate
in its partition and subsequent sale (p. 97, CFI rec.). Other properties of the estate were
sold and converted to cash, and the heirs were allowed withdrawals by way of
advances chargeable against their shares. As of July, 1967, the withdrawals by the first
set totalled P527,601.32; while the withdrawals by the second set amounted to
P180,768.35, per Consolidated Reports of the Administrators for the period from
September 2, 1962 to July, 1967 (p. 1124, CFI rec.), which were approved by the Court
on December 15, 1967 (p. 994, CFI rec.).
On January 9, 1968, the Court made an order of disposition (pp. 1039- 1049, CFI rec.)
which was later modified per Amended Decision dated September 4, 1968 [pp. 11291132, CFI rec.]. These two orders, which had both become final were later clarified in
the challenged order of Feb 15, 1969 (pp. 1190-1192, CFI rec.), which also ordered the
distribution of the residuary estate consisting of P54,669.39 deposited in the Philippine
Commercial and Industrial Bank in Bacolod City, as follows:
To the first set and the widow................. P12,081.51
To the second set...................................... 42,587.88
P54,669.39

This order of February 15, 1969 is now the subject of this appeal by certiorari. It was
filed by Brenda Javelona Debuque, the youngest legitimate heir who had already been
emancipated by her marriage to lawyer Jorge Debuque (p. 448, CFI rec.) and later he
becoming of age. The said heir, who had acquired most of the interests of the other
legitimate heirs (pp. 1227 & 1281, CFI rec.), first filed a motion for reconsideration [pp.
1204, CFI rec.] on March 11, 1969 or thirteen (13) days from receipt by her on February
26, 1969 [p. 1240, CFI rec.] of the assailed order, praying that the Court reconsider its
order so that the remaining balance of P54,669.39 belonging to the estate be
proportionately divided between the first set and the second set of heirs according to
their respective balances, which were alleged to be: (a) P49,140.52 or P39,106.66 for
the first set, depending on whether or not some P10,030.86 or P17,033.26 excess
withdrawals by the widow were to be included in the aggregate amount already
received by the legitimate heirs; and (b) P42,587.98 for the second set, which is
admitted by both parties. The movant alleges that because the total of the above
balance exceeds the remaining cash deposit of the estate, payment to the two sets of
heirs should be proportionately reduced.
The said motion likewise:
1) questions the inclusion of the excess withdrawals of the widow
[P17,033.26 or even only P10,030.86] to the over-all total withdrawals of
the first set; and
2) asks, as a corollary motion, for the deduction from the corresponding
shares of the members of the first set the amounts they owe movant
Brenda and for delivery of the same to the latter, together with her share
in the residuary estate.
The said motion was denied per order of respondent Judge dated March 31, 1969 (pp.
1226-1228, CFI rec.), received by movant Brenda through counsel on April 14, 1969
[pp- 1228 & 1242, CFI rec,].
On April 25, 1969 or eleven (11) days from receipt of such order of denial, Brenda filed
a second and last motion for reconsideration [pp. 1247-1251, CFI rec.], thus using up a
total of twenty-four days of her period to appeal.
Her second motion for reconsideration was denied "for the reasons already given in the
order of March 31, 1969" (please see par. 4, p. 24, rec.). This clearly shows that the
second motion was pro forma, since the grounds alleged were already available at the

time of the filing of the first motion (Vaswani vs. Tarachand Bros., 110 Phil. 527). Hence,
it did not interrupt her period to appeal, so that this petition should have been filed on or
before May 2, 1969 (May 1st being a legal holiday).
Even granting that the second motion were not pro forma and should therefore suspend
the running of her period to appeal, her receipt of the second order of denial on June
13, 1969 (p. 3, rec.) left her only six [6] days from said date within which to perfect her
appeal. Such being the case, this petition, which was filed on June 25, 1969 or twelve
(12) days thereafter, is obviously out of time.
But even on the merits, this petition ought not to prosper.
The petitioners are the members of the first set, Brenda having included all her
legitimate co-heirs as co-petitioners in view of her acquisition of most of their interests.
They have raised as the only issue in this petition the alleged grossly disproportionate
and unfair distribution by respondent Judge to the co-heirs consisting of petitioners on
the one hand and respondents on the other, of the decedent's residuary estate
amounting to P54,669.39, in contravention of the expressed provisions of the Civil
Code. The bone of their contention is that the respondent Judge failed to apply Articles
1085, 485, 1093, 1095, 1104 and 1019 of the Civil Code in ordering the distribution of
the residuary estate.
On the other hand, private respondents do not dispute the theory of proportionate
distribution as enjoined by the above-mention articles. They contend, however, that the
respondent Judge correctly found the respondents entitled to their full share of
P42,587.98, but the petitioners' share of P12,081.51 is actually more than what they
are entitled to, after they received and enjoyed to the exclusion of the private
respondents the following amounts due to the estate of the deceased:
P6304.31 due from the Special Administrator, Arturo L. Javelona
(member of the first set), whose remaining share was acquired by
petitioner Brenda Debuque [p. 16, rec.];
P4,823.23 share from the Hacienda Anangui, representing 1/2 of the
profits realized from the sale of the 1/3 interest held by the deceased in
the said hacienda, which he owned in common with his two brothers.
The same as sold by the first set of heirs without the proceeds having
been reported as income of the estate. The court, after hearing, found
this to be conjugal property.

P23,532.85 widow's share of the conjugal debt amounting to P47,065.71


which was fully paid by the estate.
P34.660.49 TOTAL
In other words, while petitioners insist that the computation of their 71.62% share in the
estate should be based on P794,216.99 the total value of the net estate, private
respondents contend that it should be based only on P759,556.50 because the
difference of P34,660.49 should be excluded in determining the proportionate share of
the petitioners; otherwise, they will participate twice in the abovementioned amount.

Because these relate to a factual determination of the value of the net estate as well as
the remaining shares of the parties therein, which factual determination is dependent
on the exclusion of P34,660.49 as above explained, they are deemed waived in this
direct appeal to the Supreme Court.
At any rate, We are satisfied that the lower court did not err in finding that "the shares of
the heirs in the deposit in the Philippine Commercial and Industrial Bank are as follows:
To the First Set and the Widow-Twelve thousand eighty-one pesos and
fifty-one centavos........................................... (P 12,081.51)

The trial court has apparently excluded the said amount of P34,660.49 based on the
reasons above-stated.

To the Second Set-Forty-two thousand five hundred eighty-seven pesos


and eighty-eight centavos.............................................(P42,587.88)

Since the petitioners have elected to elevate their case to Us thru the remedy of appeal
by certiorari, they are bound by the factual findings of the trial court. "A direct appeal
from the CFI binds appellant to the findings of the trial court. Because he is deemed to
have accepted the facts as found by the lower court. He may raise only questions of
law" (Lanzar vs. Guerrero Sr., 29 SCRA 107; Abuyo vs. de Suazo, 18 SCRA 600).

P54,669.39

Moreover, in appeals by certiorari, only errors or questions of law may be raised (Sec.
2, Rule 45, Rules of Court; Sec. 17, RA 296, as amended by RA 5440).
It appears, however, that the controversy lies not in the application or interpretation of
the legal provisions invoked by the petitioners, but on how much the two sets of heirs
are entitled to.
A perusal of petitioners' brief (p. 185, rec.) indicates the following assignment of errors:
1. The trial court erred in ordering the delivery of P42,587.88 to the
respondents in full payment of the balance due them and only the sum
of P12,081.51 to the petitioners in partial payment of the balance due
them (P39,109.66) knowing that the assets left of the estate was only
P54,669.39 and therefore insufficient to satisfy fully both claims;
2. The trial court erred in ignoring the request of petitioners for
proportional distribution of the available P54,669.39 whereby petitioners
requested for the delivery to them of the sum of P26,170.38 and to the
respondents the sum of P28,499. 01.

This case has been litigated between two sets of heirs who are by law supposed to be
co-owner of the estate until its partition pursuant to Articles 1078 and 484 of the New
Civil Code. When the parties entered into an amicable settlement-compromise
agreement, they made the agreement as between the two sets, namely, the legitimate
children and the widow as first party (herein petitioners) and the illegitimate "children as
party of the second part (herein private respondents) [pp. 72-73, rec.]. The parties then
agreed to divide the estate between the two contending sets in the ratio of 71.62% to
28.38% and the respective sets to divide their shares equally among themselves.
Corollarily, upon approval and finality of said agreement, the two sets will again become
co-owners of their respective shares as among themselves. Thus, the legitimate heirs
will be co-owners in equal shares of the 71.66% portion of he estate, while the
illegitimate heirs will be co-owners of their 28.38% portion until partition. Meanwhile,
they are governed by the pertinent laws on co-ownership and succession.
When the trial court issued the assailed order of February 15, 1969, the amounts of
P4,823.33 which is due from petitioner Arturo L. Javelona as Special Administrator;
P6,304.31 corresponding to the share of the deceased in the profit of P12,604.31
resulting from the sale of his 1/3 interest in Hacienda Anangui; and P23,332.85 the
widow's share in the conjugal debt of P47,065.71 which was fully paid by the estate,
were all determined to be due from members of the first set.

As pointed out above, the Special Administrator petitioner Arturo, the second eldest
legitimate heir and the widow belong to the first set. The proceeds of the sale of
Hacienda Anangui were admittedly received by the legitimate heirs [please see Deed of
Absolute Sale, pp. 67-68, rec.; par. (b) of Petitioners' Reply and Opposition to Motion to
Dismiss, p. 102, rec.]. Thus, the members of the first set are solidarily liable for the
estate's losses due to the amounts they have taken and have not restored to the estate,
pursuant to Article 947 of the New Civil Code, which is quoted hereunder:
Art. 927. If two or more heirs take possession of the estate, they shall be
solidarily liable for the loss or destruction of a thing devised or
bequeathed, even though only one of them should have been negligent.
This solidary liability should be understood to cover not only negligence (culpa) but also
fraud (dolo) and delay (mora) [Padilla's Civil Code annotated (1973), Vol. III, p. 3961].
Although the Code Commission says that this Article is for the protection of devisees
and legatees, it may be applied in relation to Article 1087, which provides as follows:
Art. 1087. In the partition the co-heirs shall reimburse one another for
the income and fruits which each one of them may have received from
any property of the estate, for any useful and necessary expenses made
upon such property, and for any damage thereto through malice or
neglect.
Certainly, the share in the P12,604.31 profit realized from the sale of the decedent's
1/3 interest in Hacienda Anangui represents income of the estate.
WHEREFORE, THE PETITION IS HEREBY DENIED, WITH COSTS AGAINST THE
PETITIONERS.

G.R. No. L-27745 October 18, 1977


MISAEL P. VERA, as Commissioner of Internal Revenue, petitioner,
vs. Hon. Judge PEDRO C. NAVARRO, in his capacity as Judge of the Court of
First Instance of Pasig, Rizal (Branch V MAGDALENA ABANTO and CAMILO
ERIBAL, as voluntary residual heirs of the Estate of the deceased ELSIE M.
GACHES; DELIA P. MEDINA, as attorney-in-fact of said heirs; BIENVENIDO A.
TAN, SR., as Executor of the Estate of ELSIE M. GACHES; PHILIPPINE NATIONAL
BANK; PHILIPPINE BANKING CORPORATION; THE OVERSEAS BANK OF
MANILA; and BANCO FILIPINO SAVINGS AND MORTGAGE BANK, respondents.
CASTRO, C.J.:t.hqw
This is a petition for certiorari, mandamus, prohibition and injunction filed by the herein
petitioner Misael P. Vera, in his capacity as Commissioner of Internal Revenue
(hereinafter referred to as "Commissioner"), against the Honorable Judge Pedro C.
Navarro, in his capacity as Judge of the Court of First Instance of Pasig, Rizal
(hereinafter referred to as "respondent Judge"), on account of three orders dated June
5, 8 and 9, 1967, which the latter issued in Special Proceedings No. 5249 entitled "In
the Matter of the Testate Estate of Elsie M. Gaches Bienvenido Tan, Executor,"
which the Commissioner maintains were issued without or in excess of jurisdiction or
with grave abuse of discretion.
It appears that one Elsie M. Gaches died on March 9, 1966 without a child. The
deceased, however, left a last will and testament in which she made the following
relevant disposition of her estate, to wit: +.wph!1
3. After payment of my just debts and funeral expenses I intact that the
balance of my property, both real and personal in the Philippines, he
distributed as follows: +.wph!1
'a) to my driver, PACITO TROCIO Ten Thousand
Pesos (P10,000.00);
b) to my lavandero, VICENTE JERODIAS One
Thousand Pesos (P1,000.00);
c) to my gardener, CRISANTO SALIPOT, JR. Five
Hundred Pesos (P500.00);
d) the balance of my estate in the Philippines shall then
be divided in half; One-half (1/2) to be given to CAMILO
ERIBAL and the other half to MISS MAGDALENA
ABANTO;
e) to MISS CONSUELO L. TAN My office table and
chair now in the library of my house, and one of the
carpets in my house to be selected by her;'

4. All my property in the United States consisting of furs, jewelry and


stocks I leave to my sister BESS LAUER widow, and at present a
resident of San Francisco, California.
On March 11, 1966, the herein respondent Judge Bienvenido Tan, Sr. (hereinafter
referred to as "Judge Tan") filed with the Court of First instance of Pasig, Rizal a petition
for the probate of the aforesaid will On Aped 21, Judge Tan was appointed as executor
of the testate estate of Elsie M. Gaches without a bond.
In a letter, dated June 3, 1966, Judge Tan informed the Commissioner that the testate
estate was worth about ten million (P10 million) pesos and that the estate and
inheritance taxes due thereon were about P9.5 million.
On June 11, 1966, the herein respondent Atty. Delia P. Medina (hereinafter referred to
as "Atty. Medina"), representing herself as the attorney-in-fact of the herein
respondents Camilo Eribal and Magdalena Abanto, filed with the probate court a motion
praying that the executor of the estate be authority to give a monthly allowance to the
voluntary heirs Abanto and Eribal from the month of May, 1966 until "the receipt of the
recommended advance of inheritance of P100,000.00 each recommended by the
Executor in his motion of June 6, 1966 and/or final distribution has been made to said
heirs of their respective shares in the estate." This prayer was granted by the probate
court in an order dated June 25, 1966 (subsequently clarified in an order dated August
11, 1966).
On July 9, 1966, the Commissioner filed with the probate court a proof of claim for the
sum "of P192,364.00 as income tax for 1965 and 1% monthly interest due from the d
Elsie M. Gaches."
On July 19, 1966, Judge Tan filed with the probate court a motion praying for authority
to make the following additional advance payments (1) To Abanto and Eribal,
P150,000.00; (2) To Bess Lauer, $75,000.00; (3) To Judge Tan as advance executor's
fees, P50,000.00; and (4) To Attys. Medina and Bienvenido Tan, Jr., P75,000.00 each
as advance attomey's fees. In this motion, Judge Tan claimed that the estate was very
liquid and that "any claims whatsoever against the Estate and the Government shall be
amply protected since over P7,000,000.00 worth of shares shall still remain to answer
therefor (Sec. 1, Rule 90, Rules of Court)." The respondent Judge granted Judge Tan's
prayer in an order dated July 23, 1966,
In a letter, dated November 4, 1966, the Commissioner advised Judge Tan to Pay to
the Bureau of Internal Revenue the sum of P1,398,436.30 as estate tax and
P7,140,060.69 as inheritance tax, the investigation of his office having allegedly
disclosed that the next value of the testate estate was P10,212,899.20. 1 Judge Tan
disputed the correctness of the assessment in a letter sent to the Commissioner.
On November 26, 1966, the Commissioner filed with the probate court a proof of claim
for the death taxes stated in the assessment notice sent to Judge Tan. On the same
date, the Commissioner also submitted to the probate court for its resolution a motion
praying: (1) for the revocation of the court's orders dated June 25, July 6, July 23 and

August 11, 1966 and all other orders granting the payment of advance inheritance,
allowances and fees; (2) for the appointment of a co-administrator of the estate to
represent the Government; and (3) for the non-disbursement of funds of the estate
without prior notice to the Commissioner. Although the records do not disclose that the
probate court specifically disposed of this motion, the said court, from its subsequent
actuations, may be considered to have impliedly denied the Commissioner's prayers for
the appointment of a co-administrator and the non-payment of advance allowances and
fees.
On January 19, 1967, the probate court authorized the conversion of the amount of
P75,000.00 previously ruled to be paid to Atty. Medina as advance attomey's fees in its
order of July 23, 1966 into allowances for Eribal and Abanto.
On April 14, 1967, with the Probate court's approval, Judge Tan paid to the Bureau of
Internal Revenue the amount of ?185,286.93 as estate tax and, on April 24, 1967, the
amount of P1,055,776.00 as inheritance tax. These payments were based on a tax
return filed by Atty. Medina on March 8, 1967 with the Bureau of Internal Revenue.
On June 3, 1967, Judge Tan submitted to the probate court for approval a final
accounting and project of partition of the testate estate. Acting thereon, the respondent
Judge issued an order, dated June 5, 1967, for the partial distribution of the estate as
follows: +.wph!1
Submitted for resolution of this Court is the Amended Final Accounting
and Project of Partition dated May 27, 1967, presented by The executor.
Atty. Paredes manifested that he has no objection to the approval
thereof provided that certain items enumerated therein be corrected or
modified, as follows: the amount of shares in the Lepanto consolidated
Mining Co. should be 6,105,429 instead of 6,015,429, as reported; the
amount of P11,537.60 reported as expenses made on January 30, 1967
should be cancelled or excluded . . . and that the time appearing as
expenses made on May 10, 1967 payable to Apolonio manifastation
illegal should be only P114,000.00 instead of P135,000.00 . . . which
manifestations were also adopted by Atty. Virgilio Saldajeno of the
Bureau of Internal Revenue, and in addition, he objected in principle to
the Executor Fees and to the Attorney's Fees as excessive but left the
matter to the discretion of the Court.
Considering, further, the manifestations of Atty. Saidajeno that him has
no objection to the partial distribution of the estate as long as it an he
shown that the rights and interests of the government can be full
protected, and it appearing from the subsequent manifestation of Atty.
Paredes, counsel for the heirs, that sufficient assets with a nutrient
market value of at least P8,000,000.00 will be left to the estate even if a
partial distribution in the amount of P3,000,000.00 is made for which
reason the rights of the government to collect whatever deficiency, taxes,

if any may be asses it may be assessed in the future the heirs have
already paid in good faith even ahead of its due dates transfer taxes in
the total amount of P1,241,062.93, the Amended Final Accounting and
Project of Partition dated May 27, 1967 may be approved, subject Lo
this following, terms and conditions:
1. The Executor is hereby discharged from any and all responsibilities
that lie has pertaining to the estate;
2. The voluntary heirs Magdalena Abanto and Camilo Eribal shill be
responsible for all taxes of any nature whatsoever which may be due the
government arising out of the transaction of the properties ol' the estate
and the environment can, if it so desires, register its tax lien in the
remaining assets after a partial distribution of the estate;
3. Bess Lauer, sister and heir of the deceased shall be fully for, all
United States taxes pertaining to her share in the estate.
WHEREFORE, subject to the above terms and conditions, entitled Final Accounting
and Project of Partition dated May 27, 1967 submitted by the Executor. as modified in
the, manifestation of Atty. Paredes and Saidajeno, is hereby approved.
1
. Pacita Trocio
P10,000.0
0
2. To Vicente Jerodias
1,000.00
3. To Vicente Crisanto salipot, Jr.
500.00
4. To Magdalena Abanto and Camilo Eribal, share and share alike
2,330.00
thru their attorney-in-fact Delia P. Medina, cash in the amont of
5. To Judge Bienvenido A. Tan, Sr.
120,000.0
0
6. To Atty. Bienvenido A. Tan, Jr.
150,000.0
0
The aforesaid amount is hereby ordered to be taken from the funds of
the estate deposited with the Philippine National bank.
As to the other properties remaining after this partial distribution,
consisting of the following:
A. BANK DEPOSITS:
1
.
Philippine Banking
559,147.41
Corporation
2.
Philippine National Bank
238,5000.00
3.
Overseas Bank of Manila
700,000.00
4.
Banco Filipino Savings &
581.00
Mortgage Bank
5.
Refund from expenses
32,537.60

B. HOUSE AND LOT LOCATED AT NO. 50 TAMARIND ROAD,


FORBES PARK, MAKATI, RIZAL;
C. SHARES OF STOCK IN THE FOLLOWING:
1
. Lepanto Consolidated Mining Co.
1,105,429 shares
2. San Miguel Corp.
16,692 shares
(common)
3. San Miguel Corp.
500 shares
(preferred)
4. Central Azucarera del Pilar
17,755 shares
5. Manufacturas Textile Industriales de
10,368 shares
Filipinas, Inc.
6. Consolidated Mines, Inc.
85,858 shares
7. Mayon Metal Corporation
5,000 shares
8. Soliangco & Co Inc.
25 shares
9. San Juan Heights
5 shares
10. Metropolitan Insurance Co.
443 shares
11. Realty Investment Inc.
652 shares
(10 shares, management & 642 common)
The same shall be turned over and delivered to the attorney-in-fact of
the voluntary heirs. Atty. Delia P. Medina, to be held by her to answer for
whatever deficiency estate and inheritance taxes may still be due from
the estate and the heirs in favor of the government.
SO ORDERED.
Pasig, Rizal, June 5,1967.+.wph!1

R
R
O

+
.

w
p
h
!
1
Judge

(
S
g
d
.)
P
E
D
R
O
C
.
N
A
V
A

On the same day (that is, June 5, 1967), the Commissioner, having been informed in
advance about the foregoing order by certain undisclosed sources, issued warrants of
garnishment against the funds of the estate deposited with the Philippine National
Manial, the overseas Bank of Manila, and the Philippine Banking Corporation, on the
strength of sections 315-330 of the National Internal Revenue Code.
On June 7, 1967, Atty. Medina filed in the probate court a petition for the discharge of
the writs of punishment issued by the commissioner. On June 8, 1967, the respondent
Judge issued an order lifting the wants in question.
On June 9, 1967, the Philippine National Bank filed a motion in the probate court
praying that it be authority to deposit with the said court the money in its hands in view
of the conflicting claims of the parties over the funds in dispute. On the same day (that
is, June 9, 1967), the respondent Judge issued an order denying the said motion and
threatening the bank officials who refuse to implement its orders of June 5 and 8, 1967
with contempt. Atty. Medina was consequently able to withdraw the sum of
P2,330,000.00 from the PNB. A copy of this order of June 9, 1967 as well as the orders
of June 5 and 8, 1967 were received by the Commissioner on June 13, 1967.
On June 16, 1967, the Commissioner filed a motion for reconsideration (supplemented
on June 22, 1967) of the orders of the probate court dated June 5, 8 and 9, 1967. On
July 6, 1967, however, the Commissioner, on the belief that the probate court's
resolution on its motion was not legally necessary, filed with this Court the instant
petition for certiorari, mandamus, prohibition and injunction against the aforesaid orders
of the respondent Judge. The petition at bar is based on the following propositions:
(1) That the distributive shares of an heir can only be paid after full payment of the
death taxes. As this case subsequently progressed before this Court, the position of the
Commissioner would seem to be that the deficiency income taxes due and payable
during the lifetime of the deceased should also be paid first.

(2) While partial distribution of the estate of a deceased may allowed, a bond must be
filed by the distributees to secure the payment of the transfer taxes. Subsequently,
however, the Commissioner changed his position, stating that such distribute may be
made so long as the payment of the taxes due the government is "provided for," citing
section 1, rule go of the Rules of Court in relation to sections 95 (c), 97, 103, 106 and
107 (c) the National lnternal Revenue Code.
(3) That the executor of an estate cannot be discharged without the payment of estate
and inheritance taxes. The Commissioner later modified his stand on this ProPosition in
line with the view that it is sufficient if the payment of the said taxes is "Provided for.,,
(4) That the delivery of properties of the estate to a stranger [that is, to the voluntary
heirs herein] is not sanctioned by law. Later, as the case at bar Progressed, and in view
of a compromise offer made by the respondents Abanto and Eribal to pay the taxes
being claimed by the Bureau of Internal Revenue, the Commissioner advanced the
view that this proposition is already moot and academic.
(5) That the respondent Judge has no authority to quash or dissolve writs of
garnishment issued by the Commissioner. Subsequently, however, the Commissioner
reversed his stand on this point and stated that the probate court may so dissolve said
writs of punishment as the assets in question were then in custodia legis, citing
Collector vs. Vda. de Codeniera L-9675, Sept. 28, 1957.
Taking stock of the Commissioner's complaint that the disputed orders Were issued
without or in excess of jurisdiction or with grave abuse of discretion, the herein
respondents Atty. Medina and Judge Tan put up a number of factual and legal
arguments, the material ones of which may be stated, in sum, as follows:
(1) The Commissioner's notice of assessment, dated November 10, 966, was based on
wrong premises and valuation of the assets in question; in fact, the Commissioner had
agreed during the pretrial conference in the probate court to reconsider certain items
therein;
(2) The allowance granted to Abanto and Eribal were taken solely from the income of
the estate, a fact admitted by Atty. Saldajeno of the Bureau of Internal Revenue; it is
claimed that in 1965 the estate had an income of P41 1,000.00 and over P750,000.00
in 1966, which could more than cover the questioned allowances;
(3) Eribal and Abanto are willing and bound themselves to assume the responsibility for
the payment of the taxes due against the estate except for the properties located in the
United States which should be charged against Bess Lauer;
(4) The Commissioner does not object to the partition of the estate in question provided
that enough assets are left to pay the taxes against the estate;
(5) The estate has sufficient assets with which to pay the taxes being claimed by the
government;
(6) There was nothing unusual in the institution of Abanto and Eribal as residual heirs of
the deceased; Abanto was the testator's special nurse, companion, secretary and cook

from 1945 until Elsie M. Gaches death in March, 1966; Eribal, on the other hand, was
the deceased's cook, caretaker, companion and driver since 1929;
(7) The grant of allowances was never contested below and cannot now be raised in
the-instant proceedings;
(8) Adequate safeguards were specified in the probate court's order of June 5, 1967 to
cover the tax claims; and
(9) There had been no full distribution of the estate in question without payment of the
transfer taxes since the said taxes are being disputed by the heirs.
In a reply filed on September 7, 1967, the Commissioner stated that he had issued a
revised assessment dated August 24, 1967 and that, furthermore, there were due from
the estate deficiency income taxes for the years 1961 to 1965 in the total sum of
P1,182,296.16, for which reason the estate should not be ordered distributed until the
same is fully satisfied. In a rejoinder, Judge Tan claimed that the August 24, 1967
assessment could still be reduced considerably. The contents of the mentioned revised
assessment which was addressed to Atty. Medina are, inter alia, as follows: +.wph!
1
Madam:
... I have the honor to advise that in a reinvestigation conducted by this
Office, for transfer tax purposes, it was ascertained that she left real and
personal properties in the sums of P377,912.50 and P5,963,822.31
respectively, or a gross estate of P9,341,734.81. The amounts of
P193,892.38, P462,022.83 and Pl,226,783.53, representing accrued
household and medical expenses, funeral expenses and income taxes
(1961-1965) payable, respectively, or a total of P1,882,198.74, were
allowed as deductions resulting in a net taxable estate in,the sum of
P7,459.536.07 subject to estate and inheritance taxes.
In view thereof, there are hereby further assessed the sums of P891,673.68 and
P4,353,972.87 as deficiency estate and inheritance taxes and penalty still due on the
transmission of the decedent's estate, after, crediting the sums of P185,286.73 and
P1,055,776.00, which were paid on April 4, 1967 and April 24, 1967, details of which
are shown hereunder:
Estate tax
Pl,076.960.41
Less: Amount Paid
185,286.7
Total
P891,673.69
Inheritance tax
Corporation CPA Certificate
Total

5,448.87
300.00
P5,409,748.87

Less Amount Paid

1,055,776.00

Deficiency Inheritance Tax & Penalty


P4,353,972.87
xxx xxx xxx
The deadlines for the payment of the aforementioned transfer taxes without penalty
were December 9, 1967 for the estate tax and March 9, 1968 for the inherit tax.
On Sepember 9, 1967, Atty. Medina riled with this Court a pleading captioned
"Compliance and Offer of Compromise to Terminate this Case" in which she stated the
following:+.wph!1
xxx xxx xxx
4. Although respondents voluntary heirs intend to assail and question the
correctness of said assessment only insofar as the same has disallowed
the deductions claimed by them for personal services rendered by
various persons in the total sum of P366,800.00, foregoing thereby other
possible objections to the other items just so this case can be earlier
disposed of, said repondents, nevertheless, are willing to pay even
before these due dates the entire amount-specified in said assessment,
but under protest insofar as the same has disallowance is concerned, in
order to already terminate and dispose of this case before this
Honorable Court.
To pay the taxes in question, Atty. Medina prayed in her offer of that she and Abanto
and Eribal be authorize to make use of the funds of the estate on deposit with the
Philippine National (P238,500.00), the Banking Corporation (P559,147.41), the Banco
Filipino savings and Mortgage Bank (P581.00), and the Bank of Manila (P700,000.00),
and to gradually dispose of and sell the shares of stock representing of the delegate
with an estimated market value of P2,154,026.36. Also included among the assets for
which authority to sell was being procured in the said offer of were 2,442,000 Lepanto
Consolidated Co. which Abanto and Eribal with the probate court niether this Court
issued a pre injunction in the case at bar on july 10, 1967 ordering, among others, Atty.
Medina, Abanto and Eribal to restore to the court a quo the amount of P2,330,000.00
withdrawn from the Philippine National Bank pursuant to the questioned orders of the
probate court, and every other money or property revived by them by of said
questioned orders. The mentioned Lepanto shares had then an estimated market value
of P2,588,520.00. It should bear mention, at this point, that the money withdrawn from
the Philippine National Bank was not returned by Atty. Medina, Abanto or Eribal to the
probate court, these respondents having prayed this Court that the deposit of the
mentioned stocks be as full compliance by them with the writ of pre injunction issued by
this Court.
On September 19, 1967, this Court issued a resolution requiring the Commissioner to
submit a memorandum on how he arrived at his original assessment of more than ?
8.83 million and the revised assessment of only about ?6.48 million, showing a reduced
difference of more than P2 million. The Commissioner submitted to this Court the

required memorandum on May 25, 1968, the important items and figures described in
which may be summed up comparatively as follows: +.wph!1
ESTATE OF ELSIER GACHES
ASSETS
ORIGINAL
REVISED
ASSESSMENT
ASSESSMENT
Cash in bank Philippine
Pl,172.635.62
P1,172,635.62
Foreign (US$ P3.95)
559,335.00
559,335.00
CarsLincoln Pl8,000.00
Volkswagen 7,000.00
(Vauxhalll)
25.000.00
12,000.00
Furnitures
30,000.00
30,000.00
Shares of stock
7,923,576.23
7,189,851.69
Forbes Park lot
(at P144.73/sq. in.)
383,202.35
(at P97.50/sq.m.)
258,862.50
House ------- P111,850.00
Swimming Pool
5,000.00
Fence -------- 2,200.00
119,050.00
119,050.00
TOTAL ASSETS
P10,212,899.20
P9,341,734.81
+.wph!1
LIABILITIES AND DEDUCTIONS
Estimated Income Tax
Payable (1965)
P192,364.00
(1961-1965)
P1,882,783.53
Aaccrued medical expenses
13,000.00)
Funeral expenses
73,320.00)
193,392.38
Judicial exercises
331,026.40
462,022.83
TOTAL LIABS. &
DEDUCTIONS
P610,190.60
P1,882,198.74
TRANSFER TAXES PAYABLE
Gross Estate
P10,212,899.20
P9,341,734.81
Less: Laibs. & Deductions
610,190.60
1,882,198.74
Net Taxable Estate
P9,602,708.60
P7,459,536.07
Less Estate'tax Due
P 1,398,436.30
Pl,076,960.41
Estate Subj. to Inh. Tax
P 8,204,272.30
P6,382,575.66
Distribution of Hereditary
Estate

C. Salipot, Jr.
P 500.00
P 500.00
V. Jerodias
1,000.00
1,000.00
P. Trocio
10,000.00
10,000.00
Bess Lauer
672,305.00
672,305.00
M. Abanto
3,760,233.65
2,849,385.33
C. Eribal
3,760,233.65
2,849,385.33
Inheritance Tax Due
C. Salipot, Jr.
P10.00
P 10.00
V. Jerodias
20.00 20.00
P. Trocio
600.00
600.00
Bess Lauer
192,186.75
192,186.75
M. Abanto
3,473,621.97
2,608,316.06
C. Eribal
3,473,621.97
2,698,316.06
Total inheritance Tax due
P 7,140,060.69
P5,409,448.87
Add: Estate Tax Due
P 1,398,436.30
Pl,076,960.41
TOTAL TRANSFER
TAXES DUE
P8,538,496.99
P6,486,409.28
On November 17, 1967, this Court authorized the herein respondents Abanto, Eribal
and Atty. Medina to withdraw funds of the estate deposited with the Philippine Banking
Corporation (P191,673,68) and the Overseas Bank of Manila (P700,000.00) in the form
of cashier's checks payable to the Commissioner for the payment of the estate tax still
unpaid under the terms of the revised assessment.
On November 23, 1967, the Solicitor General filed with this court a manifestation
expressing his conformity, in behalf of the Commissioner, to the offer of compromise
dated September 9, 1967 made by Atty. Medina, subject to certain conditions, such as,
that the cash in the banks of the estate as well as the proceeds to be realized from the
sale of the shares of stock should be turned over to the Commissioner for the payment
of the taxes due against the estate and the heirs thereof. This manifestation was first
opposed by the Acting Commissioner of Internal Revenue on the ground that the
Commissioner (who was then abroad) had actually requested the Solicitor General not
to agree to the mentioned offer of compromise; however, the Solicitor General
subsequently said that the Commissioner's conformity was given to him orally.
On December 5, 1967, Atty. Medina filed with this Court a petition to declare the
Overseas Bank of Manila in contempt for allowing the renewal, without court authority,
of the time deposit of P700,000.00 with the said bank for another year. In a
supplemental motion filed on December 8, 1967, Atty. Medina also prayed that the said
bank and those responsible for extending the maturity date of said time deposit be held
liable for the payment of whatever surcharges, interest and penalties may be imposed
as a consequence of the late payment of the balance of the estate tax assessed
against the estate. It appears that the time deposit in question was held by the said
bank under two certificates, one for P100,000.00 to mature on May 12, 1967, and the

other, for P600,000.00 to mature on June 16, 1967. Judge Tan, however, extended the
maturity date of said time deposits to May 12, 1968. The certificates of time deposit
covering the said funds had been endorsed in favor of the Commissioner in payment of
the unpaid balance of the estate then December 7, 1967) amounted to P700,000.00.
Commmoner, however. mentioned the respondents End an Abanto through their
counsel that his Office - +.wph!1
... regrets that the same cannot be accepted as payment of the
deficiency estate tax in this case since they cannot, at present or on
before December 9, 1967, be. converted into cash. However, we are
holding said certificates of time deposit for possible application in
payment of the unpaid balance of the deficiency estate tax in this case
as soon as said certificates can be converted into cash. It will be
understood in this connection that if the balance of the deficiency estate
tax in this case is not paid on or before December 9, 1967, the same
shall be subject to the interest on deficiency, 5% surcharge and 1%
monthly interest for deliquency.
According to Judge Tan, he caused the extension of the maturity date of the said
deposit but that in doing so he acted in good faith in that the testate estate then had
ample funds and assets and the said time deposit earned a higher interest than a
savings deposit; that he needed no specific court authority for the purpose; and that he
had a gentleman's agreement with the officials of the bank that said deposit could be
withdrawn in advance, such being the custom in banking circles. The Overseas Bank of
Manila, on the other hand, in answer to Atty. Medina's mentioned petition, claimed that
the deposit in question was renewed before the bank received any letter demanding its
release. In view of this impasse and the fast approaching deadline for the payment of
the estate tax, Atty. Medina requested the Commissioner to credit P700,000.00 to the
amount previously paid as inheritance tax; but, apparently, this request was not
honored by the Commissioner.
On January 26, 1968, Atty. Medina filed with this Court a manifestation in which she
alleged that even as the proposed joint manifestation between the parties which was
supposed to describe the matters agreed upon between them and the Commissioner
during a conference hearing held on January 24, 1968 had not yet been shown to her,
she already wished to express her principals, conformity to pay, but under protest, the
deficiency estate tax of P700,000.00 plus surcharges, interest and penalties due
thereon and the inheritance tax in the amount of P4,161,986.12 appearing, to Atty.
Medina, in the mentioned assessment notice dated August 24, 1967; that she was
likewise agreeable to pay, under protest however, the income taxes for 1961 to 1965
against the estate in the demand letter of the Commissioner dated August 29, 1967 in
the amount of P1,175,974.51 plus whatever interest, surcharges and penalties were
due'thereon; and that she was also agreeable to being authority to sell such properties
of the estate as may be necessary for the mentioned -

On the following day, however, that is, January 27, 1968, the herein respondents Eribal,
Abanto and Atty. Medina, on the one hand, and the Commissioner and the Solicitor
General, on the other, filed with this Court a joint manifestation which, inter alia, reads
as follows:+.wph!1
l. That the respondent taxpayers will pay the estate, inheritance and
deficiency income taxes covered by existing assessments; which are
due and collectible from the estate of Elsie M. Gaches, including the
delinquency penaltiesthereon, but without prejudice to any right of the
taxpayer to contest or protest the said assessments at the proper time
and in the proper court;
2. That the respondents Delia P. Medina, Magdalena Abanto and Camilo
Eribal shall submit to this Honorable Court an inventory of all the
properties and assets of the estate ... ;
3. That is order to generate the necessary funds for the purpose of
paying the said taxes and delinquency penalties, so much of the assets
of the estate ... shall be sold ...
4. That respondent Delia P. Medina, . and. Mr. Rodolfo U. Arrano
Supervising Revenue Examiner of the Bureau of Internal Revenue, ...
are hereby proposed to be constituted as the authorized agents of the
parties herein to effect the sale ...;
5. That the said agents shall be direct to sell the assets of the estate ... ;
6. That all negotiations and transactions for the sale of the assets of the
estate shall be made jointly by the authorized agents ... ;
7. That no disposition of any property or assets of the estate shall be
effected except for the foregoing purpose;
8. That this case shall not be terminated until ... the above mentioned ...
taxes and delinquency penalties are fully paid; and liquidated;
9. That the parties pray for the approval of the foregoing propositions.
On February 6, 1968, this Court, acting on the abovement manifestation of Atty. Medina
and the at manifestation of the Parties, issued a resolution authorizing Atty. Medina to
pay, amt, under at, the transfer and in taxes collectible from the estate, including the
accopanying delinquency penalties. A Medina was given the necessary authority to
collect and receive funds payable to the estate in question and to sell such a thereof as
may be necessary.
On February 10, 1968, a motion to declare in contempt Lepanto Consolidated Mining
Co. was filed by Atty. Medina on t ground that the said corporation refused to tum over
to dividends payable to the testate estate unless the Commissioner first lifted his
garnishment order on said dividends.
On February 16, 1968, this Court issued a resolution suspendi the writs to preliminary
junction issued by this Court on July and 17, 1967 and all warrants of garnishment

issued by the Commissioner relative to the estate of Elsie M. Gaches, said suspension
to be effective until such time that Atty. Medina, End and Abanto shall save fully paid
the transfer and income tax including the penalties thereon, covered by existing
assessment Atty. Medina thereafter submitted to this Court performance reports on her
activities relative to the authority given her.
On March 9, 1968, Atty. Medina filed with this Court manifestation stating that she
received a demand letter dated March 9, 1968 from the Commissioner for the payment
of the following 1'756 900- 00 as estate tax, including penalties; (2) P192,186.75 as
inheritance tax corresponding to the share of Bess Lauer; and (3) P451.435.91 as
balance of the income tax for the years 1961 to 1965 Atty. Medina claimed the said
demands to be erroneous for the following reasons' (1) as to the estate tax, the time
deposit in the Overseas Bank of Manila of P700,000.00 plus interest earned of
P60,000.00 as of March 9, 1968 would more than cover the said tax and the certificates
of time deposits were already endorsed to the Cmmissioner on December 6, 1967; (2)
as to the inheritance tax, she (that is. he principals Abanto and Eribal) was not
responsible therefore, as the resolution of this Court dated February 6, 1968 required
her "to pay only the estate, inheritance and in income taxes, under protest covered by
existing assessments, against the Estate, and against the heirs Magdalena Abanto and
Camilo Eribal;" in a supplemental motion, Atty. medina further argued that Bess Lauer
alone was solely responsible for the payment of the inheritance tax on her share and
not the decedent's estate in the Philippines, and that the properties of the testate estate
in the United States of America which consisted of shares of stock and deposits in
banks, being personal properties, were to be excluded from the computation of the
gross estate of the deceased in the Philippines and the computation of the Philippine
estate and inheritance taxes because, under philippine law, the sites of those properties
is the place where they are located, citing Article 16 of the new Civil Code which she
she argued, abandoned the doctrine of mobilia sequuntur personal embodied in Article
19 of the old Civil Code; and (3) as to tile deficiency income tax for 1961-1965, she had
paid the same in the total amount of P1,182,296.16 as of March 9, 1968, which was the
amount stated in the assessment letter of the Commissioner cited August 9, 1967.
According to Atty. Medina, the payment of the taxes was made in the following manner:
on February 27, she paid a total of ?838,518.62 as follows: the income tax
(P715,619.46) in full; interest (P106,855.29) in full, compromise penalty (P5.,000.00) in
full and surcharges P1,052.07) in. part only; and, on March 8, 1968. the amount of
P343,773.54 as payment of the remaining surcharges, Consequently, she argued the
the surcharges and interest, if any were still due, could legally, accrue only from
September 29, 1967 up to February 27, 1968 and only on the tax proper.
On April 16, 1968, a counter-manifestation was filed with this court by the
Commissiorner to the above-metioned manifestation according to the Commissioner,
(that is under existing assessments that is under the letter of demand of August 24 and
29, 1967)
Estate tax (BalanceP700,000.00 (x)

Inheritance tax
4,353,927.87 (xx)
Total Estate and
Inheritance taxes
P5,053,927.87
Deficiency income taxes
for 1961 to 1965
P1,175,974.51 (xxx)
Delinquency penalties for late filing
of income tax return and late
payment of
income tax for 1965 per return
6,321.65 (xxxx)
filedTotal deficiency income taxes for
1961 to 1965 and the delinquency
penalties of income tax 1965 per
return
P1,182,296.16
GRAND TOTAL
P6,236,269.03
+.wph!1
(x) pIus 5% surcharge and 1% monthly interest thereon from December
9, 1967 until full payment thereof; (xx) plus 5% surcharge and 1%,
monthly interest thereon, if the same is not paid in full on or before
March 9, 1968; (xxx) plus 5% surcharge and 1% monthly interest
thereon from August 29, 1967 until full payment thereof; and (xxxx) pIus
additional 1% monthly interest from September 29, 1967 until full
payment thereof.
Further, the Commissioner alleged that after taking into consideration the payments
made by Atty. Medina, the balances as of March 9, 1968 of the death and income taxes
still compatible were as follows:
Estate Tax
Balance of the estate tax
P700,000.00
5%, surcharge
35,000.00
1% monthly interest from
12/9/67 to 3/9/68
21,000.00
Total
P 756,000.00
plus additional 1% monthly interest
from March 9, 1968 until full payment
thereof.
Inheritance Tax
Inheritance tax due and collectible
per letter of demand dated August 24,
1967 (Annex "A")
P4,353,972.87
Less: Payments of inheritance Tax

on March 1 and March 6, 1968 per O.R.


2519938 and 2520026, respectively
Inheritance taxs due and collectible
plus 5% surcharge and 1% monthly
interest thereon from March 8, 1968
until full payment.
Deficiency Income Taxes
Deficiency income taxes from 1961
to 1965 per letter of demand dated
August 29, 1967 plus 5% surcharge and
1% monthly interest up to March 1968
Less: Payments made on February
27, 1968 and March 8, 1968 under O.R.
207001 and 207002
Deficiency income taxes still due
and collectivele
plus additional 1% monthly interest
thereon from March 8, 1968 until full
payment.

4,161,986.12
P191,986.75

P1,289,818.17
P1,182,296.16
P107,522.01

The Commissioner also explained that the i taxes paid by Atty. Medina in the total
amount of P1,182,296.16 "included only the 1/2% monthly interest On deficiency with
respect to the deficiency income taxes for 1961 to 1965 and the 1% monthly Interest for
delinquency up to September 29, 1967 with respect to the income tax for 1965 which
was paid per return, Out did not include the 5% surcharge and 1% monthly interest for
delinquency from August 29, 1967 until full Payment with respect to the income tax for
the 1965 return." The Commissioner consequently prayed that Atty. Medina be ordered
to pay: +.wph!1
(1) The amount of P756,000.00 as balance of the estate tax, 5%
surcharge and 1% monthly interest from December 9, 1967 to March 9,
1968, plus additional 1% monthly interest from March 9, 1968 until full
payment;
(2) The amount of P191,986.75 as balance of the inheritance tax, plus
5% surcharge and 1% monthly interest thereon from March 9, 1968 until
full payment; and
(3) The amount of P107,522.01 as balance of the deficiency income
taxes, 5% surcharge and 1% monthly interest for delinquency up to M
arch 8, 1968, plus additional 1% monthly interest thereon from March 8,
1968 until full payment ... ;
On August 23, 1968, Atty. Medina filed a manifestation with this Court adverting to the
refusal of the Overseas Bank of Manila to permit the withdrawal of the time deposit of

the testate estate in the said bank in spite of the fact that the extended maturity date of
said deposit had may expired. Atty. Medina payed that the bank Ida as one boss able
the deposit of the funds of is well as the who made i of the estate of Elsie M. Gaches
with the said bank be declared in contempt. on September 18, 1968, the Central Bank
Of the Philippines filed with this Court a comment on the urgent manifestation of Atty.
Medina concerning the deposit in question. The Central Bank, which according to the
Overseas Bank of Manila had restrained it from paying its time deposits to the bank's
depositors, averred that this Court's resolution of November 17, 1967 merely authorized
Atty. Medina to withdraw the deposit from the said bank and did not order the bank to
pay the time deposit in question. Moreover, according to the Central Bank, the
nonpayment of the said deposit was not wilful as the Overseas Bank of Manila was in a
state of insolvency. A comment was filed on October 11 1968 by the Overseas Bank of
Manila stating that the majority stockholders of the bank filed a petition against the
Central Bank for certiorari. prohibition and mandamus in this Court in L-29352 entitled
"Emerito M. Ramos, et at. vs. Central Bank;" 2 that the time deposit in question was an
unrecorded transaction; and that the Central Bank prohibited the bank to do business
due to its distressed financial condition, for which reason it could not give preference of
the payment of the said deposit as it might prejudice other creditors of the bank.
On November 11, 19681, Atty. Medina filed with this Court a M. motion ,- reiterating a
previous one to allow the payment of the announced of P6.000.00 to Atty. Manuel M.
Paredes whom she and tile other herein respondent herein Abanto and Eribal
hired as counsel in collection with the settlement proceedings of Elsie M. Gaches
estate. On March 29, 1969. pursuant to a resolution of this Court, Atty. Paredes
ssubmitted knitted a memorandum on the nature and extent for the legal services he
had rendered to tile herein respondents Atty. Medina Eribal and Abanto.
On June 26, 1971, Abanto and Eribal Jointly wrote the Chief Justice, expressing
willingness and agreement to pay the amount due tile government as taxes against the
estate and the heirs thereof, however, the two respondents herein subsequently
retracted their statement in the said letter, claiming they signed and sent the same
without knowing and understanding its effect and consequences.
A perusal in depth of the facts of the instant case discloses quite plainly that the
respondent Judge committed a grave abuse of discretion amounting to lack of
jurisdiction in issuing its orders of June 5, 8 and 9, 1967. Section 103 of the National
Internal Revenue Code (hereinafter referred to as "Tax Code") unequivocally provides
that "No judge shall authorize the executor or judicial administrator to deliver a
distributive share to any party interested in the estate unless it shall appear that the
estate tax has been paid." 3 The aforesaid orders of the respondent Judge are clearly in
diametric opposition to the mentioned Section 103 of the Tax Code and, consequently,
the same cannot merit approval of this Court.
While this Court thus holds that the questioned orders are not in accordance with
statutory requirements, the fundamental question raised herein regarding the
objectionable character of the probate court's mentioned orders has opened other

issues which, not alone their importance to jurisprudence, but the indispensability of
forestalling needless delays when those issues are raised anew, have, perforce,
persuaded this Court that their complete and final adjudication here and now is properly
called for. Said issues may be specificaly framed as follows:
(1) Should the herein respondent heirs be required to pay first the inheritance tax
before the probate court may authorize the delivery of the hereditary share pertaining to
each of them?
(2) Are the respondent heirs herein who are citizens and residents of the Philippines
liable for the payment of the Philippine inheritance tax corresponding to the hereditary
share of another heir who is a citizen and resident of the United States of America. said
share of the latter consisting of personal (cash deposits and, shares) properties located
in the mentioned court
(3) Does the assignment of a certificate of time deposit to the comissioner of Internal
Revenue for the purpose of paying t I hereby the estate tax constitute payment of such
tax?
(4) Should the herein respondent heirs be held liable for the payment of surcharge and
interest on the amount (P700,000.00) representing the face value of time deposit
certificates assigned to the Commissioner which could not be converted into cash?
Aside from the foregoing, there are also other incidental questions which are raised in
the present recourse, viz.,
(5) What should be the liability of the respondents herein on the contempt charges
respectively lodged against them?
(6) What should be a reasonable fee for the counsel of the respondents Atty. Medina,
Eribal and Abanto for professional services rendered In connection with the settlement
of the estate of Elsie M. Gaches?
1. On the matter of the authority of a probate court to allow distribution of an estate
prior to the complete Nuidation of the inheritance tax, the Tax Code apparently lacks
any provision substantially Identical to the mentioned Section 103 thereof. There are
provisions of the Tax Code, e.g., Section 104, which makes it the duty of registers of
deeds not to register the transfer to any new owner of a hereditary estate unless
payment of the death taxes sham be shown; Section 106, which imposes a similar
obligation on business establishments; and Section 107, which penalizes the executor
who delivers to an heir or devise, and the officers and employees of business
establishments who transfer in their books to any new owner, any property forming part
of a hereditary estate without the payment of the death taxes first being shown; but
those provisions by themselves do not clearly establish that the purchase and object of
the statute is to make the payment of the inheritance tax a pre-condition to an order for
the distribution and delivery of the decedent's estate to the lawful heirs there. The cloud
of vagueness in the statute, however, is not entirely unreachable. Section 1, Rule 90 of
the Rules of Court erases this hiatus in the statute by providing thus: +.wph!1

Section 1. When order for distribution of residue made. When the


debts, funeral charges, and expenses of administration, the allowance to
the widow, and inheritance tax, if any, chargeable to the estate in
accordance with law, have been paid, the court, on the application of the
executor or administrator, or of a person interested in the estate, and
after hearing upon notice, shall assign the residue of the estate to the
persons entitled to the same, naming them and the proportions, or parts,
to which each is entitled, and such persons may demand and recover
their respective shares from the executor or administrator, or any person
having the same in his possession. If there is a controversy before the
court as to who are the lawful heirs of the deceased person or as to the
distributive shares to which each person is entitled under the law, the
controversy shall be beard and decided as in ordinary cases.
No distribution shall be allowed until the payment of the obligations
above mentioned has been made or provided for, unless the
distributees, or any of them, give a bond, in a sum to be fixed by the
court, conditioned for the payment of said obligations within such time as
the court directs.
Under the provisions Of the aforequoted Rule, the distribution of a decedent's assets
may only be ordered under any of the following three circumstances, namely, (1) when
the inheritance tax, among others, is paid; (2) who bond a suffered bond is given to
meet the payment of the tax and all the other options of the nature enumerated in the
above-cited provision; or (3) when the payment of the said tax and at the other
obligations mentioned in the said Rule has been provided for one of these thru camar
as the satisfaction of the when tax due from the festate is were present when the
question orders were issued in the case at bar. Although the respondent Judo did make
a condition in its order of June 5, 1967 that the distribution of the estate of Elsie M.
Gaches (except the cash deposits of more than P2 million) shall be trusted to Atty.
Medina for the payment of whatever taxes may be due to the government from the
estate and the heirs them to, this Court cannot subscribe to the proposition that the
payment of the tax due was thereby adequately provided for. In the first place, the order
of June 5, l967 was, for all intents and , a complete distribution of the estate to the heirs
for, the executor who is supposed to take care of the estate was absolutely discharged
the attorney's fees for the of a lawyer who presumably acted as legal counsel for the
estate in the court below were ordered paid as were also the fees for the executor's the
cash funds of the estate were red paid to the cash and the non-cash (real property and
shares of stock) properties were likewise ordered delivered to Atty. Medina whose
participation in the said proceedings was in the capacity of an attorney-in-fact of the
herein respondent Eribal and Abanto. In short, the probate court virtually withdrew its
custodial jurisdiction over the estate which is the subject of settlement before it. In the
second place the respondent Judge, in the distribution of the properties of the estate in
question, relief solely upon the mere mandestation of the counsel for the heirs Eribal

and Abanto that them were affiant of the estate with which to pay the taxes due to the
government. There is no evidence on record that would show that the probate court
ever made a serious attempt to de what the values of the different assets the
correctness of that such properties shall be preserved for the satisfaction of those case
In the third place that main of pesos taxes were being called by the Bureau of Inc.
Revenue, the least reasonable thing that the probate court should have done was to
require the heirs to deposit the amount of inheritance tax being claimed in a suitable
institution or to authorize the sale of non-cash assets under the court's control and
supervision.
The record is likewise bereft of any evidence to show that sufficient bond has been filed
to meet this particular outstanding obligation.
2. The liability of the herein respondents Eribal and Abanto to pay the inheritance tax
corresponding to the share of Bess Lauer in the inheritance must be negated, The
inheritance tax is an imposition created by law on the privilege to receive
property. 4 Consequently, the scope and subjects of this tax and other related matters in
which it is involved must be traced and sought in the law itself. An analysis of our tax
statutes supplies no sufficient indication that the inheritance tax, as a rule, was meant
to be the joint and solidary liability of the heirs of a decedent. Section 95(c) of the Tax
Code, in fact, indicates that the general presumption must be otherwise. The said
subsection reads thus: +.wph!1
(c) xxx xxx xxx
The inheritance tax imposed by Section 86 shall, in the absence of
contrary disposition by the predecessor, be charged to the account of
each beneficiary, in proportion to the value of the benefit received, and in
accordance with the scale fixed for the class or group to which is
pertains: Provided, That in cases where the heirs divide extrajudicially
the property left to them by their predecessor or otherwise convey, sell,
transfer, mortgage, or encumber the same without being the estate or
inheritance taxes within the period prescribed in the preceding
subsections (a) and (b), they shall be solidarity liable for the payment of
the said taxes to the extent of the estate they have received.
The statute's enumeration of the specific cases when the heirs may be held solidarity
liable for the payment of the inheritance tax is, in the opinion of this Court, a clear
indication that beyond those cases, the payment of the inheritance tax should be taken
as'the individual responsibility, to the extent of the benefits received, of each heir.
3. And the effect of the indorsement of the time deposit certificates to the
Commissioner, the same cannot be held to have extinguished the estate's liability for
the estate tax. In the first place,in accepting the indorsement and delivery of the said
certificates, the Commissioner expressly gave notice that his Office +.wph!1
... Regrets that the same cannot be accepted as payment of the
deficiency estate tax in this case may they cannot, at present or on or

therefore December 9, 1967, be converted into cash. However, we are


holding said certificates of time deposit for possible application in
payment of the unpaid balance of the deficiency estate tax in this case
,is soon as said certificates can be converted into cash. ...
In the second place, a time deposit certificate is a mercantile document and is
essentially a promissory note. 5 By the express terms of Article 1249 of the Civil Code
of the Philippines, the use of this medium to clear an obligation will "produce the effect
of payment only when they have been cashed, or when through the fault of the creditor
they have been impaired." From the records of the case at bar, the Commissioner as
well as the herein respondents Atty. Medina, Eribal and Abanto spared no time trying to
collect the value of said certificates from the Overseas Bank of Manila but all to no
avail. Consequently, the value of the said certificates (P700,000.00) should still be
considered outstanding.
4. The estate of Elsie M. Gaches is likewise liable for the payment of the interest and
surcharges on the said amount of P700.000.00 imposed under Section 101 (a) (1) and
(c), respectively, of the Tax Code. 6
The Interest charge for 1% per month imposed under Section 101 (a) (1) of the Tax
Code is essentially a commotion to the State for delay in the payment of the tax due
thereto7 As for the accountant use by the tax payer of funds that nightday shall be in the
government's funds. 8 As the indorsement and delivery of the mentioned time deposit
certificates to the did not result in the payment of the estate tax (for which it was in the
respondents estate is fluently liable for the interest charge imposed in the Tax Code.
The estate cannot likewise be exempted from the payment of the 5% surcharge
imposed by Section 101 (c) of the Tax Code. While there are cases in this jurisdiction
holding that a surcharge shall not be visited upon a taxpayer whose failure to pay the
tax on time is in good faith, 9 this element does not appear to be present in the case at
bar. The Commissioner, as aforesaid, fully informed the respondents Atty. Medina,
Eribal and Abanto of the condition to this acceptance of the said time deposit
certificates. The Commissioner, in fact, advised them in the same letter that "It will be
understood in this connection that if the balance of the deficiency estate tax in this case
is not paid on or before December 9, 1967, the name shall be subject to the interest on
deficiency, 5% surcharge and 1% monthly interest for deficiency." Moreover, Judge Tan
himself, as executor of the estate of Elsie M. Gaches, specifically admitted that he was
the one who caused the extension (and consolidation) of the maturity dates of the two
time deposit certificates in question (one for P100,000.00 to mature on May 12, 1967
and the other for P600,000.00 to mature on June 16, 1967) to May 12, 1968,
It will be worthwhile to mention also, in this connection, that when Atty. Medina applied
to this Court for authorize to the amount of P700,000.00 from the Overseas Bank of
Manila on September 9, 1967, the resolution of this Court dated November 17, 1967,
approve her request authorized her to withdraw the said amount in the form of cashier's
checks payable to the Commissioner. Apparently, because the Overseas Bank of

Manila refused to issue such checks or to allow her to withdraw said amount in view of
the extension of the nuturity date of the deposit in question, Atty. Medina thought that
by simply assigning the time deposit certificates to the Commissioner, she would be
deemed to have paid the estate's obligation in its corresponding amount. However, as
aforesaid the Commissioner was also unable to convert said amount to cash and he
gave announce to that effect to Atty. Medina. Since the refusal of the Overseas Bank of
Manila to snow the withdrawal of the said deposit was then well-known to the parties, it
saw to reas that the tentatives of the estate who stand to be benefited. therefrom, such
as the respondents Eribal and Abanto, should have forthwith asked for authority to pay
the from other funds of the estate. Atty. Medina was, in fact, given the authority by this
Court to sell assets of the estate for the payment of the taxes due to the State, but she
never tried to pay the equivalent amount of P700,000.00 in question from the proceeds
of the Wm she made afterwards. Moreover, it will also be noted that the respondents
EAbal and Abanto, during the pendency of this case, had in their actual ion at least
P2.3 million (the amount they were able to withdraw from the Philippine National Bank
on account of the questioned orders) which they could have very well used for the
payment of the estate tax. They, however, opted to put the same to other uses.
5. We now consider the several petitions for contempt riled in the case at bar, namely,
(a) against the Philippine National Bank on account for allowing Atty. Medina to
withdraw P2,330,000.00 in contravention of the writ of punishment issued by the
Commissioner; (b) against the officer of the Overseas Bank of Manila for allowing the
extension of the maturity date of the mentioned time deposit of P700,000.00 and for
refusing to pay the same after the extended term expired; (c) against Judge Tan who
renewed the maturity date of the said time deposits; (d) against the Lepanto
Consolidated Mining Co. for refusing to turn over dividends payable to the estate of
Elsie M. Gaches unless the Commissioner first lifted his punishment order; and (e)
against the herein respondents Atty. Medina, Eribal and Abonto for citing shares of
stock with the probate court instead of the cash amount of P2,330,000.00 which they
withdrew from the renewed National Bank on account of the questioned orders of the
probate court, contrary to the resolutions of this Court dated July 10 and 17, 1967.
(a) The contempt charge against the officials of the Philippine National Bank is without
merit, it appearing to the satisfaction of this Court that they excited reasonable efforts
not to disobey the writ of garnishing issued by the Commissioner. Indeed, said officials
merely acted in obedience to the order of the probate court which threatened them with
contempt of court after they moved to be allowed to deposit with the said probate court
the money of the of Elsie Gaches deposited with the said bank. The commssioner
himself, through the Solicitor General, admitted later that its writ of garnishment cannot
be superior to that of the probate court,s order as the estate in Question was then
in custodia legis.
(b) The contempt charges against the officials of the Overseas Bank of Manila likewise
merit dismissal. In the case of the renewal of the term of the time deposits in question,
the said extension was made by no less than the executor of the estate himself- The

renewal of said term may be considered as purely an act of administration for the
enhancement (due to the higher interest rates) of the value of the estate, and the
officials of the bank cannot consequently be blamed or acting favorably on the
executor's application. Judge Tan himself explained that he did what he did honest the
belief that it would redound to the benefit of the estate on the account of the higher
interest rate on time deposits.
With reference, to the refuse of the bank's officials to allow the witldrawal of time
deposit in question after the extended term expired on May 12, 1968, this Court takes
notice of the fact, as stated in our decision in Ramos vs. Central Bank (L-293250, Oct.
4, 1971; 41 SCRA 565), that as early as November 20, 1967 the Central Bank required
the Overseas Bank of Manila, in view of its distressed financial condition, to execute a
voting trust agreement in order to bail it out through a change of management and the
promise of fresh funds to replenish the bank's financial portfolio. The Overseas Bank of
Manila was not able to normalize its operations in spite of the voting trust agreement
for, on July 31, 1968, it was excluded by the Central Bank from inter-bank clearing; on
August 1, 1968, its operations were suspended; and on August 13, 1968, it was
completely forbidden by the Central Bank to do business preparatory to its forcible
liquidation. Under the circumstances, this Court is satisfied with the explanation that to
allow Atty. Medina to withdraw the said time deposits after the extended term would
have worked an undue prejudice to the other depositors and creditors of the bank.
(c) The contempt charge against Judge Tan is also not meritorious. There is no
sufficient and convincing evidence to show that he renewed the maturity date of the
time deposits in question maliciously or to the prejudice of the interest of the estate.
(d) The Lepanto Consolidated Mining Company is likewise entitled to exoneration from
the contempt charge lodged against it. It is refusing to turn over to Atty. Medina stock
dividends payable to the estate of Elsie M. Gaches, it is evident that the said
corporation acted in good faith in view of the writ of garnishment issued to it by the
Commissioner. Moreover, on February 16, 1968, this Court passed a resolution
suspending temporarily the warrants of punishment issued by the Commissioner, and it
does not appear that thereafter the turnover of the stock dividends to the estate was
refused.
(e) With reference to the charge for contempt against the respondents Atty. Medina,
Eribal and Abanto, although admittedly the resolutions of this Court dated July 10 and
17, 1967 were not strictly complied with by the said respondents, it appears clearly that
they immediately deposited with the probate court shares of stock with a fairly stable
liquidity value of P2,588,520.00. In any case, the main objective of the instant petition is
to assure the State that the assessed tax obligations shall be paid and, from the
records, more than P2 million had already been paid to the State during the pendency
of the instant proceeding, in this Court.
6. With reference to the attorney's fees to be paid to Atty. Manuel M. Paredes, this court
is of the opinion, after a careful study of the statement of services rendered by said

counsel to the respondents Eribal and Abanto which was submitted to this Court, that
the amount of Fifty Thousand Pesos (P50,000.00) is fair and reasonable. The payment
of this amount, however, is the personal liability of the said respondents Eribal and
Abanto. and not that of the estate of Elsie M. Gaches, as the said counsel was hired by
the said respondents to give legal aid to them in connection with the settlement of the
various claims preferred in the probate court and in this Court.
7. The Court's intended adjudication of the main issue has been rendered academic by
supervening events which dictate that the court refrain from issuing any further order
relating thereto. On July 18, 1977 a "Manifestation and Compliance" was filed by the,
respondent Delia P. Medina which states that a compromise payment of P700,000 as
all estate tax, evidence by an official receipt (annex A of the Manifestation), was
accepted and duly approved by Acting Commissioner of Internal Revenue Efren I.
Plana (annex B of the same Manifestation), and that "with the said compromise
payment of P700,000, all estate, inheritance and deficiency income taxes . . . including
pertinent delinquency penalties thereof have been fully paid and liquidated, aggregating
to P7,929,498.55 ..." No objection thereto was interpored by any of this parties
concerned despite due notice thereof. This was further supplemented by a
communication, dated July 19, 1977, of Deputy Commissioner Conrado P. Diaz,
informing the Register of Deeds of Pasig, Metro Manila, that the Gaches estate has
already paid all the estate and inheritance taxes assessed against it, and that,
consequently, the notice of tax then inscribed on the property and property rights of the
estate can now be considered cancelled. With the full settlement of the tax claims, the
requirements of the law have been fully met, and it has unnecessary for the Court to
issue orders relative to the main issue.
ACCORDINGLY, the respondent Delia P. Medina is to deliver the remaining assets of
the estate to the voluntary heirs in the proportions adjudicated in the will and to submit
a report of compliance. On the incidental issues, the Court renders judgment as for:
(1) The amount of FIFTY THOUSAND (P50,000.00) PESOS is hereby awarded to
Manuel M. Paredes as legal fee for his services,
the same to be Paid by the respondent End will the estate of Abanto, now
(2) The contempt charges against the officials of the Philippine National Bank and the
Overseas Bank of Manila, Judge Bienvenido Tan, Sr., and Lepanto Consolidated Co.
are hereby ordered dismissed;
(3) The authority given to the respondent Delia P. Medina in the resolution of the court
dated February 6, 1968, to pay the death and income taxes, including delinquency
penalties, claimed by the State and, for that, to withdraw all cash deposits in various
banks and sell such properties of the estate as my be necessary, is hereby terminated;
and
(4) The writs of preliminary injunction issued by the Court pursuant to its resolutions
dated July 10 and 17, 1967 are hereby dissolved.
No costs.

G.R. No. L-23638

October 12, 1967

DIONISIO FERNANDEZ, EUSEBIO REYES and LUISA REYES, petitioners, vs.


ISMAELA DIMAGIBA, respondent.

After trial on the formulated issues, the Court of First Instance, by decision of June 20,
1958, found that the will was genuine and properly executed; but deferred resolution on
the questions of estoppel and revocation "until such time when we shall pass upon the
intrinsic validity of the provisions of the will or when the question of adjudication of the
properties is opportunely presented."

---------------------------------------G.R. No. L-23662

October 12, 1967

MARIANO REYES, CESAR REYES, LEONOR REYES and PACIENCIA


REYES, petitioners, vs. ISMAELA DIMAGIBA, respondent.
Jose D. Villena for petitioners.
Antonio Barredo and Exequiel M. Zaballero for respondent.

Oppositors Fernandez and Reyes petitioned for reconsideration, and/or new trial,
insisting that the issues of estoppel and revocation be considered and resolved;
whereupon, on July 27, 1959, the Court overruled the claim that proponent was in
estoppel to ask for the probate of the will, but "reserving unto the parties the right to
raise the issue of implied revocation at the opportune time."
On January 11, 1960, the Court of First Instance appointed Ricardo Cruz as
administrator for the sole purpose of submitting an inventory of the estate, and this was
done on February 9, 1960.

REYES, J.B.L., Actg. C.J.:


The heirs intestate of the late Benedicta de los Reyes have petitioned for a review of
the decision of the Court of Appeals (in CA-G. R. No. 31221-R) affirming that of the
Court of First Instance of Bulacan, in Special Proceeding No. 831 of said Court,
admitting to probate the alleged last will and testament of the deceased, and overruling
the opposition to the probate.
It appears from the record that on January 19, 1955, Ismaela Dimagiba, now
respondent, submitted to the Court of First Instance a petition for the probate of the
purported will of the late Benedicta de los Reyes, executed on October 22, 1930, and
annexed to the petition. The will instituted the petitioner as the sole heir of the estate of
the deceased. The petition was set for hearing, and in due time, Dionisio Fernandez,
Eusebio Reyes and Luisa Reyes and one month later, Mariano, Cesar, Leonor and
Paciencia, all surnamed Reyes, all claiming to be heirs intestate of the decedent, filed
oppositions to the probate asked. Grounds advanced for the opposition were forgery,
vices of consent of the testatrix, estoppel by laches of the proponent and revocation of
the will by two deeds of conveyance of the major portion of the estate made by the
testatrix in favor of the proponent in 1943 and 1944, but which conveyances were
finally set aside by this Supreme Court in a decision promulgated on August 3, 1954, in
cases G.R. Nos. L-5618 and L-5620 (unpublished).

On February 27, 1962, after receiving further evidence on the issue whether the
execution by the testatrix of deeds of sale of the larger portion of her estate in favor of
the testamentary heir, made in 1943 and 1944, subsequent to the execution of her
1930 testament, had revoked the latter under Article 957(2) of the 1950 Civil Code (Art.
869 of the Civil Code of 1889), the trial Court resolved against the oppositors and held
the will of the late Benedicta de los Reyes "unaffected and unrevoked by the deeds of
sale." Whereupon, the oppositors elevated the case to the Court of Appeals.
The appellate Court held that the decree of June 20, 1958, admitting the will to probate,
had become final for lack of opportune appeal; that the same was appealable
independently of the issue of implied revocation; that contrary to the claim of
oppositors-appellants, there had been no legal revocation by the execution of the 1943
and 1944 deeds of sale, because the latter had been made in favor of the legatee
herself, and affirmed the decision of the Court of First Instance.
Oppositors then appealed to this Court.
In this instance, both sets of oppositors-appellants pose three main issues: (a) whether
or not the decree of the Court of First Instance allowing the will to probate had become
final for lack of appeal; (b) whether or not the order of the Court of origin dated July 27,
1959, overruling the estoppel invoked by oppositors-appellants had likewise become
final; and (c) whether or not the 1930 will of Benedicta de los Reyes had been impliedly

revoked by her execution of deeds of conveyance in favor of the proponent on March


26, 1943 and April 3, 1944.
As to the first point, oppositors-appellants contend that the order allowing the will to
probate should be considered interlocutory, because it fails to resolve the issues of
estoppel and revocation propounded in their opposition. We agree with the Court of
Appeals that the appellant's stand is untenable. It is elementary that a probate decree
finally and definitively settles all questions concerning capacity of the testator and the
proper execution and witnessing of his last will and testament, irrespective of whether
its provisions are valid and enforceable or otherwise. (Montaano vs. Suesa, 14 Phil.
676; Mercado vs. Santos, 66 Phil. 215; Trillana vs. Crisostomo, 89 Phil. 710). As such,
the probate order is final and appealable; and it is so recognized by express provisions
of Section 1 of Rule 109, that specifically prescribes that "any interested person may
appeal in special proceedings from an order or judgment . . . where such order or
judgment: (a) allows or disallows a will."
Appellants argue that they were entitled to await the trial Court's resolution on the other
grounds of their opposition before taking an appeal, as otherwise there would be a
multiplicity of recourses to the higher Courts. This contention is without weight, since
Rule 109, section 1, expressly enumerates six different instances when appeal may be
taken in special proceedings.
There being no controversy that the probate decree of the Court below was not
appealed on time, the same had become final and conclusive. Hence, the appellate
courts may no longer revoke said decree nor review the evidence upon which it is
made to rest. Thus, the appeal belatedly lodged against the decree was correctly
dismissed.
The alleged revocation implied from the execution of the deeds of conveyance in favor
of the testamentary heir is plainly irrelevant to and separate from the question of
whether the testament was duly executed. For one, if the will is not entitled to probate,
or its probate is denied, all questions of revocation become superfluous in law, there is
no such will and hence there would be nothing to revoke. Then, again, the revocation
invoked by the oppositors-appellants is not an express one, but merely implied from
subsequent acts of the testatrix allegedly evidencing an abandonment of the original
intention to bequeath or devise the properties concerned. As such, the revocation
would not affect the will itself, but merely the particular devise or legacy. Only
the total and absoluterevocation can preclude probate of the revoked testament
(Trillana vs. Crisostomo, supra.).

As to the issue of estoppel, we have already ruled in Guevara vs. Guevara, 98 Phil.
249, that the presentation and probate of a will are requirements of public policy, being
primarily designed to protect the testator's, expressed wishes, which are entitled to
respect as a consequence of the decedent's ownership and right of disposition within
legal limits. Evidence of it is the duty imposed on a custodian of a will to deliver the
same to the Court, and the fine and imprisonment prescribed for its violation (Revised
Rule 75). It would be a non sequitur to allow public policy to be evaded on the pretext of
estoppel. Whether or not the order overruling the allegation of estoppel is still
appealable or not, the defense is patently unmeritorious and the Court of Appeals
correctly so ruled.
The last issue, that of revocation, is predicated on paragraph 2 of Article 957 of the Civil
Code of 1950 (Art. 869 of the Code of 1889), which recites:
Art. 957. The legacy or devise shall be without effect:
(1) . . . .
(2) If the testator by any title or for any cause alienates the thing bequeathed or
any part thereof, it being understood that in the latter case the legacy or devise
shall be without effect only with respect to the part thus alienated. If after the
alienation the thing should again belong to the testator, even if it be by reason of
nullity of the contract, the legacy or devise shall not thereafter be valid, unless
the reacquisition shall have been effected by virtue of the exercise of the right of
repurchase;
xxx

xxx

xxx

It is well to note that, unlike in the French and Italian Codes, the basis of the quoted
provision is a presumed change of intention on the part of the testator. As pointed out
by Manresa in his Commentaries on Article 869 of the Civil Code (Vol. 6, 7th Ed., p.
743)
Este caso se funda en la presunta voluntad del testador. Si este, despues de
legar, se desprende de la cosa por titulo lucrativo u oneroso, hace desaparecer
su derecho sobra ella, dando lugar a la presuncion de que ha cambiado de
voluntad, y no quiere que el legado se cumpla. Mas para que pueda presumirse
esa voluntad, es necesario que medien actos del testador que la indiquen. Si la
perdida del derecho sobre la cosa ha sido independiente de la voluntad del

testador, el legado podraquedar sin efecto, mas no en virtud del numero 2 del
articulo 869, que exige siempre actos voluntarios de enajenacion por parte del
mismo testador.
As observed by the Court of Appeals, the existence of any such change or departure
from the original intent of the testatrix, expressed in her 1930 testament, is rendered
doubtful by the circumstance that the subsequent alienations in 1943 and 1944 were
executed in favor of the legatee herself, appellee Dimagiba. In fact, as found by the
Court of Appeals in its decision annulling these conveyances (affirmed in that point by
this Supreme Court in Reyes vs. Court of Appeals and Dimagiba, L-5618 and L-5620,
promulgated on July 31, 1954), "no consideration whatever was paid by respondent
Dimagiba" on account of the transfers, thereby rendering it even more doubtful whether
in conveying the property to her legatee, the testatrix merely intended to comply in
advance with what she had ordained in her testament, rather than an alteration or
departure therefrom.1Revocation being an exception, we believe, with the Courts below,
that in the circumstances of the particular case, Article 957 of the Civil Code of the
Philippines, does not apply to the case at bar.
Not only that, but even if it were applicable, the annulment of the conveyances would
not necessarily result in the revocation of the legacies, if we bear in mind that the
findings made in the decision decreeing the annulment of the subsequent 1943 and
1944 deeds of sale were also that
it was the moral influence, originating from their confidential relationship, which
was the only cause for the execution of Exhs. A and B (the 1943 and 1944
conveyances). (Decision, L-5618 and L-5620).

If the annulment was due to undue influence, as the quoted passage implies, then the
transferor was not expressing her own free will and intent in making the conveyances.
Hence, it can not be concluded, either, that such conveyances established a decision
on her part to abandon the original legacy.
True it is that the legal provision quoted prescribes that the recovery of the alienated
property "even if it be by reason of the nullity of the contract" does not revive the
legacy; but as pointed out by Scaevola (Codigo Civil, Vol. XV, 4th Ed., pp. 324-325) the
"nullity of the contract" can not be taken in an absolute sense.2 Certainly, it could not be
maintained, for example, that if a testator's subsequent alienation were avoided
because the testator was mentally deranged at the time, the revocatory effect ordained
by the article should still ensue. And the same thing could be said if the alienation
(posterior to the will) were avoided on account of physical or mental duress. Yet, an
alienation through undue influence in no way differs from one made through violence or
intimidation. In either case, the transferor is not expressing his real intent,3 and it can
not be held that there was in fact an alienation that could produce a revocation of the
anterior bequest.
In view of the foregoing considerations, the appealed decision of the Court of Appeals
is hereby affirmed. Costs against appellants Reyes and Fernandez. So ordered.

S-ar putea să vă placă și